Download as pdf or txt
Download as pdf or txt
You are on page 1of 704

TOPIC-WISE PRACTICE

CRITICAL REASONING

CRITICAL
REASONING
TOPIC-WISE
PRACTICE

GMAT
SANDEEP GUPTA
800/800 GMAT,
The Foremost GMAT Trainer in Asia
Founder of Top-One-Percent
Harvard Admit

www.top-one-percent.com
www.top-one-percent.com | info@top-one-percent.com | +91-97395-61394

CR Inference Questions Mega Practice


By Sandeep Gupta | GMAT 800/800, Harvard Final Admit
Part 1 – 50 questions

1. In the United States, about $5,200 per person per year is spent on health care, while in Britain the amount is
about half that. A recent study indicated that middle-aged white Americans have a significantly higher rate of
diabetes and heart disease than do middle-aged white Britons. Even after eliminating from the study the lifestyle
differences of diet, exercise, smoking, and drinking, the data showed that the Americans have poorer health
than their British counterparts. The statements above, if true, best support which of the following assertions?
A. Health care spending in the United States should be reduced by 50%.
B. More expensive health care causes a higher incidence of certain diseases.
C. The money spent on health care in the United States is not being used effectively.
D. The average health care spending for middle-aged white Americans is probably less than the average health
care spending for Americans in general.
E. Something other than diet, exercise, smoking, and drinking must account for the difference in health for the
two groups in the study.

2. Spokesperson: In the 2006 election of the city mayor, 55% of the voters were female. All the voters were
between ages 18 and 70 and 2/3 of them supported the incumbent mayor. The incumbent mayor won the
election with a substantially greater number of votes than any other candidate. If the statements made by the
Spokesperson are true, then which of the following must be true?
A. At least 1/2 of the female voters supported the incumbent mayor.
B. The incumbent mayor received stronger support from the female voters than from the male voters.
C. There were no other candidates in the election who received more than 30% of all the votes.
D. 45% of the voters in the election were male and none of them were 75 years old.
E. If the proportion of male and female voters in the city remains the same, the incumbent mayor is also likely
to win the next election.

3. In 2003, the Making Hits Record Company spent 40% of its total budget on the production of ten albums, 30%
of its budget on the marketing of these albums, and the remainder of its budget on overhead costs. In the same
year, the Song Factory Record Company spent 20% of its total budget on the production of 10 albums and 60%
of its budget on the marketing of these albums. Making Hits sold a total of 800,000 copies of the ten records it
produced in 2003, while the Song Factory sold a total of 1,600,000 copies of the ten records it produced in
2003. Assuming each company met its budget, which of the following conclusions is best supported by the
information given above?
A. The amount of money spent on marketing is directly related to the number of copies sold.
B. Making Hits spent more money on the production of its albums in 2003 than did the Song Factory.
C. Song Factory’s total revenue from the sale of albums produced in 2003 was higher than that of Making Hits.
D. In 2003, Making Hits spent a larger percentage of its budget on overhead costs than did the Song Factory.
E. The Song Factory sold more copies of its 2003 albums than Making Hits did because the Song Factory spent a
higher percentage of its budget on the marketing of its albums.
4. Due to high jet fuel costs, airline carriers are looking for new ways to increase revenues and thereby counteract
declining profits. Airline A has proposed increasing the number of passengers that can fit on its airplanes by
creating several standing room only “seats” in which passengers would be propped against a padded backboard
and held in place with a harness. This proposal, since it relates to passenger safety, cannot be implemented
without prior approval by the Federal Aviation Administration. The above statements, if true, indicate that
Airline A has made which of the following conclusions?
A. The addition of standing room only “seats” will generate more revenue than the cost of ensuring that these seats
meet safety standards.
B. The Federal Aviation Administration will approve Airline A’s specific proposal.
C. The revenue generated by the addition of standing room only “seats” is greater than the current cost of jet fuel.
D. There are no safer ways in which Airline A can increase revenues.
E. Passenger safety is less important than increasing revenue.

5. Most antidepressant drugs cause weight gain. While dieting can help reduce the amount of weight gained while
taking such antidepressants, some weight gain is unlikely to be preventable. The information above most
strongly supports which one of the following?
A. A physician should not prescribe any antidepressant drug for a patient if that patient is overweight.
B. People who are trying to lose weight should not ask their doctors for an antidepressant drug.
C. At least some patients taking antidepressant drugs gain weight as a result of taking them.
D. The weight gain experienced by patients taking antidepressant drugs should be attributed to lack of dieting.
E. All patients taking antidepressant drugs should diet to maintain their weight.

6. Columnist: The winner of this year’s national spelling bee won by correctly spelling the spoken word
Ursprache, which means “fame” in German. Given the richness of our language, why must we resort to words
taken from modern foreign languages to challenge our best spellers? Ursprache is listed in our dictionary, as
are words from many other foreign languages, but future spelling bees should limit themselves to words in our
dictionary that have been anglicized in all aspects because spelling English words, not knowledge of linguistics
and international phonetics, is the point of these contests. Which of the following can most reasonably be
inferred from the argument above?
A. The spelling contest winner knew how to spell most of the anglicized words in the dictionary.
B. Foreign words are more difficult than anglicized words for all contestants to spell.
C. Spelling contestant winners should be determined by their facility with all aspects of language.
D. To spell foreign words, contestants must recognize the language and know its pronunciation.
E. The English language contains more borrowed words than most other languages.

7. Muscular strength is a limited resource, and athletic techniques help to use this resource efficiently. Since top
athletes do not differ greatly from each other in muscular strength, it follows that a requirement for an athlete
to become a champion is a superior mastery of athletic techniques. Which one of the following most accurately
expresses the conclusion of the argument?
A. Only champion athletes have a superior mastery of athletic techniques.
B. Superior muscular strength is a requirement for an athlete to become a champion.
C. No athlete can become a champion without a superior mastery of athletic techniques.
D. The differences in muscular strength between top athletes are not great.
E. Athletic techniques help athletes use limited resources efficiently.
8. According to a recent study on financial roles, one-third of high school seniors say that they have “significant
financial responsibilities.” These responsibilities include, but are not limited to, contributing to food, shelter,
or clothing for themselves or their families. At the same time, a second study demonstrates that a crisis in
money management exists for high school students. According to this study, 80% of high school seniors have
never taken a personal finance class even though the same percentage of seniors has opened bank accounts and
one-third of these account holders have bounced a check. Which of the following conclusions can be properly
drawn from the statements above?
A. High schools would be wise to incorporate personal finance classes into their core curricula.
B. At least one-third of high school seniors work part-time jobs after school.
C. The number of high school seniors with significant financial responsibilities is greater than the number of
seniors who have bounced a check.
D. Any high school seniors who contribute to food, shelter, or clothing for themselves or their families have
significant financial responsibilities.
E. The majority of high school students have no financial responsibilities to their families.

9. Analyst: Creative professionals, such as clothing designers, graphic designers, and decorators, often have very
poor managerial skills and do not succeed when they try to run their own businesses. In fact, most of these
creative types are less skilled in business than is the average white-collar professional who does not work in a
creative field. Generally, creative talent and business acumen rarely go hand in hand. If the analyst’s argument
is taken as true, which of the following statements can properly be concluded?
A. No successful businesspeople are creative.
B. Some creative types are not less skilled at business than is the average white-collar worker who is not creative.
C. Creativity precludes success in business.
D. Any white-collar worker who is not creative is more successful in business than any creative professional.
E. Business is not a creative endeavor.

10. Advocates insist that health savings accounts are an efficient method to reduce medical expenses. However,
widespread adoption of these accounts will soon undermine the public’s health. One reason for this is that most
people will be reluctant to deplete their accounts to pay for regular preventive examinations, so that in many
cases a serious illness will go undetected until it is far advanced. Another reason is that poor people, who will
not be able to afford health savings accounts, will no longer receive vaccinations against infectious diseases.
The statements above, if true, most support which of the following?
A. Wealthy individuals will not be affected negatively by health savings accounts.
B. Private health insurance will no longer be available.
C. Most diseases are detected during regular preventive examinations.
D. Some people without health savings accounts are likely to contract infectious diseases.
E. The causal relationship between an individual’s health and that person’s medical care has been adequately
documented.
11. Albinism is a rare genetic condition that inhibits the production of melanin, or pigmentation, in the skin and
hair. People born with albinism are unusually susceptible to sunburn, melanoma, and a range of other health
issues that are generally connected to excessive exposure to the sun. The statements above, if true, provide the
most support for which of the following conclusions?
A. People born with albinism develop other biological protections against melanoma and other sun-related health
issues.
B. Humans with a high production of melanin can easily ignore health issues related to exposure to the sun.
C. When a non-albino person gets sunburn, the amount of melanin produced by that person decreases.
D. In humans, melanin plays a role in protecting the skin from developing sunburn and other sun-related ailments.
E. It is not possible for a person born with albinism to adopt other artificial protective measures against excessive
exposure to the sun.

12. Celiac disease results from an inability of the digestive tract, specifically the small intestine, to absorb gluten,
a protein found in wheat, barley, and certain other grains. The body’s immune system attacks the gluten as if
the protein were a harmful pathogen, often resulting in serious damage to the intestinal lining. People who
suffer from celiac disease must eliminate gluten from their diets. Symptoms of the disease include abdominal
cramps, bloating, and anemia. If the statements above are true, which of the following assertions can be made
on the basis of them?
A. Anyone who suffers from celiac disease will experience anemia.
B. Eliminating gluten from one’s diet will cure celiac disease.
C. People experiencing abdominal cramps, bloating, and anemia have celiac disease.
D. Gluten is found only in grains.
E. The human body cannot always recognize harmless substances.

13. Mayville Airport and Newcomb Airport have the same number of flight departures each day. Mayville Airport
experiences 26 departure delays per 100 flights, while Newcomb Airport experiences 20 departure delays per
100 flights. When delays caused by bad weather are disregarded, Mayville Airport has 5 fewer departure delays
per 100 flights than Newcomb Airport does. Which of the following conclusions is best supported by the
information given above?
A. Bad weather causes a greater number of departure delays at Mayville Airport than at Newcomb Airport.
B. On average, the weather at Mayville Airport is worse than it is at Newcomb Airport.
C. Mechanical problems cause a greater number of delays at Newcomb Airport than at Mayville Airport.
D. The fleet of airplanes leaving from Newcomb Airport is better equipped to handle inclement weather than the
fleet of airplanes leaving from Mayville Airport.
E. Mayville Airport experiences a greater number of arrival delays per 100 flights than Newcomb Airport does.

14. The head baker at Barry’s Bagels can either purchase flour in-person from the local flour mill, Larry’s Local
Mill, or order a shipment of flour from an out-of-state mill, Isadore’s Interstate Mill. The cost of the flour from
Isadore’s Interstate Mill is 10 percent less than the cost of the flour from Larry’s Local Mill. Even after shipping
and handling fees are added, it is still cheaper to order flour that has to be shipped from Isadore’s than to buy
flour locally from Larry’s. The statements above, if true, best support which of the following assertions?
A. Production costs at Isadore’s Interstate Mill are 10 percent below those at Larry’s Local Mill.
B. Buying flour from Isadore’s Interstate Mill will eliminate 10 percent of the local flour mill jobs.
C. The shipping and handling fees for a batch of flour purchased from Isadore’s Interstate Mill are less than 10
percent of the cost of an identical batch of flour purchased from Larry’s Local Mill.
D. The shipping and handling fees for a batch of flour purchased from Isadore’s Interstate Mill are more than 10
percent of the cost of Isadore’s flour.
E. Isadore’s Interstate Mill produces flour 10% more efficiently than Larry’s Local Mill does.
15. Psychologist: Although studies of young children have revealed important facts about the influence of the
environment on language acquisition, it is clear that one cannot attribute such acquisition solely to
environmental influences: innate mechanisms also play a role. So, the most reasonable question that ought to
be studied is whether _______.
Which one of the following most logically completes the passage?
A. language acquisition can ever be fully explained
B. innate mechanisms are a contributing factor in language learning
C. language acquisition is solely the product of innate mechanisms
D. parents and peers are the most important influence on a child’s learning of a language
E. innate mechanisms play a more important role in language acquisition than a child’s immediate environment

16. According to a recent magazine article, of those office employees who typically work 8 hours at the office each
day but sometimes say that they will work at home on a particular day, 25 percent actually work less than one
hour. At the same time, over 90 percent of those same office employees believe they are more productive
working at home than working in their office. The statements above, if true, best support which of the following
conclusions about the office employees discussed in the article?
A. On average, the office employees working at home for a day work fewer hours than office employees working
at the office.
B. 10 percent of the office employees are less productive working from home than working in their office.
C. At least 15 percent of the office employees do not define productivity exclusively in terms of the number of
hours worked.
D. At least 25 percent of the office employees can complete the same amount of work in one hour at home as in 8
hours at the office.
E. Some of the office employees make statements regarding their productivity that are not in fact true.

17. On Monday, Daisy’s Lemonade Stand sold lemonade at 20 cents per cup. The Lemon Shack sold lemonade at
30 cents per cup. At the end of the day, Daisy’s Lemonade Stand and the Lemon Shack reported identical
revenues and identical profits. The statements above best support which of the following assertions?
A. On Monday, Daisy’s Lemonade Stand sold fewer cups of lemonade than did the Lemon Shack.
B. The Lemon Shack sells higher quality lemonade than does Daisy’s Lemonade Stand.
C. On Monday, Daisy’s Lemonade Stand and the Lemon Shack incurred identical costs to run their businesses.
D. In general, lemonade consumers prefer the lemonade at Daisy’s Lemonade Stand to the Lemonade at the Lemon
Shack.
E. The Lemon Shack would not increase its revenues by lowering its prices.

18. Government restrictions have severely limited the amount of stem cell research American companies can
conduct. Because of these restrictions, many American scientists who specialize in the field of stem cell
research have signed long-term contracts to work for foreign companies. Recently, Congress has proposed
lifting all restrictions on stem cell research. Which of the following conclusions can most properly be inferred
from the information above?
A. At least some foreign companies that conduct stem cell research work under fewer restrictions than some
American companies do.
B. Because American scientists are under long-term contracts to foreign companies, there will be a significant
influx of foreign professionals into the United States.
C. In all parts of the world, stem cell research is dependent on the financial backing of local government.
D. In the near future, American companies will no longer be at the forefront of stem cell research.
E. If restrictions on stem cell research are lifted, many of the American scientists will break their contracts to
return to American companies.
19. As many as 98,000 people die each year due to medical error. In a campaign to reduce lethal errors, thousands
of hospitals introduced six key changes, including rapid-response teams, re-checks of patient medication, and
new guidelines for preventing infection. The campaign estimated that, over an 18-month period, more than
100,000 lives were saved as a direct result of the program. Which of the following can be most properly inferred
from the above statements?
A. Doctors and nurses should be more careful when doing their jobs.
B. The campaign saved all of the people who otherwise would have died due to medical error in that time period.
C. In the future, no one will die because of medical error.
D. If the campaign had not been implemented, more than 100,000 people might have died during the 18-month
period due to medical error.
E. The key changes initiated by the campaign will continue to be implemented in the future.

20. Federal law prohibits businesses from reimbursing any employees for the cost of owning and operating a private
aircraft that is used for business purposes. Thus, many American companies themselves purchase private
aircraft. The vast majority of the business aviation fleet is owned by small and mid-size businesses, and flights
are strictly for business purposes, with mostly mid-level employees on board. These companies and their boards
of directors are in full compliance with the law and with what is best for their businesses. Which of the following
can be most properly inferred from the statements above?
A. The Federal law in question costs businesses money.
B. Most executives would rather fly on company owned planes than on commercial airlines.
C. Large businesses usually have their executives fly first or business class on commercial flights.
D. Upper-level executives are less often in compliance with the law.
E. By not receiving any reimbursement for these flights, the mid-level executives on board are complying with
the law.

21. Antoine: The alarming fact is that among children aged 19 years and younger, the number taking antipsychotic
medicines soared 73 percent in the last four years. That is greater than the increase in the number of adults
taking antipsychotic medicines during the same period.
Lucy: But the use of antipsychotic drugs by adults is considered normal at the current rate of 11 adults per
1,000 taking the drugs. In contrast, the number of children on antipsychotic medication last year was 6.6 per
1,000 children. Lucy’s argument is structured to lead to which of the following as a conclusion?
A. The current level of antipsychotic drug use in children is abnormally high.
B. The fact that the number of children taking antipsychotic medicines increased 73 percent over the last four years
is not an indication that the current level of use is abnormally high.
C. If only 6.6 out of every 1,000 children are taking an antipsychotic medication, the increase in the use of such
medicines cannot be the percentage Antoine cites.
D. It is unlikely that the increase in the use of antipsychotic medicines by children will continue at the same rate.
E. If the number of children taking antipsychotic drugs is given as a certain number, the actual rate of such drug
use is even higher.
22. The ability to analyze genomes—sequences of DNA—has grown more and more sophisticated. Scientists are
able to examine the biological past in finer detail and with greater accuracy. A new analysis of the genetic links
between early humans and chimpanzees has led to a hypothesis that the two species diverged more recently
than previous estimates indicated. This gives credence to a very startling new theory: the ancestors of humans
and chimpanzees might have hybridized to produce the lineage from which modern humans eventually
developed. What can be inferred from the statements above?
A. Hybridized species combine the best of both lineages.
B. Political and religious pressures will affect the course of future research.
C. Future research will enable scientists to determine the exact characteristics of this hybrid ancestor of modern
man.
D. Earlier genome analysis established genetic links between early humans and chimpanzees.
E. Human DNA is more complex than that of chimpanzees.

23. Prediction, the hallmark of natural sciences, appears to have been possible by reducing phenomena to
mathematical expressions. Some social scientists also want the power to predict accurately and assume they
ought to perform the same reduction. But this would be a mistake; it would neglect data that are not easily
mathematized and thereby would only distort the social phenomena. Which one of the following most accurately
expresses the main conclusion of the argument?
A. The social sciences do not have as much predictive power as the natural sciences.
B. Mathematics plays a more important role in the natural sciences than it does in the social sciences.
C. There is a need in the social sciences to improve the ability to predict.
D. Phenomena in the social sciences should not be reduced to mathematical formulas.
E. Prediction is responsible for the success of the natural sciences.

24. Under a new clean air proposal, the government has decided to tighten controls on the release of certain toxic
chemicals, including benzene, formaldehyde, and other carcinogens, by chemical plants. The stated purpose of
this proposal is to reduce cancers caused by air pollution. Yet, the chemical industry, rather than the
government, is responsible for monitoring the implementation of the proposal. If the past actions of certain
polluters in the chemical industry are any indication of future behavior, the net result of the new proposal will
be an increase, rather than a decrease, in carcinogens released into the air. The author is arguing that ____.
A. no chemical companies can be trusted to follow the clean air proposal
B. the chemical industry is responsible for releasing the majority of carcinogens into the air
C. allowing self-monitoring for the new clean air proposal will result in the opposite of its intended consequence
D. to ensure effective implementation, the government should always monitor the execution of its proposals
E. benzene and formaldehyde are two of the most hazardous cancer-causing chemicals
25. Since 1995, Congress has exempted oil companies that have leases issued by the federal government allowing
them to drill for deep-water oil off the Gulf of Mexico from royalty payments as an incentive to spur
development in times of low oil and gas prices. These leases were supposed to have included a provision that
reinstates the royalties should the market prices of oil and gas exceed a certain level. Because of an error by
the federal government, however, the language that reinstates the royalties is missing from the more than 1,100
leases issued by the U.S. government in 1998 and 1999. Since the market price of oil and gas has recently risen
far above the threshold levels, this error could allow the oil companies to reap a windfall of more than $10
billion through the life of the leases. In response, the government is pressuring the oil companies to renegotiate
the leases. The executives of the oil companies strongly oppose renegotiation; all have issued statements stating
that they expect the government to honor the terms of the contracts and that renegotiating a duly signed
agreement would set a bad precedent. Which of the following statements best reflects the position of the oil
company executives?
A. Opportunity seldom knocks twice.
B. Do unto others as you would have done unto you.
C. One man’s loss is another man’s gain.
D. You don’t change the rules in the middle of the game.
E. Revenge is so sweet.

26. Only those students who maintain 2.5 grade point averages are allowed to participate in school sports. Amy is
captain of the school’s tennis team, so she must have at least a 2.5 GPA. Which of the following statements best
summarizes the main point of the above argument?
A. Students who don’t maintain a 2.5 GPA can’t participate in sports.
B. Amy is a good tennis player.
C. The school should only require a minimum GPA of 2.0 to participate in sports.
D. Amy has earned at least the minimum GPA required to participate on a school sports team.
E. Amy wouldn’t be captain of the tennis team if her GPA were lower.

27. Companies are often torn between the benefits of focusing on one major product or service and the drawbacks
of relying too heavily on one primary source of income. While narrow focus can provide a company with an
advantage over competitors that offer a wider range of products or services, an undiversified income stream
can leave a company susceptible to major fluctuations in cash flow. We can see this tension realized when, for
example, ________________.
Which of the following best completes the passage below?
A. a local messenger service known for its speedy deliveries is forced to lay off twenty percent of its work force
after a rise in local taxes encourages many local businesses to move out of state
B. an advertising agency loses one of its clients
C. a holding company that owns a car rental agency and a national doughnut chain is now interested in purchasing
a professional basketball team
D. a construction company opts to use non-union labor to increase its profits
E. a specialty sandwich store decides to open franchises throughout the country that will focus on using local
ingredients
28. Calorie restriction, a diet high in nutrients but low in calories, is known to prolong the life of rats and mice by
preventing heart disease, cancer, diabetes, and other diseases. A six-month study of 48 moderately overweight
people, who each reduced their calorie intake by at least 25 percent, demonstrated decreases in insulin levels
and body temperature, with the greatest decrease observed in individuals with the greatest percentage change
in their calorie intake. Low insulin level and body temperature are both considered signs of longevity, partly
because an earlier study by other researchers found both traits in long-lived people. If the above statements are
true, they support which of the following inferences?
A. Calorie restriction produces similar results in humans as it does in rats and mice.
B. Humans who reduce their calorie intake by at least 25 percent on a long-term basis will live longer than they
would have had they not done so.
C. Calorie intake is directly correlated to insulin level in moderately overweight individuals.
D. Individuals with low insulin levels are healthier than individuals with high insulin levels.
E. Some individuals in the study reduced their calorie intake by more than 25 percent.

29. Last January, in an attempt to lower the number of traffic fatalities, the state legislature passed its “Click It or
Ticket” law. Under the new law, motorists can be pulled over and ticketed for not wearing their seat belts, even
if an additional driving infraction has not been committed. Lawyers and citizens’ groups are already protesting
the law, saying it unfairly infringes on the rights of the state’s drivers. Law enforcement groups counter these
claims by stating that the new regulations will save countless additional lives. Which of the following inferences
is best supported by the passage above?
A. Prior to the “Click It or Ticket” law, motorists could not be stopped simply for not wearing a seat belt.
B. The “Click It or Ticket” law violates current search and seizure laws.
C. Laws similar to “Click It or Ticket” have effectively reduced traffic fatalities in a number of states.
D. The previous seatbelt laws were ineffective in saving lives.
E. Law enforcement groups, rather than citizens groups, should determine how to best ensure the safety of
motorists.

30. In a certain state, hunting permits require the hunter to be at least 18 years old, possess a valid driver’s license
or state identification, and have completed a safety program within the past 5 years. The hunter must also sign
a legal document pledging not to consume alcohol while hunting. Which of the following can be correctly
inferred from the above statements?
A. All states have the same requirements for hunting permits.
B. Hunters under the age of 21 don’t need to sign the alcohol pledge because they are not legally allowed to drink
alcohol under any circumstances.
C. If a person last completed the safety program six years ago, he will have to complete the program again before
he can be eligible for a permit.
D. Someone who isn’t 18 years old cannot obtain a hunting permit in this state.
E. Hunting is such a dangerous activity that state controls and requirements are necessary to ensure that nobody
gets hurt.

31. The public often protests when an unregulated service industry is found to be corrupt. However, regulation
often leads to increased costs for the consumer. Fewer companies survive in a regulated market, leading to
decreased competition and higher prices. The public then responds negatively to the increased costs of these
services. The statements above best support which of the following?
A. Service industries should not be regulated.
B. The public should not protest unregulated services.
C. Only unregulated services are subject to public protest.
D. The public is sometimes the cause of its own complaints.
E. Decreased competition always leads to increased prices.
32. A certain medication used to treat migraine headaches acts by blocking pain receptors in the brain. When a
person takes the medication within one hour after ingesting grapefruit or grapefruit juice, however, the
effectiveness of the medication is significantly diminished. Researchers have determined that the grapefruit
contains a compound that alters the shape of the pain receptors, with the result that the medication can no longer
bind with them completely. Which of the following conclusions could be most properly drawn from the
information given above?
A. If one takes the medication more than an hour after ingesting grapefruit, its effectiveness is not diminished.
B. Ingesting grapefruit after taking the medication does not diminish the effectiveness of the medication.
C. There is only one type of pain receptor in the brain.
D. The medication is fully effective only when it properly binds with its target pain receptors.
E. It is not possible to design a medication for migraine headaches that can bond with the altered receptors.

33. Educator: Like any other difficult pursuit, music requires intense study and practice in order for one to become
proficient. But many school music programs encourage only children who demonstrate early aptitude to
continue studying music, while children who are not especially musical are directed towards other activities.
Having learned to think of themselves as musically inept, these children do not devote any time to music and
thus deprive themselves of the opportunity to develop a latent talent. The educator’s statements, if true, would
best support which of the following conclusions?
A. Music education should not devote special attention to talented students.
B. Everyone has the potential to learn music.
C. Talent is not always apparent at an early age.
D. Children are particularly sensitive to criticism from adults.
E. All children should study music.

34. Improved technology and equipment often result in fewer injuries during high-risk activities such as rock
climbing and scuba diving. But participant education also plays a large role in reducing the number of injuries
sustained during these activities. People who are poorly trained in these activities run a much higher risk of
injury even if provided with the latest and best equipment. Which of the following can be properly inferred
from the information above?
A. Training is a more important safety factor than equipment in high-risk activities.
B. People who are properly trained in their activities do not sustain injuries.
C. The safety benefits of the latest equipment can be offset by inadequate preparation.
D. Rock climbing and scuba diving are riskier than any other activities.
E. People with the latest equipment often neglect proper training.

35. The new heart scans offer patients significant benefits. They can be completed in a fraction of the time required
for an angiogram, with no recovery time necessary. Furthermore, the scans are more sensitive and can identify
problem areas that an angiogram might not perceive. However, heart scans use more radiation than most
diagnostic procedures, and can cause undue concern over and treatment for the harmless abnormalities often
picked up by such sensitive technology. Which of the following conclusions is best supported by the
statements above?
A. A heart scan is safer than an angiogram procedure.
B. Patients should not be concerned about heart abnormalities that appear in a heart scan.
C. A heart scan could result in indirect harm by causing a patient to undergo risky, unnecessary procedures.
D. An angiogram is the more appropriate of the two procedures for most patients.
E. The heart scan is a more expensive procedure than the angiogram.
36. The importance of the ozone layer to terrestrial animals is that it entirely filters out some wavelengths of light
but lets others through. Holes in the ozone layer and the dangers associated with these holes are well
documented. However, one danger that has not been given sufficient attention is that these holes could lead to
severe eye damage for animals of many species. Which one of the following is most strongly supported by the
statements above, if they are true?
A. All wavelengths of sunlight that can cause eye damage are filtered out by the ozone layer where it is intact.
B. Few species of animals live on a part of the earth’s surface that is not threatened by holes in the ozone layer.
C. Some species of animals have eyes that will not suffer any damage when exposed to unfiltered sunlight.
D. A single wavelength of sunlight can cause severe damage to the eyes of most species of animals.
E. Some wavelengths of sunlight that cause eye damage are more likely to reach the earth’s surface where there
are holes in the ozone layer than where there are not.

37. One of the most vexing problems in historiography is dating an event when the usual sources offer conflicting
chronologies of the event. Historians should attempt to minimize the number of competing sources, perhaps by
eliminating the less credible ones. Once this is achieved and several sources are left, as often happens, historians
may try, though on occasion unsuccessfully, to determine independently of the usual sources which date is
more likely to be right. Which one of the following inferences is most strongly supported by the information
above?
A. We have no plausible chronology of most of the events for which attempts have been made by historians to
determine the right date.
B. Some of the events for which there are conflicting chronologies and for which attempts have been made by
historians to determine the right date cannot be dated reliably by historians.
C. Attaching a reliable date to any event requires determining which of several conflicting chronologies is most
likely to be true.
D. Determining independently of the usual sources which of several conflicting chronologies is more likely to be
right is an ineffective way of dating events.
E. The soundest approach to dating an event for which the usual sources give conflicting chronologies is to
undermine the credibility of as many of these sources as possible.

38. The increasing complexity of scientific inquiry has led to a proliferation of multi-authored technical articles.
Reports of clinical trials involving patients from several hospitals are usually coauthored by physicians from
each participating hospital. Likewise, physics papers reporting results from experiments using subsystems
developed at various laboratories generally have authors from each laboratory. If all of the statements above
are true, which one of the following must be true?
A. Clinical trials involving patients from several hospitals are never conducted solely by physicians from just one
hospital.
B. Most reports of clinical trials involving patients from several hospitals have multiple authors.
C. When a technical article has multiple authors, they are usually from different institutions.
D. Physics papers authored by researchers from multiple laboratories usually report results from experiments using
subsystems developed at each laboratory.
E. Most technical articles are authored solely by the researchers who conducted the experiments these articles
report.
39. Some environmentalists question the prudence of exploiting features of the environment, arguing that there are
no economic benefits to be gained from forests, mountains, or wetlands that no longer exist. Many
environmentalists claim that because nature has intrinsic value it would be wrong to destroy such features of
the environment, even if the economic costs of doing so were outweighed by the economic costs of not doing
so. Which one of the following can be logically inferred from the passage?
A. It is economically imprudent to exploit features of the environment.
B. Some environmentalists appeal to a non-economic justification in questioning the defensibility of exploiting
features of the environment.
C. Most environmentalists appeal to economic reasons in questioning the defensibility of exploiting features of
the environment.
D. Many environmentalists provide only a non-economic justification in questioning the defensibility of exploiting
features of the environment.
E. Even if there is no economic reason for protecting the environment, there is a sound non-economic justification
for doing so.

40. Some argue that laws are instituted at least in part to help establish a particular moral fabric in society. But the
primary function of law is surely to help order society so that its institutions, organizations, and citizenry can
work together harmoniously, regardless of any further moral aims of the law. Indeed, the highest courts have
on occasion treated moral beliefs based on conscience or religious faith as grounds for making exceptions in
the application of laws. The statements above, if true, most strongly support which one of the following?
A. The manner in which laws are applied sometimes takes into account the beliefs of the people governed by those
laws.
B. The law has as one of its functions the ordering of society but is devoid of moral aims.
C. Actions based on religious belief or on moral conviction tend to receive the protection of the highest courts.
D. The way a society is ordered by law should not reflect any moral convictions about the way society ought to be
ordered.
E. The best way to promote cooperation among a society’s institutions, organizations, and citizenry is to institute
order in that society by means of law.

41. Newtonian physics dominated science for over two centuries. It found consistently successful application,
becoming one of the most highly substantiated and accepted theories in the history of science. Nevertheless,
Einstein’s theories came to show the fundamental limits of Newtonian physics and to surpass the Newtonian
view in the early 1900s, giving rise once again to a physics that has so far enjoyed wide success. Which one of
the following logically follows from the statements above?
A. The history of physics is characterized by a pattern of one successful theory subsequently surpassed by another.
B. Long-standing success or substantiation of a theory of physics is no guarantee that the theory will continue to
be dominant indefinitely.
C. Every theory of physics, no matter how successful, is eventually surpassed by one that is more successful.
D. Once a theory of physics is accepted, it will remain dominant for centuries.
E. If a long-accepted theory of physics is surpassed, it must be surpassed by a theory that is equally successful.
42. The solidity of bridge piers built on pilings depends largely on how deep the pilings are driven. Prior to 1700,
pilings were driven to “refusal,” that is, to the point at which they refused to go any deeper. In a 1588 inquiry
into the solidity of piers for Venice’s Rialto Bridge, it was determined that the bridge’s builder, Antonio Da
Ponte, had met the contemporary standard for refusal: he had caused the pilings to be driven until additional
penetration into the ground was no greater than two inches after twenty-four hammer blows. Which one of the
following can properly be inferred from the passage?
A. The Rialto Bridge was built on unsafe pilings.
B. The standard of refusal was not sufficient to ensure the safety of a bridge.
C. Da Ponte’s standard of refusal was less strict than that of other bridge builders of his day.
D. After 1588, no bridges were built on pilings that were driven to the point of refusal.
E. It is possible that the pilings of the Rialto Bridge could have been driven deeper even after the standard of
refusal had been met.

43. Every moral theory developed in the Western tradition purports to tell us what a good life is. However, most
people would judge someone who perfectly embodied the ideals of any one of these theories not to be living a
good life—the kind of life they would want for themselves and their children. The statements above, if true,
most strongly support which one of the following?
A. Most people desire a life for themselves and their children that is better than a merely good life.
B. A person who fits the ideals of one moral theory in the Western tradition would not necessarily fit the ideals of
another.
C. Most people have a conception of a good life that does not match that of any moral theory in the Western
tradition.
D. A good life as described by moral theories in the Western tradition cannot be realized.
E. It is impossible to develop a theory that accurately describes what a good life is.

44. Cézanne’s art inspired the next generation of artists, twentieth-century modernist creators of abstract art. While
most experts rank Cézanne as an early modernist, a small few reject this idea. Françoise Cachin, for example,
bluntly states that such an ascription is “overplayed,” and says that Cézanne’s work is “too often observed from
a modern point of view.” Which one of the following statements is most strongly supported by the information
above?
A. Cézanne’s work is highly controversial.
B. Cézanne was an early creator of abstract art.
C. Cézanne’s work helped to develop modernism.
D. Modern art owes less to Cézanne than many experts believe.
E. Cézanne’s work tends to be misinterpreted as modernist.

45. Light is registered in the retina when photons hit molecules of the pigment rhodopsin and change the molecules’
shape. Even when they have not been struck by photons of light, rhodopsin molecules sometimes change shape
because of normal molecular motion, thereby introducing error into the visual system. The amount of this
molecular motion is directly proportional to the temperature of the retina. Which one of the following
conclusions is most strongly supported by the information above?
A. The temperature of an animal’s retina depends on the amount of light the retina is absorbing.
B. The visual systems of animals whose body temperature matches that of their surroundings are more error-prone
in hot surroundings than in cold ones.
C. As the temperature of the retina rises, rhodopsin molecules react more slowly to being struck by photons.
D. Rhodopsin molecules are more sensitive to photons in animals whose retinas have large surface areas than in
animals whose retinas have small surface areas.
E. Molecules of rhodopsin are the only pigment molecules that occur naturally in the retina.
46. Flavonoids are a common component of almost all plants, but a specific variety of flavonoid in apples has been
found to be an antioxidant. Antioxidants are known to be a factor in the prevention of heart disease. Which one
of the following can be properly inferred from the passage?
A. A diet composed largely of fruits and vegetables will help to prevent heart disease.
B. Flavonoids are essential to preventing heart disease.
C. Eating at least one apple each day will prevent heart disease.
D. At least one type of flavonoid helps to prevent heart disease.
E. A diet deficient in antioxidants is a common cause of heart disease.

47. Sharks have a higher ratio of cartilage mass to body mass than any other organism. They also have a greater
resistance to cancer than any other organism. Shark cartilage contains a substance that inhibits tumor growth
by stopping the development of a new blood network. In the past 20 years, none of the responses among
terminal cancer patients to various therapeutic measures has been more positive than the response among those
who consumed shark cartilage. If the claims made above are true, then each of the following could be true
EXCEPT:
A. No organism resists cancer better than sharks do, but some resist cancer as well as sharks.
B. The organism most susceptible to cancer has a higher percentage of cartilage than some organisms that are less
susceptible to cancer.
C. The substance in shark cartilage that inhibits tumor growth is found in most organisms.
D. In the past 20 years many terminal cancer patients have improved dramatically following many sorts of therapy
E. Some organisms have immune systems more efficient than a shark’s immune system.

48. Journalist: A free marketplace of ideas ensures that all ideas get a fair hearing. Even ideas tainted with prejudice
and malice can prompt beneficial outcomes. In most countries, however, the government is responsible for over
half the information released to the public through all media. For this reason, the power of governments over
information needs to be curtailed. Everyone grants that governments should not suppress free expression, yet
governments continue to construct near monopolies on the publication and dissemination of enormous amounts
of information. Which one of the following most accurately expresses the conclusion of the journalist’s
argument?
A. The freedom of the marketplace of ideas is in jeopardy.
B. Preserving a free marketplace of ideas is important.
C. The control that governments have over information needs to be reduced.
D. Ideas that have malicious content or stem from questionable sources can be valuable.
E. Governments have near monopolies on the dissemination of many kinds of information.

49. I agree that Hogan’s actions resulted in grievous injury to Winters. And I do not deny that Hogan fully realized
the nature of his actions and the effects that they would have. Indeed, I would not disagree if you pointed out
that intentionally causing such effects is reprehensible, other things being equal. But in asking you to concur
with me that Hogan’s actions not be wholly condemned I emphasize again that Hogan mistakenly believed
Winters to be the robber who had been terrorizing west-side apartment buildings for the past several months.
Which one of the following most accurately expresses the conclusion of the argument?
A. Hogan should not be considered responsible for the injuries sustained by Winters.
B. The robber who had been terrorizing west-side apartment buildings should be considered to be as responsible
for Winters’s injuries as Hogan.
C. The actions of Hogan that seriously injured Winters are not completely blameworthy.
D. Hogan thought that Winters was the person who had been terrorizing west-side apartment buildings for the last
few months.
E. The actions of Hogan that seriously injured Winters were reprehensible, other things being equal.
50. Last month OCF, Inc., announced what it described as a unique new product: an adjustable computer
workstation. Three days later ErgoTech unveiled an almost identical product. The two companies claim that
the similarities are coincidental and occurred because the designers independently reached the same solution
to the same problem. The similarities are too fundamental to be mere coincidence, however. The two products
not only look alike, but they also work alike. Both are oddly shaped with identically placed control panels with
the same types of controls. Both allow the same types of adjustments and the same types of optional
enhancements. The main point of the argument is that
A. the two products have many characteristics in common
B. ErgoTech must have copied the design of its new product from OCF’s design
C. the similarities between the two products are not coincidental
D. product designers sometimes reach the same solution to a given problem without consulting each other
E. new products that at first appear to be unique are sometimes simply variations of other products
Answers and Solutions Part 1

1.
To make a valid assertion from the information in the argument, we can only rely on the facts that we are given: 1) A study
found that middle-aged white Americans are sicker than middle-aged white Britons, 2) the researchers eliminated the lifestyle
differences of diet, exercise, smoking, and drinking as causes of the health difference, and 3) average health care spending
per person is higher in the United States than in Britain. If any additional information is needed in order to support an
assertion, then it is not a valid conclusion.

Be careful to avoid making unintentional assumptions! The health care costs given in the argument are the average costs
per person per year in each nation, yet the study only looked at the health of middle-aged white people. It is unclear exactly
what amount of money is spent on health care for the age group and race studied. For example, the average health care
spending in the United States may include dramatically higher costs for care of premature infants, so it is possible that health
care expenditures for the middle-aged white Americans in the study are actually the same as, or even less than, expenditures
for their British counterparts.

(A) Reducing health care spending in the United States by 50% would equalize the amount of money spent on health care
in the United States and Britain. There is an assumption made that there is some reason to do so, perhaps in the hopes that
American health will consequently improve to the level of British health, or perhaps simply to save money. The facts given in
the argument are not sufficient to support this assertion.

(B) Although the recent study indicated that the middle-aged white Americans have poorer health than their British
counterparts despite apparently more expensive health care, there is no evidence that the cost of the health care is a cause
of health or sickness for either group.

(C) It is possible that health care in the United States costs more because the money is being used ineffectively. This assertion
is one potential explanation for the poorer health of the Americans in the study, despite apparently higher spending on health
care. However, more information is needed to conclusively make this assertion, such as proving that money is currently being
wasted, and on what. Additionally, it is unclear exactly how much money is spent on health care for the age group and race
studied, so the poorer health of the American patients does not necessarily tell us anything about the effectiveness of the
money spent on them.

(D) As mentioned previously, the health care costs given in the argument are the average per person per year in each nation,
yet the study only looked at the health of middle-aged white people. It is unclear exactly what amount of money is spent on
health care for the age group and race studied. While it is reasonable to wonder whether the average health care spending
for middle-aged white Americans is less than the average health care spending for Americans in general, we don’t have
enough information to conclude that this is the case.

(E) CORRECT. The study revealed some differences in the health of middle-aged white Americans and middle-aged white
Britons. The study did not indicate the reason for the difference. However, the researchers did eliminate the lifestyle
differences of diet, exercise, smoking, and drinking as causes of the health difference. Thus, it can be conclusively asserted
that something else (other than diet, exercise, smoking and drinking) must account for the difference in health for the two
groups in the study.

2.

This argument essentially asks us to find a conclusion that must be true based on the facts presented by the spokesperson.
When you analyze the answer choices, remember that the correct answer to questions of this type must be the claim that
does not require any additional assumptions.

(A) This statement does not have to be true, since the female voters represented only slightly more than a half of the
electorate. For example, the mayor would be able to receive 2/3 of all voters if he received all the votes of the male voters
(i.e. 45%) and just some substantial portion of the female voters (say 40%).

(B) This statement does not have to be true. For example, the incumbent mayor could have received the vast majority of the
male votes and a smaller share of the female votes.
(C) Since the incumbent mayor received 2/3 (i.e. approximately 67%) of the votes, the maximum number of votes received
by any other candidate could have been around 33%, which is still substantially less than the number of votes received by
the incumbent mayor. Since we do not know how many candidates participated in the election and the proportion of votes
received by each candidate, we cannot conclude that no other candidate received more than 30% of all votes.

(D) CORRECT. Since 55% of the voters were female, the remaining 45% were male; also, since all the voters were between
ages 18 and 70, no voters, either male or female, could have been 75 years old. This answer choice does not require any
additional assumptions and therefore must be true.

(E) Even if the proportion of the male and female voters remains the same, their preferences may change substantially and
there is no guarantee that they will vote in the same way in the next election.

3.
When drawing a conclusion, we must remember not to conclude too much; i.e., do not make unwarranted assumptions. In
this case, we are looking for the conclusion that must be true based only on the information given in the passage without
requiring any additional assumptions.

(A) While we are given information about the percentage of the total budgets spent on marketing, we have no information
about the actual amount of money either company spent on marketing.

(B) While we are given information about the percentage of the total budgets spent on production, we have no information
about the actual amount of money either company spent on production.

(C) Because we have no information on the sale price per copy for either company, we cannot make any conclusions about
the revenue generated by either company. It’s very possible that Making Hits sold its copies at twice the price of the Song
Factory copies, in which case the revenues for the two companies would be the same.

(D) CORRECT. Since Making Hits spent 40% of its budget on production, 30% on marketing, and the rest on overhead, we
can conclude that Making Hits spent 30% of its budget on overhead. Since the Song Factory spent 20% of its budget on
production and 60% on marketing, and met its budget, it could not have spent more than 20% on overhead. Therefore,
Making Hits spent a higher percentage of its budget on overhead than did the Song Factory.

(E) A valid conclusion must be true. While it is possible, and perhaps even likely, that the percentage of the budget spent on
marketing was a driver of sales, this is not necessarily true; there are many other factors that could have affected sales. For
example, it is possible that the Song Factory sold more copies of its 10 albums because the music was better than the music
produced at Making Hits, and not because the Song Factory spent a higher percentage on marketing.

4.
Only two pieces of information are given about Airline A's standing room "seats" proposal. First, that it is geared toward
increasing revenue in order to counteract declining profits. And second, that, since the proposal relates to passenger safety,
it must be approved by the Federal Aviation Administration. Airline A must have concluded that the cost of implementation
of its proposal is less than the revenue that the new seats will generate.
(A) CORRECT. Since Airline A knows that its proposal would have to comply with safety standards, it must have concluded
that the cost of compliance is worth it. In other words, the only way for Airline A to achieve its goal of increasing profit is to
implement ideas that will generate more revenue than they cost. Airline A must therefore have concluded that the standing
room only "seats" meet this criteria.

(B) The statements in the passage imply nothing about whether Airline A believes that the Federal Aviation Administration
will approve the proposal. Although Airline A must believe that the proposal has a chance of being approved (otherwise it's
unlikely to have proposed it), the airline might have proposed its specific plan knowing that it might not be approved or, that
it might have to be changed in certain ways.

(C) Airline A's goal is simply to "counteract declining profits" caused by the high cost of jet fuel. This does not mean, however,
that the proposal must fully mitigate the cost of jet fuel. As long as the proposal increases revenue without a corollary increase
in cost, it will in some way (even if it's relatively small) counteract declining profits.
(D) The passage does not mention any other ways that Airline A has considered increasing revenue. Therefore, it is impossible
to conclude anything about Airline A's perception of its standing room "seats" proposal to any other ideas.

(E) The statements in the passage do not address Airline A's view regarding the safety of the standing room only "seats". It
is very possible that Airline A views its proposal as safe and sees no conflict between passenger safety and increasing revenue,
much less that it has made any determination about the relative importance of these two issues.

5. The correct answer choice is (C). The causal relationship in this problem appears in the premise, and the argument is
structured as follows: Premise: Most antidepressant drugs cause weight gain. Premise: Dieting can help reduce the amount
of weight gained while taking such antidepressants. Conclusion: Some weight gain is unlikely to be preventable. Note that
the causal premise specifically states that “most” antidepressants cause weight gain, not necessarily all antidepressants. Also,
the second premise specifically refers to antidepressants causing weight gain (the use of “such” indicates this). The second
premise also indicates that the amount gained can be reduced, not that dieting can stop weight gain. Perhaps the
antidepressants cause a twenty pound weight gain, but dieting can reduce that to a ten pound total gain. The question stem
is a Must Be True, and thus you must accept the stimulus information and find an answer that is proven by that information.
Answer choice (A): This is an Exaggerated answer. The stimulus indicates that most antidepressants cause weight gain,
leaving open the possibility that some do not. This answer choice references any antidepressant drug. Further, the stimulus
does not address the role of a physician or the advisability of prescribing certain drugs under certain conditions. The benefits
of prescribing an antidepressant that causes weight gain to an overweight patient may well outweigh the negatives (pun
intended). Answer choice (B): This is also an exaggerated answer. The stimulus allows for antidepressants that do not cause
weight gain. Answer choice (C): This is the correct answer. Some individuals taking antidepressants that cause weight gain
will gain weight even though dieting can reduce the amount of the gain. Answer choice (D): This is an Opposite answer. The
stimulus and correct answer both indicate that people taking the weight gain-causing antidepressants will gain weight
regardless of whether they diet. Thus, the weight gain cannot be attributed to a lack of dieting. Answer choice (E): This
answer is too strong. Not all patients necessarily take antidepressants that cause weight gain, so those that do not might not
need to diet to maintain their weight.

6.
This argument concludes that spelling bees should only use anglicized words in the dictionary because spelling should be the
only criterion that determines the winner. To make this point, the author must believe that correctly spelling spoken foreign
words requires knowledge of the phonetics of the particular language.

(A) This choice does not have to follow from the argument, as a correct inference must. The winner did not have to know
how to spell most of the anglicized words in the dictionary, only the ones that she was given.

(B) This choice does not have to follow from the argument, as a correct inference must. All contestants don’t have to find
foreign words more difficult. For instance, the child of German parents might find it easy to spell Ursprache.

(C) This choice does not have to follow from the argument, as a correct inference must. In fact, it must follow from the
argument that contestants should be judged only by their spelling ability, not by their facility with "all aspects of language."

(D) CORRECT. The author wants to exclude foreign words because spelling bees should be based only on spelling ability, not
on "knowledge of linguistics and international phonetics." Thus, the author must believe that spelling foreign words correctly
when they are spoken requires knowledge of the phonetics of the foreign language in question.

(E) This choice does not have to follow from the argument, as a correct inference must. While this might be factually true,
no where in the argument is there any comparison between the number of words in English and that of any other language.
7.
The correct answer choice is (C). Unlike the first two questions in this set, this is a Main Point question. As with all Main Point
questions, if you follow the Primary Objectives, the question should be easy and you should have a solid answer in mind
before attacking the answer choices. The first sentence of the argument is a premise. The second sentence is divided into a
premise and a conclusion: the premise is introduced by the indicator “since,” and the conclusion of the argument is introduced
by the indicator “it follows that.” As stated in the argument, the conclusion is “a requirement for an athlete to become a
champion is a superior mastery of athletic techniques.” Answer choice (A): This is a Mistaken Reversal of the conclusion. Note
how right away the test makers are trying to test you to see if you will fall for an answer that uses the elements of the
conclusion but in the wrong relationship. You must be on guard at all times! Answer choice (B): Although this answer is likely
to be true in the real world, this is not stated in the argument. The only comment made on muscle strength is that top
athletes do not differ greatly from each other in muscular strength. Answer choice (C): This correct answer is a paraphrase
of the conclusion, and the diagram for this answer choice is the same as the diagram of the conclusion. Answer choice (D):
This is a classic incorrect Main Point question answer choice. Although the author would agree with this statement, this is not
the main point of the argument. Answer choice (E): This answer is similar to answer choice (D). Again, the answer choice
repeats part of the argument, but this answer does not capture the main point of the argument.

8.
This passage relates information from two studies concerning high school seniors: the first discusses the financial
responsibilities of high school seniors, while the second explains the coursework in finance taken by typical high school
seniors. On the GMAT, a proper response to a draw-a-conclusion question must be directly supported by evidence from the
passage.

(A) Although it might be true that schools would be wise to educate students in finance, this is an opinion; it doesn't
necessarily need to follow from the given evidence. A conclusion must be directly supported by evidence from the passage
without any additional information or assumptions.

(B) The fact that one-third of high school seniors claim “significant financial responsibilities” to their families does not
necessarily mean that these same students work “part-time jobs after school.” There are many possible ways that these
students might earn money for their families. If they do work, they might work on weekends or over the summer, for example.

(C) CORRECT. The first study states that one-third of all high school seniors have significant financial responsibilities to their
families. The second study states that 80% of seniors have opened a bank account, and of this 80%, one-third has bounced
a check. The number of seniors that has bounced a check (one-third of 80%) is fewer than the number of seniors with
significant financial responsibilities to their families (one-third of 100%).

(D) The passage states that certain high school seniors who contribute to the food, shelter, or clothing for themselves or
their families rate themselves as having significant financial responsibilities. This does not mean that any high school senior
who contributes to these categories has significant financial responsibilities.

(E) The passage states that one-third of high school seniors say that they have “significant financial responsibilities.” This in
no way indicates that the other two-thirds have “no” responsibilities. Because no information is given about the other two-
thirds of the students, a reasoned conclusion cannot be drawn about them.

9.
The analyst presents several points about the business talents of creative professionals. In drawing a conclusion from the
analyst's argument, we must be careful to choose a provable claim, whether or not this claim pulls together all the premises.
We also must avoid extending the analyst's argument or selecting statements that are too extreme. Finally, we must not
allow this process to be clouded by reactions to the content of the argument; whether or not we agree with the premises,
we have to find a provable conclusion.

(A) This choice takes the passage's claim that creativity and business acumen rarely go hand in hand to an extreme. The
analyst does not assert that absolutely no successful people are creative.

(B) CORRECT. The passage states that most creative types are less skilled in business than the average white-collar worker
who does not work in a creative field. This implies that some creative types are not less skilled than the average white-collar
worker who is not creative.
(C) This choice again takes the passage's claim that creativity and business acumen rarely go hand in hand to an extreme.
Creativity and business acumen are not mutually exclusive.

(D) The passage does not say that all white-collar workers are successful, nor does it say that no creative professionals are
successful.

(E) The passage makes a distinction between creative talent and business acumen. This does not mean that there are no
aspects of business that fall under the realm of creativity.

10.
The passage states that health savings accounts will undermine the health of the public because people will not use them for
preventive care. Furthermore, people who cannot afford them will not be able to receive even basic care such as vaccinations.
The correct answer will be a conclusion that can be supported solely by the facts stated in the argument, without relying on
outside information or additional assumptions.

(A) The argument does not provide enough information to conclude that wealthy individuals will not be affected negatively
by health savings accounts. The argument never specifically mentions wealthy individuals, just people in general.

(B) The argument does not provide enough information to conclude that private health insurance will no longer be available.
In fact, private health insurance is never mentioned.

(C) The author argues that people will not get regular preventive examinations, and will therefore not receive medical
attention until diseases are advanced. This logic, even if true, does not allow us to conclude that most diseases are detected
during regular preventive examinations.

(D) CORRECT. The argument states that "poor people, who will not be able to afford health savings accounts, will no longer
receive vaccinations". Based on this statement, it is reasonable to conclude that some people without health savings are likely
to contract infectious diseases.

(E) The argument does not provide enough information to conclude that the causal relationship between an individual's health
and that person’s medical care has been adequately documented. In fact, neither the link between medical care and health
nor documentation of such a link is directly discussed.

11.
This argument provides that albinos, or people whose bodies do not produce melanin, are unusually susceptible to solar
exposure. This suggests a connection between the production of melanin in humans and protection from sunburn and other
sun-related ailments; we should look for a conclusion that draws this connection.

(A) The argument does not indicate that people born with albinism somehow develop other natural defenses against sun-
related health issues.

(B) This conclusion is too extreme to be supported by the argument; nothing in the argument suggests that humans whose
bodies produce high levels of melanin can "easily ignore" sunburn or other sun-related health issues.

(C) There is no indication in the argument that sunburn reduces melanin production.

(D) CORRECT. The argument does strongly suggest that melanin plays some role in protecting the skin from developing
sunburn and other sun-related ailments, since albinos do not produce melanin and are unusually susceptible to sun-related
ailments.

(E) It is not suggested in the argument that an albino person could not protect him or herself from solar exposure through
artificial means, e.g. wearing protective clothing or powerful sunblock.
12.
The text tells us that celiac disease results when the body mistakes gluten for a harmful pathogen, causing damage to the
intestine. We are also told that gluten is a protein found in certain grains, and that people suffering from celiac disease must
eliminate it from their diets. Finally, we are told that symptoms of the disease include cramps, bloating, and anemia. We need
to find an answer choice that is inferable from these facts alone.

(A) Anemia is just one of several symptoms of the disease. We do not know whether everyone who has the disease will also
develop anemia.

(B) We do not know whether eliminating gluten will cure the disease, only that people with the disease must not eat gluten.
Perhaps the disease will exist anyway in a latent form.

(C) We do not know whether the symptoms mentioned are also symptoms of other conditions.

(D) We do not know whether gluten is found only in grains. It may exist in other foods as well.

(E) CORRECT. If the body mistakes gluten for a harmful pathogen, then it must be true that the body cannot always recognize
harmless substances.

13.
When drawing a conclusion, we must remember not to conclude too much. In this case, we are looking for the conclusion
that comes directly from the information given without requiring any additional assumptions.

(A) CORRECT. In general, Mayville experiences a greater number of delays per 100 flights than Newcomb does. However,
when delays caused by bad weather are discounted, Mayville has 5 fewer delays per 100 flights. Since the two airports run
the same number of flights, bad weather must cause a greater number of delays at Mayville Airport than at Newcomb Airport.

(B) While we can conclude that Mayville experiences a greater number of delays caused by bad weather, we cannot make
any conclusions about the relative weather conditions at either airport. It is very possible that the airports experience the
same weather, but that for some reason Newcomb’s airport is better equipped than Mayville’s airport to handle inclement
weather (e.g. the fleet of aircraft at Newcomb is better suited to bad weather, or the air traffic controllers at Newcomb are
more competent, etc.). Some other factor could cause the discrepancy in weather related delays aside from the weather
conditions being different.

(C) While we can conclude that Mayville experiences a greater number of delays caused by bad weather, we have no
information about delays caused by mechanical problems. There may be other reasons aside from either bad weather or
mechanical problems that account for departure delays (e.g. human error).

(D) We have no information regarding the quality of airplane fleets at either airport.

(E) The argument gives no information about arrival delays.

14.
The cost of flour from the local mill is higher than the cost of the flour from the out-of-state mill. However, when purchasing
from the out-of-state mill, Barry’s Bagels must pay shipping and handling fees that would not apply to a purchase from the
local mill. Purchasing the flour from the out-of-state mill will only be cheaper if those shipping and handling fees are smaller
than the difference in the flour costs of the two suppliers. Also, we cannot assume any additional information or move beyond
the scope of the given premises in order to find the conclusion.

(A) Lower production costs could explain the lower price of the flour from Isadore’s Interstate Mill, but there may be a variety
of other reasons. We cannot state this conclusively.

(B) It is possible that the number of local flour mill jobs would be decreased, but no evidence in the passage leads to that
conclusion.
(C) CORRECT. This statement properly identifies the point that, for ordering from an out-of-state mill to be less expensive,
the shipping and handling fees must be less than the difference in the flour costs of the two suppliers. Say, for example, that
a batch of flour costs $100 from Larry’s Local Mill. The passage tells us that the same batch would cost $90 from Isadore’s
Interstate Mill, yet when purchasing from Isadore’s, shipping and handling fees would apply. We are told that Isadore’s total
cost is cheaper than Larry’s, so mathematically that is: $90 + Shipping & Handling < $100, which means that Shipping &
Handling < $10 = 10% of the cost of flour from Larry’s.

(D) If shipping and handling fees were more than 10 percent, purchasing from the out-of-state supplier would be more
expensive, not less.

(E) Higher efficiency could explain the lower price of the flour from Isadore’s Interstate Mill, but there may be a variety of
other reasons. We cannot state this conclusively.

15. The single sentence prior to the last sentence is lengthy and contains the information required to fill in the blank at the
end of the question. The argument topic is language acquisition; the author indicates that “one cannot attribute such
acquisition solely to environmental influences” and then immediately follows that phrase by saying “innate mechanisms also
play a role.” Hence, the author feels that both environment and innate mechanisms play a role. The last sentence then
prefaces the blank by saying, “the most reasonable question that ought to be studied is whether______.” The most logical
answer would be one that addresses the relative roles played by environment and innate mechanisms in language acquisition.
Let’s examine the answer choices: Answer choice (A): The argument is not about whether language acquisition can ever be
explained, but about what influences exist on language acquisition and to what degree. Answer choice (B): The psychologist
asserts this statement at the end of the first sentence and if this were a Must Be True question, this would be the correct
answer. But, this is more than a Must Be True question and the correct answer must meet the Main Point criterion. So,
although this answer choice is true according to the psychologist, it does not capture the point of the argument as indicated
by the last sentence and is therefore incorrect. Answer choice (C): The argument does not attempt to establish that language
acquisition is solely the product of innate mechanisms, but that innate mechanisms have some influence, as does
environment. This answer choice tries to confuse test takers by going in the opposite direction of the psychologist’s statement
that “one cannot contribute such acquisition solely to environmental influences.” This does not mean that we can therefore
attribute such acquisition solely to innate mechanisms. Answer choice (D): “Parents and peers” would qualify as environmental
influences and the argument is not about determining if the environmental influence is the most important factor, but about
the relative roles played by environment and innate mechanisms in language acquisition. Answer choice (E): This is the
correct answer, and this is the only answer that addresses the relative roles of environment and innate mechanisms. Note
that the language of the answer choice could have indicated that either play a greater role because what ought to be studied
is a question that determines which is a greater influence.

16.
The passage presents information about what office employees who work 8-hour days and who have worked at home told a
certain magazine. The first piece of information is about what some of those office employees actually do: 25 percent of
office employees actually work less than an hour on days that they work at home. The second piece of information is about
what some of those office employees believe: 90 percent believe that they are more productive working at home than at the
office. A proper GMAT conclusion must be provable by those two pieces of information.

(A) The passage only provides information about the working hours of 25 percent of the office employees. The passage does
not provide any information regarding the working hours of the other 75 percent, hence, it is not possible to conclude anything
about the office employees on average. For example, it is possible that the other 75 percent of the office employees work 14
hour days when working from home. It is also possible that they work 6 hour days when working from home.

(B) The passage provides no information about the actual productivity of any of the office employees. It only provides
information about what the office employees believe about their productivity.

(C) CORRECT. 90 percent of the office employees believe that they are more productive at home than at work. At the same
time, 25 percent of the office employees actually work fewer hours when they work at home than when they work at the
office. The overlap between these two groups is at least 15 percent of all of the office employees. This group of employees
believes that they are more productive at home than at work and yet this group actually works fewer hours at home than at
work. Thus, these employees must not define productivity exclusively in terms of the number of hours worked.
(D) The passage discusses the actual work hours of 25 percent of the office employees. Then it describes the beliefs of 90
percent of office employees regarding their productivity. First, there is no necessary link between an individual's beliefs about
his or her productivity and that individual’s actual productivity; hence, no conclusion can be made regarding actual productivity
from the information about beliefs. Second, the number of hours worked alone is not an indication of productivity; it is
possible, for example, that an employee who works 1 hour is more productive in terms of work done per hour than when he
works 8 hours and yet that employee might still accomplish more total work when working 8 hours. Therefore, it is not
possible to conclude anything regarding productivity for any of the office employees.

(E) The fact that 90 percent of the office employees believe they are more productive at home than at work does not
necessarily contradict the fact that 25 percent of the office employees work fewer hours at home than at work. It is possible
to work fewer hours and still be more productive.

17.
The passage gives information about Monday’s business at two lemonade stores. The question asks us to make an assertion,
or conclusion, based on the information provided. The answer choice that requires no additional assumptions will be the
correct answer.

(A) This conclusion is incorrect. If Daisy’s sells its lemonade at a lower price than the Lemon Shack, and if the stores reported
identical revenues for the day, then Daisy’s sold more cups of lemonade than the Lemon Shack, not less.

(B) We know nothing about the quality of lemonade at either store.

(C) CORRECT. If the stores reported identical revenues and identical profits, the profit equation Profit = Revenue – Cost tells
us that their costs must have been identical as well.

(D) We know nothing about the preferences of lemonade consumers.

(E) We know nothing about the market conditions surrounding either store. Therefore, we cannot make any conclusions
about what might happen if the Lemon Shack were to lower its prices. It is very possible that the Lemon Shack could in fact
sell many more cups per day at lower prices, and it’s possible this could lead to higher revenues.

18.
In this argument, a cause-and-effect relationship is presented between American scientists signing long-term contracts with
foreign companies and the government's restrictions on stem cell research. This cause-and-effect relationship is the key to
the correct answer.

(A) CORRECT. If American scientists signed the contracts because of U.S. restrictions, we can infer that the new companies
they signed with were under fewer restrictions. Therefore, at least some foreign companies must work under fewer
restrictions than some American companies do.

(B) While it is possible that once the restrictions are banned American companies will want to hire more scientists and will
seek them overseas, there are too many unknowns between the premises we have have been given and this conclusion. It
is doubtful that an increase in the number of immigrating stem cell research scientists would have a significant impact on the
number of foreign professionals overall.

(C) This passage is about government restrictions; we are given no information about financial backing. Beware of extreme
statements such as in all parts of the world.

(D) We are not given any information regarding America's current or future position in terms of stem cell research. Though
restrictions and scientists switching companies are two issues related to a company's prosperity, we are given no information
about how these directly affect America's position.

(E) We are not given any information that will help us predict the behavior of the scientists in the future.
19.
The argument presents data about deaths due to medical errors. A campaign designed to reduce these deaths due to lethal
errors does indeed reduce the number of deaths over an 18-month period. No conclusion is presented in the body of the
argument; in fact, the question asks us to "infer" or draw a conclusion from the given statements. The conclusion, therefore,
will be found in the answer choices; our task is to find a statement that follows directly from the given statements without
introducing any new information or assumptions.

(A) While this might generally be true in the real world, the given information does not address whether doctors and nurses
are too careless in conducting their jobs.

(B) While the campaign did save a large number of people, we cannot say that every single person who would have died was
saved; this answer choice is too extreme.

(C) The argument does not provide information to make predictions about the future; in addition, this answer choice is
extreme. Common sense tells us that we cannot prevent every single medical error in the future.

(D) CORRECT. This statement can be inferred from the original argument. If the campaign saved the lives of people who
otherwise would have died of medical error, then the absence of the campaign would have meant that many of those people
might not have been saved. Notice that this answer choice is more of a restatement of the given information, rather than
what we would consider a true conclusion in the real world; this is typical of correct answer choices on GMAT inference
questions.

(E) While this sounds like a good idea, given the evidence, the argument does not provide information to make predictions
about the future.

20.
According to the statements, the companies that own private aircraft for business use are fully in compliance with the relevant
law, which is summarized. A correct inference will be a statement that must follow from at least part of the premises given.

(A) It does not have to be true that the law costs the businesses money, as no evidence about the relative costs is given.

(B) This choice is an irrelevant comparison, as the preferences of the executives are not the concern of the statements.

(C) This choice does not have to follow, as there is no information given about the travel arrangements made by large
companies. The statements only indicate that the majority of private planes are not owned by large companies.

(D) There is no information given about the travel arrangements of upper level executives and no reason to believe that
those with the companies discussed do not comply with their companies’ policies.

(E) CORRECT. If, as the statements indicate, the companies are in full compliance with this law, it must be true that the
executives following their guidelines also are.

21.
Antoine is alarmed that the number of children on antipsychotic medication has increased by 73 percent. Lucy begins her
reply with “but,” indicating that she is about to counter either Antoine’s facts or his alarm; she accepts his facts but addresses
his alarm. If the number of children taking antipsychotic medication is still within the normal range, the rate at which the
total number has increased is not cause for alarm. Lucy shows that even though the total number of children on such
medications has increased, children still take antipsychotic drugs at an even lower rate than do adults, and the current adult
rate is considered normal.

(A) Lucy’s argument is that the rate of adults taking antipsychotic drugs is normal, and the rate of children taking such drugs
is even lower. Thus, the argument does not lead to the conclusion that the current level of antipsychotic drug use in children
is abnormally high.

(B) CORRECT. This statement properly identifies the conclusion to which Lucy’s argument is leading.
(C) Lucy does not dispute Antoine’s statistics, and her argument is designed to make a conclusion about their interpretation,
not about their accuracy. Also, it is mathematically possible for the increase in the use of such medicines to be 73 percent,
as Antoine cites. An increase from 3.8 to 6.6 out of every 1,000 represents an approximately 73 percent increase.

(D) There is no information about what might happen in the future, so no conclusion can be made about whether the use of
antipsychotic medicines by children will increase or decrease, or at what rate that might happen.

(E) No information in Lucy’s argument supports the conclusion that the rate of antipsychotic drug use in children is higher
than that reported.

22.
The statements indicate that DNA analysis has become more sophisticated and thus produced better results. The passage
comments on a "new analysis" and “a very startling new theory” about the origins of modern man and chimpanzees. An
appropriate inference would focus on what must follow from those statements.

(A) This choice does not have to follow from the information given, as GMAT inferences must. While the hybridized species
must have had positive attributes, it certainly did not have to combine the best of pre-humans and chimpanzees.

(B) This choice does not have to follow. Politics and religion are outside the scope of this argument and thus the existence
of such pressures is not a given.

(C) This choice is too extreme to have to follow from the statements. We don’t know any specifics about the capacity or
direction of the research.

(D) CORRECT. The passage states that a "new analysis of the genetic links between early humans and chimpanzees has led
to a hypothesis that the two species diverged more recently than previous estimates indicated." The fact that this is a "new"
analysis suggests that prior analysis had already established the links; also, since the new theory has altered the time frame
for the divergence, it follows that some genetic link and subsequent divergence was already ascertained.

(E) This choice does not have to follow from the information given. Nothing is said about the differences between huma n
and chimpanzee DNA. We do not know if the differences involve complexity or something else, such as the composition of
the DNA.

23. The correct answer choice is (D). Like the previous problem, the conclusion is in the middle of the argument and is not
prefaced by a conclusion indicator. Get used to seeing this format on Main Point questions! The author states that prediction
has been made possible by reducing phenomena to mathematical expressions and that some social scientists want to have
this same power. The author argues that it would be a mistake to allow social scientists to have this ability. The conclusion,
therefore, is “But this would be a mistake.” Answer choice (A): The author says, “some social scientists also want the power
to predict accurately,” so the author would likely agree with this statement. Regardless, this is not the main point of the
argument. Again, be careful with answers that are true according to the author—do they also address the main point? Answer
choice (B): The author might very well agree with this statement, although there is not enough information to assert that
this statement is true based on the stimulus (the words “more important” are a bit strong). Regardless, this answer choice
does not address the main point of the argument and is therefore wrong. Answer choice (C): While the social scientists may
believe this is true, the author’s point is a different one—that social scientists ought not to perform a mathematical reduction.
And, because the author believes that prediction is apparently made possible by reducing phenomena to mathematical
expressions, the author would likely disagree with this statement. Answer choice (D): This is the correct answer. The
conclusion states that it would be a mistake for social scientists to have the ability to reduce phenomena to mathematical
expressions. Answer choice (D) is a paraphrase of that idea. Answer choice (E): This point is not addressed in the stimulus.

24.
The government is attempting to limit the release of carcinogens by chemical plants, but it is permitting the chemical industry
itself to monitor the plan’s implementation. The author believes this to be an unacceptable proposal, since the past behavior
of certain chemical companies indicates that they will increase their emissions of toxic carcinogens.

(A) The author states only that certain chemical companies cannot be trusted; saying that no company can be trusted is too
extreme.
(B) The argument states that the chemical industry does release certain carcinogens but does not suggest that it releases
the majority of carcinogens.

(C) CORRECT. This statement properly identifies the author’s argument that the net result of the plan will be an increase of
toxic carcinogens into the air.

(D) The argument is about a certain proposal in a specific industry; it is too far reaching to argue that the author believes
the government should always monitor all of its proposals.

(E) This is not the point of the argument. Moreover, it is never suggested that these chemicals are more hazardous than
other cancer-causing chemicals.

25.
The oil executives argue that the leases should not be renegotiated because a duly executed contract should be strictly
honored. The best answer choice will reflect this argument.

(A) This is a tempting answer because the windfall from the government error is certainly an opportunistic event that is unlike
to repeat itself. However, it does not reflect the argument that the oil executives have made justifying their opposition to
renegotiating the leases and therefore is not the best answer.

(B) The oil company executives would not agree with this answer choice: should the positions be reversed, they would
certainly not want the government to hold them to a contract that contains an error unfavorable to them.

(C) While it is true that the taxpayers’ loss is the oil companies’ gain, this answer choice does not reflect the argument that
the oil executives have made justifying their opposition to renegotiating the leases and therefore is not the best answer.

(D) CORRECT. The executives argue that the government should strictly honor the agreement already in place; i.e., they
should not “change the rules in the middle of the game.”

(E) While the unexpected windfall may indeed be “sweet” to the oil executives, getting “revenge” requires that some prior
injustice was inflicted by the government upon the oil companies. There is nothing in the passage that indicates or implies
such.

26.
The argument first presents a rule (only students with at least a 2.5 GPA can play school sports) and then a fact (Amy is on
a school sports team). It then draws a logical conclusion: Amy must have at least a 2.5 GPA. Questions that ask us to
summarize the main point are asking us to restate the conclusion.

(A) Although this statement is true, it is not the main point of the argument. The main point is the conclusion that Amy must
have at least a 2.5 GPA.

(B) Amy's tennis skills are irrelevant to the entire argument.

(C) This choice states an opinion about the presented rule; it does not restate the conclusion.

(D) CORRECT. This choice restates the conclusion that Amy must have earned at least a 2.5 GPA.

(E) Amy could still be captain if her GPA were "lower," as long as her GPA didn't drop below a 2.5.

27.
The tension indicated in the argument is that between the benefits of focusing on one primary product or service and the
inherent risks of relying too heavily on an undiversified income stream. The correct answer choice must exemplify this
tension.
(A) CORRECT. This example correctly identifies a company with a clear, narrow focus – a local messenger service - that
enjoys a competitive advantage because of its reputation for speedy deliveries. At the same time, its undiversified income
stream becomes problematic when a factor outside of its control – the relocation of local businesses out of state – forces it
to lay off employees.

(B) This example does not specify any particular focus of the advertising agency nor does it demonstrate any major cash flow
consequences due to the loss of the client.

(C) In contrast to the predicament described in the passage, this example describes a company that is interested in
diversifying its holdings.

(D) The fact that the construction company is attempting to increase its profits does not illustrate the tension described in
the passage.

(E) Though the specialty sandwich store that uses local ingredients might be called a very focused company, there is no
evidence presented that this focus has had any adverse effect on cash flow.

28.
The passage provides some specific information about the effects of calorie restriction. In rats and mice, this diet is known
to prolong life by preventing diseases. In a study of moderately overweight humans, insulin levels and body temperature
decreased. A proper GMAT inference will be based on the specific information provided, without relying on any significant
assumptions.

(A) The passage states that calorie restriction in mice and rats prolongs life by preventing diseases. The human study had
much more limited findings – that calorie restriction in moderately overweight humans decreases insulin levels and body
temperature. While these traits are known to be associated with longevity, there are no data that link calorie restriction itself
to prolonged human life. Additionally, calorie restriction may have other unstated effects, unrelated to longevity. There is no
information in the passage that indicates whether these effects are the same in humans as in mice and rats. Finally, the use
of the term "humans" is far too general; the study dealt only with moderately overweight humans and so any inference would
need to be restricted to this subset of individuals.
(B) While the passage indicates that certain traits known to be associated with longevity are found in moderately overweight
humans who reduce their calorie intake, this is far removed from the conclusion that calorie intake will actually increase a
human's lifespan. Additionally, the use of the term "humans" is far too general; the study dealt only with moderately
overweight humans and so any inference would need to be restricted to this subset of individuals.

(C) The study observed that individuals with the greatest percentage decrease in their calorie intake demonstrated the
greatest decrease in insulin levels and body temperature. This shows a strong correlation between calorie intake and insulin
levels. However, this correlation is not necessarily direct. It is possible that this correlation holds, but only up to a point. For
example, it might be the case that any reduction in calorie intake over 50 percent does not result in any additional insulin
level decreases. Moreover, the passage only draws this correlation for individuals with the greatest percent decrease in calorie
intake. It is very possible that individuals with a relatively low decrease in calorie intake exhibit the exact same decrease in
insulin levels as individuals with a moderate decrease in calorie intake.

(D) The study makes no reference to the health of individuals who reduce their calorie intake. It tries to draw some connection
to the longevity of those individuals, but longevity is not the same as health. An individual could live a very long, unhealthy
life.

(E) CORRECT. The passage states that the greatest decrease in insulin levels was observed in individuals with the greatest
percentage change in their calorie intake. This means that some individuals in the study reduced their calorie intake by a
greater percentage than other individuals in the study. The passage also states that the study participants reduced their
individual calorie intakes by "at least 25 percent." Thus, one can safely infer that there were some participants who reduced
their calorie intake by more than 25 percent.
29.
The argument explains that the new “Click It or Ticket” law is generating controversy. Under the new law, drivers can be
cited for not wearing their seat belts, even in the absence of an additional driving infraction. Any acceptable inference must
be directly supported by evidence from the text.

(A) CORRECT. The entire controversy is based on the new law that allows motorists to be cited, even in the absence of an
additional infraction. Thus, it follows that prior to the passage of this law, an additional driving infraction must have been
necessary in order to stop and cite an individual for not wearing a seat belt.

(B) Search and seizure laws are never mentioned in the text. This answer choice is outside the scope of the argument.

(C) Laws in other states are never mentioned in the text. This answer choice is outside the scope of the argument.

(D) Though the text states that the new regulation might save countless additional lives, the effectiveness of the previous
laws are never mentioned.

(E) No preference is stated between law enforcement groups and the citizens' groups. This answer choice is simply an opinion
that is unsubstantiated by the text.

30.

We are asked to determine what we can infer based upon the given information; on the GMAT, whatever we infer must be
based only upon the argument itself and cannot extend beyond the scope of that argument.

(A) The argument says nothing about requirements in other states; this choice is out of scope.

(B) While it may be illegal for those under 21 to consume alcohol, the argument makes no mention of special rules for this
age category. Indeed, the argument says that "hunters" have to sign the pledge, not just "hunters aged 21 and over."

(C) CORRECT. The argument says that the hunter must have completed a safety program within the past five years. If he
has not, then he will have to do so before he can be eligible for a permit. (Note that this choice doesn't say he will get a
permit if he completes the program again; we don't know what other requirements he may not meet. We only know that he
will have to retake the program, at least, before he can become eligible.)

(D) This choice mentions someone who "isn't 18 years old." Someone who isn't 18 can be either younger than 18 or older
than 18 and, according to the argument, those older than 18 are eligible for permits if they complete all of the requirements.

(E) While this may be true as a general rule, it is outside of the scope of the argument, which mentions nothing about danger
or why these rules were enacted. In addition, any requirements cannot "ensure" that nobody ever gets hurt; they can only
help to minimize risks.

31.
The passage describes how public complaints about one issue (corruption in unregulated service industries) can have
consequences (increased costs) that lead to new public complaints. The correct answer will tie together this information
without assuming too much.
(A) This answer choice is too extreme. While regulation does often result in increased costs, which consumers don't like, the
regulation presumably halts the corruption, which consumers also don't like.
(B) This answer choice is a judgment call based on the idea that the increased costs are less desirable than the corruption.
The passage does not provide information to support this claim.
(C) The passage doesn't address what types of services are subject to public protest. In addition, the word "only" is extreme.
(D) CORRECT. This answer summarizes what the passage describes – a cycle of public discontent. Notice that the correct
“inference” or “conclusion” here isn't much more than a summary of the premises in the passage.
(E) In the passage, the author says "regulation often leads to increased costs for the consumer" due to decreased competition.
Always is not the same as often. This answer choice is too extreme.
32.
For questions asking us to draw a conclusion, we need to base the conclusion only on the information presented; we cannot
add any new information or assumptions. In addition, remember that we are not required to use all of the information
presented in the premises, though the more information we can include, the better (as long as we don't take it too far!).
(A) The passage says that grapefruit or grapefruit juice ingested within an hour will "significantly" diminish the effectiveness
of the medication. This does not mean that grapefruit or grapefruit juice ingested more than an hour before the medication
is ingested will have no effect on the medication; it may have a mild effect.

(B) The passage speaks only of taking the medication after ingesting grapefruit; it says nothing about the effects of eating
grapefruit after taking the medication.

(C) The passage implies that the pain receptors that the specific medication works on are the pain receptors that are affected
by grapefruit. This does not mean that these are the only pain receptors in the brain.

(D) CORRECT. The passage says that the medication has been shown to be less effective when taken after grapefruit
consumption. Furthermore, grapefruit has been shown to affect the binding of the medication to pain receptors. It can be
concluded that effective binding is needed to enable the effectiveness of the medication.

(E) This choice is out of scope. We are given no information about possible research or design of new migraine medications.

33.
We are asked to draw a conclusion based on the educator's statements. The educator states that children who demonstrate
early talent for music are encouraged to pursue it while children who do not show such talent are not encouraged and thus
deprive themselves of the opportunity to develop a latent talent. We must find an answer choice that is based only on these
statements.
(A) This choice states that music education should not devote special attention to talented students. This goes beyond the
scope of the educator's statements.
(B) This choice states that everyone has the potential to learn music. This goes beyond the scope of the educator's statements.
The educator is arguing against classifying students as musically inept at an early age because they might have latent talent
that is not showing itself. He is not necessarily saying that everyone has the potential to learn music.
(C) CORRECT. By referring to the latent talent that some children may be neglecting, the educator is implying that not all
talent shows its face at an early age.
(D) The fact that children who are directed towards other activities have learned to think of themselves as musically inept
doesn't mean that children are particularly sensitive to criticism from adults. The being "directed towards other activities" is
not necessarily best characterized criticism, and furthermore, it is not just children that tend to think themselves incapable of
something if they don't partake in that activity.
(E) The educator is not necessarily claiming that all children should study music.
The correct answer is C.

34.
The correct answer is C. The text states that preparation, like equipment, is a major factor in the risk of injury during high-
risk activities. People who are poorly trained run a higher risk of injury even if provided with the best equipment. From this
we can infer that whatever benefits might derive from the best equipment can be negated by improper training. This is choice
C.

35.
The correct answer is C. The passage that heart scans take less time than angiograms and do not require recovery time.
They are also more sensitive than angiograms. But they use more radiation than other diagnostic procedures. And finally,
their sensitivity can result in detection of harmless abnormalities that may worry patients. We are asked to find among the
choices a conclusion that can be drawn on the basis of that information.
Choice A states that a heart scan is safer than an angiogram. We are given no information about the relative safety of the
procedures. Incorrect.
Choice B states that patients should not be concerned about heart abnormalities that appear in a heart scan. This
misrepresents the information in the passage. The passage simply stated that the scans may pick up harmless abnormalities,
not that all abnormalities are harmless. Incorrect.
Choice C states that a heart scan could result in indirect harm by causing a patient to undergo risky unnecessary procedures.
This is supported by the passage, which states that the scans could result in undue concern and treatment. Correct.
Choice D states an angiogram is the appropriate treatment for most patients. The information in the passage does not support
this. In fact, if anything, the passage would seem to support the contradictory assertion that heart scans are more appropriate
for most patients. Incorrect.
Choice E states that a heart scan is more expensive. We are given no information about cost. Incorrect.

36. When reading the stimulus, your eye should be drawn to the modifier and indicator words, which are underlined below:
The scope of the stimulus is relatively broad, and aside from the word “entirely,” most of the modifiers are not absolute.
Now, look at the rest of the problem and see how several of the answer choices attempt to prey upon those who did not
read the stimulus closely. Here are the question stem and corresponding answer choices for the stimulus above: With the
previous discussion in mind, let us analyze the answer choices: Answer choice (A): The very first word—“all”—should be a
red flag. Nowhere in the stimulus do we have support for stating that all damaging wavelengths are filtered out by the ozone
layer. The stimulus only states that the ozone layer filters “some” wavelengths and lets others through. Some of those that
are filtered are dangerous, as indicated by the last sentence. Surprisingly, about 10% of all test takers select this answer
choice. Answer choice (B): We know that many animal species could suffer severe eye damage, and from this we can infer
that some of them live in areas threatened by the ozone layer. We do not know that few of the species live in non- threatened
areas. Do not forget the Fact Test—it will eliminate any answer choice without support. Answer choice (C): Nothing in the
passage proves this answer choice. If you selected this answer thinking that “many” implied “not all,” then you made a
simple, correctable mistake. “Many” can include “all.” Answer choice (D): Again, watch those modifiers! One reason the
answer choice is incorrect is because it references “most” species when the stimulus only discusses “many” species. Answer
choice (E): This is the correct answer. We can follow the chain of connections in the stimulus to prove this answer: the
ozone layer filters some wavelengths of light; holes in the ozone layer are dangerous, but one previously overlooked danger
of the holes is possible eye damage for many species. From these two statements we can infer that the holes must be letting
some damaging wavelengths of light through. This is essentially what answer choice (E) states. The lesson from this question
is simple: read closely and pay strict attention to the modifiers used by the author. Even though you must read quickly, the
test makers expect you to know exactly what was said, and they will include answer choices specifically designed to test
whether you understood the details.

37. The correct answer choice is (B). The stimulus is a fact set and offers a solution for dating an event when the usual
sources offer conflicting chronologies: 1. Minimize the number of competing sources, possibly by eliminating the less credible
ones. 2. Independent of the usual sources, determine which date is more likely to be right. Notice how the test makers throw
in the word “historiography” in order to be intimidating. As usual, you do not need to know the meaning of this word (or any
unusual word) in order to continue with the problem. The remainder of the sentence makes clear that dating an event is the
point of discussion, and you can comfortably connect the “historio” word root to “date an event” and “historians” and
confidently move on with a good idea that historiography is connected to history in some way. By definition, historiography
is the writing of history. Answer choice (A): The stimulus discusses dates where there is conflict between sources. In no way
does the stimulus support answer choice (A). Answer choice (B): This is the correct answer. As stated in the last sentence,
historians are on occasion unsuccessful in determining independently the date of an event. If the usual sources offered are
in conflict about the date of a particular event and an analysis independent of the usual sources fails to confirm a date, then
a date cannot be reliably determined for the event. Answer choice (C): About one-third of all test takers choose this answer.
The stimulus speaks specifically of dating an event when the usual sources offer conflicting chronologies. The stimulus does
not discuss dating an event when there is no conflict of chronologies, and most likely many dates could be set with certainty
in the absence of any conflict. With this in mind, the language of the answer choice becomes problematic because “attaching
a reliable date to any event” would not “require determining which of several conflicting chronologies is most likely to be
true.” Answer choice (D): The language of the answer choice is too strong in saying that an independent determination is an
ineffective way of dating events. There is simply not enough information about what constitutes a “determination independent
of the usual sources” to say it is ineffective. Answer choice (E): This is another tricky answer, and just under a quarter of test
takers incorrectly select this answer. The answer claims that the soundest approach to dating an event is to undermine the
credibility of as many of the competing sources as possible. First, the stimulus suggests that the historian should, perhaps,
eliminate the less credible ones. No mention is made of eliminating as many as possible, and the stimulus indicates that
several remaining sources are to be expected. Second, that same section discusses eliminating less credible sources, not
undermining the credibility of those sources.
38. Answer (B). Answer choice (A): The stimulus never discusses who conducts the studies, only who authors the reports.
Thus, there is no proof for this answer choice and it fails the Fact Test. Even if you mistook “conducted” for “reported,” the
answer choice is still incorrect because the stimulus indicates that reports involving patients from several hospitals are usually
coauthored physicians from each hospital. Although “usually” could mean “always,” it does not have to, and hence it is
possible that a clinical trial could be reported by physicians from just one hospital. Answer choice (B): This answer choice is
a direct paraphrase of the second sentence. The second sentence states, “Reports of clinical trials involving patients from
several hospitals are usually coauthored by physicians from each participating hospital.” Answer choice (B) translates “usually”
into “most,” and “coauthored by physicians from each participating hospital” into “multiple authors.” Thus, the answer choice
passes the Fact Test and is correct. Answer choice (C): This is a Shell Game answer choice. Although the stimulus says there
has been a proliferation of multi-authored technical articles, no comment is made about the frequency of multi-authored
technical articles. In the next sentence, a frequency—“usually”—is given, but only for multi-authored clinical trial reports. The
test makers give you hard data about the clinical trial reports, and then try to entice you into picking a broader answer
involving technical reports. Answer choice (C) shows how the Shell Game can occur in the stimulus as well as in the answer
choices. The stimulus of this problem switches from “technical articles” to “reports of clinical trials.” Answer choice (C) plays
on that substitution. Answer choice (D): This is a Reverse answer that contains a complex pair of reversed elements when
matched against the stimulus. Let us compare the stimulus and the answer choice, using italics to indicate the reversed parts:
The stimulus states, “physics papers reporting results from experiments using subsystems developed at various laboratories
generally have authors from each laboratory.” Answer choice (D) states, “Physics papers authored by researchers from
multiple laboratories usually report results from experiments using subsystems developed at each laboratory.” The reversed
pair has two notable features: 1. The numbers are reversed—authors from each laboratory have become researchers
(authors) from multiple laboratories, and subsystems from various laboratories have become subsystems from each
laboratory. In a nutshell, the “various” and “each” elements have been reversed in the sentences. 2. The pair also reverses
logical position within the argument, as the stimulus states that the experiments generally have authors from each laboratory
and the answer choice states that the researchers usually report experiments from each laboratory. Answer choice (E): As
with answer choice (C), we do not know enough about technical articles to support this answer choice.

39. Answer (B). The “Some environmentalists question...” construction at the start of the stimulus does not lead to the
usual counter- conclusion because the stimulus does not contain an argument. This is a very interesting stimulus because
the author repeats the opinions of others and never makes an assertion of his or her own. When a stimulus contains only the
opinions of others, then in a Must Be True question you can eliminate any answer choice that makes a flat assertion without
reference to those opinions. For example, answer choice (A) makes a factual assertion (“It is...”) that cannot be backed up
by the author’s survey of opinions in the stimulus—the opinions do not let us know the actual facts of the situation. Answer
choice (E) can be eliminated for the very same reason. Answer choices (B), (C), and (D) each address the environmentalists,
and thus each is initially a Contender. Answer choice (B): This is the correct answer. The second sentence references the
views of many environmentalists, who claim that “nature has intrinsic value” (for example, beauty). This view is the non-
economic justification cited by the answer choice. This answer can be a bit tricky because of the convoluted language the
test makers use. “Questioning the defensibility of exploiting features of the environment” is a needlessly complex phrase. A
more direct manner of writing that phrase would be “attacking the exploitation of the environment.” To increase the difficulty
of this problem, this language was then repeated in answer choices (C) and (D). Answer choice (C): We only know the
opinions of “some” and “many” environmentalists, and these numbers do not provide enough information to discern the views
of “most” environmentalists, which is the term used in the answer choice (“many” is not the same as “most”). Answer choice
(D): This answer choice cannot be proven. While we know that many environmentalists claim a non-economic justification,
we do not know that that is the only justification they provide. When you are reading a stimulus, keep a careful watch on the
statements the author offers as fact, and those that the author offers as the opinion of others. In a Must Be True question,
the difference between the two can sometimes be used to eliminate answer choices.
40. Answer (A). Unlike many Must Be True question stimuli, this stimulus contains an argument. The conclusion is in the
second sentence: “the primary function of law is surely to help order society so that its institutions, organizations, and citizenry
can work together harmoniously, regardless of any further moral aims of the law.” The stimulus also begins with the “Some
argue that...” construction, and as usual, is followed by a conclusion that argues against the position established in the first
sentence (see “A Commonly Used Construction” in Chapter Two if this sounds unfamiliar). The last sentence is a premise that
proves to be key for choosing the correct answer. Answer choice (A): This correct answer is largely a paraphrase of the last
sentence. Answer choice (B): While the author certainly agrees with the first part of the sentence, in the second part the
phrase “devoid of moral aims” is too strong to be supported by the information in the stimulus. The last sentence indicates
that morality has some effect on the law and invalidates the “devoid” claim. Answer choice (C): This is an exaggerated
answer. Although the last sentence indicates that religious faith has been grounds for making exceptions in the application
of law, the stimulus does not indicate that actions based on religious or moral belief tend to receive the protection of the
highest courts. Answer choice (D): The author indicates that the “primary function” of law is to help order society; the author
does not indicate that this is the one and only function of law. The answer choice overstates the case by saying that a society
ordered by law should not reflect any moral convictions about the ordering. Answer choice (E): No mention is made of the
“best way” to promote cooperation, only that the primary function of law is to promote such cooperation.

41. The correct answer choice is (B). The stimulus tells the story of recent physics theories: Newtonian physics was
preeminent for over two centuries, and despite widespread acknowledgment and confirmation it was surpassed by Einsteinian
physics in the early 1900s. Answer choice (A): The two theories cited in the stimulus are not sufficient to form a pattern,
which is the basis of answer choice (A). Answer choice (B): This is the correct answer. As shown by the case of Newtonian
physics, success and substantiation is no guarantee of dominance. Answer choice (C): This is an exaggerated answer that
takes one instance and exaggerates it into a pattern. Although Newtonian physics was surpassed, this does not prove that
every theory of physics will be eventually surpassed. The answer goes farther than the facts of the stimulus and fails the Fact
Test. Answer choice (D): Like answer choice (C), this answer goes too far. Although some theories of physics have been
dominant for centuries, there is no guarantee that every theory will be dominant for that long. Answer choice (E): Even
though Einsteinian physics has enjoyed wide success in surpassing Newtonian physics, nowhere in the stimulus is there
evidence to prove that each theory must be surpassed by an equally successful theory.

42. The correct answer choice is (E). This interesting stimulus contains two definitions of “refusal:” an initial definition that
implies refusal is a point at which pilings will go no further, and then a second, contemporary standards definition of refusal
that reveals that refusal is a point at which additional penetration into the ground is no greater than two inches after twenty-
four hammer blows. The stimulus is a fact set, and thus there is no conclusion present. Answer choice (A): Although there
was an inquiry into the solidity of the piers of the Rialto Bridge, the results of that inquiry are not disclosed. The only other
information we are given is that the pilings of the Rialto Bridge met the contemporary standard of refusal, but this is not
sufficient to indicate whether the pilings of this particular bridge were safe. Hence, this answer fails the Fact Test and is
incorrect. Answer choice (B): Similar to answer choice (A), we have insufficient information to make this judgment. Answer
choice (C): This answer is somewhat opposite of the information in the stimulus, which states that Da Ponte had met the
contemporary standard of refusal. Answer choice (D): This is another Opposite answer. The stimulus indicates that bridges
built prior to 1700 were driven to the point of refusal. Answer choice (E): This is the correct answer. As stated in the stimulus,
“he had caused the pilings to be driven until additional penetration into the ground was no greater than two inches after
twenty-four hammer blows.” The statement indicates that additional penetration was possible with a sufficient number of
hammer blows.

43. The correct answer choice is (C). This is a fact set. Note the strength of the modifiers in this stimulus—“every,” “most,”
and “any.” We should be able to use this narrow scope to support a fairly strong statement, but be careful: the test makers
know this too and they will supply several answer choices that are worded strongly. Make sure you select an answer that
conforms to the facts. Answer choice (A): The phrase “better than a merely good life” goes beyond the statements in the
stimulus. Answer choice (B): This answer is incorrect because we are not given information about how the moral theories
are different, or if they different at all. The only detail we are told is that the theories all have one thing in common—they
tell us what a good life is. Since the answer choice makes a claim based on differences between theories, it cannot be correct.
Answer choice (C): This is the correct answer. At first glance, this answer choice may seem a bit strong in saying the
conception would not match that of any moral theory. But, as discussed above, we can support this because the stimulus
uses very strong language, specifically stating “most people would judge someone who perfectly embodied the ideals of any
one of these theories not to be living a good life.”. Answer choice (D): This answer is worded strongly but it quickly fails the
Fact Test. Nothing is said to indicate that the life described by one of the moral theories cannot be realized. Answer choice
(E): This answer also has strong language, but it goes too far in saying that it is impossible to develop a theory that accurately
describes a good life.
44. The correct answer choice is (C) The final three problems in this section are harder than the previous five. This problem
is answered correctly by about 45% of test takers and is classified as difficult (the hardest GMAT questions have success
rates under 20%. Fortunately, questions this difficult appear infrequently). Students can miss questions for a variety of
reasons: 1. The stimulus is difficult to understand. 2. The question stem is difficult to classify (very rare) or confusing. 3. The
correct answer is deceptive, causing students to avoid it. 4. One (or more) of the incorrect answers is attractive, drawing
students to it. Given that the stimulus is a simple fact set and that none of the incorrect answers attracted more than 15%
of test takers, the difficulty in this problem apparently lies in the correct answer. Answer choice (A): The controversy in the
stimulus is about the categorization of Cézanne as an artist, not about Cézanne’s work. Further, even if the answer did
correctly reference the categorization controversy, the answer would still be suspect because of the word “highly.” The
stimulus indicates that only a small few reject the categorization of Cézanne as an early modernist and most experts accept
it. Answer choice (B): The stimulus asserts that Cézanne inspired the creators of abstract art, not that Cézanne himself
created abstract art. Answer choice (C): This correct answer is a paraphrase of the first sentence. The deceptiveness of this
answer lies in two areas: 1. The substitution of “develop” for “inspire.” Some students feel the word “develop” is too strong,
but if Cézanne inspired the creators of the next generation of art then he helped develop it. 2. The use of the word
“modernism.” Some students are thrown off by “modernism” because they expect to see “abstract” instead. The stimulus is
careful about saying “twentieth-century modernist creators of abstract art.” Notice how the test makers use answer choice
(B)—which mentions “abstract”—to subtly prepare you to make this error. Answer choice (D): The first sentence indicates
that Cézanne inspired the modernist creators. The rest of the stimulus discusses a disagreement about the categorization of
Cézanne that is not resolved in favor of either group. Hence, there is no way for us to determine if modern art owes less to
Cézanne than many experts believe. Answer choice (E): The word “tends” is the problem in this answer choice. Logically,
“tends” means “most.” So, according to answer choice (E), Cézanne’s work is usually misinterpreted as modernist. The
stimulus disagrees with this view: only a “small few” reject the categorization of Cézanne as a modernist whereas the majority
accepts it. Further, the disagreement in the stimulus involves art experts, and from their view we would dispute answer
choice (E). Answer choice (E) can also be understood as involving all interpretation of Cézanne’s work—whether by art expert
or not—and from this perspective the answer is still unsupported since the views of others are not discussed in the stimulus.

45. The correct answer choice is (B). The stimulus is a fact set. Part of the difficulty with this problem is the scientific subject
matter. Many people are intimidated by the mention of rhodopsin, with which they are unfamiliar. As with the flavonoids in
problem #1 of the chapter text, you do not need to know what rhodopsin is to complete the problem. The stimulus can be
broken into several easily digestible parts: Premise: Light is registered in the retina when photons hit rhodopsin molecules
and the molecules change shape. Premise: Due to normal molecular motion, rhodopsin molecules sometimes change shape
without having been hit by light. This change causes errors in the visual system. Premise: The amount of molecular motion
is directly proportional to the temperature of the retina. Answer choice (A): The stimulus does not indicate that the
temperature of the retina depends on the amount of light. It could easily be affected by other factors, such as body
temperature. Answer choice (B): This is the correct answer and just over 40% of test takers correctly choose this answer. To
prove this answer, you must link together several pieces of information. First, the last sentence of the stimulus shows that
the amount of rhodopsin molecular motion is directly proportional to the temperature of the retina, and the second sentence
of the stimulus shows that this motion causes visual errors, so the higher the retinal temperature, the more errors in the
visual system. The answer choice ties body temperature (remember, the retina is a body part) to the temperature of the
surroundings and then rightly notes that hot surroundings would cause more visual errors than cold surroundings if body
temperature matched those surroundings. Answer choice (C): This was the most popular incorrect answer, and just under a
quarter of test takers fell for this answer. The answer is wrong because we do not know that temperature causes the
rhodopsin to react more slowly. Higher retinal temperature causes the rhodopsin molecules to change shape, but no mention
is made of reaction time. This answer falls under the “New information” category. Answer choice (D): Another New
Information answer choice. Similar to answer choice (C), this answer fails the Fact Test because no information is given about
the surface area of the retina. Answer choices (C) and (D) are great examples of how an answer can contain information
unmentioned by the stimulus. These answers are somewhat attractive because there is nothing actively wrong about them
and thus they could be true. To avoid them, always keep in mind that your goal is to find the answer that must occur based
on the information in the stimulus. Answer choice (E): While the stimulus focuses on rhodopsin, no indication is given that
rhodopsin is the only naturally occurring pigment molecule—there could be others.
46. Answer (D). We can make the determination that since there is no conclusion in the stimulus, this is a fact set and not
an argument. In this case the stimulus is short, and can be broken down into three components: First Statement: Flavonoids
are a common component of almost all plants. Second Statement: A specific variety of flavonoid in apples has been found to
be an antioxidant. Third Statement: Antioxidants are known to be a factor in the prevention of heart disease. The scope of
the stimulus—especially if that scope is broad—often helps eliminate one or more of the answer choices. Answer choice (A):
This is an interesting answer choice, and most people take a moment before categorizing this as a Loser. The answer choice
could be true, but it is too broad to be supported by the facts: nowhere are we told that a diet of fruits and vegetables will
help prevent heart disease (and in this sense the answer fails the Fact Test). Perhaps apples are the only fruit with the
antioxidant flavonoid and there is nothing beneficial about other fruits and vegetables. And, eating a diet of fruits and
vegetables is no guarantee that the diet includes apples. Regardless, this answer choice can be especially attractive because
it plays on the general perception that fruits and vegetables are good for you. Answer choice (B): This answer is also a Loser.
Nothing in the stimulus supports the rather strong statement that flavonoids are essential to preventing heart disease. Answer
choice (C): Many people hold this answer as a Contender and then move on to answer choice (D). As it will turn out, this
answer is incorrect because the language is too strong: the stimulus only stated that apples contain an element that was a
factor in preventing heart disease, not that they definitely will prevent heart disease. Answer choice (D): This answer is the
closest to our pre-phrase, and this is the correct answer. Notice how the language of this answer choice—“helps to prevent”—
matches the stimulus language—“factor in the prevention.” Answer choice (E): This answer choice also could be true, but it
cannot be correct because the stimulus makes no mention of the causes of heart disease. Just because an antioxidant can
help prevent heart disease does not mean that a lack of antioxidants causes heart disease. Notice how the scope of the
stimulus plays a role in how we attack the answer choices. The language of the stimulus is relatively broad—“almost all,”
“factor in the prevention,”—and the author shies away from making definite statements. Because the stimulus does not
contain much in the way of direct, absolute information, selecting an answer choice that contains a direct, absolute statement
is difficult to justify. This reasoning helps us eliminate answer choices (B) and (C), both of which contain strong statements
that are ultimately unsupportable (literally, they both fail the Fact Test because they are too strong).

47. As with most Cannot Be True questions, the stimulus does not contain an argument. Instead, a fact pattern is presented
and you are tested on your knowledge of those facts. Let’s review each statement, sentence-by-sentence, keeping in mind
that in Cannot questions you accept the statements in the stimulus as true: Statement: Sharks have a higher ratio of
cartilage mass to body mass than any other organism. This is a very broad, global statement indicating that no other organism
has a higher ratio of cartilage mass to body mass than sharks. Statement: They [sharks] also have a greater resistance to
cancer than any other organism. his is another very broad, global statement indicating that no other organism has a greater
resistance to cancer than sharks. Statement: Shark cartilage contains a substance that inhibits tumor growth by stopping
the development of a new blood network. This statement is narrower, and focuses only on shark cartilage. Since no
information is given about the cartilage of other organisms, it is possible that other organisms contain the tumor-inhibiting
substance mentioned in this statement. Statement: In the past 20 years, none of the responses among terminal cancer
patients to various therapeutic measures has been more positive than the response among those who consumed shark
cartilage. This statement is also narrower than the first two, but broader than the last statement. While it is specific in stating
that no therapeutic measures have received more positive response than shark cartilage, the statement is limited to the past
20 years and to terminal cancer patients. Note also that just because shark cartilage has received a more positive response
than any other therapy does not mean that other therapies were unsuccessful—they could have worked very well but not
quite as well as shark cartilage. Of the four statements above, the first two are global and can never be violated. The last
two are possible sources of wrong answers as they are specific enough to eliminate certain statements, but open enough to
allow for a variety of others. The makers of the test love to play with these “edges,” and you should make sure that the
answer you select directly violates a statement in the stimulus. Answer choice (A): This is the correct answer. The answer
violates the second sentence of the stimulus, where the author indicates that sharks have a “greater resistance to cancer
than any other organism.” This statement means that no other organisms matches or exceeds the cancer resistance of a
shark. Answer choice (B): This answer focuses on the susceptibility of an organism to cancer. Other than stating that sharks
are the least susceptible to cancer (greatest resistance = least susceptibility), we know nothing about cancer susceptibility of
any other organism. Hence, this answer is possibly true and therefore incorrect. Answer choice (C): The third statement in
the stimulus notes that shark cartilage contains the inhibiting substance. There is no mention that other organisms do not
have the substance or that the substance is the primary reason that sharks are cancer resistant. Thus, this answer choice
could be true and is incorrect. Answer choice (D): From the discussion of the fourth statement you know this answer choice
could be true. Accordingly, it is incorrect. Answer choice (E): The stimulus addresses sharks and cancer resistance. Although
cancer resistance would logically have some connection to the immune system, cancer resistance is only one aspect of the
immune system and therefore other organisms could have an immune system that is overall more efficient than the shark’s
immune system. Consequently, this answer choice is possibly true and therefore incorrect.
48. The conclusion to this argument is the fourth sentence, which begins with the conclusion indicator “For this reason...”
By applying the Primary Objectives you should have identified this conclusion while reading, and then, upon classifying the
question stem you should have looked for a paraphrase of this sentence. Answer choice (C) fits the bill, and is the correct
answer. Answer choice (A): The author would agree with this statement but this is not the Main Point of the argument;
rather, it is closer to a premise that might support the conclusion. Incidentally, the author’s general agreement with this
answer choice is signaled by the use of “however” in the third sentence. In the stimulus, the author begins by mentioning
that a free marketplace of ideas, including dangerous ideas, ensures a fair hearing of ideas. In the third sentence, the author
then says, “however, the government is responsible for over half the information released to the public,” indicating the author
feels the government is a threat to this free marketplace. Answer choice (B): The author would also agree with this statement,
but again this is not the Main Point of the argument. As discussed in the analysis of answer choice (A), the author believes
that the freedom of the marketplace of ideas is at risk, and in stating that we should curtail the government’s power over
information, the author assumes that preserving a free marketplace of ideas is important. Thus this answer choice would be
better described as an unstated premise that supports the conclusion. Answer choice (C): This is the correct answer.
Remember, any answer that is a paraphrase of the conclusion of the argument will be the correct answer to a Main Point
question. Answer choice (D): The stimulus specifically notes that malicious or prejudicial ideas can “prompt beneficial
outcomes.” The outcome of an idea is different than stating the ideas themselves “can be valuable.” Answer choice (E): The
stimulus states that “governments continue to construct near monopolies on the publication and dissemination of enormous
amounts of information.” This phrasing is not the same as answer choice (E), which asserts that the government already has
a monopoly on the dissemination of many kinds of information. The lesson learned from this particular problem is that
you must isolate the conclusion and then look for a paraphrase of that conclusion.

49. The conclusion of this argument is difficult to identify because the author does not use a traditional conclusion indicator.
The first three sentences are admissions by the author regarding the nature of Hogan’s actions. The fourth sentence contains
the conclusion and a premise, and the conclusion is that “Hogan’s actions should not be wholly condemned.” If you struggled
to identify the conclusion, consider how you might have applied the Conclusion Identification Methodology to the pieces of
the argument. For example, consider the two parts of the last sentence. If you thought one of them might be the conclusion,
place one as the conclusion and the other as a premise, as follows: “Because I ask that Hogan’s actions not be wholly
condemned, therefore I emphasize again that Hogan mistakenly believed Winters to be the robber who had been terrorizing
west-side apartment buildings for the past several months.” Does that configuration sound right? No. Try again by reversing
the premise and conclusion pieces: “Because Hogan mistakenly believed Winters to be the robber who had been terrorizing
west-side apartment buildings for the past several months, therefore I ask you to concur with me that Hogan’s actions not
be wholly condemned.” The relationship now sounds much more logical. Answer choice (A): The author admits that Hogan
fully realized his actions and the author asks that “Hogan’s actions not be wholly condemned.” Both of these statements are
counter to the idea that Hogan should not be considered responsible for Winter’s injuries. Answer choice (B): The only
reference to the robber is that Hogan mistakenly believed that Winters was the robber. Thus, there is no evidence in the
stimulus to support this answer. Answer choice (C): This correct answer is a paraphrase of the conclusion of the argument.
Answer choice (D): According to the information in the stimulus, this answer must be true. Regardless, the answer is still
incorrect because it fails to summarize the author’s main point. This type of answer—one that is true but misses the main
point—is frequently featured as an incorrect answer in Main Point questions. Answer choice (E): Like answer choice (D), this
statement is true according to the stimulus. But, it is incorrect because it does not capture the main point.

50. The correct answer choice is (C). Like the majority of Main Point question stimuli, the argument does not contain a
traditional conclusion indicator. Thus, you must look at the pieces of the argument in order to determine the point the author
is making. In this case, the conclusion is “The similarities are too fundamental to be mere coincidence.” Use the Conclusion
Identification Methodology to help establish that point if you are unsure. The argument uses the fact that the two workstations
are similar and were released in the same time-span to assume that the similarity is not caused by coincidence. Answer
choice (A): This is a repeat of a premise of the argument, not the main point. As mentioned in the discussion, in Main Point
questions you should expect to see incorrect answers that repeat premises from the argument. Answer choice (B): The
statement does not pass the Fact Test. The scenario could be reversed: OCF could have copied Ergotech. Regardless, this is
not the main point. Answer choice (C): This correct answer is a paraphrase of the conclusion. Answer choice (D): This would
undermine the argument and thus it cannot be the main point. Answer choice (E): Although the author would likely agree
with this statement, this does not capture the main point, which addresses the two named products.
Inference Questions Part 2
Directions: The questions in this section are based on the reasoning contained in brief statements or passages. For some questions, more
than one of the choices could conceivably answer the question. However, you are to choose the best answer; that is, the response that most
accurately and completely answers the question. You should not make assumptions that are by commonsense standards implausible,
superfluous, or incompatible with the passage.

1. A century in certain ways is like a life, and as the end of 3. Educator: If there is a crisis in education today, it is one
a century approaches, people behave toward that century of maintaining quality. People love to reduce
much as someone who is nearing the end of life does serious learning to degrees and certificates. But
toward that life. So just as people in their last years one also can obtain these credentials by plodding
spend much time looking back on the events of their life, through courses without ever learning much of
people at a century’s end . value. When that happens, the credentials one
receives are almost meaningless.
Which one of the following most logically completes
the argument? If the educator’s statements are true, then which one of
the following must be true?
(A) reminisce about their own lives
(B) fear that their own lives are about to end (A) Increasingly, institutions are granting meaningless
(C) focus on what the next century will bring degrees and certificates.
(D) become very interested in the history of the (B) It has become easier for students to complete
century just ending their coursework without learning anything of
(E) reflect on how certain unfortunate events of the importance.
century could have been avoided (C) Educational institutions should cease to grant
degrees and certificates.
2. Members of large-animal species must consume (D) Degrees and certificates do not guarantee
enormous amounts of food to survive. When climatic that a person has acquired much worthwhile
conditions in their environment deteriorate, such animals knowledge.
are often unable to find enough food. This fact helps (E) A person benefits from an education only to the
make large-animal species more vulnerable to extinction extent that he or she invests effort in it.
than small-animal species, which can maintain greater
populations on smaller amounts of food. 4. Most veterinarians, and especially those at university
veterinary research centers, have a devoted interest in
The statements above, if true, most support which one of the biological sciences. But most veterinarians choose
the following? their profession primarily because they love animals.
(A) The maximum population size that an animal Among persons who are seriously interested in
species could maintain on any given amount of biological science but lack any special love for animals,
food is the main factor determining whether that one does not find any prominent veterinarians.
species will become extinct. If all of the statements above are true, which one of the
(B) The vulnerability of an animal species to following CANNOT be true?
extinction depends at least in part on how much
food individuals of that species must consume to (A) Some veterinarians have a greater love for
survive. biological science than for individual animals.
(C) When conditions deteriorate in a given (B) Most veterinarians love animals and have an
environment, no small-animal species will interest in biological science.
become extinct unless some large-animal species (C) Prominent veterinarians at some veterinary
also becomes extinct. research centers are intensely devoted to
(D) Within any given species, the prospects for the biological sciences but do not feel any
survival of any particular individual depend pronounced affection for animals.
primarily on the amount of food that individual (D) Few veterinarians at university research centers
requires. chose their profession primarily because they
(E) Whenever climatic conditions in a given love animals.
environment are bad enough to threaten large- (E) Most veterinarians who are not prominent regard
animal species with extinction, small-animal an understanding of the biological sciences as
species are able to find enough food to survive. the most important quality for success in their
profession.
7. Although fiber-optic telephone cable is more expensive
to manufacture than copper telephone cable, a telephone
5. All societies recognize certain rules to be so crucial that
they define those rules as duties, such as rules restricting network using fiber-optic cable is less expensive overall
violence and those requiring the keeping of agreements. than a telephone network using copper cable. This is
Contained in the notion of a duty is the idea that its because copper cable requires frequent amplification
fulfillment is so fundamental to a properly functioning of complex electrical signals to carry them for long
society that persons obligated by it cannot be excused on distances, whereas the pulses of light that are
the ground that its fulfillment would be harmful to their transmitted along fiber-optic cable can travel much
self-interest. This shows that . farther before amplification is needed.
Which one of the following most reasonably completes The above statements, if true, most strongly support
the argument? which one of the following?
(A) all societies overrate the benefits of certain (A) The material from which fiber-optic cable is
rules, such as those governing the keeping of manufactured is more expensive than the copper
agreements from which copper cable is made.
(B) all societies have certain rules that no people are (B) The increase in the number of transmissions of
capable of following complex signals through telephone cables is
(C) all societies recognize the possibility of clashes straining those telephone networks that still use
between individual self-interest and the copper cable.
performance of duty (C) Fiber-optic cable can carry many more signals
(D) a properly functioning society will recognize that simultaneously than copper cable can.
some duties take priority over others (D) Signals transmitted through fiber-optic cable
(E) societies have no right to expect people always to travel at the same speed as signals transmitted
perform their duties through copper cable.
(E) The cost associated with frequent amplification of
6. Essayist: Many people are hypocritical in that they signals traveling through copper cable exceeds
often pretend to be more morally upright than the extra manufacturing cost of fiber-optic cable.
they really are. When hypocrisy is exposed,
hypocrites are embarrassed by their moral lapse, 8. According to the rules of the university’s housing
which motivates them and others to try to become lottery, the only students guaranteed dormitory rooms
better people. On the other hand, when hypocrisy are fourth-year students. In addition, any fourth-year
persists without exposure, the belief that most student on the dean’s list can choose a dormitory room
people are good is fostered, which motivates most before anyone who is not a fourth-year student.
people to try to be good.
Which one of the following inferences is most strongly
The essayist’s statements, if true, most strongly support supported by the rules described above?
which one of the following?
(A) Benizer is a fourth-year student who is not on the
(A) The existence of hypocrisy encourages people to dean’s list, so she is not guaranteed a dormitory
believe that no one is morally blameless. room.
(B) The existence of hypocrisy encourages people to (B) Ivan and Naomi are both fourth-year students but
make efforts to live by moral standards. only Naomi is on the dean’s list. Therefore, Ivan
(C) The existence of hypocrisy in some people can choose a dormitory room before Naomi.
encourages others to fall into moral lapses. (C) Halle, a third-year student, is on the dean’s list.
(D) The hiding of hypocrisy is a better way of Thus, she is guaranteed a dormitory room.
motivating people to try to be good than is the (D) Gerald and Katrina are both on the dean’s list
exposing of it. but only Gerald is a fourth-year student. Thus,
(E) There is no stronger motivator for people to try to Gerald can choose a dormitory room before
be good than the exposing of hypocrisy. Katrina.
(E) Anissa is a fourth-year student who is on the
dean’s list. Thus, since Jehan is a second-year
student who is also on the dean’s list, he can
choose a dormitory room before Anissa.
11. Criminals often have an unusual self-image. Embezzlers
9. The solidity of bridge piers built on pilings depends
often think of their actions as “only borrowing money.”
largely on how deep the pilings are driven. Prior to
Many people convicted of violent crimes rationalize
1700, pilings were driven to “refusal,” that is, to the
their actions by some sort of denial; either the victim
point at which they refused to go any deeper. In a 1588
“deserved it” and so the action was justified, or “it
inquiry into the solidity of piers for Venice’s Rialto
simply wasn’t my fault.” Thus, in many cases, by
Bridge, it was determined that the bridge’s builder,
criminals’ characterization of their situations, .
Antonio Da Ponte, had met the contemporary standard
for refusal: he had caused the pilings to be driven until Which one of the following most logically completes the
additional penetration into the ground was no greater passage?
than two inches after twenty-four hammer blows. (A) they ought to be rewarded for their actions
Which one of the following can properly be inferred (B) they are perceived to be the victim of some other
from the passage? criminal
(C) their actions are not truly criminal
(A) The Rialto Bridge was built on unsafe pilings.
(D) the criminal justice system is inherently unfair
(B) The standard of refusal was not sufficient to
(E) they deserve only a light sentence for their crimes
ensure the safety of a bridge.
(C) Da Ponte’s standard of refusal was less strict than
12. In a study, infant monkeys given a choice between two
that of other bridge builders of his day. surrogate mothers—a bare wire structure equipped with
(D) After 1588, no bridges were built on pilings that a milk bottle, or a soft, suede-covered wire structure
were driven to the point of refusal. equipped with a milk bottle—unhesitatingly chose the
(E) It is possible that the pilings of the Rialto Bridge latter. When given a choice between a bare wire
could have been driven deeper even after the structure equipped with a milk bottle and a soft, suede-
standard of refusal had been met. covered wire structure lacking a milk bottle, they
unhesitatingly chose the former.
10. Psychologist: Although studies of young children have
revealed important facts about the influence of the Which one of the following is most supported by the
environment on language acquisition, it is clear that one information above?
cannot attribute such acquisition solely to environmental (A) Infant monkeys’ desire for warmth and comfort is
influences: innate mechanisms also play a role. So, the nearly as strong as their desire for food.
most reasonable question that ought to be studied is (B) For infant monkeys, suede is a less convincing
whether . substitute for their mother’s touch than animal
fur would be.
Which one of the following most logically completes the
passage? (C) For infant monkeys, a milk bottle is a less
convincing substitute for their mother’s teat than
(A) language acquisition can ever be fully explained suede is for their mother’s touch.
(B) innate mechanisms are a contributing factor in (D) For infant monkeys, a milk bottle is an equally
language learning convincing substitute for their mother’s teat as
(C) language acquisition is solely the product of suede is for their mother’s touch.
innate mechanisms (E) Infant monkeys’ desire for food is stronger than
(D) parents and peers are the most important their desire for warmth and comfort.
influence on a child’s learning of a language
(E) innate mechanisms play a more important role in
language acquisition than a child’s immediate
environment
15. Lathyrism, a debilitating neurological disorder caused
13. Most antidepressant drugs cause weight gain. While
by the consumption of the legume Lathyrus sativus, is
dieting can help reduce the amount of weight gained
widespread among the domestic animals of some
while taking such antidepressants, some weight gain is
countries. Attempts to use rats to study lathyrism have
unlikely to be preventable.
generally failed. Rats that ingested Lathyrus sativus did
The information above most strongly supports which not produce the symptoms associated with the disorder.
one of the following?
Which one of the following is most strongly supported
(A) A physician should not prescribe any by the information above?
antidepressant drug for a patient if that patient is
(A) The physiology of rats is radically different from
overweight.
that of domestic animals.
(B) People who are trying to lose weight should not
(B) The rats did not consume as much Lathyrus
ask their doctors for an antidepressant drug.
sativus as did the domestic animals that
(C) At least some patients taking antidepressant drugs contracted lathyrism.
gain weight as a result of taking them.
(C) Not all animal species are equally susceptible to
(D) The weight gain experienced by patients taking
lathyrism.
antidepressant drugs should be attributed to lack
(D) Most of the animals that can contract lathyrism
of dieting.
are domestic.
(E) All patients taking antidepressant drugs should
(E) Laboratory conditions are not conducive to the
diet to maintain their weight.
development of lathyrism.
14. Gene splicing can give rise to new varieties of farm
animals that have only a partially understood genetic
makeup. In addition to introducing the genes for
whichever trait is desired, the technique can introduce
genes governing the production of toxins or carcinogens,
and these latter undesirable traits might not be easily
discoverable.
The statements above, if true, most strongly support
which one of the following?
(A) All toxin production is genetically controlled.
(B) Gene splicing to produce new varieties of farm
animals should be used cautiously.
(C) Gene splicing is not effective as a way of
producing new varieties of farm animals.
(D) Most new varieties of farm animals produced by
gene splicing will develop cancer.
(E) Gene splicing will advance to the point where
unforeseen consequences are no longer a
problem.
17. Physical education should teach people to pursue
16. These days, drug companies and health professionals
healthy, active lifestyles as they grow older. But the
alike are focusing their attention on cholesterol in the
focus on competitive sports in most schools causes
blood. The more cholesterol we have in our blood, the
most of the less competitive students to turn away
higher the risk that we shall die of a heart attack. The
from sports. Having learned to think of themselves as
issue is pertinent since heart disease kills more North
unathletic, they do not exercise enough to stay healthy.
Americans every year than any other single cause. At
least three factors—smoking, drinking, and exercise— Which one of the following is most strongly supported
can each influence levels of cholesterol in the blood. by the statements above, if they are true?
Which one of the following can be properly concluded (A) Physical education should include noncompetitive
from the passage? activities.
(B) Competition causes most students to turn away
(A) If a person has low blood cholesterol, then that
from sports.
person’s risk of fatal heart disease is low.
(C) People who are talented at competitive physical
(B) Smoking in moderation can entail as great a risk
endeavors exercise regularly.
of fatal heart disease as does heavy smoking.
(D) The mental aspects of exercise are as important as
(C) A high-cholesterol diet is the principal cause of
the physical ones.
death in North America.
(E) Children should be taught the dangers of a
(D) The only way that smoking increases one’s risk
sedentary lifestyle.
of fatal heart disease is by influencing the levels
of cholesterol in the blood.
18. Raising the humidity of a room protects furniture,
(E) The risk of fatal heart disease can be altered by
draperies, and computers from damage caused by
certain changes in lifestyle.
excessively dry air. Further, it can make people feel
warmer, helps the body’s defenses against viruses, and
alleviates some skin rashes.
Each of the following is supported by the information
above EXCEPT:
(A) Humidity can be bad for computers.
(B) A room can be too dry for the optimal
maintenance of its furnishings.
(C) Dry air can feel cooler than humid air of the same
temperature.
(D) Increased humidity can be beneficial to the skin.
(E) The human immune system can benefit from
humidity.
21. The largest volcano on Mars rises 27 kilometers above
19. Shortly after Isaac Newton circulated some of his
the surrounding plain and covers an area roughly the
theories of light in 1672, his colleague Robert Hooke
size of Romania. Even if the Earth’s gravity were as
claimed that most of those theories were based on
low as the gravity of Mars is, no volcano of such size
Hooke’s own work. A modern reader might interpret
could exist on Earth, for the Earth’s crust, although of
Newton’s famous comment, “if I have seen further it is
essentially the same composition as that of Mars, is too
by standing on the shoulders of giants,” as a conciliatory
thin to support even a small fraction of that mass and
gesture acknowledging indebtedness to Hooke and
would buckle under it, causing the mountain to sink.
other contemporary scientists for some of his theories.
Conciliatory gestures acknowledging indebtedness If the statements above are true, which one of the
were uncharacteristic of Newton, however, and in his following must also be true on the basis of them?
day such allusions to “giants” typically referred to the (A) The surface of Mars is less subject to forces of
ancient Greeks, not to contemporary scientists. erosion than is the surface of the Earth.
The statements in the passage, if true, most strongly (B) The highest volcanoes on Mars occur where its
support which one of the following? crust is thickest.
(C) On average, volcanoes on Mars are higher than
(A) Newton did not intend the quoted comment to
those on Earth.
be an acknowledgment that his theories of light
(D) The crust of Mars, at least at certain points on the
were largely derived from Hooke’s.
planet, is thicker than the crust of the Earth.
(B) Newton did not take credit for any advances that
(E) At least some of the Earth’s volcanoes would be
Hooke made in the theory of light.
larger than they actually are if the Earth’s crust
(C) Newton did not believe that any of Hooke’s
were thicker than it is.
theories of light were based on those of the
ancient Greeks.
22. Twenty professional income-tax advisors were given
(D) Newton intended to credit some contemporary
identical records from which to prepare an income-tax
scientists other than Hooke for some of the
return. The advisors were not aware that they were
advances that Newton made in the theory of
dealing with fictitious records compiled by a financial
light. magazine. No two of the completed tax returns agreed
(E) Newton was not familiar with Hooke’s work on with each other, and only one was technically correct.
the theory of light.
If the information above is correct, which one of the
20. Lack of exercise produces the same or similar bodily following conclusions can be properly drawn on the
effects as aging. In fact, the physical changes that basis of it?
accompany aging can often be slowed down by (A) Only one out of every twenty income-tax returns
appropriate exercise. No drug, however, holds any prepared by any given professional income-tax
promise for slowing down the changes associated with advisor will be correct.
aging. Therefore, . (B) The fact that a tax return has been prepared by a
Which one of the following provides a logical professional income-tax advisor provides no
completion to the passage above? guarantee that the tax return has been correctly
prepared.
(A) taking drugs has the same effect on aging as does
(C) In order to ensure that tax returns are correct, it
a lack of exercise
is necessary to hire professional income-tax
(B) people who do not exercise are likely to need advisors to prepare them.
drugs to sustain their health
(D) All professional income-tax advisors make
(C) appropriate exercise can prevent the physical
mistakes on at least some of the tax returns they
changes associated with aging prepare.
(D) people who do not exercise when they are young
(E) People are more likely to have an incorrectly
will gain few benefits from beginning to exercise
prepared tax return if they prepare their own tax
at a later age
returns than if they hire a professional income-
(E) if the physical changes of aging are to be slowed, tax advisor.
it is more practical to rely on exercise than on
drugs
25. Not surprisingly, there are no professors under the age of
23. Many people limit the intake of calories and cholesterol eighteen. And, as well known, no one under eighteen can
in their diet in order to lose weight and reduce the vote legally. Finally, some brilliant people are professors,
level of cholesterol in their blood. When a person loses some are legal voters, and some are under eighteen.
weight, the fat cells in that person’s body decrease in
size but not in number. As they decrease in size, fat cells If the statements above are true, then on the basis of
spill the cholesterol they contain into the bloodstream. them which one of the following must also be true?
Therefore, a person who goes on a low-calorie, low (A) No professors are eighteen-year-olds.
cholesterol diet . (B) All brilliant people are either professors, legal
Which one of the following most logically completes the voters, or under eighteen.
argument? (C) Some legal voters are not professors.
(D) Some professors are neither legal voters nor
(A) might at first have an increased level of brilliant people.
cholesterol in his or her blood (E) Some brilliant people are neither professors nor
(B) will not lose weight any faster than will a person legal voters.
whose diet is high in calories
(C) might lose more weight by going on a low
calorie, high-cholesterol diet than by going on
the low-calorie, low-cholesterol diet
(D) will not decrease the size of his or her fat cells
(E) will both decrease the level of cholesterol in his
or her blood and gain weight

24. Jones is selling a house to Smith. The contract between

the two specifies that for up to a year after ownership is


transferred, Jones will be responsible for repairing any
“major structural defects,” defined as defects in the roof
or roof-supporting components of the house, that might
be found. Jones is not responsible for any other repairs.
The house has a truss roof, which means that the only
walls that support the roof are the exterior walls.
It can be properly concluded from the information above
that
(A) Jones did not know of any defects in the roof or
roof-supporting components of the house at the
time the contract was written
(B) although other components of the house may
contain defects, the roof and roof-supporting
components of the house are currently free from
such defects
(C) the contract does not oblige Jones to repair any
defects in the house’s non-exterior walls after
ownership of the house has been transferred
(D) Smith will be obligated to repair all structural
defects in the house within a year after
ownership is transferred, except those for which
Jones is responsible
(E) in the past Jones has had to make repairs to some
of the house’s exterior walls
28. Essayist: Every contract negotiator has been lied to by
26. The chances that tropical storms will develop in a given someone or other, and whoever lies to anyone is
area increase whenever the temperature of a large body of practicing deception. But, of course, anyone who
water in that area exceeds 26 degrees Celsius to a depth has been lied to has also lied to someone or other.
of about 60 meters. If the amount of carbon dioxide in the
Earth’s atmosphere continues to increase, the If the essayist’ s statements are true, which one of the
temperatures of all of the Earth’s waters will rise, with the following must also be true?
result that the number of large bodies of water whose (A) Every contract negotiator has practiced deception.
temperatures exceed 26 degrees Celsius to a depth of (B) Not everyone who practices deception is lying to
about 60 meters will eventually be greater than it is today. someone.
The statements above, if true, most strongly support (C) Not everyone who lies to someone is practicing
which one of the following conclusions? deception.
(D) Whoever lies to a contract negotiator has been
(A) There are likely to be more tropical storms if lied to by a contract negotiator.
the amount of carbon dioxide in the Earth’s (E) Whoever lies to anyone is lied to by someone.
atmosphere continues to increase.
(B) Tropical storms can occur only when the air 29. Astronomers have long thought that the irregularity in
temperature exceeds 26 degrees Celsius. the orbit of the planet Neptune was adequately explained
(C) The number of large bodies of water whose by the gravitational pull exerted on Neptune by the
temperatures exceed 26 degrees Celsius to a planet Pluto. The most recent observations of Pluto,
depth of about 60 meters is greater today than it however, indicate that this planet is much too small to
ever was. exert the amount of gravitational pull on Neptune that
(D) The ferocity of tropical storms does not depend astronomers once thought it did.
on the amount of carbon dioxide in the Earth’s
atmosphere. If the statements above are true, they provide the most
(E) Any increase in the temperatures of the Earth’s support for which one of the following?
oceans would cause the amount of carbon (A) Neptune is somewhat larger than scientists once
dioxide in the atmosphere to increase as well. believed it to be.
(B) The orbit on Neptune is considerably more
27. Most adults in country X consume an increasing amount irregular than scientists once thought it was.
of fat as they grow older. However, for nearly all adults (C) There exists another, as yet undiscovered planet
in country X, the percentage of fat in a person’ s diet
stays the same throughout adult life. with an orbit beyond that of Pluto.
(D) The gravitational pull of Pluto is not the sole
The statements above, if true, most strongly support which cause of Neptune’s irregular orbit.
one of the following conclusions about adults in country X? (E) Further observations of Pluto will eventually
(A) They generally consume more fat than do people show it to be even smaller than it is now thought
of the same age in other countries. to be.
(B) They generally eat more when they are older than
they did earlier in their adulthood.
(C) They generally have diets that contain a lower
percentage of fat than do the diets of children in
country X.
(D) They tend to eat more varied kinds of food as
they become older.
(E) They tend to lose weight as they become older.
32. Prolonged exposure to nonionizing radiation—
30. All of the cargo ships of the Blue Star Line are over
electromagnetic radiation at or below the frequency
100 meters long, and all of its passenger ships are under
100 meters long. Most of the ships of the Blue Star Line of visible light—increases a person’s chances of
were built before 1980. All of the passenger and cargo developing soft-tissue cancer. Electric power lines as
ships of the Gold Star line were built after 1980, and all well as such electrical appliances as electric blankets
are under 100 meters long. The dockside facilities of and video-display terminals are sources of nonionizing
Port Tropica, which is open only to ships of these two radiation.
lines, can accommodate only those ships that are less Which one of the following conclusions is best
than 100 meters long. The S.S. Coral is a cargo ship supported by the statements above?
that is currently docked at Port Tropica.
(A) People with short-term exposure to nonionizing
If the statements above are true, which one of the radiation are not at risk of developing soft-tissue
following must be true on the basis of them? cancers.
(A) The S.S. Coral was built after 1980. (B) Soft-tissue cancers are more common than other
(B) The S.S. Coral belongs to the Blue Star Line. cancers.
(C) Port Tropica is served only by cargo ships. (C) Soft-tissue cancers are frequently cured
(D) Port Tropica is not served by ships of the Blue spontaneously when sources of nonionizing
Star Line. radiation are removed from the patient’s home.
(E) All of the ships of the Blue Star Line are older (D) Certain electrical devices can pose health risks
than any of the ships of the Gold Star Line. for their users.
(E) Devices producing electromagnetic radiation
31. In a study of the relationship between aggression and at frequencies higher than that of visible light
television viewing in nursery school children, many do not increase a person’s risk of developing
interesting interactions among family styles, aggression, soft-tissue cancers.
and television viewing were found. High aggression
occurred in both high-viewing and low-viewing children 33. French divers recently found a large cave along the coast
and this seemed to be related to parental lifestyle. High- of the Mediterranean Sea. The cave is accessible only
achieving, competitive, middle-class parents, whose through an underwater tunnel. The interior of the cave is
children did not watch much television, had more completely filled with seawater and contains numerous
aggressive children than parents who planned their lives large stalagmites, which are stony pillars that form when
in an organized, child-centered way, which included drops of water fall repeatedly on a single spot on a cave
larger amounts of television viewing. floor, leaving behind mineral deposits that accumulate
over time.
Which one of the following conclusions is best
supported by the passage? The information above most strongly supports which
one of the following?
(A) Low levels of television viewing often lead to
high levels of aggression among children. (A) The Mediterranean Sea was at a higher level in
(B) The level of aggression of a child cannot be the past than it is now.
predicted from levels of television viewing (B) The water level within the cave is higher now
alone. than it once was.
(C) If high-achieving, competitive, parents were more (C) The French divers were the first people who knew
child-centered, their children would be less that the tunnel leading to the cave existed.
aggressive. (D) There was once an entrance to the cave besides
(D) High levels of television viewing can explain high the underwater tunnel.
levels of aggression among children only when (E) Seawater in the Mediterranean has a lower
the parents are not child-centered. mineral content now than it had when the
(E) Parental lifestyle is less important than the stalagmites were being formed.
amount of television viewing in determining the
aggressiveness of children.
36. Frequently, people who diet to lose weight become
34. Besides laying eggs in her own nest, any female wood
trapped in a vicious cycle. When those people diet, they
duck will lay an egg in the nest of another female wood
not only lose weight, but their bodies become used to
duck if she sees the other duck leaving her nest. Under
fewer calories and become accustomed to functioning at
natural nesting conditions, this parasitic behavior is
that lower rate of caloric intake. As a result, when they
relatively rare because the ducks’ nests are well hidden.
stop dieting and go back to eating amounts of food that
However, when people put up nesting boxes to help would have just maintained their weight in the days
the ducks breed, they actually undercut the ducks’ before the diet, they take in far more calories than they
reproductive efforts. These nesting boxes become so need. Those excess calories produce excess weight.
crowded with extra eggs that few, if any, of the eggs in
those boxes hatch. The passage above best supports which one of the
following conclusions about people who diet to lose
The statements above, if true, most strongly support weight?
which one of the following?
(A) They are bound to end up weighing more than
(A) Female wood ducks will establish nests in nest
when they started dieting.
boxes only when natural nesting sites are not
(B) They should not diet if they desire to maintain
available.
their reduced weight.
(B) Nesting female wood ducks who often see other
(C) They must not go back to eating pre-diet amounts
female wood ducks are the most successful in
of food if they are to maintain their weight at the
their breeding efforts.
reduced level resulting from dieting.
(C) The nesting boxes for wood ducks have less
(D) They will have to eat even less than the amount of
space for eggs than do natural nesting sites.
food allowed by their diets if they are to avoid
(D) The nesting boxes would be more effective gaining weight after they stop dieting.
in helping wood ducks breed if they were
(E) They never can go back to their pre-diet caloric
less visible to other wood ducks than they
intake without regaining all of the weight lost by
currently are.
dieting.
(E) Nesting boxes are needed to supplement the
natural nesting sites of wood ducks because of
37. The basic ingredients from which cement is made are
the destruction of much of the ducks’ habitat.
both cheap and plentiful. Materials as common as
limestone and clay will do. Nevertheless, the price of
35. The crux of creativity resides in the ability to
cement is influenced by the price of oil, because turning
manufacture variation on a theme. If we look at the
the basic ingredients into cement in high- temperature
history of science, for instance, we see that every idea
kilns uses large amounts of energy.
is built upon a thousand related ideas. Careful analysis
leads us to understand that what we choose to call a new Which one of the following can be logically inferred
theme or a new discovery is itself always and without from the passage?
exception some sort of variation, on a deep level, of (A) Oil is one of the basic ingredients that make up
previous themes. cement.
If all of the statements in the passage are true, each of (B) Oil is a source of energy for some of the kilns
the following must also be true EXCEPT: used in the making of cement.
(C) The higher the price of cement rises, the higher
(A) A lack of ability to manufacture a variation on a
the price of clay rises.
previous theme connotes a lack of creativity.
(D) Whenever oil prices rise, cement prices drop.
(B) No scientific idea is entirely independent of all
(E) A given amount of cement costs no more than the
other ideas.
total cost of its basic ingredients.
(C) Careful analysis of a specific variation can reveal
previous themes of which it is a variation.
(D) All great scientific discoverers have been able to
manufacture a variation on a theme.
(E) Some new scientific discoveries do not represent,
on a deep level, a variation on previous themes.
40. People often pronounce a word differently when asked
38. Some people claim that elected officials must avoid even to read written material aloud than when speaking
the appearance of impropriety in office. Yet since actions spontaneously. These differences may cause problems
that give the appearance of impropriety are not
for those who develop computers that recognize speech.
necessarily improper, the only reason for an elected
Usually the developers “train” the computers by using
official to avoid the appearance of impropriety is to
samples of written material read by the people who will
maintain public approval and popularity. No one,
be using the computer.
however, not even a public official, has an obligation to
be popular or to maintain public approval. The observations above provide most evidence for the
conclusion that
The argument is structured so as to lead to which one of
the following conclusions? (A) it will be impossible to develop computers that
decode spontaneous speech
(A) No elected official has an obligation to avoid the
(B) when reading written material, people who have
appearance of impropriety.
different accents pronounce the same word in
(B) All elected officials have a vested interest in
the same way as one another
maintaining a high public approval rating.
(C) computers may be less reliable in decoding
(C) Elected officials who have been scrupulous in
spontaneous speech than in decoding samples
satisfying the obligations of their office should
that have been read aloud
ensure that the public is aware of this fact.
(D) a “trained” computer never correctly decodes the
(D) The public never approves of an elected official
spontaneous speech of a person whose voice
who appears to have behaved improperly in
sample was used to train it
office.
(E) computers are now able to interpret oral speech
(E) Elected officials who abuse the power of their
without error
office have an obligation at least to appear to be
fulfilling the obligations of their office.

39. If a country’s manufacturing capacity is fully utilized,


there can be no industrial growth without new capital
investment. Any reduction in interest rates produces new
capital investment.
Which one of the following can be properly concluded
from the statements above?
(A) Interest rates might in exceptional cases be
reduced without there being any subsequent
investment of new capital.
(B) A reduction in interest rates might cause a
precondition for industrial growth to be met.
(C) If a country’s manufacturing capacity is
underutilized, interest rates should be held
constant.
(D) New capital investment that takes place while
interest rates are rising cannot lead to industrial
growth.
(E) Manufacturing capacity newly created by capital
investment needs to be fully utilized if it is to
lead to industrial growth.
43. Naturalist: To be dependable, the accounting framework
41. Advertisers have learned that people are more easily
used by national economists to advise the
encouraged to develop positive attitudes about things
government must take into account all of our
toward which they originally have neutral or even
nation’s assets; but the current accounting
negative attitudes if those things are linked, with
framework used by our national economists
pictorial help rather than exclusively through prose, to
assigns no value to government-owned natural
things about which they already have positive attitudes.
resources, which are clearly assets.
Therefore, advertisers are likely to .
The naturalist’s statements, if true, most strongly support
Which one of the following most logically completes the
which one of the following?
argument?
(A) Economists’ indifference toward the destruction
(A) use little if any written prose in their of natural resources will lead policymakers to
advertisements make poor decisions.
(B) try to encourage people to develop positive
(B) Naturalists and economists disagree about
attitudes about products that can be better
whether natural resources have value.
represented pictorially than in prose
(C) The accounting framework used by national
(C) place their advertisements on television rather
economists is not reliable.
than in magazines
(D) Natural resources are a vital economic asset for
(D) highlight the desirable features of the advertised
every nation.
product by contrasting them pictorially with
(E) Changes in the environment have a value that is
undesirable features of a competing product
not represented in any accounting framework.
(E) create advertisements containing pictures of
things most members of the target audience like
44. Critics have argued that because Freudianism holds that
people have unconscious desires that can defeat their
42. Environmentalist: Discarding old appliances attempts to follow rational life plans, it is incompatible
can be dangerous: refrigerators contain with the predominantly rationalistic spirit of Western
chlorofluorocarbons; electronic circuit boards and philosophical and psychological thought. But it is a
cathode-ray tubes often contain heavy metals like central tenet of Freudianism that through psychoanalysis
lead; and old fluorescent bulbs contain mercury, one can become conscious of one’s previously
another heavy metal. When landfills are operated unconscious desires, enabling one to avoid being
properly, such materials pose no threat. However, defeated by them. Therefore, .
when landfills are not operated properly, lead and
mercury from them contaminate groundwater, Which one of the following most logically completes the
for example. On the other hand, when trash is argument?
incinerated, heavy metals poison the ash and (A) Freudianism does not run counter to the
escape into the air. rationalistic mainstream of Western
The environmentalist’s statements, if true, most strongly philosophical and psychological thought
support which one of the following inferences? (B) Freudianism holds that people can always achieve
happiness through psychoanalysis
(A) Old fluorescent bulbs should be recycled.
(C) Freudianism may be the beginning of a new trend
(B) Appliances containing heavy metals should not
in Western philosophical and psychological
be incinerated.
thought
(C) Chlorofluorocarbons are harmful to the
(D) psychoanalysis provides one with a rational life
atmosphere.
plan
(D) Newer appliances are more dangerous to the
(E) Freudianism reflects the predominantly
environment than older ones.
rationalistic spirit of Western philosophical and
(E) Appliances should be kept out of landfills. psychological thought more than any other
psychological theory
47. Rosen: One cannot prepare a good meal from bad food,
45. Some argue that laws are instituted at least in part to produce good food from bad soil, maintain good
help establish a particular moral fabric in society. But soil without good farming, or have good farming
the primary function of law is surely to help order without a culture that places value on the proper
society so that its institutions, organizations, and maintenance of all its natural resources so that
needed supplies are always available.
citizenry can work together harmoniously, regardless of
any further moral aims of the law. Indeed, the highest Which one of the following can be properly inferred
courts have on occasion treated moral beliefs based from Rosen’s statement?
on conscience or religious faith as grounds for making (A) The creation of good meals depends on both
exceptions in the application of laws. natural and cultural conditions.
The statements above, if true, most strongly support (B) Natural resources cannot be maintained properly
which one of the following? without good farming practices.
(C) Good soil is a prerequisite of good farming.
(A) The manner in which laws are applied sometimes (D) Any society with good cultural values will have a
takes into account the beliefs of the people good cuisine.
governed by those laws. (E) When food is bad, it is because of poor soil and,
(B) The law has as one of its functions the ordering ultimately, bad farming practices.
of society but is devoid of moral aims.
(C) Actions based on religious belief or on moral 48. An instructor presented two paintings to a class. She
conviction tend to receive the protection of the said that the first had hung in prestigious museums but
highest courts. the second was produced by an unknown amateur. Each
(D) The way a society is ordered by law should not student was asked which painting was better. Everyone
reflect any moral convictions about the way selected the first. The instructor later presented the same
society ought to be ordered. two paintings in the same order to a different class. This
(E) The best way to promote cooperation among time she said that the first was produced by an unknown
a society’s institutions, organizations, and amateur but the second had hung in prestigious
citizenry is to institute order in that society by museums. In this class, everyone said that the second
means of law. painting was better.
The statements above, if true, most strongly support
46. Newtonian physics dominated science for over two
which one of the following?
centuries. It found consistently successful application,
becoming one of the most highly substantiated and (A) Most of the students would not like any work of
accepted theories in the history of science. Nevertheless, art that they believed to have been produced by
Einstein’s theories came to show the fundamental limits an unknown amateur.
of Newtonian physics and to surpass the Newtonian (B) None of the claims that the instructor made about
view in the early 1900s, giving rise once again to a the paintings was true.
physics that has so far enjoyed wide success. (C) Each of the students would like most of the
paintings hanging in any prestigious museum.
Which one of the following logically follows from the
(D) In judging the paintings, some of the students
statements above?
were affected by what they had been told about
(A) The history of physics is characterized by a the history of the paintings.
pattern of one successful theory subsequently (E) Had the instructor presented the paintings without
surpassed by another. telling the students anything about them, almost
(B) Long-standing success of substantiation of a all of the students would have judged them to be
theory of physics is no guarantee that the theory roughly equal in artistic worth.
will continue to be dominant indefinitely.
(C) Every theory of physics, no matter how
successful, is eventually surpassed by one that is
more successful.
(D) Once a theory of physics is accepted, it will
remain dominant for centuries.
(E) If a long-accepted theory of physics is surpassed,
it must be surpassed by a theory that is equally
successful.
51. In speech, when words or sentences are ambiguous,
49. Commentator: In the new century, only nations with all gesture and tone of voice are used to indicate the
the latest electronic technology will experience intended meaning. Writers, of course, cannot use
great economic prosperity. The people in these
gesture or tone of voice and must rely instead on style;
nations will be constantly bombarded with images
the reader detects the writer’s intention from the
of how people in other countries live. This will
arrangement of words and sentences.
increase their tendency to question their own
customs and traditions, leading to a dissolution of Which one of the following statements is most strongly
those customs and traditions. Hence, in the new supported by the information above?
century, the stability of a nation’s cultural identity (A) The primary function of style in writing is to
will likely . augment the literal meanings of the words and
Which one of the following most logically completes the sentences used.
commentator’s argument? (B) The intended meaning of a piece of writing is
indicated in part by the writer’s arrangement of
(A) depend on a just distribution of electronic
words and sentences.
technology among all nations
(C) It is easier for a listener to detect the tone of a
(B) decrease if that nation comes to have a high level
speaker than for a reader to detect the style of a
of economic wealth
writer.
(C) be ensured by laws that protect the customs and
(D) A writer’s intention will always be interpreted
traditions of that culture
differently by different readers.
(D) be threatened only if the people of that culture
(E) The writer’s arrangement of words and sentences
fail to acquire the latest technical skills
completely determines the aesthetic value of his
(E) be best maintained by ensuring gradual
or her writing.
assimilation of new technical knowledge and
skills
52. For newborns of age four to six weeks whose mothers
have been the primary caregivers, the following is true:
50.
When the newborns are crying due to hunger or other
This question has been removed. similar discomfort, merely hearing the mother’s voice
will lead to a temporary halt in crying, while the voices
of others do not have this effect.
Which one of the following is most reasonably
supported by the information above?
(A) Babies more easily learn to recognize the voices
of their mothers than the voices of other people.
(B) A mother’s voice is the first thing a baby learns to
recognize.
(C) Babies associate the voice of the primary
caregiver with release from discomfort.
(D) Often only a primary caregiver can provide
comfort to a newborn.
(E) Discomfort in newborns is best relieved by
hearing the mother’s voice.
55. Nutritionist: Many people claim that simple
53. Consultant: Most workers do not have every item they
carbohydrates are a reasonable caloric
produce judged for quality, but each piece a
replacement for the fatty foods forbidden to
freelance writer authors is evaluated. That is why
those on law-fat diets. This is now in doubt.
freelance writers produce such high-quality work.
New studies show that, for many people, a high
The consultant’s statements, if true, most strongly intake of simple carbohydrates stimulates an
support which one of the following? overproduction of insulin, a hormone that is
involved in processing sugars and starches to
(A) A piece authored by a freelance writer is
create energy when the body requires energy, or,
generally evaluated more strictly than the
when energy is not required, to store the resulting
majority of items most workers produce.
by-products as fat.
(B) By having every piece of their work evaluated,
some workers are caused to produce high-quality Which one of the following is most strongly supported
work. by the nutritionist’s statements?
(C) No other workers produce higher quality work
(A) People on low-fat diets should avoid consumption
than do freelance writers.
of simple carbohydrates if they wish to maintain
(D) Only freelance writers have every item they the energy that their bodies require.
produce evaluated for quality.
(B) People who produce enough insulin to process
(E) Some workers produce high-quality work in spite
their intake of simple carbohydrates should not
of the fact that not every item they produce is
feel compelled to adopt low-fat diets.
judged for quality.
(C) People who consume simple carbohydrates
should limit their intake of foods high in fat.
54. The same task triggers different levels of awareness of
(D) People who wish to avoid gaining body fat
one’s surroundings, called environmental awareness, in
should limit their intake of foods high in simple
different individuals. Mathematical puzzles, for example,
carbohydrates.
cause most people to increase such an awareness. Some
(E) People who do not produce an excessive amount
people—those who formulate the answer visually,
of insulin when they consume foods high in
imagining the numbers in their mind’s eye—will, in
simple carbohydrates will not lose weight if they
an attempt to freeze the picture, experience a decrease restrict only their intake of these foods.
in environmental awareness while solving the puzzle.
Other people’s environmental awareness may rise during
56. The axis of Earth’s daily rotation is tilted with respect to
the exercise, because their brains are signaling a rest at the plane of its orbit at an angle of roughly 23 degrees.
the end of every stage of problem solving. That angle can be kept fairly stable only by the
Which one of the following is most strongly supported gravitational influence of Earth’s large, nearby Moon.
by the information above? Without such a stable and moderate axis tilt, a planet’s
climate is too extreme and unstable to support life.
(A) There are some people for whom mathematical Mars, for example, has only very small moons, tilts at
puzzles do not cause an increase in their level of wildly fluctuating angles, and cannot support life.
environmental awareness.
(B) People who visually formulate answers differ If the statements above are true, which one of the
from other problem solvers in that the former are following must also be true on the basis of them?
aware of their surroundings. (A) If Mars had a sufficiently large nearby moon,
(C) People tend to be more aware of their Mars would be able to support life.
surroundings when solving mathematical (B) If Earth’s Moon were to leave Earth’s orbit,
problems than when solving nonmathematical Earth’s climate would be unable to support life.
problems. (C) Any planet with a stable, moderate axis tilt can
(D) Mathematical problem solvers who rely on visual support life.
techniques become aware of their surroundings (D) Gravitational influences other than moons have
only during periods of rest. little or no effect on the magnitude of the tilt
(E) Mathematical problem solving requires frequent angle of either Earth’s or Mars’s axis.
periods of rest in the form of increased (E) No planet that has more than one moon can
awareness of the problem solver’s surroundings. support life.
59. Nutritionist: A study revealed that although most adults
57. Poppy petals function to attract pollinating insects. The
estimated their diets to correspond closely with
pollination of a poppy flower triggers the release into
the recommendations of standard nutritional
that flower of a substance that causes its petals to wilt
guidelines, most of their diets did not come
within one or two days. If the flower is not pollinated,
close to those recommendations. Both women
the substance will not be released and the petals will
and men underestimated the amount of fat in
remain fresh for a week or longer, as long as the plant
their diets and overestimated their intake of most
can nourish them. Cutting an unpollinated poppy flower
other foods. In most food categories, especially
from the plant triggers the release into the flower
fruits and vegetables, women’s diets did not meet
of the same substance whose release is triggered by the recommendations. Men underestimated their
pollination. fat intake by half, and though they met the
The statements above, if true, most strongly support recommendations for breads, they fell short in all
which one of the following? other categories.
(A) Pollinating insects are not attracted to wilted Which one of the following is most strongly supported
poppy flowers. by the information offered by the nutritionist?
(B) Even if cut poppies are given all necessary
(A) Both men and women in the study misjudged
nutrients, their petals will tend to wilt within a
their compliance with the nutritional guidelines
few days.
in every food category.
(C) Flowers of all plants release the substance that
(B) In the study, more men than women were aware
causes wilting when they are cut, although the
that in some food categories their diet failed to
amount released may vary.
reflect the recommendations closely.
(D) The pollen on pollinated poppy flowers prevents
(C) Women in the study were more aware than men
their petals from absorbing the nutrients carried
were of the recommended intake of breads.
to them by their stems.
(D) Men in the study estimated their daily intake of
(E) Poppy plants are unable to draw nutrients from
fruits and vegetables to be significantly lower
soil or water after the substance that causes
than it in fact was.
wilting has been released.
(E) Most men in the study did not consume the
amounts of fruits and vegetables that the
58. We should do what will make others more virtuous and
nutritional guidelines recommend.
not do what will make others less virtuous. It is an irony
of human existence that praise makes those who are less
60.
virtuous more virtuous, while it makes those who are
more virtuous less virtuous. And, of course, none except This question has been removed.
the more virtuous deserve praise.
From the statements above, if true, which one of the
following can be properly inferred?
(A) We should withhold praise from those who
deserve it least.
(B) We should not fail to praise those who deserve it
most.
(C) We should praise those who do not deserve it and
withhold praise from those who deserve it.
(D) We should praise everyone, regardless of whether
or not they deserve it.
(E) We should withhold praise from everyone,
regardless of whether or not they deserve it.
63. In 1712 the government of Country Y appointed a
61. Clothes made from natural fibers such as cotton, unlike
censor to prohibit the publication of any book critical
clothes made from artificial fibers such as polyester
of Country Y’s government; all new books legally
often shrink when washed at high temperatures. The
published in the country after 1712 were approved by a
reason for this shrinkage is that natural fibers are tightly
censor. Under the first censor, one half of the book
curled in their original state. Since the manufacturer
manuscripts submitted to the censor were not approved
of cloth requires straight fibers, natural fibers are
for publication. Under the next censor, only one quarter
artificially straightened prior to being made into cloth.
of the book manuscripts submitted were not approved,
High temperatures cause all fibers in cloth to return to
but the number of book manuscripts that were approved
their original states.
was the same under both censors.
Which one of the following is most strongly supported
It the statements in the passage are true, which one of
by the information above?
the following can be properly concluded from them?
(A) Washing clothes made from natural fibers at low
(A) More books critical of Country Y’s government
temperatures causes the fibers to straighten
were published before the appointment of the
slightly.
first censor than after it.
(B) High temperatures have no effect on the
(B) The first censor and the second censor prohibited
straightness of fibers in clothes made from a
the publication of the same number of book
blend of natural and artificial fibers.
manuscripts.
(C) Clothes made from natural fibers stretch more
(C) More book manuscripts were submitted for
easily than do clothes made from artificial fibers.
approval to the first censor than to the second.
(D) If natural fibers that have been straightened
(D) The second censor allowed some book
and used for cloth are curled up again by high
manuscripts to be published that the first censor
temperatures, they cannot be straightened again.
would have considered critical of Country Y’s
(E) Artificial fibers are straight in their original state. government.
(E) The number of writers who wrote published
62. In Yasukawa’s month-long study of blackbirds, the
manuscripts was greater under the first censor
percentage of smaller birds that survived the duration of
than under the second.
the study exceeded the percentage of larger birds that
survived. However, Yasukawa’s conclusion that size is a
64. The Green Ensemble, a nonprofit theater group, has
determinant of a blackbird’s chances of survival over a always been financially dependent on contributions from
month-long period is probably mistaken, since smaller corporations and would have been forced to disband this
blackbirds are generally younger than larger ones. year if any of its corporate sponsors had withdrawn their
The statements above, if true, support which one of the financial support. But the Green Ensemble has not only
following inferences? been able to continue in operation throughout the year,
but has recently announced its schedule for next year.
(A) Among the blackbirds that survived the month-
long study, there was no relation between size Which one of the following is a conclusion that can be
and age. properly drawn from the information above?
(B) Larger blackbirds of a given age are actually (A) None of the Green Ensemble’s corporate sponsors
more likely to survive over a one-month period withdrew their financial support of the group this
than are smaller blackbirds of the same age. year.
(C) Among blackbirds of the same size, a difference (B) Earlier this year the Green Ensemble found
in age probably does not indicate a difference in other sources of funding for next year, making
chances of survival over a one-month period. the group less dependent on corporations for
(D) Among blackbirds of the same age, a difference financial support.
in size may not indicate a difference in chances (C) During this year corporate funding for the Green
of survival over a month-long period. Ensemble has been steadily increasing.
(E) With a larger sample of blackbirds, the (D) This year corporate funding was the source of
percentage of smaller birds that survive a more than half of the Green Ensemble’s income.
one-month period would be the same as the (E) Corporate funding for nonprofit theater groups
percentage of larger birds that survive. like the Green Ensemble has recently increased.
67. Early pencil leads were made of solid graphite mined
65. Words like “employee,” “payee,” and “detainee” support
in Cumberland, in Britain. Modern methods of
the generalization, crudely stated, that words with the
manufacturing pencil leads from powdered graphite
ending -ee designate the person affected in the specified
are the result of research sponsored by the government
way by an action performed by someone else. The word
of France in the 1790s, when France was at war with
“absentee” seems to be a direct counterexample: it ends
Britain and thus had no access to Cumberland graphite.
in -ee, yet, if it makes sense here to speak of an action at
all—that of absenting oneself, perhaps—the word can The information above most strongly supports which
refer to the person who actually performs that action. one of the following?
Clearly, however, putting forward the following resolves (A) The world’s only deposit of graphite suitable for
the impasse: if a word with the ending -ee refers to one manufacture of pencils is in Cumberland, in
party in a two-party transaction, it refers to the party at Britain.
which the other party’s action is directed. (B) In the 1790s, France’s government did not know
The reasoning in the argument could have remained of any accessible source of solid graphite
unchanged in force and focus if which one of the appropriate to meet France’s need for pencils.
following had been advanced as a counterexample in (C) One of the causes of war between France and
place of the word “absentee”? Britain in the 1790s was the British
government’s attempt to limit the amount of
(A) honoree
Cumberland graphite being exported to France.
(B) appointee
(D) Government-sponsored research frequently gives
(C) nominee
rise to inventions that are of great benefit to
(D) transferee
society.
(E) escapee
(E) Even today, all pencil leads contain Cumberland
graphite.
66. Much of today’s literature is inferior: most of our
authors are intellectually and emotionally inexperienced,
68. A person’s dietary consumption of cholesterol and
and their works lack both the intricacy and the focus
fat is one of the most important factors determining
on the significant that characterize good literature.
the level of cholesterol in the person’s blood (serum
However, Hypatia’s latest novel is promising; it shows a
cholesterol). Serum cholesterol levels rise proportionally
maturity, complexity, and grace that far exceeds that of
to increased cholesterol and fat consumption until that
her earlier works.
consumption reaches a threshold, but once consumption
Which one of the following statements is most strongly of these substances exceeds that threshold, serum
supported by the information in the passage? cholesterol levels rise only gradually, even with dramatic
increases in consumption. The threshold is one fourth the
(A) Much of today’s literature focuses less on the
consumption level of cholesterol and fat in today’s
significant than Hypatia’s latest novel focuses on
average North American diet.
the significant.
(B) Much of today’s literature at least lacks the The statements above, if true, most strongly support
property of grace. which one of the following?
(C) Hypatia’s latest novel is good literature when
(A) The threshold can be lowered by lowering the
judged by today’s standards.
dietary consumption of cholesterol and fat.
(D) Hypatia’s latest novel is clearly better than the
(B) People who consume an average North American
majority of today’s literature.
diet cannot increase their consumption of
(E) Hypatia’s latest novel has at least one property
cholesterol and fat without dramatically
of good literature to a greater degree than her increasing their serum cholesterol levels.
earlier works.
(C) People who consume half as much cholesterol
and fat as in the average North American diet
will not necessarily have half the average serum
cholesterol level.
(D) Serum cholesterol levels cannot be affected by
nondietary modifications in behavior, such as
exercising more or smoking less.
(E) People who consume less cholesterol and fat
than the threshold cannot reduce their serum
cholesterol levels.
71. Wild cheetahs live in the African grasslands. Previous
69. The Levant—the area that borders the eastern
estimates of the size that the wild cheetah population
Mediterranean-was heavily populated in prehistoric
must be in order for these animals to survive a natural
times. The southern Levant was abandoned about 6,000
disaster in the African grasslands region were too small,
years ago, although the northern Levant, which shared
and the current population barely meets the previous
the same climate, remained heavily populated. Recently
estimates. At present, however, there is not enough
archaeologists have hypothesized that the sudden
African grassland to support a wild cheetah population
depopulation in the southern Levant was due to an
larger than the current population.
economic collapse resulting from deforestation.
The statements above, if true, most strongly support
If the statements above are true and the archaeologists’
which one of the following conclusions?
hypothesis is correct, which one of the following
CANNOT be true? (A) Previous estimates of the size of the existing wild
cheetah population were inaccurate.
(A) The sheep and goats herded by the peoples of the
(B) The cheetah’s natural habitat is decreasing in
southern Levant until 6,000 years ago grazed
size at a faster rate than is the size of the wild
extensively on the seedlings and saplings of
cheetah population.
indigenous tree species.
(C) The principal threat to the endangered wild
(B) Trees were used in the production of lime plaster, cheetah population is neither pollution nor
a building material used extensively throughout hunting, but a natural disaster.
the southern Levant until 6,000 year ago.
(D) In the short term, the wild cheetah population
(C) Organic remains from the northern Levant reliably
will be incapable of surviving a natural disaster
indicate that tree species flourished there without
in the African grasslands.
interruption during the period when the southern
(E) In regions where land is suitable for cheetah
Levant was being abandoned.
habitation, more natural disasters are expected
(D) Carbon dating of organic remains from the to occur during the next decade than occurred
southern Levant reliably demonstrates that there during the past decade.
were no forests present in that area prior to
6,000 years ago.
72. Muriel: I admire Favilla’s novels, but she does not
(E) Since there are few traces of either quarried stone
deserve to be considered great writer. The point is
or of mud brick in buildings excavated in the
that, no matter how distinctive her style may be,
southern Levant, it is likely that the buildings
her subject matter is simply not varied enough.
built there prior to 6,000 years ago were made
almost entirely of timber. John: I think you are wrong to use that criterion. A great
writer does not need any diversity in subject
70. All people prefer colors that they can distinguish easily matter; however, a great writer must at least have
to colors that they have difficulty distinguishing. Infants the ability to explore a particular theme deeply.
can easily distinguish bright colors but, unlike adults, John’s statements commit him to which one of the
have difficulty distinguishing subtle shades. A brightly following positions?
colored toy for infants sells better than the same toy in
subtle shades at the same price. (A) Even if the subject matter in Favilla’s writings is
not particularly varied, she should not thereby be
Which one of the following conclusions is most strongly excluded from being considered a great writer.
supported by the information in the passage? (B) Even if Favilla cannot explore any particular
(A) Infants prefer bright primary colors to bright theme deeply in her writings, she should not
secondary colors. thereby be excluded from being considered a
(B) Color is the most important factor in determining great writer.
which toys an infant will prefer to play with. (C) If Favilla has explored some particular theme
(C) Individual infants do not have strong preferences exceptionally deeply in her writings, she
for one particular bright color over other bright deserves to be considered a great writer.
colors. (D) If the subject matter in Favilla’s writings were
(D) The sales of toys for infants reflect the exceptionally varied, she would not deserve to
preferences of infants in at least one respect. be considered a great writer.
(E) Toy makers study infants to determine what (E) If Favilla’s writings show no evidence of a
colors the infants can distinguish easily. distinctive style, she does not deserve to be
considered a great writer.
75. Everyone sitting in the waiting room of the school’s
73. When soil is plowed in the spring, pigweed seeds that athletic office this morning at nine o’clock had just
have been buried in the soil all winter are churned up to registered for a beginners’ tennis clinic. John, Mary, and
the surface and redeposited just under the surface. The
Teresa were all sitting in the waiting room this morning
brief exposure of the seeds to sunlight stimulates
at nine o’clock. No accomplished tennis player would
receptors, which have become highly sensitive to
register for a beginners’ tennis clinic.
sunlight during the months the seeds were buried in the
soil, and the stimulated receptors trigger germination. It the statements above are true, which one of the
Without the prolonged darkness, followed by exposure following must also be true on the basis of them?
to sunlight, the seeds do not germinate. (A) None of the people sitting in the school’s athletic
The statements above, if true, most strongly support office this morning at nine o’clock had ever
which one of the following statements about a field that played tennis.
will be plowed in the spring and in which pigweed seeds (B) Everyone sitting in the school’s athletic office this
have been buried in the soil all winter? morning at nine o’clock registered only for a
beginners’ tennis clinic.
(A) Fewer pigweed plants will grow in the field if it is
(C) John, Mary, and Teresa were the only people
plowed only at night than if it is plowed during
who registered for a beginners’ tennis clinic
the day.
this morning.
(B) Fewer pigweed plants will grow in the field if it is
(D) John, Mary, and Teresa were the only people
plowed at all than if it is plowed only at night.
sitting in the waiting room of the school’s
(C) Fewer pigweed plants will grow in the field if it
athletic office this morning at nine o’clock.
is plowed just before sunrise than if it is plowed
(E) Neither John nor Teresa is an accomplished tennis
just after sunset.
player.
(D) The pigweed seeds that are churned up to the
surface of the soil during the plowing will not
76. The Rienzi, a passenger ship, sank as a result of a hole
geminate unless they are redeposited under the in its hull, possibly caused by sabotage. Normally, when
surface of the soil. a holed ship sinks as rapidly as the Rienzi did, water
(E) All of the pigweed seeds that are already on the does not enter the ship quickly enough for the ship to be
surface of the soil before the field is plowed will fully flooded when it reaches the ocean floor. Full
germinate. flooding can be achieved, however, by sabotage. Any
ship that sinks deep into the ocean when not fully
74. The increase in the price of jet fuel is due to a sharp flooded will implode. Deep-sea photographs, taken of
decrease over the past year in the supply of jet fuel the sunken Rienzi where it rests on the ocean floor,
available relative to demand. Nonetheless, the amount of reveal that the Rienzi did not implode.
jet fuel available for sale is larger today than it was last
year. Which one of the following must be true on the basis of
the information above?
If the statements above are true, which one of the
following conclusions can be properly drawn on the (A) The Rienzi was so constructed as to reduce the
basis of them? risk of sinking by impact.
(B) If the Rienzi became fully flooded, it did so only
(A) The demand for jet fuel has increased over the after it reached the ocean floor.
past year. (C) If the Rienzi was not sunk by sabotage, water
(B) The fuel efficiency of jet engines has increased flooded into it unusually fast.
over the past year. (D) If the Rienzi had sunk more slowly, it would have
(C) The number of jet airline flights has decreased imploded.
over the past year. (E) The Rienzi was so strongly constructed as to
(D) The cost of refining petroleum for jet fuel has resist imploding under deep-sea pressure.
increased over the past year.
(E) The supply of petroleum available for jet fuel has
decreased over the past year.
79. The frozen, well-preserved body of a man was recently
77. In a democratic society, when a political interest group
discovered in a glacier as the glacier melted. Artifacts
exceeds a certain size, the diverse and sometimes
conflicting economic interests that can be found in found on the body reveal that the man lived at least
almost any large group of people tend to surface. Once 4,000 years ago. The nature of the glacier indicates that
these conflicting interests have surfaced, they can make the man died on virtually the same spot where his body
it impossible for the political interest group to unite was discovered. If the body had not been sealed in the
behind a common program. Yet to have the political glacier shortly after the man died, or if the body had
impact necessary to influence legislation, a group must thawed in the intervening millennia, it would not still be
be united. preserved.

The statements above, if true, most strongly support Which one of the following is most strongly supported if
which one of the following views? all of the statements above are true?
(A) Political interest groups are generally less (A) None of the artifacts found on the body were
influential when their membership is expanding more than 4,000 years old.
than when it is numerically stable. (B) The man whose body was discovered in the
(B) For a democratic society to function effectively, glacier did not freeze to death.
it is necessary for political groups within that (C) The glacier that melted to reveal the ancient is at
society to make compromises with each other. least 4,000 years old.
(C) Politicians can ignore with impunity the (D) The artifacts found on the frozen body would not
economic interests of very large groups of have been preserved if they had not also been
people. frozen.
(D) A political interest group can become ineffective (E) The global climate is, on average, warmer now
by expanding to include as wide a membership than it was 4,000 years ago.
as possible.
(E) Political interest groups generally start out 80. A tiny, tree-living thrips is the first species identified
effectively but lose their effectiveness over time. that can reproduce both by laying eggs and by
bearing live young, although for any one instance of
78. Fossilized bones from the hominids Australopithecus reproduction, a given female will use only one of the
robustus and Homo erectus were chemically analyzed. two methods. Interestingly, biologists have observed that
The robustus bones contained a lower ratio of strontium all of the offspring of this insect that are hatched from
to calcium than did the erectus bones. The lower the eggs are females, and all of the offspring that are live-
ratio of strontium to calcium in fossilized hominid born are males. It has also been noted that any particular
bones, the more meat the hominid had in its diet. live-born brood will comprise fewer individuals than any
H. erectus is known to have eaten meat. particular brood hatched from eggs. However, a much
larger proportion of male offspring than of female
The statements above, if true, most strongly support offspring survives to adulthood, and among thrips
which one of the following? reaching adulthood the ratio of males to females is about
(A) The diet of A. robustus included at least some even.
meat. Which one of the following can be properly inferred
(B) The meat in the diet of H. erectus was higher in about the species of thrips described in the passage?
strontium than was the meat in the diets of other
hominids. (A) It is the only species capable of using two
(C) The diet of H. erectus was richer in calcium than different methods of reproduction.
was the diet of A. robustus. (B) Any female of the species that can reproduce by
(D) The diets of A. robustus and H. erectus both laying eggs can also reproduce by bearing live
contained less strontium than calcium. young but not necessarily vice versa.
(E) The process of fossilization altered the ratio (C) On average, across the species, more of the
of strontium to calcium in the bones of both offspring are born by hatching from eggs than
A. robustus and H. erectus. are born live.
(D) For the species as a whole, there are, over time,
about as many instances of live-born broods as
there are broods hatched from eggs.
(E) There are females that use only one of the two
methods of reproduction over the course of their
lives.
83. A number of measures indicate the viability of a
81. To be great, an artwork must express a deep emotion,
nation’s economy. The level and rate of growth of
such as sorrow or love. But an artwork cannot express
an emotion that the artwork’s creator is incapable of aggregate output are the most significant indicators, but
experiencing. unemployment and inflation rates are also important.
Further, Switzerland, Austria, Israel, Ireland, Denmark,
Which one of the following can be properly inferred and Finland all have viable economics, but none has
from the statements above? a very large population. Switzerland and Austria each
(A) A computer can create an artwork that expresses have populations of about seven million; the other
sorrow or love only if it has actually experienced populations are at least one-fourth smaller.
such an emotion. Which one of the following is most strongly supported
(B) The greatest art is produced by those who have by the information above?
experienced the deepest emotions.
(C) An artwork that expresses a deep emotion of its (A) A nation’s economic viability is independent of
creator is a great artwork. the size of its population.
(D) As long as computers are constructed so as to be (B) Having a population larger than seven million
incapable of experiencing emotions, they will ensures that a nation will be economically
not create great artworks. viable.
(E) Only artworks that succeed in expressing deep (C) Economic viability does not require a population
emotions are the products of great artists. of at least seven million.
(D) A nation’s population is the most significant
82. In the troposphere, the lowest level of the earth’s contributor to the level and rate of growth of
atmosphere, the temperature decreases as one progresses aggregate output.
straight upward. At the top, the air temperature ranges (E) A nation’s population affects the level and rate of
from –50 degrees Celsius over the poles to –85 degrees growth of aggregate output more than it affects
Celsius over the equator. At that point the stratosphere unemployment and inflation rates.
begins, and the temperature stops decreasing and instead
increases as one progresses straight upward through
the stratosphere. The stratosphere is warmed by ozone.
When an ozone particle absorbs a dose of ultraviolet
sunlight, heat is generated.
If the statements above are true, which one of the
following must also be true?
(A) The troposphere over the poles is thicker than the
troposphere over the equator.
(B) It is warmer at the top of the stratosphere over the
poles than it is at the top of the stratosphere over
the equator.
(C) The temperature in the middle part of the
stratosphere over the North Pole is at least as
great as the temperature in the middle part of the
stratosphere over the equator.
(D) The temperature at any point at the top of
the stratosphere is at least as great as the
temperature at the top of the troposphere directly
beneath that point.
(E) Depletion of the earth’s ozone layer would
increase the air temperature in the stratosphere
and decrease the air temperature in the
troposphere.
86. Most people invest in the stock market without doing
84. Zachary: The term “fresco” refers to paint that has any research of their own. Some of these people rely
been applied to wet plaster. Once dried, a fresco solely on their broker’s advice, whereas some others
indelibly preserves the paint that a painter has make decisions based merely on hunches. Other people
applied in this way. Unfortunately, additions do some research of their own, but just as often rely
known to have been made by later painters only on their broker or on hunches. Only a few always
have obscured the original fresco work done by do their own research before investing. Nonetheless, a
Michelangelo in the Sistine Chapel. Therefore, in majority of investors in the stock market make a profit.
order to restore Michelangelo’s Sistine Chapel
paintings to the appearance that Michelangelo If the statements in the passage are true, which one of
intended them to have, everything except the the following must also be true?
original fresco work must be stripped away. (A) Some people who make a profit on their
investments in the stock market do so without
Stephen: But it was extremely common for painters of
doing any research of their own.
Michelangelo’s era to add painted details to their
own fresco work after the frescos had dried. (B) Most people who invest in the stock market either
rely solely on their broker or make decisions
Stephen’s response to Zachary, if true, most strongly based merely on hunches.
supports which one of the following? (C) Some people who do investment research on their
(A) It is impossible to distinguish the later painted own, while just as often relying on their broker
additions made to Michelangelo’s Sistine Chapel or on hunches, make a profit in the stock market.
paintings from the original fresco work. (D) Most people who invest in the stock market without
(B) Stripping away everything except doing any research of their own make a profit.
Michelangelo’s original fresco work from the (E) Most people who rely solely on their broker rather
Sistine Chapel paintings would be unlikely to than on hunches make a profit in the stock market.
restore them to the appearance Michelangelo
intended them to have. 87. When an invading insect threatens an ant colony’s
(C) The painted details that painters of territory or food sources, the ants will vigorously swarm
Michelangelo’s era added to their own fresco over the invader, biting or stinging it. This defensive
work were not an integral part of the completed tactic can effectively deter even aggressive flying insects,
paintings’ overall design. such as wasps. Ants do not attack all insects within their
(D) None of the painters of Michelangelo’s era who territory, however. For example, riodinid caterpillars
made additions to the Sistine Chapel paintings commonly live harmoniously among South American
was an important artist in his or her own right. ants. These caterpillars, which are a favorite prey of
(E) Michelangelo was rarely satisfied with the wasps, produce secretions the ants consume as food.
appearance of his finished works. Which one of the following is most strongly supported
by the information above?
85. Politician: My opponents argue that the future of our
city depends on compromise—that unless the (A) The secretions produced by riodinid caterpillars
city’s leaders put aside their differences and are chemically identical to substances secreted
work together toward common goals, the city by plants on which South American ants also
will suffer. However, the founders of this city feed.
based the city’s charter on definite principles, and (B) South American ants are more likely to be
successful in defending their food sources and
anyone who compromises those principles betrays
territory against intruders than are ants that live
the city founders’ goals. What my opponents
elsewhere.
are advocating, therefore, is nothing less than
betraying the goals of the city’s founders. (C) With the sole exception of riodinid caterpillars,
South American ants will vigorously attack any
Critic: I’m afraid your argument is flawed. Unless organism other than ants that use riodinids as a
you’re assuming that the differences among the source of food.
city’s leaders are differences of principle, your (D) Among insect species that inhabit South America,
argument depends on a misleading use of the wasps are the only kinds of organism other than
term . ants that use riodinid caterpillars as a source of
Which one of the following provides the most logical food.
completion of the critic’s statement? (E) Riodinid caterpillars in South America that live
among ants are less likely to be attacked by
(A) betray wasps than those that do not live among ants.
(B) common
(C) compromise
(D) principles
(E) opponents
90. Editorial: The government claims that the country’s
88. Two hundred randomly selected subjects were asked, nuclear power plants are entirely safe and hence
“Have you ever awakened, seemingly paralyzed, with a that the public’s fear of nuclear accidents at these
sense of a strange presence in the room?” Forty percent plants is groundless. The government also
answered yes. A randomly selected control group of 200 contends that its recent action to limit the nuclear
different subjects in the same study were asked simply if industry’s financial liability in the case of nuclear
they remembered ever waking up seemingly paralyzed. accidents at power plants is justified by the need
Only 14 percent of the control group answered yes. to protect the nuclear industry from the threat of
Which one of the following statements is most bankruptcy. But even the government says that
supported by the information above? unlimited liability poses such a threat only if
injury claims can be sustained against the
(A) Experiencing a sense of a strange presence in a industry; and the government admits that for such
room in some way causes subjects to feel as claims to be sustained, injury must result from a
though they are paralyzed. nuclear accident. The public’s fear, therefore, is
(B) The number of subjects who had awakened with well founded.
a sense of a strange presence in the room was
greater in the first group than in the control If all of the statements offered in support of the editorial’s
group. conclusion correctly describe the government’s position,
(C) If the reports of the first group of subjects were which one of the following must also be true on the basis
accurate, approximately 60 percent of them of those statements?
had never awakened with a sense of a strange (A) The government’s claim about the safety of the
presence in the room. country’s nuclear power plants is false.
(D) At least some of the randomly selected subjects (B) The government’s position on nuclear power
of the study gave inconsistent reports. plants is inconsistent.
(E) The tendency of subjects to report a recollection (C) The government misrepresented its reasons for
of an event can sometimes be increased by acting to limit the nuclear industry’s liability.
suggesting circumstances that accompanied the (D) Unlimited financial liability in the case of nuclear
event. accidents poses no threat to the financial security
of the country’s nuclear industry.
89. Chelas and Stelma are required to leave their respective (E) The only serious threat posed by a nuclear
stations immediately to pursue any prisoner who accident would be to the financial security of the
attempts to escape from their sector. Furthermore, unless nuclear industry.
they are pursuing such a prisoner, Chelas and Stelma are
forbidden to leave their stations until their replacements
have arrived. On May 11 at 9 P.M., when Chelas and
Stelma finished the four-hour shift in their sector and
their replacements arrived, it was discovered that Chelas
had violated these rules and Stelma had not.
If the statements above are true, each of the following
could be true EXCEPT:
(A) Chelas and Stelma were at their respective
stations at 9 P.M.
(B) Stelma left her station before 9 P.M. but Chelas
did not.
(C) Chelas left his station before 9 P.M. but Stelma
did not.
(D) A prisoner attempted to escape from Chelas and
Stelma’s sector at 7 P.M. and neither Chelas nor
Stelma left his or her station before 9 P.M.
(E) A prisoner attempted to escape from Chelas and
Stelma’s sector at 7 P.M. and both Chelas and
Stelma left their stations before 9 P.M.
93. Under the influence of today’s computer-oriented
91. Editorialist: Drivers with a large number of demerit
culture, publishing for children has taken on a flashy
points who additionally have been convicted of a
serious driving-related offense should either be new look that emphasizes illustrations and graphic
sentenced to jail or be forced to receive driver design; the resulting lack of substance leads to books
re-education, since to do otherwise would be to that are short-lived items covering mainly trendy
allow a crime to go unpunished. Only if such subjects. The changes also include more humorous
drivers are likely to be made more responsible content, simplification of difficult material, and a
drivers should driver re-education be narrower focus on specific topics.
recommended for them. Unfortunately, it is Which one of the following is most strongly supported
always almost impossible to make drivers with a by the information above?
large number of demerit points more responsible
drivers. (A) The inclusion of humorous material and a
narrower focus detract from the substance of a
If the editorialist’s statements are true, they provide the children’s book.
most support for which one of the following? (B) The substance of a children’s book is important to
(A) Drivers with a large number of demerit points its longevity.
who have been convicted of a serious driving- (C) Children of the computer generation cannot
related offense should be sent to jail. concentrate on long, unbroken sections of prose.
(B) Driver re-education offers the best chance of (D) Children judge books primarily on the basis of
making drivers with a large number of demerit graphic design.
points responsible drivers. (E) The lack of substance of a children’s book is
(C) Driver re-education is not harsh enough unlikely to be important to its popularity.
punishment for anyone convicted of a
serious driving-related offense who has also 94. The nature of English literature reflects the rich and
accumulated a large number of demerit points. diverse vocabulary of the English language, which
(D) Driver re-education should not be recommended resulted from the dual influence of the Anglo-Saxon
for those who have committed no serious and, later, French languages. The French language,
driving-related offenses. though, is a direct descendant of Latin, with few traces
(E) Drivers with a large number of demerit points but of the Celtic language spoken by the pre-Roman
no conviction for a serious driving-related inhabitants of the area: the hallmark of French literature
offense should receive driver re-education rather is its simplicity and clarity.
than jail. Which one of the following can be most reasonably
inferred from the information above?
92. Critic: Most chorale preludes were written for the
organ, and most great chorale preludes written for (A) The origin of English played a role in shaping
the organ were written by J. S. Bach. One of English literature.
Bach’s chorale preludes dramatizes one hymn’s (B) The vocabulary of the Anglo-Saxon language was
D perspective on the year’s end. This prelude is richer than that of the French language.
agonizing and fixed on the passing of the old (C) The vocabulary of English is larger than the
year, with its dashed hopes and lost opportunities. vocabulary of French.
It does not necessarily reveal Bach’s own attitude (D) Simple and clear literature cannot be written in a
toward the change of the year, but does reflect the language with a rich and diverse vocabulary.
tone of the hymn’s text. People often think that (E) English literature and French literature have had
artists create in order to express their own little influence on one another.
feelings. Some artists do. Master artists never do,
and Bach was a master artist.
If the critic’s statements are true, then on the basis of
them which one of the following CANNOT be true?
(A) Bach believed that the close of the year was not a
time for optimism and joyous celebration.
(B) In composing music about a particular subject,
Bach did not write the music in order to express
his own attitude toward the subject.
(C) In compositions other than chorale preludes, Bach
wrote music in order to express his feelings
toward various subjects.
(D) Most of Bach’s chorale preludes were written for
instruments other than the organ.
(E) Most of the great chorale preludes were written
for instruments other than the organ.
97. Special kinds of cotton that grow fibers of green or
95. Ideally, scientific laws should display the virtues of brown have been around since the 1930s but only
precision and generality, as do the laws of physics. recently became commercially feasible when a long-
However, because of the nature of their subject matter, fibered variety that can be spun by machine was finally
laws of social science often have to use terms that are bred. Since the cotton need not be dyed, processing
imprecise: for example, one knows only vaguely what is plants avoid the expense of dyeing and the ecological
meant by “republicanism” or “class.” As for generality, hazards of getting rid of leftover dye and by-products.
laws that apply only in certain social systems are
typically the only ones possible for the social sciences. Which one of the following can be properly inferred
from the passage?
Which one of the following statements is most strongly
supported by the information above? (A) It is ecologically safer to process long-fibered
cotton than short-fibered cotton.
(A) All else being equal, a precise, general scientific (B) Green and brown cottons that can be spun only
law is to be preferred over one that is not by hand are not commercially viable.
general. (C) Hand-spun cotton is more ecologically safe than
(B) The social sciences would benefit if they machine-spun cotton.
redirected their focus to the subject matter of the (D) Short-fibered regular cottons are economically
physical sciences. competitive with synthetic fabrics.
(C) Terms such as ‘‘class” should be more precisely (E) Garments made of green and brown cottons are
formulated by social scientists. less expensive than garments made of regular
(D) Social scientists should make an effort to cotton.
construct more laws that apply to all societies.
(E) The laws of social science are invariably not truly 98. Researchers studying artificial sweeteners have long
scientific. claimed that the perception of sweetness is determined
by the activation of a single type of receptor on the
96. In all mammalian species, the period of a young tongue called a sweetness receptor. They have also
mammal’s life in which it is most frequently playful claimed that any given individual molecule of substance
coincides with the period of most rapid growth of the can activate at most one sweetness receptor and that the
neural connections in the mammal’s brain that give fewer molecules that are required to activate a receptor,
rise to various complex patterns of movement, posture, the sweeter that substance will be perceived to be. Now,
and social response. Indeed, the neural connections the researchers claim to have discovered a substance
created by frequent play during this period later become of which only one molecule is needed to activate any
indispensable for the mammal’s survival and well-being sweetness receptor.
as an adult.
Which one of the following conclusions is most strongly
The statements above, if true, serve LEAST well as supported by the researchers’ claims, if all of those
evidence for which one of the following? claims are true?
(A) Young mammals of species that are preyed on by (A) The more sweetness receptors a person has on
other animals are likely to engage in forms of his or her tongue, the more likely it is that that
sudden mock flight, bolting away from locations person will find sweet sensations pleasurable.
where no predators are to be found. (B) In sufficient quantity, the molecules of any
(B) The young of nonmammalian species such as substance can activate a sweetness receptor.
fish, reptiles, and birds do not normally engage (C) No substance will be found that is perceived to be
in playful behavior that serves the function sweeter than the substance the researchers have
served by play in the development of young discovered.
mammals. (D) A substance that does not activate a sweetness
(C) Adult mammals are more likely to engage in receptor will activate a taste receptor of another
interactive play with their young if they engaged type.
in similar forms of play when they themselves (E) The more molecules of a substance that are
were young. required to activate a single sweetness receptor,
(D) Mammals that cannot engage in certain common the more bitter that substance will be perceived
forms of play when young are likely to show to be.
certain deficits that limit their subsequent
success as adults.
(E) Young mammals of predatory species tend to
practice in their play inoffensive versions of
motions and actions that are useful in finding
and catching prey when these mammals become
adults.
101. Until about 400 million years ago, fishes—the first
99. Chronic back pain is usually caused by a herniated or
true swimmers—were jawless. Their feeding methods
degenerated spinal disk. In most cases the disk will have
were limited to either sucking in surface plankton
been damaged years before chronic pain develops, and
or sucking in food particles from bottom mud. With
in fact an estimated one in five people over the age of 30
the development of biting jaws, however, the life of
has a herniated or degenerated disk that shows no
fishes changed dramatically, since jaws allowed them
chronic symptoms. If chronic pain later develops in such
actively to pursue prey, to seize it in their jaws, and to
a case, it is generally brought about by a deterioration of
manipulate it between their teeth. The jawed fishes then
the abdominal and spinal muscles caused by insufficient
developed along two main lines: one retained cartilage
exercise.
for its skeletons, for example, sharks and rays; the other
The statements above, if true, most strongly support adopted bone as its principal skeletal material.
which one of the following? From the latter group evolved the most abundant and
diverse of all of today’s vertebrate groups, the “teleosts,”
(A) Four out of five people over the age of 30 can be
some 21,000 species, which vary from barracudas to
sure they will never develop chronic back pain.
sea horses.
(B) People who exercise their abdominal and spinal
muscles regularly are sure to be free from If all of the statements in the passage are true, which
chronic back pain. one of the following must also be true?
(C) Patients rarely suffer even mild and fleeting back
(A) Fish are the primary prey of all jawed fishes.
pain at the time that a spinal disk first becomes
(B) The jawless fishes did not prey upon other fish.
herniated or degenerated.
(C) Teleosts do not feed upon particles found in
(D) Doctors can accurately predict which people who
bottom mud.
do not have chronic back pain will develop it in
(D) Jawless fishes did not have cartilage as their
the future.
skeletal material.
(E) There is a strategy that can be effective in
(E) Jawless fishes became extinct approximately 400
delaying or preventing the onset of pain from a
million years ago.
currently asymptomatic herniated or degenerated
spinal disk.
102. Some people take their moral cues from governmental
codes of law; for them, it is inconceivable that
100. Mature white pines intercept almost all the sunlight something that is legally permissible could be immoral.
that shines on them. They leave a deep litter that dries
Those whose view is described above hold inconsistent
readily, and they grow to prodigious height so that, even D beliefs if they also believe that
when there are large gaps in a stand of such trees, little
light reaches the forest floor. For this reason, white pines (A) law does not cover all circumstances in which
cannot regenerate in their own shade. Thus, when in one person morally wrongs another
a dense forest a stand of trees consists of nothing but (B) a legally impermissible action is never morally
mature white pines, it is a fair bet that . excusable
Which one of the following most logically concludes the (C) governmental officials sometimes behave illegally
argument? (D) the moral consensus of a society is expressed in
its laws
(A) the ages of the trees in the stand do not differ (E) some governmental regulations are so detailed
from each other by much more than the length that they are burdensome to the economy
of time it takes a white pine to grow to maturity
(B) the land on which the stand is now growing had
been cleared of all trees at the time when the
first of the white pines started growing
(C) competition among the trees in the stand for
sunlight will soon result in some trees’ dying
and the stand thus becoming thinner
(D) other species of trees will soon begin to colonize
the stand, eventually replacing all of the white
pines
(E) any differences in the heights of the trees in the
stand are attributable solely to differences in the
ages of the trees
105. Editorialist: News media rarely cover local politics
103. Proponents of the electric car maintain that when the
thoroughly, and local political business is usually
technical problems associated with its battery design
conducted secretively. These factors each tend
are solved, such cars will be widely used and, because
to isolate local politicians from their electorates.
they are emission-free, will result in an abatement of the
This has the effect of reducing the chance that any
environmental degradation caused by auto emissions.
particular act of resident participation will elicit a
But unless we dam more rivers, the electricity to positive official response, which in turn
charge these batteries will come from nuclear or coal- discourages resident participation in local politics.
fired power plants. Each of these three power sources
produces considerable environmental damage. Thus, the Which one of the following is most strongly supported
electric car . by the editorialist’s statements?
Which one of the following most logically completes the (A) Particular acts of resident participation would be
argument? likely to elicit a positive response from local
politicians if those politicians were less isolated
(A) will have worse environmental consequences than from their electorate.
its proponents may believe
(B) Local political business should be conducted
(B) will probably remain less popular than other
less secretively because this would avoid
types of cars
discouraging resident participation in local
(C) requires that purely technical problems be solved politics.
before it can succeed
(C) The most important factor influencing a resident’s
(D) will increase the total level of emissions rather
decision as to whether to participate in local
than reduce it
politics is the chance that the participation will
(E) will not produce a net reduction in environmental elicit a positive official response.
degradation
(D) More-frequent thorough coverage of local
politics would reduce at least one source of
104. Modern science is built on the process of posing discouragement from resident participation in
hypotheses and testing them against observations— local politics.
in essence, attempting to show that the hypotheses
(E) If resident participation in local politics were not
are incorrect. Nothing brings more recognition than
discouraged, this would cause local politicians to
overthrowing conventional wisdom. It is accordingly
be less isolated from their electorate.
unsurprising that some scientists are skeptical of the
widely accepted predictions of global warming. What is
106. Philosopher: Nations are not literally persons; they have
instead remarkable is that with hundreds of researchers
no thoughts or feelings, and, literally speaking,
striving to make breakthroughs in climatology, very few
they perform no actions. Thus, they have no
find evidence that global warming is unlikely.
moral rights or responsibilities. But no nation
The information above provides the most support for can survive unless many of its citizens attribute
which one of the following statements? such rights and responsibilities to it, for nothing
else could prompt people to make the sacrifices
(A) Most scientists who are reluctant to accept the
national citizenship demands. Obviously, then, a
global warming hypothesis are not acting in
nation .
accordance with the accepted standards of
scientific debate. Which one of the following most logically completes the
(B) Most researchers in climatology have substantial philosopher’s argument?
motive to find evidence that would discredit the
(A) cannot continue to exist unless something other
global warming hypothesis.
than the false belief that the nation has moral
(C) There is evidence that conclusively shows that the rights motivates its citizens to make sacrifices
global warming hypothesis is true.
(B) cannot survive unless many of its citizens have
(D) Scientists who are skeptical about global warming
some beliefs that are literally false
have not offered any alternative hypotheses to
(C) can never be a target of moral praise or blame
explain climatological data.
(D) is not worth the sacrifices that its citizens make
(E) Research in global warming is primarily driven
on its behalf
by a desire for recognition in the scientific
(E) should always be thought of in metaphorical
community.
rather than literal terms
109. In older commercial airplanes, the design of the control
107. If the price it pays for coffee beans continues to
panel allows any changes in flight controls made by one
increase, the Coffee Shoppe will have to increase its
prices. In that case, either the Coffee Shoppe will begin member of the flight crew to be immediately viewed
selling noncoffee products or its coffee sales will by the other crew members. In recently manufactured
decrease. But selling noncoffee products will decrease aircraft, however, a crew member’s flight control
the Coffee Shoppe’s overall profitability. Moreover, the changes are harder to observe, thereby eliminating a
Coffee Shoppe can avoid a decrease in overall routine means for performing valuable cross-checks.
profitability only if its coffee sales do not decrease. As a result, the flight crews operating recently
manufactured airplanes must inform each other verbally
Which one of the following statements follows logically about flight control changes much more frequently.
from the statements above?
The statements above, if true, most strongly support
(A) If the Coffee Shoppe’s overall profitability which one of the following?
decreases, the price it pays for coffee beans will
have continued to increase. (A) How frequently an airplane’s flight crew
(B) If the Coffee Shoppe’s overall profitability members will inform each other verbally about
decreases, either it will have begun selling flight control changes depends in large part on
noncoffee products or its coffee sales will have how long it takes to perform those changes.
decreased. (B) In recently manufactured aircraft, the most
(C) The Coffee Shoppe’s overall profitability will valuable means available for performing cross-
decrease if the price it pays for coffee beans checks involves frequent verbal exchanges of
continues to increase. information among the flight crew members.
(D) The price it pays for coffee beans cannot decrease (C) In older commercial airplanes, in contrast to
without the Coffee Shoppe’s overall profitability recently manufactured airplanes, flight crew
also decreasing. members have no need to exchange information
(E) Either the price it pays for coffee beans will verbally about flight control changes.
continue to increase or the Coffee Shoppe’s (D) The flight crew members operating a recently
coffee sales will increase. manufactured airplane cannot observe the flight
control changes made by other crew members by
108. Most successful entrepreneurs work at least 18 hours a viewing the control panel.
day, and no one who works at least 18 hours a day has (E) How often flight crew members must share
time for leisure activities. But all happy entrepreneurs information verbally about flight control
have time for leisure activities. changes depends in part on what other means
If the statements above are true, each of the following for performing cross-checks are available to the
could be true EXCEPT: crew.

(A) Anyone who has no time for leisure activities


works at least 18 hours a day.
(B) Some entrepreneurs who work at least 18 hours a
day are successful.
(C) Some happy entrepreneurs are successful.
(D) Some entrepreneurs who work at least 18 hours a
day are happy.
(E) Some successful entrepreneurs work less than 18
hours a day.
112. The purpose of a general theory of art is to explain every
110. Forester: The great majority of the forests remaining in aesthetic feature that is found in any of the arts.
the world are only sickly fragments of the fully Premodern general theories of art, however, focused
functioning ecosystems they once were. These primarily on painting and sculpture. Every premodern
fragmented forest ecosystems have typically lost general theory of art, even those that succeed as theories
their ability to sustain themselves in the long of painting and sculpture, fails to explain some aesthetic
term, yet they include the last refuges for some of feature of music.
the world’s most endangered species. To maintain The statements above, if true, most strongly support
its full complement of plant and animal species, a which one of the following?
fragmented forest requires regular interventions
by resource managers. (A) Any general theory of art that explains the
aesthetic features of painting also explains those
The forester’s statements, if true, most strongly support of sculpture.
which one of the following? (B) A general theory of art that explains every
(A) Most of the world’s forests will lose at least aesthetic feature of music will achieve its
some of their plant or animal species if no one purpose.
intervenes. (C) Any theory of art that focuses primarily on
(B) Unless resource managers regularly intervene in sculpture and painting cannot explain every
most of the world’s remaining forests, many of aesthetic feature of music.
the world’s most endangered species will not (D) No premodern general theory of art achieves its
survive. purpose unless music is not art.
(C) A fragmented forest ecosystem cannot sustain (E) No premodern general theory of art explains any
itself in the long term if it loses any of its plant aesthetic features of music that are not shared
or animal species. with painting and sculpture.
(D) A complete, fully functioning forest ecosystem
can always maintain its full complement of plant 113. All social systems are based upon a division of
and animal species even without interventions economic roles. The values of a social system are
by resource managers. embodied in the prestige accorded persons who fill
(E) At present, resource managers intervene regularly various economic roles. It is therefore unsurprising that,
in only some of the world’s fragmented forest for any social system, the introduction of labor-saving
ecosystems. technology that makes certain economic roles obsolete
will tend to undermine the values in that social system.
111. Many successful graphic designers began their careers Which one of the following can most reasonably be
after years of formal training, although a significant concluded on the basis of the information above?
number learned their trade more informally on the job.
But no designer ever became successful who ignored the (A) Social systems will have unchanging values
wishes of a client. if they are shielded from technological
advancement.
If all of the statements above are true, which one of the
(B) No type of technology will fail to undermine the
following must also be true?
values in a social system.
(A) All graphic designers who are unsuccessful have (C) A social system whose values are not susceptible
ignored the wishes of a client. to change would not be one in which technology
(B) Not all formally trained graphic designers ignore can eliminate economic roles.
clients’ wishes. (D) A technologically advanced society will place
(C) The more attentive a graphic designer is to a little value on the prestige associated with an
client’s wishes, the more likely the designer is to economic role.
be successful. (E) A technological innovation that is implemented
(D) No graphic designers who learn their trade on the in a social system foreign to the one in which
job will ignore clients’ wishes. it was developed will tend to undermine the
(E) The most successful graphic designers learn their foreign social system.
trade on the job.
116. To be horrific, a monster must be threatening. Whether
114. Multiple sclerosis is an autoimmune disease: white or not it presents psychological, moral or social dangers,
blood cells attack the myelin sheath that protects nerve or triggers enduring infantile fears, if a monster is
fibers in the spinal cord and brain. Medical science now physically dangerous then it is threatening. In fact, even
has a drug that can be used to successfully treat multiple a physically benign monster is horrific if it inspires
sclerosis, but the path that led medical researchers to
this drug was hardly straightforward. Initially, some revulsion.
scientists believed attacks characteristic of multiple Which one of the following logically follows from the
sclerosis might be triggered by chronic viral infections. statements above?
So, in 1984 they began testing gamma interferon, one of
the body’s own antiviral weapons. To their horror, all the (A) Any horror-story monster that is threatening is
multiple sclerosis patients tested became dramatically also horrific.
worse. The false step proved to be instructive however. (B) A monster that is psychologically dangerous, but
that does not inspire revulsion, is not horrific.
Which one of the following is LEAST compatible with (C) If a monster triggers infantile fears but is not
the results of the gamma interferon experiment? physically dangerous, then it is not horrific.
(A) Gamma interferon stops white blood cells from (D) If a monster is both horrific and psychologically
producing myelin-destroying compounds. threatening, then it does not inspire revulsion.
(B) Administering gamma interferon to those without (E) All monsters that are not physically dangerous,
multiple sclerosis causes an increase in the but that are psychologically dangerous and
number of white blood cells. inspire revulsion, are threatening.
(C) Medical researchers have discovered that the
gamma interferon level in the cerebrospinal 117. Though many insects die soon after reproducing for
fluid skyrockets just before and during multiple the first time, some may live for years after the survival
sclerosis attacks. of the next generation has been secured. Among the
(D) It has now been established that most multiple latter are some insects that work for the benefit of the
sclerosis sufferers do not have chronic viral ecosystem—for example, bees.
infections. Which one of the following can be properly inferred
(E) The drug now used to treat multiple sclerosis from the information above?
is known to inhibit the activity of gamma
interferon. (A) Survival of the species, rather than of the
individual, is the goal of most insect
115. People should avoid taking the antacid calcium populations.
carbonate in doses larger than half a gram, for despite its (B) Insects that do not play a vital role in the
capacity to neutralize stomach acids, calcium carbonate ecosystem are more likely to die after
can increase the calcium level in the blood and thus reproducing for the first time.
impair kidney function. Moreover, just half a gram of (C) Most bees live well beyond the onset of the
it can stimulate the production of gastrin, a stomach generation that follows them.
hormone that triggers acid secretion. (D) Those bees that reproduce do not always die soon
after reproducing for the first time.
Which one of the following is most strongly supported (E) Most insects are hatched self-sufficient and do
by the information above? not need to be cared for by adult insects.
(A) Cessation of gastrin production is a more
effective method of controlling excess stomach
acid than is direct neutralization of stomach
acid.
(B) People who avoid taking more than half a gram
of calcium carbonate are less likely than average
to suffer from impaired kidney function.
(C) Doses of calcium carbonate smaller than half a
gram can reduce stomach acid more effectively
than much larger doses do.
(D) Half a gram of calcium carbonate can causally
contribute to both the secretion and the
neutralization of stomach acids.
(E) Impaired kidney function may increase the level
of calcium in the blood.
120. Researchers have discovered that caffeine can be as
118. Light is registered in the retina when photons hit
physically addictive as other psychoactive substances.
molecules of the pigment rhodopsin and change the
Some people find that they become unusually depressed,
molecules’ shape. Even when they have not been struck
drowsy, or even irritable if they do not have their
by photons of light, rhodopsin molecules sometimes
customary dose of caffeine. This is significant because
change shape because of normal molecular motion,
as many people consume caffeine as consume any one of
thereby introducing error into the visual system. The
the other addictive psychoactive substances.
amount of this molecular motion is directly proportional
to the temperature of the retina. Which one of the following can be logically concluded
from the information above?
Which one of the following conclusions is most strongly
supported by the information above? (A) There is no psychoactive substance to which
more people are physically addicted than are
(A) The temperature of an animal’s retina depends on addicted to caffeine.
the amount of light the retina is absorbing.
(B) A physical addiction to a particular psychoactive
(B) The visual systems of animals whose body
substance will typically give rise to diverse
temperature matches that of their surroundings
psychological symptoms.
are more error-prone in hot surroundings than in
(C) Not all substances to which people can become
cold ones.
physically addicted are psychoactive.
(C) As the temperature of the retina rises, rhodopsin
(D) If one is physically addicted to a psychoactive
molecules react more slowly to being struck by
substance, one will become unusually depressed
photons.
when one is no longer ingesting that substance.
(D) Rhodopsin molecules are more sensitive to
(E) If alcohol is a physically addictive psychoactive
photons in animals whose retinas have large
substance, there are not more people who
surface areas than in animals whose retinas have
consume alcohol than consume caffeine.
small surface areas.
(E) Molecules of rhodopsin are the only pigment
molecules that occur naturally in the retina.

119. Viruses can have beneficial effects. For example, some


kill more-complex microorganisms, some of which
are deadly to humans. But viruses have such simple
structures that replacing just a few of a beneficial virus’s
several million atoms can make it deadly to humans.
Clearly, since alterations of greater complexity than this
are commonly produced by random mutations, any virus
could easily become dangerous to humans.
If the statements above are true, then each of the
following statements could also be true EXCEPT:
(A) Random mutation makes some deadly viruses
beneficial to humans.
(B) Some organisms of greater complexity than
viruses are no more likely than viruses to
undergo significant alterations through random
mutation.
(C) Some microorganisms that are more complex
than viruses are beneficial to humans.
(D) Some viruses that fail to kill other viruses that are
deadly to humans are nevertheless beneficial to
humans.
(E) No virus that is deadly to organisms of greater
complexity than itself is beneficial to humans.
123. Quasars—celestial objects so far away that their light
121. Commentator: A political constitution that provides the
takes at least 500 million years to reach Earth—have
framework for the laws of a nation must be
interpreted to apply to new situations not been seen since 1963. For anything that far away to
envisioned by its authors. Although these appear from Earth the way quasars do, it would have to
interpretations express the moral and political burn steadily at a rate that produces more light than 90
beliefs of the interpreters, they are usually billion suns would produce. But nothing that burns at a
portrayed as embodying the intentions of the rate that produces that much light could exist for more
authors. This fiction is vital because without it the than about 100 million years.
illusion, so necessary for political stability, that If the statements above are true, which one of the
laws are the bequest of a long tradition rather following must also be true on the basis of them?
than the preferences of contemporary politicians
would vanish. (A) Instruments in use before 1963 were not sensitive
enough to permit quasars to be seen.
Which one of the following is most strongly supported (B) Light from quasars first began reaching Earth
by the commentator’s statements, if they are true? in 1963.
(A) If the people of a nation do not believe that the (C) Anything that from Earth appears as bright as a
laws under which they live express the intentions quasar does must produce more light than would
of their political leaders, that nation will become be produced by 90 billion suns.
more politically unstable. (D) Nothing that is as far from Earth as quasars
(B) Political instability will increase if the people of a are can continue to exist for more than about
nation cease to believe that their constitution is 100 million years.
being interpreted consistently with the intentions (E) No quasar that has ever been seen from Earth
of its authors. exists any longer.
(C) Political instability will ensue if people come to
believe there is a divergence between the beliefs 124. Sharks have a higher ratio of cartilage mass to body
of the authors of their constitution and those of mass than any other organism. They also have a greater
their present political leaders. resistance to cancer than any other organism. Shark
(D) A written constitution preserves the illusion that cartilage contains a substance that inhibits tumor growth
laws are the bequest of a long tradition rather by stopping the development of a new blood network. In
than the creations of modern politicians. the past 20 years, none of the responses among terminal
(E) The perceived lack of a long legal tradition in cancer patients to various therapeutic measures has
a nation makes the political stability of that been more positive than the response among those who
nation dependent upon the fiction that its present consumed shark cartilage.
political leaders share the intentions of the If the claims made above are true, then each of the
authors of the constitution of that nation. following could be true EXCEPT:

122. Some planning committee members—those representing (A) No organism resists cancer better than sharks
the construction industry—have significant financial does, but some resist cancer as well as sharks.
interests in the committee’s decisions. No one who is on (B) The organism most susceptible to cancer has
the planning committee lives in the suburbs, although a higher percentage of cartilage than some
many of them work there. organisms that are less susceptible to cancer.
(C) The substance in shark cartilage that inhibits
If the statements above are true, which one of the
tumor growth is found in most organisms.
following must also be true?
(D) In the past 20 years many terminal cancer patients
(A) No persons with significant financial interests in have improved dramatically following many
the planning committee’s decisions are not in the sorts of therapy.
construction industry. (E) Some organisms have immune systems more
(B) No person who has significant financial interest efficient than a shark’s immune system.
in the planning committee’s decisions lives in
the suburbs.
(C) Some persons with significant financial interests
in the planning committee’s decisions work in
the suburbs.
(D) Some planning committee members who
represent the construction industry do not work
in the suburbs.
(E) Some persons with significant financial interests
in the planning committee’s decisions do not live
in the suburbs.
127. Ethicist: A person who treats others well is more
125. If the economy is weak, then prices remain constant worthy of praise if this treatment is at least
although unemployment rises. But unemployment rises partially motivated by feelings of compassion
only if investment decreases. Fortunately, investment is
than if it is entirely motivated by cold and
not decreasing.
dispassionate concern for moral obligation. This
If the statements above are true, then which one of the is so despite the fact that a person can choose to
following must be false? do what is morally right but cannot choose to
have feelings.
(A) Either the economy is weak or investment is
decreasing. If the ethicist’s statements are true, then each of the
(B) If unemployment rises, the prices remain following could be true EXCEPT:
constant.
(A) Only actions that are at least partially the result of
(C) The economy is weak only if investment a person’s feelings should be used in measuring
decreases. the praiseworthiness of that person.
(D) Either the economy is weak or prices are (B) If a person feels compassion toward the people
remaining constant.
affected by that person’s actions, yet these
(E) Either unemployment is rising or the economy is actions diminish the welfare of those people,
not weak. that person does not deserve praise.
(C) Only what is subject to a person’s choice should
126. Some environmentalists question the prudence of be used in measuring the praiseworthiness of
exploiting features of the environment, arguing that there that person.
are no economic benefits to be gained from forests,
(D) Someone who acts without feelings of
mountains, or wetlands that no longer exist. Many
compassion toward those affected by the actions
environmentalists claim that because nature has intrinsic
is worthy of praise if those actions enhance the
value it would be wrong to destroy such features of the
welfare of the people affected.
environment, even if the economic costs of doing so
(E) If someone wants to have compassion toward
were outweighed by the economic costs of not doing so.
others but does not, that person is worthy of
Which one of the following can be logically inferred praise.
from the passage?
128. A poem is any work of art that exploits some of the
(A) It is economically imprudent to exploit features musical characteristics of language, such as meter,
of the environment. rhythm, euphony, and rhyme. A novel, though it may be
(B) Some environmentalists appeal to a noneconomic a work of art in language, does not usually exploit the
justification in questioning the defensibility of musical characteristics of language. A symphony, though
exploiting features of the environment. it may be a work of art that exploits the musical
(C) Most environmentalists appeal to economic characteristics of sounds, rarely involves language. A
reasons in questioning the defensibility of limerick, though it may exploit some musical
exploiting features of the environment. characteristics of language, is not, strictly speaking, art.
(D) Many environmentalists provide only a
noneconomic justification in questioning The statements above, if true, most strongly support
the defensibility of exploiting features of which one of the following?
the environment. (A) If a creation is neither a poem, nor a novel, nor a
(E) Even if there is no economic reason for protecting symphony, then it is not a work of art.
the environment, there is a sound noneconomic (B) An example of so-called blank verse, which does
justification for doing so. not rhyme, is not really a poem.
(C) If a novel exploits meter and rhyme while
standing as a work of art, then it is both a novel
and a poem.
(D) Limericks constitute a non-artistic type of poetry.
(E) If a symphony does not exploit the musical
characteristics of sound, then it is not a work
of art.
132. Whenever she considers voting in an election to select
129. Pollen and other allergens can cause cells in the nose one candidate for a position and there is at least one
to release histamine, a chemical that inflames nasal issue important to her, Kay uses the following principle
tissue and causes runny nose, congestion, and sneezing. in choosing which course of action to take: it is
Antihistamines minimize these allergy symptoms acceptable for me to vote for a candidate whose opinions
by blocking the action of histamine. In addition, differ from mine on at least one issue important to me
antihistamines have other effects, including drowsiness. whenever I disagree with each of the other candidates on
However, histamine plays no role in the processes by even more such issues; it is otherwise unacceptable to
which colds produce their symptoms. vote for that candidate. In the upcoming mayoral
If the statements above are true, which one of the election, the three candidates are Legrand, Medina, and
following must also be true? Norton. There is only one issue important to Kay, and
only Medina shares her opinion on that issue.
(A) Pollen and other allergens do not cause colds.
(B) Colds are more difficult to treat than allergies. If the statements in the passage are true, which one of
(C) Antihistamines, when taken alone, are ineffective the following must also be true about Kay’s course of
against congestion caused by colds. action in any election to select one candidate for a
(D) The sleeplessness that sometimes accompanies position?
allergies can be effectively treated with (A) If there are no issues important to her, it is
antihistamines. unacceptable for her to vote for any candidate in
(E) Any effect antihistamines may have in reducing the election.
cold symptoms does not result from blocking the (B) If she agrees with each of the candidates on most
action of histamine. of the issues important to her, it is unacceptable
for her to vote for any candidate in the election.
130. Anthropologist: Violence is an extreme form of (C) If she agrees with a particular candidate on only
aggression, and is distinct from the self- one issue important to her, it is unacceptable for
expression sufficient for survival under normal her to vote for that candidate.
conditions. Human beings in certain situations (D) If she disagrees with each of the candidates
react to unpleasant stimuli with violence—but on exactly three issues important to her, it is
only because they are conditioned by their culture unacceptable for her to vote for any candidate in
to react in this manner. the election.
Each of the following can be logically inferred from the (E) If there are more issues important to her on which
anthropologist’s statements EXCEPT: she disagrees with a particular candidate than
there are such issues on which she agrees with
(A) Not all aggression is violent. that candidate, it is unacceptable for her to vote
(B) The self-expression required for survival is for that candidate.
generally non-aggressive.
(C) Some behaviors are influenced by the cultures in
which human beings live.
(D) In normal circumstances, human beings can
survive by responding nonviolently.
(E) Violent behavior is a product of one’s cultural
environment.

131. Only poetry cannot be translated well, and therefore it is


poets who preserve languages, for we would not bother
to learn a language if we could get everything written in
it from translation. So, since we cannot witness the
beauty of poetry except in the language in which it is
composed, we have motivation to learn the language.
The information above provides the LEAST support for
which one of the following?
(A) All nonpoetic literature can be translated well.
(B) One purpose of writing poetry is to preserve the
language in which it is written.
(C) Some translations do not capture all that was
expressed in the original language.
(D) The beauty of poetry is not immediately
accessible to people who do not understand the
language in which the poetry was written.
(E) Perfect translation from one language to another
is sometimes impossible.
135. At night, a flock of crows will generally perch close
133. Numismatist: In medieval Spain, most gold coins were
together in a small place—often a piece of wooded
minted from gold mined in West Africa, in the land—called a roost. Each morning, the crows leave the
area that is now Senegal. The gold mined in this
roost and fan out in small groups to hunt and scavenge
region was the purest known. Its gold content of
the surrounding area. For most flocks, the crows’
92 percent allowed coins to be minted without hunting extends as far as 100 to 130 kilometers (60 to 80
refining the gold, and indeed coins minted from miles) from the roost. Normally, a flock will continue to
this source of gold can be recognized because occupy the same roost for several consecutive years, and
they have that gold content. The mints could when it abandons a roost site for a new one, the new
refine gold and produced other kinds of coins that roost is usually less than eight kilometers (five miles)
had much purer gold content, but the Senegalese away:
gold was never refined.
Of the following claims, which one can most justifiably
Which one of the following inferences about gold coins be rejected on the basis of the statements above?
minted in medieval Spain is most strongly supported by
the information the numismatist gives? (A) Crows will abandon their roost site only in
response to increases in the population of the
(A) Coins minted from Senegalese gold all contained flock.
the same weight, as well as the same proportion,
(B) When there is a shortage of food in the area in
of gold.
which a flock of crows normally hunts and
(B) The source of some refined gold from which scavenges, some members of the flock will
coins were minted was unrefined gold with a begin to hunt and scavenge outside that area.
gold content of less than 92 percent.
(C) Most of the hunting and scavenging that crows do
(C) Two coins could have the same monetary value
occurs more than eight kilometers (five miles)
even though they differed from each other in the
from their roost.
percentage of gold they contained.
(D) Once a flock of crows has settled on a new
(D) No gold coins were minted that had a gold
roost site, it is extremely difficult to force it to
content of less than 92 percent.
abandon that site for another.
(E) The only unrefined gold from which coins could
(E) When a flock of crows moves to a new roost site,
be minted was Senegalese gold.
it generally does so because the area in which it
has hunted and scavenged has been depleted of
134. If this parking policy is unpopular with the faculty, then food sources.
we should modify it. If it is unpopular among students,
we should adopt a new policy. And, it is bound to be
unpopular either with the faculty or among students. 136. Essayist: Wisdom and intelligence are desirable
qualities. However, being intelligent does not
If the statements above are true, which one of the imply that one is wise, nor does being wise imply
following must also be true? that one is intelligent. In my own experience, the
people I meet have one or the other of these
(A) We should attempt to popularize this parking
qualities but not both.
policy among either the faculty or students.
(B) We should modify this parking policy only if this If the essayist’s statements are true, then each of the
will not reduce its popularity among students. following could be true EXCEPT:
(C) We should modify this parking policy if
(A) Most people are neither intelligent or wise.
modification will not reduce its popularity with
(B) Most people are both intelligent and wise.
the faculty.
(C) No one is both wise and intelligent.
(D) If the parking policy is popular among students, (D) No one is either wise or intelligent.
then we should adopt a new policy.
(E) Many people are intelligent and yet lack wisdom.
(E) If this parking policy is popular with the faculty,
then we should adopt a new policy.
139. In the past, the railroads in Ostronia were run as regional
137. For the condor to survive in the wild, its breeding
monopolies and operated with little regard for what
population must be greatly increased. But because only
a few eggs can be produced by a breeding pair over their customers wanted. In recent years, with improvements to
lifetime, any significant increase in the number of birds the Ostronian national highway network, the railroad
depends upon most of these eggs hatching, which is companies have faced heavy competition from long-
extremely unlikely in the wild due to environmental distance trucking companies. But because of government
dangers. One possible way to eliminate the effects of subsidies that have permitted Ostronain railroad
these factors is to breed the birds in captivity and companies to operate even while incurring substantial
subsequently return them to the wild. losses, the companies continue to disregard customers’
needs and desires.
Which one of the following is most strongly supported
by the information above? If the statements above are true, which one of the
following must also be true on the basis of them?
(A) The condor as a species will eventually become
extinct in the wild. (A) If the government of Ostronia ceases to subsidize
(B) The best way to save the condor from extinction railroad companies, few of those companies will
is to breed it in captivity. continue to operate.
(C) It is almost impossible to eliminate all the (B) Few companies in Ostronia that have received
environmental threats to the eggs of condors. subsidies from the government have taken
(D) If more condor eggs do not hatch, the condor as a the needs and desires of their customers into
species will not survive in the wild. account.
(E) The most feasible way to save the condor from (C) Without government subsidies, railroad
extinction is to increase egg production. companies in Ostronia would have to increase
the prices they charge their customers.
138. Biologist: Some speculate that the unusually high (D) The transportation system in Ostronia is no more
frequency of small goats found in island efficient today than in was in the past.
populations is a response to evolutionary pressure (E) In recent years, some companies in Ostronia that
to increase the number of goats so as to ensure a have had little regard for the desires of their
diverse gene pool. However, only the reproductive customers have nonetheless survived.
success of a trait influences its frequency in a
population. So, the only kind of evolutionary 140. Explanation must be distinguished from justification.
pressure that can reduce the average size of the Every human action potentially has an explanation; that
members of a goat population is that resulting is, with sufficient knowledge it would be possible to
from small goats achieving greater reproductive give an accurate description of the causes of that action.
than their larger cousins. An action is justified only when the person performing
the action has sufficient reasons for the action.
The biologist’s view, if true, provides the most support According to many psychologists, even when there is a
for which one of the following? justification for an action, that justification often forms
(A) The evolutionary pressure to ensure a diverse no part of the explanation. The general principle,
gene pool could have the effect of increasing the however, is that only an action whose justification, that
frequency of a gene for small size. is, the reasons for the action, forms an essential part of
(B) The unusual frequency of small goats in island its explanation is rational.
populations is not a result of the greater If the statements in the passage are correct, which one of
reproductive success small goats possess when the following can be properly concluded from them?
space is limited.
(C) Contrary to what some believe, large goats (A) When a human action is justified, that action has
achieve greater reproductive success than small no explanation.
goats even when space is limited. (B) If there are any reasons among the causes of an
(D) The evolutionary pressure to ensure a diverse action, then that action is rational.
gene pool does not have the effect of increasing (C) Some psychologists believe that the justification
the frequency of a gene for small size. for an action never forms an essential part of its
(E) A diverse gene pool cannot be achieved in a explanation.
goat population unless the average size of its (D) There are actions whose causes cannot be
members is reduced. discovered.
(E) If any human actions are rational, then reasons
must sometimes be causes of actions.
143. Nearly all mail that is correctly addressed arrives at its
141. Each December 31 in Country Q, a tally is made of the destination within two business days of being sent. In
country’s total available coal supplies—that is, the total fact, correctly addressed mail takes longer than this only
amount of coal that has been mined throughout the when it is damaged in transit. Overall, however, most
country but not consumed. In 1991 that amount was mail arrives three business days or more after being sent.
considerably lower than it had been in 1990.
Furthermore, Country Q has not imported or exported If the statements above are true, which one of the
coal since 1970. following must be true?

If the statements above are true, which one of the (A) A large proportion of the mail that is correctly
addressed is damaged in transit.
following must also be true on the basis of them?
(B) No incorrectly addressed mail arrives within two
(A) In Country Q, more coal was mined in 1990 than business days of being sent.
was mined in 1991. (C) Most mail that arrives within two business days
(B) In Country Q, the amount of coal consumed in of being sent is correctly addressed.
1991 was greater than the amount of coal mined (D) A large proportion of mail is incorrectly
in 1991. addressed.
(C) In Country Q, the amount of coal consumed (E) More mail arrives within two business days of
in 1990 was greater than the amount of coal being sent than arrives between two and three
consumed in 1991. business days after being sent.
(D) In Country Q, the amount of coal consumed
in 1991 was greater than the amount of coal 144. Scientists have recently discovered that, in doses
consumed in 1990. massive enough to kill cells, almost any chemical is
(E) In Country Q, more coal was consumed during carcinogenic because cell death causes rapid division
the first half of 1991 than was consumed during among surviving cells, promoting cancer-causing
the first half of 1990. mutations. A few chemicals are also carcinogenic
without causing cell death. Until now studies of the
142. Curator: The decision to restore the cloak of the central carcinogenicity of food additives have always involved
figure in Veronese’s painting from its present red administering to test animals doses of additives larger
to the green found underneath is fully justified. than the largest possible human exposure levels and
Reliable x-ray and chemical tests show that the massive enough to kill large numbers of cells in the
red pigment was applied after the painting had animals, and then observing the animals’ cancer rates.
been completed, and that the red paint was not
mixed in Veronese’s workshop. Hence it appears If the statements above are true, which one of the
likely that an artist other than Veronese tampered following conclusions is most strongly supported by
with Veronese’s painting after its completion. them?
(A) In the doses heretofore used in carcinogenicity
Art critic: But in a copy of Veronese’s painting made
studies of food additives, cell death often does
shortly after Veronese died, the cloak is red. It is
not occur in test animals.
highly unlikely that a copyist would have made
so major a change so soon after Veronese’s death. (B) Until now results of carcinogenicity studies
encouraged overestimation of the degree to
The art critic’s response to the curator would provide which some food additives are carcinogenic.
the strongest support for which one of the following (C) Truly carcinogenic chemicals usually cause some
conclusions? immediate cell death, even in small doses.
(A) The copy of Veronese’s painting that was made (D) Carcinogenic chemicals are usually absorbed in
soon after the painter’s death is indistinguishable small doses in the natural environment.
from the original. (E) Most of the food additives that are now banned
(B) No painting should be restored before the because of carcinogenicity could safely be used
painting is tested with technologically in human foods.
sophisticated equipment.
(C) The proposed restoration will fail to restore
Veronese’s painting to the appearance it had at
the end of the artist’s lifetime.
(D) The value of an artist’s work is not necessarily
compromised when that work is tampered with
by later artists.
(E) Veronese did not originally intend the central
figure’s cloak to be green.
146. Decision makers tend to have distinctive styles.
145. Unless the residents of Glen Hills band together, the
proposal to rezone that city will be approved. If it is, the One such style is for the decision maker to seek the
city will be able to build the water and sewer systems widest possible input from advisers and to explore
that developers need in order to construct apartment alternatives while making up his or her mind. In fact,
houses there. These buildings would attract new decision makers of this sort will often argue vigorously
residents, and the increased population would probably for a particular idea, emphasizing its strong points and
result in overcrowded schools and would certainly result downplaying its weaknesses, not because they actually
in roads so congested that new roads would be built. believe in the idea but because they want to see if their
Neither new roads nor additional schools could be built real reservations about it are idiosyncratic or are held
without substantial tax increases for the residents of independently by their advisers.
Glen Hills. Ultimately, this growth might even destroy Which one of the following is most strongly supported
the rural atmosphere that makes Glen Hills so attractive. by the statements above?
Which one of the following can be properly concluded
(A) If certain decision makers’ statements are quoted
from the passage?
accurately and at length, the content of the quote
(A) If the citizens of Glen Hills band together, could nonetheless be greatly at variance with the
developers will not build apartment houses. decision eventually made.
(B) If developers build apartment houses in Glen (B) Certain decision makers do not know which ideas
Hills, there will be substantial tax increases for they do not really believe in until after they have
the residents of Glen Hills. presented a variety of ideas to their advisers.
(C) If the rezoning proposal does not pass, the rural (C) If certain decision makers dismiss an idea out
atmosphere in Glen Hills will not be lost. of hand, it must be because its weaknesses are
(D) If developers do not build apartment houses in more pronounced than any strong points it may
Glen Hills, the taxes of the residents of Glen have.
Hills will not increase substantially. (D) Certain decision makers proceed in a way that
(E) If developers do not build apartment houses in makes it likely that they will frequently decide
Glen Hills, the schools of Glen Hills will not be in favor of ideas in which they do not believe.
overcrowded and roads will not be congested. (E) If certain decision makers’ advisers know the
actual beliefs of those they advise, those
advisers will give better advice than they would
if they did not know those beliefs.
47. A 95. A 143. D
48. D 96. B 144. B
1. D 49. B 97. B 145. B
2. B 50. * 98. C 146. A
3. D 51. B 99. E
4. C 52. C 100. A
5. C 53. B 101. B
6. B 54. A 102. A
7. E 55. D 103. A
8. D 56. B 104. B
9. E 57. B 105. D
10. E 58. C 106. B
11. C 59. E 107. C
12. E 60. * 108. D
13. C 61. E 109. E
14. B 62. D 110. A
15. C 63. C 111. B
16. E 64. A 112. D
17. A 65. E 113. C
18. A 66. E 114. A
19. A 67. B 115. D
20. E 68. C 116. E
21. D 69. D 117. D
22. B 70. D 118. B
23. A 71. D 119. E
24. C 72. A 120. E
25. E 73. A 121. B
26. A 74. A 122. E
27. B 75. E 123. E
28. A 76. C 124. A
29. D 77. D 125. A
30. A 78. A 126. B
31. B 79. C 127. C
32. D 80. C 128. C
33. B 81. D 129. E
34. D 82. D 130. B
35. E 83. C 131. B
36. C 84. B 132. D
37. B 85. C 133. B
38. A 86. A 134. E
39. B 87. E 135. E
40. C 88. E 136. D
41. E 89. D 137. D
42. B 90. B 138. D
43. C 91. A 139. E
44. A 92. C 140. E
45. A 93. B 141. B
46. B 94. A 142. C
Inference Questions food we require. But that truism doesn’t follow from this set
of statements, which are on the topic of species survival
generally, not the ability of any one creature to survive. (Also,
(D) ’s “depend primarily on” is no more justified than (A)’s “the
1. (D) main determining factor.”)
Unlike other Inference questions, you can often pre-phrase (E) draws a distinction that’s way out of line. It should make
an answer to Inference questions that ask you to complete an sense that during an environmental catastrophe, a large
argument. species is at greater risk than a small one, because the latter
This type of Inference question is very similar to a Main Point needs less food to survive. That’s a far cry from (E)’s confident
question, except that it asks us to find a conclusion that prediction that if the big creature is threatened, the small one
isn’t stated. We can do that by focusing on the scope of the will pull through. If things are bad enough, maybe all species
argument and asking what it’s all leading up to. This argument of all sizes will be wiped out; who knows? 49
starts by equating a century and a life, and noting that people
behave the same way towards the end of each. So (the last 3. (D)
sentence states), just as people at the end of their lives reflect Evaluate the choices in Inference questions by relating them
on their life, people at the end of a century . . . reflect on the to the stimulus.
century. That prediction fits with the argument’s comparison Don’t be fooled by the “if” in the first sentence. This question
between a life and a century, and can be found in (D). doesn’t require any formal logic. Instead, the educator
(A) distorts the stimulus; people reflect on what is ending, bemoans the loss of quality in today’s education, telling
which is not always their life. us how the emphasis on degrees has opened the door to
(B) Fear is outside the scope of the stimulus. obtaining meaningless credentials by completing courses
without leaning anything. The Method for Inference
(C) is the opposite of what the stimulus suggests will happen.
questions tells us not to spend time predicting, unless we can
(E) is also a distortion—there’s nothing in the argument about do so easily, so we’ll move straight into the choices:
second-guessing the events of the past century.
(A) The author tells us what has to happen for credentials
to be meaningless, but never says that more meaningless
2. (B)
credentials are being granted. Eliminate.
LR Inference questions are usually based on a set of premises,
(B) Again, the educator doesn’t claim that it’s now easier for
not an argument. Yet often the right answer is essentially a
students to get a meaningless degree. He only tells us that it’s
summary, i.e., the “main idea” of the facts presented.
possible to do so. Eliminate.
If, as we’re told, larger species are more vulnerable to extinction
(C) is far too extreme. The educator warns of the danger of a
than are smaller species on the grounds of the relative amounts
particular type of degree or certificate, but doesn’t advocate
of food each needs to survive, then species survival must have
the abolishment of all degrees and certificates. Eliminate.
at least some connection to the food source of each individual
member of that species (since, of course, each individual eats (D) Well, if it’s possible to obtain a degree “without ever
its own food, or starves, depending on prevailing conditions). learning much of value,” as the author says, then it’s certainly
(B) offers a summary of this idea, “vulnerability . . . at least in true that a degree alone doesn’t guarantee someone knows
part” acting as a nice parallel to the stimulus’s “This fact helps anything worthwhile. (D) must be true, and is correct. We can
make . . . more vulnerable.” quickly eliminate the last choice:

(A) Nothing in the stimulus supports any contention about any (E) The effort invested to obtain a degree doesn’t necessarily
particular behavior being “the main factor” in any particular relate to the benefits it bestows. After all, “plodding through
phenomenon. Indeed, the tone of the stimulus (“helps courses” sounds like an effort, but it leads to what the author
make . . . more vulnerable”) is quite tentative. calls a meaningless degree

(C) sets the extinction of a large species as a necessary


4. (C)
condition for the extinction of a smaller one. Say what?
There’s no hint of that kind of connection in the stimulus, Inference questions don’t have to ask for what must be true;
not to mention the fact that (C), like (A), is too certain in its they can ask you to find what could or cannot be true as well.
tone. “No small-animal species will become extinct” is far too This atypical Inference question tells us to look for a choice
unqualified an assertion. that CANNOT be true. If the correct answer is impossible based
(D) is in a sense self-evidently true: for the human animal as on the statements in the stimulus, then the four wrong choices
well as any other, of course our survival hinges on how much will be possible—in other words, they all could be true. We’ll
keep that in mind when we evaluate the answer choices.
“Most” is a clue that formal logic will play a role in a question, This sentence can be paraphrased as, “If a rule is a duty,
just like “if” or “only if.” Here, we find that most veterinarians then people can’t be excused from following it just because it
(meaning at least half of all veterinarians) have a strong would conflict with their self-interest.” In other words, you’ve
interest in biological science. Most veterinarians also choose got to do your duty even if it clashes with your self-interest. We
their profession primarily because of their love for animals. can combine this statement with the first one to get (C).
Be careful when there are two “most” statements about a (A) The stimulus never suggests that any duties are
single group. While there must be some overlap between the “overrated,” even if our own personal experience might
two characteristics—here, there must be some veterinarians suggest otherwise.
that have a strong interest in biology and also chose their (B) is way too extreme. The only thing we know about “all
profession because of their love for animals—the overlap societies” is that they define certain rules as duties. Even
doesn’t have to include the majority of the group. This concept if some of those duties might conflict with someone’s self-
is often tested on the GMAT. interest, that doesn’t necessarily mean that “no people are
Finally, we hear about “people who are seriously interested in capable of following” them, and rules that aren’t duties are
biological science but lack any special love for animals.” This outside the scope.
group includes no prominent veterinarians. Note the qualifier (D) “Some duties take priority over others”? The stimulus
“prominent” In the argument’s most definite statement. doesn’t compare different duties. Some people might think
This last group could include some veterinarians without that self-interest rises to the level of a duty, which would set
violating either of the first two statements, as long as those up this conflict, but the author never makes that connection.
veterinarians aren’t prominent ones. Armed with all of this
(E) contradicts the stimulus—if fulfillment of a duty is
information, let’s test the choices:
“fundamental to a properly functioning society,” so much so
(A) could be true. Even those vets that love animals and that duty is more important than self-interest, then societies
chose their profession on the basis of that love could still love must have the right to expect people to perform their duties.
biological sciences even more. Eliminate.
(B) is possible. The two characteristics of most vets could 6. (B)
overlap in a majority of vets, even though they don’t have to. Remember that “when” can signal Formal Logic.
Eliminate this choice, but note: if this question had asked for
This essayist would be hard pressed to imagine a world where
what must be true, (B) would be a particularly dangerous (and
someone wasn’t being hypocritical, and frankly, he isn’t alone.
common) trap.
But he seems to suggest that some good will come of it. We
(C) directly contradicts the last sentence. We know that there can translate his statements into formal logic by changing
are no prominent veterinarians among the people who are “when” to “if”:
interested in biological science but don’t love animals. (C)
If hypocrisy is exposed, then the hypocrites are embarrassed
cannot be true, and is correct. For the record:
and they and others are motivated to be better people. And
(D) could be true. The vets at university research centers if hypocrisy is not exposed, people believe that most people
don’t have to be part of the majority of vets who chose their are good, which motivates most people to try to be good. It
profession because they love animals. (D) is another trap that sounds like hypocrisy actually motivates people to be good,
deals with the overlap between the two different majorities of whether it is exposed or not.
veterinarians. Eliminate.
(A) is certainly tempting given what we know in the real world,
(E) Vets who aren’t prominent and what they consider but remember, we can’t use that information on the GMAT.
important for success are way outside the scope of the The question of blame (or the lack thereof) is outside the
argument. scope. Eliminate.
(B) Here we go. If hypocrisy exists, whether it is exposed or
5. (C)
not, people are motivated to try to be better people. This
If a stimulus includes language that suggests necessity, then follows very easily from the combination of the two formal
it includes Formal Logic. logic statements in the stimulus. This is the correct answer.
The first sentence of this stimulus involves a clear “All X are Y” (C) is the reverse of what the stimulus says. According to the
construction: All societies have certain rules that they define author, hypocrisy encourages morality, not lapses in it.
as duties. But the second sentence also sets up a Formal
(D) This is a classic irrelevant comparison. We don’t know
Logic construction, even though the language isn’t what we’re
whether hiding or exposing hypocrisy is a better motivator; we
used to.
only know that one motivates people “to try to be good” and
“Contained in the notion of a duty” tells us that we’re about one motivates people “to try to become better people.”
to get a characteristic of all duties—something necessary.
(E) “No stronger motivator” is too extreme. If we don’t know (E) Anissa is in the same situation as (D)’s Gerald: each is a
that one motivation is better than the other (like in the last dean’s list fourth year. She too gets a dorm room choice before
choice), we certainly don’t know that one is the best. any non-fourth year—such as Jehan. So (E) not only is not
inferable, it contradicts the text.
7. (E)
Inferences can be properly drawn only within the scope of the 9. (E)
information presented. Although refusal is defined as “the point at which [the pilings]
Fiber-optic networks cost less “overall” than copper networks, refused to go any deeper,” we also learn that Da Ponte met
even though one key component of the former, the cost of the the standard for refusal by driving his pilings until “additional
cable, is higher. The only way that the need for copper cable penetration . . . was no more than 2 inches after twenty-four
amplification can cause this price differential—and the author hammer blows.” Although it would be difficult to predict the
does use the words “This is because”—is if, as (E) alleges, the correct answer before you look at the choices, it’s clear that
amplification costs exceed the fiber-optic manufacturing costs. there is a difference between the absolute, “ideal” definition
of refusal, and the standard for refusal in Da Ponte’s time. (E)
(A) We can’t infer that fiber-optic cable costs more to
addresses this distinction nicely. It’s possible that the pilings
manufacture because its raw material is more expensive.
could have been driven deeper, (though probably not much
Maybe that has to do with the greater cost of labor or
deeper, and it would have been a real pain to do so). This
machinery.
inference follows directly from the definition of the standard
(B) The author’s focus is solely on cost comparisons; she for refusal.
shows no interest in revealing any “strains” on either cable
(A) is completely unfounded—if anything, the pilings were
system . . .
more likely to be safe, since they meet the standards. (B) is
(C) . . . nor is any time spent on the relative capacities of the also off base, since we know nothing about the safety of the
systems. bridge as a whole. (C) introduces other bridge builders, and
(D) Like the previous two choices, (D) brings up an issue— so moves out of the scope of the argument. (D) is even more
signal speed—that is outside the scope of the discussion, so sweeping, and introduces all the bridges built after 1558,
no inference can be drawn about it. though the argument provides us with no information about
that topic.
8. (D)
Translate formal logic statements into simple if/then terms. 10. (E)
The two statements in the stimulus translate as follows. If This is a somewhat unusual problem type in that the passage
you’re guaranteed a dorm room, then you’re a fourth year (it is arguing for a particular course of study, rather than a
does not mean the reverse—that if you’re a fourth year you’re final deduction. Nevertheless, we can see that the evidence
guaranteed a room—which causes problems in the choices, presented in the passage leads to a specific conclusion. It is
as we’ll see). And If you’re a dean’s list fourth year, then you established that innate mechanisms and environment both
can choose a dorm room before anyone 3rd year and below. play a role in language acquisition. The best course of study
Apply each of these statements to the concrete situations in (for those interested) should then be which factor is more
the choices. influential. (E) states this by asking if innate mechanisms are
more important than environment.
(A) As a fourth year (dean’s list or not), Benizer is a member of
the only group of people who get dorm room guarantees. But (A) is too general. The fact that language acquisition has more
that just means that she’s eligible for same; she may or may than one influence hardly suggests the necessity of a study
not receive such a guarantee. Eliminate. into whether the process is explainable at all. (B) and (C)
both suggest studies into something that is taken for granted
(B) As a dean’s list fourth year, Naomi gets precedence over
in the passage—that innate mechanisms influence language
any non-fourth years. But Ivan is a fourth year too. We have no
acquisition. (D) discusses specific elements of the language
way of determining the order of room choice between the two
environment (“parents and peers”) not mentioned in the
of them. Eliminate.
passage, and ignores innate mechanisms altogether.
(C) is contradicted by sentence 1: Since she’s not a fourth
year, Halle is most certainly not guaranteed a dorm room. 11. (C)
Eliminate.
The passage describes the rationalizations of embezzlers
(D) Gerald’s fourth year dean’s list status definitely entitles and violent criminals. The specifics vary, but all three of the
him to choose a dorm room over non-fourth-year Katrina. So rationalizations attempt to explain why the actions are not
(D) is inferable and is correct. For the record: really crimes. We then need to complete a sentence about the
criminals’ own characterizations of their situations. Although ways, and if you know your wrong answer types, they should
it might be tough to predict exactly, we can recognize that the jump out at you. Sentence 1 suggests a consequence of using
common theme in the three rationalizations matches (C)—the gene splicing to create new animal varieties; it sounds bad,
criminals’ actions are not truly criminal. and sentence 2 confirms that it’s indeed bad: One might not
The rationalizations deal only with the actions themselves, so be able to detect the genetic bases of poisons or cancer-
we can’t really infer much about what the criminals feel should causing agents in these new strains. (A) goes way too far in
follow these actions, whether reward (A) or punishment (E). its scope to all toxin production, but (B) follows as naturally
Even though the criminals may feel they have committed no as can be from the two sentences: If they’re true, then one
crimes, they never imply that they are themselves victims had better gene splice with care. Of the other choices, (C)
(B). (D) is a somewhat tempting wrong answer choice, but is a 180—the effectiveness of gene splicing for creating
note that it refers to a very general principle, rather than “the new animal varieties is a given—and (D) and (E) go awry
criminals’ characterizations of their situations.” by appealing to future consequences when the argument
is concerned with the here and now of the gene-splicing
12. (E) technique.
When both figures have milk, the monkey goes with the plush
15. (C)
suede model. When it’s a choice between suede and milk,
milk wins. What can we conclude? All things being equal, the This is not an argument, but a factual report on the
monkeys go for the comfort, but when it’s comfort versus food, ineffectuality of using rats to study lathyrism. Some countries’
the monkeys choose food. (E) sums this up nicely. domestic animals get it by eating this legume, but since rats
are an exception, (C) must be true. There cannot be universal
We know that the monkeys’ desire for comfort is less
susceptibility to lathyrism, because rats don’t seem to get it.
compelling than their desire for food, but we don’t know
enough to speculate whether it’s “nearly as strong as their That rats differ from other animals in their susceptibility to one
desire for food” (A). (B) speculates on whether animal fur disease in one way does not support a grand claim of radically
would be “more convincing” than suede, but we don’t know different physiology (A). The amount of legume consumed
the first thing about animal fur from this passage, so we (B) is mentioned nowhere in the stimulus, so nothing can
can forget about (B). (C) and (D) both speculate on the more be inferred about it. The incidence of lathyrism among wild
“convincing substitute” for a mother’s touch and teat, but they animals (D) could be widespread or could be minimal (no way
both get the order of preference wrong. In the experiment, to tell); and for all we know, the rats in question were fed the
the monkey preferred the milk over the suede when forced to legume and got a clean bill of health far from any laboratory,
choose, but (C) presents the suede as more compelling, and so (E)’s conclusion about lab conditions is uncalled for.
(D) presents them as equally compelling.
16. (E)
13. (C) We’re looking for a proper conclusion, so it behooves us to
(C) merely paraphrases the last sentence’s assertion that at read through the stimulus with an eye towards where it’s all
least some people taking antidepressants are going to gain a heading. The topic is blood cholesterol, a relevant concern
little weight. “these days.” Sentence 2 offers what most would accept as
conventional wisdom: the higher one’s cholesterol, the greater
No injunction to doctors, of any sort, is built into the
the risk of a heart attack. The following sentence tells us
paragraph (A), nor is any recommendation to patients (B). It
why this is important (heart disease is the number one killer
may be that the alleviation of one’s depression outweighs
in North America), and the passage ends with three factors
(pardon the pun) any additional poundage accompanying it.
that affect cholesterol. Not much of an argument; more like a
Or maybe not. But we can deduce nothing from the paragraph
collection of facts. But that makes sense, since it’s something
either way. (D) contradicts the first sentence: It’s the drugs
that “can be properly concluded from the passage” that
that cause the weight gain, not “lack of dieting.” And (E) both
we’re after anyway. It’s hard to prephrase exactly what that
contradicts the text (the implication is that weight gain is
conclusion might be, but you may have had a pretty good
inevitable and hence that weight maintenance is impossible)
feeling that it would have something to do with the factors
and presumes that those taking antidepressants must care
listed at the end. After all, the abrupt ending kind of begs the
about the accompanying weight gain, when in fact they
question “what about these factors? Why mention these?”
might not.
Only (B), (D) and (E) relate specifically to these factors, and
the first two conclude more about smoking than we can infer
14. (B)
from the passage (see below). But (E) works: Blood cholesterol
Only the technical nature of this stimulus might be off- influences heart disease, and the factors that influence blood
putting, because the four wrong answers go wrong in almost
classical
cholesterol are lifestyle choices. Put it together, and it’s 18. (A)
proper to conclude that the risk of fatal heart disease can be This unusual Inference question asks us for a statement that
influenced by changes in such choices. the stimulus doesn’t support, meaning that the four wrong
(A) Nowhere is it stated or implied that cholesterol level is choices are all inferable from the text. You have a couple of
the only factor influencing heart disease, so it’s too hasty to options here—read for the four inferable ones, or seek the
conclude that low blood cholesterol means low risk of fatal odd-man-out, outside the scope choice—but either way you
heart disease. Perhaps hereditary or other factors would put a must read the stimulus closely. Note especially the qualified
low cholesterol person at high risk. language: “can make,” “some skin rashes.”
(B) and (D), as mentioned above, distort the smoking issue. (A) might very well have leapt right out as the non-inferable
(B) compares smoking in moderation with smoking heavily, choice. According to the first sentence, it is too-dry air
but nothing in the passage allows us to distinguish between that is bad for computers, because higher humidity
the two. (D) is a classic extreme answer choice, signaled by the “protects . . . [them] from damage.” It is therefore the opposite
phrase “the only way. . .”: We can certainly infer that smoking of (A) that is suggested in the passage.
increases one’s risk of fatal heart disease by influencing (B) , too, stems from the first sentence. Another thing that
blood cholesterol. But can we conclude that this is the only warrants protection from too-dry air is furniture and draperies.
way smoking can have this effect? No. The information in the (B) just restates this portion of sentence 1 in slightly more
passage doesn’t discount other possible ways smoking may formal language.
cause heart disease.
(C) , (D), and (E) are all inferable from sentence 2. If
(C) also goes too far, playing on the notion that heart disease higher humidity “can make people feel warmer,” it’s just the
is the single biggest killer in North America. While high flip side that dryer air can feel cooler (C). If higher humidity
cholesterol is a factor in heart disease, we can’t even tell can “help the body’s defenses against viruses,” then (E) is
whether it’s the most significant factor. It’s therefore a far a slightly more formal restatement of that fact. And (D) is just
stretch to conclude that a high-cholesterol diet is the principal a rewrite of the clause asserting that humidity can alleviate
cause of death in North America. rashes.
• Listen for the “temperature” of the author’s
17. (A) statements—is s/he moderate or impassioned,
The question stem tells us we’re looking for an inference, so qualified or extreme?—especially in Inference
we should take the statements in the stimulus as true and questions, where tone can be all-important.
see what follows from them. The first sentence describes
some goals of physical education. But some schools turn 19. (A)
noncompetitive students away from sports by focusing For Inference questions, eliminate answer choices that
on competitive activities. Those students don’t exercise contradict the stimulus or are unknowable.
enough later. So, while sports and physical education are
Newton, if he acknowledged a debt to anyone, credited the
good things, focusing on competitive activities alienates a
ancient Greeks. But we can be sure Newton made no such
part of the population these activities are designed to help. acknowledgments to any “contemporary scientists,” including
So (A) is strongly supported. If physical education included Hooke. Given that, (A) must be true.
noncompetitive activities, maybe those noncompetitive
(B) The argument is about acknowledging intellectual debt.
students will get some of the benefits.
What Newton took credit for is never mentioned.
(B) We don’t know how many students are competitive and
how many aren’t, so we don’t have enough information to (C) again veers outside of the argument’s scope. At issue is
Newton’s indebtedness, not Hooke’s.
support (B).
(D) contradicts the stimulus. The reference to “giants”
(C) We know the noncompetitive ones don’t exercise enough,
probably refers to the Greeks, but certainly not to any
but that doesn’t mean that the competitive ones do. For all we
contemporary scientists.
know, they slack off for other reasons.
(E) is unknowable. Newton could have been aware of
(D) Mental vs. physical? That’s outside the scope. This
Hooke’s work on light and still not have acknowledged any
question is about gym class.
indebtedness to him.
(E) is a statement one could hardly argue with, but the
stimulus doesn’t provide any real support for it. Sure, the 20. (E)
author believes that children should be taught to pursue
“Therefore” indicates that we are to fill in the blank with the
healthy lifestyles. But the dangers of sedentary lifestyles are
most appropriate conclusion to that which precedes it. The key
one step removed from this.
here is noticing that the 3rd sentence brings in a new term.
The first two sentences deal with the relationship of exercise (E)’s hypothetical cause-and-effect is not reasonable. That
and aging, and spell out that exercise can slow down the aging Earth’s crust cannot support a huge volcano comparable to
process, while lack of exercise can produce aging. Then comes Mars’s biggest is by no means evidence that volcanoes bigger
sentence 3 with the first reference to drugs, and the assertion than Earth’s would be inevitable if the crust were thicker.
that no drug now or in the works can retard aging. Clearly, we That’s a huge leap in logic. (Perhaps a thicker crust would
need to relate the issue of drugs to the exercise/aging nexus, make such volcanoes possible, but that doesn’t make them
and even if you couldn’t pre-phrase it, (E) seems to add the inevitable.)
whole thing up. Since no drug can do the job whereas some
exercise may, then indeed, the person wanting to slow down 22. (B)
his aging would do well to try exercise rather than drugs. This question asks you to draw a proper conclusion based on
(A) is an unwarranted inference from sentence 3. That using the information provided in the stimulus, which means you
drugs cannot slow down aging doesn’t at all imply that drug need to find an inference. Twenty professional income-tax
use can produce aging in the same way that lack of exercise advisors were given fake financial information. Each advisor
can. (A) connects sentence 3 and the rest of the paragraph in prepared the return differently, and only one was technically
a most unwarranted way. correct. We can’t form an exact pre-phrase for this question,
(B) is an even grosser distortion of sentence 3 than (A) is. All but the correct answer is certainly not going to be a testament
we hear about drugs is that no drug can, or promises to, slow to the accuracy of professional advisors. This should lead
aging. In no sense need that mean that drugs are needed you to (B), as the evidence provided strongly suggests that
by those who don’t exercise. (And where does “sustain their professional tax advisors don’t necessarily prepare returns
health” shine in? It’s never mentioned in the stimulus, that’s correctly.
for sure.) (A) cannot be inferred because each individual advisor only
(C) , (D) Both choices fall short of what we need prepared one return. We have no idea how often an individual
because each one fails to mention drugs, and we need to tie income-tax advisor fills out a return correctly. If the advisor in
the concept of drugs to the rest of the argument. Beyond question is the one in the study who got it right, perhaps that
that, (C) distorts the stimulus by raising the issue of person will nail twenty out of twenty returns.
“prevention” when the stimulus merely talks about “slowing (C) goes against what’s being said in the stimulus. This
aging down”; and none of the evidence supports (D)’s out- experiment doesn’t exactly inspire confidence in professional
of-left-field speculation about what happens when exercise advisors. The stimulus shows that tax advisors often prepare
starts late in life. returns incorrectly, so hiring a professional would not ensure
correctness.
21. (D) (D) cannot be inferred because there was one correct return.
Mentally picture that supersized Mars volcano. Even if you For all we know, that advisor never makes an error when
can’t conceive of how high 27 kilometers would be, just preparing returns.
knowing that the thing is the size of a European country (E) compares the accuracy of individuals to that of advisors.
should suggest the scale. The author’s claim is that even Since the stimulus never mentions the accuracy of individuals,
with equal gravity on the two planets, no such volcano could we can’t make that comparison.
exist on Earth—and why? Because while the crusts of Earth
and Mars are made of the same stuff, Earth’s crust “is too 23. (A)
thin to support” such a mountain. But since Mars has such
We’re looking for something that must logically follow, given
a mountain, it has to follow that Mars has a thicker crust, at
that someone goes on a diet low in calories and cholesterol.
least where the giant volcano is, keeping it supported. A pretty
The right approach is to follow the chain of events described,
safe inference, provided by (D).
starting with the weight loss that sentence 1 says is the
(A) Erosion is outside the scope. The author gives us no reason hoped-for result of many people on such a diet. When weight
to infer that the erosion process has anything to do with loss occurs, one’s fat cells get smaller (though no less
that giant volcano on Mars, or what would happen to such a numerous), and those shrunken cells pour their cholesterol
mountain on Earth. into the blood. It follows, then, that though the person has lost
(B) , (C) The scope of this argument is one high Mars mountain weight on a low-cholesterol diet, he might well see a higher
and the reason why its counterpart couldn’t be found here. blood cholesterol level at first, at least until other, unspecified
Other Mars mountains, whether what’s going on beneath them mechanisms kick in and result in the lower level that is the
(B) or how they stack up relative to Earth’s (C), have nothing to other goal of the diet.
do with the issue at hand.
While you might not have pre-phrased (A) (and indeed, pre- (D) The terms of the contract are about Jones’s limited
phrasing is often difficult in Inference questions), its logic responsibilities for repairs, not Smith’s. To name but two
should seem on-point once you read it. “Might” is the right possible alternatives to (D), Smith might choose to ignore
word; more certain we cannot be, since a low-calorie, low- some defect and not repair it at all; or perhaps something in
cholesterol diet wouldn’t necessarily bring about weight loss local law might mandate that a government entity pay for the
and the entire chain of events described. But if such a diet repair of certain defects. Either is possible, so (D) need not
led to weight loss, then the person might see a temporary be so.
cholesterol increase, as the paragraph suggests and choice (A) (E) There’s no implication that any part of the house has
describes. ever suffered any structural defects whatsoever. Remember,
(B) , (C) Each brings up a comparison irrelevant to the this whole thing is hypothetical: the conditions are set in
evidence presented: between low- and high-calorie diets (B), motion if there’s a defect after the house is transferred to
and between low- and high-cholesterol diets (C). We get no Smith. Nothing in the stimulus speaks to repairs conducted in
evidence as to what happens when a diet is high in cholesterol the past.
or calories. For the record, note that another problem with (B)
is its irrelevant reference to the speed of weight loss. 25. (E)
(D) would be false if the person were to lose weight on this The stem tells us we’re looking for an inference, and the
diet: The paragraph says flatly that his fat cells would shrink. stimulus is just one formal logic statement after another. The
And if he lost no weight, well, who knows what would happen best approach on questions like these is to read through the
to his fat cells? (D) is therefore either a 180 or outside the formal logic statements, paraphrasing them as you go, and
scope, depending on the person’s weight loss. But either way, see if any deductions jump out immediately. If that doesn’t
it’s wrong. happen, hit the choices. The first two sentences tell us two
(E) Nothing in the stimulus describes a process for gaining things about people under the age of 18: They can’t be profs
weight, on this diet or any other, regardless of what happens and they can’t vote. The next sentence gives us three new
to the blood cholesterol. (E), if anything, goes against the facts about “brilliant” people. Some are profs, some can vote,
grain of the passage: If many people go on low-calorie, low- and some are under 18. Great. From this, a few deductions
cholesterol diets specifically to lose weight, why would we may have jumped out at you, but you were probably better off
expect such a person to gain weight as a result of such a diet? tackling the choices:
(A) We know that no profs are under 18, but some might be
24. (C) exactly 18.
In the sale of Jones’ house to Smith, the seller is going (B) We know that some brilliant people fall into those
to have a continuing responsibility for one aspect of the categories, but there might also be brilliant, non-academic,
house, at least for a limited time. For up to a year, Jones non-voting senior citizens out there.
will be responsible for repairing any defects in the roof or (C) and (D) are true in real life, but aren’t inferable based on
roof-supporting components—those are what are defined the stimulus. For all we know, all legal voters are profs (C).
as “major structural defects.” And we’re told that in this Similarly, it could be that all profs are either legal voters,
particular house, the only “roof-supporting components” are brilliant, or both.
the exterior walls. The most useful sentence (and we’re sure
(E) Since some under 18’s are brilliant, and under 18’s can’t
that Jones is glad of it too) is the third one: those are all the
vote and can’t be profs, we can deduce that some brilliant
repairs that Jones is responsible for. It follows that Jones will
people (the under 18’s) are neither legal voters nor profs. So
be responsible for any needed repairs to the truss roof and the
(E) must be true and is therefore correct.
exterior walls that support it, but not any non-exterior walls,
and so (C) properly defines the situation after Smith takes over
26. (A)
the house.
We’re looking for where the argument is leading. Just keep the
(A) Awareness of defect is a classic component of contracts
facts straight and add the different pieces of evidence together
such as the one between Smith and Jones, but we aren’t given
to see what can be concluded. We are told that a storm is
any information as to this condition’s applicability to this
more likely to occur when a large body of water exceeds 26
particular contract, so (A) isn’t inferable.
degrees Celsius at 60 meters. Then we’re told that increased
(B) The entire stimulus is speculative: it’s all about Jones’s amounts of carbon dioxide in the atmosphere will raise the
responsibilities if there prove to be any defects. We can’t be temperature of Earth’s water resulting in more large bodies of
sure how the roof and exterior walls are looking these days; water with temperatures warmer than 26 degrees Celsius at 60
they could be rock-solid or shaky. meters. So, what can we conclude? Forget about the specific
numbers for a second; they’re constants so we need not and anyone who has been lied to also has lied to someone.
obsess over them. We can simply think of “26 degrees Celsius So, every contract negotiator has lied to someone else.
at 60 meters” as a condition. The argument boils down to this: Whoever lies to anyone is practicing deception, so every
more carbon dioxide causes the exact condition that is said contract negotiator has practiced deception. That’s (A).
to increase the likelihood of tropical storms. We can therefore (B) could be true, but isn’t a valid inference. We know that
conclude that if the carbon dioxide keeps mounting, then every lie is an act of deception, but it might also be true that
there will, indeed, be a greater likelihood of tropical storms, every act of deception involves a lie to someone.
as (A) puts it. Why? Because as we’ve seen, there will be more
(C) is contradicted by the stimulus. Whoever lies to anyone is
bodies of water that satisfy the condition that makes these
practicing deception, no exceptions.
storms more likely.
(D) and (E) could be true, but the stimulus leaves open the
(B) is outside the scope. This choice is concerned with air
possibility that there are people that have not been lied to
temperature while the argument discusses water temperature.
(although they couldn’t be contract negotiators).
(C) We can’t tell from the evidence that there are now more
bodies of water that exceed 26 degrees Celsius than there 29. (D)
“ever was,” nor is this the conclusion the author is trying to
Astronomers used to think that Pluto’s gravitational pull
get at. For all we know there may have been more such bodies
caused the irregularity in Neptune’s orbit. Now, however, it
of water sometime in the past. Whether we are currently
appears that Pluto is too small to exert the gravitational pull
setting a record for the number of bodies of water that exceed
on Neptune that astronomers once thought it could. That
26 degrees Celsius is not a concern of this author.
suggests (D): that astronomers were incorrect in attributing
(D) The author is concerned with the likelihood, not the the irregularity in Neptune’s orbit entirely to Pluto, since that
ferocity, of tropical storms. attribution was based on an exaggerated estimate of Pluto’s
(E) distorts, and in fact, reverses, the information in the gravitational power.
argument. We’re told that increased amounts of carbon (A) Nothing in the stimulus supports any conclusion about
dioxide lead to the Earth’s water heating up. This choice Neptune’s size; the passage only deals with Pluto’s size and
claims the opposite, that increases in the temperature of Pluto’s gravitational pull.
the Earth’s water causes carbon dioxide to increase in the
(B) misses the point. The reason that it looks like Pluto isn’t
atmosphere. This is not only not inferable, but is also far from
entirely responsible for the irregularity in Neptune’s orbit
the conclusion the author is trying to lead us toward.
is that Pluto’s gravitational pull on Neptune appears to be
weaker than previously thought, not that Neptune’s orbital
27. (B)
irregularity is larger.
For adults in country X, the percentage of fat in their diet stays
(C) goes too far. While it seems that something else is
about the same as they grow older. However, most adults in
affecting Neptune’s orbit, there’s no way to determine that this
country X consume more fat as they grow older. Well, if the
is the work of a faraway undiscovered planet.
percentage of fat is the same, then the ratio of fat to nonfat
food must be the same, and if the amount of fat goes up (E) There’s nothing to indicate that (E)’s prediction is true—
over time, then the amount of nonfat food must go up by a there’s no reason to think that Pluto will continue to appear
comparable percentage. Put it all together and you get (B)— smaller and smaller.
most adults in country X eat more when they are older as
compared to their earlier adulthood. This is the only way in 30. (A)
which the percentage of fat intake can remain constant while We need to combine the statements to deduce a fact. The
the amount of fat eaten increases. concrete rule is in the last sentence, so let’s begin there and
(A), (C), and (D) Other countries? Non-adults in country-X? The work backwards:
variety of foods? These are all out of the scope. We’ve got the S.S. Coral, a cargo ship that’s docked at Port
(E) sounds pretty unlikely given all the extra food they’re Tropica. Well, if it’s docked at Port Tropica, then we know that
packing in. In any case, the conclusion in (E) cannot be it’s less than 100 meters long, and we know that it belongs
inferred from the facts given. either to the Blue Star Line or the Gold Star Line. Can it belong
to the Blue Star Line? No: all Blue Star cargo ships are more
28. (A) than 100 meters long. Therefore, we’ve got the S.S. Coral, a
cargo ship less than 100 meters long that belongs to the Gold
The key to this question is recognizing the formal logic
Star Line. Great—scan the choices like you would in a Logic
statements in the stimulus and forming a chain to arrive at the
Game at this point. No go; neither of these deductions is
deduction in (A). Every contract negotiator has been lied to,
there, which means we have to go further. Sure enough, we’re that doesn’t mean that less exposure reduces risk, let alone
told that all Gold Star cargo and passenger ships were built eliminates it as (A) would have it.
after 1980. Therefore, the S.S. Coral was built after 1980, and (B) , (C) Other cancers (B) and cancer cures (C) are outside the
(A) is our answer. scope of this argument, which is about the increased risk of
(B) can’t be true, as we’ve deduced above; no cargo ship less soft-tissue cancers only.
than 100 meters long can belong to the Blue Star Line. (E) is also outside the scope, but perhaps less obviously
(C) and (D) needn’t be true. There’s no reason Port Tropica so than (B) or (C). The statements are about the cancer risk
can’t be open to Blue Star passenger ships or, for that matter, caused by a kind of radiation at or below the frequency of
to Gold Star passenger ships, which are both less than 100 visible light. Whether there’s any cancer risk associated with
meters long. radiation higher than that frequency is an open question.
(E) Although most Blue Star ships were built before 1980, that
means that definitely some Blue Star ships were built on or 33. (B)
after 1980, which means that some Blue Star ships may be The phrase “most strongly supports” is GMAT-speak for
newer than some Gold Star ships. “inference,” and figuring out which choice must be true is
easier if you mentally picture the situation described. If the
31. (B) cave’s stalagmites are formed by water dropping onto a cave
Is there a connection between the level of television viewing floor, and the cave is currently submerged, then it has to be
and aggression in children? A recent study found that high true that the water level within the cave was lower at some
aggression was found in kids that watch a lot of TV and in time in the past. How else could the stalagmites have been
kids that don’t watch a lot of TV. Further, the study also found formed in the manner described?
that parents who planned their lives in a “child-centered way” (A) No evidence suggests that the height of the Mediterranean
(which, according to the author, included lots of TV) had less itself is any different from what it once was. Indeed, the
aggressive kids than competitive middle-class parents who sea’s height could have been the same (or higher or
didn’t allow as much TV. So, methods of child-rearing have lower) throughout history, with no damage to the author’s
a role to play in determining child aggression, which means information.
that TV viewing cannot be the whole story. In other words, a (C) fails the Denial Test. Even if (C) is false—even if others
child’s level of aggression cannot be predicted from levels of knew about the cave tunnel before the divers did—the
TV viewing alone, choice (B). stimulus isn’t challenged.
(A), (C), and (E) contain dubious claims of causality. From the (D) Nothing in the paragraph necessitates an additional cave
stimulus, there’s no way to tell whether TV watching by itself entrance now or at any other time. The author’s concern is
leads to more or less aggression, (A). Similarly, just because what went on in the cave, not how or why anything entered or
the “competitive” approach is correlated with more aggressive left it.
children doesn’t mean that the competitive method caused
(E) seems to imply that the stalagmites, formed by mineral
the extra aggression. For all we know, those competitive kids
deposits, left the Mediterranean mineral-poor. But for all we
could have been born that way, so we can’t infer (C).
know the mineral content of the sea was replenished long ago.
As for (E), there’s no way to tell whether TV viewing or parental
lifestyle makes a bigger difference in child aggression. We still 34. (D)
don’t know whether TV watching by itself leads to more or less
This isn’t exactly a Paradox question, but there is a puzzlement
aggression, and there’s no way to compare it to the general
at the heart of the facts described. Most duck nests are well
effects of parental lifestyle.
hidden under natural conditions, so it’s rare for one duck to
(D) jumps to conclusions. The explanatory power of TV lay an egg in the empty nest of another (which they will do, if
watching is still an open issue, and there’s no way to tell if they see the nest unoccupied). However, when an unnatural
that explanatory power is limited to children of child-centered condition is imposed—the creation of man-made nesting
parents. boxes—the ducks lay eggs hither and yon, and that Egg-a-thon
impedes all the hatching. It follows that the nesting boxes
32. (D) must not be as well-hidden as the relatively undisturbed
(D) succinctly sums up the correlation implied in the stimulus. empty nests in the wild, and that—as (D) says—if they were
Nonionizing radiation can increase one’s cancer risk, and less conspicuous, the breeding efforts would be enhanced.
certain electrical devices contain that radiation. Hence, (D). (A) , (E) The facts make no representation that nesting boxes
(A) Once again, the “fallacy of denying the antecedent” is at are erected either in the absence or presence of natural
work. True, more radiation exposure raises cancer risk, but nests (A). Nor do they mention habitat destruction, or the
reason why ducks might need breeding help (E). And anyway, calories to maintain your weight than it did before you started
a fact about how, when, and why nests are established says dieting. Eat the amount of food you did before the diet, and
nothing about how many eggs are laid in them. your weight will increase. Truly a vicious cycle. (C) perfectly
(B) We know what happens when a female duck sees another captures the essence of the stimulus.
leave her nest: she lays an egg in the neighbor’s spot. But (A) distorts that idea of “excess weight” in the stimulus’ last
what happens when ducks see ducks goes unmentioned. sentence. The vicious cycle leads one to gain back weight that
(C) As far as we can tell, easily-visible nests in the wild would was lost while on the diet. Will the dieter regain all the lost
be just as crammed with extra eggs as the easily-visible nesting weight plus additional pounds? The stimulus doesn’t say.
boxes. No inference can be made about their relative sizes. (B) is an odd juxtaposition of two ideas. Why would the author
want to deter anyone from dieting in the first place? And
35. (E) maintaining one’s lower weight is a post-diet concern.
According to the author, a necessary component of creativity is (D) is another non sequitur. Nothing in the stimulus suggests
the ability to manufacture variations on a theme—to produce that once you’ve lost weight through dieting, you must further
deviations on a motif; to create alterations on a pattern; to reduce your food intake to keep from gaining weight.
invent extrapolations on a phenomenon (we think you get the (E) Based on the stimulus, we don’t know whether there is any
idea). And note the passage structure: the second sentence way out of the “vicious cycle” described here. Moreover, even
provides an evidence example, highlighted by the Keyword if the vicious cycle were unavoidable, we still don’t know if the
phrase “for instance.” The third sentence merely rehashes in dieter will regain all of the weight lost by dieting, were she to
even stronger terms (“always and without exception”) the idea resume her pre-diet eating habits.
from the first sentence, leading to the conclusion that every
new theme, idea, or discovery is in some way or another a
37. (B)
variation on a previous theme or idea. The stem instructs us to
find the one statement that cannot be inferred from this “rule” We’re told that despite the cheapness of cement’s ingredients,
(and that’s in fact what if boils down to—if it’s a new idea, then its price is influenced by oil’s price, the reason being the
it must be a variation on something older), and we have no high energy needed to turn the ingredients into cement. The
choice but to try out the choices. The first four turn out to be implication is that oil is needed for that transformation, at
valid deductions of the rules in the first and third sentences, least in some cement making, and that’s answer choice (B).
as we’ll see below. (E), on the other hand, is a direct violation: Use the Denial Test: if choice (B) were false—if no cement
kilns used oil—then how would the cost of oil have anything
contrary to (E), any new scientific discovery must be a variation
to do with the cost of cement?
on a previous theme.
(A) The very use of the Contrast Keyword “Nevertheless” belies
(A) The first sentence boils down to the fact that the ability
this inference. “Oil is nevertheless an influence, despite the
mentioned is necessary for creativity. Therefore, no “variation
cheapness of cement’s ingredients;” in other words, oil is no
on theme” ability, no creativity.
part of the composition of cement but a part of its cost.
(B) paraphrases the rule in the conclusion: if all ideas are
(C) and (D) Both choices go too far. That cement’s price is
linked to a past idea, then it’s true that no idea is completely
influenced by oil’s price is a modest claim, one that cannot
independent.
be read as suggesting a flat-out direct (C) or inverse (D)
(C) finds another way to say the same thing: if everything new relationship.
must be a variation on something old, it must be possible
(E) The author implies that at least one factor in cement
(theoretically, at least) to trace the new idea back to its
making—the needed energy—is both separate from cement’s
evolutionary predecessors.
ingredients and relevant to cement’s cost. So (E) is a highly
(D) Isn’t it clever how the choices themselves are mostly unlikely inference.
variations on the theme of the passage, which itself is about
the variation on themes? If every discovery involves a variation 38. (A)
on a previous theme, then every discoverer must possess the
The question stem is a fancy way of asking us “so, where’s
ability to manufacture such variations.
she going with this?” The first sentence begins with “some
people claim . . .”and the second sentence begins with the
36. (C)
word “yet.” Believe it or not, you can pretty much answer the
Pay close attention to the scope of the stimulus in an question from this information alone. We know from reading
Inference question. the stem up front that our job is to fill in the conclusion,
Does dieting lower your metabolic rate? This speaker says yes, and the sentence structure of the first two lines should have
once you lose weight through dieting, your body needs fewer helped you to deduce it. Think about it: if someone says to
you “some people claim X,” and follows that up with the word (C) is an out-of-place policy recommendation. The argument
“but” or “however” or “yet,” what do you think is going to never discusses what should be done with interest rates or
come next? The speaker thinks NOT X!! Replace X with “elected anything else.
officials must avoid the appearance of impropriety in office,” (D) There are no restrictions on the source of new capital
and NOT X, the conclusion that the word “yet” screams for, investment. The stimulus deals only with falling rates; since
becomes choice (A). The rest of the stimulus supports this we’re told nothing of rising rates, it’s totally possible that new
conclusion. The only possible reason for X (avoid appearance capital investment that takes place while interest rates are
of impropriety) is Y (maintain public approval and popularity). rising could lead to industrial growth.
No one has an obligation for Y, therefore the conclusion NOT X
(E) This answer choice scrambles the terms of the argument.
is confirmed (no elected official has an obligation to avoid the
The argument never mentions “manufacturing capacity newly
appearance of impropriety).
created” and there is no requirement that it be “fully utilized.”
(B) The author states that the only reason to avoid the
appearance of impropriety is to maintain public approval. The 40. (C)
author never states, however, that all elected officials have
The issue here is the different ways in which people pronounce
a vested interest in a high public approval rating. The author
words when reading as opposed to speaking off-the-cuff, and
merely states that if officials want to maintain public approval,
how that difference impacts on computer speech recognition.
they should avoid impropriety. Some politicians could actually
A conclusion based on these observations should therefore
have a vested interest in a low rating. Also, don’t read more
address this issue, and (C) flows logically; if the computers
into a question than exists: “vested interest” is not the same
are programmed from read speech, they will likely have a
as “maintaining public approval,” and “high” public approval
more difficult time decoding a different type of speech (i.e.,
rating is not the same as public approval and popularity.
spontaneous).
(C) and (E) are way too specific: the author never singles out
(A) is a little too extreme; the computers probably could
and discusses “good” elected officials (C) or “bad” elected
decode spontaneous speech, just not as well as words read
officials (E), so any conclusions relating directly to these sub-
off a page. Also, the passage deals with the problems inherent
groups of elected officials is out of place in this argument.
in the programming today—it makes no mention of what is
(D) The author states that the only reason an official may ever possible in the future.
want to avoid impropriety is to maintain public approval,
(B) No mention of different accents was made—the statements
but this does not mean, as (D) states, that the public never
deal with differing speech patterns of the same person.
approves of an official who appears to have behaved
improperly. It’s possible, according to the stimulus, that an (D) Once again, too extreme—just because something is more
official can appear to have behaved improperly and still be likely to be a problem doesn’t mean that it will never work.
approved of by the public. (E) We’re told nothing to suggest that speech-recognizing
computers are flawless. Besides, the scope of the argument
39. (B) covers computers in the immediate or near future (check
The stem tells us to draw a conclusion from the formal logic- it out).
type statements found in the stimulus. The major key is for
you to recognize that the first statement is a statement of 41. (E)
necessity: “there can be no growth without investment” Ordinarily, you should move straight to evaluating the choices
means that if capacity is full, growth needs, or requires, in an Inference question; but if a prediction jumps out at you,
investment. That is, under full capacity, investment is feel free to scan for it.
necessary to bring about growth, although there may be other Advertisers have found that people will develop more positive
factors that are needed as well. Now we can integrate the attitudes about things when they’re pictorially linked to
second sentence: reducing interest rates produces investment, things those people already have positive feelings about.
the very thing we just saw is necessary for growth. Bottom This question lends itself very well to prediction. Since it’s an
line: reducing rates won’t guarantee growth, but it will lead to advertiser’s job to engender positive feelings in his market,
one necessary factor for growth—investment. In other words, what’s he likely to do? That’s right—create pictorial links to
as answer choice (B) has it, a reduction in rates allows for a things people have positive feelings about. That’s (E).
condition necessary for growth to come about.
(A) The stimulus didn’t tell us anything negative about prose,
(A) contradicts the stimulus. Any reduction in interest rates only that it worked better when there were also pictorial links.
produces new capital investment—no exceptions. Eliminate.
(B) Distortion—it’s the associations we need to create resources, but the author stops short of that question so no
pictorially; it’s the advertisers’ methods, not their products, inference can be made.
which are at issue. (D) is far too broadly stated. Nowhere does the author bring
(C) Both television and magazines can include visual images, up whether assets are “vital” or not, and “every nation” goes
so this is an irrelevant distinction. beyond the scope as well.
(D) Nothing in the stimulus leads us to believe there’s a (E) Where do “changes in the environment” shine in? Never
benefit in negative associations or contrast. mentioned by the author, so impossible as the basis of an
inference.
42. (B)
Often the line between an Inference and an author’s 44. (A)
conclusion is fine indeed. Endeavor to reduce complex text to simple terms.
The environmentalist paints a grim portrait of what happens The cited critics see a disjunction between Freudianism (which
when trash is incinerated: poison from heavy metals escapes says that our unconscious desires can trump our rationality) and
into the air. And since many appliances contain heavy metals, the mainstream of Western thought (which is predominantly
surely incinerating them would be a mistake: that’s (B), a rational). But the author goes on to point out that Freudians
restatement of the danger cited in the environmentalist’s first also hold that learning about one’s unconscious desires can
sentence. conquer them—and, inferably, allow rationality to rule. The only
(A) You may know that recycling is a real-world alternative to logical conclusion is that Freudianism is not at odds with the
incineration, but since the term is never mentioned, (A) can’t mainstream of Western thought—and that’s (A).
possibly qualify as a statement that “must be true” or “must (B) “Happiness” as a term is never mentioned in the stimulus,
follow.” so it cannot possibly be part of the logical fill-in.
(C) Clearly the author thinks that chlorofluorocarbons (C) goes into the realm of future prediction, something the
are dangerous, because she cites their presence in stimulus never does. Anyway, the issue is the coexistence of
refrigerators right after saying “Discarding old appliances Freudianism and Western thought, not the influence of the
can be dangerous:”—and note the colon there. Trouble former on the latter.
is, chlorofluorocarbons aren’t necessarily harmful to the (D) That psychoanalysis can help one to avoid being defeated
atmosphere, as (C) would have it. Maybe they are just an by unconscious desires doesn’t mean that it must be able to
earthbound nuisance. Insofar as this stimulus is concerned, provide “a rational life plan.” That’s a wholly different issue.
we only know of heavy metals’ threat to the atmosphere.
(E) offers an unwarranted and irrelevant comparison between
(D) For all we can tell, newer appliances may contain equal Freudianism and other (unmentioned) psychological theories.
amounts of heavy metals as older ones, and hence pose
exactly the same threat. (D) is a classic irrelevant comparison. 45. (A)
(E) is a 180. On the evidence of the passage, landfills—at least For most Inference questions, the correct answer is something
well-operated ones—are a far better final resting place for that must be true based on the stimulus.
appliances than the incinerator.
As with many Inference questions, especially those with longer
stimuli, the correct answer to this Inference question draws
43. (C)
on only part of the stimulus. In fact, it can be inferred from
Be on the lookout for conditions necessary for a result. just one of the several statements in the stimulus. The last
All the naturalist says is that a condition necessary for a sentence states that courts, when applying laws, sometimes
dependable accounting framework (i.e., full accounting bend them in response to moral beliefs. (A) is correct because
of assets) isn’t being currently met, since publicly owned it states this fact in a different way—by speaking more
national resources have been left out. The only conclusion generally, but still within the scope, about how moral beliefs
to be drawn is that the current accounting framework is not affect the application of these laws.
dependable—or “not reliable,” choice (C). (B) starts off with information stated in the second sentence,
(A) ’s prediction is utterly unjustified, since the author restricts but then becomes too extreme in the second half when it says
himself solely to the necessary condition for dependability, law is “devoid” of moral aims, a notion the author does not
and his assessment as to whether that condition is being met. support.
All present tense, no future. (C) is outside the scope because the author never mentions
(B) seems to be edging toward an assessment as to why “protection” of any kind of actions. While the last sentence
economists have failed to factor in publicly owned natural says courts may change their application of laws based on
moral considerations, it does not discuss actions based on the result of bad soil and farming practices (E). Rosen’s
moral considerations. argument leaves open the possibility of having good farming
(D) departs from the scope by including the notion that and soil, and still having bad food.
laws “should not reflect any moral convictions.” The author
doesn’t get close to making a recommendation—the passage 48. (D)
merely states facts. This answer exhibits a very common GMAT Both classes unanimously said that the painting they were
inference trap: it is possible the author feels this way, but the told was prestigious was actually a better painting. What can
right answer to an Inference question is always something that we conclude from this? The classes seemed to be influenced
must follow from the stimulus; being likely is not enough. by what they were told about the painting, and (D) says
(E) is incorrect for two reasons: it improperly reverses the exactly this.
author’s statement in the second sentence and it is extreme. (A), (C), and (E) all overstep the bounds of the evidence by
The author says the law definitely orders society; he does not making unwarranted assumptions about what might happen
say anything about whether this way of ordering society is the in hypothetical cases. We only know what happened in this
“best” way of doing so. Remember that the right answer is one case, so we can’t say for sure what would happen if they
never stronger in tone than the stimulus. saw other paintings (A) and (C), or had been told something
else (E). We also can’t say that nothing the instructor said was
46. (B) true (B). (In fact, since the instructor reversed the descriptions
We know that Newtonian physics was a highly substantiated of the paintings for the second class, there’s a good chance
theory that was dominant for over two centuries, and that that the instructor told the truth about a given painting in
it was then supplanted by a physics based on Einstein’s either the first or the second class.)
theories. It’s therefore very safe to say that “long-standing
success or substantiation . . . is no guarantee that the theory 49. (B)
will continue to dominate indefinitely,” choice (B). The chain of events described goes something like this:
All four of the wrong answer choices make unfounded latest technology and great economic prosperity →
generalizations. The information provided is not sufficient to bombarded with images of other countries →
conclude that the entire history of physics is characterized questioning of customs and traditions →
“by one successful theory subsequently surpassed by another dissolution of these customs and traditions
(A).” After all, there could be other periods when two or more
competing theories are both partially accepted, for example. The question then asks about the stability of a nation’s
We also don’t have enough information to conclude every cultural identity. According to the chain above, technology and
theory of physics will eventually be surpassed (C), that any prosperity will eventually lead to a dissolution of customs and
dominant theory will remain so for centuries (D), or that traditions, so (B) follows directly from the information in the
a theory can only be surpassed by an equally successful passage.
theory (E). (A) might be tempting at first, but remember not to add
anything not contained in the passage—we might speculate
47. (A) that distributing technology fairly would stabilize cultural
Rosen presents a formal logic chain that would look like this: identities, but, according to the passage, it would just assure
that the cultural identities of all nations would dissolve.
good meal → good food → good soil → good farming → (C) also might make sense, but it doesn’t follow from the
culture that maintains natural resources information in the passage, since laws to preserve cultural
(A) states that a good meal depends on cultural and natural identity are never mentioned. (D) and (E) are both 180 answer
conditions. We can follow Rosen’s chain of cause and effect to choices, since they both endorse technological development
see that this is true. to preserve cultural identity, in direct contradiction to the
chain of events described in the passage.
(B) states that natural resources depend on farming practices.
According to Rosen, it’s the other way around—farming
50.
practices depend on a culture which values natural resources.
(C) also reverses the chain of causation, since it’s farming This question has been removed.
that’s a prerequisite for good soil. (D) confuses necessity with
sufficiency. Although good cultural values are necessary for a
good cuisine, they are not sufficient, i.e., they do not
guarantee a good cuisine. The fact that good food depends
on soil and farming practices does not mean that bad food is
necessarily
51. (B) (D) is a flagrant misreading of sentence 1’s second clause. The
The stimulus deals with the difference between speaking and statement “All freelance writers are evaluated” cannot be read
writing. In the former, tone of voice and gesture help indicate to mean “Only freelance writers are evaluated,” as (D) would
meaning, but in writing that role falls to authorial style, have it. Finally, (E), the author takes no position on the quality
defined here as “the arrangement of words and sentences.” of workers other than freelance writers; notwithstanding that
The communication of a writer’s intention, therefore, is most workers don’t enjoy quality control, they still might
assisted by this arrangement, which is exactly what (B) states. produce plenty of high-quality work.

Nowhere does the stimulus suggest a hierarchy (“primary


54. (A)
function”) of style’s purposes (A). (C) is an unwarranted
comparison, pitting oral tone and written style against each After defining the concept of environmental awareness and
other. Notice the extreme language in the last two answer focusing on how that awareness is triggered, the stimulus
choices. (D) states that readers “will always” interpret a goes on to explain that most people’s is triggered by doing a
writer’s intention differently (no, not when the writing is not math puzzle, but that some (we’re in sentence 3 now) find it
ambiguous, and even then, the “always” goes too far). And decreased—and that’s all that correct choice (A) is asserting.
(E) suggests that style “completely determines” aesthetic (B) is 180 degrees off: Visual-answer-formulators, we’re told,
value— that’s too strong, and outside the scope as well. have less awareness of their surroundings. (C) proposes
the irrelevant distinction between math and non-math
52. (C) problem solving; the latter are never mentioned. (D) is a
“Most reasonably supported” tells you it’s inference. gross distortion of the stimulus’s last sentence: That those
whose awareness rises take “a rest at the end of every stage”
That the crying of a discomfited infant is temporarily quelled
does not imply that those whose awareness decreases only
by the sound of the mother’s voice, but not that of others,
become aware during rest periods. (E) mixes details from every
doesn’t support that it learns that voice more readily, (A), or
sentence of the stimulus and makes an utter hash of them.
first before anything else, (B); the learning process is wholly
outside the scope of the brief passage. We cannot infer that
55. (D)
the mother “provide[s] comfort,” (D), under the circumstances
described, because all that happens is that the child stops Our task is to find the statement that’s most strongly
crying for the moment. For all we know, the discomfort supported, so it’s reasonable to expect that prephrasing an
continues. And since the source of the discomfort might be answer may not be possible. The plan is therefore to gather
some medical condition that a doctor for example, or some up the facts, understand them in your own words, and move
talcum powder, might best relieve, (E)’s suggestion that the on to the choices. The nutritionist describes a claim that’s
mother’s voice is always the best relief should strike us as subsequently disputed by “new studies”: replacing fatty foods
somewhat absurd. with carbohydrates may not work for those on low-fat diets, as
once believed, because eating a lot of carbs produces extra
That leaves (C), which is the most benign of the answer
insulin, which in turn acts on sugars and starches to provide
choices as befits the single, simple given example. The most
energy and, more importantly, produce fat when energy levels
we can deduce is that the mother’s voice sends a message
are sufficient. There’s one key element here that we overlook
that relief is forthcoming, hence the temporary cessation
at our own risk: it is a “high-intake” of carbs that triggers this
of tears. Inferably, if the mother were unable to relieve the
result. Bearing this in mind, we move on to the choices:
discomfort, the crying would begin again.
(A) As far as we know, insulin-produced energy and fat become
53. (B) an issue only with a high intake of simple carbs. We don’t
know what happens with a normal or minimal intake of carbs,
Rather than predict anything, we can simply check each choice
so we can’t infer that people on low-fat diets should “avoid”
against the stimulus and look for that which must be true. (A)
simple carbs altogether if they want to keep up their energy.
need not, since the strictness of evaluation is never mentioned
Perhaps a small to normal intake of simple carbs for such
and hence need not be the reason for freelancers’ high-quality
people is just the ticket.
work. But (B) echoes the author’s overall point. The workers
(B) is talking about are the freelance writers. They, at least, are (B) Nothing in the stimulus concerns the reasons to go on a
always evaluated and “that is why” they produce (they “are low-fat diet, so (B) operates beyond the scope. The issue here
caused to produce”) high-quality work. For the record: is how certain eating habits may affect those already on such
a diet.
(C) makes an irrelevant and unwarranted distinction between
freelancers and others; for all we know, other types of workers (C) has it all backwards: the nutritionist is concerned with the
may produce even better work than do freelance writers. And question of whether those who are limiting their intake of fatty
foods should consume simple carbs, not the other 57. (B)
way around. The stimulus for this Inference question, like many others,
(D) is the choice supported by the stimulus. We can interpret presents not an argument but a set of facts. This one lists
“limit their intake of foods high in simple carbohydrates” to three conditions or applications that can affect a poppy
mean “not including a high intake of simple carbs in their flower. (1) When a poppy is pollinated, its petals wilt in a day
diet.” And that’s just what the nutritionist would recommend or two because of some substance released into the flower.
to people wishing to avoid gaining body fat, since consuming (2) If a poppy’s not pollinated, then as long as it’s fed, its
above a normal intake of simple carbs will add body fat due to petals won’t wilt for a week or so. (3) If the unpollinated
the extra insulin produced. flower is cut from the plant, the same substance is released
(E) The people in (E) are either the exceptions to the rule or as in pollination. We can infer that the cut flower’s petals will
are eating small amounts of foods high in simple carbs and therefore wilt, because it gets that substance . . . but if you
not getting a high enough intake to trigger the extra insulin didn’t see this right away, all you needed to do was attack the
production. In either case, who’s to say that such individuals choices in search of the one that must be true.
will not lose weight if they don’t restrict their intake of other (A) The behavior of insects, and what influences it, are outside
foods? Perhaps they eat normal amounts of all other foods, the scope of this discussion of what happens to the plants
and restricting their intake of simple carbs will allow them to after (or instead of) pollination.
lose weight. We just can’t tell. (B) is what we want. A cut flower, according to the last
sentence, will have its mystery substance released, and
56. (B) according to sentence 2, its petals will therefore wilt in a
The weird 23-degree angle at which the earth rotates can matter of days. (Incidentally, notice that (B)’s reference
be maintained, asserts the author, “only” by the Moon’s to nutrients is a red herring, since nourishment is only
gravitational influence. That “only,” of course, signifies a mentioned in terms of unpollinated flowers. Nourishment
necessary condition: without that gravitational pull, Earth’s notwithstanding, cut flower petals will wilt fast. Period.)
angle of axis would go cockeyed. And without that stable angle (C) Since this passage is wholly about poppy plants, there’s
(say lines 5–6) the planetary climate would lose its ability no conceivable way to use it to make an inference about “all
to support life. So that’s the chain of cause and effect that plants.” This choice should have given you little pause.
culminates in (B): Loss of the Moon loss of the angle of axis
(D) , (E) Each of these goes far beyond the stimulus’
loss of a life-supporting climate.
scope to focus on issues that might be relevant to the
(A), (C) The last sentence cites Mars as an example of a planet poppy but not to the author’s set of facts. Each choice is too
lacking the kind of helpful moon gravity that leads to a stable technical
axis angle. But it doesn’t follow that if Mars had the right and unsupported by hard evidence to qualify as an inference here—
sized moon for gravitational help, and then had a stable angle we simply don’t know enough about the biological mechanisms
of axis, that it would be able to support life. Remember that involved to affirm either one.
while a moon’s gravitational aid is necessary for the support
of life, necessity is not sufficiency. Other factors—the right 58. (C)
kind of atmosphere, for instance—might yet be needed for life
A correct inference will stick to the scope and language of the
to thrive on Mars. So (A)’s conclusion is overly optimistic. The
original text.
same is true for (C), which generalizes to all planets.
We “should” do whatever it takes to make people more
(D) That a moon can (and does) have a strong influence on
virtuous and avoid things which make people less so. Praise
its planet doesn’t ipso facto exclude the influence of other
makes the more virtuous less virtuous and the less virtuous
factors. Consider that the Moon’s strong gravitational influence
more virtuous. Oh, and one more thing. The more virtuous are
keeps Earth’s angle of axis “fairly stable,” while with Mars’s
the people who actually deserve the praise. Got all that? The
little moons, Mars fluctuates wildly. Well, it’s quite possible
correct answer is going to glue two or more of these concepts
that other factors contribute to Earth’s stability, as well as
together. By the way, you may have noticed that all the answer
keeping Mars from spinning out of control. No way to tell.
choices begin “we should.” That tells us that the correct
(E) It’s not the number of Mars’s moons, but their net inference is going to have to make use of the first sentence in
gravitational influence that is cited as a factor in the absence some way.
of Martian life.
(A) ’s recommendation is unknowable, as the stimulus never
makes any statement about those who deserve praise the
least. Eliminate.
(B) , like (A), is unknowable. Who deserves praise the most is a
concept never broached.
(C) By not praising those who deserve praise (the more 61. (E)
virtuous) and praising those who don’t (the less virtuous), (C) The stimulus here, you should realize, is not an argument but
doesn’t corrupt the more virtuous but makes the less virtuous rather a set of facts, on the topic of fabrics and what happens
more so. It’s the correct answer and makes the world a better to them under high temperatures. Natural fibers, we’re told,
place. For the record: are tightly curled up, but fabric requires straight fibers, so the
(D) presents the generous solution of praising everyone. natural ones first have to be straightened; when washed under
But that would include the more virtuous, making them high heat such fibers “return to their original [state],” i.e., their
less virtuous, thereby contradicting the first sentence of the tightened-up state, and thus shrink. What about artificial
stimulus. fibers, mentioned only once, in sentence 1? They, like all
(E) ’s stingy solution withholds praise that could make the less fabrics (see the final sentence), “return to their original states”
virtuous more virtuous, again contradicting the first sentence. under high heat. But since fabrics made with artificial fibers
don’t shrink, it follows that artificial fibers must be straight to
59. (E) begin with—hence the absence of shrinkage. (E) has that right.
Don’t underestimate “supported” in the wording of Inference Notice how the four wrong choices all deviate far from the
questions. The correct answer to Inference questions must author’s scope:
be true. (A) No inference about low wash temperatures can be made
According to this nutritionist, although most adults reported from this paragraph. Low heat is never alluded to.
that their diets met standard nutritional guidelines, that just (B) In the same way, the issue of natural + artificial blends is
wasn’t true. Everybody said they ate less fat than they did. beyond the author’s scope. (In any event, the natural fibers
They ate fewer fruits than they said. Women didn’t eat enough would likely still behave under high heat the way the author
vegetables. Men did get enough bread, but “they fell short in describes, and hence would shrink, and hence would be
all other categories.” Men must have therefore fallen short in affected by high heat, contrary to (B). Either way, (B) is hardly
“fruits and vegetables,” just as (E) states. inferable.)
(A) Men, in correctly reporting their performance in the bread (C) brings in the irrelevant issue of stretching. Like (A)’s low
requirement, didn’t misrepresent their performance in every temperatures, it’s never raised, hence cannot be the basis of
category. an GMAT inference.
(B) stakes a numerical claim on two groups never mentioned: (D) speculates hypothetically as to what would happen if one
women and men who actually did report some deficiency in tried to straighten natural fibers a second time. But the author
their own diets. never goes there.
(C) is unknowable since we’re never told how women fared in
the bread category.
(D) exaggerates the wording of the stimulus. We’re told that
men fell short in all “other” categories (besides bread), but
(D)’s insistence that they fell significantly short just isn’t
supported by the stimulus.

60.
This question has been removed.
62. (D) (E) brings up the sample size, an issue way outside the
Here’s something relatively rare—an Inference question that’s scope of the author’s logic. And in any case, we can infer
based on an actual argument, as opposed to a set of facts that, contrary to (E), even in a larger sample group, a higher
or assertions. Actually, what we have here is an argument percentage of smaller birds would survive.
inside an argument: the author assembles evidence to rebut a
scientist’s conclusion. In any event, the mandate is the same 63. (C)
as in any Inference question: to locate a statement that must This Inference question begins with a major turning point
be true based on the stimulus. in the cultural life of Country Y: All books legally published
Let’s start with Yakusawa, whose conclusion that size helps there after 1712 received a censor’s approval. Then the
to determine a blackbird’s survival chances is based on his experiences of Country Y’s first two censors are compared.
month-long study, during which a higher percentage of smaller Censor #1 disapproved 50% of the books submitted to him.
birds survived than larger birds. That’s a correlation, and Censor #2 disapproved only 25% of the books he received,
Yakusawa is clearly using that correlation to reason towards and yet (a hint of a paradox here) both censors approved the
partial cause and effect: the birds’ size, he believes, had same number of books. Well, the only way this can be true is
something to do with whether they lived or died. To rebut if Censor #1 simply had more books submitted to him—and
this reasoning, our author points out that smaller birds—the that’s exactly what correct choice (C) says. This is the rare
ones who had a higher survival rate—are generally younger Inference question whose answer you might actually have pre-
than larger birds. The author is implying, in other words, that phrased, if you had sensed the seeming contradiction in the
it’s not size but age that is the survival determinant: that stimulus’ last two sentences. Still unclear? Pick numbers. The
younger birds simply tend to live longer than older birds, size number of books approved under each censor was the same,
notwithstanding. the last clause states, so say that number was 60. Those 60
books represent 75% of the books submitted to Censor #2
All we need to do is compare each of the choices to the
(remember, he disapproved 25%), so the total submitted to
statements just analyzed. Since the entire stimulus is
Censor #2 was 80. Now: Censor #1 also approved 60 books,
composed of two arguments that disagree, chances are the
but that represented 50% of his total. So, 120 books must
right answer will be rather narrow in scope: it has to follow
have been submitted to him—more than Censor #2. No matter
from the stimulus, and cannot disagree with it.
which number you pick, it works out that Censor #1 must have
(A) is flatly contradicted by the stimulus. Since smaller birds reviewed more books.
tend to be younger, and smaller birds lived longer, the traits of
(A) We are given no pre-1712 data whatsoever, so we cannot
smallness and youth certainly seem to have correlated in that
conclude that the naming of the censor in that year saw a
study, and rather well at that.
change in the number of critical books published, let alone
(B) twists the implication of the age factor that our author whether that change was + or –.
introduces. His simple suggestion that birds’ survival risk is
(B) is impossible given the conditions in the stimulus. Given
based more on age than on size cannot yield (B)’s conclusion
the numbers we picked above, Censor #1 prohibited 60 books,
that given two birds of the same age, their size will determine
while Censor #2 prohibited 20. That disparity will always be
their survival chances. Indeed, the author seems to want to
the case.
take the size issue off the table altogether.
(D) All we are playing in the stimulus is a numbers game:
(C) Au contraire, (C) is a sentiment with which the author
percentages and numbers of approved and disapproved
would categorically disagree. Age differences do determine
books. Any disparity between numbers can be explained by
survival chances; that’s his whole point. And since we’re
any number of factors, (D)’s being only one of many. This
asked to make an inference from the author’s argument, that
conclusion is far from inferable.
which contradicts the argument cannot be correct.
(E) subtly shifts the scope from number of manuscripts to
(D) Here’s what we want: something narrow in scope and easy
number of writers. While we do know that the number of
to swallow (no pun intended; these are blackbirds). Reflecting
rejected manuscripts was greater under Censor #1, we don’t
the author’s view that it’s age rather than size that has an
know the number of writers attached to these.
impact on survival, (D) is right in asserting that when two birds
are the same age, their size difference may not mean that they
64. (A)
have different survival chances (and notice how qualified that
“may not” is; it allows for exceptions, and hence is easier to Another Inference question; another set of assertions rather
accept as true). than an argument per se. The nonprofit Green Ensemble,
which gets its major funding from corporate contributors,
would have folded this year had any corporate sponsors (A) , for example, makes an accusation that can’t be supported,
bowed out. Instead, the Ensemble has lasted out the year. since the author says nothing about whether Hypatia’s new
From those two facts alone, we can deduce (A), in simple if/ novel has any “focus on the significant” whatsoever. Yes, the
then + contrapositive form: book has maturity, complexity, and grace. What’s its focus? No
If any corporate sponsors had withdrawn their financial help, way to tell.
the company would not have survived. (B) “Grace,” as a concept, is mentioned as a property that
Hypatia’s early work lacks and that her new novel possesses.
The company has survived; therefore—? Choice (A).
But we cannot apply it to modern literature in general as (B)
(B) No information is provided on next year’s funding. All we’re would have it, because the author fails to do so.
told is that the Ensemble has announced next year’s schedule,
which may be sheer optimism, or may be based on their (C) , (D) All the author is committed to is the tentative judgment

current level of corporate support, or may as (B) says hinge on that Hypatia’s new novel is “promising,” and the affirmation
other financial sources. Or maybe not. No way to tell. that it has met one necessary condition of good literature:
It’s “complex,” a synonym for “intricate.” We cannot leap to
(C) is not necessarily true. This year’s corporate funding
either (C)’s or (D)’s comparison of the book’s quality relative
could’ve been stable year-round.
to the modern norm. Each choice is way too extreme and
(D) Where does this “one-half” figure come from? Nothing in unsupported.
the stimulus even remotely refers to any such statistic.
(E) is all that’s left, and it is indeed inferable. Since “intricacy”
(E) Groups other than the Green Ensemble are wholly outside and “complexity” are synonyms, then that is the “property of
this argument’s scope. good literature” that Hypatia’s novel, at minimum, possesses.

65. (E) 67. (B)


Very clever, and hopefully you were clever in handling it. All Consider the relationship between the two sentences in the
you had to do was ask yourself the right question to test each stimulus, and tie them together: pencils were made from
choice; and the relevant question is: Does this word signify Cumberland graphite up until the 1790’s. Since the war dried
something being done to someone, or some action one takes up the Cumberland graphite supply, the French government
oneself? In order to maintain the “force and focus” of the sponsored research that led to the use of powdered graphite
argument, the word we choose to replace “absentee” must and, ultimately, modern methods of pencilmaking. Fine. Now
function exactly as that word does; that is, it must denote an we must confront each choice in turn.
action one takes oneself. Let’s scan the list:
(A) has two problems. For one thing, it’s written in present
(A) An “honoree” is honored by someone or some group. tense, whereas the stimulus discusses only the availability
(B) An “appointee” is appointed by someone or some group. of Cumberland graphite prior to the 19th century; for all we
(C) A “nominee” is nominated by someone or some group. know, pencil makers later discovered other sources that are
in use today. The other problem is its assertion that graphite
(D) A “transferee” is transferred by someone or some group.
of all types was solely based in Cumberland. Well, where did
(E) An “escapee” is not escaped by someone or some group. the powdered graphite that French researchers were studying
There you have it—(E) it is. come from? It could’ve been powder left over from using the
Cumberland solid graphite, true, but it’s just as likely that
66. (E) sources of powdered graphite were found elsewhere. In the
This Inference question starts with an assertion that lots end, then, (A) is too far-off topic and too broad to be inferable.
of today’s literature is poor. How come? The work lacks (B) makes sense. If, during the time of the war with Britain,
“intricacy” and a “focus on the significant,” because of France had known of a source of solid graphite other than
today’s authors’ inexperience. The implication is that those Cumberland, why would the government have sponsored
two traits are necessary for bringing about good literature. research into the use of the powdered form? Why wouldn’t
By contrast, the author finds the newest novel by Hypatia to they have just purchased, and used, the solid graphite from
be “promising,” because it is more mature, complex, and the alternative source? That the research began when the
graceful than her earlier work. As we compare the choices Cumberland graphite dried up strongly supports the idea that
to the stimulus in search of that which must be true, the key no alternatives were available.
is to recognize the scope shifts that the author commits. In
(C) Hah! There’s no reason to suspect that Britain and France
sentence 2, he is not comparing Hypatia’s works to that of
went to war over pencils.
other modern authors, but rather comparing her new novel
to her earlier stuff. And the terms of sentences 1 and 2 don’t
match up precisely.
(D) is a general statement about the “frequent” value (D) commits the same (one might add, classic) scope error
of government research, way too broad a claim to be as does (A). A description of a process in no sense implies
supported by this one example. Besides which, it’s hard ways of reversing or changing that process. Beyond that,
to justify deeming an alternative way of making pencils an “nondietary” factors are never brought up.
“invention . . . of great benefit.” (E) For all we know, people who are consuming fat and
(E) “Even today”? Come now. The passage evidence takes us cholesterol at lower levels than the threshold can further
no later than the 1790’s. That era saw the origin of “modern decrease their serum cholesterol levels by consuming
methods,” yes, but we’re told nothing more about those even less. After all, below the threshold, serum cholesterol
methods. increases and decreases are proportional to intake, so there’s
no reason to believe that such people couldn’t reduce their
68. (C) cholesterol level as (E) maintains.
Here’s one more “non-argument”: that is, a set of assertions
and claims from which we are to draw an inference. Our best 69. (D)
bet is to make our way through the individual sentences, This passage, like many stimuli for Inference questions,
understanding the gist of each; and be sure that you don’t get represents not an argument per se but rather a set of related
bogged down in author jargon: claims or assertions. We’re told that in prehistoric times, both
Sentence 1: Serum (blood) cholesterol levels are affected by halves of the Levant had the same climate but suffered a very
how much cholesterol and fat one eats. different fate: The north stayed heavily populated, while the
south was abandoned 6,000 years ago. What explains this
Sentence 2: As you eat more cholesterol and fat, your serum
oddity? Well, we’re told to take the archaeologists’ hypothesis
cholesterol level rises at the very same pace, but only up to a
as true: A clearing of the forests in the south led to economic
point. At that certain “threshold” point, you can go on to eat
ruin, which in turn led to the disappearing (presumably
a ton of cholesterol and fat and yet you’ll only see slow serum
migrating) population. Fair enough.
cholesterol increases. Interesting, huh?
Since the correct answer is that which “CANNOT be true,” it
Sentence 3 spells out what the threshold is: 1/4 of the
follows that the four wrong choices can or must be true. (And
cholesterol and fat level in the average American diet. In other
therefore, the four wrong choices need not be deducible
words, once you have eaten 25% of the average American’s
from the stimulus, as long as they’re consistent with it.) You
level of cholesterol and fat (at the same time experiencing
may find it easiest to look for the right answer—that which is
a proportional increase in serum cholesterol), then things
impossible—directly; but you always have the option to seek
change: From that point on, you can eat lots more of the stuff
out the choices that can or must contain truth, and whittle
with less effect on your blood. Now let’s compare these facts
them down that way instead:
with the allegations in the choices:
(A) That up until 6,000 years ago the fauna of the southern
(A) misunderstands the purpose. The stimulus has gone to
Levant fed on forest flora is eminently consistent with the
great pains to describe a process, but never explores how that
stimulus. As written, the choice implies the strong economic
process might be altered or reversed. Anyway, the threshold
importance of those trees, the ones that the archaeologists
point has to do with the action of cholesterol and fat on
believe somehow disappeared in a way that led to the ultimate
people’s blood in general, and is independent of what any
abandonment of the area.
individual eats.
(B) is a different, but equally consistent, assertion of the
(B) Quite the contrary. If you consume the average American
economic importance of trees prior to 6,000 years ago. The
diet, then you are eating 100% of its cholesterol and fat. But
disappearance of the trees that made a major contribution to
once you passed the 25% mark, you stopped seeing dramatic
building might well cause an economic collapse.
increases in your serum cholesterol level. So, the stimulus
contradicts (B)’s apocalyptic announcement. (C) speaks to the different relative fates of the north and
south. That the north maintained its tree species while the
(C) If, as (C) says, you’ve eaten one-half of the cholesterol-
south was being abandoned is entirely consistent with the
and-fat content in the average American diet, then you’re
chain of events that the archaeologists posit. Assuming the
way past the threshold beyond which serum cholesterol level
importance of trees to the economy (and we have no reason
rises at the same rate as consumption. Therefore, while the
to doubt that), (C)’s fact helps to explain why the north kept
people in (C) have doubled (1/4 times two) their cholesterol
its population while the south lost its people. In any event,
and fat consumption, they’re at the point where their serum
(C) can be true in light of the stimulus, and that’s all we need
cholesterol level is going up only “gradually.” Inferably,
in order to reject it.
consumption has outpaced the rise in serum cholesterol as
the second sentence describes. (C) is correct.
(D) If indeed the south was treeless prior to 6,000 years a natural disaster. We know this because past estimates of
ago, then maybe it did suffer an economic collapse the population level necessary for survival were low, and we
leading to depopulation, but not for the reason cited by the barely even have that number. Therefore, we can conclude
archaeologists: Remember, their idea is that deforestation that at least in the short term, the wild cheetah population
was the first step in the process, and that couldn’t be so if the will be incapable of surviving a natural disaster in the African
southern Levant was treeless before it was abandoned. Since grasslands, choice (D).
(D) directly contradicts a central tenet of the hypothesis that (A) Read carefully: the estimate of the size of the population
we’re to take as true, it must be the right answer here. For the (the actual number of cheetahs) wasn’t inaccurate—the
record: estimate of the size needed to survive a natural disaster was
(E), for all its gabbiness, simply and essentially does what (A) inaccurate. An importance difference.
and (B) do: provide evidence that trees were economically (B) Although we know that there isn’t enough grassland to
valuable prior to 6,000 years ago. That (again) would be support more cheetahs now, we don’t know that their habitat
consistent with, and indeed would help to explain, why the is decreasing. Similarly, we know that we have too few
southern Levant collapsed after deforestation. cheetahs, but we can’t conclude from this that their population
is decreasing. And even if we could infer that the habitat and
70. (D) cheetah population were both decreasing, we would still need
A question stem that asks for a “most strongly supported more information to be able to conclude anything about their
conclusion” is asking for an inference. So, we need to attack relative rate of decrease.
the stimulus looking to combine statements to find what must (C) Hunting? Pollution? The scope of this argument is very
be true. All people prefer colors that are easy to tell apart. narrow; the author is only concerned with natural disasters
Babies have an easy time distinguishing bright colors but and never compares the severity of this threat to any others.
a hard time distinguishing subtle shades. Since babies are
(E) The author isn’t concerned with predicting the locations or
people, we can now conclude that babies prefer bright colors
likelihood of natural disasters—she’s concerned only with the
to subtle shades. Finally, we learn that brightly colored toys for
cheetah’s chances of surviving any natural disaster that might
infants sell better than their shady counterparts. Since babies
occur.
prefer bright colors, there must be some connection between
the opinions of babies and toy sales. Presumably, the infants
72. (A)
aren’t buying the toys, but we can conclude, as (D) does,
that toy sales reflect the preferences of babies, at least with We’re looking for a statement with which John must agree,
respect to the “bright color vs. subtle shade” distinction. which is another way of asking for an inference. John made
two claims. First, he maintains that great writers do not need
(A) Primary colors? Secondary colors? These terms come out of
diversity in subject matter. Second, he claims that great writers
nowhere, so we can’t infer anything about them.
do need to be able to explore a theme in-depth. As is often the
(B) goes too far. Yes, color does play a role, but there may be case with Inference questions, forming a prephrase is difficult,
other more important factors at play (so to speak). so there’s little to do except try out the choices. Choice (A)
(C) is too specific. The stimulus refers to bright colors in paraphrases John’s first claim: since John believes that great
general, and not individual colors in that category. writers do not need variety in subject matter, he must also
(E) might be the case, but we don’t know why toy sales are the believe that a writer without such variety could still be a great
way they are. Maybe toy manufacturers used some other kind writer.
of market research, or maybe they did no research at all. (B) Au contraire, John believes that great writers must be able
to explore a theme deeply, so Favilla’s failure to do so would
71. (D) disqualify her from greatness.
We’re looking for a conclusion that can be derived from (C) confuses necessity and sufficiency. John believes that
the argument, so let’s follow the line of reasoning to see exploring a theme deeply is necessary for greatness, but he
where it leads. The author tells us three things: 1) previous might not think it is sufficient. For all we know, John might
estimates of the number of cheetahs needed to survive a impose other requirements.
natural disaster were too low; 2) we barely have that number (D) John doesn’t believe that variety should be required, but
of cheetahs; and (3), there isn’t enough grassland right now that doesn’t mean that John believes that variety in subject
for more cheetahs. We’re interested in where the author is matter is a disadvantage.
going with all this evidence, so let’s piece it together: since,
(E) scrambles some terms in Muriel’s argument. John never
for the time being, at least, the grassland cannot support any
claims that distinctiveness is necessary for greatness.
more cheetahs, there are currently too few cheetahs to survive
73. (A) that the overall amount of jet fuel decreased during the year.
Don’t be alarmed by the lengthy stem—it simply refers to the Yet the second sentence says that there’s actually more jet
type of field described in the stimulus. We’re asked to identify fuel available than last year. This forces one to realize that
the statement that is supported by what we learn about such the first sentence didn’t in fact say that the amount of jet
a field, so it is important to keep the facts straight from the fuel was down, but rather that the supply decreased relative
beginning. We’re told that pigweed seeds rest in the soil to the demand. Well, since actual supply is up, the only way
in the dark until plowing occurs in spring. At that time, they that supply could have decreased relative to demand is if the
get moved up near the surface and are exposed briefly to demand is also up. That’s the only way both sentences of the
sunlight. After this brief exposure to the sunlight, the seeds passage can exist without contradiction. Demand is up, choice
germinate. It is important to note that the seeds actually need (A) gets the point.
the months of darkness because during that time they become (B) The stimulus said that the price increase was purely a
highly sensitive to light, making them more receptive to the matter of supply and demand. There’s nothing that allows us
sun later. It is this fact that allows us to infer (A), that plowing to conclude anything about advances in engine technology.
at night would result in fewer pigweed plants than would (C) is also outside the scope; nothing in the stimulus says
plowing during the day. Since the seeds need the sunlight to anything about the number of flights made by jet airlines.
germinate, plowing during the day would ensure that as many
(D) and (E) focus on a supposed component of jet fuel,
seeds as possible got exposure to sunlight. Night plowing,
petroleum, about which we know absolutely nothing. Thus,
however, would prevent the seeds from receiving sunlight, and
there’s no way for us to conclude anything about petroleum
we would expect less exposure to sunlight to result in fewer, or
from the statements in the passage.
even no, germinations.
(B) We don’t have enough information to make the comparison 75. (E)
in (B). For all we know, no plowing and night plowing equally
Combining the first two statements, we can deduce that John,
deny the seeds sunlight, in which case we would expect no
Mary, and Teresa must have signed up for the beginners’
plants in either case.
tennis clinic. Since, according to the third sentence, there is
(C) The seeds in a field plowed just after sunset ostensibly no possible overlap between accomplished players and those
couldn’t get more sunlight than the seeds in a field plowed signing up for such a clinic, Mary, and Teresa (and anyone else
just before. So, if anything, this choice has it backwards; we in that waiting room) can’t be accomplished players. (E) sees
would expect more seeds from the “before sunrise” field. fit to mention only two of those folks, but that doesn’t make
However, it’s still hard to know what to make of (C), because (E) any less true.
plowing just before sunrise technically could even deny these
(A) goes too far. They could have played tennis before, but still
seeds any sunlight. Who knows how long the seeds remain
not be accomplished.
above ground before they’re deposited back below? What
if it’s only one second? “Just before” sunrise is ambiguous; (B) “Only” is what kills this choice. Who knows what other
it doesn’t tell us how much before sunrise. No need to get classes or clinics these folks have signed up for, beyond the
bogged down with the ambiguities, however. No matter how beginners’ tennis clinic?
we interpret it, (C) just isn’t inferable. (C) , (D) Any number of other people beyond the three
(D) We do not know, because the argument does not tell us, mentioned could have been sitting there (D) and hence could
that the seeds won’t germinate at all unless they settle just have signed up for the clinic (C).
under the surface. We’re told only what happens when the
seeds are redeposited after exposure—they grow. But this 76. (C)
doesn’t exclude the possibility that they might germinate if The wording of this difficult stimulus is somewhat formal in
they settle on top of the soil. nature; and so, choosing a formal statement or two isn’t a bad
(E) 180: The stimulus tells us that the seeds need a period of place to start—the last two sentences, for instance. We’re
underground darkness to become receptive to the sunlight. told that a sinking, only partially flooded ship will implode;
Presumably, if seeds are on the surface the whole time, they then we’re told that the Rienzi didn’t do so. See the basis of a
will not become receptive to the light, will not respond to the contrapositive here? “If partially flooded, then implode; since
sun, and will not germinate. not implode . . .” It must be true, then, that the Rienzi was in
fact fully flooded. Otherwise, it would have imploded. Now
74. (A) back up a bit. Sabotage can achieve full flooding, and the
Rienzi could have blown up in that way. But if it wasn’t blown
The sooner you pick up on the crucial word here, the better.
up by sabotage—and here comes correct choice (C)—then how
Which word is it? The word “relative.” If you miss this word,
in the world could it have sunk fully flooded? Only one way:
you’d possibly get the impression from the first sentence
it must have flooded with unusual quickness. The second (E) Whatever effectiveness a political interest group may have
sentence says that, ordinarily, water doesn’t enter a rapidly- at its inception, it is excessive growth, not the mere passage
sinking ship fast enough to flood it fully. But fully-flooded the of time, which will cause a loss of effectiveness, according to
Rienzi was, so if (as (C) says) sabotage is ruled out, then water this author.
must have entered the ship with unusual rapidity. That’s the
only explanation. 78. (A)
Confirming that the wrong choices need not be true may be The most modest answer choice is often a valid inference.
easier than the explanation of (C), above: The stimulus tells us that A. robustus bones contained a lower
(A) and (E) The method or intent of the ship’s construction is ratio of strontium to calcium than did bones from H. erectus.
never even alluded to; hence it’s totally irrelevant to the issue Further, the lower the ratio, the more meat the hominid ate.
of how and why the ship sank as it did. Finally, we are told that H. erectus is known to have been a
(B) When the ship became fully flooded isn’t the issue; it’s meat eater. Answer choice (A) makes the very modest claim
how it could have become fully flooded, given that it didn’t that the diet of A. robustus included at least some meat. Given
implode. what we have been told, A. robustus must have eaten more
meat than H. erectus, and so answer choice (A) must be true.
(D) Yes, the Rienzi sank rapidly and didn’t implode. But we
cannot deduce that, therefore, had it sunk slowly it would have (B) and (C) Don’t fall into the trap of assuming that just
imploded. Implosion—insofar as we’re told—is a matter of the because two things are correlated, one must have caused the
degree to which the ship has flooded; there’s no mention of other. Nothing in the stimulus tells us exactly how diet and
sinking speed as a condition of implosion. So (D) makes an strontium–calcium ratios are related, so we have no way of
unsupported connection. knowing whether the meat consumed by H. erectus contained
more strontium than that consumed by A. robustus (B) or
77. (D) whether the diet of H. erectus contained more calcium than
that of A. robustus (C).
When the stimulus contains a series of formal logic
statements, look for ways the statements can be combined. (D) Again, with the diet! Not only do we not have any basis for
tying the calcium and strontium levels in these hominids to
This wordy stimulus boils down to a simple chain of logic:
anything in their diets, but also, whether or not the strontium-
if a political interest group gets too large, then conflicting
to-calcium ratio is a fraction greater than or less than one
economic interests of the members will surface. If conflicting
makes absolutely no difference to the information we have
interests surface, then the group cannot unite behind a
been given in this stimulus.
common program. If the group possesses political impact
necessary to influence legislation, then it must be united. The (E) The stimulus neither says nor implies anything about
first two claims blend easily—if a political interest group gets the fossilization process affecting the ratio of strontium to
too large, then the group cannot be united. How can we bring calcium. This answer choice goes far beyond the scope of the
in the third claim? stimulus.

The contrapositive, formal logic’s knight in shining armor,


79. (C)
comes to the rescue. The contrapositive of the third claim is
that if a political interest group cannot be united, then it will The right answer choice for an Inference question must
not be able to influence legislation. Now we can combine all be true.
three statements to see that if a political interest group gets The stimulus tells us that the frozen man lived at least 4,000
too large, it will not be able to influence legislation. Answer years ago (based on artifacts found with his body), and the
choice (D) nicely paraphrases this combination of the three nature of the glacier indicates that the man died on the spot
claims. where his body was discovered. We are also told that had
(A) is an irrelevant comparison. According to our speaker, the glacier not frozen the body shortly after the man died, or
if the group becomes too large, it will not be influential. No had the glacier thawed before this discovery, the body would
distinction is made between groups that are expanding versus not have been preserved. Put it all together, and that glacier
those that are numerically stable. must be at least as old as the body, that’s at least 4,000 years
old. (C)
(B) A society’s effective functioning is outside the scope of the
author’s argument. (A) The stimulus says that the artifacts found on the body are
at least 4,000 years old. That is entirely consistent with some
(C) The author says nothing about what happens to politicians
of the artifacts being more than 4,000 years old.
who ignore the economic interests of large groups of people.
This answer choice is not at all supported by the stimulus.
(B) The stimulus makes no mention of the man’s cause of (D) It’s possible that there could be equal numbers of male
death, so we cannot rule out the possibility that he froze to and female broods, with fewer males being born than females
death shortly before the glacier encased his body. are hatched, but the greater proportion of males surviving to
(D) The fact that the body would not have been preserved had adulthood then balances out the proportion of male to female
the glacier thawed prior to the present time is good enough adult thrips. However, this need not be true.
to guarantee that the glacier is at least 4,000 years old. It is (E) It’s possible that there are some females who use only one
not necessary for the artifacts to be just as perishable as the of the two methods of reproduction over the course of their
man’s body. lives, but this need not be true.
(E) While this answer choice may be true in the real world,
nothing in the stimulus requires global warming as compared 81. (D)
to 4,000 years ago. So long as the climate in the area around In Formal Logic questions, beware of answer choices that
the glacier did not warm sufficiently to thaw the glacier before confuse necessity and sufficiency.
the discovery of the body, it does not matter whether the You may not have immediately noticed the Formal Logic in
average global climate is warmer or cooler or the same as it this question, since it does not appear in the classic if/then
was 4,000 years ago. formulation. But you should learn to recognize the language of
necessity: anything that “must” happen in order for something
80. (C) else to occur is the result in a Formal Logic statement. Thus,
Critical reading skills are especially important in Inference we can translate the first sentence of this stimulus as, “If an
questions. artwork is great, then that artwork expresses a deep emotion.”
Based on the stimulus, we know that a particular species of Remember to form the contrapositive: “If an artwork does not
thrips can reproduce either by laying eggs or by bearing live express a deep emotion, then it is not a great work of art.”
young, although any particular brood of young will be either The double negative in the second sentence cancels itself
hatched from eggs or born live. All offspring hatched from out, so we get, “If an artwork expresses an emotion, then
eggs are female, and all offspring born live are male. Any the artwork’s creator must be capable of experiencing that
particular live-born brood will be smaller than any particular emotion.” The contrapositive would state that, “If an artwork’s
brood hatched from eggs. Thus, we can conclude that given creator is incapable of experiencing an emotion, then their
equal numbers of broods, there will be more female than male work cannot express that emotion.”
offspring. As in most Inference questions that involve Formal Logic, we
The author, however, is not done with us. He continues, can combine the statements once we have translated them.
telling us that a larger proportion of male (remember, that’s Here, the stimulus tells us that an artwork’s creator must be
the live-born offspring) than female offspring (hatched from capable of experiencing any emotion expressed by their work,
eggs) survive to adulthood. Interestingly enough, among these and so the creator of a great work of art must be capable of
thrips reaching adulthood, the proportion of males to females experiencing the deep emotion necessary to such an artwork.
is about equal. Let’s use this new statement to evaluate the choices:
The best way to handle such a long, complicated stimulus is (A) was probably tempting, since we think of computers
to tackle each answer choice in order, and ask whether or not as incapable of experiencing emotion. But notice that the
that answer choice is a valid inference (must be true). stimulus only refers to the “capacity to” experience emotion,
(A) The thrips are the first species found that can reproduce by while (A) insists that the actual experience of the emotion is
either egg-laying or live birth. That doesn’t necessarily mean necessary. That’s why (A) does not have to be true, and can be
that they are the only species capable of doing so. eliminated.
(B) The stimulus says that any particular female will use (B) does not have to be true. While we know that great art
only one method of reproducing per brood, but there is no must express deep emotion, the stimulus never makes the
information about whether individual females who reproduce correlation between depth of emotion and greatness of art that
in one way are or are not capable of reproducing in the (B) does. Eliminate.
other way. (C) ’s logic is backward, and so confuses necessity and
(C) If a greater proportion of live born (male) thrips reach sufficiency. The stimulus says that the expression of deep
adulthood than the proportion of hatched (female) thrips, but emotion is a necessary quality of great art, but (C) claims that
the proportion of male to female adult thrips is about equal, it such an expression is sufficient to consider an artwork great.
must be the case that more thrips are hatched than are born Don’t be fooled by the introduction of computers in (D). The
live. (C) must be true based on our stimulus, and it is the choice focuses on computers’ inability to experience emotion
correct answer choice. For the record: and how it relates to their ability to produce great art. Sure, if
a computer cannot experience emotion, then according to the (D) is contradicted by the second sentence’s assertion of the
logic of the stimulus, they cannot create art that expresses any “most significant indicators” of viability, and (E) creates an
emotion, and thus computer-created art cannot be great. (D) unwarranted (and outlandish) comparison among four of the
must be true, and is correct. factors mentioned.
(E) Watch out for the distinction between great artwork and
a great artist. No connection between the two is made in the 84. (B)
stimulus, so (E) is out of scope. Asked and answered. As noted above, by pointing out
Zachary’s assumption, Stephen implies that it’s possible—
82. (D) indeed probable—that Michelangelo added paint to his
Anything outside the scope of an Inference stimulus could be own fresco, and thus stripping away “everything except”
true, or could be false; we don’t know anything about it. his original work “would be unlikely” to achieve the artist’s
intention (B). Notice that the extreme language of the stimulus
Don’t be frightened by the technical language in this Inference
(“everything except”) can be contrasted with the moderate
stimulus. The information it contains is actually pretty simple:
language of the correct answer (“would be unlikely”); that
the troposphere gets colder as you go straight up, and the
contrast makes this answer choice absolutely correct.
stratosphere gets warmer as you go straight up. We also find
out the range of temperature at the top of the troposphere, (A) says it would be impossible to distinguish between the
and the reason why the stratosphere is warmer: ozone. With original fresco and later painting, which is what we’ve called a
those things in mind, let’s evaluate the choices: “could vs. should scope shift.” That it might not be possible to
distinguish original and later work is irrelevant to the question
(A) could be false. The thickness of the troposphere isn’t part
of what should be done to restore the artist’s intent. (C) is a
of the stimulus. We only know the temperatures at the poles 180, in that it would tend to support Zachary’s position that
and the equator, and that it must be warmer underneath those
only the original fresco work is illustrative of original intent. (D)
points. Eliminate.
and (E) are outside the scope, bringing in unimportant issues
(B) and (C) could be false. We know that the temperature of importance and satisfaction, respectively.
increases as you go up in the stratosphere, but we don’t know
how much it increases by the time you get to the top, or to the 85. (C)
middle. It’s possible that the stratosphere over the equator
For a fill-in-the-blank question, be sure to plug in your choice
could warm up more quickly than the stratosphere over the
and read the entire stimulus through, to check whether you’re
poles, which could make up for the difference in temperature
correct.
at the top of the troposphere. Eliminate.
“Compromise” has two different meanings, and each
(D) must be true. If the temperature in the stratosphere
shows up in the politician’s remarks. The opponents
increases as you go straight up, then the temperature at the
seek compromise meaning to work together, each giving
top of the stratosphere must be higher than the temperature at
up a little of their differences toward a common goal.”
the top of the troposphere directly beneath the same point.
Yet in the next sentence the author accuses them of
(E) could be false. We know ozone is responsible for the “compromising principles,” meaning cravenly imperiling or
warming in the stratosphere, so ozone depletion would surrendering. The critic most logically would point out this
probably decrease the air temperature in the stratosphere, not misuse of the verb “compromise” (C), in order to imply that
the other way around. the politician is criticizing his opponents for something that he
has virtually acknowledged as praiseworthy.
83. (C)
“Betray,” (A), is used in the second sense of compromise only.
(A) —Not enough data are provided to make this determination. “Common” goals, (B), are those mutually held, and that’s
If anything, the fact that each of the six economically viable the only sense in which that word is used. We don’t need
countries named has a relatively small population suggests to know what the “principles,” (D),areto recognize that the
that population size may correlate with viability. term is used in only one sense. And the “opponents,” (E),
(B) —No countries, economically viable or not, with a are evidently well-known to the politician and are given no
population greater than 7M are mentioned, so (B) is ambiguity.
impossible to deduce.
(C) —Must be true, since four economically viable countries 86. (A)
are named, each of whose populations is no larger than As usual in an inference question, the stimulus is not an
1/4 of 7M. argument but a mere set of statements, these dealing with
the different ways in which people make their investment
decisions. Also, as usual, not every statement is needed to
lead to the correct answer, and here correct choice (A) comes 88. (E)
out of the first and last sentences only. Most investors don’t Stick close to the text of the stimulus when evaluating
do their own research (that’s sentence 1) but most investors Inference answer choices.
make a profit (that’s sentence 5). Put them together, and it
The study cited in this question is a rarity on the GMAT.
must be true that at least one investor who doesn’t do his own
There is no indication that either the sample population or
research must make a profit; there’s got to be some overlap
the control group were not a representative sample of the
between the two groups.
population—the groups are both large and randomly selected.
(B) ’s conclusion is unwarranted, because we can’t be sure So, we should focus on the findings of the study, and not how
that the majority, as (B) contends, only make use of those two it was conducted. A small percentage of the control group
listed strategies; many investors may use word of mouth or a remembered waking up seemingly paralyzed, but a larger
TV or newspaper recommendation, or any one of a thousand percentage of the experimental group remembered waking up
other sources not mentioned here. And (C), (D), and (E) are seemingly paralyzed with a sense of a strange presence in the
all unwarranted because we don’t know anything about the room. The only difference between the groups is the addition
composition of the group mentioned in sentence 5—those who of the “strange presence” in the experimental group, so that
make a profit. Their specific characteristics go undescribed, difference must somehow account for the larger percentage
except for the fact that most of them do no research. That’s of people recalling such an event. Let’s use this knowledge to
why (A) is right and the others wrong. evaluate the choices:
(A) suggests a causal relationship between the strange
87. (E)
presence and the feeling of paralysis, but it ignores the fact
An answer choice must be true if it combines two or more that the study asked whether the subjects had ever awakened
different pieces of evidence from an Inference stimulus. to such a feeling. By extending the question at hand to any
Like many Inference stimuli, this one doesn’t really have a sense of a strange presence, sleeping or waking, (A) goes
conclusion. It is just a long string of evidence, and our job is to beyond the scope of the argument. Eliminate.
look at the connections within that evidence to find something (B) We have no way of knowing how many people in the
else that must be true. This evidence deals with ants’ behavior control group had awakened with a sense of a strange
in protecting their territory and food against predators. We presence in the room, since they were only asked about
find out that the tactic of swarming invaders can “effectively waking up with a sense of paralysis. Eliminate.
deter” flying insects like wasps, which prey on the riodinid
(C) Perhaps some of the 60% of the group that answered
caterpillars used by some South American ants for food. Let’s
no to the question had awakened with a sense of a strange
use this information to evaluate the choices:
presence in the room but not seemingly paralyzed. By omitting
(A) The secretions produced by riodinid caterpillars are used the paralysis that was mentioned in the stimulus, this choice
for food by South American ants, but that doesn’t necessarily distorts the information we were given. Eliminate.
mean that they are chemically identical to other things that the
(D) There is no reason to think that some of the subjects were
ants use for food. Eliminate.
being inconsistent unless the two groups were composed of
(B) The stimulus makes no comparison between South the same people, which is not the case. Eliminate.
American ants and other ants. Eliminate.
(E) may sound vague at first, but it explains the difference
(C) The stimulus tells us that ants will swarm over insects that between the groups. More people in the first group recalled an
threaten their territory, but (C) refers to any organism and is event (waking up seemingly paralyzed) when circumstances
thus too extreme. Eliminate. accompanying it (a sense of a strange presence in the room)
(D) We know that wasps and ants use the riodinid caterpillars were suggested. (E) must be true, and is correct.
for food, but we can’t be sure that no other organisms use the
same caterpillars for food. Eliminate. 89. (D)
(E) remains, and must be true. We can be sure that riodinid An Inference question doesn’t always ask for what
caterpillars living among ants are less likely to be attacked must be true.
successfully by wasps than caterpillars living elsewhere, since A quick characterization of the choices tells us that the four
the stimulus tells us that ants can effectively deter wasps from wrong ones will all be things that could be true, and that the
threatening their food sources and riodinid caterpillars are correct answer must be false. This means we’re looking for
among the ants’ food sources. (E) is correct. an answer choice that directly contradicts the stimulus in
some way. The first two sentences of the stimulus set up two
different Formal Logic statements: If any prisoner attempts
to escape from Chelas and Stelma’s sector, then they must
leave their stations in pursuit, but unless they are in pursuit industry’s liability. The government says it wishes to protect
of a prisoner, they cannot leave their stations until relieved. the industry against the threat of bankruptcy; there’s nothing
The final sentence of the stimulus gives us a concrete piece to suggests that this is not its true intention. (If it claimed it
of information—at the end of a shift, Chelas had violated the was limiting the industry’s liability to ultimately benefit “the
rules and Stelma had not. We don’t know exactly how the people” in the form of some trickle-down cost savings or some
violation occurred, but we’ll look through the choices to see baloney or other like that, that would be a different story.)
what exactly could (and could not) have happened. (D) Au contraire, the government acts as if unlimited financial
(A) could be true. Perhaps Chelas left his station in violation of liability in the case of an accident would pose a serious threat
the rules before 9 P.M. Eliminate. to the industry—the threat of bankruptcy.
(B) could be true. Stelma could have pursued a prisoner trying (E) The people think, and the government implies, that injury
to escape (and thus left her station). If Chelas did not pursue can result from nuclear accidents. Surely, we couldn’t say
that prisoner, he would have violated the rules. Eliminate. that the nuclear industry’s pocketbook would be the only
(C) could be true. Chelas left his station in violation of the victim in the event of an accident; the threat to life and limb
rules, and Stelma did not. Eliminate. would be serious as well.
(D) must be false, and is correct. If this were the case, both
91. (A)
Chelas and Stelma would have violated the rules. For the
record: This one has a formal logic feel to it. There are two possibilities
for these violators: they should either get jail or re-education.
(E) could be true. Chelas could have left for a reason besides
But re-education works only if it will make them more
pursuing the prisoner—say, if he knew he’d violated the rules
responsible drivers. In other words, if re-education will work,
and was trying to get away.
then it will make them more responsible drivers. However,
it is almost impossible for these drivers to become more
90. (B)
responsible. What follows? Well, if these drivers won’t get
We’re to accept that the government’s position is accurately more responsible, then re-education won’t work. But then the
portrayed, and need to infer what must be true on the basis only option for these drivers is jail. (A) says just that.
of that portrayal. While it’s usually difficult to prephrase
(B) Au contraire, re-education doesn’t look like such a great
answers to Inference questions, perhaps a general sense of
option, according to the last sentence of the stimulus.
contradiction jumped out at you? After all, the government
does appear to be speaking out of both sides of its PR office. (C) Harshness isn’t the issue here; effectiveness is.
On the one hand we have assurances to the public that (D) , (E) Drivers who haven’t committed (or been convicted of) a
nuclear power is perfectly safe. At the same time, however, serious driving-related offense are outside the scope. We don’t
the government is taking an action that seems to be necessary know what should happen to them.
only if injury claims resulting from a nuclear accident can
be sustained. If there’s really no danger of injury, then such 92. (C)
claims can’t be sustained, and no limits to the industry’s The question stem asks you to find the choice that must be
financial liability in case of accidents would be necessary. But false, so the best approach is to read through the stimulus
the government has acted to protect the industry. Evidently, once but be prepared to check back with it when you consult
the government’s pronouncements to the public regarding the the choices. (There’s far too much detail to expect to be able
safety of the plants doesn’t match its own beliefs underlying to handle the choices without checking back.) As it turns out,
its action to limit the nuclear industry’s financial liability. the correct answer comes from the last few sentences. Master
Something doesn’t jibe here. As (B) puts it, the government’s artists never create in order to express their own feelings. Bach
position on nuclear power plants (specifically relating to the is a master artist, so he never created works (chorale preludes
safety issue) is inconsistent. or not) to express his feelings. So (C) is the impossible choice.
(A) The government claims that nuclear power plants are safe, (A) could be true, since we don’t know what Bach’s feelings
but takes an action that suggests that it believes they may were, and his creations provide no evidence of them.
not be entirely safe. The action, however, doesn’t prove the
(B) is right in line with the stimulus, which says that master
safety claim is false; the government itself may not know and
artists such as Bach don’t create in order to express their
may just be hedging its bets by securing the industry against
feelings.
financial liability.
(D) We know that most chorale preludes were written for the
(C) Nothing indicates that the government is being
organ, and most great organ chorale preludes were written
disingenuous regarding its reasons to limit the nuclear
by Bach, but we don’t know anything about the relationship
between Bach’s chorale preludes and Bach’s chorale preludes (B) might be true based on the stimulus, but that’s not good
written for the organ. (D) could be true, since Bach could have enough. No such comparison is ever made explicitly.
written lots of chorale preludes for the banjo, for all we know. (C) No mention of the relative sizes of French and English
(E) Close, but no cigar. We know that most chorale preludes vocabularies is ever made.
were written for the organ, but most great chorale preludes (D) is unknowable. The author never says that English
could have been written for other instruments, irrespective of literature is not simple and clear.
who wrote them.
(E) is unknowable. The author focuses on the influence that
the French language has had on the English language, but
93. (B)
never mentions the effect the respective literatures have had
Not much of an argument here, just a set of assertions on the on each other.
topic of contemporary children’s publishing. Sentence 1 is the
meatier of the two, asserting that the flashy, highly-illustrated 95. (A)
nature of today’s kids’ books reduces their “substance,” which
An inference must reflect the content and tone of the
in turn “leads to books that are short lived [and] trendy . . .”
stimulus.
This cause and effect is summed up neatly in correct choice
(B)—which some students never even got to, because they This chatty, accessible stimulus is straightforward to
were suckered in by (A). Whether or not you were tricked in paraphrase. Scientific laws should be general. But alas,
that way, read on. the laws of social science often (necessarily) use vague
terms. Does the author chastise the social scientist for this
(A) is tempting because it seems to link “flashy” elements to
vagueness? Nope, no criticism, just straight reporting. (A)
the problem of reduced book substance. But notice that (A)’s
nicely sums this all up.
conditions—humor and narrow focus—come out of sentence
2, which simply lists some more changes wrought on kids’ (B) ’s value judgment is simply nowhere to be found.
publishing by the computer age. The author never links (C) , like (B), attributes to the author an opinion that simply
sentence 2’s list to the book-substance issue of sentence 1, so isn’t there.
neither may we. (D) attributes to the author a recommendation that’s never
(C) That kids’ books today are simple and narrow and look made.
flashy doesn’t mean that it’s because kids can’t handle longer (E) contradicts the stimulus. The author states that scientific
and tougher prose. Anyway, the author ascribes the changes laws “should” display virtues of precision and generality, but
in kids’ books not to the kids themselves but to the overall that’s often impossible in the social sciences. So, precision
computer culture. and generality are ideals to strive to, but not a requirement for
(D) Where does the issue of children judging books shine a law to be scientific.
in? (D) has the same problem as (C): As far as we know, the
change is not a reaction to children’s needs or judgments, but 96. (B)
rather to the mandates of a computer-oriented culture. An unusual question that you must stop to translate: Which of
(E) We learn only about major trends in the industry; we don’t the five choices does the stimulus provide LEAST evidence for?
know what makes these books popular. Perhaps the short- It means that the stimulus provides BETTER evidence for four
lived substance-lite stories are popular, perhaps not. Nothing of them. In other words, four of the five choices are supported
in the stimulus supports the assertion in (E). by the paragraph’s evidence.
As we read the stimulus, we see that it makes two distinct
94. (A) points: A young mammal’s playful period coincides with the
An GMAT inference is a statement that must be true based growth of key neural connections; and those play-created
on the stimulus. Watch out for answer choices that mix up connections are indispensable for adult survival. In short, the
the language of the stimulus. author is asserting that a correlation (the first point) is there
to create a necessary condition for survival (the second point).
Remember, there’s no argument to look for in an Inference
So: We are to take each choice in turn as a conclusion, and
question. Simply accept each statement as true. The diversity
ask ourselves, does the stimulus answer the question “Why?”
of English reflects the fact that both Anglo-Saxon and French
Is the stimulus evidence for the conclusion? In four of the five
languages influenced its development. French, on the other
cases, the answer will be yes. Check them in turn:
hand, being a direct descendent of Latin, has a literature that
is simple and clear. (A) asserts that young mammals of prey species bolt away
in mock flight from imaginary predators. Does the stimulus
(A) is a direct paraphrase of the first line, and therefore must
explain why? Sure: This sounds like a kind of playful activity
be true. We’re done. For the record:
that could teach skills that would keep the animal alive when 98. (C)
predators are around. In other words, (A) links youthful play, (C) is correct—the researchers must have stumbled upon the
learned behavior, and adult survival in the way that the Ultimate Sweetness. How come? We’re told that the fewer the
stimulus describes. molecules activating a sweetness receptor, the sweeter the
(B) , however, does not, and is the correct answer. And this substance, and this new discovery activates a receptor with
is just a scope issue, isn’t it? The evidence is about the play just one molecule. How can you have fewer than one? Some
period and neural connections and survival of mammals. other substance might come along and be seen as just as
It has nothing to do with non-mammal species, so it can’t sweet, but not sweeter.
support any notion about them. Bingo. For the record: (A) “Pleasurability” is outside the scope of the claims. Not
(C) asserts that adult mammals who played a lot when everyone may find sweetness pleasurable.
young are more likely to play with their own young. Does the (B) There’s no way to tell whether “any” substance can
paragraph explain that? Sure: Those adults developed, during activate a sweetness receptor. (B) is a distortion of sentence 2,
youthful play, neural connections that increased their survival which states that “any” substance can activate one sweetness
and well-being, and hence are likely to pass that process along receptor at most. That’s the maximum. Some substances may
to their own young. activate none at all.
(D) is almost an exact contrapositive of the original paragraph. (D) Other receptor types are outside the scope of the claims.
Its assertion that “Those who cannot engage in certain This is all about sweetness only.
types of play have certain survival deficiencies” more or less
(E) is a case of the “fallacy of denying the antecedent.” Though
contraposes into “Those without survival deficiency engaged
it’s true that the fewer required molecules, the sweeter the
in the right kind of play”—exactly what the stimulus is
substance, we cannot infer that the more the molecules, the
implying.
less sweet. The whole concept of “bitterness” never comes up
(E) , like (A), provides an example of play (here, the in the stimulus, and on those grounds you could have tossed
“inoffensive . . . motions and actions”) that can readily translate (E) right away.
into useful adult skills in the way that the stimulus describes.
99. (E)
97. (B)
A disquieting situation for people thirty-something on up
The most potent word in the stimulus—potent because we is discussed in this question. A herniated or degenerated
should always be alert to its use during the GMAT—is “only,” spinal disk is cited as the cause of most chronic back pain,
as in “only recently became commercially feasible when.” but in most cases, the injury occurs years before the chronic
“Only,” as we know, always signifies necessity, and can be symptoms develop. In fact, roughly one in every five people
read in context here to mean that a necessary condition of the over the age of 30 has such an injury yet shows no chronic
commercial feasibility of these special green and brown cotton symptoms. If pain suddenly develops in such a case, the
fibers is that they “be spun by machine.” Hand-spinning author says that it’s generally because insufficient exercise
fails to meet that necessary condition and thus must not be weakens certain muscles. You may not be able to prephrase
commercially viable. And that’s exactly what (B) is saying. a perfect inference; but if you’re over 30 or approaching it,
(A) We can probably infer from sentence 1 that a short-fibered you might have had the sudden inspiration to hit the gym
green or brown cotton is difficult or impossible to spin by following your mental workout in the Training Library. This
machine. But how does that relate to ecological safety? What thought should help you to recognize the correct choice:
seems to be unsafe is the dyeing process, which has nothing anyone over 30 without any signs of back pain may actually
to do with fiber length. have a herniated or degenerated disk, but since the onset
(C) The notion of ecological safety is related to the dyeing of pain from this injury is generally caused by insufficient
process, not the distinction between hand-spun and machine- exercise, there’s probably a strategy (getting sufficient
spun cotton. Hand-spun cotton that was dyed could be just as exercise) that will delay or possibly even prevent the pain from
dangerous to the environment. coming—choice (E).

(D) Synthetics? Where do they shine in? (D) is out of the scope. (A) is too precise to be inferable. First of all, the “one in
five” figure cited in the stimulus is an estimation, while the
(E) Green and brown cotton that is machine-spun surely
figures in this choice reflect certainties: “...can be sure they
does “avoid the expense of dyeing,” but there’s many a slip
will never .....” Secondly, how do we know there aren’t other
between the raw materials and the finished product, and we
sources of chronic back pain besides those resulting from the
have no idea what the relative costs of garments made with
specific spinal cord injuries cited here? We don’t, and so (A) is
the colored and regular cottons will turn out to be.
not supported by the argument.
(B) is wrong for the same reason as (A): there can be other sunlight it needs (C); and there need not be any other species
causes of chronic back pain, so exercising abdominal and making their way into the grove (D).
spinal muscles regularly can’t guarantee a lack of such (E) sounds a little like (A), but is quite different. Though
pain. Furthermore, the author states that such pain is roughly the same age, the trees in the grove could be many
generally brought on by insufficient exercise, which means different heights, and for many different reasons. Maybe some
it’s still possible for one who exercises these muscles insect blight or root rot causes some of them to have stopped
regularly to develop chronic back pain from a herniated or growing.
degenerated disk.
(C) Note the scope shift: chronic (long term, persistent) back 101. (B)
pain may take a while to develop in many cases after such We’re looking for an inference, a statement that must be true
an injury, but it’s still perfectly possible for many or even all based on the statements in the passage. In such questions,
patients to experience mild or fleeting pain at the time of the answer can come from anywhere in the passage, and
the injury. we may not even know beforehand what part of the passage
(D) From a group of people with herniated or degenerated to specifically focus on. Even isolating the conclusion may
spinal disks, doctors can probably venture a guess as to which not help greatly, since the right answer to a strict Inference
ones are likely to develop chronic back pain based on which question like this could come from a seemingly obscure piece
ones exercise regularly. However, in a general group of people of evidence, or even an irrelevant aside. In this case, correct
without chronic back pain, how is a doctor to know which ones choice (B) is inferable solely from the second sentence: if
(roughly one in five, but even then, only in a group over 30) the feeding methods of jawless fishes were “limited to either
even have the spinal disk problems? Nothing from the passage sucking in surface plankton or sucking in food particles from
suggests (D). bottom mud,” then we can be 100% sure that these jawless
fishes didn’t prey upon other fish. As simple as that—which
100. (A) is not to say it’s obvious. Many test-takers passed on (B)
Picture that grove of mature white pines. They’re real tall because it seemed outside the scope, or just not important
and bushy, and cut off most sunlight below. It’s pretty dark enough. But sure enough, the passage could end with the
down there, and impossible for new white pines to grow (be word “mud” in sentence two and (B) would still be inferable—
“generated”) in the thick dark litter. It is therefore a “fair it may get lost in the verbosity and onslaught of facts in the
bet” that the trees in the grove are approximately the same rest of the passage, but there’s no denying that fish whose
age—indeed, it’s virtually certain that if the ages are different, food choices are LIMITED to sucking EITHER plankton OR mud
the differences aren’t much greater than the time it takes for particles surely aren’t preying on other fish.
a white pine to mature. That’s because a younger tree would (A) The only thing we’re told about jawed-fish in relation to
have to have been generated beneath the mature grove, prey is that they’re able to pursue prey, and use their jaws and
something the author calls impossible. If you still don’t see it, teeth on it. What kind of prey we’re talking about here isn’t
look at it this way. Suppose it takes 10 years for a white pine mentioned, so there’s no way we can infer it’s primarily fish.
to mature. Suppose, too, that the oldest pines in the grove are (C) Why not? How do we know this? We absolutely don’t.
30 years old. That would mean that those oldest trees matured Nowhere does it say that jawless fishes had a monopoly
20 years ago, which in turn would mean that the other trees throughout history on bottom mud particles—teleosts can
could easily be anywhere from 20 to 30 years old: all of those partake too.
trees would have generated at much the same time and
(D) This is simply answer choice (C) in reverse—just because
matured together. However, few if any trees in the grove could
some jawed fishes had or have cartilage as their skeletal
be much younger than 20 years old, because then they’d
material doesn’t mean that jawless fishes did not; maybe they
have to have been generated in the shade of mature trees, a
did.
possibility that (again) the author denies.
(E) No; 400 million years ago is the date given for the first
(B) True, the grove contains only pines. But it’s quite possible
development of biting jaws in fishes, but we have no reason
that the land there once boasted all manner of other trees,
to believe that the entire population of jawless fishes
trees that died in the shade of the mature pines or from some
disappeared instantly on or around that date. That is, nothing
other cause.
in the stimulus precludes the possibility that some classes of
(C) and (D) The author is concerned with whether white pines jawless fishes remained alive for quite some time while jawed
can regenerate in their own shade, not with their life span; yet fished evolved from other classes of fish.
both of these choices point to some sort of pine tree demise.
For all we know, every mature pine in the grove gets all the
102. (A) point that the electric car isn’t a panacea; even if it reduces
This question has a short but dense stimulus. What’s the auto emissions, it could lead to environmental damage
author saying? Some people get their moral standards from from other sources. The end of that last sentence should be
governmental codes of law. These people cannot imagine that something like (A), pointing out the negative environmental
something that is legal could be immoral. In other words, for consequences of the electric car.
these people, if something is legal, then it must be moral. This (B) The argument never links environmental damage to lack of
is one of those formal arguments in casual clothing, and if you popularity, so this choice doesn’t fit what we already know.
remember your if/then statements, this should be no sweat. (C) is true, but it’s a detail from the very beginning of the
So, we need to find a statement that is logically inconsistent argument. The author would have to take us in a circle to make
with the stimulus statement—something that, if people held this the conclusion of his argument, and he doesn’t do so.
both views at the same time, would give them an inconsistent (D) and (E) have the same problem: we know that the electric
outlook. If something is legal, then it is moral, they say. What’s car has some environmental problems that not everyone has
the contrapositive of this statement? If something is not moral, considered, but we don’t know anything about the net effect
then it is not legal. For someone to hold logically consistent of that problem. The argument never forecloses the possibility
views, they must believe the contrapositive of their beliefs as that total emissions will drop or that net environmental
well, right? Well, choice (A) is inconsistent with this belief, and degradation will decrease.
therefore correct, because it implies that it’s not necessarily
true that if an act is immoral, it is illegal, since law doesn’t 104. (B)
cover all immoral circumstances.
Don’t strain to predict an answer in Inference questions,
(B) says that if something is not legal, it’s not moral. While unless the prediction comes easily, just test each choice to
that’s not necessarily true in this context, it’s also not see whether it must be true.
inconsistent, so it can’t be the right answer.
It’s refreshing to hear global warming discussed in terms
(C) says that government officials do illegal things sometimes; of the scientific method, rather than in the political terms
well, government officials are beyond the realm of relevance. that usually accompany the issue. Perhaps that’s why the
(D) talks about laws as the moral consensus of a society, but test-maker chose to include this question. Anyway, the
the stimulus just tells us that people take moral cues from the author points out that the scientific method is designed to
law, not that laws are made based on people’s moral codes. test hypotheses by attempting to prove that they are
(E) is way out there— it’s beyond the scope since the economic incorrect, and that succeeding in disproving a conventional
implications of government regulations mean nothing to us. theory is the most prestigious accomplishment for a
scientist. Thus, it’s not surprising that a number of scientists
103. (A) are attempting to disprove the conventional wisdom
regarding predictions of
“Complete the argument” Inference questions are very closely
global warming—the surprising thing is that none of them have
related to Main Point questions.
yet succeeded.
A glance at the end of this argument shows that we’re trying to
We could make all sorts of predictions for where else the
complete a sentence that begins with the Conclusion Keyword,
argument might go, or what else must be true according to the
“thus.” It should be clear that we need to determine the
author. But our time will be better spent combing through the
conclusion of the argument to answer this question correctly,
answer choices for something that must be true based on the
and that means we should probably think through a variation
statements we already have:
on One Sentence Test: if we were to sum all this up
in one sentence, what would that sentence say? We’ve already (A) There’s no evidence that scientists involved in the global
got the beginning of the sentence, so it shouldn’t be too hard warming debate, on either side, haven’t been acting in
to figure out the end. Let’s look at the earlier parts of the accordance with the accepted standard. This choice is outside
argument to see where they lead. the scope, and is thus incorrect.
First, we learn that the electric car isn’t widely used (B) The stimulus does indeed point out a substantial motive
because of technical problems—but once those problems for scientists to discredit the global warming hypothesis: the
are solved recognition that would come with success in doing so. (B)
and electric cars are popular, auto emissions will drop and the must be true, and is the correct answer. Let’s quickly eliminate
associated environmental degradation will drop along with the last choices:
them. But, the author warns, the power for all those electric (C) is far too extreme—there’s no evidence that global warming
cars doesn’t just come out of thin air. It comes from coal is true, only a lack of evidence that would prove it false.
plants, nuclear plants, and dams, all of which come with their (D) Alternative hypotheses are outside the scope of the
own environmental problems. So, the author leads us to the stimulus, which deals only with the conventional hypothesis.
(E) is also extreme: the stimulus says that a desire for 106. (B)
recognition is a driving force behind global warming Inference questions that ask for the logical conclusion to the
research, but not the primary force. argument are good targets for prediction.
The language gets a little tough here; just take it piece by
105. (D)
piece. Nations aren’t persons. Thus, they don’t have moral
An Inference on the GMAT is something that must be rights and responsibilities. BUT, for a nation to survive,
true based on the information in the stimulus. its citizens have to think it does. All that means that if
As is often the case, this Inference stimulus doesn’t contain a citizens recognize that a nation has no moral rights and
full argument. We simply get a series of facts and have to try to responsibilities, it won’t survive; or, if it’s going to survive,
link them together to find a deduction, or look for an answer citizens have to think (erroneously) that it does have some
choice that restates part of the stimulus. The correct answer moral rights and responsibilities. That’s (B).
will be a statement that must be true based on the information (A) Introduces a new element; the answer choice that
we’re given, and the four wrong answers will all be statements “logically completes” the argument will be based on the
that could be false. evidence provided in the argument. Eliminate.
The stimulus itself is a series of disheartening facts about (C) Draws a conclusion from one statement within the
the news media and local politics. The news media doesn’t argument; we’re specifically asked here for the logical
cover local politics, and local politics are conducted in conclusion to the argument itself. Eliminate.
secret; these two factors each work to isolate local politicians
(D) Outside the scope; the argument isn’t about value
from the electorate. This isolation reduces the chances that
judgments.
residents can influence local politics, which discourages their
participation, which (we can guess) leads to a vicious circle (E) The “always” in this answer choice is too Extreme—it also
where there is less and less participation in democracy. As takes us beyond the scope of the argument, which is about
soon as we’re finished mourning the death of the republic, one particular belief.
we’ll work our way through the choices:
107. (C)
(A) goes too far. Perhaps, if politicians were less isolated,
resident participation would be more likely to elicit a response When confronted with a Formal Logic question, translate and
than it is now, but we can’t be sure that the response would combine the statements sentence by sentence.
be likely. The first three sentences are pretty straightforward, if you
(B) adds an opinion to the stimulus that isn’t really present. break them down one at a time:
The author might agree with this statement, but for all we If the cost of coffee beans continues to increase, the Coffee
know, he might also think that less resident participation is a Shoppe will have to raise prices. (If CB up → P up)
good thing! Without any statements one way or another, we If the Coffee Shoppe has to raise prices, either it will add non-
just can’t be sure. coffee products or coffee sales will decline. (If P up → NC or
(C) is too extreme. A positive official response is clearly a CS down)
factor in resident participation, but it need not be the most If it adds non-coffee products, profitability will decrease.
important factor. (If NC → Pr down)
(D) The rarity of media coverage of local politics is cited as a The last sentence takes a bit more work, but the investment is
factor that isolates local politicians from the electorate, and worth the payoff:
this isolation discourages resident participation . . . so adding
The Coffee Shoppe can avoid a decrease in profitability only if
more coverage of local politics would reduce the isolation,
coffee sales do not decrease.
which would reduce the discouragement. (D) ties together
different pieces of the argument in a way that logically follows, That means: profitability will not decrease only if coffee sales
so it is correct. do not decrease. Translate “only if” into “then,” and we get,
“if profitability does not decrease, then no coffee sales will not
(E) We can quickly eliminate this last choice as extreme.
decrease.” (If Pr not down → CS not down)
Isolation from the electorate discourages participation; there’s
no evidence that if we reduced the discouragement, the The contrapositive of this last statement (If CS down →
isolation would also drop. Pr down): if coffee sales decrease, then profitability will
decrease too.
Combine all these statements, and it looks like the Coffee
Shoppe is in trouble; if the price of coffee beans keeps going
up, it will have to either add non-coffee products or increase
the price of coffee, and either way, its overall profitability will 109. (E)
drop. That’s (C). Let’s test this against the choices: Every GMAT question comes with one correct answer
(A) is a classic reversal. We know that if coffee bean prices choice and four terrible ones.
continue to increase, profitability will decline, but the reverse Don’t be fooled by a stimulus that asks you which answer
is not necessarily true. Eliminate. choice is “most strongly supported.” The GMAT will never give
(B) Likewise, we know that certain events will trigger a decline you a set of answer choices where one choice is very strongly
in profitability, but nothing here precludes other causes. supported, another choice is somewhat supported, two more
(D) is all right except that it says “decrease” where it should of the choices have a little bit of support, and the last choice
say “increase.” This is a trap for the careless reader. isn’t supported at all. The correct answer to any Inference
question is supported by the stimulus (it must be true) and the
(E) Coffee sales increasing isn’t referenced at all in the
incorrect choices are not supported by the stimulus at all (they
stimulus, nor is it the necessary result of anything we’re given.
could be or must be false).
108. (D) This stimulus gives us an insight into the cockpits of newer
and older commercial airplanes. The older planes were
When working with formal logic, always start with the most
designed so that all of the crew members could immediately
concrete, definite statements; save the more slippery ones for
view any changes in the controls made by any one of the crew
later.
members, but the newer planes are designed differently. In the
Starting with the most concrete statements: If everyone who new planes, it is more difficult for the entire crew to see one
works 18+ hours a day has no time for leisure activities (that’s member’s changes to the controls, so the flight crews have to
the second clause, rewritten in positive terms), and if, clause talk to each other more often about the control changes they
3, all happy entrepreneurs have time for leisure activities, make. Using that information, we can evaluate the choices to
then there are two groups that cannot overlap: those working see which one must be true.
18+ hours a day, and happy entrepreneurs. That impossible
(A) sounds good up until it says the frequency of verbal
overlap makes (D) correct for this “could be true EXCEPT”
communication depends on “how long it takes to perform
question. There cannot be any of the type (D) describes,
those changes” in the flight control settings. The length of
because all the happy entrepreneurs have time for leisure
time it takes to make the changes is outside the scope of the
activities, while all of those working 18+ hours a day do not.
argument. Eliminate.
Clause 1, the “most” statement, offers the fact that a majority
(B) Calling verbal exchanges of information “the most valuable
of successful (a new term!) entrepreneurs work 18+ hours
means available for performing cross-checks” is much more
a day. This isn’t very precise, because it leaves room for all
extreme than the stimulus. Just because a certain course of
sorts of successful entrepreneurs, though in the minority, to
action is necessary doesn’t mean it’s the best course of action
work many fewer hours than 18, and have time for leisure
available. Eliminate.
activities and be happy, to boot. What this first clause most
accomplishes is setting up the wrong choices. (C) is also more extreme than the stimulus. There’s no
evidence that crews in older airplanes had absolutely no
(A) could be true, because nothing stands in the way of a 1:1
need to discuss flight control changes, only that crews of new
identity between those working 18+ hours a day and those
airplanes must discuss such changes “more frequently” than
with no time for leisure activities. Those two groups could be
crews of older airplanes. Eliminate.
one and the same.
(D) The stimulus says that flight control changes in recently
(B) actually has to be true. (B) is speaking of people who,
manufactured aircraft “are harder to observe,” not that they
according to clause 1, are in the majority of successful
are impossible to observe. Once again, (D) is more extreme
entrepreneurs—namely those who work 18+ hours a day.
than the stimulus and can be eliminated.
(C) could be true: there could be entrepreneurs who are
(E) Finally, something that must be true based on the stimulus.
both happy and successful. They would be part of clause 1’s
The “other means for performing cross-checks” in older aircraft
(implied) minority: those entrepreneurs who are successful yet
was the control panel design, which allowed each member
work less than 18 hours a day (which leaves them available for
of the flight crew to see another member’s changes to the
the leisure activities that correlate with happiness).
controls. That “routine” means for performing cross-checks
(E), even more than (C), speaks of clause 1’s minority. While was removed in the newer aircraft, leading to an increase in
most successful entrepreneurs work 18+ hours a day, some verbal communication as an alternative. (E) is correct.
certainly could work less.
110. (A) 112. (D)
Be sure to translate and combine any Formal Logic (A) creates an irrelevant distinction between painting and
statements. sculpture—irrelevant because the statements hinge on a
It is often the case that the answer to an Inference question distinction between music and other arts.
comes from a single sentence in the stimulus, or even a (B) isn’t necessarily true, because a general history of art,
fragment of a sentence, and this question is no exception. We having explained every aesthetic feature of music, would have
learn in this stimulus that most of the world’s forests have to go on to do likewise for other arts. (B) falls short of that.
fragmented ecosystems and cannot sustain themselves in (C) isn’t necessarily true because its topic is “any theory of
the long term, but harbor many endangered species. The final art,” whereas the statements all deal with any “general theory
sentence’s “requires” clues us into a Formal Logic statement: of art.” A small, but powerful and real, distinction.
If a fragmented forest is to maintain all of its plant and animal
(D) is deducible from the third and first sentences. If a
species, then resource managers must regularly intervene.
premodern general theory must neglect music, and if the
The contrapositive tells us that if resource managers do not
purpose of such a theory is to explain every art, then it can
intervene regularly, then the fragmented forests will lose at
never achieve that purpose . . . unless, of course, music were
least some of their plant or animal species. Using these facts,
somehow exempted from the definition of art. (D) gets all of
we can evaluate the choices:
that right.
(A) is a perfect combination of two facts in the stimulus: most
(E) is not inferable, partly because (D) is (there can only be one
of the world’s forests are fragmented, and fragmented forests
right answer!) and partly because the author never mentions
will lose some species without intervention. (A) must be true,
which aesthetic features of music fail to be explained by
and is correct. For the record:
postmodern general theory. Maybe they’re the ones shared
(B) is a bit too specific. We know that fragmented forests with painting and sculpture, and maybe they’re not.
harbor the world’s most endangered species, and these
forests will lose some species if resource managers do 113. (C)
not intervene, but the most endangered species wouldn’t
Expressed as a simple if/then statement, the last sentence
necessarily be the ones lost.
reads “If technology makes economic roles obsolete, then
(C) is a distortion of the facts in the stimulus. A fragmented it will undermine that society’s values.” (C) is merely the
forest cannot sustain itself, regardless of whether it loses any contrapositive of that statement, and hence (C) must be true
more species. based on the stimulus.
(D) and (E) Complete, fully functioning ecosystems are outside The author argues that labor-saving technology is more or less
the scope of the argument, as are the places where resource sufficient for changing social values, but (A) believes that it’s
managers currently intervene. necessary, and thus that such technology’s absence will lead
to unchanged values: a classic GMAT blunder. (B) goes too far
111. (B) in indicting all technology; only that which “makes certain
When attacking a formal logic question, start with the most economic roles obsolete” is indicted as undermining social
concrete statement and build on it. values. The author never spells out what “undermined values”
According to sentence 2, a necessary condition for successful means, so (D) is wrong to characterize the phenomenon
graphic designing is not ignoring clients’ wishes. So, all of as meaning “little value placed on prestige.” Nothing is
the successful graphic designers mentioned in sentence 1— mentioned in the stimulus about the importation of one
including the ones who were formally trained—do not ignore technology to a foreign society (E).
clients’ wishes, and that’s what (B) states.
114. (A)
(A) A classic necessity/sufficiency misunderstanding. That
success requires not ignoring clients’ wishes does not imply The stimulus begins in the present tense—defining MS and
that lack of success means ignoring them. There are many pointing to the drug that can now successfully treat it—and
other possible reasons for a designer’s failure. then takes us back to the past, to the “hardly straightforward
path” that led to the drug. Tests with the antiviral gamma
(C) There’s nothing in the stimulus that supports the direct
interferon, prompted out of the belief that MS has a viral
proportion described in (C).
source, made patients’ MS worse but were “instructive”—
(D) This isn’t necessarily true. The unsuccessful designers inferably, they led to a breakthrough.
among those trained on the job might ignore clients’ wishes.
That which is “LEAST compatible with” the experiment is
(E) No hierarchy (i.e., “most successful”) is implied by the contradictory to it, and that’s (A): We already know that in MS,
statements taken together or separately. white blood cells attack protective myelin. If (A) is right, and
gamma interferon prevents white blood cells from destroying If a monster is physically benign yet inspires revulsion,
myelin, why would gamma interferon make the patients’ MS then it’s horrific.
“dramatically worse?” It would make no sense for gamma
The rest of the stimulus is a lot of irrelevancy—note the
interferon to have such a terrible effect, given that the
phrase “Whether or not,” which means that everything else
substance blocks the progress of MS.
in the clause is irrelevant to the discussion. No prephrasing
All of the others are quite consistent with the experiment advisable; attack the choices in turn.
as described. Gamma interferon’s creation of white blood
(A) commits the common logical error of reversing a
cells (B) might well have contributed to the awful impact
statement’s (the first one’s) if- and then-clauses. (A) is merely
of the antiviral on MS patients in 1984; at least there’s no
testing whether you properly translated the first sentence. (B)
contradiction as in (A). (C) deepens the correlation between
leaves out the issue of whether the monster in question is
gamma interferon and MS that the 1984 experiment implied.
threatening, so its horrific nature remains up in the air. (C) isn’t
(D) might well have been the “instruction” that the last
necessarily true, since a physically benign (e.g., not physically
sentence implies was the result of the gamma interferon
dangerous) monster could inspire revulsion and hence be
experiment. As for (E), if gamma interferon has been proved to
horrific. (D), too, is not necessarily true; (D)’s if-clause is totally
be part of the cause of MS—which is certainly possible, given
self-contained in that it describes a threatening and horrific
its effect on patients in 1984—then (E) would be an expected
monster, so inspiring revulsion may or may not be part of the
effect of this successful anti-MS drug.
package.
115. (D) (E) rewards your patience. It’s inferable based on the last and
first sentences, in that order: If physically benign monsters
Expect the answer to an Inference question to emerge from
inspire revulsion (as (E)’s monsters do), they’re horrific; and
one or two statements combined.
if they’re horrific they are threatening; and that’s exactly
This is a rare instance of an inference question that’s based what (E) concludes. That (E) tosses in the irrelevant issue of
on an actual argument rather than a mere set of premises. psychological danger is of no importance.
Nonetheless, the right answer simply combines two pieces of
evidence. Yes, calcium carbonate can neutralize stomach acid, 117. (D)
but just gram can produce gastrin that triggers stomach acid.
An inference can simply be a rewrite of one stimulus sentence.
In other words, gram can create that which it is being used to
neutralize, choice (D). Bees are explicitly named as an insect species that can live for
years after ensuring their survival through reproduction, so (D)
Comparing methods of neutralizing stomach acid, (A), is
has to be true; the bees (D) describes are the ones who are
irrelevant to the author’s conclusion in the first sentence,
working to benefit the ecosystem.
which is a recommendation against excessive ingestion of
calcium carbonate because of its ill effects. (B) represents a Nothing about “most” insect populations can be inferred
classic logical fallacy. Yes, more calcium carbonate can lead from this highly tentative paragraph, in which the adjectives
to impaired kidney function, but it doesn’t logically follow that “many,” “some,” and “some” are used in that order.
avoiding calcium carbonate can reduce the risk of impaired That knocks out (A), (C), and (E). As for (B), it makes an
kidney function. (C) is tempting, but it is possible that larger unwarranted connection between the two stimulus sentences.
doses of the antacid will reduce stomach acid more effectively Those insects that don’t die after their first reproduction
than very small ones; true, over gram will lead to gastrin and may or may not “play a vital role in the ecosystem,” and the
more acid, but perhaps not enough to overshadow the amount argument says nothing about any insect’s “likelihood” of
neutralized in the first place. (E) has the calcium and impaired dying in the first place.
kidney function relationship exactly backward.
118. (B)
116. (E) When an argument deals with a complex physical process,
Attack formal logic questions with confidence, or skip them make mental pictures to render it clearer.
with confidence. Picture the rhodopsin molecules on the retina. (They can
At the heart of this initially confusing set of statements are look like whatever you want them to look like.) Picture them
three if/then statements, rewritten in order as: hit by photons of light. Picture the molecules—hit by light or
not—changing shape as they move and thus “introducing error
If a monster is horrific, then it’s threatening. into the visual system.” Now picture the effect of temperature
If a monster is physically dangerous, on that movement and that error factor: If temperature and
then it’s threatening. molecular movement are directly proportional, then when one
goes up, the other goes up. Hence (B): As the temperature
goes up, molecular movement will increase and, in turn, the of what happens when an addictive substance is suspended,
error factor will be intensified. (The first 13 words of (B) are but that issue too never comes up during the brief discussion.
designed to render moot the possible objection that other
temperature factors might influence what’s going on in 121. (B)
the eye.) Be on the lookout for uncommon ways the GMAT can
(A) sounds plausible—doesn’t light make things hotter? — phrase Formal Logic statements.
but is not supported by the text, which never mentions any We’re all used to seeing if/then statements on Logic Games,
factors that alter temperature. All we know is that as retina and even at times in the Logical Reasoning section of the
temperature goes up, rhodopsin molecules’ motion increases; GMAT. But not all Formal Logic statements will be phrased
does that make them react slower to photon attack, (C)? No in the if/ then form. Any statement that sets up a necessary
way to tell. Surface area, (D), is plausibly relevant to activity condition, such as something that “must be true” for another
on the retina but never mentioned here. And there may very thing to happen, is implicitly creating a Formal Logic
well be other pigment molecules in the retina, (E), though they statement. The result of the Formal Logic statement will be
don’t necessarily relate to the light-registration process. the necessary component, while the trigger will be the
condition that requires it.
119. (E)
The commentator’s evidence uses this construction to set up
In “could be true EXCEPT” questions, boldly seek out the an important Formal Logic statement in the final sentence. Up
choice that contradicts something in the stimulus. to this point, the commentator has informed us that political
Since the wrong choices are possible—that is, consistent with constitutions must be interpreted to deal with unforeseen
the stimulus—there’s an 80% chance that each answer choice situations, and that the new interpretations are usually
is consistent. Look for the “contradictor.” Correct answer (E) portrayed as embodying the authors’ intentions. We learn
simply contradicts the first sentence, since that sentence in the final sentence that this illusion—the idea that new
points to a virus that’s beneficial to humans yet deadly to interpretations stem from the original authors’ intentions—is
more complex microorganism, yet (E) denies the possibility “necessary for political stability.” In other words, if we are to
that such a virus exists. have political stability, then we must also have the illusion
Random mutations, we’re told, can cause alterations in that new laws come from a long political tradition instead of
viruses; and while some of those alterations can turn deadly the preferences of contemporary politicians. We can quickly
there’s no reason why others couldn’t benefit humans as make the contrapositive (If the illusion vanishes, then we
well (A). The author never describes how likely it is for a virus won’t have any more political stability) and use it and the
or some more complex critter to undergo random mutation, original statement in our evaluation of the choices.
nor does he discuss viruses that fail to kill, so (B) and (D), (A) Close, but no cigar. According to our contrapositive,
respectively, are possible. And while viruses are simple and political instability results from people not believing that
can benefit humans, there’s no reason why more complex the laws reflect the intentions of the original authors of their
organisms can’t be beneficial too, (C). constitution, not of their political leaders. Eliminate.
(B) Bingo. If people believe that their constitution is not being
120. (E) interpreted consistently with the intentions of its authors—if
(E) is a straightforward inference from the last sentence. We’re the illusion is shattered—political instability will result. (B) is
told that “as many people consume” caffeine as consume our contrapositive, and is thus correct. For the record:
any “other physically addictive psychoactive substance.” (C) The issue discussed in the stimulus is not whether the
So, assuming, as (E) does, that alcohol qualifies as one current political leaders hold the same beliefs as the authors
such “other” substance, there may be the same number of of the constitution, but whether people believe that the
consumers of caffeine as of alcohol, but the latter cannot constitution is being interpreted according to the authors’
possibly outnumber the former. intentions. We don’t know how the political leaders’ beliefs
The stimulus leaves the door open for many psychoactive may or may not affect political stability.
substances more physically addictive than caffeine, contrary (D) The written constitution is not cited as the source of the
to (A). (B) goes too far in making an inference about that illusion.
which is “typical” of addictive substances, since all of the
(E) The commentator never claims that a nation’s dependence
symptomatic evidence concerns caffeine alone. The distinction
on this fiction is the result of any perceived lack of a long legal
that (C) proposes between psychoactive and non-psychoactive
tradition.
substances couldn’t be further from the author’s mind, so it
cannot be part of a proper inference. And (D) gets into the area
122. (E) 123. (E)
Two sentences here, one beginning with the word “Some,” Here’s a rare Inference question that’s vulnerable to
the other with “No one.” Add to that an Inference stem and prephrasing: quasars burn so hot that they can’t last for more
answer choices that begin with either “No” or “Some” and the than 100 million years, but light from quasars takes at least
result is unmistakable—this is classic formal logic. It’s possible 500 million years to get here. Quasars have only been seen
to combine the statements, just like we do in Logic Games, in since 1963, and so any quasar light that anyone on Earth
order to deduce what must be true. The two most helpful terms has seen has to be at least 500 million years old, and so,
in the short stimulus are “some” and “no one”—the former we as (E) points out, that quasar itself cannot exist any more.
understand to mean “at least one,” while the latter excludes By the time its light hits us, it’s been dead for at least 400
all members of a group from a particular situation. “Many” is million years.
simply not as helpful, because we have no way of telling which (A) could be true, but we don’t know why quasars weren’t seen
people this “many” refers to. So, it’s a good strategy to look before.
to combine the first sentence with the first part of the second
(B) Light from quasars was noticed in 1963, but could have
sentence. Were you able to do this? If so, you should have had
reached Earth before that.
no trouble scanning for the correct answer. If not, try it now
before reading on. (C) Quasars have to produce lots of light to appear the way
they do given that they’re so far away. But presumably objects
The first sentence tells us that at least one planner/
that are much closer could appear as bright as a quasar.
construction rep (nothing wrong with shortening the terms
Quasars from 500 million light years away don’t appear
to that) has an interest in the decisions. But no planner lives
brighter than a camera flash from 12 inches away! —and it’s
in the suburbs, so if we combine these facts, we can say
doubtful that your trusty Kodak produces more light than 90
conclusively that there must be at least one non-
billion suns.
suburbanite
(the planner from above) who has an interest in the decisions. (D) For all we know, lots of things (like dead rock, for example)
That’s the same as saying that some (at least one) persons could exist for more than 100 million years, no matter where
interested in the decision don’t live in the suburbs, choice (E). it is. It’s the heat that puts the limit on a quasar’s life, not the
You may have noticed another deduction: it must be true that distance from Earth.
some construction reps don’t live in the suburbs, since the
planners in question, none of whom live in the suburbs, are 124. (A)
“those representing the construction industry.” This is an excellent question for reinforcing your understanding
(A) Getting rid of the double negative, (A) translates into “all of the concept of consistency vs. inconsistency. We are looking
persons with significant interest in the decisions are in the for the one choice that cannot be true, meaning that the other
construction industry.” This need not be true—nothing forbids four can be true—meaning that those four are consistent with
non-construction industry members outside of the committee the text.
from having major interest in the committee’s decisions. (A) (A) Happily, we don’t have to search for long. (A)’s statement
requires a strategically-inserted “only” in the first sentence in that “some [organisms] resist cancer as well as sharks” is flatly
order to be true. contradictory to sentence 2’s claim that sharks “have a greater
(B) No one on the committee itself lives in the suburbs, resistance to cancer than any other organism” (emphasis
sure, but, as with (A), nothing prevents people not on the ours). That sentence and (A) cannot both be true. So (A) is
committee from having a financial interest in the committee’s what we want.
decisions. And who knows where those people live? —it could (B) The organism that is most susceptible to cancer (which
be anywhere, suburbs included. Again, it would take an “only” could be any organism except the shark) may very well have a
somewhere in the first sentence to make (B) work. higher percentage of cartilage than some other organisms. The
(C) , (D) As mentioned above, this notion of “many” stimulus deals only with sharks, while (B) is so general that it
working in the suburbs is vague—which ones are they? We can refer to virtually any organisms.
simply don’t know, which is what creates the possibility that (C) Since sentence 3 doesn’t assert that only shark cartilage
both (C) and contains this tumor-inhibiting substance, it may very well be
(D) are false. In (C), it’s possible that the “many” planners true that many or indeed most creatures possess it, but that
working in the suburbs don’t coincide at all with the “some” other factors cause those other organisms to have a greater
who have a significant financial interest in the decisions. cancer risk than does the shark.
If these latter folks were the only ones interested in the (D) Sentence 4 asserts that no response to cancer therapy
decisions, then (C) would be false. Likewise, for (D): nothing among terminal patients has been more positive than shark
prevents all of the planner/construction reps from working in cartilage, which must mean that other therapies have been
the suburbs. Again, this all comes back to the ambiguity in
this context of the word “many.”
less positive. But it still could be true that those responses to (C) must be true. A weak economy means a rise in
other therapies have been “dramatic”—albeit less dramatic unemployment, which in turn means decreased investment.
than the shark cartilage results. So (D) offers no contradiction Hence (C) is deducible when sentences 1 and 2 are combined.
to the stimulus. This one is selected by students who forget what they’re being
(E) The concept of “immune system” is never mentioned in the asked for.
stimulus, so it could easily be true that the shark’s immune (D) The first clause in (D) can’t be true—we deduced that the
system is inferior to those found in other organisms. This economy is NOT weak. However, the second clause could be
doesn’t contradict the claim that sharks have the greatest true, as we saw that prices may or may not remain constant.
cancer resistance of all organisms, since we’re not told, and So overall, (D) contains a statement that could be true and is
cannot infer, what causes either the disease or the sharks’ therefore not what we seek here.
superior resistance to this one particular disease. (E) Again, as in (D), the first clause cannot be true
(unemployment is NOT rising), but the second clause must be
125. (A) true—the economy indeed is not weak, as we deduced above.
This is a formal logic question, as signaled by “if/then” and (E) is therefore a statement that must be true.
“only if,” which means we need to translate the statements • On the GMAT, whenever you encounter an if/then
into similar form and go on to make whatever deduction is statement, think through its contrapositive. And
available to us. whenever you encounter an “only if,” turn it into an
Sentence 1 says that a weak economy is sufficient to bring if/then and think through its contrapositive. Get into
these habits!
about two effects: constant prices and rising unemployment.
Okay for now. Sentence 2’s “only if” signals a necessary
126. (B)
condition, so we can translate the sentence into if/then terms,
to wit: Instead of offering an argument of his own, our stimulus
author is reporting two arguments made by others on
If unemployment rises, then investment decreases. the topic of exploiting (in the sense of “making use of”)
Now formulate the contrapositive: aspects of our earth. “Some” environmentalists (sentence
1) believe that exploiting our resources may not make sense
If investment is not decreasing, because you can’t benefit economically from resources
then unemployment is not rising. that have disappeared. “Many” of them (sentence 2) argue
And look! —sentence 3 confirms the latter “if” to be the case: that exploitation is wrong because it’s wrong to destroy the
Investment, indeed, is not decreasing. Hence, from sentence “intrinsic value” of nature, irrespective of the cost-to-benefits
2’s contrapositive, we can deduce that unemployment indeed ratio. We can’t tell whether there’s any overlap between those
is not rising. And that has an impact on sentence 1, whose two groups of environmentalists—that is, whether any of them
contrapositive must read: hold both views (which is certainly plausible). But we can be
sure that at least the members of the group in sentence 2
If either prices are not constant or unemployment argue against exploitation on noneconomic grounds; and they
is not rising, then the economy is not weak. are the “some” mentioned in correct choice (B).
And that leads us to a final deduction. The latter “if” has (A), (E) Each of these choices relies on the author’s having
just been confirmed—it is a fact that unemployment isn’t taken a firm position on the issue. (A) would be inferable if the
rising—hence we must conclude that the economy is not author agreed with the environmentalists of sentence 1, while
weak. So, of the four factors in the stimulus—economy, (E) is in line with an advocate of sentence 2’s position. But in
prices, unemployment, and investment—we are sure of the fact, our author takes no position at all, and it would therefore
status of three of them. Only prices is up for grabs; they be premature to infer these hard and fast judgments simply
may or may not remain constant. Now: Keep in mind that the from the author’s presentation of the two opinions.
right answer must be false, so the four wrong choices either (C) is discardable because nothing in the stimulus brings up
could or must be true. And no sooner do we start looking that the concept of “most” environmentalists. We can infer that
we see that: there are people who hold each of the positions mentioned,
(A) is impossible. Its second clause is flatly contradicted by and as we’ve said it’s possible that some hold both. But what
stimulus sentence 3, and its first clause is contradicted by our are the views of the majority? No way to tell.
final deduction that “the economy is not weak.” Neither of (D) is similarly too precise on the number breakdown, implying
(A)’s conditions is possible, so (A) is what we are looking for. that there must be some environmentalists who hold only the
For the record: view cited in sentence 2 and who disagree with the sentence
(B) is possible, because unemployment’s rise has no effect, as
far as we’re told, on the constancy of prices.
1 position. But no such environmentalists need exist, let alone with just those traits. Such a novel would have to qualify as
“many” of them. a poem, even though (as the author told us) novels rarely do.
Since the hypothetical novel in (C) meets the definition of
127. (C) poetry, that artwork would be both a novel and a poem, and
For EXCEPT questions, take the time to characterize the (C) is correct. For the record:
answer choices. (D) is subtle. Most of us do think of a limerick (the special
The wrong answer choices “could be true,” but the correct five-line form that usually includes some racy language) as
answer will directly contradict part of the stimulus. First, a poem, and since the limerick is deemed non-artistic in the
a quick lesson in ethics: a person who treats others well stimulus, (D) would seem to be inferable. But by the author’s
because of feelings of compassion rather than moral own definition, a limerick is not a poem—a poem has to be
obligation is more worthy of praise. This is true “despite the a work of art, and the author denies the limerick any artistic
fact” that a person can’t choose what feelings they have, but status. So (D)’s sentiment isn’t in line with the author’s
can choose to do the morally right thing. Predicting is tough, definitions.
so go straight to the answer choices: (E) thinks the issue is, What constitutes a work of art?, when
(A) is an unknown, but certainly could be true. Eliminate. the issue really is: What constitutes a poem? The symphony in
(E) cannot qualify as a poem under the author’s definition, but
(B) , by introducing the concept of the welfare of somebody,
it could qualify as art, a concept that the author never defines.
gets into uncharted territory, and is therefore unknowable.
Eliminate.
129. (E)
(C) contradicts the stimulus. The author argues that feelings
As standard a stem as they come for this garden-variety
a person have should be used to judge how worthy a person
Inference question; expect you’ll probably have to bypass
is of praise, but then goes on to point out that feelings are
pre-phrasing and will need to evaluate each choice. Did
beyond people’s control. This is our answer, but for the record:
you paraphrase the stimulus? Hopefully you condensed it
(D) Notice that in the statement the ethicist only comments on to something like this: Some things (pollen etc.) cause the
doling out relative amounts of praise. The ethicist never says that release of histamines which cause allergy symptoms. Anti-
the cold-hearted moralist isn’t worthy of any praise, only that hists fight the hists and relieve symptoms, but may cause
the warmth of compassion may earn you a few more worthiness drowsiness. Hists, however, are not implicated in producing
points. symptoms brought on by colds. (And how is it exactly that
(E) Again, unknowable and therefore possible. the trillion-dollar medical research industry that can grow
humans in test tubes and clone living things still can’t cure
128. (C) the sniffles?) Now, maybe we were premature to rule out pre-
As in most Inference questions, this stimulus gives us a group phrasing—there does seem to be something brewing regarding
of related assertions from which we are to derive a statement what hists and anti-hists cause and what they relieve (and
that has to be true. Here, the author defines a poem as a work kudos to those who were able to make the connection up
of art that uses some of the musical aspects of language (four front!), but it’s probably just a bit beyond our grasp. Moving
are listed, lines 2–3). Then each of the remaining sentences right to the choices is most likely the best way to go:
speaks to a different type of work, and comments on its poetic (A) We don’t know that . . . Pollen et al cause allergy
qualities and/or artistry. Let us take up each choice in turn. symptoms through the release of histamines, but just because
(A) does mention three of the types of creative work with which histamines don’t cause cold symptoms doesn’t mean that
the author deals, but we cannot conclude that only these pollen and allergens cannot cause colds via some other
types of creative work qualify as art, as (A) would have it. mechanism.
What about plays? Operas? Sonatas? Short stories? —to name (B) We know nothing from the stimulus about how to treat
four other creative forms that the author never mentions, and colds, so there’s no way to tell how treating colds compares
that could qualify as art. with treating allergies.
(B) To assert that blank verse lacks one of poetry’s four (C) Another classic case of “what DON’T you know?”: We know
“musical characteristics of language” doesn’t mean that blank antihistamines help relieve allergy symptoms. We don’t know
verse doesn’t possess any of the others—meter, rhythm, what they can or cannot do for colds.
euphony; and hence we cannot deduce that blank verse fails (D) A clever choice, but hopefully disposed of by cleverer
to qualify as poetry under the author’s definition. students. Antihistamines minimize “these allergy
(C) The author defines a poem as a work of art that makes use symptoms. . .”, which means the ones listed; and
of the music of language, and (C)’s if-clause asserts a novel sleeplessness is not among them. There’s simply no way to
tell whether antihistamines can treat sleeplessness brought 131. (B)
on by allergies. Why so clever? Because it’s a takeoff on the Yuck. This “all can be inferred except” question was a good
drowsiness side effect that sometimes accompanies the use of candidate for skipping, at least the first time around. When
antihistamines. Sure, they both deal with sleep-related issues, you do get to a question like this, the best approach is to read
but nothing suggests that a possible side effect would serve the stimulus, just to get a sense of the topic and scope, and
as an effective treatment of a related symptom. (See second then match each choice to the language of the stimulus. So
bullet point below.) what choice isn’t supported? Only (B): we know that poets
(E) We saw in (C) that we can’t tell what effect antihistamines preserve language, but that doesn’t mean that one purpose
would have on cold symptoms, but (E), paraphrased correctly, of writing poetry is to preserve the language in which it was
poses a hypothetical: IF antihistamines reduce cold symptoms written. Preservation could be an unintended beneficial side
(and we don’t know that they do), THEN it’s not accomplished effect. As for the others:
by blocking histamines. That absolutely must be true, because (A) Since only poetry cannot be translated well, it follows that
the final sentence just got finished telling us categorically all nonpoetic literature can be translated well.
that histamines play no part in producing cold symptoms. Not
(C) , (E) At least in the case of poetry, some translations must
sure? Take an analogous example: If aliens play no part in car
be missing something that was expressed in the original.
crashes, and if police have reduced car crashes, it cannot be
the result of rounding up aliens. (We didn’t say it would be a (D) is inferable from the statement “” we cannot witness
realistic example.) the beauty of poetry except in the language in which it is
composed.”
130. (B)
132. (D)
Finding the answer choice that is not inferable may be
considerably easier and faster than identifying the four that The structure here is interesting: a long (eight-line) sentence
are. First the anthropologist distinguishes between “violence” articulates a somewhat complex principle that Kay follows
and the “self-expression” normally needed for survival, and at election time. Then we get two short sentences revealing
then he posits that we react with violence under certain concrete facts about a particular mayoral election. We’d better
circumstances “only because” our culture conditions us to do start by understanding the principle fully—and the scope
so. The implication here (derived from asking yourself why the proves to be central.
anthropologist has linked up these two ideas) is that when a Given that an election is being held to select one candidate
human resorts to violence, it’s not normally out of a need to and that at least one issue is important to Kay (lines 1–3),
preserve himself, it’s because his culture is somehow pushing the principle is really exploring the following specific and
him to do so. But to contrast “violence” and “self-expression” limited question: “When may I, Kay, in good conscience vote
as the anthropologist does is not necessarily to contrast for someone with whom I disagree?” (Note lines 5–6 of the
“extreme aggression” and “non-aggression,” which is why (B) stimulus: “a candidate whose opinions differ from mine on at
is not a justified inference. He doesn’t define “self-expression” least one issue.” Note, too, that the principle never discusses
enough for us to determine to what extent it is aggressive, candidates with whom Kay’s in total agreement. Presumably
“generally” or otherwise. The other four choices all follow from she has no dilemma about voting for any of them.) In short,
the argument: Kay has set up a litmus test of sorts, for situations when she
(A) That “violence is an extreme form of aggression” implies is considering voting for a candidate with whom she has “at
that there are forms of aggression that are nonviolent. That’s least one” major area of disagreement.
all that (A) is saying. Here’s how you should have understood the protocol: given
(C) The “behaviors” that this choice is talking about at least that Kay is not in 100% agreement with any one candidate,
include the violent behavior that sentence 2 says is culturally she feels that it’s OK to vote for someone she disagrees with
conditioned. Since “some” means “at least one,” only one (let’s call him Candidate X) as long as she has more areas of
example is needed to support (C)’s inference. disagreement with the other candidates. That’s lines 6–7—see
how that works? In other words, if her disagreements with
(D) Whatever the term “self-expression” means, we know
the other candidates outnumber her disagreements with
that it is (1) “distinct” from violence, meaning that it’s non-
Candidate X, she feels it’s OK to vote for X. Otherwise—no way.
violent; and (2) normally “sufficient for survival.” That’s (D) in
a nutshell. Good enough. Now let’s attack the question, which involves
only the principle itself. The right answer is an inference—
(E) flatly restates the second sentence of the stimulus.
that which “must also be true”—so we’re not going to try to
Violence is culturally conditioned, the anthropologist says.
prephrase an answer. Let’s just start with the briefest choices,
which happen to be the first three, and see which of the five Instead, let’s take the choices in turn—perhaps starting with
follows logically: the briefest ones; that’s a good habit to get into:
(A) need not be true because it fails to meet one of the (D) Since the use of gold less pure than 92% is never
preconditions under which the principle will apply—namely, discussed, we can make no inference about it. Contrary to
that this be an election in which “at least one issue [is] (D), it’s quite possible that gold coins were minted from
important to Kay.” We have no idea what protocol she follows gold that was less pure than either natural Senegalese, or
(staying home? voting at random? voting by party?) when no refined, gold.
issues are important to her. (E) is also unwarranted, because nothing is mentioned about
(B) is incomplete. OK, so Kay agrees with each candidate the requirements of minting. Contrary to (E), it’s eminently
on “most of the [major] issues.” That’s not good enough. possible that the Spanish used unrefined gold from many
We cannot use the principle to draw a conclusion until her other parts of the world as the basis of some of their coins.
disagreements with each are enumerated. If she disagrees OK, on to the lengthier choices now:
with one candidate on fewer major issues than the others, (A) There’s no evidence for (A). The distinctiveness of
then the principle says she may vote for that candidate. If Senegalese coins has to do with their purity, not their weight
not, not. So (B) doesn’t tell us enough to infer whether its per coin or proportion of gold per coin, two issues that are
conclusion must be true or not. never discussed.
(C) is incomplete too. Kay may vote for a candidate with (B) is inferable from a combination of portions of sentences 2,
whom she has but a single area of agreement, so long as 3, and 4, like so: thanks to the refinement process (sentence
she has more disagreements on major issues with the other 4), the Spanish were able to mint coins with gold purer than
candidates. (C), like (B), fails to enlighten us about the relative natural Senegalese. But if (sentence 4 again) Senegalese
numbers of disagreements—the linchpin of her principle—so gold was never refined, then those purer coins must have
we cannot draw its conclusion for sure. been minted from refined gold from other parts of the world.
(D) , on the other hand, does enumerate those And since gold from Senegal was the naturally purest at 92%
disagreements, and allows us to draw its conclusion. If Kay (sentences 2 and 3), then that gold from “other places” must
has the same number of disagreements with each have been less naturally pure than 92%. That’s the gold to
candidate—which which (B) refers.
under (D), she does—then she’s stuck. The principle says (C) could be true, but is not inferable. For all we know, the only
that she can only vote for any one of them if she has more factor influencing monetary value was gold content.
major disagreements with the others. But here the number
of disagreements with each candidate is the same. So (D) is 134. (E)
right: by virtue of the principle, she cannot properly vote for
The stimulus for this formal logic Inference question ends
any of them.
with its most straightforward sentence: this parking policy
(E) ’s comparison may seem relevant but is really will be unpopular with somebody, whether it be the faculty or
bogus. We are not asked by the principle to compare the the students (or both, since that can’t be logically ruled out,
numbers of agreements and disagreements between Kay at least on the basis of the last sentence alone). The first two
and a single candidate. Again, the principle kicks in and sentences tell us the outcome of the policy’s being unpopular
gives us guidance only when Kay’s disagreements with each with each group: if the faculty, the policy should be modified;
of the candidates are compared. if the students, a new policy should be adopted. (And what
about that “or both” possibility? Well, while common sense
133. (B) suggests that one could not do both things—modify the
The topic of the numismatist’s litany of facts is the gold that existing policy and adopt a new one—the point might be
was used to make medieval Spanish coins. His scope focuses arguable. Happily, though, the answer choices never insist that
on the purity of gold, and especially on the purest of the pure, we go there.)
the gold from present-day Senegal in West Africa. Senegalese (A) The verb “popularize” appears nowhere in the stimulus,
gold, we’re told, was so pure—92% gold—that it didn’t have so in this formal logic question we are immediately ready
to be refined, and even today it’s highly identifiable as to trash this choice. And we should. Even if “popularize”
Senegalese in its coin form. We’re also told that the refining means “make the unpopular popular”—which it certainly
process can lead to purer gold—i.e., gold that is 93+ %—but need not mean—(A)’s effort would be wasted, because the
Senegalese gold was as pure as one could get naturally. policy will continue to be unpopular with at least one of the
This question seeks an inference about medieval Spanish gold two groups.
coins, and as usual with Inference questions, we are ill-served
by trying to predict an answer that could come from anywhere.
(B) , (C) “Reducing” popularity is no more a part of this a new home. How difficult it is to force a flock to move is
stimulus than was choice (A)’s “popularize.” We must stick anyone’s guess.
excruciatingly close to the terms of formal logic, and both (B)
and (C) veer too far afield. The terms of the stimulus involve 136. (D)
popularity and unpopularity—period. No levels or gradations We need to find the choice that must be false, which means
are mentioned. the other choices either could be true or must be true. This
(D) Not at all! If, as (D) says, the policy is popular among stimulus looks manageable. Intelligence and wisdom are
students, then it must be unpopular with the faculty (sentence different qualities that aren’t necessarily related, and the
3: it’s got to be unpopular with at least one group), in which people the author knows have one or the other but not both.
case the present policy should be modified, not a new one Notice that this distinction pertains only to the author’s own
adopted. experience. So, we don’t have much to use in forming a pre-
(E) has it right—and fortunately so, since we’re out of choices. phrase, but we should recognize (D) as impossible. Since
If, as (E) says, the policy is popular among faculty, then it must people known by the author have exactly one of the qualities,
be unpopular with the students (again, sentence 3 mandates it isn’t possible that no one is either wise or intelligent. At
the policy’s unpopularity with at least one group), in which least the people known by the author have one of those
case sentence 2’s if/then kicks in, and yes, a new policy qualities, so (D) cannot be true.
should be adopted. (A), (B) We’re told only about people known by the author, so
we can’t draw any inferences about “most people.”
135. (E) (C) Similarly, some people could be both wise and intelligent.
A claim that could be justifiably rejected is a claim that The author hasn’t cited an example of such a person, but they
conflicts with the stimulus. So, despite the complicated could be out there somewhere.
nature of the stem, this question is really asking us to infer (E) could be true, even within the author’s limited circle of
what is not likely based on the information in the passage. friends. Even there, many people have one quality but not
The stimulus isn’t short, but you can get it down to the gist: both, which leaves open the possibility that many have
crows search for food up to 130 kilometers from their roost; intelligence but not wisdom.
when they move, they move a relatively short distance away.
You may have formed a pre-phrase here, and you don’t need 137. (D)
to draw overlapping circles to do it: When the crows move,
The survival of the condor in the wild depends on an increase
their hunting area is basically the same. So, it’s unlikely that
in the breeding population. This means that if the condor is
they’re moving in reaction to the food supply. So (E) is a claim
to survive, then the population must increase. An increase in
that sounds fishy. If crows move because their hunting area
the breeding population depends on most of the condor eggs
has been stripped bare, then why wouldn’t they move farther
hatching, which means that if the population is to increase,
away? They would have to move farther than eight kilometers
then most of the eggs must hatch. Connecting the dots, this
in order to change their hunting area significantly. Since
means that if the condor is to survive in the wild, then most of
they move only a little, (E) can be rejected, which makes it
its eggs must hatch. Choice (D) is merely the contrapositive of
correct here.
this statement. If most of the eggs don’t hatch, then, sadly, the
(A) We have reason to believe that the reason crows move condor will not survive in the wild.
often isn’t related to food supply, but we don’t know what
(A) Not necessarily. The condor may still have a chance, thanks
does cause them to move. It could be population alone, as (A)
to breeding programs like the one described in the stimulus.
indicates. We just don’t know.
(B) takes the last sentence to an extreme conclusion: all we
(B) could be true. The stimulus suggests that changing the
know is that the breeding program is one possible way to save
roost location typically isn’t caused by food shortages, but
the condor. Other better means may exist as well.
what would happen in the event of a food shortage is a
question left unresolved here. (C) is tricky. We know that environmental factors make it highly
unlikely that most of the eggs will hatch, which suggests that
(C) seem likely and is at least possible. If crows travel as far as
environmental factors will be a threat in the wild. However,
130 kilometers to hunt for food, we might reasonably expect
the whole point of captive breeding programs is to eliminate
them to do much of their hunting farther than eight kilometers
environmental threats, and so for the eggs in such programs,
from the roost.
all environmental threats have been eliminated.
(D) We know that when left alone, a flock of crows tends to
(E) contradicts the stimulus, which claims that there is a fairly
stick to one roost for a while, until it leaves voluntarily for
low upper limit to the number of eggs that individual condors
can produce. So, increasing egg production may not be a No evidence is present to support any inference about the
viable method at all. customer service history of those various firms.
(C) Another variation of denial of the antecedent, ‡ la (A). We
138. (D) cannot be sure what would happen if and when the subsidies
The biologist recites a theory about why there are so many were removed, because all we know is what is happening
small goats on islands, then presents evidence to undermine with the subsidies in place. (C) takes the certain position that
that theory. “However” indicates that the author disagrees prices would have to go up if the subsidies were removed, but
with the basis of the original theory (that the need for a where’s the evidence for that prediction? In fact, any number
diverse gene pool caused the high number of small goats). The of other alternatives could come into play.
author argues instead that only reproductive success (and not (D) Huge (and classic) scope shift here, from the argument’s
anything else, including a diverse gene pool) can influence a interest in the railroads’ customer service history to (D)’s
trait, like size. So, the author is arguing that the only reason interest in their efficiency.
we would see more small goats is if small goats enjoy greater
reproductive success. And since the biologist believes that 140. (E)
this is the only reason we could have more small goats, we
Much easier than it looks, if you remember that many
can infer that pressure for a diverse gene pool does not affect
Inference questions simply require the translation of a single
size. Choice (D) states just that.
sentence, and if you notice that the “only” in sentence 5
(A) might be tempting because it supports the speculation that makes that sentence a prime candidate for translation. It
the biologist is arguing against—that the drive for a diverse states that an action can be rational only if its justification
gene pool can affect goat size. Remember, though, that we (reason) is a part of its explanation (cause). In other words,
want to support the biologist’s view, not the other view. if it’s rational, then its reason must be a part of its cause. In
(B) introduces an idea that does not appear in the stimulus, other words, (E).
“limited space,” and is also inconsistent with the biologist’s (A) Another sentence prime for translation is sentence 3,
view. The biologist believes that the frequency of small goats which tells us that a necessary condition for justification is
is a result of their greater reproductive success. (B) states that that there be enough reasons for an act. But (as the author
frequency is not the result of reproductive success. himself states) the explanation of that action is a whole
(C) also introduces the concept of “limited space” which is different issue. One sentence earlier we were told that every
never mentioned in the stimulus. More importantly, it focuses action—justified or otherwise—can potentially be explained, a
on large goats. The argument is about what causes goats to be sentiment that (A) contradicts.
small—either the need for a diverse gene pool or reproductive (B) mistakes a necessary condition for a sufficient one. To be
success. What causes goats to be large is outside the scope of rational, an action needs a connection between its reason
this argument. and its cause. It doesn’t follow that (as (B) asserts) any action
(E) The biologist argues that small size is driven by whose reason and cause are connected is ipso facto rational.
reproductive success. He does not state that there is any link (C) That “many psychologists” believe something does not
between size and a need for gene pool diversity. (E) muddles imply that “some psychologists” believe otherwise.
the speculation that the biologist argues against.
(D) sort of goes against the thrust of the argument that every
action’s cause can potentially be discovered, given enough
139. (E)
information.
The important—indeed the only—point to notice is that
nothing has changed, that Ostronia’s railroad companies 141. (B)
have always treated customers like dirt and continue to do
Based on the information about Country Q’s tally, what must
so, regardless of current economic difficulties. That’s what (E)
be true? First off, what’s this tally all about? On the last day
sums up pretty neatly. All the other choices distort stuff thrown
of every year, the Qians (Qers? Qannites?) calculate the tally
in to distract us from that one key point.
as the amount of coal mined in Q but not consumed. An easy
(A) commits the “fallacy of denying the antecedent.” Even way to think of the tally is as the total amount “left over” at
though the subsidies do permit the railroads to operate, it the end of each year—the new amount in the country’s overall
doesn’t follow that not having the subsidies will not permit stockpile. Now, if the tally is lower in 1991 than it was in
them to operate. 1990, that means that there is less coal left over at the end of
(B) This argument is solely about the railroad subsidies, but 1991 than was left over at the end of 1990; the stockpile has
(B) ’s scope is companies in general that have gotten subsidy. dwindled. This in turn means that in 1991, coal consumption
must have outpaced coal mining, choice (B). This is the only
way—barring the export of coal which is ruled out by the last 143. (D)
sentence—that 1991’s tally can be far lower than 1990’s. Our job here is to draw an inference from the several timing
(A), (C), (D), (E) The tally represents a comparison between statements contained in the stimulus. Here’s the situation:
the amount of coal mined and consumed. Any change in nearly all mail that’s correctly addressed arrives within two
the tally from year to year therefore represents a change in business days. The only time correctly addressed mail takes
this comparison. All of the wrong choices offer unsupported more than two business days is if it is damaged in transit.
comparisons between one of the elements across years; in Overall, however, most mail takes three days or more to get to
other words, mining in 90 vs. mining in 91, or consumption its destination. How can we reconcile this last idea with the
in 90 vs. consumption in 91. But all we have is information statement in the first two sentences? Like this: we must infer
on the tally, a number that includes both factors. Without that most mail is not correctly addressed, choice (D). Think
raw numbers, we can’t tell how the consumption numbers about it this way: the correctly addressed mail is a subset
compared from 1990 to 1991; we need to know the amount of the larger group called “overall” mail. And if most mail in
mined before making the judgments in (C) and (D). The this larger group takes three business days or more, yet most
opposite holds for (A); we need the consumption figures of the correctly addressed mail arrives within two business
before concluding how much was mined in 90 as opposed days, then there must be another group contained in the
to 91. (E) has an even greater problem: it introduces a new “overall” mail group that drives the “overall” arrival number
concept—the first half of 90 and 91—which we know nothing up. The only other group besides “correctly addressed mail” is
about. “incorrectly addressed mail,” and there must be a lot of it out
there if the number in the last sentence is correct.
142. (C) (A) can be true, but need not be; so, we can cross it off.
By asking for a conclusion supported by the art critic’s (B) There must be a lot of incorrectly addressed mail taking
response, this question essentially asks you to plant a big three business days or more in order to drive the overall
“Therefore” in front of each answer choice and choose the number up, but that doesn’t mean that every incorrectly
one that most logically follows from the critic’s remarks. In addressed piece of mail arrives this late. That is, it is possible
response to the claim that the change from red to green is for some incorrectly addressed pieces to arrive within two
justified (which is the curator’s main point), the critic points business days.
out counterevidence suggesting that Veronese’s intended color
(C) Most correctly addressed mail arrives within two business
was in fact red. The upshot, therefore—as (C) states—is that
days. We can’t switch this around and say that most mail
changing the color from red to green is not likely to restore the
arriving within two business days is correctly addressed. This
painting to its state during the painter’s life. In the end, the
choice may sound good at first; but if you look at it really hard,
restoration would not be “justified.”
you’ll see this is backwards and therefore cannot be inferred.
(A) ’s sweeping assertion about the Veronese copy does not It’s possible that most mail that shows up within two business
follow from the critic’s limited evidence about one particular days is incorrectly addressed.
element of the copy—the cloak.
(E) is an overly precise claim that we have no way of verifying
(B) Testing, whether sophisticated or crude, is outside the
from the information in the passage.
scope here. And the critic is concerned with the merits of one
proposed Veronese restoration, not the merits of restoration in 144. (B)
general.
The right answer choice for an Inference question will
(D) “Tampering”? The critic is talking about a Veronese copy. frequently involve the combination of two or more claims
And that copy has nothing to do with “the value of” the made in the stimulus.
original Veronese. Furthermore, the very specific topic and
In this stimulus, we learn that just about any chemical is
scope are the merits of one proposed change to a Veronese
carcinogenic, if given in doses large enough to kill cells. We
painting.
also learn that until now, studies of the carcinogenicity of food
(E) Mentally inserting the word “Therefore” between the critic’s
additives were performed by administering to test animals
statement and choice (E) should confirm why the latter is doses of the additives that were not only larger than the
wrong. There’s a big gap between the evidence about the post-
largest possible human exposure levels, but doses that were
mortem copy and (E)’s proposed conclusion about Veronese’s
large enough to kill many cells in the test animals’ bodies.
“original intentions.”
Combine those two claims, and it must be true that until
quite recently, carcinogenicity studies of food additives have
overestimated the carcinogenicity of those substances, (B).
(A) is a 180. The stimulus specifically states that previous makers say might not necessarily be consistent with the
carcinogenicity studies of food additives used doses large decisions they ultimately make.
enough to cause cell death in the test animals. (B) That some decision makers present contrary ideas (in order
(C) may very well be true in the real world. However, the to test out what they actually believe in) implies that they, at
stimulus does not provide us with any information that would least, do have an idea of how they feel before they approach
allow us to distinguish between truly carcinogenic substances their advisers.
and those that are not truly carcinogenic. (C) The passage is based on decision making through
(D) This answer choice may also be true in the real world discussion, so the nature of the decision in this one is outside
(perhaps depending upon where one lives), but there is no of the scope.
support for it in the stimulus. (D) Some decision makers may argue in favor of an idea they
(E) goes beyond the information of the stimulus. While the don’t believe in for the purpose of confirming their suspicions,
stimulus clearly tells us that the way carcinogenicity studies but this in no way suggests that they will end up settling for an
of food additives have been conducted likely overstates the idea in which they don’t believe.
carcinogenicity of some food additives, we cannot tell whether (E) The decision makers seek to test their beliefs by listening
this is true of “most” food additives that have already been to others. If the advisers knew the actual beliefs of the
banned due to concerns about carcinogenicity. decision makers in advance, this would probably skew their
advice towards what they think the speaker wants to hear, and
145. (B) thus defeat the decision maker’s purpose.
The argument is a chain of cause-and-effect—a prediction of
what the author believes will happen if the city gets rezoned:
new water and sewer systems, and then new apartment
houses will be built; schools will become overcrowded;
over-congested roads will lead to new roads. We learn in the
next to last sentence that a bigger tax bite will be needed
for those civic improvements. Hence answer choice (B) is a
proper inference: if the new apartment buildings (a result of
the rezoning) are built, then the tax bite (a necessary condition
for the civic improvements that will follow those apartment
buildings) will increase.
(A) is a classic distortion. “Unless [they] band together, the
[rezoning] will be approved” means that in the absence of
the citizens banding together, the rezoning will go through.
We cannot, however, presume that that banding together
would necessarily halt the rezoning (and all the subsequent
developments). Maybe they would all go through anyway.
(C), (D), and (E) The stimulus more or less argues that if the
rezoning plan goes through, Glen Hills’ rural atmosphere might
be destroyed (by the sequence of events described). We are
not, however, permitted to assume that the failure of the plan
will ensure the continuation of that rural atmosphere—but
that’s just what (C) does. Likewise, if the apartment buildings
are built, taxes will go up; but (D) just denies both terms.
That’s logically forbidden, as is (E)’s inference that not building
the apartment buildings will result in no overcrowded schools
or congested roads.

146. (A)
We’re told that one type of decision maker will sometimes
advocate a position just to see how others react to it rather
than because of a commitment to that position. This supports
(A), which states that the substance of what some decision
Paradox Questions
Directions: The questions in this section are based on the reasoning contained in brief statements or passages. For some questions, more
than one of the choices could conceivably answer the question. However, you are to choose the best answer; that is, the response that most
accurately and completely answers the question. You should not make assumptions that are by commonsense standards implausible,
superfluous, or incompatible with the passage. After you have chosen the best answer, blacken the corresponding space on your answer sheet.

1. After replacing his old gas water heater with a new, 3. Consumer activist: When antilock brakes were first
pilotless, gas water heater that is rated as highly introduced, it was claimed that they would
efficient, Jimmy’s gas bills increased. significantly reduce the incidence of multiple-car
collisions, thereby saving lives. Indeed, antilock
Each of the following, if true, contributes to an
brakes have reduced the incidence of multiple-
explanation of the increase mentioned above EXCEPT:
car collisions. I maintain, however, that to save
(A) The new water heater uses a smaller percentage lives, automobile manufacturers ought to stop
of the gas used by Jimmy’s household than did equipping cars with them.
the old one.
Which one of the following, if true, most helps to
(B) Shortly after the new water heater was installed,
resolve the apparent conflict in the consumer activist’s
Jimmy’s uncle came to live with him, doubling
statements?
the size of the household.
(C) After having done his laundry at a laundromat, (A) Drivers and passengers in automobiles with
Jimmy bought and started using a gas dryer antilock brakes feel less vulnerable, and are thus
when he replaced his water heater. less likely to wear seat belts.
(D) Jimmy’s utility company raised the rates for gas (B) Under some circumstances, automobiles with
consumption following installation of the new traditional brakes stop just as quickly as do
water heater. automobiles with antilock brakes.
(E) Unusually cold weather following installation (C) For inexperienced drivers, antilock brakes are
of the new water heater resulted in heavy gas easier to use correctly than are traditional
usage. brakes.
(D) Antilock brakes are considerably more expensive
2. Statistical studies show that last year there was the to manufacture than are traditional brakes.
greatest drop in the violent crime rate over the course of (E) Antilock brakes are no more effective in
a year since such statistics were first gathered. But they preventing multiple-car accidents than in
also reveal that at the same time public anxiety about preventing other kinds of traffic accidents
violent crime substantially increased.
Which one of the following, if true, most helps to
resolve the apparent discrepancy described above?
(A) Longer prison sentences were the primary cause
of the decrease in the violent crime rate over the
course of last year.
(B) As in the past, last year’s increase in public
anxiety about violent crime has been
consistently underreported in the news media.
(C) Most people can realistically assess the likelihood
that they will become victims of violent crime.
(D) People who feel the most anxiety about violent
crime usually live in areas with relatively high
violent crime rates.
(E) The proportion of violent crimes covered in the
news media nearly doubled over the course of
last year.
6. A study of plaque buildup on teeth used three randomly
4. Recent investigations of earthquakes have turned up a assigned groups of people who brushed their teeth twice
previously unknown type of seismic shock, known as a day for a year. People in Group 1 used the same
a displacement pulse, which is believed to be present toothbrush all year. People in Group 2 used the same
in all earthquakes. Alarmingly, high-rise buildings are toothbrush all year but sterilized it each month. People
especially vulnerable to displacement pulses, according in Group 3 used a new, sterile toothbrush each month.
to computer models. Yet examination of high-rises At the end of the year, people in Groups 1 and 2 had the
within cities damaged by recent powerful earthquakes same amount of plaque buildup as each other, while
indicates little significant damage to these structures. people in Group 3 had less plaque buildup.
Which one of the following, if true, contributes to a Which one of the following, if true, most helps to
resolution of the apparent paradox? explain the relative amounts of plaque buildup found in
(A) Displacement pulses travel longer distances than the three groups?
other types of seismic shock. (A) The buildup of plaque on teeth, which brushing
(B) Scientific predictions based on computer models twice a day helps to prevent, is accelerated by
often fail when tested in the field. the growth of bacteria on toothbrushes that
(C) While displacement pulses have only recently remained unsterilized for more than a month.
been discovered, they have accompanied all (B) The stiffness of the bristles on new toothbrushes,
earthquakes that have ever occurred. which the mechanical action of brushing
(D) The displacement pulses made by low- and destroys after several months, inhibits the
medium-intensity earthquakes are much less buildup of plaque.
powerful than those made by the strongest (C) The people who did the study measured the
earthquakes. amount of plaque buildup by a new method not
(E) Computer models have been very successful in usually employed by dentists.
predicting the effects of other types of seismic (D) Before they joined the study, some of the people
shock. in Group 3 had been in the habit of brushing
their teeth only once a day.
5. In a study, parents were asked to rate each television (E) The people in Group 2 and Group 3 brushed their
program that their children watched. The programs teeth as vigorously as did the people in Group 1.
were rated for violent content on a scale of one to five,
with “one” indicating no violence and “five” indicating 7. The water of Lake Laberge, in Canada, currently
a great deal. The number of times their children were contains high levels of the pesticide toxaphene.
disciplined in school was also recorded. Children who Authorities are puzzled because toxaphene was banned
watched programs with an average violence rating of in North America in the early 1980s and now is used
three or higher were 50 percent more likely to have been only in a few other parts of the world.
disciplined than other children.
Which one of the following, if true, does most to explain
Each of the following, if true, helps to explain the why the water of Lake Laberge currently contains high
statistical relationship described above EXCEPT: levels of toxaphene?
(A) Children who are excited by violent action (A) Levels of pesticides in the environment often
programs on television tend to become bored continue to be high for decades after their use
with schoolwork and to express their boredom ends.
in an unacceptable fashion. (B) Lake Laberge’s water contains high levels of
(B) When parents watch violent programs on other pesticides besides toxaphene.
television with their children, those children (C) Toxic chemicals usually do not travel large
become more likely to regard antisocial behavior distances in the atmosphere.
as legitimate. (D) North American manufacturers opposed banning
(C) Parents who rated their children’s television toxaphene.
viewing low on violence had become (E) Toxic chemicals become more readily detectable
desensitized to the violence on television by once they enter organisms the size of fish.
watching too much of it.
(D) Children learn from violent programs on
television to disrespect society’s prohibitions
of violence and, as a result, are more likely
than other children to disrespect the school
disciplinary codes.
(E) Parents who do not allow their children to watch
programs with a high level of violence are more
likely than other parents to be careful about
other aspects of their children’s behavior.
8. After 1950, in response to record growth in worldwide 10. Scientists agree that ingesting lead harms young
food demand, farmers worldwide sharply increased children. More lead paint remains in older apartment
fertilizer use. As a result, the productivity of farmland buildings than newer ones because the use of lead paint
more than doubled by 1985. Since 1985, farmers have was common until only two decades ago. Yet these same
sought to increase farmland productivity even further. scientists also agree that laws requiring the removal of
Nevertheless, worldwide fertilizer use has declined lead paint from older apartment buildings will actually
by 6 percent between 1985 and the present. increase the amount of lead that children living in older
apartment buildings ingest.
Which one of the following, if true, most helps to
resolve the apparent discrepancy in the information Which one of the following, if true, most helps to
above? resolve the apparent discrepancy in the scientists’
beliefs?
(A) Since 1985 the rate at which the world’s
population has increased has exceeded the rate at (A) Lead-free paints contain substances that make
which new arable land has been created through them as harmful to children as lead paint is.
irrigation and other methods. (B) The money required to finance the removal of
(B) Several varieties of crop plants that have become lead paint from apartment walls could be spent
popular recently, such as soybeans, are as in ways more likely to improve the health of
responsive to fertilizer as are traditional grain children.
crops. (C) Other sources of lead in older apartment buildings
(C) Between 1950 and 1985 farmers were able to are responsible for most of the lead that children
increase the yield of many varieties of crop living in these buildings ingest.
plants. (D) Removing lead paint from walls disperses a great
(D) After fertilizer has been added to soil for several deal of lead dust, which is more easily ingested
years, adding fertilizer to the soil in subsequent by children than is paint on walls.
years does not significantly improve crop (E) Many other environmental hazards pose greater
production. threats to the health of children than does lead
(E) Between 1975 and 1980 fertilizer prices paint.
temporarily increased because of labor disputes
in several fertilizer-exporting nations, and 11. Although marathons are 26.2 miles (42.2 kilometers)
these disputes disrupted worldwide fertilizer long and take even world-class marathoners over 2 hours
production. to run, athletes who train by running 90 minutes a day
fare better in marathons than do those who train by
9. When astronomers observed the comet Schwassman- running 120 minutes or more a day.
Wachmann 3 becoming 1,000 times brighter in
Each of the following, if true, contributes to an
September 1995, they correctly hypothesized that
explanation of the difference in marathon performances
its increased brightness was a result of the comet’s
described above EXCEPT:
breaking up—when comets break up, they emit large
amounts of gas and dust, becoming visibly brighter as a (A) The longer period of time that one runs daily, the
result. However, their observations did not reveal comet greater the chances of suffering adverse health
Schwassman-Wachmann 3 actually breaking into pieces effects due to air pollution.
until November 1995, even though telescopes were (B) The longer the period of time that one runs daily,
trained on it throughout the entire period. the easier it is to adjust to different race lengths.
(C) The longer the run, the more frequent is the
Which one of the following, if true, most helps to
occurrence of joint injuries that significantly
resolve the apparent conflict in the statements above?
interfere with overall training.
(A) Comets often do not emit gas and dust until (D) Runners who train over 90 minutes per day grow
several weeks after they have begun to break up. bored with running and become less motivated.
(B) The reason comets become brighter when they (E) Runners who train over 90 minutes per day
break up is that the gas and dust that they emit deplete certain biochemical energy reserves,
refract light. leaving them less energy for marathons.
(C) Gas and dust can be released by cracks in a
comet even if the comet is not broken all the
way through.
(D) The amount of gas and dust emitted steadily
increased during the period from September
through November.
(E) The comet passed close to the sun during this
period and the gravitational strain caused it to
break up.
14. Most doctors recommend that pregnant women eat a
12. A company that produces men’s cologne had been nutritious diet to promote the health of their babies.
advertising the product in general-circulation magazines However, most babies who are born to women who ate
for several years. Then one year the company decided to nutritious diets while pregnant still develop at least one
advertise its cologne exclusively in those sports medical problem in their first year.
magazines with a predominantly male readership. That
year the company sold fewer bottles of cologne than it Which one of the following, if true, does most to resolve
had in any of the three immediately preceding years. the apparent discrepancy in the information above?

Which one of the following, if true, best helps to explain (A) Women who regularly eat a nutritious diet while
why the sale of the company’s cologne dropped that pregnant tend to eat a nutritious diet while
year? breast-feeding.
(B) Most of the babies born to women who did not
(A) Television advertising reaches more people than eat nutritious diets while pregnant develop no
does magazine advertising, but the company serious medical problems later in childhood.
never advertised its cologne on television (C) Babies of women who did not eat nutritious diets
because of the high cost. while pregnant tend to have more medical
(B) The general-circulation magazines in which problems in their first year than do other babies.
the company had placed its advertisements (D) Medical problems that develop in the first year
experienced a large rise in circulation recently. of life tend to be more serious than those that
(C) Most men do not wear cologne on a regular basis. develop later in childhood.
(D) Women often buy cologne as gifts for male (E) Many of the physicians who initially
friends or relatives. recommended that pregnant women consume
(E) Successful advertisements for men’s cologne nutritious diets have only recently reaffirmed
often feature well-known athletes. their recommendation.

13. Last year a large firm set a goal of decreasing its 15. Many elementary schools have recently offered
workforce by 25 percent. Three divisions, totaling computer-assisted educational programs. Students’
25 percent of its workforce at that time, were to be reactions after several years have been decidedly mixed.
eliminated and no new people hired. These divisions Whereas students have found computers very useful
have since been eliminated and no new people have in studying arithmetic, they have found them of little
joined the firm, but its workforce has decreased by help in studying science, and of no help at all with their
only 15 percent. reading and writing skills.
Which one of the following, if true, contributes most to Which one of the following, if true, most helps to
an explanation of the difference in the planned versus explain the students’ mixed reactions?
the actual reduction in the workforce?
(A) Students in these schools began reading and
(A) The three divisions that were eliminated were doing arithmetic before learning to use
well run and had the potential to earn profits. computers.
(B) Normal attrition in the retained divisions (B) Of the disciplines and skills mentioned, the
continued to reduce staff because no new people exactness of arithmetic makes it most suitable to
were added to the firm. computer-assisted education.
(C) Some of the employees in the eliminated (C) Many elementary school teachers are reluctant to
divisions were eligible for early retirement and use computer technology in their classrooms.
chose that option. (D) Young students are more likely to maintain
(D) As the divisions were being eliminated some of interest in training programs that use the newest
their employees were assigned to other divisions. computers and video graphics than in those that
(E) Employees in the retained divisions were forced do not.
to work faster to offset the loss of the eliminated (E) The elementary schools have offered more
divisions. computer-assisted programs in reading and
writing than in arithmetic and science.
16. A certain moral system holds that performing good 18. North American eastern white cedars grow both on cliff
actions is praiseworthy only when one overcomes a faces and in forests. Cedars growing on exposed cliff
powerful temptation in order to perform them. Yet this faces receive very few nutrients, and rarely grow bigger
same moral system also holds that performing good than one-tenth the height of cedars growing in forests,
actions out of habit is sometimes praiseworthy. where they benefit from moisture and good soil. Yet few
eastern white cedars found in forests are as old as four
Which one of the following, if true, does the most to
hundred years, while many on cliff faces are more than
reconcile the apparent conflict in the moral system
five hundred years old.
described above?
Which one of the following, if true, most helps to
(A) People who perform good actions out of habit
explain the difference in the ages of the cedars on cliff
have often acquired this habit after years of
faces and those in forests?
having resisted temptation.
(B) Most people face strong moral temptation from (A) The conditions on cliff faces are similar to those
time to time but few people have to endure it in most other places where there are few tall
regularly. trees.
(C) People virtually always perform actions they (B) In areas where eastern white cedars grow, forest
think are good, regardless of what other people fires are relatively frequent, but fires cannot
may think. reach cliff faces.
(D) Since it is difficult to tell what is going on in (C) Trees that are older than a few hundred years start
another person’s mind, it is often hard to know to lose the protective outer layer of their bark.
exactly how strongly a person is tempted. (D) The roots of cedars on cliff faces lodge in cracks
(E) It is far more common for people to perform in the cliff, and once the roots are so large that
good actions out of habit than for them to they fill a crack, the tree is unable to grow any
do so against strong temptation. taller.
(E) Eastern white cedar wood is too soft to be used
17. To acquire a better understanding of the structure for firewood or modern buildings, but it is
and development of the human personality, some occasionally used to make furniture.
psychologists study the personalities of animals.
19. The local fair held its annual photography contest and
Each of the following, if true, contributes to an
accepted entries from both amateurs and professionals.
explanation of the practice mentioned above EXCEPT:
The contest awarded prizes in each of several categories.
(A) The actions of humans and animals are believed As it turned out, most of the prizes in the contest were
to be motivated by similar instincts, but these won by amateurs.
instincts are easier to discern in animals.
Each of the following, if true, could by itself constitute
(B) The law forbids certain experiments on humans
an explanation of the outcome of the photography
but permits them on animals.
contest EXCEPT:
(C) It is generally less expensive to perform
experiments on animals than it is to perform (A) Many more of the entries in the contest were
them on humans. from amateurs than were from professionals.
(D) Proper understanding of human personality (B) The judges in the contest were amateurs, and
is thought to provide a model for better amateurs tend to prefer photographs taken by
understanding the personality of animals. other amateurs.
(E) Field observations of the behavior of young (C) Amateurs tend to enter their best photographs
animals often inspire insightful hypotheses about while professionals tend to save their best work
human personality development. for their clients.
(D) Each category in the contest was restricted to
amateurs only or professionals only, and there
were more categories open to amateurs.
(E) Three times as many amateurs entered the contest
as had entered in any previous year.
22. The common ancestors of Australian land- and tree-
20. The symptoms of hepatitis A appear no earlier than dwelling kangaroos had prehensile (grasping) tails and
60 days after a person has been infected. In a test of a long opposable thumbs, attributes that are well-adapted
hepatitis A vaccine, 50 people received the vaccine to tree-dwelling but offer kangaroos few advantages
and 50 people received a harmless placebo. Although on land. It is hardly surprising, therefore, that land-
some people from each group eventually exhibited dwelling kangaroos eventually lost these attributes;
symptoms of hepatitis A, the vaccine as used in the test what is puzzling is the fact that all modern tree-dwelling
is completely effective in preventing infection with the kangaroos now lack them as well.
hepatitis A virus.
Which one of the following, if true, most helps explain
Which one of the following, if true, most helps resolve the puzzling fact cited above?
the apparent discrepancy in the information above?
(A) Modern tree-dwelling kangaroos must back down
(A) The placebo did not produce any side effects that tree trunks slowly and carefully, but the common
resembled any of the symptoms of hepatitis A. ancestors of modern tree-and land-dwelling
(B) More members of the group that had received the kangaroos used their opposable thumbs to
placebo recognized their symptoms as symptoms descend trees quickly headfirst.
of hepatitis A than did members of the group that (B) Modern tree-dwelling kangaroos are smaller than
had received the vaccine. most modern land-dwelling kangaroos but larger
(C) The people who received the placebo were in than their common ancestors.
better overall physical condition than were the (C) Modern tree-dwelling kangaroos’ tails cannot
people who received the vaccine. grasp branches, but they are somewhat longer
(D) The vaccinated people who exhibited symptoms and more flexible than those of modern land-
of hepatitis A were infected with the hepatitis A dwelling kangaroos.
virus before being vaccinated. (D) Modern tree-dwelling kangaroos are descended
(E) Of the people who developed symptoms of from species of land-dwelling kangaroos that
hepatitis A, those who received the vaccine had been land-dwellers for many generations
recovered more quickly, on average, than those before modern tree-dwelling kangaroos started
who did not. to develop.
(E) Modern tree-dwelling kangaroos have smaller
21. The blues is a modern musical form whose lyrics and weaker hind legs than modern land-dwelling
usually address such topics as frustration, anger, kangaroos, and they move more slowly on land
oppression, and restlessness. Yet blues musicians claim than do modern land-dwelling kangaroos.
to find joy in performing, and the musicians and fans
alike say that the blues’ overall effect is an affirmation
of life, love, and hope.
Each of the following, if true, helps to resolve the
apparent conflict in the passage EXCEPT:
(A) The sharing of blues music serves to create a
cohesive, sympathetic social network.
(B) Blues musicians who do not draw on their
personal tragedies are no more successful than
blues musicians who do.
(C) The irony and wit found in the blues provide a
sense of perspective on life’s troubles.
(D) The realization that other people share one’s
plight is helpful in dealing with life’s problems.
(E) The conversion of personal sorrow into an artistic
work can have a cathartic effect on artists and
their audiences.
23. From the tenth century until around the year 1500, there 25. Throughout the Popoya Islands community pressure is
were Norse settlers living in Greenland. During that exerted on people who win the national lottery to share
time, average yearly temperatures fell slightly their good fortune with their neighbors. When people
worldwide, and some people claim that this temperature living in rural areas win the lottery, they invariably
drop wiped out the Norse settlements by rendering throw elaborate neighborhood feasts, often wiping out
Greenland too cold for human habitation. But this all of their lottery winnings. However, in the cities,
explanation cannot be correct, because Inuit settlers lottery winners frequently use their winnings for their
from North America, who were living in Greenland own personal investment rather than sharing their good
during the time the Norse settlers were there, continued fortune with their neighbors.
to thrive long after 1500.
Which one of the following, if true, contributes most to
Which one of the following if true, most helps explain an explanation of the difference between the behavior of
why the Norse settlements in Greenland disappeared lottery winners in rural areas and those in cities?
while the Inuit settlements survived?
(A) Twice as many Popoyans live in rural areas as
(A) The drop in average yearly temperature was live in the city.
smaller in Greenland than it was in the world as (B) Popoyan city dwellers tend to buy several lottery
a whole. tickets at a time, but they buy tickets less
(B) The Norse settlers’ diet, unlike that of the Inuit, frequently than do rural dwellers.
was based primarily on livestock and crops that (C) Lottery winners in rural areas are notified of
were unable to survive the temperature drop. winning by public posting of lists of winners,
(C) There were settlements in North America during but notification in the city is by private mail.
the fifteenth century that were most likely (D) Families in rural areas in the Popoyas many
founded by Norse settlers who had come from contain twelve or fourteen people, but city
Greenland. families average six or seven.
(D) The Inuit and the Norse settlements were (E) Twice as many lottery tickets are sold in rural
typically established in coastal areas. areas as are sold in the city.
(E) The Norse community in Norway continued to
thrive long after 1500. 26. The highest-ranking detectives in the city’s police
department are also the most adept at solving crimes. Yet
24. In an effort to boost sales during the summer months, in each of the past ten years, the average success rate for
which are typically the best for soft-drink sales, Foamy the city’s highest-ranking detectives in solving criminal
Soda lowered its prices. In spite of this, however, cases has been no higher than the average success rate
the sales of Foamy Soda dropped during the summer for its lowest-ranking detectives.
months.
Which one of the following, if true, most helps to
Each of the following, if true, contributes to reconciling resolve the apparent paradox?
the apparent discrepancy indicated above EXCEPT:
(A) The detectives who have the highest success rate
(A) The soft-drink industry as a whole experienced in solving criminal cases are those who have
depressed sales during the summer months. worked as detectives the longest.
(B) Foamy Soda’s competitors lowered their prices (B) It generally takes at least ten years for a detective
even more drastically during the summer to rise from the lowest to the highest ranks of
months. the city’s detective force.
(C) Because of an increase in the price of sweeteners, (C) Those detectives in the police department who
the production costs of Foamy Soda rose during are the most adept at solving criminal cases are
the summer months. also those most likely to remain in the police
(D) A strike at Foamy Soda’s main plant forced department.
production cutbacks that resulted in many stores (D) The police department generally give the criminal
not receiving their normal shipments during the cases that it expects to be the easiest to solve to
summer months. its lowest-ranking detectives.
(E) The weather during the summer months was (E) None of the lowest-ranking detectives in the
unseasonably cool, decreasing the demand for police department had experience in solving
soft drinks. criminal cases prior to joining the police
department.
28. A survey of alumni of the class of 1960 at Aurora
27. Preschool children who spend the day in daycare University yielded puzzling results. When asked to
nurseries are ill more often than those who do not. indicate their academic rank, half of the respondents
They catch many common illnesses, to which they are reported that they were in the top quarter of the
exposed by other children in the nurseries. However, graduating class in 1960.
when these children reach school age, they tend to be ill Which one of the following most helps account for the
less often than their classmates who did not spend the apparent contradiction above?
day in daycare nurseries during their preschool years.
(A) A disproportionately large number of high-
Which one of the following, if true, best explains the ranking alumni responded to the survey.
discrepancy in the information above? (B) Few, if any, respondents were mistaken about
(A) There are many common infectious illnesses that their class rank.
circulate quickly through a population of school- (C) Not all the alumni who were actually in the top
age children, once one child is infected. quarter responded to the survey.
(B) Those children who have older siblings are likely (D) Almost all of the alumni who graduated in 1960
to catch any common infectious illnesses that responded to the survey.
their older siblings have. (E) Academic rank at Aurora University was based
(C) By school age, children who have been in daycare on a number of considerations in addition to
nurseries have developed the immunities to average grades.
common childhood illnesses that children who
have not been in such nurseries have yet to
develop.
(D) The number of infectious illnesses that children
in a daycare nursery or school develop is
roughly proportional to the number of children
in the facility, and daycare nurseries are smaller
than most schools.
(E) Although in general the illnesses that children
contract through contact with other children at
daycare nurseries are not serious, some of those
illnesses if untreated have serious complications.
29. A study conducted over a 6-month period analyzed daily 31. Industry experts expect improvements in job safety
attendance and average length of visit at the local art training to lead to safer work environments. A recent
museum. The results showed that when the museum was survey indicated, however, that for manufacturers who
not featuring a special exhibition, attendance tended to improved job safety training during the 1980s, the
be lower but patrons spent an average of 45 minutes number of on-the-job accidents tended to increase in
longer in the museum than when it was featuring a the months immediately following the changes in the
special exhibition. training programs.
Each of the following, if true, could help to explain Which one of the following, if true, most helps to
the differing average lengths of visits to the museum resolve the apparent discrepancy in the passage above?
EXCEPT:
(A) A similar survey found that the number of
(A) Visitors to the museum during special exhibitions on-the-job accidents remained constant after job
tend to have narrower artistic interests, and do safety training in the transportation sector was
not view as many different exhibits during their improved.
visit. (B) Manufacturers tend to improve their job safety
(B) A plan to extend normal museum hours during training only when they are increasing the size
special exhibitions was considered but not of their workforce.
enacted during the period studied. (C) Manufacturers tend to improve job safety training
(C) Many people who go to special exhibitions go only after they have noticed that the number of
simply for the prestige of having been there. on-the-job accidents has increased.
(D) Admission tickets to the special exhibitions at the (D) It is likely that the increase in the number of
museum are issued for a specific 1-hour period on-the-job accidents experienced by many
on a specific day. companies was not merely a random fluctuation.
(E) Many people who go to special exhibitions are on (E) Significant safety measures, such as protective
organized tours and do not have the opportunity equipment and government safety inspections,
to browse. were in place well before the improvements in
job safety training.
30. Carrots are known to be one of the best sources of
naturally occurring vitamin A. However, although 32. In the city of Glasgow, Scotland, trade doubled between
farmers in Canada and the United States report 1750, when the first bank opened there, and 1765, when
increasing demand for carrots over the last decade, the government regulations on banking were first
number of people diagnosed with vitamin A deficiency implemented in Scotland.
in these countries has also increased in that time.
Each of the following, if true, could contribute to an
Each of the following, if true of Canada and the United explanation of the doubling described above EXCEPT:
States over the last decade, helps to resolve the apparent
(A) The technological revolution that started in the
discrepancy described above EXCEPT:
early eighteenth century in England resulted in
(A) The population has significantly increased in increased trade between England and Scotland.
every age group. (B) Reductions in tariffs on foreign goods in 1752 led
(B) The purchase of peeled and chopped carrots has to an increase in imports to Glasgow.
become very popular, though carrots are known (C) The establishment of banking in Glasgow
to lose their vitamins quickly once peeled. encouraged the use of paper money, which made
(C) Certain cuisines that have become popular use financial transactions more efficient.
many more vegetable ingredients, including (D) Improvements in Scottish roads between 1750
carrots, than most cuisines that were previously and 1758 facilitated trade between Glasgow and
popular. the rest of Scotland.
(D) Carrot consumption has increased only among (E) The initial government regulation of Scottish
those demographic groups that have historically banks stimulated Glasgow’s economy.
had low vitamin A deficiency rates.
(E) Weather conditions have caused a decrease in the
availability of carrots.
35. After the United Nations Security Council authorized
33. Conscientiousness is high on most firms’ list of traits military intervention by a coalition of armed forces
they want in employees. Yet a recent study found that intended to halt civil strife in a certain country,
laid-off conscientious individuals are less likely to find
the parliament of one UN member nation passed a
jobs within five months than are their peers who shirked
resolution condemning its own prime minister for
their workplace responsibilities.
promising to commit military personnel to the action.
Each of the following, if true, helps to resolve the A parliamentary leader insisted that the overwhelming
apparent paradox above EXCEPT: vote for the resolution did not imply the parliament’s
(A) People who shirk their workplace responsibilities opposition to the anticipated intervention; on the
are less likely to keep the jobs they have, so contrary, most members of parliament supported the
there are more of them looking for jobs. UN plan.
(B) Conscientious people tend to have a greater than Which one of the following, if true, most helps to
average concern with finding the job most suited resolve the apparent discrepancy presented above?
to their interests and abilities.
(A) The UN Security Council cannot legally commit
(C) Resentment about having been laid off in spite
the military of a member nation to armed
of their conscientiousness leads some people to
intervention in other countries.
perform poorly in interviews.
(B) In the parliamentary leader’s nation, it is the
(D) People who are inclined to shirk their workplace
constitutional prerogative of the parliament, not
responsibilities are more likely to exaggerate
of the prime minister, to initiate foreign military
their credentials, leading prospective employers
action.
to believe them to be highly qualified.
(C) The parliament would be responsible for
(E) Finding a job is less urgent for the conscientious,
providing the funding necessary in order
because they tend to have larger savings.
to contribute military personnel to the UN
intervention.
34. Raisins are made by drying grapes in the sun. Although
(D) The public would not support the military
some of the sugar in the grapes is caramelized in the
action unless it was known that the parliament
process, nothing is added. Moreover, the only thing
supported the action.
removed from the grapes is the water that evaporates
(E) Members of the parliament traditionally are more
during the drying, and water contains no calories or
closely attuned to public sentiment, especially
nutrients. The fact that raisins contain more iron per
with regard to military action, than are prime
calorie than grapes do is thus puzzling.
ministers.
Which one of the following, if true, most helps to
explain why raisins contain more iron per calorie than
do grapes?
(A) Since grapes are bigger than raisins, it takes
several bunches of grapes to provide the same
amount of iron as a handful of raisins does.
(B) Caramelized sugar cannot be digested, so its
calories do not count toward the calorie content
of raisins.
(C) The body can absorb iron and other nutrients
more quickly from grapes than from raisins
because of the relatively high water content of
grapes.
(D) Raisins, but not grapes, are available year-round,
so many people get a greater share of their
yearly iron intake from raisins than from grapes.
(E) Raisins are often eaten in combination with other
iron-containing foods, while grapes are usually
eaten by themselves.
36. High cholesterol levels are highly correlated with 38. Radial keratotomy (RK), a surgery that is designed to
cardiovascular disease. In the Italian town of Limone, reshape the cornea so that light focuses correctly on the
however, each of the residents has had high cholesterol retina, is supposed to make eyeglasses or contact lenses
levels for many years, and yet they have not developed that correct for nearsightedness unnecessary. Yet a study
cardiovascular disease. of patients who have undergone RK shows that some of
them still need to wear glasses or contact lenses.
Which one of the following, if true, most helps to
explain the unusual health condition of the residents of Each of the following, if true, would help to resolve the
Limone? apparent discrepancy in the information above EXCEPT:
(A) Persons who come from families that have (A) As the eye heals from an operation to correct
enjoyed great longevity tend not to develop nearsightedness, it may in fact overcorrect,
cardiovascular disease. causing the person to be farsighted.
(B) The stress and pollution found in large cities (B) The more severe a patient’s nearsightedness, the
exacerbate existing cardiovascular disease, but less effective the corneal reshaping of RK will
there is little stress or pollution in Limone. be in correcting the problem.
(C) The residents of Limone have normal blood sugar (C) Occasionally an RK patient’s eyes may heal
levels, and very low blood sugar levels tend to differently, causing a difference in the two eyes’
cancel out the cardiovascular effects of a high visual acuity that can be overcome only with
cholesterol level. corrective lenses.
(D) The residents of Limone inherited from common (D) RK patients who originally suffered from only
ancestors a blood protein that prevents vascular mild nearsightedness may, if the cornea does not
blockage, which is a cause of cardiovascular heal evenly, develop an astigmatism that requires
disease. corrective lenses.
(E) Olive oil is a staple of the diet in some parts of (E) Those who choose to undergo RK tend to be as
Italy, but unlike butter, olive oil is a nearsighted before this operation as those who
monosaturated fat, and monosaturated fats do choose not to undergo RK.
not contribute to cardiovascular disease.

37. Shortly after the power plant opened and began


discharging its wastewater into the river, there was
a decrease in the overall number of bass caught by
anglers downstream from the plant.
Each of the following, if true, could explain the decrease
described above EXCEPT:
(A) The discharged wastewater made the river more
attractive to fish that are the natural predators of
bass.
(B) The discharged water was warmer than the
normal river temperature, leading the bass to
seek cooler water elsewhere.
(C) Because of the new plant, access to the part of the
river downstream from the plant was improved,
leading to an increase in the number of anglers
fishing for bass.
(D) Because of the new plant, the level of noise
downstream increased, making that section of
the river a less popular place for anglers to fish.
(E) The discharged wastewater created turbulence
that disrupted the vegetation of the river
downstream, destroying some of the bass’s
natural habitat.
weight.
39. The indigenous people of Tasmania are clearly related
to the indigenous people of Australia, but were separated
from them when the land bridge between Australia and
Tasmania disappeared approximately 10,000 years ago.
Two thousand years after the disappearance of the land
bridge, however, there were major differences between
the culture and technology of the indigenous Tasmanians
and those of the indigenous Australians. The indigenous
Tasmanians, unlike their Australian relatives, had no
domesticated dogs, fishing nets, polished stone tools,
or hunting implements like the boomerang and the
spear-thrower.
Each of the following, if true, would contribute to an
explanation of differences described above EXCEPT:
(A) After the disappearance of the land bridge the
indigenous Tasmanians simply abandoned
certain practices and technologies that they had
originally shared with their Australian relatives.
(B) Devices such as the spear-thrower and the
boomerang were developed by the indigenous
Tasmanians more than 10,000 years ago.
(C) Technological innovations such as fishing nets,
polished stone tools, and so on, were imported to
Australia by Polynesian explorers more recently
than 10,000 years ago.
(D) Indigenous people of Australia developed hunting
implements like the boomerang and the spear-
thrower after the disappearance of the land
bridge.
(E) Although the technological and cultural
innovations were developed in Australia more
than 10,000 years ago, they were developed by
groups in northern Australia with whom the
indigenous Tasmanians had no contact prior to
the disappearance of the land bridge.

40. A clothing manufacturer reports that unsalable garments,


those found to be defective by inspectors plus those
returned by retailers, total 7 percent of the garments
produced. Further, it reports that all of its unsalable
garments are recycled as scrap, but the percentage of
garments reported as recycled scrap is 9 percent.
Which one of the following, if true, could contribute
most to explaining the discrepancy between the reported
percentages?
(A) Garments with minor blemishes are sent to
wholesale outlets for sale at discounted prices
and are not returned for recycling.
(B) The percentage of garments returned by retail
outlets as unsalable includes, in addition to
detective merchandise, garments in unacceptable
styles or colors.
(C) Some inspectors, in order to appear more
efficient, tend to overreport defects.
(D) The total number of garments produced by the
manufacturer has increased slightly over the
past year.
(E) Unsalable garments are recorded by count,
but recycled garments are recorded by
41. A stingray without parasites is healthier than it
would be if it had parasites. Nevertheless, the
lack of parasites in stingrays is an indicator
that the ecosystem in which the stingrays live
is under environmental stress such as
pollution.
Which one of the following, if true, most helps
to reconcile the discrepancy indicated above?
(A) During part of their life cycles, the
parasites of stingrays require as
hosts shrimp or
oysters, which are environmentally
vulnerable organisms.
(B) A stingray is a free-ranging predator that
feeds on smaller organisms but has few
predators itself.
(C) A parasite drains part of the vitality of its
host by drawing nourishment from the
host.
(D) An ecosystem can be considered stressed
if only a few species of very simple
organisms can live there.
(E) Since the life of parasites depends on that
of their host, they need to live without
killing their host or else not reproduce
and infect other individuals before their
own host dies.
42. Numismatist: In medieval Spain, most gold coins were 43. A study of the difference in earnings between men and
minted from gold mined in West Africa, in the women in the country of Naota found that the average
area that is now Senegal. The gold mined in this annual earnings of women who are employed full time
region was the purest known. Its gold content of is 80 percent of the average annual earnings of men
92 percent allowed coins to be minted without who are employed full time. However, other research
refining the gold, and indeed coins minted from consistently shows that, in Naota, the average annual
this source of gold can be recognized because earnings of all employed women is 65 percent of the
they have that gold content. The mints could average annual earnings of all employed men,
refine gold and produced other kinds of coins that
Which one of the following, if also established by
had much purer gold content, but the Senegalese
research, most helps explain the apparent discrepancy
gold was never refined.
between the research results described above?
As a preliminary to negotiating prices, merchants selling
(A) In Naota, the difference between the average
goods often specified that payment should be in the
annual earnings of all female workers and the
coins minted from Senegalese gold. Which one of the
average annual earnings of all male workers has
following, if true, most helps to explain this preference?
been gradually increasing over the past 30 years.
(A) Because refined gold varied considerably in (B) In Naota, the average annual earnings of
purity, specifying a price as a number of women who work full time in exactly the
refined-gold coins did not fix the quantity same occupations and under exactly the same
of gold received in payment. conditions as men is almost the same as the
(B) During this period most day-to-day trading was men’s average annual earnings.
conducted using silver coins, though gold coins (C) In Naota, a growing proportion of female
were used for costly transactions and long- workers hold full-time managerial, supervisory,
distance commerce. or professional positions, and such positions
(C) The mints were able to determine the purity, and typically pay more than other types of positions
hence the value, of gold coins by measuring pay.
their density. (D) In Naota, a larger proportion of female workers
(D) Since gold coins’ monetary value rested on the than male workers are part-time workers, and
gold they contained, payments were frequently part-time workers typically earn less than
made using coins minted in several different full-time workers earn.
countries. (E) In ten other countries where the proportion of
(E) Merchants obtaining gold to resell for use in women in the work force is similar to that of
jewelry could not sell the metal unless it was Naota, the average annual earnings of women
first refined. who work full time ranges from a low of
50 percent to a high of 90 percent of the average
annual earnings of men who work full time.
46. Between 1977 and 1987, the country of Ravonia lost
44. Council member: The preservation of individual about 12,000 jobs in logging and wood processing,
property rights is of the utmost importance to representing a 15 percent decrease in employment in the
the city council. Yet, in this city, property owners country’s timber industry. Paradoxically, this loss of jobs
are restricted to little more than cutting grass occurred even as the amount of wood taken from the
and weeding. Anything more extensive, such as forests of Ravoinia increased by 10 percent.
remodeling, is prohibited by our zoning laws.
Which one of the following, if true, most helps to
Which one of the following provides a resolution to resolve the apparent paradox?
the apparent inconsistency described by the council
member? (A) Not since the 1950’s has the timber industry been
Ravonia’s most important industry economically.
(A) Property owners are sometimes allowed (B) Between 1977 and 1987, the total number of
exemptions from restrictive zoning laws. acres of timberland in Ravonia fell, while the
(B) It is in the best interest of property owners to demand for wood products increased.
maintain current laws in order to prevent an (C) Since 1977, a growing proportion of the timber
increase in their property taxes. that has been cut in Ravonia has been exported
(C) The city council places less importance on as raw, unprocessed wood.
property rights than do property owners. (D) Since 1977, domestic sales of wood and wood
(D) An individual’s property rights may be infringed products have increased by more than export
upon by other people altering their own property. sales have increased.
(E) Zoning laws ensure that property rights are not (E) In 1977, overall unemployment in Ravonia was
overly extensive. approximately 10 percent; in 1987, Ravonia’s
unemployment rate was 15 percent.
45. The hearts of patients who are given vitamin E before
undergoing heart surgery are less susceptible to certain 47. A certain type of insect trap uses a scented lure to attract
postoperative complications than are the hearts of rose beetles into a plastic bag from which it is difficult
patients who are not given vitamin E before heart for them to escape. If several of these traps are installed
surgery. From six hours after surgery onward, however, in a backyard garden, the number of rose beetles in
the survivors from both groups show the same level the garden will be greatly reduced. If only one trap is
of heart function, on average, and also recover equally installed, however, the number of rose beetles in the
well. Despite this lack of long-term effect, doctors often garden will actually increase.
recommend vitamin E for patients about to undergo
heart surgery. Which one of the following, if true, most helps to
resolve the apparent discrepancy?
Which one of the following, if true, most helps to
explain the doctors’ recommendation? (A) The scent of a single trap’s lure usually cannot be
detected throughout a backyard garden by rose
(A) Postoperative complications pose the greatest beetles.
threat to patients during the first six hours after (B) Several traps are better able to catch a large
heart surgery. number of rose beetles than is one trap alone,
(B) Postoperative complications occur six or more since any rose beetles that evade one trap are
hours after surgery for some patients who have likely to encounter another trap if there are
not been given vitamin E. several traps in the garden.
(C) It sometimes takes less than six hours for a (C) When there are several traps in a garden, they
patient’s heart function to return to normal after each capture fewer rose beetles than any single
heart surgery. trap would if it were the only trap in the garden.
(D) Certain categories of patients are inherently less (D) The presence of any traps in a backyard garden
likely to develop postoperative complications will attract more rose beetles than one trap
than other patients are. can catch, but several traps will not attract
(E) Many patients who are told that they are being significantly more rose beetles to a garden than
given vitamin E actually receive a medically one trap will.
inert substance. (E) When there is only one trap in the garden, the
plastic bag quickly becomes filled to capacity,
allowing some rose beetles to escape.
48. During the nineteenth century, the French academy 50. A safety report indicates that, on average, traffic
of art was a major financial sponsor of painting and fatalities decline by about 7 percent in those areas in
sculpture in France; sponsorship by private individuals which strict laws requiring drivers and passengers to
had decreased dramatically by this time. Because the wear seat belts have been passed. In a certain city, seat
academy discouraged innovation in the arts, there was belt laws have been in effect for two years, but the city’s
little innovation in nineteenth century French sculpture. public safety records show that the number of traffic
Yet nineteenth century French painting showed a deaths per year has remained the same.
remarkable degree of innovation.
Which one of the following, if true, does NOT help
Which one of the following, if true, most helps to resolve the apparent discrepancy between the safety
explain the difference between the amount of innovation report and the city’s public safety records?
in French painting and the amount of innovation in
(A) Two years ago, speed limits in the city were
French sculpture during the nineteenth century?
increased by as much as 15 kph (9 mph).
(A) In France in the nineteenth century, the French (B) The city now includes pedestrian fatalities in its
academy gave more of its financial support to yearly total of traffic deaths, whereas two years
painting than it did to sculpture. ago it did not.
(B) The French academy in the nineteenth century (C) In the time since the seat belt laws were passed,
financially supported a greater number of the city has experienced a higher-than-average
sculptors than painters, but individual painters increase in automobile traffic.
received more support, on average, than (D) Because the city’s seat belt laws have been so
individual sculptors. rarely enforced, few drivers in the city have
(C) Because stone was so much more expensive complied with them.
than paint and canvas, far more unsponsored (E) In the last two years, most of the people killed in
paintings were produced than were unsponsored car accidents in the city were not wearing seat
sculptures in France during the nineteenth belts.
century.
(D) Very few of the artists in France in the nineteenth 51. In the decade from the mid-1980s to the mid-1990s,
century who produced sculptures also produced large corporations were rocked by mergers,
paintings. reengineering, and downsizing. These events
(E) Although the academy was the primary sponsor significantly undermined employees’ job security.
of sculpture and painting, the total amount of Surprisingly, however, employees’ perception of their
financial support that French sculptors and own job security hardly changed over that period.
painters received from sponsors declined during Fifty-eight percent of employees surveyed in 1984 and
the nineteenth century. 55 percent surveyed in 1994 stated that their own jobs
were very secure.
49. Vervet monkeys use different alarm calls to warn each
Each of the following contributes to an explanation of
other of nearby predators, depending on whether the
danger comes from land or from the air. the surprising survey results described above EXCEPT:
Which one of the following, if true, contributes most (A) A large number of the people in both surveys
to an explanation of the behavior of vervet monkeys work in small companies that were not affected
described above? by mergers, reengineering, and downsizing.
(B) Employees who feel secure in their jobs tend to
(A) By varying the pitch of its alarm call, a vervet think that the jobs of others are secure.
monkey can indicate the number of predators (C) The corporate downsizing that took place during
approaching. this period had been widely anticipated for
(B) Different land-based predators are responsible for several years before the mid-1980s.
different numbers of vervet monkey deaths. (D) Most of the major downsizing during this period
(C) No predators that pose a danger to vervet was completed within a year after the first
monkeys can attack both from land and from the survey.
air. (E) In the mid-1990s, people were generally more
(D) Vervet monkeys avoid land-based predators by optimistic about their lives, even in the face of
climbing trees but avoid predation from the air hardship, than they were a decade before.
by diving into foliage.
(E) Certain land-based predators feed only on vervet
monkeys, whereas every predator that attacks
vervet monkeys from the air feeds on many
different animals.
54. Each child in a group of young children read aloud
52. When a community opens a large shopping mall, it often both a short paragraph and a list of randomly ordered
expects a boost to the local economy, and in fact a large words from the paragraph. The more experienced
amount of economic activity goes on in these malls. readers among them made fewer pronunciation errors in
Yet the increase in the local economy is typically much whichever task they performed second, whether it was
smaller than the total amount of economic activity that the list or the paragraph. The order in which the two
goes on in the mall. tasks were performed, however, had no effect on the
Which one of the following, if true, most helps to performance of beginning readers, who always made
explain the discrepancy described above? fewer pronunciation errors when reading the paragraph
than when reading the list.
(A) When large shopping malls are new, they attract a
lot of shoppers but once the novelty has worn off, Which one of the following, if true, most helps
they usually attract fewer shoppers than does the to explain why the order in which the tasks were
traditional downtown shopping district. performed was not significant for the beginning readers?
(B) Most of the money spent in a large shopping mall (A) Because several words were used more than once
is spent by tourists who are drawn specifically in the paragraph but only once in the list, the list
by the mall and who would not have visited the was shorter than the paragraph.
community had that mall not been built. (B) In reading the paragraph, the more experienced
(C) Most of the jobs created by large shopping malls readers were better at using context to guess at
are filled by people who recently moved to the difficult words than were the beginning readers.
community and who would not have moved had (C) The more experienced readers sounded out
there been no job offer in the community. difficult words, while the beginning readers
(D) Most of the money spent in a large shopping mall relied solely on context to guess at difficult
is money that would have been spent elsewhere words.
in the same community had that mall not (D) Both tasks used the same words, so that the
been built. words the children read in whichever task was
(E) Most of the jobs created by the construction of a performed first would be recognized in the
large shopping mall are temporary, and most of second task.
the permanent jobs created are low paying. (E) The beginning readers made more pronunciation
errors than the more experienced readers did in
53. People always seem to associate high prices of products reading both the paragraph and the list.
with high quality. But price is not necessarily an
indicator of quality. The best teas are often no more
expensive than the lower-quality teas.
Which one of the following, if true, does most to
explain the apparent counterexample described above?
(A) Packing and advertising triple the price of all
teas.
(B) Most people buy low-quality tea, thus keeping its
price up.
(C) All types of tea are subject to high import tariffs.
(D) Low-quality teas are generally easier to obtain
than high quality teas.
(E) The price of tea generally does not vary from
region to region.
55. During 1991 the number of people in the town of 57. In recent years the climate has been generally cool in
Bayburg who received municipal food assistance northern Asia. But during periods when the average
doubled, even though the number of people in Bayburg daily temperature and humidity in northern Asia were
whose incomes were low enough to qualify for such slightly higher than their normal levels the yields of
assistance remained unchanged. most crops grown there increased significantly. In the
next century, the increased average daily temperature
Which one of the following, if true, most helps to
and humidity attained during those periods are expected
resolve the apparent discrepancy in the information
to become the norm. Yet scientists predict that the yearly
above?
yields of most of the region’s crops will decrease during
(A) In 1990 the Bayburg Town Council debated the next century.
whether or not to alter the eligibility
Which one of the following, if true, most helps to
requirements for the food assistance program but
resolve the apparent paradox in the information above?
ultimately decided not to change them.
(B) In 1990 the Bayburg social service department (A) Crop yields in southern Asia are expected to
estimated the number of people in Bayburg remain constant even after the average daily
who might be eligible for the food assistance temperature and humidity there increase from
program and then informed the Bayburg Town recent levels.
Council of the total amount of assistance likely (B) Any increases in temperature and humidity
to be needed. would be accompanied by higher levels of
(C) During 1991 many residents of a nearby city lost atmospheric carbon dioxide, which is vital to
their jobs and moved to Bayburg in search of plant respiration.
work. (C) The climate in northern Asia has generally been
(D) During 1991 the number of applicants for food too cool and dry in recent years for populations of
assistance in Bayburg who were rejected on many crop insect pests to become established.
the basis that their incomes were above the (D) In many parts of Asia, the increased annual
maximum allowable limit was approximately the precipitation that would result from warmer and
same as it had been in 1990. wetter climates would cause most edible plant
(E) During 1991 Bayburg’s program of rent species to flourish.
assistance for low-income tenants advertised (E) The recent climate of northern Asia prevents
widely and then informed all applicants about many crops from being farmed there during
other assistance programs for which they would the winter.
be qualified.
58. In the country of Boldavia at present, only 1 percent
56. Of all the photographs taken of him at his wedding, of 80-year-olds are left-handed, whereas 10 percent of
there was one that John and his friends sharply 50-year-olds and 15 percent of 20-year-olds are left-
disagreed about. His friends all said that this particular handed. Yet over the past 80 years, the proportion of
picture did not much resemble him, but John said that Boldavians who are born left-handed has not increased,
on the contrary it was the only photograph that did. nor have Boldavian attitudes toward left-handedness
Which one of the following, if true about the undergone any important changes.
photograph, most helps to explain John’s disagreement Which one of the following, if true, most helps to
with his friends? explain the variation in incidence of left-handedness
(A) It, unlike the other photographs of John, showed among Boldavians?
him in the style of dress he and his friends (A) In Boldavia, where men have a shorter average
usually wear rather than the formal clothes he life expectancy than do women, left-handedness
wore at the ceremony. is less common among women than among men.
(B) It was the only photograph taken of John at his (B) In Boldavia, left-handed people are no more
wedding for which the photographer has used a likely than right-handed people to be involved in
flash. accidents
(C) It was a black-and-white photograph, whereas the (C) Ambidexterity is highly valued in Boldavia.
other photographs that showed John were mostly (D) The birthrate in Boldavia has decreased slowly
color photographs. but steadily over the course of the past 80 years.
(D) It was unique in showing John’s face reflected (E) Left-handed people have never accounted for
in a mirror, the photographer having taken the more than 18 percent of the population of
photograph over John’s shoulder. Boldavia.
(E) It was one of only a few taken at the wedding that
showed no one but John.
60. Box office receipts at movie theaters increased
59. In one-round sealed-bid auctions, each prospective buyer 40 percent last year over the previous year. Thus, the
submits in strict confidence a single bid, and the sale film industry overall evidently is prospering. Yet twice
goes to the highest bidder. A reserve price—a minimum as many movie theaters went bankrupt last year as in the
price below which there will be no sale—can be set by preceding two years combined.
the seller as protection against a token bid, which is an
unreasonably low bid sometimes made by a bidder who Which one of the following, if true, most helps to
gambles that there will be no other bid. Paradoxically, resolve the apparent discrepancy in the information
such protection is most needed when extremely desirable above?
items are auctioned off this way. (A) Films cost, on average, twice as much to produce
Which one of the following, if true about one-round today as they did ten years ago.
sealed-bid auctions, most helps to explain why the (B) Ticket prices at some theaters fell last year.
generalization characterized above as paradoxical holds? (C) Those of last year’s films that were successful
were very profitable films that were shown by
(A) The bidder who submitted the winning bid on exclusive engagement at only a selection of the
an item cannot, on being informed of this fact, largest theaters.
decline acceptance of the item in favor of the (D) The amount of money spent on film advertising
next-highest bidder. increased greatly last year over the year before,
(B) The identity of unsuccessful bidders is not and the majority of this expense was absorbed
disclosed unless those bidders themselves by the producers and the distributors of films,
choose to disclose it. not by the theater owners.
(C) The reserve price on an extremely desirable item (E) In general, an increase in a theater’s box office
is generally set high enough to yield a profit for receipts for any year is accompanied by an
the seller even if the winning bid just matches it. increase in that theater’s profits from snack-food
(D) Prospective buyers of an extremely desirable item and soft-drink sales for that year.
can often guess quite accurately who at least
some of the other prospective buyers are.
(E) Prospective buyers tend to believe that, in order
to be successful, a bid on an extremely desirable
item would have to be so high as not to make
economic sense.
61. It is clear that humans during the Upper Paleolithic 63. Environmental scientist: It is true that over the past
period used lamps for light in caves. Though lamps can ten years, there has been a sixfold increase in
be dated to the entire Upper Paleolithic, the distribution government funding for the preservation of
of known lamps from the period is skewed, with the wetlands, while the total area of wetlands needing
greatest number being associated with the late Upper such preservation has increased only twofold
Paleolithic period, when the Magdalenian culture was (although this area was already large ten years
dominant. ago). Even when inflation is taken into account,
the amount of funding now is
Each of the following, if true, contributes to an
at least three times what it was ten years ago.
explanation of the skewed distribution of lamps
Nevertheless, the current amount of government
EXCEPT:
funding for the preservation
(A) Artifacts from early in the Upper Paleolithic of wetlands is inadequate and should
period are harder to identify than those that be augmented.
originated later in the period.
Which one of the following, if true, most helps to
(B) More archaeological sites have been discovered
reconcile the environmental scientist’s conclusion with
from the Magdalenian culture than from earlier
the evidence cited above?
cultures.
(C) More efficient lamp-making techniques were (A) The governmental agency responsible for
developed by the Magdalenian culture than by administering wetland-preservation funds
earlier cultures. has been consistently mismanaged and run
(D) Fire pits were much more common in caves early inefficiently over the past ten years.
in the Upper Paleolithic period than they were (B) Over the past ten years, the salaries of scientists
later in that period. employed by the government to work on the
(E) More kinds of lamps were produced by the preservation of wetlands have increased at a rate
Magdalenian culture than by earlier cultures. higher than the inflation rate.
(C) Research over the past ten years has enabled
62. A patient complained of feeling constantly fatigued. It scientists today to identify wetlands in need of
was determined that the patient averaged only four to preservation well before the areas are at serious
six hours of sleep per night, and this was determined risk of destruction.
to contribute to the patient’s condition. However, the (D) More people today, scientists and nonscientists
patient was not advised to sleep more. alike, are working to preserve all natural
resources, including wetlands.
Which one of the following, if true, most helps to
(E) Unlike today, funding for the preservation of
resolve the apparent discrepancy in the information
wetlands was almost nonexistent ten years ago.
above?
(A) The shorter one’s sleep time, the easier it is to
awaken from sleeping.
(B) The first two hours of sleep do the most to
alleviate fatigue.
(C) Some people required less sleep than the eight
hours required by the average person.
(D) Most people who suffer from nightmares
experience them in the last hour of sleep before
waking.
(E) Worry about satisfying the need for sufficient
sleep can make it more difficult to sleep.
65. Ditrama is a federation made up of three autonomous
64. Smoking in bed has long been the main cause of home regions: Korva, Mitro, and Guadar. Under the federal
fires. Despite a significant decline in cigarette smoking revenue-sharing plan, each region receives a share of
in the last two decades, however, there has been no federal revenues equal to the share of the total
comparable decline in the number of people killed in population of Ditrama residing in that region, as shown
home fires. by a yearly population survey. Last year, the percentage
of federal revenues Korva received for its share
Each one of the following statements, if true over
decreased somewhat even though the population survey
the last two decades, helps to resolve the apparent
on which the revenue-sharing was based showed that
discrepancy above EXCEPT:
Korva’s population had increased.
(A) Compared to other types of home fires, home If the statements above are true, which one of the
fires caused by smoking in bed usually cause following must also have been shown by the population
relatively little damage before they are survey on which last year’s revenue-sharing in Ditrama
extinguished. was based?
(B) Home fires caused by smoking in bed often break
out after the home’s occupants have fallen (A) Of the three regions, Korva had the smallest
asleep. number of residents.
(C) Smokers who smoke in bed tend to be heavy (B) The population of Korva grew by a smaller
smokers who are less likely to quit smoking than percentage than it did in previous years.
are smokers who do not smoke in bed. (C) The populations of Mitro and Guadar each
(D) An increasing number of people have been killed increased by a percentage that exceeded the
in home fires that started in the kitchen. percentage by which the population of Korva
(E) Population densities have increased, with the increased.
result that one home fire can cause more deaths (D) Of the three regions, Korva’s numerical increase
than in previous decades. in population was the smallest.
(E) Korva’s population grew by a smaller percentage
than did the population of at least one of the
other two autonomous regions.
47. D
48. C
1. A 49. D
2. E 50. E
3. A 51. B
4. B 52. D
5. C 53. B
6. B 54. C
7. A 55. E
8. D 56. D
9. C 57. C
10. D 58. A
11. B 59. E
12. D 60. C
13. D 61. E
14. C 62. E
15. B 63. E
16. A 64. B
17. D 65. E
18. B
19. E
20. D
21. B
22. D
23. B
24. C
25. C
26. D
27. C
28. A
29. B
30. C
31. B
32. E
33. A
34. B
35. B
36. D
37. C
38. E
39. B
40. E
41. A
42. A
43. D
44. D
45. A
46. C
Paradox Questions last year. We don’t know, though the shrewd guess would be
that it dropped there just as it did generally. In any event, the
paradox deals with an overall increase in public anxiety vs. an
overall decline in violent crime, so focusing exclusively on the
1. (A) behavior in high-crime pockets, as (D) does, is off the point.
Characterize the choices carefully in EXCEPT questions.
3. (A)
Here, the four wrong answer choices will all help to resolve the
Paradox. The oddball correct answer will either have no effect The correct answer to a Paradox question must show how
or aggravate the confusion. the apparently conflicting elements of the stimulus can occur
together.
Job one in a Paradox question is always to identify what
seems to be wrong. Here, Jimmy bought a new, allegedly With all of the car commercials touting antilock brakes, it
more efficient gas water heater, but his bills went UP. would be easy to think that they’re a feature every car should
What does that mean? Either the new water heater doesn’t have, but the consumer activist thinks otherwise. Even though
compare as favorably with the old one as Jimmy had hoped, she admits that antilock brakes have reduced the incidence of
or there’s some other factor driving up his gas bills. All of multiple-car collisions, she believes that auto manufacturers
the choices except (A) provide an alternative explanation as should stop equipping cars with the brake systems “to save
to why Jimmy’s gas bills have increased (and so are readily lives.” Therein lies the problem. It doesn’t seem to make
eliminated); (A) is in keeping with what we would have much sense that antilock brakes reduce the incidence of
expected from the new, higher efficiency water heater and it certain types of car accidents and yet removing them would
does nothing to explain the higher bills. save lives. But that’s the point of this question. Our job is to
find the answer choice that explains how these conflicting
(B) More usage = higher cost; that’s one outside factor that
statements can both be true—how antilock brakes can both
could explain the increase. Eliminate.
prevent accidents and result in more fatalities. (A) gives us
(C) Again, increased usage explains the increased cost. Eliminate. this explanation. If people in cars with antilock brakes don’t
(D) Higher rates could mean a higher gas bill even though wear their seatbelts, then they are more likely to die in any
Jimmy’s usage might have decreased—another outside factor accidents that do occur.
that resolves our confusion. Eliminate. (B) and (D) give us additional reasons to remove antilock
(E) Just like (B) and (C), (E) explains the increased cost by brakes in favor of traditional brakes, but neither one explains
giving us a reason for increased usage. Eliminate. how antilock brakes lead to greater fatalities in auto accidents.
If anything, (C) argues that antilock brakes shouldn’t be
2. (E) removed. If inexperienced drivers (the most dangerous drivers
Always try to predict the kind of statement that would allow on the road) find antilock brakes easier to use than traditional
two apparently contradictory phenomena to coexist. brakes, then antilock brakes make these drivers safer.
It does seem odd that last year saw both an unprecedented (E) is an irrelevant comparison between the effects of antilock
drop in the violent crime rate and a huge increase in people’s brakes on the incidence of different kinds of accidents. In
worry about violent crime. You’d think they’d be reassured. fact, we know from the stimulus that antilock brakes reduce
There must be some external factor that jacks up people’s the incidence of multiple-car accidents, so if (E) were true, the
fears, and (E) provides it: the press. Heavy coverage of violent brakes would lead to fewer other accidents as well, arguing
crimes would certainly explain why people are more fearful against the activist’s recommendation.
than the reduced crime rate should make them.
(A) doesn’t speak at all of public reaction to crime, and of 4. (B)
course the right answer must do so, since it’s the public’s The correct answer to a Paradox question must show how
anxiety that is so paradoxical. both sides of the paradox can coexist.
(B) The reporting of public anxiety is after the fact: we’re This stimulus tells us that scientists recently discovered a
concerned with what raised that anxiety in the first place, not new type of seismic shock, a “displacement pulse,” which
the way in which the anxiety was reported afterwards. is thought to occur in all earthquakes. We further learn that
(C) , if true, deepens the paradox. These so-called displacement pulses are especially dangerous to high-rise
realistic assessors of the risk of victimhood should be less, buildings; at least, that’s true according to computer models.
not more, anxious, given the statistics cited in the stimulus. Yet high-rises in cities that have been damaged recently by
earthquakes don’t show any damage, contrary to what the
(D) correlates high anxiety with high crime areas, which begs
computer models would suggest, and therein lies the paradox.
the question of what became of violent crime in those areas
As we evaluate the choices, we have to ask, “Does this tell us include the behavior of the kids which is, after all, at the heart
how the high-rises could have survived the pulses, which the of the stimulus. (C) is what we want—the choice that has no
computer models tell us are so dangerous?” impact on the statistical relationship cited. (C) is correct. For
(A) would, if anything, deepen the paradox. If displacement the record:
pulses travel further than other types of seismic shock, we (D) speaks to a specific effect of violent TV (i.e., disrespect
would expect them to damage buildings in a wider area around for restrictions on violent behavior) that if taken into the
every earthquake. Yet we know that high-rises within cities hit schoolroom and schoolyard would, again, surely explain more
by earthquakes show “little significant damage.” Eliminate. frequent disciplining there.
(B) Hmmm…if computer models fail in the field, that could (E) tackles the issue from another angle. (E)’s parents allow
mean that the models concerning displacement pulses their kids to watch, inferably, only 1–2 level violence on
weren’t accurate, and that displacement pulses really aren’t TV, and thus are more likely to care for the kids’ behavior
all that dangerous to high-rises. This choice would resolve the generally. This means that those other parents, who allow 3+
paradox. For the record: level watching, tend to be less careful, and hence their kids
(C) would again deepen the paradox. If displacement pulses are more likely to deserve disciplining at school. None of the
have always been around, then they should have damaged four wrong choices confirms causality here, but in different
high-rises in more earthquakes than just the “recent” ones ways (A), (B), (D), and (E) all point to commonality between the
discussed in the stimulus. two distinct phenomena in question.
(D) Even if low- and medium-intensity earthquakes don’t pack
6. (B)
much of a displacement pulse, the buildings in the stimulus
were hit by “powerful” earthquakes, which would presumably When the right answer resolves a paradox, depend on one or
still have a stronger pulse. This doesn’t help to explain why more wrong choices to deepen it.
the high-rises weren’t damaged. It’s interesting that Groups 1 and 2 had about equal plaque
(E) again deepens the paradox by suggesting that the buildup when their approaches were so different from each
computer models are usually accurate—the opposite of (B). other, with Group 2 doing monthly sterilizing of the same
toothbrush while Group 1 just made their brushes last a year.
5. (C) And for all that, Group 3 had the best results, using a new
sterile brush monthly. There must have been something about
The relationship of two phenomena need not be causal, but
the brushes’ newness—the only factor unique to Group 3—that
one can find connections between them that could strengthen
removed plaque and (B) tells us what it was. Only Group 3
their causal nature.
would benefit from the stiff bristles that (B) describes.
The relationship here isn’t a paradox per se (in that the higher
(A) This choice deepens the paradox. Based on (A), Groups 1
likelihood of school disciplining doesn’t on its face contradict
and 2 should have different plaque buildups, since the latter’s
those kids’ greater exposure to violence), but the question
brushes were sterilized and the former’s weren’t.
works in much the same way; and certainly, the language
“helps to explain” is what we’ve come to expect with (C) The uniqueness or ordinariness of the measurement
paradoxes. method is irrelevant to the differing results, because the same
method was used for all three groups.
Prediction may or may not help you here. Look for some sort of
connection between violent TV and bad school behavior, and (D) Bad brushing habits in the past are irrelevant to the study.
remember that (1) the parents were the ones who rated the And even if they were relevant, Group 3 was the one with the
programs on the 1–5 violence scale, and (2) kids who don’t best plaque results, so (D) would only make the outcome more
watch “3+” level TV are watching “1–2” level. mysterious.
(A) Among the four wrong choices, (A) may be the clearest (E) If true, this finding only blurs the differences among the
explainer of the relationship. Those kids watching highly violent three groups. And if vigorousness of brushing is relevant to
TV are, according to (A), likely to behave in ways that would lead plaque, which it may or may not be, then (E) only intensifies
to disciplining, i.e., unacceptable acting-out. Eliminate. the paradox.
(B) As we noted, the parents rated the programs for their
7. (A)
violence, which implies that they had to watch them. If
watching them with kids legitimizes violence, as (B) asserts, When you see a Paradox question think, “How can this be?”
then the resulting antisocial behavior, if manifested in school, First summarize the apparent contradiction and then try to
would be likely to lead to disciplining. Eliminate. predict a possible explanation.
(C) Here are the parents again, but (C) addresses the effect of First, the paradox: the authorities can’t understand how the
violent TV on the parents themselves while making no effort to lake could still contain high toxaphene levels even though
toxaphene was banned more than 20 years ago. What would
how. (C) explains how the brightness could have increased
explain why the toxaphene is still present? One explanation is
prior to the observation of the comet’s breakup, and removes
that the poison lingers even after people stop using it, which is
all conflicts among the statements.
answer (A). You might have predicted other explanations, such
as some companies’ continued illegal use, or the presence of (A) is a 180 in that it deepens the mystery of how the
the poison on goods imported from overseas. As soon as you brightness could have begun in advance of the November
come up with a prediction, scan the choices; if your prediction sighting. (D), too, if anything deepens the paradox, since it
isn’t among them, make another quick prediction if you can. describes brightness-creating activity prior to November. Two
If you can’t make another prediction or if you weren’t able to choices relate to irrelevant causes of brightness (B) and comet
predict when you first summarized the stimulus, just move breakup (E). Neither speaks to the disjunction between the
to the choices. Remember that the one right answer to every September hypothesis and November observations.
Paradox question resolves the contradiction entirely.
10. (D)
Choice (B)’s “other pesticides” is irrelevant to the continued
presence of the banned toxaphene, so this choice is outside The author finds it paradoxical that while ingesting lead is
the scope. dangerous to kids, removing lead apartment paint is even
more so. That is only paradoxical if there’s nothing in the
How far toxins usually travel in the atmosphere is also
removal process that enhances the danger, but (D) points out
irrelevant, because we’re focused on the toxaphene in the
an even more dangerous consequence of lead paint removal;
lake, which means (C) is also outside the scope.
so, if (D) is true, the statements don’t contradict each other.
(D) Who cares which parties favored or opposed the ban?
The scope of the argument is lead and its removal, not
We only need to explain why the stuff is still in the lake. This
the replacement of lead paint with other paints (A) or the
choice is, again, outside the scope.
existence of other environmental hazards (E). The wisdom of
(E) is outside the scope for several reasons, one of which is allocating funds in one particular way (B) has nothing to do
that the stimulus never suggests that organisms of any size with the safety of lead and of paints removal. And no matter
are involved in the detection of toxaphene. Also, the argument what the source (C) of the dangerous lead is, the greater
never discussed the “how” of toxaphene detection. danger of the paint removal remains puzzling.

8. (D) 11. (B)


From 1950 to 1985, farmers used more fertilizer to increase At first it may be surprising that those who run for shorter
productivity. After 1985, farmers still wanted to increase time periods (90 mins. vs. 2 hours) do better in 2-hour-plus
productivity, but they actually started using less fertilizer— marathons. But if longer practice runs damage one’s health
pretty surprising since you’d think they would increase (A), jeopardize joint health (C), decrease motivation (D),
fertilizer to increase productivity. After all, it worked before. and drain biochemical energy (E), who can be surprised
How can we explain this apparent paradox? If, for some that shorter runs correlate better with success? (B) doesn’t
reason, fertilizer wouldn’t increase productivity after 1985, have any impact on the paradox. Indeed, since according
even though it had worked in the past, then the farmers’ to the stimulus a marathon is run at one fixed distance, the
actions make sense. (D) provides just such a situation, and so increased ability to “adjust to different race lengths” would be
explains the paradox nicely. irrelevant. So (B) is correct.
(A), (B) and (C) all deepen rather than explain the mystery:
(A) reinforces the need for increased productivity and never 12. (D)
addresses fertilizer, while (B) introduces several new plants Why, the question asks us to consider, would sales of men’s
that respond well to fertilizer, and (C) reiterates that fertilizer cologne drop so precipitously from one year to the next upon
was effective from 1950 to 1985. (E) explains why farmers switching the advertising from general to men’s magazines?
might need to use less fertilizer, but only in the period 1975– Well, it’s only a surprise if men are the ones doing the
1980. We only care about after 1985, so (E) is not relevant purchasing. If in fact women buy men a lot of cologne (D), then
here. it’s understandable—the buyers don’t see the ads, as they
probably did in the past.
9. (C)
Bringing in another ad medium (A) only impacts the situation
A simple rephrasing: How can astronomers in September have if we know what effect the medium would be on purchases,
“correctly” attributed the comet’s greater brightness to its and (A) fails to indicate anything about the impact of TV ads
breakup, when the breakup wasn’t observed until two months on cologne sales. (B) reinforces the notion that the switch
later? If the breakup had begun earlier and unobserved, that’s to men’s magazines was ill-advised, but doesn’t suggest
why sales dropped afterward. The wearing of cologne (C) is
irrelevant to the purchasing of same, and unless we know something about the nature of those disciplines vs. that which
something about the content of the ads—which we don’t—(E)’s the computer is good for. (B) speaks to the superior ability of
reference to athletes is similarly irrelevant. the computer to teach arithmetic over the other fields.
(A) has reading (where the computer is useless) and arithmetic
13. (D) (where it’s useful) appearing before computer skills, a
The firm eliminated three divisions totaling 25% of its workers, coincidence that at best is irrelevant and at worst deepens
and hired nobody new. And yet its net decrease of staff was the paradox. (C) and (D) avoid the comparisons among the
only 15%. The decrease should’ve been 25%; what happened disciplines altogether. And the fact that schools have offered
to that other 10%, if they weren’t laid off? The mystery is more computer programs in fields in which students find
solved, as many paradox questions can be, by recognizing the computers to be of no help, (E), renders the schools’ judgment
scope shift in the stimulus. The firm set out to decrease its questionable but doesn’t explain the students’ reactions.
workforce, but eliminated divisions (not necessarily workers).
So (D) has it right: Some of the workers in those three 16. (A)
divisions must have been reassigned elsewhere, and that’s “Reconcile the conflict” signals a paradox.
why they’re still on the payroll.
It seems, at first, contradictory that a necessary condition of
Three of the choices have no effect whatsoever on resolving praiseworthy good deeds is that one overcome temptation
this numerical mystery, which involves neither profit (A), to perform them, and yet habitual good actions (i.e.,
alternative reasons for leaving (C), nor productivity (E). (B) is those lacking that necessary condition) can nevertheless
a 180, for it would deepen the paradox; “normal attrition” be praiseworthy. What needs to happen is to reconcile
plus the stimulus layoffs should have led to a staff reduction the seeming contradiction between overcoming temptation
greater than 25%. and acting out of habit, and that’s what (A) does: The
habitual praiseworthy acts have been done in the context of
14. (C) overcoming temptation—years ago. So, the necessary condition
It seems paradoxical for doctors to recommend a nutritious has in fact been met in both cases.
diet for pregnant women when their babies are still likely to The length of temptation endured, (B), is never brought up
have at least one medical problem in their first year. This is in the author’s moral system, and the author distinguishes
only puzzling if the recommendation is designed to prevent all between one’s perception of good, (C), and what she perceives
post-natal medical problems. Consider, if you will, pregnant as objectively good. The necessary condition is “powerful
women who don’t follow the recommendation—who don’t temptation,” so (D)’s objection (How powerful is “powerful”?)
eat nutritiously. If the latter women have babies with multiple gets us nowhere. And the frequency of each of the types
first-year problems, then the recommendation makes perfect of good acts, (E), begs the question of how a condition
sense. (C) resolves the apparent paradox by implying that necessary for goodness can seemingly be absent in one type
following a nutritious diet is surely better for the baby than the of goodness. Only (A) hits it head on.
alternative.
Whether women who eat nutritiously during pregnancy also 17. (D)
eat well when they’re breast-feeding (A) is a potentially “Contributes to an explanation” signals a paradox.
interesting point, and if they didn’t, that would explain why
Why would scientists study animal personalities in order
the babies still get sick. But (A) goes the other way and thus
to gain insight about human personalities? Four of the five
only makes the doctors’ recommendation even more puzzling.
choices explain why, and probably you could have predicted
(B) is an even more obvious 180, providing no support at
some of them. If animal experiments were easier to carry
all for the recommendation. If the kids of poor eaters grow
out effectively, (A); permitted where human experiments
up healthy, why recommend a good diet? (D) presents an
were forbidden, (B); cheaper, (C); or just generally revelatory
irrelevant comparison; the relative seriousness of medical
of the “human animal,” (E); such experiments would be
problems is not at issue. (E) is also irrelevant in its suggestion
understandable. (D) reverses what we need, providing a
that how recently the advice was “reaffirmed” has something
rationale for studying humans in order better to understand
to do with its wisdom.
animals.
15. (B)
18. (B)
“Explain…mixed reactions” tells you to look for, and smooth
The essence of the paradox about the two sets of cedars
over, a contradiction.
goes like this: One set grows in a hospitable forest locale
Why would students find computers useful in arithmetic, but and another grows on hardscrabble cliff faces, yet the cliff
less so in science and not at all in reading and writing? Must be
cedars seem to be thriving better—in general they’re older sure you understand, in your own words, the nature of the
than the forest cedars. Some outside factor must account for discrepancy before moving on to the answer choices. In this
the difference in longevity, and you might not have predicted case, the discrepancy is signaled by the Keyword “although”
forest fires but certainly should not have been surprised to in the last sentence. The paradox is that even though the
encounter (B). That forests are far more susceptible to tree- vaccine used in this test is completely effective in preventing
killing blazes than are cliffs would readily account for the hepatitis, members of both groups exhibited symptoms of
longer life of the cliff cedars. hepatitis A. Now, it’s not surprising that members of the
(A) offers information only on the cliff dwellers and only on placebo group may have contracted the disease; after all,
their height, and none of that is germane to the comparison they weren’t vaccinated. But how could the members of the
at issue. Even if we assume, and we have no reason to do group that received the vaccine exhibit symptoms of hepatitis
so, that losing protective bark endangers tree life (C), the if we’re told the vaccine is completely effective in preventing
same phenomenon probably affects both cliff dwellers and it? Try to answer that yourself, and then look to the answer
forest dwellers, so (C) gets us nowhere. (D), like (A), leads choices for a similar explanation. (D) is correct because it
to an explanation of the cliff dwellers’ short stature but has gives a perfectly logical explanation: the people who exhibited
no effect on the forest vs. cliffs paradox. The use of cedar symptoms contracted hepatitis before they were vaccinated.
wood in general (E) is even further off the mark; this is about Remember, the stimulus tells us that the symptoms appear
indigenous growth, not end use. no earlier than 60 days after a person has been infected.
Therefore, if a person were infected on day 1, and received
19. (E) the vaccine on day 10, we would expect that person to exhibit
symptoms sometime after he or she was vaccinated.
Explaining an unexpected outcome uses the same skill as
resolving a paradox. (A) doesn’t give us any help. In fact, it tells us that some
members of the placebo group probably did have hepatitis,
Even though the question stem does not tell us to look
because if (A) is true it rules out the possibility that their
for a paradox, we know from the fact that we need to find
hepatitis symptoms were just side effects produced by the
an explanation that we will be using the same skill. The
placebo.
outcome of the photography contest is paradoxical because
it is unexpected: in a contest entered by both amateurs and (B) is irrelevant. It’s not important whether or not members
professionals, the amateurs won most of the prizes. Four of “recognized” their symptoms as symptoms of hepatitis.
the answer choices will explain this outcome, and the correct Remember, we just want to find out how the vaccinated group
answer will not. contracted the disease. Don’t lose sight of that.

(A) would explain the result, since we might expect amateurs (C) It doesn’t matter what condition the people who received
to win more of the prizes if many more of the entries in the the vaccine were in. The fact is, they received the vaccine,
contest were from amateurs than from professionals. Eliminate. the vaccine is completely effective in preventing infection,
and they still exhibited symptoms of the disease. (C) doesn’t
(B) If the judges were biased toward amateur photography, as
explain that paradox.
(B) suggests, that could explain the outcome. Eliminate.
(E) We want to know how these people contracted the disease.
(C) suggests that the amateurs’ best work was up against
Information on how quickly they recovered is outside the
lesser efforts by the professional photographers, so the
scope of the argument.
contest was not an even match. (C) would explain the
outcome. Eliminate.
21. (B)
(D) is the most obvious explanation in the choices; if each
Eliminate any answer choice that even partially resolves the
category was exclusively amateur or exclusively professional
paradox in a Paradox/EXCEPT question.
and there were more amateur categories, then of course the
amateurs would win more prizes. Eliminate. How can one enjoy playing the blues? The four wrong choices
will show us how. The GMAT leaves none of life’s big questions
(E) remains, and must be the correct answer. If more
unexplored.
amateurs entered this year than in previous years, then more
professionals also could have entered, or the comparatively (A) Providing a “cohesive, sympathetic social network” could
few professionals could still have won more of the prizes. certainly contribute to a positive experience. Eliminate.
(E) does not explain the outcome of the contest. (B) , in focusing on success, leaves the question of happiness
up in the air. This does nothing to resolve the paradox, and is
20. (D) our answer. For the record:
For this question we need to resolve the apparent discrepancy. (C) , in providing a “perspective on life’s troubles,”
Remember, the first step in this question type is to make certainly could help explain the music’s appeal.
(D) offers up the group therapeutic value of the blues. doesn’t work, the author writes, because Inuit settlers in
(E) ’s cathartic effect can only be a good thing. Greenland thrived during the same period. So why did the
Inuit prosper while the Norse were wiped out? That’s the
22. (D) paradox.

This difficult paradox question may set you back for a few (B) offers a satisfactory answer. If the Norse settlers based
moments, but once you’ve correctly defined the paradox, you their diet on livestock and crops that perished during the
should be able to evaluate the answer choices and find the period of extreme cold, but the Inuit ate foods that the cold
right answer. didn’t much affect, that could explain why the Inuit survived in
Greenland around 1500 while the Norse were eliminated.
What is the paradox here? We know that modern tree-dwelling
and land-dwelling kangaroos had a common ancestor. The (A) doesn’t help resolve the paradox. No matter how much or
ancestor possessed several physical attributes that help tree- how little the temperature changed in Greenland compared to
dwelling animals but not land-dwellers. It’s easy to see why the rest of the world, it still leaves unexplained why the Inuit
modern land-dwelling kangaroos might have lost tree-dwelling survived the change but the Norse didn’t.
attributes, but why would modern tree-dwelling kangaroos (C) Even if all the Norse in Greenland moved to North America
have lost them also? To get a clearer picture of this paradox, in around 1500 (which itself is more extreme than what’s
you might imagine a tree that is forked at its base, with the supported by the choice), that doesn’t explain why the Norse
base representing the common ancestor, one fork representing got out of Greenland while the Inuit were able to thrive there.
land-dwelling kangaroos, and the other fork representing (D) If the Inuit and Norse settlements both occupied the same
the tree-dwelling kind. If that picture truly represents the general areas, then the Norse disappearance becomes even
evolutionary tree of kangaroos, then we indeed have a more enigmatic, and the paradox deepens.
paradox on our hands. But what if that picture is inaccurate?
(E) Like (C), this is irrelevant to an explanation of the paradox.
What if the common ancestor first evolved into a land-dwelling
No matter what the conditions were in Norway, we still don’t
kangaroo, after which the kangaroo lost its prehensile tail and
know why the Inuit survived in Greenland but the Norse
opposable thumbs? If the land-dweller then gave rise to a tree-
disappeared.
dwelling variety, as stated in (D), the mystery of the vanishing
tree-traits has been solved.
24. (C)
(A) contributes nothing new to what we already know: The
This Paradox question asks you to pick the choice which does
fact that modern tree-dwelling kangaroos have lost those
NOT provide an explanation for the apparent discrepancy.
early adaptations and seem to be inconvenienced as a result
The discrepancy isn’t hard to spot: Although Foamy Soda
is perfectly consistent with the info in the passage, and thus
lowered its prices during the summer, its sales dropped. The
offers no resolution to the mystery of why they lost those
result is surprising because we would normally have expected
attributes.
sales to increase given the lower price. A quick look through
(B) The size of modern kangaroos and their ancestors tells us the answer choices reveals that (C) does nothing to explain
nothing about why modern tree-dwellers have lost helpful tree- the paradox. This choice refers to the increased cost of the
dwelling features. If you took the logical leap that the size of soda, but says nothing relevant about its sales. While cost is
modern tree-dwellers has something to do with the fact that certainly pertinent to the amount of profit the company earns
they may not need those lost attributes today, then you went from the soda, nothing in the stimulus allows us to infer how
too far. production costs may impact on sales. (C) is therefore the
(C) sheds no light on the mystery. Even if modern tree-dwelling choice that does not resolve the paradox.
kangaroos have tails that are slightly more flexible than the Each of the other choices provides a plausible explanation
tails of modern land-dwelling kangaroos, why did they lose for why Foamy Soda’s sales might have dropped despite the
the fully prehensile use of their tails when such tails would be decrease in prices:
advantageous to them?
(A) and (E) point to factors that affected the entire industry,
(E) These other differences between modern tree-dwelling and thus making Foamy’s poor performance easier to understand.
land-dwelling kangaroos are immaterial to the question of why If soft drinks sold poorly across the board (A), it’s no wonder
tree-dwellers lost attributes that are helpful to tree-dwellers. that Foamy Soda did too. And if the demand for all soft drinks
decreased, as (E) indicates, it’s more understandable that
23. (B) Foamy Soda’s sales would suffer.
Some people claim that a worldwide temperature drop wiped (B) and (D) point to special circumstances that might have
out Norse settlements in Greenland during the 1500s because contributed significantly to Foamy’s decline in sales. If
no humans could survive the cold there. But this explanation the company faced harsh competition from price-slashing
competitors (B), it’s not surprising that Foamy’s sales were 27. (C)
down—people were buying the competitors’ brands. And if When there’s a “discrepancy,” or something unexpected or
a strike caused production cutbacks curtailing the amount surprising occurs, we’ve got a Paradox question.
of Foamy Soda that actually reached the stores during the
The first step is to identify the paradox. Here, preschoolers
summer (D), the mystery is not so puzzling anymore: It makes
who go to daycare are sick more often than preschoolers
sense that less product would produce decreased sales.
who don’t go to daycare. However, once the children enter
elementary school, the daycare kids get sick less often than
25. (C)
their classmates who did not go to daycare. How can this be?
Here’s an odd situation: While lottery winners in both the city Perhaps the daycare children built up immunity to common
and country are subject to peer pressure to share their wealth childhood illnesses when they shared their germs and toys in
with neighbors, the pressure only works in the latter. City preschool, so that by the time they reached elementary school
winners resist it and spend their winnings on themselves. How they were no longer susceptible to the germs that elementary
come? There must be some mitigating factor at work on the schoolchildren pass around. Answer choice (C) says just that.
city winners, but prephrasing just what that factor might be is
(A) explains how schoolchildren in general can get ill, but
practically impossible.
doesn’t tell us how the daycare graduates avoid catching the
Therefore, your best bet here was to analyze the choices in turn common childhood illnesses.
until you came to (C), whose mitigating factor is the way winners
(B) The distinction in the stimulus is between children who
are publicized. Since winning the lottery in the city is a private
went to daycare and those who did not go to daycare. Bringing
matter, it makes sense that those winners might be better able
a new distinction (older siblings) into the equation doesn’t
to resist the pressure to share than those in the country, where
explain the differing rates of infection between the original
winners’ identities are common knowledge. Notice that the
subgroups of children.
wrong choices are paired off in terms of their irrelevancy:
(D) This answer choice ignores the time element of the
(A) and (D) focus on the number of people in the country
stimulus. The paradox is not that children in daycare have
relative to the city. (Indeed, (A) and (D) are virtually identical,
fewer illnesses than children in school; it’s that children who
a solid clue that both are incorrect.) But the real issue is the
went to daycare before attending elementary school have fewer
way in which peer pressure works differently, and that doesn’t
illnesses than their classmates who did not go to daycare.
hinge on the number of people involved.
(E) discusses the severity of illnesses passed around day-
(B) and (E), by the same token, each focus on the numbers
care centers, but utterly fails to address the discrepancy. The
of tickets and ticket buyers. But having more or fewer tickets
stimulus says nothing about the severity of illness in either
in play doesn’t contribute to an explanation of why winners
the daycare population or the non-daycare population, so this
behave as they do.
information is outside the scope of the stimulus.

26. (D)
28. (A)
It does seem a contradiction that the elite group of detectives
How can 50% of the respondents be among the top 25%
who are “most adept” at crime solving have no better a
of the class? This only seems to be a contradiction: If the
success rate than the run-of-the-mill detectives. But an
number of respondents was smaller than the total number of
assumption being made here is that both groups’ caseload is
graduates, and if many of those who responded were students
identical, that both are assigned the same crime solving tasks.
with high academic rank, then it’s quite possible that half of
If, as (D) says, that assumption is false—if in fact the run-of-
the respondents graduated in the top 25% of the class. The
the-mill detectives are simply assigned easier cases to solve—
only “puzzle” here was if you didn’t see that the respondents
then the contradiction disappears. Indeed, (D) would mean
could be a subset of the total set of graduates, or if you
that the Èlite group gets harder cases to solve and yet enjoys
assumed that everyone in the class responded. (Then it’d be
the same success rate as the run-of-the-mill group, making the
a puzzle, all right! —see (D), below.) Anyhow, (A) makes it all
Èlite clearly worthy of their “most adept” status.
clear: The group of respondents included more than its share
(A), (B), and (C) all speak to the same irrelevant issue the of people who graduated with high ranks.
length of detectives’ tenure. All three utterly beg the question
(B) reinforces the accuracy of the numbers, without making it
of the relative crime solving skills of the two detective groups.
clearer how 50% of respondents fits into 25% of graduates.
(E) takes up a different, but equally irrelevant, issue—the
(C) tells us that some of the 25% who graduated at the top
prior experience of the run-of-the-mill group. It’s irrelevant,
of the class weren’t respondents, which makes it harder to
of course, because the alleged paradox only concerns the
see how 50% of the respondents came from the top 25% of
detectives’ skills while on the force.
the class.
(D) is even worse than (C). If the number of respondents to explain, this counterintuitive pair of developments. Note
roughly = the number of graduates, then we’d have more than that the author seems to think that a cause and effect ought
1/4 of the graduates saying that they were in the top 25th %ile to be at work—that is, the greater demand for carrots ought to
of graduates. Rather than explaining an apparent discrepancy, lead to less vitamin A deficiency. We’ll see.
(D) creates a real one. (A) A major increase in across-the-board population could, in
(E) deals with the criteria for academic rank, which is outside and of itself, explain why a vitamin deficiency could be more
the scope of the argument. We need help with the numbers, widespread irrespective of the demand for carrots. To put it
and (E) gives us no information that explains how 1/2 of another way, (A) drives a wedge between the cause and effect
respondents can be 1/4 of graduates. that the author implies in the stimulus. Eliminate.
(B) redefines the increased demand for carrots to focus on
29. (B) the kind of carrots that lose their vitamin A in processing.
Sometimes it’s easier to reach the correct answer to an Eliminate.
EXCEPT question by eliminating answer choices that are (C) does little more than explain the greater demand for
clearly wrong. carrots (i.e., they’re needed for these newly popular cuisines),
Four of the answer choices to this question will explain the but fails to address the vitamin A issue. So (C) is what we’re
different average lengths of visits to the museum, but the one looking for—the neutral answer. For the record:
we’re looking for won’t do so. This difference stems from the (D) drives a different kind of wedge between the two allegedly
special exhibitions the museum sometimes offers. When there causal phenomena: those demanding more carrots are only
is not a special exhibition, the museum’s attendance is lower, those whose vitamin A levels are traditionally low. Everyone
but the patrons tend to spend a longer time at the museum. else is eating no more carrots than before, and hence could
We can eliminate any answer choice that accounts for the experience an increase in vitamin A deficiency.
difference, so let’s evaluate the choices:
(E) goes beyond the sheer demand for carrots to suggest that
(A) Visitors who come to see the special exhibitions don’t supply is down. A resulting vitamin A deficiency on the part of
view as many exhibits, so it would make sense that they don’t the public would be most logical.
spend as much time at the museum. (A) helps to explain the
difference, so we can eliminate it. 31. (B)
(B) A plan to extend museum hours that wasn’t even put One or more choices in a Paradox question will serve to
into place couldn’t possibly affect how long visitors stay in deepen the paradox.
the museum. (B) wouldn’t explain the difference, so it is our
The 1980s example seems to be a contradiction to the
correct answer. Let’s quickly see how the last three choices
generalization that improved safety training leads to a
explain the difference in how long different visitors spent at
safer workplace: in the ’80s, accidents went up right
the museum:
after training improved. But if (B) is true—if the training
(C) You can get the prestige of having been somewhere improvements were instituted, as they tend to be, when more
just by walking in and walking out, so visitors just in it for employees were being hired—then who can be surprised
that purpose don’t need to stay long. (C) helps explain the at an increase in accidents during the window (“months
difference. immediately following”) between the hiring and the training’s
(D) If admission to special exhibitions has a time limit, then effectiveness?
visitors to those exhibits could easily have their visits cut short (A) A constant rate of job accidents deepens the paradox,
when compared to regular visitors. We can eliminate (D). acting as a direct contradiction to the expectation of the
(E) Without the opportunity to browse, it makes sense that industry experts.
visitors to special exhibitions wouldn’t spend as much time (C) This contradicts the statements in the stimulus: we’re
looking at different exhibits as regular patrons. Eliminate this expressly told that the training came first, followed by the
choice. immediate increase in accidents.
(D) If, as (D) suggests, the 1980s experience wasn’t just a blip
30. (C)
on the radar, then it’s even more paradoxical.
When four of the five choices resolve a paradox, remember
(E) Other improvements to safety notwithstanding, the training
you can choose to toss out those four or look for the neutral
that would make for a safer workplace seems to have done the
one directly.
opposite. (E) leaves the mystery unexplained.
A classic paradox: vitamin A deficiency is up even though
demand for carrots—a great source of vitamin A—is up as well.
How can that be? Four of the five choices will explain, or help
32. (E) The paragraph is about rates but (A) is about amounts; its
When you see “explain,” think Paradox. When there is comparison has no effect on the per calorie iron content.
no obvious paradox in the stimulus, think about what Meanwhile, (C), (D), and (E) all describe events about or
circumstances might be relevant to the events described. surrounding the ingestion of grapes and raisins. But the
Remember to characterize the choices for every EXCEPT paradoxical difference in iron-per-calorie exists long before
question. anyone eats the fruits, so these three choices are all beside
the point.
The four wrong answers to this question will all contain
possible explanations, or at least information that is relevant
35. (B)
to the circumstances in the stimulus. The one right answer,
then, will either be outside the scope of the argument, or will The paradox or “discrepancy” is that a country’s parliament
be a 180. All we know is that trade doubled between 1750 was all for the UN decision to send in an international
and 1765, that the first bank opened in 1750, and that the peacekeeping force, yet censured its own prime minister for
first banking regulations were implemented in 1765. Note promising troops to that force. (B) resolves the dilemma by
that the banking regulation came after the “doubling” that the implying that it wasn’t the promise of troops that irked the
question stem asks us to explain, so that banking regulation parliament, but rather the unconstitutional involvement of the
cannot by itself be relevant to the doubling. And since this prime minister in that promise. It’s not at all paradoxical, in
is an EXCEPT question, our first thought should be to check light of (B), for a parliament to say “We’re in favor of sending
for an answer that tries to use that regulation in some way. troops, but we not you, prime minister, are supposed to make
Choice (E) says that the initial banking regulation stimulated that judgment.”
the economy, which can’t help explain the doubling that came (A) sidesteps the issue of why the nation’s parliament was
before it, so this is our answer. angry at the p.m., and doesn’t even focus on that nation at all.
Each of choices (A), (B), (C), and (D) offers an explanation of The implication in (C) that the parliament is irked at having to
something that happened before or during the relevant time fork over the cash still doesn’t explain why they focused their
and could have contributed (or did contribute) to the trade ire on the p.m., or why they were gung-ho for the UN mission
doubling mentioned in the passage. in the first place. The public (D) is irrelevant to the paradox,
and anyway, the parliament did support the action so where’s
33. (A) the evidence that the public wouldn’t know that? (E), like (D),
wrongly brings in public sentiment. As relevant as the public’s
We learn that conscientious employees are less likely to find
views almost certainly would be in the real world, they have
new jobs within five months. Only (A) doesn’t help to explain
nothing to do with the seemingly contradictory stances of the
why this is. The fact that there are more non-conscientious
parliament as outlined here.
people looking for jobs doesn’t explain why any particular
non-conscientious person would find a job more quickly than
36. (D)
a conscientious person.
Remember that correlation never proves causation.
All of the other answer choices help to explain the apparent
paradox. If conscientious people are more picky about the jobs Usually, the GMAT shows correlation by telling us that
they choose (B) or have a less pressing financial need (E), then two things happened at the same time, or that one thing
we could see why it would take them longer to find jobs. If happened and then another thing followed. Here, they come
the conscientious perform poorly in interviews (C), or if the right out and use the word correlation, which should help
non-conscientious lie to appear more qualified (D), it is less you by reminding you that a correlation doesn’t imply cause
surprising that the non-conscientious are hired more quickly. and effect. People who live in Limone have high cholesterol,
but haven’t developed heart disease, despite the fact that
34. (B) heart disease and high cholesterol are correlated. So, there is
probably another factor which accounts for their relatively
What’s “puzzling” is that grapes and raisins sound identical—
good health, some other reason why they don’t have heart
the caramelized sugar and the loss of water seem to be the
trouble.
only differences—and yet the latter have “more iron per
(D) gives us that reason, a genetic quirk common to Limone
calorie.” Those differences must not be negligible, however;
that protects against heart disease. We should all be so lucky.
there must be something else going on. (B) provides it by
asserting that the caramelized sugar’s calories aren’t part of (A) Perhaps the residents of Limone come from long-lived
the raisin’s calorie count. As a result, we can infer that while families, and perhaps not. We don’t know from the stimulus
the grape and the raisin have the same amount of iron, the whether or not (A) would even apply to the residents of
latter’s fewer counted calories mean that it has more iron per Limone.
calorie.
(B) deals with “existing” cardiovascular disease, but the (A) Overcorrection and the resulting farsightedness would
residents of Limone “have not developed cardiovascular provide a reason why some RK patients would still need
disease.” corrective lenses. Eliminate.
(C) flat out tells us it doesn’t apply to the argument. Very low (B) suggests that RK may not always be completely effective,
blood sugar could account for Limone’s lack of cardiovascular which means it could correct some, but not all, of a patient’s
disease, except that the residents of Limone have normal nearsightedness. Eliminate.
blood sugar. (C) and (D) give clear reasons why some RK patients would
(E) Whether or not Limone is one of the parts of Italy that uses need corrective lenses. Eliminate.
olive oil, the residents of Limone still have high cholesterol. (E) makes an irrelevant comparison between patients who
Again, (E) doesn’t help to explain why people in Limone don’t choose RK and those who do not choose RK, which has
have cardiovascular disease. nothing to do with the results of the operation. This is the
correct answer.
37. (C)
Characterize the choices whenever you see the word EXCEPT. 39. (B)
Four of the answer choices will explain the decrease described The question stem tells us that we need to explain the
in the argument, and will be wrong choices. The fifth choice differences, just a variation on the resolve the paradox
will not explain the decrease, and is the answer we are looking theme. But remember, if an answer choice does explain the
for. The decrease in question is in the number of bass caught difference, it’s incorrect. We’re looking for the one that has
by anglers downstream from a new power plant. We might take no effect. We know that the Tasmanians and Australians
some time predicting reasons why a new power plant might are related, and we know that they were separated about
lower the number of bass in a stream, or why the anglers 10,000 years ago, when a land bridge disappeared. Within two
might have failed to catch as many as before, or we might just thousand years, there were significant differences between the
start looking through the choices to find four reasons why the Tasmanians and Australians. Each one of the incorrect answer
number of bass caught has dropped. choices will explain how those differences came about, but
(A) If the predators of bass are more prevalent in the water (B) doesn’t, so it’s correct. If the Tasmanians developed those
since the power plant began to operate, then it naturally weapons over 10,000 years ago, that doesn’t explain why they
follows that the number of bass in the river would have don’t have them now. It does explain why the Australians have
declined. (A) would explain the decrease. Eliminate. them, since the Australians were linked to the Tasmanians
by the land bridge when the weapons were developed. But it
(B) explains the decrease by giving us a reason why the bass
doesn’t explain why the Tasmanians no longer have them. So
are no longer in this part of the river. Eliminate.
(B) is the correct answer.
(C) reverses the situation. If the power plant’s operation made
Answer choices (A), (C), (D), and (E) all adequately explain how
it so that more anglers could fish in the river, we would expect
the Australians could have ended up with certain technological
them to catch more bass, not fewer. (C) would deepen the
innovations that the Tasmanians did not have.
paradox, which makes it the correct choice. For the record:
(D) If the section of the river in question was less attractive to 40. (E)
anglers, then it stands to reason there would be fewer anglers
The four wrong answer choices in a Paradox question will be
fishing, which would explain the decrease in the number of
out of scope or deepen the mystery.
fish caught. Eliminate.
Here’s a puzzle for you. How does 7% equal 9%? Both
(E) Less habitat means fewer bass to catch. Eliminate.
percentages purport to measure the fraction of unsalable
garments, all of which are recycled as scrap. The only possible
38. (E)
resolution is, as (E) suggests, that the two percentages are
A paradox is a statement that is contradictory, yet true; a calculated differently. Percentages calculated by count and
resolution of the paradox will explain how it can be true. percentages calculated by weight will, in general, be different
RK surgery is supposed to make eyeglasses or contact lenses if garments are of varying sizes.
for nearsightedness unnecessary, but in some cases people (A) Only unsalable garments are of concern here. Since
who have had the surgery still need contact lenses or glasses. garments with minor blemishes are sold, they are outside of
Four of the answer choices will resolve this discrepancy, while the scope of the argument.
the correct answer will not. We can resolve it by finding some
(B) does not resolve the paradox. What constitutes unsalable
reason why they would still need the correction.
does nothing to explain why the statistics differ.
(C) Could the discrepancy arise from overzealous inspectors? …and that’s what we find in choice (A). Picture yourself as a
No, because both the 7% and the 9% purport to measure the medieval merchant. Choice (A) tells us that you couldn’t be
same thing at the same time. sure of the purity, hence the worth, of any given refined-gold
(D) is of no help either, since it’s clear from the argument that coin. For that reason, it would be crazy, wouldn’t it, to fix a
both quantities are being calculated for the same year. price before you knew how you’d be paid? You might get fair
value or even more for your goods, if you were paid in refined
41. (A) coins that were really, really pure—but you also might get far
less, if the refined coins were trash. At least if you specified
Scope shifts are often the key to resolving discrepancies,
“Pay me in Senegalese gold,” you would always know
and they play a key role here as well. Parasites are bad for
how much gold would be present in coins made from that
stingrays. (What a surprise.) But their absence is bad news
unrefined, pure-as-the-day-it-was-mined gold. And that would
for the ecosystem in which the stingrays live. That’s the shift.
explain your making such a precondition.
How could parasites be good for the ecosystem even though
they are bad for some of the residents of that ecosystem? A (B) Silver? Bah. Never mentioned by the numismatist. That
precise pre-phrase would be tough, but something connecting silver tended to be the metal of everyday commerce and gold
parasites to the good of the ecosystem would do the job. That’s the metal of major trade says nothing about why merchants
what (A) does. If (A) is true, and stingray parasites depend on would express a preference for one type of gold over another,
other environmentally vulnerable organisms, then the presence or over silver for that matter.
of the parasite is an indicator of the presence of those other (C) , by lumping all gold coins together, brings us no closer
organisms. The parasites are still bad news for the stingray, but to understanding the preference for Senegalese gold. If
they could be good news for the ecosystem. If the parasites anything, (C)’s standard against which all coins could be
are around, then the species they depend on must be present measured might make refined-gold coins—those of
as well, which supports the notion that the system isn’t under demonstrably greatest purity—more desirable than ones
stress. But if the parasites are missing, then maybe those minted from Senegalese gold.
other species are missing, too, and this in turn would support (D) The coins from other countries are no more relevant
the notion that environmental stresses have taken a toll on here than silver was in (B). Where coins were minted is less
vulnerable organisms such as the shrimp and the oyster. significant than the origin of the gold they were minted from,
(B) is useless background information. You’d have to make and on this topic (D) is silent. It thus doesn’t help explain why
up a complicated story to connect the predatory habits of money made from Senegalese gold was the preferred method
stingrays to anything in this stimulus, but you know that of payment.
making up complicated stories isn’t rewarded on the (E) makes the merchants’ preference even more mysterious.
GMAT. Why would merchants needing refined gold for jewelry covet
(C) explains why parasites are bad for stingrays (or any hosts), coins whose gold was never refined?
but we knew that already.
(D) is too broad: A general statement about what might 43. (D)
constitute environmental stress doesn’t help us to resolve the The first step in resolving an apparent discrepancy is to
puzzle posed in the stimulus—why is something that’s bad locate the discrepancy. What are the seemingly contradictory
for an inhabitant of an environment a positive sign for the claims? Women in Naota who are employed full time earn only
environment as a whole? 80% as much as full-time employed males. However, as a
(E) explains why parasites cannot be too hostile to their hosts, group, employed women in Naota earn only 65% as much as
but doesn’t connect stingray parasites to the ecosystem in employed men in Naota. These statements seem to conflict
general. Like (C), (E) offers an interesting tidbit about parasite because they are both about the comparative earnings of men
life, but doesn’t connect that info to the main gist of the and women in Naota, but upon closer inspection, we see that
argument. they are talking about different things. “Full-time workers” is
a subset of all workers, and there’s no reason to assume that
42. (A) what must be true of the subset must be true of the larger
group. So, the resolution of this apparent discrepancy will
Why, this question asks, would merchants demand payment in
somehow address the distinction between employed women
Senegalese gold before even fixing prices? Well, we know that
and full-time employed women. Since the full-time employed
some coins contained gold far purer than natural Senegalese
women do comparatively better, there must be something
stuff, but those were minted from refined gold. There must
about the non-full-time employed women that is dragging the
have been something about never-refined Senegalese gold
average down. (D) provides such an explanation. If women
that merchants prized up front. That’s what we have to seek in
are more likely to be part-time workers (a.k.a. “non-full-time
the right answer…
employed”) who are typically paid less, then the earnings of council’s aim to preserve individual property rights. That’s the
employed women in general will be dragged down. So even inconsistency we’re trying to explain away.
if full-time employed women are earning 80% of full-time (B) Preventing an increase in property taxes and seeing to
employed men’s salaries, it could be that the part-time women the best interest of property owners is outside the scope
workers are earning even less and bringing down the overall of the argument, which centers on the conflict between the
average. supposed aim to preserve individual rights and the zoning
(A) might be part of an explanation of why the earnings laws that appear to obstruct those rights.
of employed women trail the earnings of employed men, (C) makes an irrelevant comparison. Knowing who places more
but ignores the issue of why full-time employed women importance on property rights, the council members or the
do comparatively better in relation to their full-time male property owners, does not affect the original argument, and
counterparts than all women do in relation to all men. thus does nothing to explain away the apparent inconsistency.
(B) involves a subset of full-time women workers: those who (E) is the main source of the paradox: zoning laws limit
work exactly the same occupations and under exactly the property rights. The issue to reconcile is how these restrictive
same conditions as men. We have no idea what percentage of zoning laws co-exist with the city council’s supposed aim of
full-time working women this subset comprises. The fact that preserving individual property rights. (E) simply repeats one
these women make nearly the same amount of money as the half of the paradoxical situation.
corresponding men is not surprising, but it does nothing to
clear up the general discrepancy between the earnings of full- 45. (A)
time working women and all working women. This remains a
Identification of the two seemingly incompatible observations
mystery; the best (B) may do is explain why the 80% figure is
is crucial in answering a Paradox question.
as high as it is.
The mystery we need to solve: patients who are given
(C) may explain why women in general are doing better than
vitamin E before heart surgery are less susceptible to
they used to, but also offers no insight on the discrepancy in
postoperative complications, although as soon as six hours
question.
after surgery, they do no better than patients who were not
(E) Other countries are outside the scope. given vitamin E before heart surgery. So why bother? Answer
choice (A) tells us that those first six hours after surgery are
44. (D) critical, and thus justifies giving patients vitamin E before
Here we must resolve the seeming contradiction between heart surgery.
the city council’s commitment to “individual property rights” (B) Whatever happens to patients after the six-hour time
and the fact that home owners are not allowed to make many period in which vitamin E makes a difference can’t possibly
changes to their properties because of zoning laws. These explain the recommendation to take vitamin E.
two positions seem to be contradictory because the notion of
(C) assumes without any support in the stimulus that a return
“individual rights” is often interpreted to mean “freedom to do
to normal heart function ends the six-hour period during
what one wants”—and the prohibition in question seems to
which vitamin E makes a difference for heart patients. If the
operate against that freedom. But what if the prohibition acts
assumption were justified, this answer choice deepens the
in some way to protect property rights; that is, if a person’s
mystery. The fact that some patients regain their normal heart
individual rights are protected, not restricted, by limiting
function in less than six hours makes the benefit of taking
what everyone can do? It protects a worker’s individual
vitamin E appear even more elusive.
rights, for example, to prohibit smoking in confined spaces.
Some people are not allowed to do what they want to do, (D) Like (C), this answer choice says nothing about why
namely smoke, but the individual’s right to breathe easy is doctors might recommend vitamin E to their heart surgery
preserved. Applying this line of thinking to the argument, we patients despite the lack of long-term benefit. Moreover,
can reconcile the apparent inconsistency if we can show that evidence that certain categories of patients are less likely
the restraint somehow works in the favor of individual property to develop postoperative complications than other patients
owners, and not against them. If (D) is true, then the zoning doesn’t necessarily tell us anything about the specific group
laws effectively do preserve individual property rights by we are interested in: heart surgery patients.
protecting individuals against infringement by others. If this is (E) If many patients (were they patients about to undergo heart
the case, the zoning laws no longer appear to conflict with the surgery?) receive a placebo instead of real vitamin E, that tells
city council’s aim, and the inconsistency has been resolved. us nothing about why doctors might recommend real vitamin E
(A) A periodic exemption from the zoning laws doesn’t show prior to heart surgery.
how the essence of the zoning laws can be reconciled with the
46. (C) (A) not only fails to explain the given paradox, it actually adds
A paradox, of course, is a seeming contradiction, and at to the mystery: Why would more beetles result from using one
first blush it does seem as if the decrease in wood workers trap if the beetles can’t even detect the scent of a single trap?
contradicts the increase in wood yield over the same period. (B) makes logical sense; one would expect several traps to
One possible resolution would be greater work efficiency on have this kind of advantage over a single trap. But if you
the part of the workers who remained employed, but that chose answer choice (B) on this basis alone, you forgot what
doesn’t appear as an answer choice. The other resolution, question you were answering. (B) shores up one element of
the one we get in answer choice (C), is that the workers and the stimulus (use several traps, and the number of beetles
the yield essentially have nothing to do with each other. will be greatly reduced), but entirely ignores the crux of the
Given that at least some of the 12,000 laid-off workers were discrepancy—we’re still in the dark as to why laying down a
wood processors, there’d be no paradox if the amount of single trap increases the number of beetles.
unprocessed wood went up after those workers left. (C) Here’s information that like (B), stands to reason, yet does
(A) Hearkening back to the 1950s does us no good, nor does nothing to erase the mystery in question.
the timber industry’s status relative to other industries. It’s all (E) OK, in this scenario, beetles check in, but some DO check
irrelevant. out (remember that roach motel commercial?). How does this
(B) presents a different inverse relationship—acres down, lead to an overall increase in the number of beetles in the rose
demand up. In and of itself this is no paradox (demand ought garden? This choice shows promise, but doesn’t go far enough
to go up when acreage decreases), and since answer choice to make a necessary connection that allows us to say “aha!
(B) doesn’t relate to the worker/wood yield issue, it doesn’t that explains it!”.
help us resolve the paradox we’ve got.
(D) ’s domestic use vs. exported wood contrast is totally 48. (C)
outside the scope, since the author never alludes to different Get as much information as you can from the question stem.
potential destinations for the wood. The stem for this question doesn’t just tell us that it’s a
(E) An increase in overall Ravonian unemployment that Paradox question; it also tells us exactly what the paradox
parallels the one in the timber industry gets us no closer to is. Before we even look at the stimulus, we know we’re trying
resolving the dilemma presented in the paragraph. to explain a difference in the amount of innovation into
different art forms, painting and sculpture, in nineteenth
47. (D) century France. From the stimulus, we learn that there was far
As soon as you see “apparent discrepancy” in the stem, you more innovation in painting than there was in sculpture. The
know what to expect: Something in the passage is going to reason appears to be the desires of the academy of art. We
seem odd—an unusual circumstance, a strange result—and learn that the academy was the main sponsor of French art in
it’s up to you recognize the choice that clears the whole thing the nineteenth century, and that it discouraged innovation.
up. The apparent discrepancy isn’t too hard to spot: Several Our job is to figure out why the academy’s sponsorship
traps, and the number of rose beetles decreases—so far discouraged innovation in sculpture, but wasn’t able to do so
so good. Only one trap, however, and the number of little in painting. We find a reason in (C): due to the relative costs
buggers increases. Huh? It’s odd, it’s strange, we’ve located of their media, painters didn’t need nearly as much money as
the apparent paradox. You’d think that one trap would reduce sculptors, so they could create unsponsored works. In other
the number of beetles, just not by as much as several traps. words, the academy of art could control sculptors, but there
You may not be able to precisely prephrase an answer, but were a large number of independent painters who were doing
you should be able to get a feel for what the right answer most or all of the innovating.
will sound like: There’s something about the mechanism of (A) and (B) would actually deepen the paradox, since it
this trap that makes them work well in bunches but horribly would suggest that the academy would have greater ability to
by themselves. That’s the only notion you need to recognize discourage innovation in painting than in sculpture.
answer choice (D)—throw any trap down, even just one, and (D) The overlap between sculptors and painters is irrelevant;
more beetles than one trap can handle are lured to the site. we’re concerned with the amount of innovation in each art
That explains why using only one trap will cause a net increase form as a whole, not the innovations of individual artists.
in beetles. But using several traps won’t attract many more
beetles than one trap will, and now the capacity for catching (E) Decreasing financial support might explain why artists
were more dependent on the academy of art, but it would,
the little guys is increased. Answer choice (D) explains why
if anything, only deepen the paradox of innovation in one
using several traps together does the job, while using just one
supported art form and not another.
trap alone makes the problem worse.
49. (D) security were surprisingly similar. The reason this is surprising
To explain a paradox or situation, interpret the differences in is that we would expect the workers in 1994 to feel much less
the circumstances before proceeding to the choices. secure, having experienced so many mergers, layoffs, etc.
All of the incorrect answer choices will help to explain how
Land-based predators prompt a different vervet monkey alarm
the results could be similar. The one that doesn’t is (B). How
call than do airborne predators. Why would that be? There
the employees feel about the jobs of others is irrelevant. The
must be some difference between the land and air threats that
argument concerns one’s confidence in one’s own job security.
demands different calls. (D) provides it: different calls allow
(B) is therefore correct.
the vervet monkeys to either climb a tree or hide in foliage,
depending on which threat approaches. (A) If the people surveyed came from small firms that weren’t
affected by mergers, reengineering and downsizing, then we
(A) Mere pitch changes unrelated to the origin of the predator
wouldn’t expect their perceptions to change as a result of what
don’t explain the behavior as the question demands.
took place at larger firms.
(B) By only speaking of the land-based predators, (B) cannot
(C) The fact that all this activity had been widely anticipated
fully explain the difference in behavior.
before the mid-1980s explains why the people surveyed in
(C) Different predators evidently require different calls. This 1984 would be just as insecure as people in 1994. While the
has nothing to do with whether a predator can approach in earlier survey respondents hadn’t experienced downsizing, the
different ways. fact that they could see it coming would probably be enough
(E) This one is so complicated that its sheer density might to make them feel insecure.
make it tempting. But how could the difference between one (D) If most of the downsizing was completed within a year after
predator that exists on an exclusive vervet monkey diet and the first survey, we wouldn’t expect the 1994 respondents to
others that have more universal tastes be relevant here? be too worried about their jobs; most of the bad things had
happened nine years earlier. So again, we wouldn’t expect
50. (E) much difference between the 1984 survey and the 1994
The facts: 1) Seat belt laws usually lower traffic fatalities. survey.
2) In the city in question, strict laws have been in effect (E) gives us a reason why the 1994 survey numbers were
for 2 years. 3) The safety records show that the number of higher than we might have expected. If (E) is true, that people
fatalities has stayed the same. There are many statements were more optimistic even in the face of hardships, then that
which could help explain this apparent paradox, and in fact would push the number of people who felt secure higher than
all of the answer choices except (E) do just this. (E) states that we would expect.
most of the people killed weren’t wearing their seatbelts, and
so only deepens the mystery by making us wonder “Why didn’t 52. (D)
the traffic laws encourage more people to wear their seat
The question stem tells us that we need to explain the
belts?”
discrepancy, so we can attack the stimulus looking for that
(A) points out the possibility that other, unrelated traffic discrepancy. The Keyword “Yet” identifies the surprising
regulations like higher speed limits might counteract the situation. Large malls generate a lot of economic activity, but
seatbelt laws, while (B) points out that differing accounting they don’t seem to add much to the local economy. From here,
methods could mean that the figures for the two periods may a prephrase might have struck you: perhaps malls don’t add
not be comparable. Note that seat belt laws decrease the much because (as (D) suggests) the dollars that are spent
percentage of traffic fatalities. If the city had an increase in the there would have been spent elsewhere in the community if
total number of drivers, the actual number of traffic fatalities the mall did not exist. So (D), if true, would explain why malls
might stay constant while the percent decreases (C). It’s don’t add as much to the local economy as one would think,
also apparent that laws may have little effect if they are not even though lots of cash is spent there.
enforced (D).
(A) and (E) suggest that the amount of activity generated by
a mall may be less than one might expect, but the stimulus
51. (B)
already tells us that malls generate a lot of activity. The
The question stem tells us that we’re looking to explain mystery is: why doesn’t that activity (however much it is)
surprising survey results. This just means that we have to translate into a corresponding increase in the local economy?
resolve the paradox. So, you should have first identified the On that question, (A) and (E) have nothing to offer.
paradox: even though massive downsizing occurred from
(B) and (C) both deepen the mystery by suggesting ways that
the mid-1980s to the mid-1990s, which the author tells us
malls could offer boosts to the local economy.
undermined employees’ job security, results from surveys
taken in 1984 and 1994 asking employees about their job
53. (B) in whichever task comes first. The fact that the list was shorter
A brief stimulus gives us an exception to a generality to than the paragraph applies to both groups equally.
ponder: It seems as if high price is always associated with (B) If the difference was that more experienced readers were
high quality, but tea doesn’t conform to this general rule— better at using context, why didn’t they always do better with
great tea is often no pricier than lower-quality teas. So, we’re the paragraph task? This is essentially the opposite of correct
looking for a reason why a better tea may not be priced higher choice (C).
than an inferior tea. Perhaps you were able to prephrase (D) might work if both groups performed better with the
an answer; it must have something to do with the pricing second task, but au contraire! Beginning readers performed
mechanism. But the reason can be just about anything, so it’s better with the first task when it was the paragraph task.
not worth racking our brains for too long before moving to the Again, this choice applies equally to both groups, and doesn’t
choices. Let’s evaluate: help explain the difference in performance.
(A) , (C) and (E) all contain general information about tea that (E) is outside the scope of the information presented in the
doesn’t distinguish in any way between the primo stuff and stimulus. We’re comparing the number of errors between tasks
the garbage. They’re simply too broad: Nothing about “all performed by each group, not between groups.
teas” (A), “all types of tea” (C), or “the price of tea generally”
is going to help us explain the apparent counterexample. 55. (E)
(B) and (D), however, get more specific: (B) focuses on a A discrepancy exists between the greater number of people
characteristic of low-quality tea that affects its price; that receiving food help, and the unchanging number of people
seems relevant enough. And (D) offers a comparison between qualifying for such help, only if those two groups are the same—
low- and high-quality teas. The problem with (D)’s comparison, if, in other words, the assumption is made that everyone who
however, is that it doesn’t include anything about the major qualifies, receives aid. But of course, there may be many
issue here, price. We can read in our own knowledge of supply more qualified citizens than aid recipients, in the same way
and demand, but if anything, that would force us to conclude that many more people are registered than actually vote. Now,
that the high-quality stuff is rare and should therefore be even if a reason exists as to why the recipient number might double
more expensive compared to the bad stuff. We can get fancy from one year to the next, then there’s no contradiction at all.
with (D), but it only brings us further from the explanation we And (E) provides such a reason: More eligible people knew
seek. (B) is therefore the winner, and it does make sense: If about the food aid in 1991 than in the previous year, in which
high demand for the low-quality tea keeps its price up, it’s case many of them might have applied for (and received) the
easier to understand why other tea may be better quality-wise, aid, thus explaining the extraordinary increase.
but priced in the same range.
(A) , (D) Something must have changed between 1990 and
1991 in order for the number of aid recipients to have
54. (C)
doubled. Any statement suggesting that things stayed the
This difficult “explain the phenomenon” question might be a same tends to deepen the paradox rather than resolve it.
good candidate for a quick guess, especially if you’re running
(B) speaks to how the budget estimates were made—utterly
out of time. If you’re not careful, a question like this one can
irrelevant to the issue at hand (e.g., how many people
sap a lot of your remaining time.
received aid).
What do we know about how the two groups of youngsters
(C) shows an increase in the number of unemployed residents
performed these tasks? The more experienced readers made
of Bayburg, a group that might well be in need of aid. But
fewer errors in whichever task they performed second. So, for
evidently this increase did nothing to increase the number
them, the essential ingredient was task order, not task type.
of eligible residents, because the stimulus asserts that that
The beginning readers always did better with one particular
number was flat. (Apparently as many eligible residents moved
task—reading the paragraph—and order made no difference.
out of Bayburg in 1991 as moved in.)
(C) helps to make sense of these facts. If, as (C) has it, the more
experienced readers sounded out words the first time they saw
56. (D)
them, whether in a paragraph or in a list, it stands to reason
that the next time they saw the same words, they’d make fewer John and his friends seem to have a difference of opinion—all
errors. But if the beginning readers relied on context to figure of them agree that a certain photo doesn’t look like him, while
out difficult words, then it makes sense that they would do he says it’s the only one in a series that does. We have put the
better with the paragraph, which provides such context. formal logic elements in boldface italics, because you know
that an GMAT LR question isn’t going to hinge on opinion;
(A) doesn’t explain why beginning readers always do better in
it’s got to revolve around a matter of logic. Why is John so
the paragraph task, while more experienced readers do better
insistent (and alone) on this being the one good likeness? We
need
an answer that both singles out this particular snapshot and century. However, “many parts of Asia” is fairly vague, and
provides a logical reason for the conflict over resemblance, may not include northern Asia, and “edible plant species” may
and (D) provides both. A photo taken from behind, a photo only be a subset of crops in general. (D), while possibly a 180
of his mirror image, will look like John to John because—in choice, is mostly outside the scope of the argument.
contrast to the face that he presents to everybody else—a (E) Scope shift! The issue is how much the yield will increase
mirror image is the way John sees himself, when shaving etc. or decrease, not the season in which the crops are farmed.
And the photo is “unique,” i.e., the only one having been
taken from that perspective. 58. (A)
(A) and (B) each do only half the job: Yes, each one does set this Don’t be too quick to dismiss an answer choice that seems
one photo apart from the others. But while his wearing everyday outside the scope of the stimulus—it may very well resolve
casual clothes (A) might prompt John to say “That’s the real the paradox.
me,” there’s no inherent reason why his friends would disagree.
The mystery posed by this question is why the proportion
(After all, they always see him in casual clothes, too.) And the
of left-handed Boldavians drops so significantly as the
use of a flash (B) might make that photo qualitatively different
population ages. The GMAT writers eliminate the two most
from all the rest, but wherefore the disagreement between John
obvious explanations: the proportion of Boldavians who are
and his pals? (B) leaves that unaddressed.
born left-handed has not changed, and neither have Boldavian
(C) and (E), meanwhile, can’t be correct because they don’t attitudes toward left-handedness (meaning that at no relevant
explicitly distinguish the one photo in question from all of time were naturally left-handed Boldavian children forced to
the others. Beyond that, like (A) and (B), each one presents use their right hand to fit in socially).
a special feature of that one photo but fails to suggest why
(A) seems outside the scope of the stimulus at first glance—
the friends and John might part company on the issue of true
how would gender differences in life expectancy explain the
likeness.
discrepancy? But the end of the answer choice ties the new
information to the stimulus quite nicely. If Boldavian men are
57. (C)
more likely than Boldavian women to be left-handed, and the
The word “paradox” in the stem tells us to keep our eyes peeled men die earlier, it would not be surprising for the proportion of
for a surprising result or finding, which in this case comes to left-handed Boldavians to diminish as the population ages.
us in the form of a prediction: Why would the scientists predict
(B) deepens the mystery. If left-handed people are no more
that crop yields will decrease during the next century when it
likely than right-handed people to be involved in accidents,
appears that the beneficial factors for crop growth (increased
that eliminates another possible explanation for the
temperature and humidity) will become the norm? One would
discrepancy between older and younger Boldavians, with
expect, under such conditions, for the yields to increase. Thus,
respect to the proportion of left-handed individuals.
the paradox. It’s difficult to prephrase an exact solution to the
paradox, but we can prephrase the general notion that some (C) Someone who is ambidextrous would not be classified as
other factor caused by the increased heat and humidity may being left-handed. A high value placed on ambidexterity might
adversely affect the crop yield. (C) follows from this idea: if influence the proportion of ambidextrous individuals, but it
would not make a difference in the proportion of left-handed
(C) is true, and the cool and dry climate has kept pests away,
individuals in the Boldavian population.
then increased heat and humidity may very well increase the
likelihood that pests will thrive, and thus take a bite out of crop (D) This answer choice invites us to confuse absolute numbers
yields. Under these circumstances, the scientists’ prediction is with percentages. A declining birthrate would reduce the
more understandable, and thus what seemed to be a paradox is number of left-handed Boldavians over time, but the stimulus
resolved. is all about proportions, and so a decline in the population
(A) What happens in southern Asia has no bearing on this wouldn’t affect the proportions at issue in this question.
argument unless a relevant connection can be made between (E) Information about the ceiling on the proportion of left-
southern and northern Asia. The stimulus gives us no basis for handed Boldavians tells us nothing about why the proportion
inferring such a connection, so (A) offers no help in resolving has declined so precipitously as the population has aged.
the paradox.
(B) To the contrary. (B) only deepens the mystery since it 59. (E)
provides a situation whereby the expected climate changes The correct answer to a Paradox question adds a piece of
would increase crop yields, contrary to the scientists’ evidence that harmonizes seemingly contrary information.
prediction. Why should the seller of an especially desirable item at an
(D) , if anything, reinforces the paradox by providing further auction be particularly interested in the protection offered by
evidence that the crop yields should increase in the next the setting of a minimum price for a successful bid? The usual
justification for setting such a reserve price for an item is that it as well, but that just deepens the mystery relating to those
protects the seller from buyers who make a token bid, gambling theaters that went bankrupt.
that nobody else will bid on the object. The right answer choice
will tell us that there might be an incentive for a prospective 61. (E)
buyer to “gamble” that nobody else will bid on a particularly The very term “skewed distribution” implies that by rights we
desirable object. (E) does just that: if prospective buyers tend should find evidence of cave lamps from every period of the
to believe that only an unreasonably high bid will be successful, Upper Paleolithic era. And pretty much evenly spaced, too, not
they might not bother bidding at all, leaving an opportunity for a predominantly from the late period. Since four of the choices
buyer who makes a token bid to be the high bidder. help to explain the skewing, the right answer must either
(A) deepens the mystery. If the high bidder is stuck with the deepen the mystery or lie outside its scope. (E) proves to be
object, and cannot defer to the next-highest bidder, the seller the latter, since the argument about the evidence of lamps has
is guaranteed whatever that high bid is. The high bidder would nothing to do with any variety or sameness of lamps, in the
not have an incentive to defer to a lower bidder unless he’d Magdalenian culture or any other. This is a numbers paradox.
made a bid in excess of the reasonable value of the object, There’d be little surprise at finding few cave lamps from the
which would be good for the seller (who would have no need early Upper Paleolithic period if we knew that identified artifacts
for the “protection” offered by setting a reserve price). of all types from that period are rare (A), or that caves from
(B) Whether the unsuccessful bidders are ever identified that period may have relied on fire pits as an alternative light
publicly tells us nothing about the relationship between the source (D). As for (B) and (C), both deal with the Magdalenians,
reserve price and protection of the seller. mentioned in the stimulus only as the dominant culture in
(C) If the reserve price for any object (not just the most the period of greatest lamp evidence. The greater number of
desirable ones) did not ensure a profit to the seller, it would unearthed Magdalenian sites (B) would certainly explain the
not offer the seller any meaningful protection. discovery of so much lamp evidence there. So would the more
efficient lamp-making techniques of the Magdalenians (C), since
(D) If prospective buyers could identify other prospective
“efficiency” implies “simpler and easier to crank them out.”
buyers, they would presumably be less likely to gamble by
making a token bid, and the seller of an extremely desirable
62. (E)
object would have less, not more need of the protection
offered by a reserve price. You’d think that a fatigued person who wasn’t sleeping very
much—and whose lack of sleep was contributing to the
60. (C) fatigue—would be told to get some sleep, but that is not
the recommendation, and we need a choice that explains
Identifying the apparent contradiction makes it easier to find
why. Your search may have led you to (E) by elimination.
the right answer for a Paradox question.
“Awakening” (A) and “nightmares” (D) aren’t part of the guy’s
How could there be a doubling of the number of movie problem, and while some may need less than 8 hours (C),
theaters going bankrupt in a year where box office receipts who knows whether he’s one of them? (B) might have been
increased by 40 percent? Clearly, the theaters that are
tempting if you reasoned that based on (B), the doctors would
declaring bankruptcy are not the ones seeing the increased
seek the patient’s remedy in the first 2 sleep hours and not
receipts. Answer choice (C) points to a very limited number of bother about the later hours. But that reasoning focuses on
theaters actually seeing profits from successful films, which solving the patient’s fatigue problem, whereas the question
would explain the discrepancy. focuses on avoiding a particular solution.
(A) points to a reason why theaters in general might see lower That’s really the key to understanding (E). Regardless of
profits, but doesn’t explain how box office receipts could have what is positively done to help the guy’s fatigue problem,
increased by 40 percent despite this increase in movie-making (E) explains why the doctors would avoid directing him
costs. to sleep more: Any worry over his inability to follow the
(B) Falling ticket prices at “some” theaters might explain “prescription” would likely only make things worse.
“some” of the bankruptcies, but standing alone, provides no
explanation for the increase in box office receipts. 63. (E)
(D) Increased film advertising paid for by entities other than The author’s conclusion (signaled by the contrast structural
theater owners might explain the increased box office receipts, clue “Nevertheless”) is that funding for preservation of
but makes the increase in bankruptcies even more mystifying. wetlands should be increased despite the fact that funding
(E) tells us that those theaters that experienced increased has grown much more quickly in the past than the area of
box office receipts also saw an increase in snack bar profits wetlands needing such protection. To reconcile this conclusion
to the evidence cited, we need to find an answer choice that
explains why funding is still inadequate despite the fact circumstances, a decline in smoking wouldn’t be expected to
that it has been increasing more quickly than the total area result in a corresponding decline in home-fire deaths.
of wetlands needing protection. (E) clears up the mystery: (C) picks up on the scope shift between the first and second
If funding was almost nonexistent ten years ago, as choice sentences: There’s been a significant decline in cigarette
(E) states, then it is quite possible that even though it has smoking in general, but that doesn’t mean the decline
increased sixfold, it is still inadequate. If (E) is true, the includes the subset of people who smoke in bed. If (C) is true,
conclusion seems reasonable despite the discrepancy in we’d expect most of the bed-smokers to keep on smoking,
growth rates cited in the evidence. which would certainly help resolve the apparent discrepancy.
(A) might explain why spending for preservation of wetlands (D) and (E) both help to resolve the paradox by offering other
has risen so dramatically in the last ten years (the money has reasons why deaths from fires have increased lately. If kitchen
been wasted), but would not provide us with a reason why fires or greater population densities are responsible for more
funding should be augmented in the future. In fact, one could home-fire deaths than before, the fact that no decline in fire
argue that if the money is being spent inefficiently, then it’s deaths has accompanied the decline in cigarette smoking is
best to spend that money in areas where it can be used to far less surprising.
achieve more substantial results.
(B) may explain one of the reasons why the government is 65. (E)
spending more to preserve wetlands now than it was ten years This one turns out to be a Paradox question even though
ago, but it does not explain why we should spend even more it’s not formally announced as such. It does seem a bit
to preserve wetlands in the future. Raising scientists’ salaries paradoxical that Korva’s population went up and yet its
does not justify even more spending, especially considering percentage of federal revenues declined; after all, revenue
that the funding has increased so much over the last ten years. sharing is pegged to population, isn’t it? Well, yes and no.
Nothing in this answer choice gives us a reason to believe that As the author describes it, the $ is divvied out in terms of
current funding is inadequate and should be increased. percentage of total population, not sheer number. Korva
(C) If anything, the fact that scientists can identify wetlands in can easily have gotten a smaller % of the cash despite its
need of restoration earlier would reduce the need for drastic, population increase, if Mitro or Guadar or both also saw a
desperate measures. population increase, and by a greater proportion. Which is
(D) mentions all natural resources while the stimulus only what (E) states. To test this, use the Denial Test: If (E) is false
mentions wetlands. —if neither of the other regions saw a greater percent increase
in population than Korva—then the paradox deepens.
64. (B) (A) Even if Korva does have the smallest of the three regions’
In this question we’re faced with another apparent populations, its population last year did increase, and without
discrepancy. This time, however, we’re looking for the one knowing what happened with the population of the other two
choice that doesn’t help explain the unusual result or finding. regions we have the paradox that Korva’s money decreased
First, we get some background info: smoking in bed has long while its population went up.
been a major cause of home fires. And here’s the surprise: (B) provides information about Korva’s change only. The issue
even though cigarette smoking has significantly declined over is why Korva got a smaller piece of the pie than did Mitro
the last twenty years, the number of people killed in home and Guadar, and choice (B) doesn’t address that any more
fires hasn’t declined accordingly. There are at least four good than (A).
reasons why this is so, and we’ll see those below in the wrong (C) could have been true, but need not. The smaller % of
answer choices. But we’re looking for the one that doesn’t help cash awarded to Korva can be explained so long as either of
solve the mystery. And (B) could only deepen the mystery: if the other two regions saw a greater percentage population
the fires caused by smoking in bed tend to take place after increase. That’s what correct choice (E) is all about. One of
the home’s occupants have fallen asleep, then home fires the two regions, Mitro or Guadar, could have had a stable
caused by smoking would seem even deadlier, since it’s hard percentage of total population, or even a drop.
to evacuate your home when you’re asleep. If fires caused by
(D) Contrary to choice (D), Korva could have had the greatest
smoking are especially deadly, then it is all the more confusing
numerical population increase, and still the paradox would
that a reduction in smoking has not led to a reduction of the
remain. The revenue dollars are pegged to percent of increase.
number of people killed in home fires. So (B) is no help, which
means that it is the credited response.
(A) If (A) is true, then bed-smoking fires aren’t likely to cause
many fire-related deaths in the first place. Under these
Strengthen/Weaken Questions
Directions: The questions in this section are based on the reasoning contained in brief statements or passages. For some questions, more
than one of the choices could conceivably answer the question. However, you are to choose the best answer; that is, the response that most
accurately and completely answers the question. You should not make assumptions that are by commonsense standards implausible,
superfluous, or incompatible with the passage.

1. Despite increasing international efforts to protect the 3. To the editor:


natural habitats of endangered species of animals, the
For generations, magnificent racehorses have been
rate at which these species are becoming extinct
bred in our area. Our most valuable product, however,
continues to rise. It is clear that these efforts are wasted.
has been generations of children raised with the
Which one of the following, if true, most weakens the character that makes them winners in the contests of
argument? life. Gambling is wrong, and children raised in an
atmosphere where the goal is to get something for
(A) Scientists are better able to preserve the habitats
nothing will not develop good character. Those who
of endangered species now than ever before.
favor developing good character in children over
(B) Species that would have become extinct have
gambling on horses should vote against allowing our
been saved due to the establishment of animal
first racetrack to be built.
refuges.
(C) Scientists estimate that at least 2000 species L.E.
become extinct every year.
Which one of the following, if true, most weakens L.E.’s
(D) Many countries do not recognize the increased
argument?
economic benefit of tourism associated with
preserved natural habitats. (A) If good character is developed in children early,
(E) Programs have been proposed that will transfer the children continue to have good character in
endangered species out of habitats that are in different environments.
danger of being destroyed. (B) In other areas with gambling, parents are able to
raise children of good character.
2. Being near woodlands, the natural habitat of bees, (C) In most areas with horse racing, the percentage
promotes the health of crops that depend on pollination. of adults who gamble increases gradually from
Bees, the most common pollinators, visit flowers far year to year.
from woodlands less often than they visit flowers close (D) Children whose parents gamble do not necessarily
to woodlands. gamble when they become adults.
(E) Where voters have had the opportunity to vote on
Which one of the following, if true, most strengthens the
horse racing, they have consistently approved it.
argument?
(A) The likelihood that a plant is pollinated increases
as the number of visits from pollinators
increases.
(B) Many bees live in habitats other than woodlands.
(C) Woodlands are not the natural habitat of all
pollinators.
(D) Some pollinators visit flowers far from their
habitats more often than they visit flowers close
to their habitats.
(E) Many crops that are not near woodlands depend
on pollination.
6. Company president: Grievance procedures should
4. Legislator: To keep our food safe, we must prohibit the allow the grievant and the respondent to select
use of any food additives that have been found to a mediator who will attempt to work out a
cause cancer. resolution. Grievances are costly and mediation
Commentator: An absolute prohibition is excessive. could help to resolve many of them. However,
Today’s tests can detect a single molecule of beginning mediation fairly late in the process, as
potentially cancer-causing substances, but we our human resources department proposes, would
know that consuming significantly larger amounts be relatively ineffective.
of such a chemical does not increase one’s risk Which one of the following, if true, most helps to
of getting cancer. Thus, we should instead set a justify the company president’s criticism of the human
maximum acceptable level for each problematic resources department’s proposal?
chemical, somewhat below the level at which the
substance has been shown to lead to cancer but (A) People who file grievances are unreasonable and
above zero. would resist listening to a mediator.
(B) Many disagreements are already being solved
Of the following, which one, if true, is the logically without the intervention of a mediator.
strongest counter the legislator can make to the (C) Adversaries’ positions tend to harden as a dispute
commentator’s argument? wears on, making compromise less likely.
(A) The level at which a given food additive has been (D) Respondents tend to be supervisors who cannot
shown to lead to cancer in children is generally give in to employees without losing authority.
about half the level at which it leads to cancer in (E) The mediation process itself is likely to cost
adults. as much in time and money as the present
(B) Consuming small amounts of several different grievance procedures.
cancer-causing chemicals can lead to cancer
even if consuming such an amount of any one 7. Advertisement: At most jewelry stores, the person
cancer-causing chemical would not. assessing the diamond is the person selling it, so
you can see why an assessor might say that a
(C) The law would prohibit only the deliberate diamond is of higher quality than it really is. But
addition of cancer-causing chemicals and would because all diamonds sold at Gem World are
not require the removal of naturally occurring certified in writing, you’re assured of a fair price
cancer-causing substances. when purchasing a diamond from Gem World.
(D) For some food additives, the level at which the
substance has been shown to lead to cancer The reasoning in the advertisement would be most
is lower than the level at which the additive strengthened if which one of the following were true?
provides any benefit. (A) Many jewelry stores other than Gem World also
(E) All food additives have substitutes that can be provide written certification of the quality of
used in their place. their diamonds.
(B) The certifications of diamonds at Gem World are
5. Midlevel managers at large corporations are unlikely to written by people with years of experience in
suggest reductions in staff in their own departments even appraising gems.
when these departments are obviously overstaffed. (C) The diamonds sold at Gem World are generally of
Each of the following, if true, supports the claim above higher quality than those sold at other jewelry
EXCEPT: stores.
(D) The diamond market is so volatile that prices of
(A) The compensation paid to midlevel managers is the most expensive diamonds can change by
greater when they supervise more workers. hundreds of dollars from one day to the next.
(B) Midlevel managers have less work to do when (E) The written certifications of diamonds at Gem
their departments are overstaffed. World are provided by an independent company
(C) Staff morale and productivity often suffer when of gem specialists.
workers are laid off.
(D) Departmental workloads at most large
corporations increase and decrease significantly
and unpredictably.
(E) Many large corporations allow managers to offer
early retirement as a means of reducing staff.
8. In response to office workers’ worries about the health 10. Nutritionist: Recently a craze has developed for home
risks associated with using video display terminals juicers, $300 machines that separate the pulp of
(VDTs), researchers asked office workers to estimate fruits and vegetables from the juice they contain.
both the amount of time they had spent using VDTs and Outrageous claims are being made about the
how often they had suffered headaches over the previous benefits of these devices: drinking the juice
year. According to the survey, frequent VDT users they produce is said to help one lose weight or
suffered from headaches more often than other office acquire a clear complexion, to aid digestion, and
workers did, leading researchers to conclude that VDTs even to prevent cancer. But there is no indication
cause headaches. that juice separated from the pulp of the fruit or
Which one of the following, if true, most undermines vegetable has any properties that it does not have
the researchers’ conclusion? when unseparated. Save your money. If you want
carrot juice, eat a carrot.
(A) Few of the office workers surveyed participated
in regular health programs during the year in Which one of the following, if true, most calls into
question. question the nutritionist’s argument?
(B) In their study the researchers failed to ask the (A) Most people find it much easier to consume a
workers to distinguish between severe migraine given quantity of nutrients in liquid form than to
headaches and mild headaches. eat solid foods containing the same quantity of
(C) Previous studies have shown that the glare the same nutrients.
from VDT screens causes some users to suffer (B) Drinking juice from home juicers is less healthy
eyestrain. than is eating fruits and vegetables because such
(D) Office workers who experienced frequent juice does not contain the fiber that is eaten if
headaches were more likely than other workers one consumes the entire fruit or vegetable.
to overestimate how much time they spent using (C) To most people who would be tempted to buy
VDTs. a home juicer, $300 would not be a major
(E) Office workers who regularly used VDTs expense.
experienced the same amount of job-related (D) The nutritionist was a member of a panel that
stress as workers who did not use VDTs. extensively evaluated early prototypes of home
juicers.
9. A plausible explanation of the disappearance of the (E) Vitamin pills that supposedly contain nutrients
dinosaurs is what is known as the comet theory. A large available elsewhere only in fruits and vegetables
enough comet colliding with Earth could have caused a often contain a form of those compounds that
cloud of dust that enshrouded the planet and cooled the cannot be as easily metabolized as the varieties
climate long enough to result in the dinosaurs’ demise. found in fruits and vegetables.
Which one of the following statements, if true, most
seriously weakens the argument?
(A) One of the various schools of paleontology
adheres to an explanation for the disappearance
of the dinosaurs that is significantly different
from the comet theory.
(B) Various species of animals from the same era as
the dinosaurs and similar to them in physiology
and habitat did not become extinct when the
dinosaurs did.
(C) It cannot be determined from a study of dinosaur
skeletons whether the animals died from the
effects of a dust cloud.
(D) Many other animal species from the era of the
dinosaurs did not become extinct at the same
time the dinosaurs did.
(E) The consequences for vegetation and animals
of a comet colliding with Earth are not fully
understood.
12. Solicitor: Loux named Zembaty executor of her will. Her
11. Several companies will soon offer personalized only beneficiary was her grandson, of whom she
electronic news services, delivered via cable or was very fond. Prior to distributing the remainder
telephone lines and displayed on a television. People
to the beneficiary, Zembaty was legally required
using these services can view continually updated stories
to choose which properties in the estate should be
on those topics for which they subscribe. Since these
sold to clear the estate’s heavy debts. Loux never
services will provide people with the information, they
expressed any particular desire about the Stoke
are looking for more quickly and efficiently than printed
Farm, which includes the only farmland in her
newspapers can, newspaper sales will decline drastically
estate. Thus, it is unlikely that Loux would have
if these services become widely available.
had any objection to Zembaty’s having
Which one of the following, if true, most seriously sold it rather than having transferred it to her
weakens the argument? grandson.
(A) In reading newspapers, most people not only Which one of the following, if true, most weakens the
look for stories on specific topics but also like solicitor’s argument?
to idly browse through headlines or pictures for
(A) The estate’s debts could not have been cleared
amusing stories on unfamiliar or unusual topics.
without selling the Stoke Farm.
(B) Companies offering personalized electronic news
(B) Loux repeatedly told her grandson that she would
services will differ greatly in what they charge
take care of him in her will.
for access to their services, depending on how
(C) Loux was well aware of the legal requirements
wide a range of topics they cover.
the executor of her will would have to satisfy.
(C) Approximately 30 percent of people have never
(D) The Stoke Farm was the main cause of the
relied on newspapers for information but instead
estate’s debts.
have always relied on news programs broadcast
on television and radio. (E) Loux’s grandson had repeatedly expressed his
desire to own a farm.
(D) The average monthly cost of subscribing to
several channels on a personalized electronic
13. Medical doctor: Sleep deprivation is the cause of many
news service will approximately equal the cost
social ills, ranging from irritability to potentially
of a month’s subscription to a newspaper.
dangerous instances of impaired decision making.
(E) Most people who subscribe to personalized
Most people today suffer from sleep deprivation
electronic news services will not have to pay
to some degree. Therefore, we should restructure
extra costs for installation since the services will
the workday to allow people flexibility in
use connections installed by cable and telephone
scheduling their work hours.
companies.
Which one of the following, if true, would most
strengthen the medical doctor’s argument?
(A) The primary cause of sleep deprivation is
overwork.
(B) Employees would get more sleep if they had
greater latitude in scheduling their work hours.
(C) Individuals vary widely in the amount of sleep
they require.
(D) More people would suffer from sleep deprivation
today than did in the past if the average number
of hours worked per week had not decreased.
(E) The extent of one’s sleep deprivation is
proportional to the length of one’s workday.
14. Peter: Because the leaves of mildly drought-stressed 15. A recent study suggests that Alzheimer’s disease,
plants are tougher in texture than the leaves of which attacks the human brain, may be caused by a
abundantly watered plants, insects prefer to feed virus. In the study, blood from 11 volunteers, each of
on the leaves of abundantly watered plants. whom had the disease, was injected into rats. The rats
Therefore, to minimize crop damage, farmers eventually exhibited symptoms of another degenerative
should water crops only just enough to ensure neurological disorder, Creutzfeldt-Jakob disease,
that there is no substantial threat, from a lack of which is caused by a virus. This led the scientist who
water, to either the growth or the yield of the conducted the study to conclude that Alzheimer’s
crops. disease might be caused by a virus.
Jennifer: Indeed. In fact, a mildly drought-stressed plant Which one of the following statements, if true,
will divert a small amount of its resources from would most strengthen the scientist’s hypothesis that
normal growth to the development of pesticidal Alzheimer’s disease is caused by a virus?
toxins, but abundantly watered plants will not.
(A) Alzheimer’s disease in rats is not caused by a
Which one of the following, if true, most strengthens virus.
Peter’s argument? (B) Creutzfeldt-Jakob disease affects only motor
(A) The leaves of some crop plants are much larger, nerves in rats’ limbs, not their brains.
and therefore absorb more water, than the leaves (C) The virus that causes Creutzfeldt-Jakob disease in
of some other crop plants. rats has no effect on humans.
(B) In industrialized nations there are more crops (D) The symptoms known, respectively, as
that are abundantly watered than there are crops Creutzfeldt-Jakob disease and Alzheimer’s
grown under mild drought stress. disease are different manifestations of the same
(C) Insect damage presents a greater threat to crop disease.
plants than does mild drought stress. (E) Blood from rats with Creutzfeldt-Jakob disease
(D) Farmers are not always able to control the amount produced no symptoms of the disease when
of water that their crops receive when, for injected into other experimental rats.
instance, there are rainstorms in the areas where
their crops are growing. 16. Marmosets are the only primates other than humans
(E) Mexican bean beetles are more likely to feed on known to display a preference for using one hand rather
than the other. Significantly more marmosets are left-
the leaves of slightly drought-stressed soybeans
than oak lace bugs are to feed on the leaves of handed than are right-handed. Since infant marmosets
abundantly watered soybeans. engage in much imitative behavior, researchers
hypothesize that it is by imitation that infant marmosets
learn which hand to use, so that offspring reared by left-
handed parents generally share their parents’
handedness.
Which one of the following, if true, most supports the
researchers’ hypothesis?
(A) A study conducted on adult marmosets revealed
that many were right-handed.
(B) Right-handed marmosets virtually all have at least
one sibling who is left-handed.
(C) According to the study, 33 percent of marmosets
are ambidextrous, showing equal facility using
either their left hand or their right hand.
(D) Ninety percent of humans are right-handed, but
those who are left-handed are likely to have at
least one left-handed parent.
(E) Marmosets raised in captivity with right-handed
adult marmosets to whom they are not related
are more likely to be right-handed than left-
handed.
18. A recent national study of the trash discarded in several
17. Juan: Unlike the ancient Olympic games on which representative areas confirmed that plastics constitute a
they are based, the modern Olympics include smaller proportion of all trash than paper products do,
professional as well as amateur athletes. But since whether the trash is measured by weight or by volume.
amateurs rarely have the financial or material The damage that a given weight or volume of trash does
resources available to professionals, it is unlikely to the environment is roughly the same whether the
that the amateurs will ever offer a serious trash consists of plastics or paper products. Contrary to
challenge to professionals in those Olympic popular opinion, therefore, the current use of plastics
events in which amateurs compete against actually does less harm to the environment nationwide
professionals. Hence, the presence of professional than that of paper products.
athletes violates the spirit of fairness essential to
the games. Which one of the following, if true, most strengthens the
argument?
Michiko: But the idea of the modern Olympics is to
showcase the world’s finest athletes, regardless (A) A given weight of paper product may increase
of their backgrounds or resources. Hence, in volume after manufacture and before being
professionals should be allowed to compete. discarded as trash.
(B) According to popular opinion, volume is a
Which one of the following, if true, most seriously more important consideration than weight in
undermines Juan’s argument? predicting the impact of a given quantity of trash
(A) In general, amateur athletes tend to outnumber on the environment.
professional athletes in the modern Olympics. (C) The sum of damage caused to the environment by
(B) In certain events in the modern Olympics the paper trash and by plastic trash is greater than
best few competitors are amateurs; in certain that caused by any other sort of trash that was
other events the best few competitors are studied.
professionals. (D) The production of any paper product is more
(C) The concept of “amateur” and “professional” harmful to the environment than is the
athletics would have been unfamiliar to the production of an equal weight or volume of any
ancient Greeks on whose games the modern plastic.
Olympics are based. (E) The proportion of plastic trash to paper trash
(D) In the modern Olympics there has been no varies from one part of the country to another.
noticeable correlation between the financial or
material resources expended on the training of 19. Limited research indicates that therapeutic intervention
individual athletes and the eventual performance before the onset of mental disorders can mitigate factors
of those athletes. identified as major contributors to them. But a much
(E) Many amateur athletes who take part in more comprehensive research program is needed to
international competitions receive no financial verify these results and allow for the design of specific
or material support from the governments of the health care measures. Thus, in order to explore a
countries that the amateurs represent. potential means of cost-effectively helping people prone
to mental disorders, we should increase funding for
intervention research.
Which one of the following, if true, most strengthens the
argument?
(A) Most minor mental disorders are more expensive
to treat than other minor health problems.
(B) Prevention research can be coordinated by
drawing together geneticists, neurologists, and
behavioral scientists.
(C) Reducing known risk factors for mental disorders
is relatively inexpensive compared to the long-
term treatment required.
(D) Current funding for intervention research is now
higher than it has ever been before.
(E) Once a mental disorder disappears, there is a fair
chance that it will recur, given that complete
cures are rare.
20. Galanin is a protein found in the brain. In an experiment, 22. There should be a greater use of gasohol. Gasohol is
rats that consistently chose to eat fatty foods when a mixture of alcohol and gasoline, and has a higher
offered a choice between lean and fatty foods were octane rating and fewer carbon monoxide emissions than
found to have significantly higher concentrations of straight gasoline. Burning gasohol adds no more carbon
galanin in their brains than did rats that consistently dioxide to the atmosphere than plants remove by
chose lean over fatty foods. These facts strongly support photosynthesis.
the conclusion that galanin causes rats to crave fatty Each of the following, if true, strengthens the argument
foods. above EXCEPT:
Which one of the following, if true, most supports the (A) Cars run less well on gasoline than they do on
argument? gasohol.
(A) The craving for fatty foods does not invariably (B) Since less gasoline is needed with the use of
result in a rat’s choosing those foods over lean gasohol, an energy shortage is less likely.
foods. (C) Cars burn on the average slightly more gasohol
(B) The brains of the rats that consistently chose to per kilometer than they do gasoline.
eat fatty foods did not contain significantly more (D) Gasohol is cheaper to produce and hence costs
fat than did the brains of rats that consistently less at the pump than gasoline.
chose lean foods. (E) Burning gasoline adds more carbon dioxide to the
(C) The chemical components of galanin are present atmosphere than plants can remove.
in both fatty foods and lean foods.
(D) The rats that preferred fatty foods had the higher 23. Automobile-emission standards are enforced through
concentrations of galanin in their brains before annual inspection. At those inspections, cars are tested
while idling; that is, standing still with their engines
they were offered fatty foods. running. Testing devices measure the levels of various
(E) Rats that metabolize fat less efficiently than do pollutants as exhaust gases leave the tail pipe.
other rats develop high concentrations of galanin
in their brains. Which one of the following, if true, most strongly
indicates that current enforcement of automobile-
21. The existing works of ancient literature, science, and emission standards might be ineffective in controlling
philosophy do not represent the most original work overall pollutant levels?
produced in ancient times. The most compelling
(A) As an emission-control technology approaches
evidence for this view is that the existing works
its limits, any additional gains in effectiveness
frequently refer to other works that we no longer have.
become progressively more expensive.
Which one of the following statements, if added to the (B) The testing devices used must be recalibrated
argument, most helps to justify its conclusion? frequently to measure pollutant levels with
(A) Works that contain frequent references to other acceptable accuracy.
works tend to be derivative. (C) The adjustments needed to make a car idle
(B) Many extant works have laudable things to say cleanly make it likely that the car will emit high
about the missing works. levels of pollutants when moving at highway
(C) A surprisingly large number of ancient texts have speeds.
been irretrievably lost. (D) Most car owners ask their mechanics to make
(D) Subversive ideas tend to be suppressed whenever sure that their cars are in compliance with
they are proposed. emission standards.
(E) Most current ideas regarded as original were (E) When emission standards are set, no allowances
already proposed in ancient times. are made for older cars.
26. Jane: Television programs and movies that depict
24. The town of Springhill frequently must declare a water violence among teenagers are extremely popular.
emergency, making it temporarily unlawful to use Given how influential these media are, we have
water for such nonessential purposes as car washing. good reason to believe that these depictions cause
These emergencies could be avoided if Springhill young people to engage in violent behavior.
would introduce permanent economic incentives for Hence, depictions of violence among teenagers
water conservation. Actually, Springhill discourages should be prohibited from movies and television
conservation because each household pays a modest programs, if only in those programs and movies
monthly flat fee for any amount of water below a certain promoted to young audiences.
usage threshold, and a substantial per-liter rate only after
the threshold is reached. Maurice: But you are recommending nothing short
of censorship! Besides which, your claim that
Which one the following, if true, most strengthens the television and movie depictions of violence
argument? cause violence is mistaken: violence among
(A) The Springhill authorities do a poor job of young people predates movies and television by
enforcing its water emergency laws and many centuries.
people break the laws without incurring a Which one of the following, if true, most strengthens
penalty. Jane’s argument?
(B) The town council of Springhill recently refused to
raise the threshold. (A) The most violent characters depicted in movies
(C) The threshold is kept at a high enough level and on television programs are adult characters
to exceed the water requirements of most who are portrayed by adult actors.
households in Springhill. (B) The movies that have been shown to have the
(D) The threshold is not as high in Springhill as it is most influence on young people’s behavior are
in neighboring towns. those that are promoted to young audiences.
(E) The threshold remains at the predetermined level (C) The people who make the most profits in the
specified by law until a change is approved by movie and television industry are those who can
the Springhill town council. successfully promote their work to both young
and old audiences.
25. Market research traditionally entails surveying consumers (D) Many adolescents who engage in violent behavior
about why they buy and use particular products and had already displayed such behavior before they
brands. Observational research—actually watching were exposed to violence in movies.
consumers shopping and interacting with products— (E) Among the producers who make both movies and
is now increasingly used by market researchers to television programs, many voluntarily restrict
supplement surveys. Market researchers claim that the subject matter of films directed toward
observational research yields information about consumer young audiences.
behavior that surveys alone cannot provide.
Which one of the following, if true, provides the
strongest support for the market researchers’ claim?
(A) Even consumers who are unable to explain their
preference for or rejection of particular brands
reveal which brands they are considering by
picking up and putting down products while they
are shopping.
(B) Market researchers find that consumers
are almost always willing to participate in
observational research for which the consumer is
paid by the hour.
(C) Consumers are becoming increasingly self-
conscience about their buying habits, and some
consumers have stopped buying some items that
they normally used to buy.
(D) Market researchers say they find data collection
more enjoyable in observational research than in
survey research, because observational research
requires more creative judgment on their part.
(E) Consumers are more likely to respond to oral
surveys than they are to respond to written
questionnaires.
27. Space programs have recently suffered several setbacks 29. Generic drugs contain exactly the same active
with respect to their large projects, and much money has ingredients as their brand-name counterparts, but usually
been lost. Consequently, these grand projects should be cost much less to develop, produce, and market. So,
abandoned in favor of several small ones. generic drugs are just as effective as their brand-name
counterparts, but cost considerably less.
Which one of the following, if true, provides the most
support for the reasoning above? Which one of the following, if true, most strengthens the
argument?
(A) The cost of starting a space project increases
every year. (A) The ingredients used in the manufacture of brand-
(B) It is just as easy to revise, and even scrap, small name drugs cost no more than the ingredients
projects as it is large ones. used to produce their generic counterparts.
(C) Large projects are intrinsically more likely to fail (B) Generic drugs are no more likely than
and so are more financially risky than small brand-name drugs to suffer from defects in
projects. composition.
(D) Project managers prefer to work on small projects (C) Generic drugs are just as likely as brand-name
rather than large ones. drugs to be readily available in pharmacies.
(E) Large space projects can explore a few places (D) The higher costs of brand-name drugs underwrite
thoroughly, while small projects can investigate drug companies’ heavy investment in research.
more regions, though less thoroughly. (E) Because of advertising, doctors frequently
prescribe brand-name drugs by their brand
28. In an experiment testing whether hyperactivity is due to name, rather than by their chemical name.
a brain abnormality, the brain activity of 25 hyperactive
adults was compared to the brain activity of 25 adults 30. Nutrition education in schools once promoted daily
who were not hyperactive. The tests revealed that the consumption of food from each of the “four food
hyperactive adults had much less brain activity in the groups”: milk, meat, fruit and vegetables, and breads
premotor cortex, a region of the brain believed to control and cereals. This recommendation was, however,
action, than did the non-hyperactive adults. The dangerous to health.
experimenters concluded that diminished activity in the Each of the following, if true, provides support for the
premotor cortex is one cause of hyperactivity. critique above EXCEPT:
Which one of the following, if true, most undermines
(A) The division into four groups gave the impression
the conclusion drawn by the experimenters?
that an equal amount of each should be
(A) Some of the non-hyperactive adults in the consumed, but milk and meat tend to contain
study had children who suffer from fats that promote heart disease and cancer and
hyperactivity. should be eaten in lesser amounts.
(B) The hyperactive adults who participated in (B) The omission of fish, which contains beneficial
the experiment varied in the severity of their oils, from the names of groups in the list gave
symptoms. erroneous impression that it is less healthy as a
(C) The neuropsychologists who designed the food than is red meat.
experiment were not present when the tests were (C) A healthy diet should include the consumption
performed. of several different fruits and vegetables daily,
(D) All of the hyperactive adults in the study had but the recommendation was often interpreted as
been treated for hyperactivity with a medication satisfied by the consumption of a single serving
that is known to depress activity in some regions of a fruit or vegetable.
of the brain, while none of the non-hyperactive (D) The recommendation that some food from the
adults had been so treated. fruit and vegetable group be consumed daily
(E) The test was performed only on adults because constituted a reminder not to neglect this group,
even though the method by which the test which provides needed vitamins, minerals, and
measured brain activity is harmless to adults, it fiber.
does require the use of radiation, which could be (E) Encouraging the daily consumption of some
harmful to children. product from each of the four food groups gave
the impression that eating in that manner is
sufficient for a healthy diet, but eating in that
manner is consistent with the overconsumption
of sweets and fats.
33. Commercial passenger airplanes can be equipped with a
31. During the 1980’s Japanese collectors were very active collision-avoidance radar system that provides pilots
in the market for European art, especially as purchasers with information about the proximity of other airplanes.
of nineteenth-century Impressionist paintings. This Because the system warns pilots to take evasive action
striking pattern surely reflects a specific preference on when it indicates a possible collision, passengers are
the part of many Japanese collectors for certain aesthetic safer on airplanes equipped with the system than on
attributes they found in nineteenth-century Impressionist comparable airplanes not so equipped, even though
paintings. the system frequently warns pilots to evade phantom
Which one of the following, if true, most strongly airplanes.
supports the explanation above? Which one of the following, if true, most strengthens the
(A) Impressionist paintings first became popular argument?
among art collectors in Europe at the beginning (A) Evasive action taken in response to the system’s
of the twentieth century. warnings poses no risk to the passengers.
(B) During the 1980s, the Japanese economy (B) Commercial passenger airplanes are in greater
underwent a sustained expansion that was danger of colliding with other airplanes while on
unprecedented in the country’s recent history. the ground than they are while in flight.
(C) Several nineteenth-century Impressionist (C) Commercial passenger airplanes are rarely
painters adopted certain techniques and visual involved in collisions while in flight.
effects found in Japanese prints that are highly (D) A study by ground-based air traffic controllers
esteemed in Japan. found that 63 percent of the warnings by the
(D) During the 1960s and 1970s, the prices of system were invalid.
nineteenth-century Impressionist paintings (E) The collision-avoidance radar system is run by a
often exceeded the prices of paintings by older computerized device on the plane that scans the
European masters. sky and calculates the distances between planes.
(E) During the 1980s, collectors from Japan and
around the world purchased many paintings and 34. First legislator: Medical research is predominantly done
prints by well-known twentieth-century Japanese on groups of patients that include only men. For
artists. example, the effects of coffee drinking on health
are evaluated only for men, and studies are
32. In Yasukawa’s month-long study of blackbirds, the lacking on hormone treatments for older women.
percentage of smaller birds that survived the duration of Government-sponsored medical research should
the study exceeded the percentage of larger birds that be required to include studies of women.
survived. However, Yasukawa’s conclusion that size is a
determinant of a blackbird’s chances of survival over a Second legislator: Considerations of male/female
month-long period is probably mistaken, since smaller balance such as this are inappropriate with
blackbirds are generally younger than larger ones. respect to research; they have no place in science.

Which one of the following, if true, indicates that Which one of the following rejoinders, if true, most
the criticism of Yasukawa’s research is based on a directly counters the second legislator’s objection?
misunderstanding of it? (A) Government-sponsored research is supported by
(A) Yasukawa compared the survival chances of two all taxpayers, both male and female.
different species of blackbirds, a larger and a (B) Serving as a subject for medical research can
small species, rather than of different sizes of provide a patient access to new treatments but
birds within one species. also can subject the patient to risks.
(B) Yasukawa examined blackbirds in their natural (C) Government-sponsored medical research is often
habitat rather than in captivity. done in military hospitals or prisons that hold
(C) Yasukawa did not compare the survival chances only male residents.
of blackbirds with those of other kinds of birds. (D) The training of male and female scientists does
(D) Yasukawa noted that the larger blackbirds had not differ according to their sex.
more success in fights than did the smaller (E) Restriction to males of the patient base on which
blackbirds. data are collected results in inadequate science.
(E) Yasukawa noted that the larger blackbirds
tended to have more firmly established social
hierarchies than did the smaller blackbirds.
35. Even though trading in ivory has been outlawed by 37. Advances in photocopying technology allow criminals
international agreement, some piano makers still use with no printing expertise to counterfeit paper currency.
ivory, often obtained illegally, to cover piano keys. One standard anticounterfeiting technique,
Recently, experts have devised a synthetic ivory that, microprinting, prints paper currency with tiny designs
unlike earlier ivory substitutes, has found favor with that cannot be photocopied distinctly. Although
concert pianists throughout the world. But because counterfeits of microprinted currency can be detected
piano makers have never been major consumers of ivory, easily by experts, such counterfeits often circulate
the development of the synthetic ivory will therefore widely before being detected. An alternative, though
probably do little to help curb the killing of elephants, more costly, printing technique would print currency
from whose tusks most natural ivory is obtained. with a special ink. Currency printed with the ink would
change color depending on how ordinary light strikes it,
Which one of the following, if true, most helps to whereas photocopied counterfeits of such currency
strengthen the argument? would not. Because this technique would allow anyone
(A) Most people who play the piano but are not to detect photocopied counterfeit currency easily, it
concert pianists can nonetheless easily should be adopted instead of microprinting, despite the
distinguish between the new synthetic ivory and expense.
inferior ivory substitutes. Which one of the following, if true, most seriously
(B) The new synthetic ivory can be manufactured undermines the argument?
to resemble in color and surface texture any
of the various types of natural ivory that have (A) The longer the interval between the time a
commercial uses. counterfeit bill passes into circulation and
(C) Other natural products such as bone or tortoise the time the counterfeit is detected, the more
shell have not proven to be acceptable difficult it is for law enforcement officials to
substitutes for natural ivory in piano keys. apprehend the counterfeiter.
(D) The most common use for natural ivory is in (B) Sophisticated counterfeiters could produce
ornamental carvings, which are prized not only currency printed with the special ink but cannot
for the quality of their workmanship but also for duplicate microprinted currency exactly.
the authenticity of their materials. (C) Further advances in photocopying technology
(E) It costs significantly less to produce the new will dramatically increase the level of detail that
synthetic ivory then it does to produce any of the photocopies can reproduce.
ivory substitutes that scientists had developed (D) The largest quantities of counterfeit currency now
previously. entering circulation are produced by ordinary
criminals who engage in counterfeiting only
36. The manager of a nuclear power plant defended the briefly.
claim that the plant was safe by revealing its rate of (E) It is very difficult to make accurate estimates
injury for current workers: only 3.2 injuries per 200,000 of what the costs to society would be if large
hours of work, a rate less than half the national average amounts of counterfeit currency circulated
for all industrial plants. The manager claimed that, widely.
therefore, by the standard of how many injuries occur,
the plant was safer than most other plants where the
employees could work.
Which one of the following, if true, most calls into
question the manager’s claim?
(A) Workers at nuclear power plants are required to
receive extra training in safety precautions on
their own time and at their own expense.
(B) Workers at nuclear power plants are required to
report to the manager any cases of accidental
exposure to radiation.
(C) The exposure of the workers to radiation at
nuclear power plants was within levels the
government considers safe.
(D) Workers at nuclear power plants have filed only a
few lawsuits against the management concerning
unsafe working conditions.
(E) Medical problems arising from work at a nuclear
power plant are unusual in that they are not
likely to appear until after an employee has left
employment at the plant.
39. Asthmagon was long considered the most effective of
38. The Board of Trustees of the Federici Art Museum has the drugs known as beta-2 agonists, designed to alleviate
decided to sell some works from its collection in order asthma attacks. However, studies conducted in Rhiago
to raise the funds necessary to refurbish its galleries. between 1981 and 1987 revealed that nearly one out of
Although this may seem like a drastic remedy, the every five of the asthma patients under observation who
curator has long maintained that among the paintings took asthmagon suffered serious side effects after taking
that the late Ms. Federici collected for the museum were the drug. Citing this statistic, some doctors argue that
several unsuccessful immature works by Renoir and asthmagon should be banned as an anti-asthma drug.
Cézanne that should be sold because they are of inferior
quality and so add nothing to the overall quality of the Which one of the following, if true, most weakens the
museum’s collection. Hence, the board’s action will not case for the proposed ban of asthmagon?
detract from the quality of the museum’s collection. (A) In Rhiago, where asthmagon had been the most
Which one of the following, if true, most weakens the widely prescribed of the beta-2 agonists, the
argument? number of asthma deaths increased between
1981 and 1987.
(A) The directors of an art museum can generally (B) Many of the patients under observation to whom
raise funds for refurbishing the building in asthmagon was administered had not previously
which the museum’s collection is housed by taken a beta-2 agonist.
means other than selling part of its collection. (C) Despite the growing concern about the drug,
(B) The quality of an art collection is determined many physicians in Rhiago still prescribe
not just by the quality of its paintings, but by asthmagon to asthma suffers.
what its collection demonstrates about the (D) Among the patients observed, only those who had
development of the artistic talent and ideas of very high cholesterol counts suffered side effects
the artists represented. after taking asthmagon.
(C) The immature works by Renoir and Cézanne that (E) Asthmagon increases the severity of asthma
were purchased by Ms. Federici were at that attacks in some people because the drug can
time thought by some critics to be unimportant cause damage to heart tissues.
juvenile works.
(D) Those people who speculate in art by purchasing 40. Because quitting smoking is very stressful and leads to
artworks merely to sell them at much higher weight gain, it is difficult to do. The key to quitting,
prices welcome inflation in the art market, but however, may be as simple as replacing an unhealthy
curators of art museums regret the inflation in activity with a healthy one. In one study, half of those
the art market. attempting to quit were assigned to a smoking-cessation
(E) The best work of a great artist demands much program alone, and the other half were assigned to the
higher prices in the art market than the worst same program plus fifteen weeks of aerobic exercise. At
work of that same artist. the one-month mark, none in the first group had quit,
but 40 percent of those in the second group had not
smoked.
Each of the following, if true, provides some support for
the argument EXCEPT:
(A) Regular exercise prevents weight gain.
(B) Each group in the study included four hundred
randomly selected participants.
(C) Nonsmokers accustomed to regular exercise do
not gain weight when they stop exercising.
(D) Aerobic exercise can stimulate the brain’s
production of endorphins, which reduce tension.
(E) Of those in the second group in the study, 38
percent had not smoked at the one-year mark.
41. A neighborhood group plans to protest the closing of the 43. Antinuclear activist: The closing of the nuclear power
neighborhood’s only recreation center on the grounds plant is a victory for the antinuclear cause. It
that to do so would leave the neighborhood without also represents a belated acknowledgment by the
local access to a recreation center. “Our neighborhood power industry that they cannot operate such
already has the most residents per center of any plants safely.
neighborhood in the city,” complained one resident, “and
closing this center would make the situation Nuclear power plant manager: It represents no such
unacceptable since access to recreational facilities is a thing. The availability of cheap power from
necessity for this neighborhood.” nonnuclear sources, together with the cost of
mandated safety inspections and safety repairs,
Each of the following, if true, weakens the resident’s made continued operation uneconomic. Thus
argument EXCEPT: it was not safety considerations but economic
(A) A large number of the neighborhood’s residents considerations that dictated the plant’s closing.
are unable to travel outside their locality to gain Which one of the following, if true, most strongly
access to recreational facilities. supports the activist’s claim of victory?
(B) Children, the main users of recreational facilities,
make up a disproportionately small segment of (A) The plant had reached the age at which its
the neighborhood’s population. operating license expired.
(C) Often the recreation center in the neighborhood is (B) The mandate for inspections and repairs
open but not being used. mentioned by the manager was recently enacted
(D) Programs that are routinely filled at other as a result of pressure from antinuclear groups.
recreation centers must be canceled at the (C) The plant would not have closed if cheap power
neighborhood’s recreation center due to lack of from nonnuclear sources had not been available.
interest. (D) Per unit of electricity produced, the plant had the
(E) As people become more involved in computers highest operating costs of any nuclear power
and computer games, recreation centers are plant.
becoming increasingly less important. (E) The plant that closed had been able to provide
backup power to an electrical network when
42. Most small children are flat-footed. This failure of the parts of the network became overloaded.

foot to assume its natural arch, if it persists past early


childhood can sometimes result in discomfort and even
pain later in life. Traditionally, flat-footedness in
children has been treated by having the children wear
special shoes that give extra support to the foot, in order
to foster the development of the arch.
Which one of the following, if true, most calls into
question the efficacy of the traditional treatment
described above?
(A) Many small children who have normal feet wear
the same special shoes as those worn by flat-
footed children.
(B) Studies of flat-footed adults show that flat feet
are subject to fewer stress fractures than are feet
with unusually high arches.
(C) Although most children’s flat-footedness is
corrected by the time the children reach puberty,
some people remain flat-footed for life.
(D) Flat-footed children who do not wear the special
shoes are as likely to develop natural arches as
are flat-footed children who wear the special
shoes.
(E) Some children who are not flat-footed have hip
and lower leg bones that are rotated excessively
either inward or outward.
46. Someone who gets sick from eating a meal will often
44. A number of seriously interested amateur astronomers develop a strong distaste for the one food in the meal
have tested the new Exodus refractor telescope. With that had the most distinctive flavor, whether or not that
it, they were able to observe in crisp detail planetary food caused the sickness. This phenomenon explains
features that were seen only as fuzzy images in their why children are especially likely to develop strong
8-inch (approximately 20-centimeter) Newtonian aversions to some foods.
telescopes, even though the 8-inch telescopes, with
their wider apertures, gather more light than the 4-inch Which one of the following, if true, provides the
(approximately 10-centimeter) Exodus. Given these strongest support for the explanation?
amateur astronomers’ observational findings, any serious (A) Children are more likely than adults to be given
amateur astronomer ought to choose the Exodus if she or meals composed of foods lacking especially
he is buying a telescope for planetary observation. distinctive flavors.
Which one of the following most seriously weakens the (B) Children are less likely than adults to see a
argument? connection between their health and the foods
they eat.
(A) Telescopes of certain types will not perform well (C) Children tend to have more acute taste and to
unless they have been precisely collimated, a become sick more often than adults do.
delicate adjustment requiring deftness. (D) Children typically recover more slowly than
(B) Image quality is only one of several different adults do from sickness caused by food.
factors that, taken together, should determine the (E) Children are more likely than are adults to refuse
choice of a telescope for planetary observation. to eat unfamiliar foods.
(C) Many serious amateur astronomers have no
intention of buying a telescope for planetary 47. Opponents of allowing triple-trailer trucks to use the
observation. national highway system are wrong in claiming that
(D) The comparisons made by the amateur these trucks are more dangerous than other commercial
astronomers were based on observations made vehicles. In the western part of the country, in areas
during several different observation sessions. where triple-trailers are now permitted on some
(E) The substance used to make the lenses of Exodus highways, for these vehicles the rate of road accident
telescopes differs from that used in the lenses of fatalities per mile of travel is lower than the national
other telescopes. rate for other types of commercial vehicles. Clearly,
triple-trailers are safer than other commercial vehicles.
45. A director of the Rexx Pharmaceutical Company argued
that the development costs for new vaccines that the Which one of the following, if true, most substantially
health department has requested should be subsidized weakens the argument?
by the government, since the marketing of vaccines (A) It takes two smaller semitrailers to haul as much
promised to be less profitable than the marketing of any weight as a single triple-trailer can.
other pharmaceutical product. In support of this claim (B) Highways in the sparsely populated West are
the director argued that sales of vaccines are likely to be much less heavily traveled and consequently are
lower since each vaccine is administered to a patient far safer than highways in the national system as
only once, whereas medicines that combat diseases and a whole.
chronic illnesses are administered many times to each (C) Opponents of the triple-trailers also once opposed
patient. the shorter twin-trailers, which are now common
Which one of the following, if true, most weakens the on the nation’s highways.
support offered by the company director for the claim (D) In areas where the triple-trailers are permitted,
concerning the marketing of vaccines? drivers need a special license to operate them.
(E) For triple-trailers the rate of road accident
(A) Vaccines are administered to many more people fatalities per mile of travel was higher last year
than are most other pharmaceutical products. than in the two previous years.
(B) Many of the diseases that vaccines are designed
to prevent can be successfully treated by
medicines.
(C) Pharmaceutical companies occasionally market
products that are neither medicines nor vaccines.
(D) Pharmaceutical companies other than the Rexx
Pharmaceutical Company produce vaccines.
(E) The cost of administering a vaccine is rarely
borne by the pharmaceutical company that
manufactures that vaccine.
48. A newly developed light bulb is much more cost- 50. Pit vipers are so called because of the small pits on the
effective than conventional light bulbs: it costs only sides of their heads which function as infrared sensors,
about 3 times what a conventional light bulb costs but it giving the viper a thermal impression of objects in its
lasts up to 10 times as long as a conventional light bulb. vicinity. Because all species of pit vipers are predators,
Despite the manufacturer’s intense efforts to publicize scientists have long thought that these sensors were used
the advantages of the new bulb, one analyst predicts that to find prey. One researcher has hypothesized, however,
these new bulbs will prove to sell very poorly. that the infrared sensor serves primarily to assist the
viper in judging the size of approaching predators and
Each of the following, if true, provides support for the
how far away they are.
analyst’s prediction EXCEPT:
Which one of the following, if true, would most support
(A) The light generated by the new bulb is in the
the researcher’s hypothesis?
yellow range of the spectrum, a type of artificial
light most people find unappealing. (A) Pit vipers of both sexes have infrared sensors,
(B) Most people who purchase light bulbs prefer to and both sexes exhibit aggressive and defensive
buy inexpensive light bulbs rather than more behaviors.
durable but expensive light bulbs. (B) Pit vipers do not differ in their predatory behavior
(C) A manufacturer of one brand of conventional from closely related vipers without pits, but they
light bulb has advertised claims that the do differ markedly from these other species in
new light bulb uses more electricity than do their strategies of defense against predators.
conventional light bulbs. (C) Pit vipers are distinguishable from other viper
(D) The new bulb is to be marketed in several species not only by their pits but also by the
different quantities, ranging from packages chemical composition of their venom.
containing one bulb to packages containing four (D) Pit vipers have a well-developed sense of
bulbs. smell that they use to supplement the thermal
(E) A competing manufacturer is about to introduce impressions they receive from their infrared
a light bulb that lasts 10 times as long as sensors.
a conventional bulb but costs less than a (E) The rattle of the diamondback rattlesnake, one of
conventional bulb. the most common kinds of pit viper, functions as
a defense mechanism to intimidate predators.
49. For every 50 dogs that contract a certain disease, one
will die from it. A vaccine exists that is virtually 100 51. Public health will improve more quickly in the wake
percent effective in preventing this disease. Since the of new medical discoveries if medical researchers
risk of death from complications of vaccination is one abandon their practice of waiting until their findings are
death per 5,000 vaccinations, it is therefore safer for a published in peer-reviewed journals before informing the
dog to receive the vaccine than not to receive it. press of important research results. This is because the
Which one of the following would it be most helpful to public release of new medical information allows people
know in order to evaluate the argument? to use that information in order to improve their health,
but the peer-review process is unavoidably very slow.
(A) the total number of dogs that die each year from
all causes taken together Which one of the following, if true, most seriously
(B) whether the vaccine is effective against the weakens the argument?
disease in household pets other than dogs (A) Peer review often prevents the publication of
(C) the number of dogs that die each year from false conclusions drawn on the basis of poorly
diseases other than the disease in question conducted medical research.
(D) the likelihood that a dog will contract another (B) People often alter their life-styles on the basis of
disease such as rabies new medical information made available through
(E) the likelihood that an unvaccinated dog will the press.
contract the disease in question (C) Some improvements in public health are due to
factors other than the discovery of new medical
information.
(D) Some newspapers would be willing to publish the
results of medical research before those results
have appeared in peer-reviewed journals.
(E) Most peer-reviewed scientific journals would
refuse to give up the practice of peer review.
Questions 53–54
52. Three major laundry detergent manufacturers have
concentrated their powdered detergents by reducing Although tales of wonder and the fantastic are integral
the proportion of inactive ingredients in the detergent to all world literatures, only recently has the fantasy genre
formulas. The concentrated detergents will be sold had a commercial resurgence in North America. During the
in smaller packages. In explaining the change, the last 20 years, sales of fantasy-fiction books written for adults
manufacturers cited the desire to reduce cardboard have gone from 1 to 10 percent of total adult-fiction sales.
packaging and other production costs. Market analysts At the same time, the number of favorable reviews of fantasy
predict that the decision of these three manufacturers, books has increased markedly. Some booksellers say that
who control 80 percent of the laundry detergent market, the increased sales of fantasy books written for adults can be
will eventually bring about the virtual disappearance of traced to the increased favorable attention given the genre by
old-style bulky detergents. book reviewers.
Which one of the following, if true, most strongly
supports the prediction made by the market analysts? 53. Which one of the following, if true, undermines the
booksellers’ explanation of the growth in sales of
(A) Most smaller manufacturers of laundry detergents fantasy-fiction books for adults?
will consider it too expensive to retool factories
for the production of the smaller detergent (A) Publishers often select a manuscript on the
packages. basis of whether they think that the published
(B) Many consumers will be skeptical initially that book will receive favorable reviews by book
the recommended small amount of concentrated reviewers.
detergent will clean laundry as effectively as the (B) Few readers of fantasy fiction read book reviews,
larger amount of the old-style detergent did. and even fewer select books to purchase on the
(C) Some analysts believe that consumers will have basis of those reviews.
to pay a greater cost per load of laundry to use (C) Most booksellers are aware of what major book
the new concentrated detergent than they did to reviewers have written about recently published
use the old-style detergent. books.
(D) Major supermarkets have announced that they (D) Although the increase in the percentage of fantasy
will not charge the detergent manufacturers books sold has been substantial, publishers
less to display their detergents, even though the estimate that sales could increase even further.
detergents will take up less shelf space. (E) Many of the book reviews of new fantasy-fiction
(E) Consumers are increasingly being persuaded by novels also mention great fantasy novels of
environmental concerns to buy concentrated the past.
detergents when available in order to reduce
cardboard waste. 54. Which one of the following, if true, most strongly
supports the booksellers’ explanation of the growth in
sales of fantasy-fiction books for adults?
(A) Many experts report that on average the reading
level of book buyers has declined over the past
20 years.
(B) Because life during the past 20 years has become
complex and difficult, many readers have come
to prefer the happy endings that fantasy fiction
often provides.
(C) Some fantasy publishers take advantage of the
popularity of certain books by commissioning
similar books.
(D) Because few readers of mystery novels were
buying fantasy fiction, 10 years ago the
major publishers of fantasy fiction created an
advertising campaign directed specifically at
those readers.
(E) After fantasy fiction began to be favorably
reviewed by respected critics 20 years ago, book
buyers began to regard fantasy books as suitable
reading for adults.
55. Advertisement: Most power hedge trimmers on the 57. Which one of the following, if true, would provide the
market do an adequate job of trimming hedges, most support for the conclusion in the passage?
but many power hedge trimmers are dangerous to (A) Not all medical tests entail significant discomfort.
operate and can cause serious injury when used
(B) Sometimes, unnecessary medical tests cause
by untrained operators. Bolter Industries’ hedge
healthy people to become ill.
trimmer has been tested by National Laboratories,
(C) Some patients refuse to accept a physician’s
the most trusted name in safety testing. So, you
assurance that the patient is healthy.
know, if you buy a Bolter’s, you are buying a
(D) The more complete the series of tests performed
power hedge trimmer whose safety is assured.
in a medical checkup, the more likely it is that a
The answer to which one of the following questions rare disease, if present, will be discovered.
would be most useful in evaluating the truth of the (E) Physicians can eliminate the need to order certain
conclusion drawn in the advertisement? tests by carefully questioning patients and
(A) Has National Laboratories performed safety tests rejecting some possibilities on that basis.

on other machines made by Bolter Industries?


(B) How important to the average buyer of a power
hedge trimmer is safety of operation? 58. Several studies have shown that hospitals are not all
(C) What were the results of National Laboratories’ equally successful: patients are much more likely to die
tests of Bolter Industries’ hedge trimmer? in some of them than in others. Since the hospitals in the
(D) Are there safer ways of trimming a hedge than studies had approximately equal per-patient funding,
using a power hedge trimmer? differences in the quality of care provided by hospital
(E) Does any other power hedge trimmer on the staff are probably responsible for the differences in
market do a better job of trimming hedges than mortality rates.
does Bolter Industries’ hedge trimmer?
Which one of the following, if true, casts the most doubt
on the conclusion drawn above?
Questions 56–57
(A) The staff in some of the hospitals studied had
A physician who is too thorough in conducting a medical earned more advanced degrees, on average, than
checkup is likely to subject the patient to the discomfort the staff in the other hospitals.
and expense of unnecessary tests. One who is not thorough (B) Patient populations vary substantially in average
enough is likely to miss some serious problem and therefore severity of illness from hospital to hospital.
give the patient a false sense of security. It is difficult for (C) The average number of years that staff members
physicians to judge exactly how thorough they should be. stay on at a given job varies considerably from
Therefore, it is generally unwise for patients to have medical one hospital to another.
checkups when they do not feel ill. (D) Approximately the same surgical procedures were
performed in each of the hospitals covered in the
56. Which one of the following, if true, would most studies.
seriously weaken the argument in the passage? (E) Mortality rates for hospital patients do not vary
(A) Some serious diseases in their early stages have considerably from one region of the country to
symptoms that physicians can readily detect, another.
although patients are not aware of any problem.
(B) Under the pressure of reduced reimbursements,
physicians have been reducing the average
amount of time they spend on each medical
checkup.
(C) Patients not medically trained are unable to judge
for themselves what degree of thoroughness is
appropriate for physicians in conducting medical
checkups.
(D) Many people are financially unable to afford
regular medical checkups.
(E) Some physicians sometimes exercise exactly the
right degree of thoroughness in performing a
medical checkup.
61. Compared to us, people who lived a century ago had
59. Poor nutrition is at the root of the violent behavior of very few diversions to amuse them. Therefore, they
many young offenders. Researchers observed that in likely read much more than we do today.
a certain institution for young offenders, the violent
inmates among them consistently chose, from the food Which one of the following statements, if true, most
available, those items that were low in nutrients. In a weakens the argument?
subsequent experiment, some of the violent inmates (A) Many of the books published a century ago were
were placed on a diet high in nutrients. There was a of low literary quality.
steady improvement in their behavior over the four (B) On average, people who lived a century ago had
months of the experiment. These results confirm the link considerably less leisure time than we do today.
between poor nutrition and violent behavior. (C) The number of books sold today is larger than it
Which one of the following, if true, most strengthens the was a century ago.
argument? (D) On the average, books today cost slightly less in
relation to other goods than they did a century
(A) Some of the violent inmates who took part in the ago.
experiment had committed a large number of (E) One of the popular diversions of a century ago
violent crimes. was horse racing.
(B) Dietary changes are easier and cheaper to
implement than any other type of reform 62. The vomeronasal organ (VNO) is found inside the noses
program in institutions for young offenders. of various animals. While its structural development and
(C) Many young offenders have reported that they function are clearer in other animals, most humans have
had consumed a low-nutrient food sometime in a VNO that is detectable, though only microscopically.
the days before they committed a violent crime. When researchers have been able to stimulate VNO cells
(D) A further study investigated young offenders in humans, the subjects have reported experiencing
who chose a high-nutrient diet on their own and subtle smell sensations. It seems, then, that the VNO,
found that many of them were nonviolent. though not completely understood, is a functioning
(E) The violent inmates in the institution who were sensory organ in most humans.
not placed on a high-nutrient diet did not show
an improvement in behavior. Which one of the following, if true, most weakens the
argument?
60. With decreased production this year in many (A) It is not known whether the researchers succeeded
rice-growing countries, prices of the grain on world in stimulating only VNO cells in the human
markets have increased. Analysts blame this increase on subjects’ noses.
the fact that only a small percentage of world production (B) Relative to its occurrence in certain other animals,
is sold commercially, with government growers the human VNO appears to be anatomically
controlling most of the rest, distributing it for local rudimentary and underdeveloped.
consumption. With so little rice being traded freely, even (C) Certain chemicals that play a leading role in the
slight changes in production can significantly affect the way the VNO functions in animals in which it is
amount of rice available on world markets. highly developed do not appear to play a role in
Which one of the following, if true, would most call into its functioning in humans.
question the analysts’ explanation of the price increase? (D) Secondary anatomical structures associated with
the VNO in other animals seem to be absent in
(A) Rice-importing countries reduce purchases of rice humans.
when the price increases dramatically. (E) For many animal species, the VNO is thought to
(B) In times of decreased rice production, subtly enhance the sense of smell.
governments store more of the rice they control
and reduce their local distribution of rice.
(C) In times of decreased rice production,
governments export some of the rice originally
intended for local distribution to countries with
free grain markets.
(D) Governments that distribute the rice crop for local
consumption purchase the grain commercially in
the event of production shortfalls.
(E) During reduced rice harvests, rice-importing
countries import other kinds of crops, although
this fails to compensate for decreased rice
imports.
63. Lydia: Red squirrels are known to make holes in the 64. When several of a dermatologist’s patients complained
bark of sugar maple trees and to consume the of a rash on just one side of their faces, the
trees’ sap. Since sugar maple sap is essentially dermatologist suspected that the cause was some kind of
water with a small concentration of sugar, the external contact. In each case it turned out that the rash
squirrels almost certainly are after either water occurred on the side of the face to which the telephone
or sugar. Water is easily available from other was held. The dermatologist concluded that the rash was
sources in places where maple trees grow, so the caused by prolonged contact with telephones.
squirrels would not go to the trouble of chewing
Each of the following, if true, provides additional
holes in trees just to get water. Therefore, they are
support for the dermatologist’s diagnosis EXCEPT:
probably after the sugar.
(A) Many telephones are now manufactured using a
Galina: It must be something other than sugar, because
kind of plastic to which some people are mildly
the concentration of sugar in the maple sap is
allergic.
so low that a squirrel would need to drink an
(B) Contact between other devices and the patients’
enormous amount of sap to get any significant
faces occurred equally on both sides of their
amount of sugar.
faces.
Which one of the following, if true, most undermines (C) Most of the patients had occupations that required
the force of Galina’s attempted rebuttal of Lydia’s them to use their telephones extensively.
argument? (D) Telephones are used by most people in the
industrialized world.
(A) Squirrels are known to like foods that have a high
(E) The complaints occurred after an increase in the
concentration of sugar.
patients’ use of the telephone.
(B) Once a hole in a sugar maple trunk has provided
one red squirrel with sap, other red squirrels will
65. In ancient Mesopotamia, prior to 2900 B.c., wheat was
make additional holes in its trunk.
(C) Trees other than sugar maples, whose sap cultivated in considerable quantities, but after 2900 B.c.
production of that grain began to decline as the
contains a lower concentration of sugar than
production of barley increased sharply. Some historians
does sugar maple sap, are less frequently tapped
who study ancient Mesopotamia contend that the decline
by red squirrels.
in wheat production was due to excessive irrigation, lack
(D) Red squirrels leave the sugar maple sap that
of drainage, and the consequent accumulation of salt
slowly oozes out of the holes in the tree’s
residues in the soil.
trunk until much of the water in the sap has
evaporated. Which one of the following, if true, most helps to
(E) During the season when sap can be obtained from support the historians’ contention concerning the
sugar maple trees, the weather often becomes reasons for the decline in wheat production in ancient
cold enough to prevent sap from oozing out of Mesopotamia?
the trees.
(A) The cultivation of barley requires considerably
less water than does the cultivation of wheat.
(B) Barley has much greater resistance to the
presence of salt in soil than does wheat.
(C) Prior to 2900 B.c., barley was cultivated along
with wheat, but the amount of barley produced
was far less than the amount of wheat produced.
(D) Around 2900 B.c., a series of wheat blights
occurred, destroying much of the wheat crop
year after year.
(E) Literary and archaeological evidence indicates
that in the period following 2900 B.c., barley
became the principal grain in the diet of most of
the inhabitants of Mesopotamia.
68. Statistician: A financial magazine claimed that its
66. It has been claimed that television networks should survey of its subscribers showed that North
provide equal time for the presentation of opposing Americans are more concerned about their
views whenever a television program concerns scientific personal finances than about politics. One
issues—such as those raised by the claims of question was: “Which do you think about more:
environmentalists—about which people disagree. politics or the joy of earning money?” This
However, although an obligation to provide equal time question is clearly biased. Also, the readers of the
does arise in the case of any program concerning social magazine are a self-selecting sample. Thus, there
issues, it does so because social issues almost always is reason to be skeptical about the conclusion
have important political implications and seldom can drawn in the magazine’s survey.
definitely be settled on the basis of available evidence. If
a program concerns scientific issues, that program gives Each of the following, if true, would strengthen the
rise to no such equal time obligation. statistician’s argument EXCEPT:

Which one of the following, if true, most seriously (A) The credibility of the magazine has been called
weakens the argument? into question on a number of occasions.
(B) The conclusions drawn in most magazine surveys
(A) No scientific issues raised by the claims of have eventually been disproved.
environmentalists have important political (C) Other surveys suggest that North Americans are
implications. just as concerned about politics as they are about
(B) There are often more than two opposing views on finances.
an issue that cannot be definitely settled on the (D) There is reason to be skeptical about the results
basis of available evidence. of surveys that are biased and unrepresentative.
(C) Some social issues could be definitely settled on (E) Other surveys suggest that North Americans are
the basis of evidence if the opposing sides would concerned not only with politics and finances,
give all the available evidence a fair hearing. but also with social issues.
(D) Many scientific issues have important political
implications and cannot be definitely settled on 69. One of the advantages of Bacillus thuringiensis (B.t.)
the basis of the available evidence. toxins over chemical insecticides results from their
(E) Some television networks refuse to broadcast specificity for pest insects. The toxins have no known
programs on issues that have important political detrimental effects on mammals or birds. In addition, the
implications and that cannot be definitely settled limited range of activity of the toxins toward insects
by the available evidence. means that often a particular toxin will kill pest species
but not affect insects that prey upon the species. This
67. Journal: In several psychological studies, subjects were advantage makes B.t. toxins preferable to chemical
given statements to read that caused them to form insecticides for use as components of insect pest
new beliefs. Later, the subjects were told that the management programs.
original statements were false. The studies report,
however, that most subjects persevered in their Which one of the following statements, if true, most
newly acquired beliefs, even after being told that weakens the argument?
the original statements were false. This strongly (A) Chemical insecticides cause harm to a greater
suggests that humans continue to hold onto number of insect species than do B.t. toxins.
acquired beliefs even in the absence of any (B) No particular B.t. toxin is effective against all
credible evidence to support them. insects.
Which one of the following, if true, most undermines (C) B.t. toxins do not harm weeds that do damage to
the journal’s argument? farm crops.
(D) Insects build up resistance more readily to B.t.
(A) Regardless of the truth of what the subjects were toxins than to chemical insecticides.
later told, the beliefs based on the original (E) Birds and rodents often do greater damage to
statements were, for the most part, correct. farm crops than do insects.
(B) It is unrealistic to expect people to keep track of
the original basis of their beliefs, and to revise a
belief when its original basis is undercut.
(C) The statements originally given to the subjects
would be highly misleading even if true.
(D) Most of the subjects had acquired confirmation of
their newly acquired beliefs by the time they
were told that the original statements were false.
(E) Most of the subjects were initially skeptical of the
statements originally given to them.
70. The five senses have traditionally been viewed as 72. Expert witness: Ten times, and in controlled
distinct yet complementary. Each sense is thought to circumstances, a single drop of the defendant’s
have its own range of stimuli that are incapable of blood was allowed to fall onto the fabric. And in
stimulating the other senses. However, recent research all ten cases, the stained area was much less than
has discovered that some people taste a banana and the expected 9.5 cm2. In fact, the stained area
claim that they are tasting blue, or see a color and say was always between 4.5 and 4.8 cm2. I conclude
that it has a specific smell. This shows that such people, that a single drop of the defendant’s blood stains
called synesthesiacs, have senses that do not respect the much less than 9.5 cm2 of the fabric.
usual boundaries between the five recognized senses.
Which one of the following, if true, most undermines
Which one of the following statements, if true, most the value of the evidence for the expert witness’s
seriously weakens the argument? conclusion?
(A) Synesthesiacs demonstrate a general, systematic (A) If similar results had been found after 100 test
impairment in their ability to use and understand drops of the defendant’s blood, the evidence
words. would be even stronger.
(B) Recent evidence strongly suggests that there are (B) Expert witnesses have sometimes been known to
other senses besides sight, touch, smell, hearing, fudge their data to accord with the prosecution’s
and taste. case.
(C) The particular ways in which sensory experiences (C) In an eleventh test drop of the defendant’s blood,
overlap in synesthesiacs follow a definite the area stained was also less than 9.5 cm2—this
pattern. time staining 9.3 cm2.
(D) The synesthetic phenomenon has been described (D) Another person’s blood was substituted, and
in the legends of various cultures. in otherwise identical circumstances, stained
(E) Synesthesiacs can be temporarily rid of their between 9.8 and 10.6 cm2 of the fabric.
synesthetic experiences by the use of drugs. (E) Not all expert witnesses are the authorities in
their fields that they claim to be.
71. Standard archaeological techniques make it possible
to determine the age of anything containing vegetable 73. Unlike newspapers in the old days, today’s newspapers
matter, but only if the object is free of minerals and televised news programs are full of stories about
containing carbon. Prehistoric artists painted on murders and assaults in our city. One can only conclude
limestone with pigments composed of vegetable matter, from this change that violent crime is now out of
but it is impossible to collect samples of this prehistoric control, and, to be safe from personal attack, one should
paint without removing limestone, a mineral containing not leave one’s home except for absolute necessities.
carbon, with the paint. Therefore, it is not possible to
Which one of the following, if true, would cast the most
determine the age of prehistoric paintings on limestone
serious doubt on the conclusion?
using standard archaeological techniques.
(A) Newspapers and televised news programs have
Which one of the following, if true, most seriously
more comprehensive coverage of violent crime
weakens the argument?
than newspapers did in the old days.
(A) There exist several different techniques for (B) National data show that violent crime is out of
collecting samples of prehistoric pigments on control everywhere, not just in the author’s city.
limestone. (C) Police records show that people experience more
(B) Laboratory procedures exist that can remove all violent crimes in their own neighborhoods than
the limestone from a sample of prehistoric paint they do outside their neighborhoods.
on limestone. (D) Murder comprised a larger proportion of violent
(C) The age of the limestone itself can be determined crimes in the old days than it does today.
from samples that contain no vegetable-based (E) News magazines play a more important role
paint. today in informing the public about crime than
(D) Prehistoric artists did not use anything other than they did in the old days.
vegetable matter to make their paints.
(E) The proportion of carbon to other elements
in limestone is the same in all samples of
limestone.
76. Astronomer: Mount Shalko is the perfect site for the
74. At some point in any discussion of societal justice, the proposed astronomical observatory. The summit
only possible doctrinal defense seems to be “That is the would accommodate the complex as currently
way we do things here.” Different communities that each
designed, with some room left for expansion.
recognize the dignity and equality of all citizens will, for
There are no large cities near the mountain, so
example, nevertheless settle on somewhat different
neither smog nor artificial light interferes with
provisions for the elderly. So, we can see that general
atmospheric transparency. Critics claim that
principles of justice are never sufficient to determine the
Mount Shalko is a unique ecological site, but
details of social policies fixed within a particular state.
the observatory need not be a threat to endemic
Which one of the following statements, if true, most life-forms. In fact, since it would preclude
strengthens the argument concerning the general recreational use of the mountain, it should
principles of justice? be their salvation. It is estimated that 20,000
(A) Although two socialist states each adhered to the recreational users visit the mountain every year,
same electoral principles, one had a different posing a threat to the wildlife.
type of machine for counting ballots in public Which one of the following, if true, most weakens the
elections than the other did. astronomer’s argument?
(B) Two democratic industrial states, both subscribing
(A) More than a dozen insect and plant species
to capitalistic economic principles, differed
endemic to Mount Shalko are found nowhere
markedly in the respective proportions of land
else on earth.
they devoted to forestry.
(B) A coalition of 14 different groups, as diverse as
(C) Although each adhered to its own principles, a
taxpayer organizations and hunting associations,
democracy and a monarchy each had the same
opposes the building of the new observatory.
distribution of wealth in its population.
(C) Having a complex that covers most of the
(D) Two states founded on and adhering to similar
summit, as well as having the necessary security
principles of justice had different requirements
fences and access road on the mountain, could
that had to be met in order to be eligible for
involve just as much ecological disruption as
government-subsidized day care.
does the current level of recreational use.
(E) Two societies based on different principles of
(D) The building of the observatory would not cause
justice, each adhering to its own principles, had
the small towns near Mount Shalko eventually to
the same unemployment benefits.
develop into a large city, complete with smog,
bright lights, and an influx of recreation seekers.
75. Columnist: Polls can influence voters’ decisions, and
(E) A survey conducted by a team of park rangers
they may distort the outcome of an election since
concluded that two other mountains in the same
their results are much less reliable than the public
general area have more potential for recreational
believes. Furthermore, the publication of polls
use than Mount Shalko.
immediately prior to an election allows no
response from those wishing to dispute the polls’
findings. A ban on publishing polls during the
week prior to an election only minimally impairs
freedom of expression, and thus should be
implemented.
Which one of the following, if true, most seriously
weakens the columnist’s argument?
(A) Few people are influenced by the results of polls
published during the two weeks immediately
prior to an election.
(B) The publication of poll results would not decide
the winner of an uneven election race.
(C) The publication of poll results may remove
some voters’ motivation to vote because of the
certainty that a particular candidate will win.
(D) The publication of poll results in the last weeks
before an election draws attention to candidates’
late gains in popularity.
(E) Countries in which such a ban is in effect do not
generally have better informed citizens than do
countries in which such a ban is not in effect.
77. Many people joke about Friday the thirteenth being an 79. Advertisement for a lactase supplement: Lactase, an
unlucky day, but a study showed that in one year enzyme produced by the body, aids in the digestion of
approximately 17 percent of people scheduled to fly lactose, a natural sugar found in dairy foods. Many
canceled or did not show up for their flights on Friday subjects in an experiment who drank a liter of milk on an
the thirteenth—a rate higher than that on any other empty stomach showed signs of lactose intolerance—
day and date in that year. This shows that a significant difficulty in digesting dairy products because of
percentage of the population would rather disrupt their insufficient lactase. Thus, extrapolating from the number
travel plans than risk flying on a supposedly unlucky of subjects adversely affected, at least 50 million people
day. in North America alone should take lactase supplements.

Which one of the following statements, if true, most Which one of the following statements, if true, most
seriously weakens the argument? seriously weakens the argument?

(A) People who fly tend to be professionals who as (A) Eating solid food when drinking milk can
a group are less superstitious than the general decrease the amount of lactase produced by the
public. body.
(B) Surveys show that less than 5 percent of the (B) Most people who consume dairy products
population report that they believe that Friday consume less lactose at each meal than the
the thirteenth is an unlucky day. amount found in a liter of milk.
(C) Weather conditions at several major airports were (C) The production of lactase by the human body
severe on the Fridays that fell on the thirteenth increases with age.
in the year of the study. (D) Lactose intolerance can interfere with proper
(D) In the year of the study, automobile traffic was nutrition.
no lighter on Friday the thirteenth than on other (E) Some dairy foods, such as cheese, contain a form
Fridays. of lactose more difficult to digest than that found
(E) The absentee rate among airline workers was not in milk.
significantly higher than normal on the Fridays
that fell on the thirteenth in the year of the study. 80. Surviving seventeenth-century Dutch landscapes
attributed to major artists now equal in number those
78. Geologist: A geological fault in the mountain under attributed to minor ones. But since in the seventeenth
which the proposed nuclear waste storage facility century many prolific minor artists made a living
would be buried could, after a few thousand supplying the voracious market for Dutch landscapes,
years, cause the contents to seep out or water to while only a handful of major artists painted in the
seep in. Since nuclear waste remains dangerous genre, many attributions of seventeenth-century Dutch
for up to 25,000 years, such seepage would be landscape paintings to major artists are undoubtedly
disastrous. So, we should not place a nuclear erroneous.
waste storage facility under this mountain until Which one of the following, if true, most strengthens the
scientists investigate whether this mountain has argument?
any geological faults.
(A) Technically gifted seventeenth-century Dutch
Which one of the following arguments, if true, most landscape artists developed recognizable styles
strengthens the geologist’s argumentation? that were difficult to imitate.
(A) In a few thousand years, human civilization may (B) In the workshops of major seventeenth-century
no longer exist. artists, assistants were employed to prepare the
(B) The scientists’ investigation would conclusively paints, brushes, and other materials that the
show whether or not the mountain has any major artists then used.
geological faults. (C) In the eighteenth century, landscapes by minor
(C) The proposed facility was not initially intended to seventeenth-century artists were often simply
be used for the storage of nuclear waste. thrown away or else destroyed through improper
(D) The scientists’ investigation would increase storage.
dramatically the cost of storing nuclear waste (D) Seventeenth-century art dealers paid minor artists
under the mountain. extra money to leave their landscapes unsigned
(E) Nuclear waste could be stored in the proposed so that the dealers could add phony signatures
facility on a temporary basis. and pass such works off as valuable paintings.
(E) More seventeenth-century Dutch landscapes were
painted than have actually survived, and that is
true of those executed by minor artists as well as
of those executed by major artists.
83. A rise in the percentage of all 18-year-olds who were
81. In 1992, a major newspaper circulated throughout North recruited by the armed services of a small republic
America paid its reporters an average salary that was between 1980 and 1986 correlates with a rise in the
much lower than the average salary paid by its principal percentage of young people who dropped out of high
competitors to their reporters. An executive of the school in that republic. Since 18-year-olds in the
newspaper argued that this practice was justified, since republic are generally either high school graduates
any shortfall that might exist in the reporters’ salaries is or high school dropouts, the correlation leads to the
fully compensated by the valuable training they receive conclusion that the republic’s recruitment rates for
through their assignments. 18-year-olds depend substantially on recruitment rates
Which one of the following, if true about the newspaper for high school dropouts.
in 1992, most seriously undermines the justification Which one of the following statements, if true, most
offered by the executive? weakens the argument?
(A) Senior reporters at the newspaper earned as much (A) A larger number of 18-year-old high school
as reporters of similar stature who worked for graduates were recruited for the republic’s armed
the newspaper’s principal competitors. services in 1986 than in 1980.
(B) Most of the newspaper’s reporters had worked (B) Many of the high-technology systems used by
there for more than ten years. the republic’s armed services can be operated
(C) The circulation of the newspaper had recently only by individuals who have completed a high
reached a plateau, after it had increased steadily school education.
throughout the 1980s. (C) Between 1980 and 1986 the percentage of high
(D) The union that represented reporters at the school graduates among 18-year-olds recruited
newspaper was different from the union that in the republic rose sharply.
represented reporters at the newspaper’s (D) Personnel of the republic’s armed services are
competitors. strongly encouraged to finish their high school
(E) The newspaper was widely read throughout education.
continental Europe and Great Britain as well as (E) The proportion of recruits who had completed at
North America. least two years of college education was greater
in 1986 than in 1980.
82. Human beings have cognitive faculties that are superior
to those of other animals, and once humans become
aware of these, they cannot be made happy by anything
that does not involve gratification of these faculties.
Which one of the following statements, if true, most
calls into question the view above?
(A) Certain animals—dolphins and chimpanzees,
for example—appear to be capable of rational
communication.
(B) Many people familiar both with intellectual
stimulation and with physical pleasures enjoy
the latter more.
(C) Someone who never experienced classical music
as a child will usually prefer popular music as
an adult.
(D) Many people who are serious athletes consider
themselves to be happy.
(E) Many people who are serious athletes love
gourmet food.
84. Modern navigation systems, which are found in most of 86. The most reliable way to detect the presence of life on a
today’s commercial aircraft, are made with low-power planet would be by determining whether or not its
circuitry, which is more susceptible to interference than atmosphere contains methane. This is because methane
the vacuum-tube circuitry found in older planes. During completely disappears from a planet’s atmosphere
landing, navigation systems receive radio signals from through various chemical reactions unless it is
the airport to guide the plane to the runway. Recently, constantly replenished by the biological processes of
one plane with low-power circuitry veered off course living beings.
during landing, its dials dimming, when a passenger Which one of the following statements, if true, most
turned on a laptop computer. Clearly, modern aircraft seriously weakens the argument?
navigation systems are being put at risk by the electronic
devices that passengers carry on board, such as cassette (A) There are other ways of detecting the presence of
players and laptop computers. life on a planet.
(B) Not all living beings have the ability to
Which one of the following, if true, LEAST strengthens biologically produce methane.
the argument above? (C) We are incapable at present of analyzing a
(A) After the laptop computer was turned off, planet’s atmosphere for the presence of methane.
the plane regained course and its navigation (D) Some living beings biologically produce only
instruments and dials returned to normal. very small amounts of methane.
(B) When in use all electronic devices emit (E) Earth is the only planet whose atmosphere is
electromagnetic radiation, which is known to known to contain methane.
interfere with circuitry.
(C) No problems with navigational equipment or 87. To allay public concern about chemicals that are leaking
instrument dials have been reported on flights into a river from a chemical company’s long-established
with no passenger-owned electronic devices on dump, a company representative said, “Federal law
board. requires that every new chemical be tested for safety
(D) Significant electromagnetic radiation from before it is put onto the market. This is analogous to the
portable electronic devices can travel up to eight federal law mandating testing of every pharmaceutical
meters, and some passenger seats on modern substance for safety.”
aircraft are located within four meters of the Which one of the following, if true, most seriously
navigation systems. weakens the representative’s implied argument that the
(E) Planes were first equipped with low-power public need not be concerned about the leak?
circuitry at about the same time portable
electronic devices became popular. (A) When pharmaceutical substances are tested for
safety pursuant to federal requirements, a delay
is imposed on the entry of potentially lifesaving
85. In a town containing a tourist attraction, hotel and
restaurant revenues each increased more rapidly over the substances onto the market.
past year than did revenue from the sale of passes to the (B) Leakage from the dump has occurred in
attraction, which are valid for a full year. This led those noticeable amounts only in the last few months.
in charge of the attraction to hypothesize that visitors (C) Before the federal law requiring testing of
were illicitly selling or sharing the passes. nonpharmaceutical chemicals went into effect
recently, there were 40,000 such chemicals being
Each of the following, if true, helps to undermine the manufactured, many of them dangerous.
hypothesis of those in charge of the tourist attraction
(D) The concentration of chemicals leaking into the
EXCEPT:
river is diluted, first by rainwater and then by the
(A) During the past year other tourist attractions have water in the river.
opened up in the area. (E) The water in the river is murky because of
(B) Those possessing passes made more frequent the runoff of silt from a number of nearby
trips to the attraction last year than in previous construction projects.
years.
(C) While the cost of passes is unchanged since last
year, hotel and meal prices have risen.
(D) The local board of tourism reports that the
average length of stay for tourists remained
unchanged over the past year.
(E) Each pass contains a photograph of the holder,
and during the past year these photographs have
usually been checked.
90. It is often said that beauty is subjective. But this
88. On the basis of research with young children, a judgment has to be false. If one tries to glean the
developmental psychologist hypothesized that the skills standard of beauty of earlier cultures from the artistic
involved in copying curves must be developed before the works they considered most beautiful, one cannot but
skills involved in copying angles can be developed. be impressed by its similarity to our own standard. In
Which one of the following, if true, supports the many fundamental ways, what was considered beautiful
developmental psychologist’s hypothesis? in those cultures is still considered beautiful in our own
time.
(A) All of the children who can copy curves can also
copy straight lines. Which one of the following statements, if true, most
(B) All of the children who can copy angles can also weakens the argument?
copy curves. (A) Few contemporary artists have been significantly
(C) The ability to discriminate angles must be exposed to the art of earlier cultures.
developed before angles can be copied. (B) The arts held a much more important place in
(D) Some of the children who cannot copy curves can earlier cultures than they do in our culture.
copy angles. (C) Our own standard of beauty was strongly
(E) Young children have the cognitive processes influenced by our exposure to works that were
involved in copying angles. considered beautiful in earlier cultures.
(D) Much of what passes for important artistic work
89. In a study in which secondary school students were today would not be considered beautiful even by
asked to identify the teachers they liked the best, the contemporary standards.
teachers most often identified possessed a personality
(E) In most cultures art is owned by a small social
that constitutes 20 percent of the general public but only
elite.
5 percent of teachers. Thus, something must discourage
the people who would be the best-liked teachers from
91. Monarch butterflies spend the winter hibernating on
entering the profession.
trees in certain forests. Local environmental groups have
Which of the following, if true, most weakens the organized tours of the forests in an effort to protect the
argument? butterflies’ habitat against woodcutters. Unfortunately,
(A) People with the personality type constitute 5 the tourists trample most of the small shrubs that are
percent of the medical profession. necessary to the survival of any monarch butterflies that
(B) People with the personality type constitute 5 fall off the trees. Therefore, the tour groups themselves
percent of college students pursuing a degree in are endangering the monarch butterfly population.
education. Which one of the following would it be most useful to
(C) Students of teachers with the personality type know in evaluating the argument?
are intensely recruited for noneducational
(A) the amount of forest land suitable for monarch
professions.
butterfly hibernation that is not currently used by
(D) Students with the personality type are more likely
monarch butterflies for hibernation
to be liked by teachers than those with other
(B) the amount of wood cut each year by woodcutters
personality types.
in forests used by monarch butterflies for
(E) Teachers with the personality type are more likely
hibernation
to quit teaching than those with other personality
(C) the amount of plant life trampled by the tourists
types.
that is not necessary to the survival of monarch
butterflies
(D) the proportion of the trees cut down by the
woodcutters each year that are cut in the forests
used by monarch butterflies for hibernation
(E) the proportion of hibernating monarch butterflies
that fall of the trees
92. In defending the Hyperion School of Journalism from 93. Advances in photocopying technology allow criminals
charges that its program is of little or no value to its with no printing expertise to counterfeit paper currency.
students, the dean of the school pointed to its recent One standard anticounterfeiting technique,
success in placing students: 65 percent of its graduates microprinting, prints paper currency with tiny designs
that cannot be photocopied distinctly. Although
went on to internships or jobs in print or broadcast
counterfeits of microprinted currency can be detected
journalism.
easily by experts, such counterfeits often circulate
Which one of the following, if true, most seriously widely before being detected. An alternative, though
undermines the defense offered by the dean? more costly, printing technique would print currency
with a special ink. Currency printed with the ink would
(A) More than half of the school’s students came
change color depending on how ordinary light strikes it,
from jobs in journalism to improve their skills.
whereas photocopied counterfeits of such currency
(B) Some newspaper editors do not regard journalism
would not. Because this technique would allow anyone
schools as a necessary part of the training of a
to detect photocopied counterfeit currency easily, it
journalist.
should be adopted instead of microprinting, despite the
(C) The number of cities with more than one major
expense.
newspaper has declined sharply over the last 25
years. Which one of the following, if true, provides the most
(D) The program offered by the Hyperion School of support for the recommendation made by the argument?
Journalism is similar in quality and content to
(A) When an anticounterfeiting technique depends on
those offered by its peer institutions.
the detection of counterfeits by experts, the cost
(E) The proportion of applicants to the Hyperion of inspection by experts adds significantly to the
School of Journalism that are admitted is lower cost to society of that technique.
than it was ten years ago.
(B) For any anticounterfeiting technique to be
effective, the existence of anticounterfeiting
techniques should be widely broadcast, but
the method by which counterfeits are detected
should be kept secret.
(C) The process of microprinting paper currency
involves fewer steps than does the printing of
paper currency with the special ink.
(D) Before photocopying technology existed,
most counterfeits of paper currency were
accomplished by master engravers.
(E) Many criminals do not have access to the
advanced photocopiers that are needed to
produce counterfeits of microprinted paper
currency that cashiers will accept as real.
95. Some plants have extremely sensitive biological
94. Insects can see ultraviolet light and are known to identify thermometers. For example, the leaves of rhododendrons
important food sources and mating sites by sensing the curl when the temperature of the air around them is
characteristic patterns of ultraviolet light that these things below 0°C (Celsius). Similarly, mature crocus blossoms
reflect. Insects are also attracted to Glomosus open in temperatures above 2°C. So, someone who
spiderwebs, which reflect ultraviolet light. Thus, insects simultaneously observed rhododendrons with uncurled
are probably attracted to these webs because of the leaves, crocuses with mature but unopened blossoms,
specific patterns of ultraviolet light that these webs and a thermometer showing 1°C could determine that the
reflect. thermometer’s reading was accurate to within plus or
Which one of the following, if true, most strongly minus 1°C.
supports the argument? Which one of the following, if true, most seriously
(A) When webs of many different species of spider undermines the reasoning above?
were illuminated with a uniform source of white (A) Neither rhododendrons nor crocuses bloom
light containing an ultraviolet component, many for more than a few weeks each year, and the
of these webs did not reflect the ultraviolet light. blossoms of rhododendrons growing in any area
(B) When the silks of spiders that spin silk only do not appear until at least several weeks after
for lining burrows and covering eggs were crocuses growing in that area have ceased to
illuminated with white light containing an bloom.
ultraviolet component, the silks of these spiders (B) Many people find it unpleasant to be outdoors for
reflected ultraviolet light. long periods when the temperature is at or about
(C) When webs of the comparatively recently evolved 1°C.
common garden, spider were illuminated with (C) The climate and soil conditions that favor the
white light containing an ultraviolet component, growth of rhododendrons are also favorable to
only certain portions of these webs reflected the growth of crocuses.
ultraviolet light. (D) Air temperature surrounding rhododendrons,
(D) When Drosophila fruit flies were placed before which can grow 12 feet tall, is likely to differ
a Glomosus web and a synthetic web of similar from air temperature surrounding crocuses,
pattern that also reflected ultraviolet light and which are normally only a few inches high,
both webs were illuminated with white light by more than 2°C, even if the two plants are
containing an ultraviolet component, many of growing side by side.
the fruit flies flew to the Glomosus web. (E) Certain types of thermometers that are commonly
(E) When Drosophila fruit flies were placed before used to measure outdoor temperatures can be
two Glomosus webs, one illuminated with white extremely accurate in moderate temperature
light containing an ultraviolet component and ranges but much less accurate in warmer or
one illuminated with white light without an colder temperature ranges.
ultraviolet component, the majority flew to the
ultraviolet reflecting web.
96. The authors of a recent article examined warnings of an 98. The local agricultural official gave the fruit growers of
impending wave of extinctions of animal species within the District 10 Farmers’ Cooperative a new pesticide
the next 100 years. These authors say that no evidence that they applied for a period of three years to their pear
exists to support the idea that the rate of extinction of orchards in place of the pesticide they had formerly
animal species is now accelerating. They are wrong, applied. During those three years, the proportion of pears
however. Consider only the data on fishes: 40 species lost to insects was significantly less than it had been
and subspecies of North American fishes have vanished during the previous three-year period. On the basis of
in the twentieth century, 13 between 1900 and 1950, and these results, the official concluded that the new
27 since 1950. pesticide was more effective than the old pesticide, at
The answer to which one of the following questions least in the short term, in limiting the loss of certain fruit
would contribute most to an evaluation of to insects.
the argument? The official’s conclusion is most strongly supported if
which one of the following groups of trees did not show
(A) Were the fish species and subspecies that became
a reduction in losses of fruit to insects?
extinct unrepresentative of animal species in
general with regard to their pattern of extinction? (A) peach trees grown in the district that were treated
(B) How numerous were the populations in 1950 of with the new pesticide instead of the old
the species and subspecies of North American pesticide
fishes that have become extinct since 1950? (B) peach trees grown in the district that were treated
(C) Did any of the species or subspecies of North with the new pesticide in addition to the old
American fishes that became extinct in the pesticide
twentieth century originate in regions outside of (C) pear trees grown in the district that were treated
North America? with the old pesticide instead of the new
(D) What proportion of North American fish species pesticide
and subspecies whose populations were (D) pear trees grown in a neighboring district that
endangered in 1950 are now thriving? were treated with neither the old nor the new
(E) Were any of the species or subspecies of North pesticide
American fishes that became extinct in the (E) pear trees grown in a neighboring district that
twentieth century commercially important? were treated with the new pesticide instead of
the old pesticide
97. The play Mankind must have been written between 1431
and 1471. It cannot have been written before 1431, for in 99. A member of the British Parliament is reputed to have
that year the rose noble, a coin mentioned in the play, said, “The first purpose of good social reform is to
was first circulated. The play cannot have been written increase the sum total of human happiness. So, any
after 1471, since in that year King Henry VI died, reform which makes somebody happy is achieving its
and he is mentioned as a living monarch in the play’s purpose. Since the reform I propose would make my
dedication. constituents happy, it is a good social reform.”
The argument would be most seriously weakened if Which one of the following, if true, most seriously
which one of the following were discovered? weakens the argument attributed to the member of
Parliament?
(A) The Royal Theatre Company includes the play on
a list of those performed in 1480. (A) Different things make different people happy.
(B) Another coin mentioned in the play was first (B) The proposed reform would make a few people
minted in 1422. happy, but would not increase the happiness of
(C) The rose noble was neither minted nor circulated most other people.
after 1468. (C) The proposed reform would affect only the
(D) Although Henry VI was deposed in 1461, he was member of Parliament’s constituents and would
briefly restored to the throne in 1470. make them happy.
(E) In a letter written in early 1428, a merchant told (D) Increasing some people’s happiness might not
of having seen the design for a much-discussed increase the sum total of human happiness if
new coin called the “rose noble.” others are made unhappy.
(E) Good social reforms usually have widespread
support.
101. Eva: A “smart highway” system should be installed, one
100. Brownlea’s post office must be replaced with a larger that would monitor areawide traffic patterns and
one. The present one cannot be expanded. Land near communicate with computers in vehicles or with
the present location in the center of town is more programmable highway signs to give drivers
expensive than land on the outskirts of town. Since the information about traffic congestion and alternate
cost of acquiring a site is a significant part of the total routes. Such a system, we can infer, would result
construction cost, the post office clearly could be built in improved traffic flow in and around cities that
more cheaply on the outskirts of town. would do more than improve drivers’ tempers; it
Which one of the following, if true, most seriously would decrease the considerable loss of money
undermines the argument’s stated conclusion? and productivity that now results from traffic
congestion.
(A) The new post office will have to be built in
accordance with a demanding new citywide Luis: There are already traffic reports on the radio. Why
building code. would a “smart highway” system be any better?
(B) If the new post office is built on the outskirts of If Eva responded to Luis by saying that the current one-
town, it will require a parking lot, but if sited minute radio reports are too short to give a sufficient
near the present post office it will not. description of overall patterns of traffic congestion,
(C) If the new post office is built on the outskirts of which one of the following, if true, would most
town, current city bus routes will have to be strengthen Luis’s challenge?
expanded to provide access.
(D) If the new post office is built on the outskirts of (A) Bad weather, which radio stations report, would
town, residents will make decreased use of post cause traffic to slow down whether or not a
office boxes, with the result that mail carriers “smart highway” system was in operation.
will have to deliver more mail to homes. (B) It would be less costly to have radio stations that
(E) If the new post office is built near the center of give continual, lengthier traffic reports than to
town, disruptions to city traffic would have to install a “smart highway” system.
be minimized by taking such steps as doing (C) Radio reports can take note of congestion once
some construction work in stages at night and on it occurs, but a “smart highway” system could
weekends. anticipate and forestall it in many instances.
(D) The proposed traffic monitoring would not reduce
the privacy of drivers.
(E) Toll collection booths, which constitute traffic
bottlenecks, would largely be replaced in the
“smart highway” system by electronic debiting
of commuters’ accounts while traffic proceeded
at full speed.
102. An ingredient in marijuana known as THC has been 104. Garbage dumps do not harm wildlife. Evidence is
found to inactivate herpes viruses in experiments. In furnished by the Masai-Mara reserve in Kenya, where
previous experiments researchers found that inactivated baboons that use the garbage dumps on the reserve as a
herpes viruses can convert healthy cells into cancer food source mature faster and have more offspring than
cells. It can be concluded that the use of marijuana can do baboons on the reserve that do not scavenge on
cause cancer. garbage.
Which one of the following, if true, most seriously Each of the following statements, if true, casts doubt on
weakens the argument? the argument EXCEPT:
(A) Several teams of scientists performed the various (A) The baboons that feed on the garbage dump are
experiments and all of the teams had similar of a different species from those that do not.
results. (B) The life expectancy of baboons that eat garbage is
(B) The carcinogenic effect of THC could be significantly lower than that of baboons that do
neutralized by the other ingredients found in not eat garbage.
marijuana. (C) The cholesterol level of garbage-eating baboons
(C) When THC kills herpes viruses it weakens the is dangerously higher than that of baboons that
immune system, and it might thus diminish the do not eat garbage.
body’s ability to fight other viruses, including (D) The population of hyenas that live near
viruses linked to cancers. unregulated garbage landfills north of the reserve
(D) If chemists modify the structure of THC, THC has doubled in the last two years.
can be safely incorporated into medications to (E) The rate of birth defects for the baboon
prevent herpes. population on the reserve has doubled since the
(E) To lessen the undesirable side effects of first landfills were opened.
chemotherapy, the use of marijuana has been
recommended for cancer patients who are free
of the herpes virus.

103. It is probably not true that colic in infants is caused


by the inability of those infants to tolerate certain
antibodies found in cow’s milk, since it is often the case
that symptoms of colic are shown by infants that are fed
breast milk exclusively.
Which one of the following, if true, most seriously
weakens the argument?
(A) A study involving 500 sets of twins has found that
if one infant has colic, its twin will probably also
have colic.
(B) Symptoms of colic generally disappear as infants
grow older, whether the infants have been fed
breast milk exclusively or have been fed infant
formula containing cow’s milk.
(C) In a study of 5,000 infants who were fed only
infant formula containing cow’s milk, over 4,000
of the infants never displayed any symptoms of
colic.
(D) When mothers of infants that are fed only breast
milk eliminate cow’s milk and all products made
from cow’s milk from their own diets, any colic
symptoms that their infants have manifested
quickly disappear.
(E) Infants that are fed breast milk develop mature
digestive systems at an earlier age than do those
that are fed infant formulas, and infants with
mature digestive systems are better able to
tolerate certain proteins and antibodies found in
cow’s milk.
106. Babies who can hear and have hearing parents who
105. One hundred people listened to a single musical expose them to speech begin to babble at a certain age
composition, “Study I,” on a certain morning. Later that as a precursor to speaking. In the same way, deaf babies
day, they listened to “Study I” again and to another with deaf parents who communicate with them and with
musical composition, “Study II,” and were asked which each other by signing begin to babble in signs at the
they preferred. A substantial majority preferred “Study same age. That is, they make repetitive hand gestures
I.” These results support the hypotheses that people that constitute, within the language system of signs, the
prefer music they have heard on an earlier occasion to analogue of repeated syllables in speech.
music they are hearing for the first time.
The information above, if accurate, can best be used as
Which one of the following, if true, most strengthens the evidence against which one of the following hypotheses?
argument?
(A) Names of persons or things are the simplest
(A) Half of the members of a large group of people words in a language, since babies use them
who heard only “Study II” in the morning and before using the names of actions or processes.
heard both “Study I” and “Study II” later the (B) The development of language competency
same day preferred “Study I.” in babies depends primarily on the physical
(B) Ninety percent of the members of a large group maturation of the vocal tract, a process that
of people who listened to “Study I” and requires speech-oriented vocal activity.
“Study II” without having heard either of the (C) In the absence of adults who communicate with
compositions on any earlier occasion preferred each other in their presence, babies develop
“Study I.” idiosyncratic languages.
(C) The group of 100 people who listened to “Study (D) In babbling, babies are unaware that the sound or
I” in the morning and to both “Study I” gesture combinations they use can be employed
and “Study II” later the same day included some in a purposive way.
professional music critics. (E) The making of hand gestures by hearing babies
(D) Of 80 people who heard only “Study II” in the who have hearing parents should be interpreted
morning and heard both “Study I” and “Study as a part of their developing language.
II” later the same day, 70 preferred “Study II.”
(E) Many of the 100 people who in the morning
listened only to “Study I” complained afterward
that they had not been able to hear well from
where they had been seated.
107. Antarctic seals dive to great depths and stay submerged 109. Before 1986 physicists believed they could describe the
for hours. They do not rely solely on oxygen held in universe in terms of four universal forces. Experiments
their lungs, but also store extra oxygen in their blood. then suggested, however, a fifth universal force of
Indeed, some researchers hypothesize that for long dives mutual repulsion between particles of matter. This fifth
these seals also store oxygenated blood in their spleens. force would explain the occurrence in the experiments
of a smaller measurement of the gravitational attraction
Each of the following, if true, provides some support for
between bodies than the established theory predicted.
the researchers’ hypothesis EXCEPT:
Which one of the following, if true, most strengthens the
(A) Horses are known to store oxygenated blood in
argument that there is a fifth universal force?
their spleens for use during exertion.
(B) Many species of seal can store oxygen directly in (A) The extremely sophisticated equipment used for
their muscle tissue. the experiments was not available to physicists
(C) The oxygen contained in the seals’ lungs and before the 1970s.
bloodstream alone would be inadequate to (B) No previously established scientific results are
support the seals during their dives. incompatible with the notion of a fifth universal
(D) The spleen is much larger in the Antarctic seal force.
than in aquatic mammals that do not make long (C) Some scientists have suggested that the alleged
dives. fifth universal force is an aspect of gravity rather
(E) The spleens of Antarctic seals contain greater than being fundamental in itself.
concentrations of blood vessels than are (D) The experiments were conducted by physicists
contained in most of their other organs. in remote geological settings in which factors
affecting the force of gravity could not be
108. A 1991 calculation was made to determine what, if any, measured with any degree of precision.
additional health-care costs beyond the ordinary are (E) The fifth universal force was postulated at a time
borne by society at large for people who live a sedentary in which many other exciting and productive
life. The figure reached was a lifetime average of ideas in theoretical physics were developed.
$1,650. Thus, people’s voluntary choice not to
exercise places a significant burden on society.
Which one of the following, if true and not taken into
account by the calculation, most seriously weakens the
argument?
(A) Many people whose employment requires
physical exertion do not choose to engage in
regular physical exercise when they are not at
work.
(B) Exercise is a topic that is often omitted from
discussion between doctor and patient during a
patient’s visit.
(C) Physical conditions that eventually require
medical or nursing-home care often first
predispose a person to adopt a sedentary
lifestyle.
(D) Individuals vary widely in the amount and kind
of exercise they choose, when they do exercise
regularly.
(E) A regular program of moderate exercise tends to
increase circulation, induce a feeling of well-
being and energy, and decrease excess weight.
111. Politician: All nations that place a high tax on income
110. Parent P: Children will need computer skills to deal produce thereby a negative incentive for
with tomorrow’s world. Computers should technological innovation, and all nations in which
be introduced in kindergarten, and computer technological innovation is hampered inevitably
languages should be required in high school. fall behind in the international arms race. Those
nations that, through historical accident or the
Parent Q: That would be pointless. Technology foolishness of their political leadership, wind up
advances so rapidly that the computers used in a strategically disadvantageous position are
by today’s kindergartners and the computer destined to lose their voice in world affairs. So,
languages taught in today’s high schools would if a nation wants to maintain its value system and
become obsolete by the time these children are way of life, it must not allow its highest tax
adults. bracket to exceed 30 percent of income.
Which one of the following, if true, is the strongest Each of the following, if true, weakens the politician’s
logical counter parent P can make to parent Q’s argument EXCEPT:
objection?
(A) The top level of taxation must reach 45 percent
(A) When technology is advancing rapidly, regular before taxation begins to deter inventors and
training is necessary to keep one’s skills at a industrialist from introducing new technologies
level proficient enough to deal with the society and industries.
in which one lives. (B) Making a great deal of money is an insignificant
(B) Throughout history people have adapted to factor in driving technological innovation.
change, and there is no reason to believe that (C) Falling behind in the international arms race
today’s children are not equally capable of does not necessarily lead to a strategically less
adapting to technology as it advances. advantageous position.
(C) In the process of learning to work with any (D) Those nations that lose influence in the world
computer or computer language, children community do not necessarily suffer from a
increase their ability to interact with computer threat to their value system or way of life.
technology. (E) Allowing one’s country to lose its technological
(D) Automotive technology is continually advancing edge, especially as concerns weaponry, would be
too, but that does not result in one’s having to foolish rather than merely a historical accident.
relearn to drive cars as the new advances are
incorporated into new automobiles. 112. A person is more likely to become disabled as that person
(E) Once people have graduated from high school, ages. Among adults in the country of East Wendell,
they have less time to learn about computers and however, the proportion receiving disability benefit
technology than they had during their schooling payments shrinks from 4 percent among 55- to 64-year-
years. olds to 2 percent for those aged 65 to 74 and 1 percent
for those aged 75 and older. The explanation of this
discrepancy is that the proportion of jobs offering such a
disability benefit has greatly increased in recent years.
Which one of the following, if true about East Wendell,
shows that the explanation above is at best incomplete?
(A) The treatment of newly incurred disabilities is more
successful now than in the past in restoring partial
function in the affected area within six months.
(B) Some people receive disability benefit payments
under employers’ insurance plans, and some
receive them from the government.
(C) Medical advances have prolonged the average
lifespan beyond what it was 20 years ago.
(D) For persons receiving disability benefit payments,
those payments on average represent a smaller
share of their pre-disability income now than
was the case 20 years ago.
(E) Under most employers’ plans, disability benefit
payments stop when an employee with a
disability reaches the usual retirement age of 65.
113. Critic: Political utility determines the popularity of a 115. Ringtail opossums are an Australian wildlife species
metaphor. In authoritarian societies, the metaphor that is potentially endangered. A number of ringtail
of society as a human body governed by a head is opossums that had been orphaned and subsequently
pervasive. Therefore, the society-as-body raised in captivity were monitored after being returned
metaphor, with its connection between society’s to the wild. Seventy-five percent of those opossums
proper functioning and governance by a head, were killed by foxes, a species not native to Australia.
promotes greater acceptance of authoritarian Conservationists concluded that the native ringtail
repression than do other metaphors, such as opossum population was endangered not by a scarcity of
likening society to a family. food, as had been previously thought, but by non-native
predator species against which the opossum had not
Which one of the following statements, if true, most
developed natural defenses.
weakens the critic’s argument?
Which one of the following, if true, most strongly
(A) In authoritarian societies, the metaphor of society
supports the conservationists’ argument?
as a family is just as pervasive as the society-as-
body metaphor. (A) There are fewer non-native predator species that
(B) Every society tries to justify the legitimacy of its prey on the ringtail opossum than there are
government through the use of metaphor. native species that prey on the ringtail opossum.
(C) The metaphor of society as a human body is (B) Foxes, which were introduced into Australia over
sometimes used in nonauthoritarian societies. 200 years ago, adapted to the Australian climate
(D) Authoritarian leaders are always searching for less successfully than did some other foreign
new metaphors for society in their effort to species.
maintain their power. (C) The ringtail opossums that were raised in
(E) The metaphor of society as a human body captivity were fed a diet similar to that which
governed by a head is rarely used in liberal ringtail opossums typically eat in the wild.
democracies. (D) Few of the species that compete with the ringtail
opossum for food sources are native to Australia.
114. Thirty years ago, the percentage of their income that (E) Ringtail opossums that grow to adulthood in the
single persons spent on food was twice what it is today. wild defend themselves against foxes no more
Given that incomes have risen over the past thirty years, successfully than do ringtail opossums raised in
we can conclude that incomes have risen at a greater captivity.
rate than the price of food in that period.
116. Very powerful volcanic eruptions send large amounts of
Which one of the following, if assumed, helps most to
ash high into the atmosphere, blocking sunlight and
justify the conclusion drawn above?
causing abnormally cold temperatures for a year or
(A) The amount of food eaten per capita today is more after the eruption. In 44 B.c. there was a powerful
identical to the amount of food eaten per capita eruption of Mount Etna in Sicily. In the following year,
thirty years ago. Chinese historians recorded summer frosts and dimmed
(B) In general, single persons today eat healthier sunlight in China, thousands of miles east of Sicily. If
foods and eat less than their counterparts of these phenomena were caused by volcanic ash in the
thirty years ago. atmosphere, then the ash sent into the atmosphere by
(C) Single persons today, on average, purchase the Mount Etna’s eruption must have spread over great
same kinds of food items in the same quantities distances.
as they did thirty years ago.
In evaluating the support given for the conclusion
(D) The prices of nonfood items single persons
advanced in the last sentence of the passage, it would be
purchase have risen faster than the price of food
most important to determine whether
over the past thirty years.
(E) Unlike single persons, families today spend about (A) modern monitoring equipment can detect the
the same percentage of their income on food as precise path of volcanic ash in the atmosphere
they did thirty years ago. (B) the abnormal weather in China lasted for a full
year or longer
(C) temperatures in Sicily were abnormally cold after
Mount Etna erupted
(D) there were any volcanic eruptions near China
around the time of Mount Etna’s eruption
(E) subsequent eruptions of Mount Etna were as
powerful as the one in 44 B.c.
119. We can learn about the living conditions of a vanished
117. Essayist: Some researchers criticize British culture by examining its language. Thus, it is likely that
governmental security agencies for not releasing the people who spoke Proto-Indo-European, the
enough information about sightings of language from which all Indo-European languages
unidentified flying objects (UFOs) made by these descended, lived in a cold climate, isolated from ocean
military and intelligence agencies. Requests for or sea, because Proto-Indo-European lacks a word for
information by civilian researchers are brushed “sea,” yet contains words for “winter,” “snow,” and
aside. This leads one to suspect that there have “wolf.”
been spacecraft sighted near Earth that are
extraterrestrial in origin. Which one of the following, if true, most seriously
weakens the argument?
Which one of the following, if true, would most
strengthen the essayist’s argument? (A) A word meaning “fish” was used by the people
who spoke Proto-Indo-European.
(A) The British government is generally not (B) Some languages lack words for prominent
forthcoming with secure information. elements of the environments of their speakers.
(B) The British government would withhold (C) There are no known languages today that lack a
information pertaining to UFOs only if it were word for “sea.”
established that they were from other planets. (D) Proto-Indo-European possesses words for “heat.”
(C) The British government would deny the requests (E) The people who spoke Proto-Indo-European were
by civilian researchers to have access to the data nomadic.
only if this government had something to hide.
(D) The British government is less trusting of civilian 120. A recent study concludes that prehistoric birds, unlike
researchers than it is of military researchers. modern birds, were cold-blooded. This challenges a
(E) The British government has always attempted to widely held view that modern birds descended from
deny the existence of UFOs. warm-blooded birds. The conclusion is based on the
existence of growth rings in prehistoric birds’ bodily
118. Newspaper editorial: Many pharmaceutical companies structures, which are thought to be found only in cold-
develop “me too” drugs, drugs designed to blooded animals. Another study, however, disputes this
duplicate, more or less, the effect of another view. It concludes that prehistoric birds had dense blood
company’s product that is already on the market. vessels in their bones, which suggests that they were
Some critics object that such development active creatures and therefore had to be warm-blooded.
constitutes an unnecessary, redundant use of
resources that provides no new benefits to Which one of the following, if true, would most help to
consumers. However, the entry of “me too” drugs resolve the dispute described above in favor of one party
into the market can result in a price reduction to it?
for the drugs they resemble. Therefore, “me too” (A) Some modern warm-blooded species other than
drugs can indeed benefit consumers. birds have been shown to have descended from
Which one of the following, if true, most strengthens the cold-blooded species.
editorial’s argument? (B) Having growth rings is not the only physical trait
of cold-blooded species.
(A) Some “me too” drugs turn out to be more (C) Modern birds did not evolve from prehistoric
effective than the drugs they were designed to species of birds.
imitate. (D) Dense blood vessels are not found in all warm-
(B) If “me too” drugs were prohibited, more money blooded species.
would be available for the development of (E) In some cold-blooded species the gene that is
innovative drugs. responsible for growth rings is also responsible
(C) Pharmaceutical companies often make more for dense blood vessels.
money on a “me too” drug than on an original
drug.
(D) If all pharmaceutical companies developed “me
too” drugs, fewer innovative drugs would be
developed.
(E) Some pharmaceutical companies lose money on
the development of innovative drugs because
of the development by other companies of “me
too” drugs.
121. Paleontologist: It is widely, but falsely, held that life 123. Sociologist: Research shows, contrary to popular
began in the ocean and did not exist on land until opinion, that, all other things being equal, most
half a billion years ago. Traces of carbon 14 have people who have pets are less happy than most
been found throughout certain 1.2-billion-year- people who do not. Therefore, any person who
old rock in North America. Carbon 14 is extracted wants to be as happy as possible would do well to
by plants and microbes from the atmosphere to consider not having a pet.
fuel their chemical reactions, and is then released
when the plants and microbes die. Which one of the following, if true, most seriously
weakens the sociologist’s argument?
Each of the following, if true, provides additional
support for the paleontologist’s conclusion EXCEPT: (A) Some people who have pets are happier than most
people who do not.
(A) According to one dating technique, a few fossils (B) Most people who have no pets occasionally wish
of plants that lived on land are more than half a that they had pets.
billion years old. (C) Most people who have pets are reasonably happy.
(B) The severity of conditions in the primordial (D) Most people who have pets feel happier because
oceans would have made it difficult for life to they have pets.
begin there. (E) All people who have no pets admit to feeling
(C) Research suggests that some portions of the unhappy sometimes.
1.2-billion-year-old rocks were at one time
submerged in water, though portions clearly 124. Letter to the editor: After Baerton’s factory closed, there
never were. was a sharp increase in the number of claims filed
(D) The 1.2-billion-year-old rocks were formed from for job-related injury compensation by the
soil containing very small amounts of carbon factory’s former employees. Hence there is reason
14 that entered the soil directly from the to believe that most of those who filed for
atmosphere. compensation after the factory closed were just
(E) Uranium testing has been used to confirm that the out to gain benefits they did not deserve, and filed
age of the rocks is 1.2 billion years. only to help them weather their job loss.
122. Consumers will be hurt by the new lower ceilings on Each of the following, if true, weakens the argument
halibut catches. Given the law of supply and demand above EXCEPT:
these restrictions are likely to result in an increase in the (A) Workers cannot file for compensation for many
price of the fish. job-related injuries, such as hearing loss from
Which one of the following, if assumed, would do factory noise, until they have left the job.
most to justify the claim that the price of halibut will (B) In the years before the factory closed, the
increase? factory’s managers dismissed several employees
who had filed injury claims.
(A) The demand for halibut will not decrease (C) Most workers who receive an injury on the job
substantially after the new restrictions are file for compensation on the day they suffer the
imposed. injury.
(B) There is a connection between the supply of (D) Workers who incur partial disabilities due
halibut and the demand for it. to injuries on the job often do not file for
(C) The lost production of halibut will not be compensation because they would have to stop
replaced by increased production of other fish. working to receive compensation but cannot
(D) The demand for other fish will be affected by the afford to live on that compensation alone.
new restrictions. (E) Workers who are aware that they will soon be laid
(E) The amount of halibut consumed represents a off from a job often become depressed, making
very small proportion of all fish consumed. them more prone to job-related injuries.
127. Using fossil energy more efficiently is in the interest
125. Electrical engineers have repeatedly demonstrated that of the nation and the global environment, but major
the best solid-state amplifiers are indistinguishable from improvements are unlikely unless proposed government
the best vacuum-tube amplifiers with respect to the standards are implemented to eliminate products or
characteristics commonly measured in evaluating the practices that are among the least efficient in their class.
quality of an amplifier’s musical reproduction.
Therefore, those music lovers who insist that recorded Objection: Decisions on energy use are best left to the
music sounds better when played with the best vacuum- operation of the market.
tube amplifier than when played with the best solid-state Which one of the following, if true, most directly
amplifier must be imagining the difference in quality undermines the objection above?
that they claim to hear.
(A) It would be unrealistic to expect society to make
Which one of the following, if true, most seriously the changes necessary to achieve maximum
weakens the argument? energy efficiency all at once.
(A) Many people cannot tell from listening to it (B) There are products, such as automobiles, that
whether a recording is being played with a consume energy at a sufficient rate that persons
very good solid-state amplifier or a very good who purchase and use them will become
vacuum-tube amplifier. conscious of any unusual energy inefficiency in
(B) The range of variation with respect to the quality comparison with other products in the same
of musical reproduction is greater for vacuum- class.
tube amplifiers than for solid-state amplifiers. (C) Whenever a new mode of generating energy, such
(C) Some of the characteristics that are important as a new fuel, is introduced, a number of support
in determining how music sounds to a listener systems, such as a fuel-distribution system, must
cannot be measured. be created or adapted.
(D) Solid-state amplifiers are more compact, use less (D) When energy prices rise, consumers of energy
power, and generate less heat than vacuum-tube tend to look for new ways to increase energy
amplifiers that produce a comparable volume of efficiency, such as by adding insulation to their
sound. houses.
(E) Some vacuum-tube amplifiers are clearly superior (E) Often the purchaser of a product, such as a
to some sold-state amplifiers with respect to landlord buying an appliance, chooses on the
the characteristics commonly measured in basis of purchase price because the purchaser is
the laboratory to evaluate the quality of an not the person who will pay for energy used by
amplifier’s musical reproduction. the product.

126. Marianne is a professional chess player who hums


audibly while playing her matches, thereby distracting
her opponents. When ordered by chess officials to cease
humming or else be disqualified from professional
chess, Marianne protested the order. She argued that
since she was unaware of her humming, her humming
was involuntary and that therefore she should not be
held responsible for it.
Which one of the following, if true, most undermines
Marianne’s argument against the order?
(A) The officials of chess have little or no authority to
control the behavior of its professional players
outside of matches.
(B) Many of the customs of amateur chess matches
are not observed by professional chess players.
(C) Not all of a person’s involuntary actions are
actions of which that person is unaware.
(D) A person who hums involuntarily can easily learn
to notice it and can thereby come to control it.
(E) Not all of Marianne’s opponents are distracted by
her humming during chess matches.
128. Health association: In certain studies, most volunteers 129. Industrial scientists have hypothesized that much of the
who used to eat meat have permanently excluded nitrous oxide that contributes to air pollution originates
meat from their diets and now eat only vegetables from the burning of organic matter in forest fires. The
and fruits. These volunteers suffered no ill effects results of one extensive study in which smoke samples
and many even prefer the new regimen. So, taken from forest fires were bottled and then analyzed in
people who eat meat can change their diets to a research laboratory supports this hypothesis, since the
meatless ones and not suffer as a result. analysis showed that all of the samples contained high
concentrations of nitrous oxide.
Critic: Participants in these studies were all favorably
disposed to the substitution before they started, Which one of the following, if true, most undermines
and even so, many of them failed to make a the argument?
complete transition to a vegetarian diet.
(A) The production of synthetic products such as
The critic’s response would most seriously call into nylon is responsible for up to 10 percent of the
question which one of the following conclusions, if that nitrous oxide present in the atmosphere.
conclusion were drawn on the basis of the evidence (B) Many of the pollutants that are present in the
given by the health association? atmosphere are produced by the combustion
of fossil fuels, such as petroleum, which are
(A) The diets of most people who eat meat would be
derived from organic matter.
improved if those people ate only vegetables and
(C) Soil bacteria that feed on ammonium compounds
fruits instead.
present in organic ash are thought by some
(B) Among those who make the attempt to exclude
scientists to excrete small amounts of nitrous
meat from their diets, the more favorably
oxide that then enter the atmosphere.
disposed to the meatless regimen a person is
the more likely that person is to succeed in the (D) When in a closed container, the gases in smoke
produced by forest fires quickly react with each
attempt.
other thereby producing high concentrations of
(C) The number of people who have adopted a
nitrous oxide.
strictly vegetarian diet has increased in the last
(E) By using gas-analyzing devices mounted on
few years.
helicopters, researchers can perform on-site
(D) Participants in the study who failed to make the
analyses of smoke instead of sending smoke
transition to a vegetarian diet do not regret the
samples to laboratories for analysis.
attempt.
(E) Most people, if told by their physicians to exclude
meat from their diets and eat only vegetables and
fruits, would succeed in doing so.
132. Waste management companies, which collect waste for
Questions 130–131 disposal in landfills and incineration plants, report that
disposable plastics make up an ever-increasing
The fishing industry cannot currently be relied upon to percentage of the waste they handle. It is clear that
help the government count the seabirds killed by net fishing, attempts to decrease the amount of plastic that people
since an accurate count might result in restriction of net throw away in the garbage are failing.
fishing. The government should therefore institute a program
Which one of the following, if true, most seriously
under which tissue samples from the dead birds are examined
weakens the argument?
to determine the amount of toxins in the fish eaten by the
birds. The industry would then have a reason to turn in the (A) Because plastics create harmful pollutants when
bird carcasses, since the industry needs to know whether the burned, an increasing percentage of the plastics
fish it catches are contaminated with toxins. handled by waste management companies are
being disposed of in landfills.
130. Which one of the following, if true, most strengthens the (B) Although many plastics are recyclable, most of
argument? the plastics disposed of by waste management
(A) The seabirds that are killed by net fishing do companies are not.
not eat all of the species of fish caught by the (C) People are more likely to save and reuse plastic
fishing industry. containers than containers made of heavier
(B) The government has not in the past sought to materials like glass or metal.
determine whether fish were contaminated with (D) An increasing proportion of the paper, glass, and
toxins by examining tissue samples of seabirds. metal cans that waste management companies
(C) The government cannot gain an accurate count used to handle is now being recycled.
of the number of seabirds killed by net fishing (E) While the percentage of products using plastic
unless the fishing industry cooperates. packaging is increasing, the total amount
(D) If the government knew that fish caught by the of plastic being manufactured has remained
fishing industry were contaminated by toxins, unchanged.
the government would restrict net fishing.
(E) If net fishing were restricted by the government,
then the fishing industry would become more
inclined to reveal the number of seabirds killed
by net fishing.

131. Which one of the following, if true, most strongly


indicates that the government program would not by
itself provide an accurate count of the seabirds killed by
net fishing?
(A) The seabirds killed by net fishing might be
contaminated with several different toxins even
if the birds eat only one kind of fish.
(B) The fishing industry could learn whether the fish
it catches are contaminated with toxins if only a
few of the seabirds killed by the nets were
examined.
(C) The government could gain valuable information
about the source of toxins by examining tissue
samples of the seabirds caught in the nets.
(D) The fish caught in a particular net might be
contaminated with the same toxins as those in
the seabirds caught in that net.
(E) The government would be willing to certify
that the fish caught by the industry are not
contaminated with toxins if tests done on the
seabirds showed no contamination.
133. Samples from the floor of a rock shelter in Pennsylvania 134. There are about 75 brands of microwave popcorn on the
were dated by analyzing the carbon they contained. The market; altogether, they account for a little over half of
dates assigned to samples associated with human the money from sales of microwave food products. It
takes three minutes to pop corn in the microwave,
activities formed a consistent series, beginning with the compared to seven minutes to pop corn conventionally.
present and going back in time, a series that was Yet by weight, microwave popcorn typically costs over
correlated with the depth from which the samples came. five times as much as conventional popcorn. Judging by
The oldest and deepest sample was dated at 19,650 the popularity of microwave popcorn, many people are
years before the present, plus or minus 2,400 years. willing to pay a high price for just a little additional
Skeptics, viewing that date as too early and inconsistent convenience.
with the accepted date of human migration into North
Which one of the following statements, if true, would
America, suggested that the samples could have been
call into question the conclusion in the passage?
contaminated by dissolved “old carbon” carried by
percolating groundwater from nearby coal deposits. (A) More than 50 percent of popcorn purchasers buy
conventional popcorn rather than microwave
Which of the following considerations, if true, argues
popcorn.
most strongly against the suggestion of the skeptics?
(B) Most people who prefer microwave popcorn do
(A) No likely mechanism of contamination involving so because it is less fattening than popcorn that
percolating groundwater would have affected the is popped conventionally in oil.
deeper samples from the site without affecting (C) The price of microwave popcorn reflects its
the uppermost sample. packaging more than it reflects the quality of the
(B) Not every application of the carbon-dating popcorn contained in the package.
procedure has led to results that have been (D) The ratio of unpopped kernels to popped kernels
generally acceptable to scientists. is generally the same whether popcorn is popped
(C) There is no evidence that people were using coal in a microwave or conventionally in oil.
for fuel at any time when the deepest layer (E) Because microwave popcorn contains additives
might have been laid down. not contained in conventional popcorn,
(D) No sample in the series, when retested by the microwave popcorn weighs more than an equal
carbon-dating procedure, was assigned an earlier volume of conventional popcorn.
date than that assigned to a sample from a layer
above it.
(E) No North American site besides the one in
Pennsylvania has ever yielded a sample to which
the carbon-dating procedure assigned a date that
was comparably ancient.
137. Police commissioner: Last year our city experienced a
135. There have been no new cases of naturally occurring 15 percent decrease in the rate of violent crime.
polio in North America in recent years. Yet there are At the beginning of that year a new mandatory
approximately 12 new cases of polio each year in North sentencing law was enacted, which requires that
America, all caused by the commonly administered live all violent criminals serve time in prison. Since
oral polio vaccine (OPV). Substituting inactivated polio no other major policy changes were made last
vaccine (IPV) for most childhood polio immunizations year, the drop in the crime rate must have been
would cut the number of cases of vaccination-caused due to the new mandatory sentencing law.
polio about in half. Clearly it is time to switch from
OPV to IPV as the most commonly used polio vaccine Which one of the following, if true, most seriously
for North American children. weakens the police commissioner’s argument?

Which one of the following, if true, most weakens the (A) Studies of many other cities have shown a
argument? correlation between improving economic
conditions and decreased crime rates.
(A) If IPV replaces OPV as the most commonly (B) Prior to the enactment of the mandatory
used polio vaccine, at least a few new cases of sentencing law, judges in the city had for many
naturally occurring polio in North America will years already imposed unusually harsh penalties
result each year. for some crimes.
(B) The vast majority of cases of polio caused by (C) Last year, the city’s overall crime rate decreased
OPV have occurred in children with preexisting by only 5 percent.
but unsuspected immunodeficiency disorders. (D) At the beginning of last year, the police
(C) A child’s risk of contracting polio from OPV department’s definition of “violent crime” was
has been estimated at 1 in 8.7 million, which is broadened to include 2 crimes not previously
significantly less than the risk of being struck by classified as “violent.”
lightning. (E) The city enacted a policy 2 years ago requiring
(D) Although IPV is preferred in some European that 100 new police officers be hired in each of
nations, most countries with comprehensive the 3 subsequent years.
child immunization programs use OPV.
(E) IPV, like most vaccines, carries a slight risk of
inducing seizures in children with neurological
diseases such as epilepsy.

136. Columnist: There are certain pesticides that, even


though they have been banned for use in the
United States for nearly 30 years, are still
manufactured there and exported to other
countries. In addition to jeopardizing the
health of people in these other countries, this
practice greatly increases the health risk to U.S.
consumers, for these pesticides are often used on
agricultural products imported into the United
States.
Which one of the following, if true, most seriously
weakens the columnist’s argument?
(A) Trace amounts of some of the pesticides banned
for use in the United States can be detected in
the soil where they were used 30 years ago.
(B) Most of the pesticides that are manufactured in
the United States and exported are not among
those banned for use in the United States.
(C) The United States is not the only country that
manufactures and exports the pesticides that are
banned for use in the United States.
(D) The banned pesticides pose a greater risk to
people in the countries in which they are used
than to U.S. consumers.
(E) There are many pesticides that are banned for use
in other countries that are not banned for use in
the United States.
138. Columnist: George Orwell’s book 1984 has exercised 140. Archaeologist: A skeleton of a North American
much influence on a great number of this mastodon that became extinct at the peak of the
newspaper’s readers. One thousand readers were Ice Age was recently discovered. It contains a
surveyed and asked to name the one book that human-made projectile dissimilar to any found
had the most influence on their lives. The book in that part of Eurasia closest to North America.
chosen most often was the Bible; 1984 was Thus, since Eurasians did not settle in North
second. America until shortly before the peak of the Ice
The answer to which one of the following questions Age, the first Eurasian settlers in North America
would most help in evaluating the columnist’s probably came from a more distant part of
argument? Eurasia.
(A) How many books had each person surveyed read? Which one of the following, if true, most seriously
(B) How many people chose books other than 1984? weakens the archaeologist’s argument?
(C) How many people read the columnist’s (A) The projectile found in the mastodon does not
newspaper? resemble any that were used in Eurasia before or
(D) How many books by George Orwell other than during the Ice Age.
1984 were chosen? (B) The people who occupied the Eurasian area
(E) How many of those surveyed had actually read closest to North America remained nomadic
the books they chose? throughout the Ice Age.
(C) The skeleton of a bear from the same place and
139. Lobsters and other crustaceans eaten by humans are time as the mastodon skeleton contains a similar
more likely to contract gill diseases when sewage projectile.
contaminates their water. Under a recent proposal, (D) Other North American artifacts from the peak of
millions of gallons of local sewage each day would be the Ice Age are similar to ones from the same
rerouted many kilometers offshore. Although this would time found in more distant parts of Eurasia.
substantially reduce the amount of sewage in the harbor
(E) Climatic conditions in North America just before
where lobsters are caught, the proposal is pointless,
the Ice Age were more conducive to human
because hardly any lobsters live long enough to be
habitation than were those in the part of Eurasia
harmed by those diseases.
closest to North America at that time.
Which one of the following, if true, most seriously
weakens the argument? 141. Some people claim that every human discovery or
invention is an instance of self-expression. But what
(A) Contaminants in the harbor other than sewage are
they seem to ignore is that, trivially, anything we do
equally harmful to lobsters.
is self-expressive. So, until they can give us a more
(B) Lobsters, like other crustaceans, live longer in the
interesting interpretation of their claim, we are not
open ocean than in industrial harbors.
obliged to take their claim seriously.
(C) Lobsters breed as readily in sewage-contaminated
water as in unpolluted water. Which one of the following, if true, provides the most
(D) Gill diseases cannot be detected by examining the support for the reasoning above?
surface of the lobster. (A) All claims that are trivial are uninteresting.
(E) Humans often become ill as a result of eating (B) Most people do not take trivial claims seriously.
lobsters with gill diseases. (C) No claims that are trivial are worthy of serious
consideration.
(D) Every claim is open to both interesting and
uninteresting interpretations.
(E) Every interpretation is either trivial or
uninteresting.
144. Recently discovered prehistoric rock paintings on small
142. Dietary researcher: A recent study reports that islands off the northern coast of Norway have
laboratory animals that were fed reduced-calorie archaeologists puzzled. The predominant theory about
diets lived longer than laboratory animals whose northern cave paintings was that they were largely a
caloric intake was not reduced. In response, some description of the current diets of the painters. This
doctors are advocating reduced-calorie diets, in theory cannot be right, because the painters must have
the belief that North Americans’ life spans can needed to eat the sea animals populating the waters
thereby be extended. However, this conclusion north of Norway if they were to make the long journey
is not supported. Laboratory animals tend to to and from the islands, and there are no paintings that
eat much more than animals in their natural unambiguously depict such creatures.
habitats, which leads to their having a shorter life Each of the following, if true, weakens the argument
expectancy. Restricting their diets merely brings against the predominant theory about northern cave
their caloric intake back to natural, optimal levels paintings EXCEPT:
and reinstates their normal life spans.
(A) Once on these islands, the cave painters hunted
Which one of the following, if true, would most weaken and ate land animals.
the dietary researcher’s argument? (B) Parts of the cave paintings on the islands did not
(A) North Americans, on average, consume a higher survive the centuries.
number of calories than the optimal number of (C) The cave paintings that were discovered on the
calories for a human diet. islands depicted many land animals.
(B) North Americans with high-fat, low-calorie diets (D) Those who did the cave paintings that were
generally have a shorter life expectancy than discovered on the islands had unusually
North Americans with low-fat, low-calorie diets. advanced techniques of preserving meats.
(C) Not all scientific results that have important (E) The cave paintings on the islands were done by
implications for human health are based on the original inhabitants of the islands who ate
studies of laboratory animals. the meat of land animals.
(D) Some North Americans who follow reduced-
calorie diets are long-lived. 145. Amphibian populations are declining in numbers
(E) There is a strong correlation between diet and worldwide. Not coincidentally, the earth’s ozone layer
has been continuously depleted throughout the last 50
longevity in some species of animals.
years. Atmospheric ozone blocks UV-B, a type of
143. Although wood-burning stoves are more efficient than ultraviolet radiation that is continuously produced by the
open fireplaces, they are also more dangerous. The sun, and which can damage genes. Because amphibians
smoke that wood-burning stoves release up the chimney lack hair, hide, or feathers to shield them, they are
is cooler than the smoke from an open flame. Thus particularly vulnerable to UV-B radiation. In addition,
it travels more slowly and deposits more creosote, a their gelatinous eggs lack the protection of leathery or
flammable substance that can clog a chimney—or worse hard shells. Thus, the primary cause of the declining
ignite inside it. amphibian population is the depletion of the ozone layer.
Each of the following, if true, would strengthen the
Which one of the following, if true, most seriously
argument EXCEPT:
weakens the argument?
(A) Of the various types of radiation blocked by
(A) The most efficient wood-burning stoves produce
atmospheric ozone, UV-B is the only type that
less creosote than do many open fireplaces.
can damage genes.
(B) The amount of creosote produced depends not
(B) Amphibian populations are declining far
only on the type of flame but on how often the
more rapidly than are the populations of
stove or fireplace is used.
nonamphibian species whose tissues and eggs
(C) Open fireplaces pose more risk of severe
have more natural protection from UV-B.
accidents inside the home than do wood-burning
(C) Atmospheric ozone has been significantly
stoves.
depleted above all the areas of the world in
(D) Open fireplaces also produce a large amount of
which amphibian populations are declining.
creosote residue.
(D) The natural habitat of amphibians has not become
(E) Homeowners in warm climates rarely use
smaller over the past century.
fireplaces or wood-burning stoves.
(E) Amphibian populations have declined
continuously for the last 50 years.
146. Medical researcher: As expected, records covering the 148. Further evidence of a connection between brain
last four years of ten major hospitals indicate that physiology and psychological states has recently been
babies born prematurely were more likely to have uncovered in the form of a correlation between
low birth weights and to suffer from health electroencephalograph patterns and characteristic
problems than were babies not born prematurely. moods. A study showed that participants who suffered
These records also indicate that mothers who had from clinical depression exhibited less left frontal lobe
received adequate prenatal care were less likely to activity than right, while, conversely, characteristically
have low birth weight babies than were mothers good-natured participants exhibited greater left lobe
who had received inadequate prenatal care. activity. Thus, one’s general disposition is a result of
Adequate prenatal care, therefore, significantly the activity of one’s frontal lobe.
decreases the risk of low-birth-weight babies. Each of the following, if true, weakens the argument
Which one of the following, if true, most weakens the EXCEPT:
medical researcher’s argument? (A) Many drugs prescribed to combat clinical
(A) The hospital records indicate that many babies depression act by causing increased left lobe
that are born with normal birth weights are born activity.
to mothers who had inadequate prenatal care. (B) Excessive sleep, a typical consequence of clinical
(B) Mothers giving birth prematurely are routinely depression, is known to suppress left lobe
classified by hospitals as having received activity.
inadequate prenatal care when the record of that (C) Frontal lobe activity is not subject to variation the
care is not available. way general disposition is.
(C) The hospital records indicate that low birth weight (D) Earlier studies indicated that frontal lobe activity
babies were routinely classified as having been and emotive states are both caused by activity in
born prematurely. the brain’s limbic system.
(D) Some babies not born prematurely, whose (E) Social interaction of the kind not engaged in by
mothers received adequate prenatal care, have most clinically depressed people is known to
low birth weights. stimulate left lobe activity.
(E) Women who receive adequate prenatal care, are
less likely to give birth prematurely than are 149. Book publishers have traditionally published a few
women who do not receive adequate prenatal books that they thought were of intrinsic merit even
care. though these books were unlikely to make a profit.
Nowadays, however, fewer of these books are being
147. The interstitial nucleus, a subregion of the brain’s published. It seems, therefore, that publishers now, more
than ever, are more interested in making money than in
hypothalamus, is typically smaller for male cats than for
female cats. A neurobiologist performed autopsies on publishing books of intrinsic value.
male cats who died from disease X, a disease affecting Which one of the following statements, if true, most
no more than .05 percent of male cats, and found that seriously weakens the argument?
these male cats had interstitial nuclei that were as large
as those generally found in female cats. Thus, the size of (A) Book publishers have always been very interested
the interstitial nucleus determines whether or not male in making money.
cats can contract disease X. (B) There has been a notable decline in the quality of
books written in recent years.
Which of the following statements, if true, most (C) In the past, often books of intrinsic value would
seriously weakens the argument? unexpectedly make a sizable profit.
(A) No female cats have been known to contract (D) There have always been authors unwilling to be
disease X, which is a subtype of disease Y. published unless a profit is guaranteed.
(B) Many male cats who contract disease X also (E) In recent years, profits in the book publishing
contract disease Z, the cause of which is industry have been declining.
unknown.
(C) The interstitial nuclei of female cats who contract
disease X are larger than those of female cats
who do not contract disease X.
(D) Of 1,000 autopsies on male cats who did not
contract disease X, 5 revealed interstitial nuclei
larger than those of the average male cat.
(E) The hypothalamus is known not to be causally
linked to disease Y, and disease X is a subtype of
disease Y.
152. Taken together, some 2,000 stocks recommended on a
150. Over the last 25 years, the average price paid for a popular television show over the course of the past 12
new car has steadily increased in relation to average years by the show’s guests, most of whom are successful
individual income. This increase indicates that consultants for multibillion-dollar stock portfolios,
individuals who buy new cars today spend, on average, a performed less successfully than the market as a whole
larger amount relative to their incomes buying a car than for this 12-year period. So clearly, no one should ever
their counterparts did 25 years ago. follow any recommendations by these so-called experts.
Which one of the following, if true, most weakens the Each of the following, if true, weakens the argument
argument? EXCEPT:
(A) There has been a significant increase over the (A) Taken together, the stocks recommended on the
last 25 years in the proportion of individuals in television show performed better than the market
households with more than one wage earner. as a whole for the past year.
(B) The number of used cars sold annually is the (B) Taken together, the stocks recommended on the
same as it was 25 years ago. television show performed better for the past
(C) Allowing for inflation, average individual income 12-year period than stock portfolios that were
has significantly declined over the last 25 years. actually selected by any other means.
(D) During the last 25 years, annual new-car sales and (C) Performance of the stocks recommended on the
the population have both increased, but new- car television show was measured by stock
sales have increased by a greater percentage. dividends, whereas the performance of the
(E) Sales to individuals make up a smaller proportion market as a whole was measured by change in
of all new-car sales than they did 25 years ago. share value.
(D) Performance of the stocks recommended on the
151. A commercial insect trap consists of a small box television show was measured independently by
containing pesticide mixed with glucose, a sweet a number of analysts, and the results of all the
substance known to attract insect pests. Yet in measurements concurred.
households where this type of trap has been used (E) The stock portfolios for which the guests were
regularly for the past several years, recently installed consultants performed better for the past 12-year
traps are far less effective in eliminating insect pests period than the market as a whole.
than were traps of that type installed several years ago.
Research scientists have hypothesized that traps in 153. In Australia the population that is of driving age has
those households decreased in effectiveness because grown larger over the last five years, but the annual
successive generations of the pests developed a number of traffic fatalities has declined. This leads to
resistance to the pesticide in the traps. the conclusion that, overall, the driving-age population
of Australia consists of more skillful drivers now than
Which one of the following, if true, most seriously
five years ago.
undermines the hypothesis?
Each of the statements below, if true, weakens the
(A) In households where the traps have been used argument EXCEPT:
regularly, the proportion of insect pests that
have a natural aversion to eating glucose has (A) Three years ago, a mandatory seat-belt law went
increased with each successive generation. into effect throughout Australia.
(B) Even when only a few individuals out of an entire (B) Five years ago, Australia began a major road
generation of insects survive the effects of a repair project.
pesticide, the offspring of those individuals are (C) Because of increases in the price of fuel,
usually resistant to that pesticide. Australians on average drive less each year than
(C) After eating glucose mixed with the pesticide, in the preceding year.
insects that live in households that do not use (D) The number of hospital emergency facilities in
the trap tend to die in greater numbers than do Australia has doubled in the last five years.
insects from households where the traps have (E) In response to an increase in traffic fatalities,
been used regularly. Australia instituted a program of mandatory
(D) After the manufacturer of the traps increased the driver education five years ago.
concentration of the pesticide used in the traps,
the traps were no more effective in eliminating
household insect pests than were the original
traps.
(E) The kind of glucose used to bait the traps is one
of several different kinds of glucose that occur
naturally.
154. Anthropological studies indicate that distinct cultures 156. Speaker: Contemporary business firms need to
differ in their moral codes. Thus, as long as there are recognize that avoiding social responsibility leads
distinct cultures, there are no values shared across to the gradual erosion of power. This is Davis and
cultures. Blomstrom’s Iron Law of Responsibility: “In the
Each of the following, if true, would weaken the long run, those who do not use power in a manner
argument EXCEPT: which society considers responsible will tend to
(A) Anthropologists rely on inadequate translation lose it.” The law’s application to human
techniques to investigate the values of institutions certainly stands confirmed by history.
cultures that use languages different from the Though the “long run” may require decades
anthropologists’ languages. or even centuries in some instances, society
(B) As a result of advancing technology and global ultimately acts to reduce power when society
communication, we will someday all share the thinks it is not being used responsibly. Therefore,
same culture and the same values. a business that wishes to retain its power as long
(C) Although specific moral values differ across as it can must act responsibly.
cultures, more general moral principles, such Which one of the following statements, if true, most
as “Friendship is good,” are common to all weakens the speaker’s argument?
cultures.
(D) The anthropologists who have studied various (A) Government institutions are as subject to the Iron
cultures have been biased in favor of finding Law of Responsibility as business institutions.
differences rather than similarities between (B) Public relations programs can cause society to
distinct cultures. consider an institution socially responsible even
(E) What appear to be differences in values when it is not.
between distinct cultures are nothing more (C) The power of some institutions erodes more
than differences in beliefs about how to live in slowly than the power of others, whether they
accordance with shared values. are socially responsible or not.
(D) Since no institution is eternal, every business will
155. Historians of North American architecture who have eventually fail.
studied early nineteenth-century houses with wooden (E) Some businesses that have used power in socially
floors have observed that the boards used on the floors responsible ways have lost it.
of bigger houses were generally much narrower than
those used on the floors of smaller houses. These
historians have argued that, since the people for whom
the bigger houses were built were generally richer than
the people for whom the smaller houses were built,
floors made out of narrow floorboards were probably
once a status symbol, designed to proclaim the owner’s
wealth.
Which one of the following, if true, most helps to
strengthen the historians’ argument?
(A) More original floorboards have survived from big
early nineteenth-century houses than from small
early nineteenth-century houses.
(B) In the early nineteenth century, a piece of narrow
floorboard was not significantly less expensive
than a piece of wide floorboard of the same
length.
(C) In the early nineteenth century, smaller houses
generally had fewer rooms than did bigger
houses.
(D) Some early nineteenth-century houses had wide
floorboards near the walls of each room and
narrower floorboards in the center, where the
floors were usually carpeted.
(E) Many of the biggest early nineteenth-century
houses but very few small houses from that
period had some floors that were made
of materials that were considerably more
expensive.
158. Members of the Amazonian Akabe people commonly
157. Party spokesperson: The opposition party’s proposal to take an early-morning drink of a tea made from the
stimulate economic activity in the province by leaves of a forest plant. Although they greatly enjoy
refunding $600 million in provincial taxes to this drink, at dawn they drink it only in small amounts.
taxpayers, who could be expected to spend the Anthropologists hypothesize that since this tea is
money, envisions an illusory benefit. Since the extraordinarily high in caffeine, the explanation for the
province’s budget is required to be in balance, Akabe’s not drinking more of it at dawn is that high
either new taxes would be needed to make up caffeine intake would destroy the surefootedness that
the shortfall, in which case the purpose of the their daily tasks require.
refund would be defeated, or else workers for
the province would be dismissed. So, either the Which one of the following, if true, most seriously calls
province’s taxpayers or its workers, who are also the anthropologists’ explanation into question?
residents of the province, will have the $600 (A) The drink is full of nutrients otherwise absent
million to spend, but there can be no resulting net from the Akabe diet.
increase in spending to stimulate the province’s (B) The Akabe also drink the tea in the evening, after
economy. their day’s work is done.
The conclusion about whether there would be a resulting (C) The leaves used for the tea contain a soluble
net increase in spending would not follow if the narcotic.
(D) Akabe children are introduced to the tea in only a
(A) taxpayers of the province would spend outside weak form.
the province at least $300 million of any $600 (E) When celebrating, the Akabe drink the tea in
million refunded to them large quantities.
(B) taxpayers of the province would receive any
refund in partial payments during the year rather 159. Raising the tax rate on essential goods—a traditional
than in a lump sum means of increasing government revenues—invariably
(C) province could assess new taxes in a way that turns low- and middle-income taxpayers against the
would avoid angering taxpayers government. Hence government officials have proposed
(D) province could, instead of refunding the money, adding a new tax on purchases of luxury items such as
stimulate its economy by redirecting its spending yachts, private planes, jewels, and furs. The officials
to use the $600 million for construction projects claim that this tax will result in a substantial increase in
creating jobs around the province government revenues while affecting only the wealthy
(E) province could keep its workers and use them individuals and corporations who can afford to purchase
more effectively, with a resulting savings of such items.
$600 million in its out-of-province expenditures The answer to which one of the following questions
would be the most relevant in evaluating the accuracy of
the government officials’ prediction?
(A) Will luxury goods be taxed at a higher rate than
that at which essential goods are currently
taxed?
(B) Will the revenues generated by the proposed tax
be comparable to those that are currently being
generated by taxes on essential goods?
(C) Will sales of the luxury items subject to the
proposed tax occur at current rates once the
proposed tax on luxury items has been passed?
(D) Will the proposed tax on luxury items win
support for the government in the eyes of low-
and middle-income taxpayers?
(E) Will purchases of luxury items by corporations
account for more of the revenue generated by
the proposed tax than will purchases of luxury
items by wealthy individuals?
160. One year ago, a local government initiated an 161. The number of aircraft collisions on the ground is
antismoking advertising campaign in local newspapers, increasing because of the substantial increase in the
which it financed by imposing a tax on cigarettes of 20 number of flights operated by the airlines. Many of the
fatalities that occur in such collisions are caused not by
cents per pack. One year later, the number of people the collision itself, but by an inherent flaw in the cabin
in the locality who smoke cigarettes had declined by 3 design of most aircraft, in which seats, by restricting
percent. Clearly, what was said in the advertisements access to emergency exits, impede escape. Therefore, to
had an effect, although a small one, on the number of reduce the total number of fatalities that result annually
people in the locality who smoke cigarettes. from such collisions, the airlines should be required to
Which one of the following, if true, most helps to remove all seats that restrict access to emergency exits.
strengthen the argument? Which one of the following proposals, if implemented
(A) Residents of the locality have not increased their together with the proposal made in the passage, would
use of other tobacco products such as snuff and improve the prospects for achieving the stated objective
chewing tobacco since the campaign went into of reducing fatalities?
effect. (A) The airlines should be required, when buying
(B) A substantial number of cigarette smokers in the new planes, to buy only planes with unrestricted
locality who did not quit smoking during the access to emergency exits.
campaign now smoke less than they did before it (B) The airlines should not be permitted to increase
began. further the number of flights in order to offset
(C) Admissions to the local hospital for chronic the decrease in the number of seats on each
respiratory ailments were down by 15 percent aircraft.
one year after the campaign began. (C) Airport authorities should be required to
(D) Merchants in the locality responded to the local streamline their passenger check-in procedures
tax by reducing the price at which they sold to accommodate the increased number of
cigarettes by 20 cents per pack. passengers served by the airlines.
(E) Smokers in the locality had incomes that on (D) Airport authorities should be required to refine
average were 25 percent lower than those of security precautions by making them less
nonsmokers. conspicuous without making them less effective.
(E) The airlines should not be allowed to increase the
ticket price for each passenger to offset the
decrease in the number of seats on each aircraft.
Strengthen/ 46. C 94. E 142. A
Weaken 47. B 95. D 143. C
48. D 96. A 144. C
1. B 49. E 97. E 145. A
2. A 50. B 98. C 146. B
3. B 51. A 99. D 147. E
4. B 52. E 100. B 148. A
5. E 53. B 101. B 149. B
6. C 54. E 102. B 150. E
7. E 55. C 103. D 151. A
8. D 56. A 104. D 152. D
9. B 57. B 105. D 153. E
10. A 58. B 106. B 154. B
11. A 59. E 107. B 155. B
12. E 60. C 108. C 156. B
13. B 61. B 109. B 157. E
14. C 62. A 110. C 158. C
15. D 63. D 111. E 159. C
16. E 64. D 112. E 160. D
17. D 65. B 113. A 161. B
18. D 66. D 114. C
19. C 67. D 115. E
20. D 68. E 116. D
21. A 69. D 117. B
22. C 70. A 118. A
23. C 71. B 119. B
24. C 72. C 120. E
25. A 73. A 121. D
26. B 74. D 122. A
27. C 75. A 123. D
28. D 76. C 124. C
29. B 77. C 125. C
30. D 78. B 126. D
31. C 79. B 127. E
32. A 80. D 128. E
33. A 81. B 129. D
34. E 82. B 130. C
35. D 83. C 131. B
36. E 84. E 132. D
37. B 85. D 133. A
38. B 86. B 134. B
39. D 87. C 135. A
40. C 88. B 136. C
41. A 89. E 137. E
42. D 90. C 138. B
43. B 91. E 139. E
44. B 92. A 140. A
45. A 93. A 141. C
Strengthen/Weaken (D) is even more of a weakener than (A). Hearing that some
pollinators make more frequent visits far from their habitats

Questions starts to drive a wedge between pollinator bees and the crops
near their woodland homes.
(E) The scope here is only those pollen-based crops that do
grow near woodlands, as the author asks the question, does
1. (B) that habitat promote their health? Other habitats are thus
Broad conclusions drawn from specific evidence are easily outside the scope.
weakened.
In spite of efforts to preserve the habitats of endangered 3. (B)
species, the species are still becoming extinct; in fact, they Reduce arguments to simple terms.
are becoming extinct faster and faster. This might be enough Beyond all of his hyperbole and verbal bravado, L.E.’s point
evidence to conclude that preservation efforts are inadequate, is simple: the racetrack should be voted down because the
but that they are wasted? Certainly not. This conclusion is far proximity to gambling will damage children’s character. The
too broad. Any evidence that shows the preservation efforts assumption that gambling cannot coexist with upstanding
haven’t been a total waste—as choice (B) does—will weaken children is directly countered by (B). If (B) is true, there still
this argument. might be many reasons to oppose the racetrack, but its effect
(A) would almost strengthen the argument. If scientists are on kids isn’t necessarily one of them.
getting better and better at preserving habitats and the rate of (A) That children will continue to have good character in
extinction is still going up, that phenomenon would seem to different environments doesn’t matter if, as L.E. suggests, they
support the conclusion that preserving habitat is a wasted effort. will not be able to develop such character to begin with due to
(C) ’s fact may be alarming, but it’s irrelevant to the argument. the influence of the racetrack.
We know that the rate of extinction is going up, no matter what (C) L.E.’s central issue is the effect of a gambling environment
the number of extinctions is in any given year. on children’s character, so (C)’s omission of both children and
(D) Economic benefits of tourism? What? How does that relate character makes it doubly removed from the scope.
to the rate of extinction? This choice is way outside the scope
(D) distorts the issue into one of whether children are turned
of the argument.
into adult gamblers, losing sight of the central issue of
(E) Just because certain programs were proposed doesn’t character.
mean that they were implemented, and even if they were,
(E) That other voters were wise or foolish enough to approve
transferring species out of certain habitats isn’t necessarily
other racetracks is irrelevant to whether voting Yes is wise in
the same as protecting their habitats. We can’t tell from (E)
L.E.’s locality.
whether or not any efforts were wasted.
4. (B)
2. (A)
A logically strong (or proper) retort must stay within the topic
Start your thinking about strengthening an argument with the
and scope chosen by the opponent.
assumptions that the author is making.
To counter the legislator’s straightforward recommendation
The two claims in the argument—the conclusion that pollen-
(ban all cancer-causing food additives), the commentator
based crops are advantaged by growing near woodlands,
goes into a big song and dance about levels of additives
and the evidence that bees pay more visits to flowers close
and an (allegedly) more reasonable approach than absolute
to woodlands—don’t seem to have much to do with each
prohibition. The legislator’s retort must take on this idea
other, on their face. What connects them is the assumption
that instead of a ban, we merely need to set a maximum
that something about the frequency of bee visits advantages
acceptable level of cancer-causingagent—and (B) does the
pollen-based flowers. By affirming that assumption, (A)
job: while each individual chemical’s cancer-causing capacity
strengthens the connection between the bees’ visits and the
may be low, the ingestion of multiple chemicals might well
crops’ health.
lead to the disease. That state of affairs wouldn’t occur if the
(B) Since the author is trying to tout woodland growth as absolute ban were in effect, so by countering the commentator
beneficial for crops, the hint that pollinating bees also live on his own ground, the legislator is able to support her own
elsewhere is outside the scope at best, and a weakener at worst. original idea.
(C) Bees are the only pollinators relevant to the argument, so (A) The quarrel concerns food safety in general, so (A)’s
statements about other ones have no effect, whether or not distinction between children and adults is wholly beside the
they live in woodlands. point.
(C) , like (A), draws an unwarranted distinction—this one mediation can never be effective, while (E) implies that it
between naturally occurring cancer causers and those willfully will not be cost-effective. Neither (D) nor (E) addresses the
added. Such a distinction is never brought up. “lateness” of the proposed mediation.
(D) brings in the issue of the effectiveness of the food additive,
but the commentator hasn’t brought that up so the legislator 7. (E)
would be illogical to do so in her retort. The author states that the assessments at Gem World are
(E) , like (D), focuses on the issue of the effect of additives on more trustworthy because they are in writing. Why should
food, something that isn’t already part of the quarrel so cannot a written assessment be more objective? According to
be brought up now. the argument, the problem with most stores is that the
assessment is made by the seller of the diamond. So, the
5. (E) problem is with who makes the assessments, not whether or
not they appear in writing. We can therefore conclude that
For Strengthen EXCEPT questions, predict the wrong choices
the written assessments made at Gem World will only be
(the strengtheners) and cross them off the list.
more trustworthy if they are made by an outside party. (E)
Characterize the choices: the four wrong ones will all ensures exactly that.
strengthen the argument, so the one right answer either
The author is arguing that we can be more sure of a fair
weakens it or leaves it alone. Why would midlevel managers
price at Gem World than at most other jewelry stores. If
refuse to suggest firing staff even when their departments are
anything, (A), which states that other stores also offer written
“obviously overstaffed?”
assessments, would weaken rather than strengthen the
(A) If the midlevel managers get paid more when they have argument that Gem world is superior. (B) doesn’t address the
bigger staffs, they’d be reluctant to fire people. Eliminate. issue of self-interest and bias—the assessors at other stores
(B) If overstaffing lightens the manager’s workload, the who “might say that a diamond is of higher quality than it
manager would have an incentive to keep on extra staff. really is” might also be “people with years of experience in
Eliminate. appraising gems.” (C) only refers to the actual quality of the
(C) No doubt midlevel managers would want to avoid diamonds, and doesn’t give us reason to believe that the
damaging the morale and productivity of their employees—yet assessments of those diamonds will be more trustworthy at
another incentive to keep extra staff. Gem World. (D), which states that diamond prices change
from day to day, actually makes a verbal assessment more
(D) If workload demands rise and fall unpredictably, managers
likely to be accurate than a written one, and so weakens
would have an incentive to retain extra staff for those heavy
the argument that Gem World’s written assessments are
workload periods.
superior.
(E) must therefore be our correct choice. Sure enough, (E)
is outside the scope of the argument. The question is why 8. (D)
managers are unlikely to get rid of people, not how they do it.
The evidence boils down to one statement: “frequent VDT
Characterizing choices will lead to quick, easy points on Test Day.
users suffered from headaches more often than other office
workers did.” Any statement that provides an alternate
6. (C)
explanation for the results will weaken the argument that VDTs
The company president is all in favor of mediation. (He says cause headaches. (D) suggests that the frequent users might
that it could resolve many costly grievances.) His problem with not actually be frequent users—that is, people who often
human resources’ proposal is that the mediation would occur get headaches may be more likely to classify themselves as
too late, and so “would be relatively ineffective.” The correct frequent VDT users. If true, this would weaken the conclusion
answer choice, then, would explain why mediation, usually by offering a different reason for the correlation between those
effective, would be ineffective if it is begun late in the process. who report themselves as frequent VDT users and those who
(C) provides just such an explanation—if “positions tend to often get headaches.
harden as a dispute wears on,” then late mediation is not a
(A) is irrelevant, since it would affect VDT and non- VDT
good idea, and the company president’s criticism is justified.
users, as well as those with and without frequent headaches.
(A) addresses the process of mediation in general, and not the Since the conclusion does not make the distinction between
company president’s criticism of late mediation specifically. migraines and other headaches (B), the researchers needn’t
(B) states that many disagreements are already being solved make that distinction either. It’s not made clear how
without mediation. This hardly supports the company eyestrain (C) or job-related stress (E) correlates to headaches,
president’s claim that the proposed mediation would be too so these answer choices do not weaken the argument.
late. (D) implies that, because of office power structures,
9. (B) in the way of the eclipsing of the newspaper. Those who
The “argument” is really more of an explanation: the dinosaurs have never read newspapers anyway (C) are not part of the
became extinct because a comet stirred up enough dust author’s calculus; she’s betting that it’s the other 70 percent
to cool the planet. To weaken the argument, we often need who have relied on newspapers who will drift away. If the
to spot an alternative explanation, but here, the weakener cost of electronic data will be roughly equivalent to the cost
simply casts doubt of this explanation. We would expect of newspapers, then that can only hasten the demise of the
other animals with similar physiology to also be affected by newspaper medium, so (D) strengthens the logic. (E) is pretty
something as widespread as a climate change, so (B) casts much a strengthener, too, since it renders negligible the cost
serious doubt on the comet hypothesis by stating that these of installation, a cost that might make a newspaper reader
non-dinosaurs didn’t become extinct at the same time. think twice about switching.
The fact that some paleontologists have a different theory
12. (E)
(A) doesn’t mean the comet hypothesis is wrong—the
paleontologists could be wrong. Since the argument never To weaken an argument that the deceased “would have had
relies on skeletal remains (C), it doesn’t matter that these [no] objection to” the executors selling the farm rather than
remains are inconclusive. (D) is similar to the correct answer giving it to the grandson, we simply need counterevidence
(B), but doesn’t maintain that the animals who did not become that casts doubt as to the authors certainty. If the grandson,
extinct were physiologically similar to the dinosaurs. (If they “of whom she was very fond,” had said again and again (E)
were completely different structurally, they might survive that he wanted the farm, at the very least there’s now a 50-50
where the dinosaurs did not). (E) might be tempting, but note chance that grandma—despite her silence on the matter—
that the theory could be correct, even if the exact mechanisms would have wanted him to have it. And 50-50 is enough.
of the comet extinction are not “fully understood.” Since the executor has to clear the debts, (A) is an argument
for the sale of the farm and the grandson is beside the point.
10. (A) The vague “I’ll take care of you” (B) can’t be construed as an
The nutritionist denies the usefulness of juicing by asserting expression of the specific promise “I’ll give you the farm”; for
that there’s no nutritional difference between eating a piece all we know the grandson was taken care of in the bequest,
of produce and drinking it juiced. That’s the key evidence; albeit not with the gift of a farm. The deceased’s awareness of
sentence 3, and the gist of it is: You can eat a whole carrot, the executor’s position (C) would seem to put a greater burden
save $100, and get the same benefit. But what if eating solid on her to have expressed her wishes about the farm pre-
food is problematic? (A) offers a scenario in which people mortem. (D), meanwhile, distorts the issue of paying the debts
would be wise to purchase the juicer in order to get the vs. the cause of the debts; the latter is certainly irrelevant here.
benefits of produce.
13. (B)
As an argument against juicing, (B) is a 180. Affordability (C)
isn’t the issue; necessity is. (D) implies that the nutritionist The recommendation of employee schedule flexibility is
only knows about “early prototypes” and hence isn’t hip to designed to improve the sleep deprivation problem, so the
the current state of juicing, but the whole mess is outside the author must think that the one has something to do with the
scope. (E) draws an irrelevant comparison between vitamin other. Whether you see (B) as an assumption the author is
pills and the nutrients in produce. This is not an argument that making, or as support for an assumption, either way (B) makes
produce needn’t be consumed at all, just that one needn’t buy the conclusion more likely to follow from the evidence.
an expensive juicer to consume it. (A) actually weakens the argument, since it suggests that
fixing the sleep deprivation problem would be better
11. (A) accomplished by dealing with employees’ overwork rather
The author prophesies bad times for newspapers once more than their schedule flexibility. (C) and (E) may well be true,
people can readily obtain faster and more efficient electronic but schedule flexibility wouldn’t affect how long people sleep
data. That prediction can only hold true if one assumes that (C) or how long people work (E), but when they sleep and
there’s nothing about newspapers per se that is likely to work. (D) is a past-tense hypothetical—”what if such and
remain desirable in the wake of this electronic revolution. such hadn’t happened?”—that has no impact on the present
(A) weakens that assumption by presenting a feature of recommendation.
newspapers that electronic data cannot duplicate. If (A) is true
the predicted dire fate of newspapers may still prove to be 14. (C)
accurate, but not for the reason proposed. Peter’s argument is one of underwatering, which he implies
As long as some service is affordable, the existence of make leaves less desirable to insect predators than the
differently-priced electronic data services (B) shouldn’t stand less-tough leaves produced by abundant watering. But what
about the dangers of underwatering? For the argument to (C) Ambidexterity wouldn’t support the imitation hypothesis
work, it would be well to demonstrate that there are no unless a majority of imitating infants and imitated adults were
such dangers, or at least that they are less troublesome ambidextrous, but the 1/3 figure stops this idea in its tracks.
than the dangers caused by insect predators. That’s why (C) (D) The passing stimulus reference to humans seems solely
strengthens the logic—it assures us that we are wise to have designed to tempt us toward (D), since humans’ handedness
drought-stress (underwatering) damage take a back seat to is utterly irrelevant to that of the marmosets.
insect damage.
(A) creates an irrelevant comparison between plant leaves; 17. (D)
it makes the logic of underwatering no better and no worse. (D) directly severs the link between training resources and
(B) suggests that industrialized countries are in big insect Olympic performance in the modern era. As a result, Juan’s
predator trouble because their plants are so well watered, but pessimism that Olympic amateurs can never seriously
that’s not a point in favor of the author’s recommendation, challenge pros because of the latter’s material advantages is
just a heads-up that those big countries better get busy. highly unjustified. If (D) is true, then the pros may still have
Farmers may not be able to control rainfall, as (D) suggests, an edge over the amateurs, but not for the reason that Juan
but they are able to control how much additional water they proposes.
provide, and it’s the latter that falls within the author’s scope.
(A) offers an irrelevant distinction between numbers of
If anything (E) slightly weakens the argument, by presenting
athletes, irrelevant, that is, to the relationship of resources to
a single instance (albeit complicated by a strange, irrelevant
performance.
comparison between bugs) of a predator preference for
underwatered leaves. By conceding that certain amateurs have something of an
Olympic edge, (B) offers an occasional exception to Juan’s
15. (D) logic but not a stinging rebuke to it. In any event, the Olympic
events to which (B) refers might not be ones in which
The question stem actually gives us the conclusion, which
amateurs are pitted against pros, in which case (B) would be
is the hypothesis that Alzheimer’s disease is caused by a
irrelevant to the logic altogether.
virus. The evidence is that when rats were injected with the
blood of Alzheimer’s patients, they developed C-J disease, (C)’s domain is the ancient games only.
and C-J is caused by a virus. The assumption here, which (E) goes beyond the scope of the Olympics, and if anything
bridges the necessary gap in the argument, is that these two serves to strengthen Juan’s argument by stripping away from
diseases are related. (D) clearly shores up this assumption many amateurs even the poorest pittance of material support.
with its statement that Alzheimer’s and C-J are “different
manifestations of the same disease.” 18. (D)
Notice that none of the other choices mentions both As noted above, the conclusion is a broad one:
Alzheimer’s and C-J, which is exactly the connection that the environmentally speaking, the use of paper causes more harm
argument needs. (A) would weaken the hypothesis, and so than does the use of plastic. But all of the author’s evidence
potentially would (C), both of which talk about the differences, concerns the environmental impact of paper trash and plastic
rather than the similarities, between rats and humans. (B) and trash. What about paper vs. plastic production, which clearly
(E) focus only on C-J disease and deal with factors irrelevant to falls under the scope of general use? The only way that the
the conclusion about Alzheimer’s. argument can work is if the production issue is taken off the
table, and that’s what (D) does, by indicting the production of
16. (E) paper as causing more damage than the production of plastic.
The hypothesis that the marmosets pick up their handedness (A) is an interesting fact that seems to be leading towards a
by imitation hinges on the assumption of a causal link heavier (pardon the pun) indictment of paper, but by failing to
between the infants’ handedness and the handedness of the mention plastic and plastic’s potential (if any) for increasing in
adults allegedly being imitated. By suggesting that marmosets volume, (A)’s effect on the argument is negligible.
raised with adults who are in the minority share those adults’ (B) raises two hugely irrelevant issues: predicted impact, and
minority handedness, (E) renders the cause-and-effect much popular opinion of importance.
more likely.
Each of the remaining wrong choices raises an irrelevant
(A) is wholly consistent with the stimulus’s second sentence distinction: (C)’s is between paper + plastic and all other forms
but doesn’t affect the cause-and-effect one way or the other. of trash, and (E)’s is between different parts of the country.
(B) ’s sibling distinction raises a sensible question (why don’t Each sidesteps the essential issue—that of the environmental
these siblings share the same handedness?) that tends to cast impact of total paper vs. total plastic.
doubt on the hypothesis instead of strengthening it.
19. (C) (B) The extant works praise the missing works, so the missing
The author believes that more funding for intervention works must be the most original of their time? Not necessarily.
research is needed to see whether those with mental disorders The missing works may be great works without being
can be cost-effectively helped. The key is that phrase “cost- superlative.
effectively,” because the cost issue is raised only in the (C) The number of works lost is irrelevant to their originality.
conclusion and not in the evidence. All we get as evidence is (D) Subversive ideas may be original, but need not be. This
the statement that early identification of risk factors through choice is outside the scope.
intervention can “mitigate” those factors. The argument,
(E) Current ideas of what is original are also outside the scope
therefore, lacks support that such mitigation will be more
of the argument, which deals with works that were original in
cost-effective, which is what (C) provides—evidence that the
ancient times.
author’s recommended course of action may end up cheaper
than the alternative.
22. (C)
Of the wrong choices only (A) and (D) mention money at all.
This is a strengthen EXCEPT question which means that the
But (A) goes outside the scope to compare costs of minor
correct answer will either weaken the argument or have no
mental disorders to those of other problems, while (D) only
effect on it. The author’s conclusion is the recommendation
relates to the funding of research (and in fact tends to weaken
made in the first sentence: there should be greater use of
the argument). The feasibility of prevention research (B) and
gasohol. Why? Because it has a higher-octanerating and fewer
the possible recurrence of mental problems (E) are far removed
carbon monoxide emissions than gasoline, and the carbon
from the issue at hand.
dioxide it adds to the air is removed by photosynthesis.
Pretty straightforward, so on to the answer choices. All of the
20. (D)
incorrect answer choices will strengthen the conclusion that
The author’s alleged cause-and-effect (galanin causes a gasohol should be used instead of gasoline. (C) is correct
craving for fats in rats) is based on a correlation: The rats who because it provides an advantage that gasoline has over
opted for fat had more galanin than those who opted for lean. gasohol, namely that cars burn more gasohol per kilometer
But who’s to say that the fatty foods ingested by the former than gasoline. In other words, gasoline cars get better mileage.
rats didn’t account for the greater amounts of galanin? The This would weaken the recommendation that gasohol be used.
experiment would better support causation if (D) were true
(A), (B), (D), and (E) are all incorrect because they provide
and the differing galanin levels were a preexisting condition.
additional advantages of using gasohol over using gasoline.
In other words, by suggesting that the author’s alleged cause-
and-effect wasn’t in fact working in reverse, (D) strengthens it
23. (C)
as initially stated.
This is a difficult question stem. What’s really going on
Choice (A) restricts itself, oddly, to the fat vs. lean decision
here is that you’re going to be questioning the logic of the
and has no impact on the galanin data. It’s amount of galanin,
enforcement program. The stimulus tells you that car emission
not amount of fat (B) in the brain, that is significant in this
standards are enforced through annual inspection. At
argument. And the issue is what is happening within the
inspection, cars are tested while idling, and the test measures
brains, not within the foods (C), so (C) cannot have any effect
the levels of pollutants leaving the tail pipe. You have to
on the logic either. (E) drags in a totally irrelevant issue—rate
determine which answer choice, if true, would show why that
of metabolism—and one that is several steps removed from
particular testing program would not be effective in controlling
the key issue of cravings.
overall pollutant levels. (C) does the job by suggesting that
reducing “idling” pollution emissions will increase pollution
21. (A)
emissions while driving.
To strengthen an argument, find evidence in favor of a key
(A) The question asks us why the current method might not be
assumption.
effective in controlling pollutant levels. Discussions of the cost
This author appears to be a frustrated classicist, griping about of these measures are outside the scope.
the works we’ve lost rather than celebrating the works that
(B) This would have no impact on controlling pollutant levels,
have survived. He complains that the surviving works are
so long as the machines were in fact recalibrated. If the
not as original as the works that were lost, based solely on
answer choice told us it was impossible to calibrate the testing
the evidence that the surviving works refer to the lost works.
devices, that would certainly be a problem, but just the fact
This makes a fairly large assumption: the mere fact that the
that they have to be recalibrated frequently is not enough to
surviving works refer to other works must mean that they are
doubt the effectiveness of the program.
not the most original works of their time. (A) provides us with a
piece of evidence for that assumption.

342 MAS TERY PRACTIC E


(D) , if anything, would point toward the effectiveness of the (D) is an irrelevant comparison. We’re concerned with
program. If most car owners take the necessary measures to observational research’s value for the purchaser of the
get their cars in compliance, then it would be reasonable to research, not the conductors of the research.
expect that overall pollutant levels would be reduced. (E) is another irrelevant comparison. The issue is observational
(E) This too would probably strengthen the logic of the testing research vs. surveys, so comparisons within the survey field
program. By not making special allowances or exceptions for don’t matter.
older cars, which are presumably bigger pollutants, the plan
should help reduce pollutant levels. 26. (B)
Now we’re looking for a strengthener for Jane’s argument.
24. (C) Jane thinks that we ought to ban violence from television
The helpful Keyword “because” gets us to the heart of this programs aimed at minors, because these depictions of
argument. The author believes that Springhill discourages violence might cause real violence later. Lots of things might
water conservation because of the way it charges for water. strengthen the causal link between kids watching violent acts
How so? Springhill charges very little up to a certain amount, and kids committing violent acts, so coming up with a precise
and only then starts to charge a lot of money for water use. prephrase would be difficult. But (B) should jump off the page.
The real question, left unanswered by the author, is how high If kids are influenced most by those movies that are aimed
is this usage threshold? If it’s very low, then people start to specifically at them, then it stands to reason that preventing
pay a lot for even a little water. In this case, Springhill would them from seeing violent acts would make them less likely to
be encouraging water conservation by charging people real commit violent acts.
dollars for every extra drop. However, if (C) is true, and most (A) shifts the scope from the depiction of teenage violence
people don’t need to use enough water to get to the usage to the depiction of adult violence, which, if anything, might
threshold, then Springhill’s water system does discourage
weaken Jane’s argument by suggesting that the portrayal of
conservation. If you pay the same for your water no matter
teenage violence isn’t so bad.
how much you use, then you don’t have much of a financial
(C) is also outside the scope, dealing with issues of profit
incentive to conserve water. So (C) strengthens the argument.
rather than preventing violent acts.
(A) is irrelevant, concerning only what happens when a water
(D) could only weaken Jane’s argument by pointing to factors
emergency is declared. This argument addresses the cause of
influencing teen violence other than movies.
the water emergency.
(E) is irrelevant. So, what if some producers voluntarily
(B) and (E) are outside the scope. The issue is the effect of
tone down their productions? We don’t know why they’re
Springhill’s system of charging for water, not whether it is
doing it. Perhaps they are interested in the moral
willing to change it (B) or the legal process involved in doing
development of today’s youth, but perhaps they think
so (E).
there’s more money in “family-friendly” entertainment. (E)
(D) is an irrelevant comparison. Neighboring towns are outside is too vague to be a strengthener.
the scope. The issue is Springhill, not them. We have no
idea what the water deal is in those towns or how that could 27. (C)
possibly relate to the situation here.
In order to strengthen the argument “X is better than Y”, the
correct answer must introduce new evidence that confirms,
25. (A)
“X is indeed better than Y”. Any answer choice that makes no
The question stem says that we need to find the support for distinction between X and Y is necessarily wrong.
the market researcher’s claim. So, what is that claim? It’s in
Don’t put all your eggs in one basket, reasons this author.
the last sentence. They say that observational research
Large space programs have not done well; “consequently,”
provides information that surveys can’t. So, the correct
answer will show how we can learn something by watching large projects should be broken up into smaller projects. (C)
someone shop that we can’t learn by asking them about the rescues the author’s rather sloppy reasoning and makes the
conclusion inescapable. If large projects are inherently more
experience later.
likely to fail than small projects, then clearly, we should scrap
That’s (A).
these large projects in favor of the smaller projects.
(B) is outside the scope. The issue is whether observational
(A) makes no distinction between large and small projects,
research provides any unique benefits, and not what it takes
and therefore doesn’t tip the balance in one direction or the
to get consumers to participate in such research.
other.
(C) tells us nothing about research methods, focusing instead
on the behavior that this research is intended to measure.
(B) , in introducing criteria for which large and small projects (E) Marketing strategies of brand-name drugs say nothing
are equivalent, does nothing to suggest small projects are a about the effectiveness of generic drugs.
surer bet.
(D) may be tempting, but is irrelevant. Just because project 30. (D)
managers like to work on small projects gives no indication of Here’s an issue many of us can relate to: children were once
whether they are more likely to succeed. taught that they had to eat from the “four major food groups,”
(E) is a mixed bag. Large space projects can do Z better, but but it turns out that this was bad advice. We need to find the
smaller projects can do W better. Who knows which is more four choices that explain why this is bad advice, or the one
important? choice that doesn’t. Not much to do but hit the choices:
(A) There’s one good reason right there. We should consume
28. (D) less of some of the groups, but the division gave us the
When asked to weaken a causal argument, look for the impression that we ought to consume an equal amount of
answer choice that will provide an alternative explanation for each. And that’s a bad thing, since it leads us to consume
the given data. more milk and meat than is healthy.
According to the study, hyperactive adults have less activity (B) Yup, leaving out other important foods certainly counts as
in the premotor cortex than normal adults (correlation). So, a drawback.
reasons the study, diminished activity in this particular part (C) points out that the “four food groups” system was
of the brain must therefore be responsible for hyperactivity oversimplified, leading people to believe that just one fruit
(causation). Well, you don’t need to be a neurologist to realize or vegetable is OK, when many are needed. And no, ketchup
that there are other ways to explain the correlation. Given (D), doesn’t count. Neither do potato chips or Kool-Aid. Sorry.
the reduced activity in the premotor cortex is simply a side (D) Here’s our winner. If the “four food groups” message
effect of one treatment of hyperactivity, and therefore can’t be reminded us not to neglect important sources of nutrients,
the cause of the condition. then maybe it wasn’t so bad after all.
(A) Don’t be fooled by (A)’s supposed nature versus nurture (E) Giving people the false impression that they are eating well
rebuttal. Just because non-hyperactive adults can have when in fact they may have terrible diets certainly qualifies as
hyperactive children says nothing about the physiological providing bad advice.
cause of the hyperactivity.
(B) introduces a new distinction: the severity of hyperactivity. 31. (C)
This distinction is irrelevant to the study; people are either What we’re asked to support is the author’s explanation
hyperactive or they are not. of why 1980s Japanese art collectors bought a lot of 19th
(C) Just because the designers of the experiment weren’t present century Impressionist works. To the author, the reason lay in
doesn’t imply that the experiment was carried out incorrectly. the buyers’ preference for “certain aesthetic attributes they
(E), in explaining why only adults were used, says nothing found in [those] paintings.” But she provides no evidence for
about the interpretation of the data. that, and many other factors other than aesthetic appreciation
could explain those purchases in the ‘80s: Maybe the
29. (B) paintings were inexpensively priced; or maybe they were worth
an awful lot at the time and hence became more desirable; or
Any choice that silences a potential objection to an argument
maybe a whole bunch of them came on the market at once.
strengthens that argument.
So, the right answer must either link the taste of Japanese
Generic drugs: same active ingredients, less money. Sign me collectors to the traits of Impressionist art, or must dismiss
up! The author’s conclusion is nicely signposted: “so” generic those other reasons. Correct answer (C) does the former: The
drugs are equally effective with a lower cost. (B) preemptively “high esteem” that the Japanese feel for certain techniques
rebuts a possible problem with generic drugs. Because generic in their own country’s art would tend to explain the desire to
drugs are no more likely to suffer from defects than the name acquire European Impressionist art that picks up on those
brands, we’re even more convinced their efficacy. techniques. (C) strengthens the connection between the
(A) The cost of ingredients to the drug companies is irrelevant, buying phenomenon described and the reason cited by the
as the conclusion focuses on the impact to the consumer. author.
(C) The availability of generic drugs to pharmacies says (A) The author’s interest is in Japanese collectors, not
nothing about their effectiveness. European ones. Besides, the origin of Impressionist appeal
(D) A justification for the higher-priced brand names does has nothing necessarily to do with its appeal in the 1980s.
nothing to weaken the case for generic drugs.
(B) might explain where ‘80s Japanese collectors obtained 33. (A)
the wherewithal to purchase a lot of art. But why 19th century All we need to remember from the stimulus is that the new
Impressionism? (B) leaves that unexplained. system supposedly makes airplanes safer—even though it
(D) offers an irrelevant comparison between the appeal of two frequently causes false alarms—simply because it warns
types of art—and pulls it out of the wrong decades, to boot. pilots to take evasive action. Presumably, evasive action
(E)’s reference to collectors “around the world” has nothing to helps pilots avoid mid-air collisions, which no doubt is a
do with the issues raised in the stimulus, nor do paintings by good thing. But are there any possible drawbacks to evasive
Japanese artists. action—especially when it’s often triggered by false alarms?
Here that image of passengers flying all over the plane
32. (A) mentioned above comes into play: What if the evasive action
is itself dangerous to passengers? Surely no one would mind a
We need to identify a possible flaw in the author’s logic—
little bump on the head to avoid a crash, but is it worth being
specifically, a statement which, if true, would indicate that
thrown from your seat every time the blasted thing produces
the author has based his critique of Yakusawa’s research
a false warning? No; the argument must assume that taking
on a misunderstanding. We recall that he takes exception to
evasive action doesn’t pose a great safety risk to passengers,
Yakusawa’s conclusion that size was a determinant of the
and (A), in bolstering this assumption, effectively strengthens
birds’ survival, on the grounds that the real distinguishing
the argument. (A) doesn’t directly prove that evasive action
characteristic between the birds was their ages. In doing so,
equals safety, but at least it proves that evasive action doesn’t
the author is assuming—whether he knows it or not—that
increase the risk to passengers, which certainly makes the
all of the birds in Yakusawa’s study were roughly the same
conclusion more tenable.
in terms of their essential traits: the same species, in other
words. That’s what he understood to be the case. But if (A) (B) Airplanes are always in danger of colliding with other
is true—if in fact Yakusawa was comparing a large-blackbird airplanes, whether on the ground or in the air. Presumably,
species with a small-blackbird species—then differences in however, this sophisticated radar system is developed for and
size don’t merely reflect differences in age. If smaller birds of used in the air, so (B) is irrelevant to the argument. (On the
all ages had a better chance of surviving, then perhaps size is ground, the warning system is more likely to be the co-pilot’s
an important factor after all. (A) helps to take age out of the “hey Fred, look out!”)
equation and thus weakens the argument. (C) doesn’t strengthen the argument; if anything, it weakens
If you had trouble understanding exactly why (A) worked, it. If only one flight in a million, say, is involved in a collision
perhaps it was easier to see how each of the wrong choices while in flight, do you want your pilot to be “frequently” taking
moves into an irrelevant area: evasive action to avoid phantom airplanes?

(B) Yakusawa is measuring the survival chances of birds, so he (D) doesn’t add anything of significance to the argument.
should be looking at birds in the wild. Neglecting the issue of We already know that invalid warnings occur “frequently.”
captive birds isn’t a problem, so (B), if true, wouldn’t point out Knowing the precise percentage of false warnings doesn’t
a weakness in the argument. make the conclusion that equipped planes are safer any more
likely.
(C) Since both Yakusawa’s study and the author’s critique
concerned blackbirds only, (C)’s invocation of a comparison (E) Knowing the details of how the system works doesn’t add
with other birds has no relevance whatsoever. to our evaluation of relative safety. Yeah, fine, a computer
runs the system, but the question remains whether or not
(D) The only way “fights” might seem to be relevant is if you
passengers are safer with such a system.
inferred a connection between fighting and survival rate. But
we can’t be sure that such a connection exists. After all, not all
34. (E)
fights are fatal. And even if such a connection did exist, then
the larger birds’ success at fighting would create a paradox: The first legislator argues that government-sponsored medical
remember, Yakusawa found a greater percentage of smaller studies should include studies of women because such
birds to have lived longer. We’re not looking to cross up studies are now lacking. The second legislator responds
Yakusawa here. simply that the question of male/female balance has no
place in science. Sounds kind of rash, no? Does the second
(E) The birds’ social hierarchy is no less irrelevant than the
legislator provide any evidence for this conclusion? No. But
points raised by the other wrong choices, because it’s no less
(E) provides evidence that directly counters the conclusion.
removed from the issue of how the author has misunderstood
If using only males in medical studies results in inadequate
what Yakusawa was up to. No firm connection is inferable
science, then the question of male/female balance does have
between any kind of “social hierarchy,” whatever that means,
a place in science, and the second legislator is wrong.
and a species’ survival chances.
(A) introduces an issue that is irrelevant to the second evidence for the claim consists of a low injury rate for current
legislator’s conclusion—taxpayers. The second legislator draws workers. Since you’re looking for a choice that weakens the
a conclusion about what constitutes good science, not about manager’s claim, start by asking if the claim depends upon a
what constitutes good government. questionable assumption. Does the claim assume that nuclear
(B) The question of risks-vs.-benefits for patients used in plant injuries are just like injuries at other industrial plants?
medical research has nothing to do with the importance of Yup; perhaps the nuclear plant has fewer injuries, but those
the contribution those patients might make to the research. injuries are much more severe than the injuries sustained at
(B) offers no insight on the male-female debate that’s at the other kinds of plants. That could weaken the argument, and
heart of the dialogue and especially the second legislator’s would form a good pre-phrase, but alas, is not among the
objection. choices. What if most injuries suffered by workers at nuclear
plants develop over time, and usually don’t reveal themselves
(C) The logistics of why males currently predominate in
until after the employees have left the plant? That fact would
experiments has little if any bearing on whether or not they
also seriously weaken the argument, as it would suggest that
should predominate.
the 3.2 figure doesn’t reflect the true danger of working at the
(D) , in a classic scope shift, shifts the focus from plant. (E) takes up this issue: If (E) is true, then the 3.2 figure
participation in research to medical training. The debate does cited by the manager is very likely misleading, damaging the
not concern the training of male and female scientists, but manager’s claim regarding the relative safety of the plant.
rather the scientific adequacy of using mostly male subjects
(A) The author’s claim doesn’t preclude the possibility that the
in medical experiments.
plant’s safety record is subsidized by employees. Neither does
it mention why the plant is comparatively safe, only that it is.
35. (D)
(B) and (C) tend to strengthen the argument, not weaken it, in
We’re asked to strengthen an argument about a new form of
that they may indicate that the likelihood of long-term injury is
synthetic ivory. It seems that concert pianists like the new
being kept to a minimum.
product, but since piano manufacture accounts for a relatively
small amount of the illegal ivory trade, the new synthetic will (D) may also suggest that the plant is relatively safe, even
do little to curb the killing of elephants, the source of natural though the notion of “a few” is ambiguous; we have no way of
ivory. (D) reinforces that conclusion directly. If the most knowing how many such lawsuits are filed at other plants, and
common use for natural ivory is in ornamental carvings, and the argument does center on a comparison of safety levels. In
those carvings are prized because they’re made from authentic any case, we need not waste time debating (D)’s merits as a
materials, then it becomes even more likely that synthetic strengthener; all we need to know is that it certainly doesn’t
ivory, fine for the piano but not for the ornamental industry, weaken the manager’s claim of safety.
will do little to curb the killing of elephants.
37. (B)
(A) Even if most people who play the piano and who are not
concert pianists fall in love with the new product, it won’t What is the big advantage of the current anticounterfeiting
much help the elephants. Why? Because the amount of real method? Counterfeiters with no printing expertise can’t
ivory now being used to produce piano keys is not a major reproduce microprinting well enough to fool the experts. When
contributor to the illegal elephant trade. the author advocates replacing microprinting with special ink,
you might wonder, “How do we know that counterfeiters won’t
(B) Leans the other way —the closer the fake stuff is to the real
duplicate the special ink?” How, indeed? If (B) is true, then
stuff, the greater the likelihood that the development of the
some counterfeiters WILL get the ink and thus be able to churn
synthetic ivory will save a few elephants.
out bogus currency with abandon. At least the current method
(C) is outside the scope: The fact that other natural products thwarts exact reproduction, according to (B), whereas the
aren’t as good as ivory for piano keys has little or nothing to method the author advocates may not. If (B) is true, making
do with the effect of synthetic ivory on the killing of elephants. the switch from microprinting to the special-ink method
Again, the piano market is small, and relatively unimportant to doesn’t seem like such a good idea (that is, unless you’re one
the bigger picture here. of the bad guys).
(E) Like (B), choice (E), if anything, weakens the argument. (A) Today, counterfeit bills often circulate widely before
Anything that suggests advantages of the synthetic stuff may detection. Using the special ink would supposedly result in a
help to save a few elephants in the long run. shorter detection interval, so this choice would strengthen the
argument, not weaken it.
36. (E)
(C) If the level of detail that photocopiers can reproduce
The manager of a nuclear power plant claims that the plant increases dramatically, then it will become more and more
is safer for workers than most other plants would be. The difficult to thwart counterfeiting by using microprinting.
Therefore, this also strengthens the argument for the new drug because studies revealed that one out of five patients
anticounterfeiting method. taking the drug suffered serious side effects. We know from
(D) It doesn’t matter what kinds of criminals are responsible the stem that we’re looking to weaken the argument, so
for the bogus bills or how long they counterfeit—the fact we want to find a plausible reason why the drug shouldn’t
remains that bills they reproduce on modern photocopiers be banned. Think in terms of how the drug’s producers
circulate widely without detection. Choice (D) is thus irrelevant may respond: A one out of five problem ratio isn’t terrible,
to the argument. especially if it can be determined that the problem case was
due to factors other than asthmagon.
(E) No matter how difficult it is to estimate the overall cost of
counterfeiting, we can still compare the effectiveness of these A choice along these lines would lessen the impact of the
two methods in stopping counterfeiting. (E), however, doesn’t results of the study, and thus weaken the author’s stern
help in this respect at all. conclusion. (D) provides one such scenario. If the side effect
sufferers all had high cholesterol, their cholesterol levels may
38. (B) have contributed to their side effects. If no one with normal
cholesterol suffered side effects, asthmagon might still be
The argument is that selling some works will not detract
viable for those patients, and there would be less of a reason
from the quality of the museum’s collection because the
for taking it off the market. The author believes asthmagon is
collection includes several inferior early works by Renoir and
dangerous. (D) suggests that it may be possible for that danger
Cezanne—the assumption being that these early works will
to be controlled or even eliminated.
be the paintings sold. To weaken the argument, we simply
need to attack the notion that selling these early paintings will (A) An increase in asthma deaths in this region certainly isn’t
not detract from the quality of the museum’s collection, and a good sign, something that would effectively counter the
perhaps a prephrase came to mind—something about these author’s call for a ban. But (A) also has other problems: We
early works are good, and the loss of them would signify a don’t know whether those who died here were even taking the
drop in the quality of the collection. Choice (B) helps advance drug, regardless of the fact that it was widely prescribed.
this notion by providing an alternate definition of quality. If (B) falls into the category of “useless background
showing the development of artists is key to the quality of the information.” The fact that the patients had not taken similar
museum, then removing the immature, early works of featured drugs before does not cast doubt on the conclusion that the
artists would adversely affect the museum’s quality. Thus, (B) drug should be banned.
weakens the argument. (C) The fact that the drug continues to be prescribed does
(A) Other ways to raise funds are irrelevant to the argument, nothing to counter the author’s claim that the drug should be
which focuses specifically on the question of whether or not banned. There are many reasons why the Rhiago physicians
selling paintings to raise funds will negatively affect the quality may still rely on this drug despite the fact that it may be
of the collection. Other ways to raise money are outside the dangerous.
scope. (E) is a 180 choice that actually strengthens the argument. If
(C) The works are currently deemed immature and the drug increases the severity of some asthma attacks, this
unsuccessful, and that’s the basis of the argument. That provides all the more reason to ban it.
they were deemed as such by some critics when they were
originally purchased could only help the argument by 40. (C)
confirming the curator’s assessment. The logic in the stimulus works like this: Because quitting
(D) People’s feelings about inflation in the art market have no smoking causes stress and weight gain, taking up aerobic
bearing on how the museum’s collection would be affected by exercise will make it easier for smokers to quit. But is there a
selling certain paintings. The notion of art inflation is too far necessary connection between exercise and stress? Between
from the scope to have any bearing on this argument. exercise and weight gain? Was the study cited by the author
(E) doesn’t weaken the argument because the stimulus a valid study? Affirmative answers to any of these questions
doesn’t revolve around the amount of money the museum would strengthen the argument. Choice (C), however,
raises. Rather, the curator’s argument is that the museum can addresses none of these questions. It concerns an irrelevant
sell certain works without hurting the quality of the collection. group—nonsmokers—and the irrelevant question of what
happens to their weight when they stop exercising.
39. (D) (A) One reason that it is difficult to quit smoking, according
A brilliant name: Want your asthma to be gone? Get to the argument, is because quitting leads to weight gain. If
asthmagon! The author is not impressed, or amused. She regular exercise prevents weight gain, then quitting becomes
claims that asthmagon should be banned as an anti-asthma
easier. This supports the argument that adding a healthy shoes that have extra support to help develop an arch. What
activity to the mix is the key to quitting. would make us question the effectiveness of this treatment?
(B) Four hundred participants per group is a fairly large Essentially, we are looking for an answer that proves that
study, which makes it more likely to be valid than if, say, the this method of treatment does not develop arches in the
groups each consisted of five individuals. And knowing that children who use it. (D) does just this, by showing that there
the participants were selected at random also increases the is essentially no difference in arch development between
likelihood that the study is valid. This supports the argument. children who wear the special shoes and those who do not.
If non-wearers with flat feet are just as likely as wearers
(D) In addition to the problem of weight gain, according to the
of the gizmo to develop normal arches, then the efficacy of
argument, it is difficult to quit smoking because quitting is
the treatment must be called into question.
very stressful. If exercise reduces tension, it’s reasonable to
say that exercise does make quitting easier. (D) therefore also (A) is outside the scope because it is concerned with children
strengthens the argument. with normal arches. Why these children are wearing the
special shoes is not explained, but that’s beside the point.
(E) Hey! Not only did 40 percent of the people in the second
group quit smoking, most of them quit for a long time—at least (B) is outside the scope because this argument is about
a year. Exercise really works, right? Guess what? This supports treating children with flat feet and whether the treatment is
the argument too. successful. (B) addresses one advantage of having flat feet as
opposed to unusually high arches, but this is irrelevant to the
41. (A) treatment issue.
The question of access lies at the heart of the neighborhood (C) is also outside the scope; the issue here is the
group’s argument: the center should not be closed because effectiveness of the traditional treatment for flat-footedness,
the neighborhood needs local access to recreational facilities. not how long a cure takes.
Weigh the two parts of that argument carefully. The evidence (E) Not flat-footed? You should be suspicious already. Hip and
concerns the neighborhood’s need for access. The conclusion lower leg bones? Huh? Classic outside the scope.
concerns a specific place—the center. Is there an assumption
here? Sure—that there is a necessary connection between that 43. (B)
particular center and the neighborhood’s recreational needs. The activist claims that the plant’s closing is a victory for
In other words, the argument assumes that the neighborhood “the antinuclear cause.” So, we’re looking for something that
doesn’t have access to other facilities that could meet links the plant’s closing directly to the actions of those who
its needs equally well. If, as (A) has it, a large number of espouse that cause. According to (B), it was pressure from
the residents are unable to travel to facilities outside the antinuclear groups that forced the mandate for inspections
neighborhood, the argument would be strengthened. We’re and repairs at nuclear plants. The manager himself admits that
looking for the choice that doesn’t weaken the argument, and the cost of those mandated inspections and repairs played a
choice (A), a strengthener, fits the bill. major role in causing the shutdown of the plant. Therefore, (B)
(B) If the neighborhood contains a disproportionately small confirms the activist’s claim that the plant’s shutdown was a
number of children, then access to a recreation center may not victory for the antinuclear movement.
be as necessary as the argument suggests. This weakens the (A) provides another explanation for the plant’s closing at
argument. this particular time, one that has nothing to do with “the
(C) If the center is often open but not being used, that also antinuclear cause”—it was simply an old plant and its license
casts doubt on the notion that the center is as necessary as expired. We don’t know why the license wasn’t renewed, but
the argument suggests. nothing suggests anti-nuke activists had anything to do with it.
(D) Since programs are being canceled due to lack of interest, (C) supports the manager’s position that economic
how necessary can the center really be? competition from nonnuclear sources triggered the demise
(E) If recreation centers are declining in importance because of the plant. So much for the efforts of those working for the
computers and computer games put recreation opportunities antinuclear cause.
right in people’s homes, then access to a center becomes less (D) again suggests an economic reason for the plant’s closing;
important, and the argument again takes a hit. it was simply the most expensive plant around. That suggests
it may have closed even if there was no antinuclear cause.
42. (D) (E) gives a positive feature of the plant, a reason people might
Here we are looking for the answer that challenges the want to keep the plant open, but it tells us nothing about why
effectiveness of the traditional treatment for flat feet. So, the plant closed, so it can’t support any theory regarding an
what is this treatment? Flat-footed children must wear special “anti-nuke victory.”
44. (B) (E) is outside the scope by focusing on vaccine administration
To the author, the crisp planetary images obtained from the when the argument concerns vaccine development.
Exodus are, in and of themselves, reason for any amateur Notwithstanding (E), the profitability of vaccine development
astronomer who wants to observe planets to buy the Exodus might still be low, and hence such costs might still be worthy
for herself. But if, as (B) says, there are other factors that of subsidy.
ought to come into play in such a purchase, then maybe the
decision to buy an Exodus isn’t such a slam dunk. Given (B), 46. (C)
one would want to compare the Exodus to other telescopes in The stem offers two clues: there’s an explanation in the
terms of those “several different factors,” whatever they are, argument, and we’re to find the choice that supports it. And
before making one’s purchase decision. it’s not too tough to find the explanation—a phenomenon is
(A) “Collimation” may be a necessary condition of “certain stated in the first sentence, and then a situation explained by
types” of telescopes, but unless we know how precisely the the phenomenon is discussed in the second. First paraphrase
Exodus has been collimated—and we don’t—we can’t use (A) the phenomenon: someone who gets sick from food will often
to assess the ad’s appeal. (In fact, we can’t be sure whether develop a strong dislike of the strongest tasting part of the
the Exodus is one of those “certain types,” so who knows meal, even if that particular food didn’t cause the illness.
whether (A) applies to this situation at all?) According to the author, this helps explain why children are
“especially likely to develop strong aversions to some foods.”
(C) The conclusion is hypothetical: if an amateur is going
The best way to strengthen this argument is to bolster the
to purchase, etc., then the ad applies. Those who may not
connection between children and the factors inherent in the
choose to buy a telescope, however many or few in number,
phenomenon: getting sick, and picking out a distinctive flavor.
are simply irrelevant.
Choice (C) does this by showing these elements to be more
(D) If anything, the uniform results obtained by the Exodus at prevalent in children than in adults. If children are more likely
different times would be a tribute to its versatility and utility. than adults to get sick and better able to pick out distinctive
They would hardly cause one to reconsider the purchase flavors (that’s what’s meant by “more acute taste”), they have
decision and maybe pause, but a reason for pause is what we a greater number of opportunities to connect eating with
need from the right answer here. sickness, and even have the better taste buds to help them
(E) Like (A), too vague. So, the Exodus’ lenses differ from that lay blame on certain foods. If (C) is true, it’s easier for us to
of other telescopes. Is that good or bad? How can we use this believe that in light of the phenomenon stated in the first
to assess whether we should let the ad persuade us to rush sentence, children will be especially likely to develop food
out and buy the Exodus? aversions.
(A) and (B) are both weakeners, not strengtheners: they
45. (A) both break down the connection between children and the
Sentence 1 reveals the Rexx guy’s conclusion—that vaccine elements of the phenomenon—distinctive flavor (A) and
development deserves government subsidy—and sums up his linking food to sickness (B).
main evidence, namely that selling vaccine is by definition less (D) “Recovery” is outside the scope of the argument.
profitable than selling other drugs. The rest of the paragraph Presumably, the development of a food aversion takes place at
explains why: people use a vaccine only once, but use other the onset of the illness, so how one recovers from the illness
medicines often. So far so good, but isn’t there more to the has little bearing on the explanation in the last sentence.
issue of profitability? If as (A) says, many more people take
(E) The issue is getting sick from a meal and connecting
vaccines than take other medicines, then the possibility exists
that feeling to the most distinctive tasting part of the meal.
that it’s vaccines that may be more profitable—and hence less
Whether or not children eat only familiar foods doesn’t affect
worthy of government subsidy, which was the point in the first
the argument at all. We would need this information to
place.
somehow tie in with the aversion factors—likelihood of getting
(B) , if anything, strengthens the idea that vaccines might sick and picking out distinctive flavors—in order for (E) to have
be less profitable. (Medicinal alternatives to vaccines might any bearing on the situation at hand.
translate to a lessened need for vaccines, and thus fewer
sales.) 47. (B)
(C) Since the Rexx guy’s argument hinges on the comparison Triple-trailer trucks (say that three times fast) are more
between vaccines and other medicines, nonpharmaceutical dangerous than other commercial vehicles and should not
products are outside its scope. be allowed to use the nation’s highways. Hogwash, says our
(D) That the subsidy might benefit companies other than author, just look out west. Triple-trailer trucks (TTTs) use the
merely Rexx doesn’t affect the logic of the subsidy. highways out there and their rate of road accident fatalities
is lower than the national rate for other commercial vehicles. 49. (E)
“Clearly,” these TTTs not only aren’t more dangerous, they’re What’s worse, the disease or the vaccine? The author’s
actually safer than other commercial vehicles. How’s that for conclusion, signaled by “therefore,” is that it is safer for a
an air-tight argument? Well, how air-tight is it if (B)’s true? dog to receive the vaccine than not to receive it. The author’s
What if those highways out west are so seldom used that evidence for that conclusion is that: (1) 1 in 50 dogs who
they’re much, much safer than other highways? What happens contracts the disease dies; (2) the vaccine prevents the
to the argument’s conclusion that the TTTs are actually safer disease; and (3) only 1 in 5,000 dogs will die from the vaccine.
than other commercial vehicles? It’s certainly weakened (if not So far, the vaccine sounds like a pretty good risk, since a 1 in
blown out of the water). 5000 chance certainly sounds less risky than a 1 in 50 chance.
(A) The argument deals with the safety of the TTTs, not their But wait a minute: the disease kills 1 in 50 dogs that contract
hauling capacity. (A) is outside of the scope. it. What are the chances of contracting the disease? If all dogs
(C) The past wisdom of the TTT opponents is irrelevant. If get the disease (so 1 in 50 dies), it’s better to give all dogs
anything, (C) may look like a slight strengthener (though the vaccine because only 1 in 5,000 will die from the vaccine.
it’s really too vague for that) by showing that the opponents However, if only 1 in every 10,000 unvaccinated dogs actually
lacked good judgment in these matters in the past. gets the disease, then only 1 in 500,000 unvaccinated dogs
would die (remember 1 in 50 who have the disease die). In
(D) , if anything, would seem to strengthen the argument. If
that case, the disease would be less risky than the vaccine.
drivers of the TTTs had to get a special license (with special
Since the author’s conclusion depends upon the likelihood
requirements), then maybe they’re better drivers, which would
that an unvaccinated dog will contract the disease, (E) would
tend to make the trucks slightly safer than others in areas
be most helpful in evaluating the argument.
where they’re both operated. In any case, (D) certainly doesn’t
weaken the argument. All four wrong answer choices are outside the scope. (A)
talks about all causes of death (the argument is limited to a
(E) The argument compares the safety of TTTs to the safety of
discussion of only one disease); (B) introduces other pets;
other commercial vehicles. The TTTs could still be safer than
(C) discusses causes of death from other diseases; and, (D)
other commercial vehicles, even if their own rate of fatalities
discusses the likelihood of contracting other diseases.
has been on the rise of late.

50. (B)
48. (D)
Answer choices that are outside the scope of an argument
This all/EXCEPT question is looking for the one answer
cannot be strengtheners.
choice that does not support the prediction that the
new light bulbs will sell very poorly, even though the Although scientists have long hypothesized that the “pits”
evidence suggests that they will be more cost-effective on pit vipers help them be better predators, we are asked to
than conventional bulbs. Choice (D) has no effect on the strengthen a different hypothesis: that the infrared sensors of
argument (thus it certainly can’t support it), since there is no the pits on pit vipers serve primarily to help the snake avoid
evident relation between the number of bulbs per package becoming some other predator’s dinner.
and the bulbs’ cost-effectiveness. Therefore, nothing in (D) One way to determine whether or not the pits related more
would lead us to believe that the analyst’s prediction is to predatory or defensive behavior would be to compare the
bound to come true or fall flat. All of the wrong choices allow behavior of pit vipers to that of their relatives without pits.
us in some way to infer that people are not likely to want If the pits in fact played a role in the pit vipers’ defensive
these bulbs. But the innocuous (D) is neutral, and gets the behavior, then one would expect pit vipers to behave
point for this question. differently than other vipers without pits when it comes
(A), (B), and (C) all support the analyst’s prediction by to defensive behavior, and one would expect pit vipers to
providing plausible reasons why consumers may not choose to have no significant differences in predatory behavior, as
buy the new bulbs. compared to their relatives without pits. (B) says just that, and
strengthens the researcher’s hypothesis.
(E) tells us about a newer, even more cost-effective light bulb
that would very likely attract consumers away from the bulb (A) The argument makes no distinction between male or
discussed in the stimulus. After all, if the competitor’s newer female pit vipers, and so this answer choice is outside the
bulb is even better than the bulb in the argument, won’t many scope of the argument.
consumers choose the better bulb? (E) also supports the (C) The argument is strictly about the role played by the
analyst’s prediction. infrared sensors possessed by pit vipers. Information
about the chemical composition of vipers’ venom (or other
distinguishing characteristics) is outside the scope of and to prefer the new approach. This is all well and good, but
irrelevant to this argument. the author does neglect a full half of the supply and demand
(D) Additional characteristics of pit vipers (such as a well- equation—what about the consumers? If the consumers aren’t
developed sense of smell) that may provide information in going to buy the new detergents, the manufacturers, and this
addition to the thermal impressions they receive from their argument, are sunk. If, on the other hand, the consumers had
infrared sensors says nothing about the role played by those a reason to prefer the new approach, then the argument would
infrared sensors. This answer choice is also outside the scope be strengthened. Answer choice (E), proposing that consumers
of the argument. would prefer concentrated detergents due to environmental
concerns, fits the bill.
(E) Information about other defense mechanisms possessed
by some pit vipers says nothing about whether the infrared (A) , if anything, weakens the argument. If smaller
sensors that distinguish pit vipers from other vipers are also manufacturers were unable to adopt the new approach, then
defense mechanisms. Another answer choice that is outside the eventual disappearance of bulky detergents would be
the scope of the argument. much less likely.
(B) and (C) Both of these also lean more towards weakening
51. (A) than strengthening the argument; both paint a negative
The author argues that we should abandon peer-reviewed picture of the new detergent/consumer relationship. Initial
journals so that new medical discoveries can more quickly skepticism about the effectiveness of the concentrated
be used by the public to improve their health. In weakening detergent answer choice (B) could only weaken the argument.
this argument, we must undermine the evidence for that However, since the argument refers to the eventual
conclusion. In this case we need to provide a reason why disappearance of bulky detergents, initial skepticism wouldn’t
expediency isn’t in the public’s best interest. In (A), the damage the argument too much, making this choice mostly
journals serve some other, more useful function—they keep irrelevant without additional information. As for answer choice
potentially dangerous and ill-formed medical conclusions (C), if the analysts are correct and consumers have to pay more
from reaching the street where they might negatively impact to use the concentrated style, the law of demand suggests that
on public health. If (A) is true, a slow process is in the best bulky detergents would be more likely to stay on the market.
interests of the public and we shouldn’t rush publication. (D) A discount from major supermarkets would have been a
(B) is consistent with the author’s argument; the fact that boost to the argument. However, if supermarkets charge the
people act on the medical information they receive through same for shelf space for the new product, then it’s hard to see
the press is almost a necessary component of the argument, how this would affect the argument at all.
but technically “alter their life-styles” may be a little extreme.
In any case, in no way does this weaken the argument. 53. (B)

(C) This choice is outside the scope, plain and simple. This How fortunate for us; two questions based on a single
argument concerns improvements in public health derived explanation of a phenomenon, one to weaken that
from new medical information, period; other factors are explanation, one to strengthen it. The phenomenon in
irrelevant. question is the commercial resurgence of the fantasy genre
in North America, specifically the increase in fantasy-fiction
(D) , if anything, strengthens the argument by
books for adults. The explanation, given by booksellers, is
shoring up the viability of the author’s plan to publish
that the recent boom in this genre is due to favorable reviews
quickly.
given to these works by book reviewers. It’s not too difficult
(E) The issue isn’t peer review or no peer review— the central to prephrase a weakener for this explanation—if the buyers
issue of the argument is whether the practice of informing the of such books don’t read, care about, or base purchasing
press of new medical findings only after peer review should be decisions on such reviews, then this argument is a whole lot of
abandoned. In other words, peer-review journals can still do hot air. And that’s exactly what we get in (B).
their thing—this in and of itself doesn’t disrupt the argument,
(A) is generally irrelevant, because it leaves out the purchasers
since peer review can still be performed after the press has
of books altogether. What publishers think will receive good
been notified.
reviews is too wishy-washy to have any effect on the argument,
no less weaken it.
52. (E)
(C) The fact that booksellers are aware of the content of the
The market analysts predict that the decision of leading
reviews is also irrelevant; it has no bearing at all on the affect
manufacturers to concentrate their detergents will lead
the reviews have on the purchasing public, which is, after all,
to the virtual disappearance of the older, bulky style. The
what the booksellers’ argument is about.
concentrated detergents will be sold in smaller packages that
will save production costs, which gives manufacturers a reason
(D) is even further removed from the crux of the argument; 56. (A)
it totally ignores the main issue of why this type of book has The argument is pretty simple, summed up in the last
become so popular. sentence: patients shouldn’t get checkups unless they feel
(E) is, at best, neutral. At worst, it’s a 180—if people pay sick. (A) weakens the argument by showing that a patient
attention to the reviews, this would provide even another who may not feel ill might nevertheless benefit from an
reason (historical) for people to get into this genre. examination.
(B) doesn’t address the issue of whether patients who feel fine
54. (E) should go for a checkup. Furthermore, this choice deals with
Having thought through the weakener for the previous the amount of time a doctor spends on a checkup, while the
question, all we have to do is reverse our thinking to haul in argument is concerned with thoroughness.
this point: to strengthen the argument, we need a choice that (C) The argument discusses the doctor’s judgment of how
says that book purchasing behavior is influenced by reviews thorough to be with a patient. Knowing that a patient is no
of the kind described in the stimulus. And that sentiment more able than a physician to make that judgment doesn’t
appears in (E): as a result of critic’s favorable reviews, adult weaken that argument. Like (B), this choice doesn’t offer any
book buyers began to see the fantasy-fiction genre as a viable reason why a seemingly healthy individual should go for a
option for their reading pleasure. checkup.
(A) We have no idea how complex or simple the books in this (D) If anything, this strengthens the argument by presenting
genre are, and therefore have no way of knowing how the another good reason to avoid checkups.
general reading level of the public affects this argument.
(E) Although some physicians might exercise the right amount
(B) , (C), and (D) all may contribute in their own ways to a of thoroughness, others might not. As long as there are some
general explanation of the recent popularity of fantasy-fiction that don’t, the argument against “unnecessary” checkups still
books, but none offer any support for the booksellers’ specific holds.
argument—that the popularity of these books is due in some
way to favorable reviews. 57. (B)
One of the reasons given for healthy patients avoiding
55. (C)
checkups is that some doctors are over-thorough and hence
The advertisement reaches the conclusion that its product’s likely to perform unnecessary tests. (B) points out a possible
safety is “assured.” However, the only evidence offered on danger in these unnecessary tests and, in doing so, lends
this point is that the product has been tested by a reputable support to the notion that patients who feel fine should avoid
laboratory. If the test results were positive, then the checkups.
conclusion would seem fairly justified. However, if the test
(A) Fairly irrelevant; if anything, this weakens the argument
results were negative, then the ad would be grossly deceptive.
by softening one of the negative aspects of the over-thorough
The fact that the product has been tested is not enough; we
doctor.
need to know the results of the tests to determine the ad’s
truthfulness. (C) Those who don’t believe a physician who pronounces
them healthy undoubtedly believe themselves to be ill. The
(A) Since the ad only deals with the safety of the hedge
argument, however, is aimed at those who feel fine.
trimmer, knowing whether other products have also been
tested won’t bring us any closer to judging the ad’s validity. (D) weakens the argument, in that it demonstrates a possible
benefit of a complete medical checkup. In this case, the
(B) How the consumers perceive the safety issue is one
exhaustive testing might save a patient’s life, whereas the
step away from the argument. Whether or not people regard
passage views such testing as a hindrance.
safety as important, the hedge trimmer may still be safe or
dangerous. (E) Another choice that leans in the other direction, by offering
a reason why going to the doctor isn’t so bad. Presumably
(D) Who cares? The ad assures the safety of a hedge trimmer;
this type of doctor would be thorough without burdening the
alternative ways of trimming hedges are irrelevant.
patient with unnecessary uncomfortable and expensive tests.
(E) Knowing the answer to this question won’t help us While it may not be a very convincing weakener, it certainly
evaluate the ad, because the advertisement doesn’t claim that doesn’t support the argument either.
Bolter’s is the best for trimming hedges, only that its safety is
guaranteed. 58. (B)
The author concludes that the higher patient mortality rates
in certain hospitals as opposed to others are probably the
result of lower quality care. The evidence for this includes the violent events. That’s not very impressive. Who knows how
fact that the hospitals all had roughly the same per-patient much junk food also went down the criminals’ gullets?
funding; in other words, lack of money didn’t account for the This single instance has little or no effect on the broad
poor hospitals’ bad showing. But that eliminates only one generalization the author describes.
other possible explanation for the result. Another, equally (D) The first experiment the author describes relates, as (D)
plausible explanation why some hospitals are deadlier than does, to inmates’ voluntary dining choices. But that which
others (other than differences in the quality of care provided) people choose to eat has little or nothing to do with the effect
would severely weaken the argument. We get this plausible of their food on their behavior. That first experiment is mostly
explanation in (B): if some hospitals receive much sicker there to set up (D) and see whether you can be thrown off
patients than others, then the difference in mortality rates the track.
might reflect the difference in the severity of the patients’
illnesses rather than a difference in the quality of care the 60. (C)
hospital provides.
The most effective way to weaken an argument is to attack
(A) Even if the high death rates in some hospitals are due to one of its necessary assumptions.
the fact that the staff in these hospitals has less training, this
Why have world rice prices gone up? According to analysts—
would strengthen the argument, by making it plausible that
the ones we’re told to attack—it’s the tiny amount of rice
the poorly trained personnel are providing lower quality care.
exported freely vs. the larger amounts that governments dole
(C) is irrelevant, as we have no way of knowing from this out to their own people. This is a tidy explanation, but if (C)
statement which hospitals have the most experienced is true, then today—a time of decreased rice production—
staffs. And even if we did know, we still wouldn’t be able to governments should be exporting more rice than normal.
legitimately tie this info in with the quality-of-care issue. (C) is This destroys the analysts’ explanation, which hinges on the
simply way too vague to damage the argument. exports of rice being limited, not expanded.
(D) is a 180: it slightly strengthens the argument by (A) The argument is about why prices have gone up, not about
eliminating a possible alternative reason for the difference in the effects of an increase.
death rates—the types of surgery offered.
(B) Government rice storage is quite consistent with the
(E) essentially does the same thing as (D), by eliminating analysts’ view. Whether rice is stored or distributed locally, it’s
another possible explanation (regional differences not related still unavailable for export.
to quality of care) that otherwise could damage the argument.
(D) A government’s sharing of its own grain commercially
If mortality rates are the same from region to region, then the
would have no demonstrable effect on world markets insofar
difference in rates between hospitals can’t be due to what
as the passage describes them.
region they’re in.
(E) Other crops fall outside the scope of the argument about
59. (E) rice prices.

To strengthen a perceived cause and effect, it helps to show


61. (B)
that an absence of one factor is followed by an absence of the
other. The evidence states that people 100 years ago had few
diversions. The conclusion is that they therefore must have
The conclusion that poor nutrition causes violent behavior is
read more than we do today. However, the connection
based upon the experiment in which violent inmates who were
between having fewer diversions and reading more is
on a high-nutrient diet saw steady behavioral improvement.
fairly tenuous, and any number of facts could weaken this
But who’s to say that the one caused the other? Maybe it
connection, and therefore weaken the argument. (B) for
was coincidence; maybe there were other, more important
example, states that people 100 years ago had much less
intervening factors. The relationship is more likely if, in the
leisure time. This certainly weakens the argument. Even if
absence of the nutritious diet, violent behavior saw no change.
there’s not much to do besides read in your leisure time, if you
(E) describes the control group whose results make the
don’t have the leisure time in the first place, you don’t have
conclusion considerably more persuasive.
time to read.
(A) The amount of pre-incarceration violence is irrelevant to the
The low literary quality (A) and higher cost (D) of books 100
experiment as described.
years ago doesn’t weaken the argument. After all, people
(B) The ease and expense of implementing a course of action might settle for more expensive or worse books if there was
has nothing to do with its effectiveness. nothing else to do. The specific number of books (C) is also
(C) So, the ingestion of some healthy food came irrelevant, since the number of people alive 100 years ago is
“sometime . . . before” (How long? Days? Weeks?) some not the same as the number of people alive today, and we
thus, cannot draw conclusions about how much each person everyone uses telephones, so we might wonder why more
reads. (E) comes completely out of left field, and introduces people don’t have rashes on their faces.
the entirely irrelevant topic of horse racing. (A) notes that telephones are made up of allergenic plastic,
which would certainly support the dermatologist’s hypothesis,
62. (A) as would (B), which makes it less likely that the one-sided rash
The evidence boils down to this: When VNO cells are is provoked by something other than a telephone. (C) and (E)
stimulated, people have “subtle smell sensations.” The both ensure frequent telephone exposure in affected patients,
conclusion is that the VNO is therefore a functioning sense either because of occupation (C) or recent usage increase (E).
organ in humans. To weaken the argument, look for something
that gives a different cause for the smell sensations. Now 65. (B)
that we know what we’re looking for, we can see that (A) In 2900 B.C., Mesopotamians stopped growing wheat and
does exactly that. If other sense cells were stimulated at the started growing barley. Why? Some historians think it was due
same time as the VNO cells, then it might have been the to too much irrigation, bad drainage, and the resulting build-
other cells and not the VNO cells that caused the sensation. up of salt in the soil. This isn’t much of an explanation unless
That definitely weakens the argument that the VNO cells are a barley is somehow better suited to these conditions than
functioning sense organ. wheat, and (B) provides just such an explanation. If barley
(B) , (C), and (D) all function in similar ways. They point out can resist the salty soil, then the historians have a stronger
that the VNO in humans is rudimentary (B), or chemically argument that the crop switch was due to the soil conditions
(C) or anatomically (D) incomplete, as compared to animals listed.
with more developed VNOs. Just because it’s less advanced The problem was not drought but over-irrigation—too much
doesn’t mean that it doesn’t function to some extent, so none water, not too little. (A) states that barley needs less water,
of these choices effectively weakens the argument. (E) is a so it would weaken, not strengthen the argument. (C) would
180-answer choice, since it actually strengthens the simply let us know that the Mesopotamians grew both wheat
conclusion by providing a parallel example from other and barley before the switch in emphasis, but the fact that
species. barley was around before the switch doesn’t explain why the
switch was made, and so doesn’t strengthen the argument.
63. (D) (D) would weaken the argument by providing another, equally
To undermine Galina’s rebuttal, we first need to understand or more plausible argument for the dominance of barley. (E)
that rebuttal. She says that the squirrels can’t be after sugar, explains that barley became the predominant grain in the diet
because the concentration of sugar in maple sap is so low. of Mesopotamians after 2900 B.C.—hardly a surprise, since
To undermine this, we need to find a reason that the low people either eat what they can grow or grow what they like
concentration of sugar wouldn’t deter the squirrels. (D) to eat. In either case, the predominance of barley in the diet
provides just such a reason—if the squirrels let the sap sit doesn’t strengthen the historian’s contention that the shift was
until the concentration is much higher, then the initial low due to soil conditions.
concentration of the sap wouldn’t matter.
(A) directly tells us that squirrels like sugar, while (B) indirectly 66. (D)
hints at the same thing, but neither of these bear directly While conceding the need for equal TV time for shows about
on Galina’s rebuttal—Galina says the sap is too watery, and social issues, the author denies that need when it comes
(A) and (B) don’t address this point at all. (C) seems a little to shows about scientific issues, on the grounds that social
more relevant, since it establishes that squirrels like sugary issues are political and can’t be settled…the implication being
trees more than less sugary trees, but it still doesn’t explain that scientific issues are totally different. Any choice attacking
the fact that the sugar maple sap is too watery. (E) states the assumption that social issues and scientific issues are
that sometimes during sapping season, it’s impossible to mutually exclusive must therefore weaken the logic, and (D)
get the sap. Since we already know that squirrels consume does just that. If (D) is true, by the author’s own logic there
sap (presumably at those times when it’s a little warmer), (E) would be a strong rationale for offering equal time for at least
doesn’t bear on why squirrels eat the sap. some scientific TV shows.
(A) is a 180, strengthening the argument by driving an even
64. (D) greater wedge between social and scientific issues. The
The dermatologist thinks that telephones are provoking number of opposing views, whether it be 2, 3, or more, is not
rashes. Since this is an “all of the following EXCEPT” question, fundamental to the author’s reasons for agreeing to some
the correct response will either weaken the argument or not equal time but not all (B). By stating that “social issues almost
affect it all, while the four wrong answers will strengthen the always” are political and “seldom can…be settled,” the author
argument. (D) is a mild weakener, since it notes that almost
leaves the door open for exceptions, so when (C) points out assertion of a biased survey question to the assertion that
those exceptions his logic is undamaged. (E) ventures into the skepticism about the results is called for.
utterly irrelevant issue of what TV networks are willing to do. The choice that fails to strengthen the argument is (E). Other
The argument’s logic can hold, or falter, whether equal time is things that Americans might be concerned about—such as
granted willingly or under duress. social issues—are irrelevant to the central comparison that
the magazine made, the comparison between politics and
67. (D) personal finances that (E) utterly ignores.
According to the studies, people held on to newly acquired
beliefs long after they were told that the beliefs were based 69. (D)
on bogus evidence. To undermine the author’s indictment Using the One-Sentence Test (“which sentence would the
of people’s irrational stubbornness, we need to bolster author least like to part with?”) confirms that the conclusion
the roots of these new beliefs; that is, we need to seek is the last sentence. B.t. toxins are preferable to chemicals
“credible evidence” that the author believes to be “absent.” in controlling insect pests. Why? Because of the unlikelihood
(D) provides it, by suggesting that the people in the studies of harm to mammals, birds, and other insects that prey upon
did indeed have credible and independent confirmation of the pests. But (D) provides a compelling reason to prefer
those new beliefs. In other words, (D) serves to render moot chemicals to B.t. If insects build up resistance to B.t. faster
the bogus nature of the original evidence. The believers had than to the chemicals, that would certainly make the former
already gotten solid support from other sources. less effective over the long haul.
The correctness of the original beliefs (A) is not at issue. What (A) is tempting for those who have not read carefully, but is
is at issue is why people hang onto new beliefs—correct or actually a 180. While at first glance it seems to weaken the
not—in the face of no evidence. That we can’t expect people conclusion by suggesting that chemicals kill more insects,
to revise their beliefs when the support is undercut (B) is this shifts the scope of our desire; we don’t want to kill all
irrelevant to why people do so. (C) speaks of the nature of the insects, just pest insects. That B.t. toxins are limited to these
statements “even if true,” but it is the very falseness of the pests actually strengthens the argument. (B), (C), and (E)
statements that causes the author to derive his conclusion. all venture beyond the scope of the comparison between
(E)’s initial skepticism is irrelevant since it did give way to B.t. and chemicals. Notice that none of these three answer
firmly-held belief ultimately; if anything (E) makes it even choices focuses on that comparison; none even mentions the
more curious that people would hold onto their new beliefs chemicals.
irrationally.
70. (A)
68. (E)
“Seriously weakens” tells you that the argument is
The author’s conclusion is that one should be skeptical of vulnerable—probably in terms of its assumptions.
this magazine’s finding that North Americans care more about
After some general claims about the distinctness senses,
their personal finances than about politics. The evidence
the author gets to his main point: Synesthesiacs have
for skepticism is pretty convincing: There would seem to be
senses that fail to respect (that is, break down or cause an
a scope shift between the survey question “Which do you
overlap between) the distinctiveness of the 5 usual ones, as
think about more?” and the conclusion’s “Which are you
evidenced by instances of people smelling and tasting colors.
concerned about more?”; the buzzword “joy” seems designed
But the author is assuming that the blurring of the 5 senses is
to distort; and the sample is self-selecting. And yet we want to
the only plausible explanation for the synesthesiacs’ behavior.
strengthen the conclusion even more—in four ways!
If (A) is true, then the explanation lies most likely with
The magazine’s past credibility problems (A) don’t prove that impaired vocabulary than altered senses.
the results are dubious. But then, strengthening an argument
Other possible senses, (B), are irrelevant to an argument
doesn’t require proof, merely greater likelihood; and the
about the blurring of the other 5. That the overlapping of
magazine’s questionable past certainly makes it more likely
senses fit a pattern, (C), might strengthen the argument in that
that it would draw a questionable conclusion. (B) casts doubt
they suggest design and order. The existence of synesthesiacs
on magazine survey conclusions in general, and if “most”
isn’t in doubt, just the nature of how their senses work, so
have been rebutted it follows that any individual one probably,
their widespread appearance, (D), or the ability to cure them,
if not certainly, would be rebutted as well. (C) brings up other
(E), fall outside the scope.
surveys of the same sample population that came up with a
180-degree different result; surely at the very least (C) implies
that the issue is in doubt. And (D) confirms an assumption
that the author seems to be making when he moves from the
71. (B) the greater crime coverage as a media policy rather than a
The question type asks you to sever the connection between reflection of daily reality on the street.
evidence and conclusion. (B) concedes that the author is right about the city’s violent
The conclusion that we are to weaken speaks of the crime, and knowing that it’s not confined to his own city won’t
impossibility of using standard techniques to date prehistoric make the author breathe any easier. No comparison between
paintings on limestone, given that standard techniques cannot neighborhoods (C) is relevant whatsoever, nor is a distinction
determine age if carbon is present, and limestone contains between murder and other crimes (D)—it’s all lumped
carbon. But what if the limestone, and hence the carbon, together. The role of news magazines (E), whether positive or
could be removed? If (B) is true, then the only barrier to dating negative, is outside the scope, since this argument concerns
put forward by the author is eliminated. newspapers and TV.

(A) confuses techniques. We are concerned with those in place


74. (D)
to date the paint, not to collect the samples. (C) dates the
wrong thing—it’s the paintings, not the limestone, that need This stimulus cuts a little deeper than most, philosophically
to be dated. The absence of other painting materials, (D), has speaking. The idea introduced in the first sentence and
no effect on the logic, because the techniques only work on reinforced at the end is that any given state will have its own
vegetable matter anyway. And carbon is, so far as we know, conception of “social justice” that goes beyond any universal
the one and only impediment to dating, so its proportion set of principles about that which is just. The example they
relative to other elements, (E), is irrelevant…unless the provide—that two states may have different ideas about the
proportion of carbon is 0%, which it would be under correct just way to treat the elderly, even though both states operate
choice (B). under the same general ideals—helps to clarify matters and
also may help to identify (D) as correct. In the same way as the
72. (C) stimulus example, it is revealing that two different states have
different ideas about the just way to qualify for day care, even
The task here is to weaken the conclusion that a single drop
though their general principles of justice are the same.
of this defendant’s blood stains “much less than 9.5 square
cm of the fabric.” The evidence consists of 10 samples, each (A) and (B)’s examples don’t focus on social justice so much
of which covered about one-half of that area. But if an 11th as political justice (A) and economic justice (B); each is a poor
sample covered almost the entire 9.5 square cm, as (C) points example of the specific principle described in the stimulus. (C)
out, then the witness’s certainty about a droplet’s being sure points to a sameness between two states, which is only part of
to stain “much less than 9.5” must be reduced to almost zero. the story. And (E) has it backwards—two states with different
general justice principles but the same social justice policy.
(A) ’s hypothetical addition of 100 more examples of the same
phenomenon hardly weakens the impact of the original 10
75. (A)
cited.
Don’t cut corners when thinking about the terms of the
(B) is as it may be, but this particular expert witness may not
argument; be specific.
be one of those fudgers. And we don’t even know which side
this witness is testifying for. (The evidence sounds exculpatory Like many arguments, this one starts with a broad topic and
to us, and hence this is likely a defense witness.) gradually narrows the scope until the author comes to their
(D) ’s reference to someone else’s blood couldn’t be more
point. Here, he begins by telling us that polls can distort the
outside the scope of an argument solely dealing with that of outcome of elections by influencing voters, and that polls
the defendant. published close to election day are particularly problematic
since candidates have no chance to respond to them. This
(E) has the same problem as (B)—there’s no reason to suspect
leads to his conclusion that publishing polls should be
that this particular allegedly expert witness is in fact a phony.
banned during the week leading up to the election.

73. (A) Notice all the qualifiers that the author uses: polls “can”
influence voters’ decisions. They “may” distort the outcome
The conclusion here is rather hysterical given the relatively
of elections. We should ban the publication of polls, but only
straightforward evidence. Simply on the grounds that the
during the week just before the election. All of these may seem
media are full of violent stories, the author concludes that
insignificant, but they are actually the crux of the author’s
there’s an epidemic of violent crime going on that should
argument. In arguing that a blanket ban is necessary, the
keep all of us behind locked doors as much as possible. But if
author assumes that the publication of poll results right before
the increase in media crime coverage is actually due to some
the election will always distort the outcome of the election,
factor of taste or readership, then the link between the media
even after he said that such a distortion is only a possibility.
coverage and the incidence of crime is severed. (A) defines
This shift in scope, while subtle, is what allows us to find
the weakener. (A) breaks down this assumption by telling 77. (C)
us that polls published during the week before the election Weaken causal arguments by finding an answer choice that
don’t really have that much influence on the outcome of the mentions an alternative cause.
election. Thus, the author’s reason for banning them isn’t as
On its most basic level, this argument claims that Friday the
strong as he suggested.
13th caused a higher percentage of people to cancel their
(B) is too extreme. Even if the publication of poll results flights. To weaken this argument, we can look for the three
wouldn’t decide the winner of an election outright, it could still classic causal alternatives. Instead of X causing Y (Friday the
distort the outcome, perhaps by widening the gap between the 13th caused flight cancellations), we can look for a reversal,
candidates. or X caused Y (flight cancellations caused Friday the 13th,
(C) and (D) are both 180s. The author tells us that publishing which doesn’t sound very likely); a choice that claims the
poll results distorts the results of elections. These two relationship was a coincidence (another year had the same
choices clarify how the distortion could occur, which actually rate of cancellations on a different day); or an alternative
strengthens the argument. cause (some other reason why people cancelled their flights).
(E) is an irrelevant comparison. It doesn’t matter which We find an alternative cause in (C): severe weather could have
countries’ citizens are better informed. The publication of poll caused people to cancel their flights.
results right before an election could still distort the results of (A) The comparison between professionals and the rest of
elections. the general public is irrelevant to the argument; people still
cancelled their flights, whether they were superstitious or not.
76. (C) (B) was probably tempting, because it suggests that a much
To weaken an argument, find a choice that makes it less likely smaller percentage of people think Friday the 13th is unlucky
that the conclusion is true. than the percentage who cancelled their flights that day. But
The astronomer is convinced that Mount Shalko is the perfect (B) deals with a percentage of the population, whereas the
site for the new observatory, and gives several reasons why. argument deals with a percentage of those scheduled to fly.
Not only is the site an excellent fit for the size and scientific (D) Automobile traffic has no effect on flight cancellations.
needs of the observatory, but according to the author, it would (E) The conclusion deals with people’s willingness to alter
actually save the unique wildlife of the mountain from the travel plans, but airline workers were planning on working, not
threat posed by recreational use of Mount Shalko. That last traveling, even if traveling is part of their job.
bit of evidence sounds particularly far-fetched—building an
observatory on a mountain will actually protect the wildlife on 78. (B)
the mountain? Sounds pretty doubtful. Perhaps the critics are
The correct answer to a Strengthen question will make the
right about the ecological threat posed by the observatory. (C)
conclusion more likely to be true.
says just that: the observatory could cause as much ecological
disruption as the current recreational use. This would negate The geologist is apparently responding to a proposal for
the last piece of the astronomer’s evidence, making his a nuclear waste storage facility under a certain mountain.
conclusion less likely to be true. He argues that if a geological fault is present under the
mountain, it could precipitate a disaster in the storage of the
(A) Whether or not the endemic species are unique does not
waste. This leads him to conclude that the storage project
argue for or against siting the observatory on Mount Shalko,
shouldn’t go forward until scientists check to see if there is a
because recreational users of the mountain would threaten the
geological fault under the mountain. (B) provides an additional
species even if the observatory were not in place.
piece of evidence in favor of this argument—the scientists’
(B) The number of opposition groups has no bearing on the investigation will tell us, once and for all, whether such a fault
appropriateness of the observatory site. exists under the mountain, which will give us the information
(D) and (E) actually strengthen the author’s argument by we need.
removing potential weakeners. If the small towns near Mount (A) would actually weaken the argument, by putting forward
Shalko eventually developed into a city, the accompanying the possibility that it wouldn’t really matter if the nuclear
light and smog would compromise the atmospheric waste seeped out or water seeped in. (A) is a 180.
transparency that the observatory needs, and if other nearby
(C) The initial intent behind the facility is outside the scope.
mountains are better for recreational use, then the 200,000
What matters is the intent to use the facility for nuclear
current recreational users would be more likely to relocate
waste now.
their activities once the observatory is in place, removing the
threat to wildlife. (D) and (E) would both weaken the argument, (D) by
suggesting a reason why the scientists’ investigation should
not go forward, and (E) by suggesting that any problems with (E) makes no distinction between minor and major artists, and
the facility could be solved by removing the waste after a so cannot help the argument here.
period of time.
81. (B)
79. (B) Reporters at the newspaper are paid less than their
Look closely at the parameters of any experiment that is counterparts at other newspapers. Management responds
supposed to apply to the entire population. that this is OK, since these reporters get valuable training
The advertisement concludes that at least 50 million North that makes up for the shortfall in salaries. We’re looking for a
Americans should take lactase supplements to correct lactose choice that undermines this justification, and so the correct
intolerance, which would provide quite a large market for answer will suggest that the training these reporters receive
the supplements. But this conclusion is based entirely on doesn’t have much value after all. If (B) is true, and most
an experiment in which the subjects drank an entire liter of of the reporters have worked there for 10+ years, then the
milk on an empty stomach. Perhaps this is a new breakfast training the reporters receive ostensibly isn’t translating into
craze, but it doesn’t sound like the typical way in which milk, extra cash, since they’re still working for the same cheapskate
or any other dairy product, is consumed. In other words, the company. “Valuable training” doesn’t seem so valuable if
circumstances of the experiment weren’t necessarily relevant you’re still underpaid by industry standards, so (B) weakens
to the population at large, unless everybody drinks a liter of the argument.
milk on an empty stomach every now and again. (B) points out (A), if anything, suggests that reporters may be getting
this discrepancy as a weakness in the argument. valuable training that will pay off when they become senior
(A) would actually strengthen the argument, by suggesting that reporters (at least their salaries would then be on par with the
the subjects’ empty stomach contained more lactase than a salaries paid at competing papers).
full stomach would have and could thus have readily digested (C) suggests that business isn’t growing but tells us nothing
more lactose. Eliminate this 180. about whether reporters receive “valuable training.”
(C) The age of the subjects is outside the scope of the (D) and (E) provide useless background information. Who can
argument. tell what the difference in union representation means to the
(D) Strengthens the argument by providing a reason to avoid value of training? What difference does it make where the
lactose intolerance (perhaps by taking lactase supplements). newspaper is read? You’d have to try pretty hard to find a way
to make either of these relevant one way or the other, but that
(E) also strengthens the argument, suggesting that the lactose
kind of creativity isn’t rewarded on the GMAT.
in the experiment was easier to digest than other forms of
lactose, yet still caused problems.
82. (B)

80. (D) The question stem, in an unusual way, tells us we’re looking
for a weakener. So, we’re looking to weaken the notion
The author notices that the number of Dutch landscapes
that humans can only be made happy by gratifying their
attributed to major artists is similar to the number that are
higher cognitive functions, once they have become aware of
attributed to minor artists. This seems odd, since minor artists
those faculties. (B) contradicts this notion. If many humans
of the time outnumbered major artists by a hefty margin. All
who are aware of intellectual gratification nonetheless prefer
other things being equal, we would expect that more artists
physical pleasure, this would suggest that they can be made
would produce more surviving works. So, the author concludes
happy by something other than gratification of their higher
that many of the attributions are false. The correct answer
cognitive faculties, in which case the author’s conclusion
will bolster the notion that works by minor artists were falsely
would seem too hasty.
attributed to major artists, and that’s what (D) does. (D) says
point-blank that minor artists were paid not to sign their (A) suggests that some animals possess certain kinds of
works so that dealers could engage in the trickery the author cognitive faculties, which does not at all conflict with the
suggests. So (D) is our strengthener. author’s assertion that human cognitive faculties are simply
on a higher level. (A) is irrelevant to the argument.
(A) could only weaken the argument by suggesting that false
attributions aren’t likely to fool anyone. (C) says that humans need exposure to classical music early if
they are to prefer it later, but so what? We don’t know whether
(B) says that the major artists got lots of help, which could
classical music is one of the “higher faculties” and (C) doesn’t
explain why they were so prolific. So (B) as well could only
present us with a case of a person being aware of the higher
weaken the argument.
pleasure but choosing the baser pleasure.
(C) weakens the argument by explaining why paintings by
minor artists are relatively scarce.
(D) , (E) We don’t know whether those athletes have been (D) implies that electronic devices have the range to interfere
turned on to the higher pleasures or what pleasures they with navigational equipment, which in turn strengthens the
prefer more, so (D) and (E) don’t matter. The notion of “serious argument.
athletes” as presented in these choices is too ambiguous to (E) , if anything, would weaken the argument, and thus is
have any real effect on the argument here. correct. (E) suggests that the connection between the use
of electronic devices and navigational problems may be a
83. (C) coincidence. If (E) is true, then maybe it’s the low-power
The percentage of 18-year-olds recruited by the armed services circuitry that’s to blame, and not electronic devices.
went up at the same time that the percentage of dropouts
among young people went up. Our author concludes (as 85. (D)
many might) that these two events are connected, but the The correct choice will either strengthen the argument or have
correct answer will suggest otherwise. (C) does this job. If no effect at all.
(C) is true, then the army is increasingly going after high
Hotel and restaurant revenues outpaced sales of passes to
school graduates. In this case, recruitment rates wouldn’t be
a local tourist attraction. Those in charge of the attraction
dependent on recruitment rates for dropouts. There might
speculated that visitors were selling the passes. You don’t
be more dropouts running around in the republic, but (C)
need an MBA to come up with alternative explanations for the
suggests that the army is recruiting elsewhere.
decline in sales.
(A) is tough, but it refers to raw numbers rather than
(A) Competing tourist attractions opening up in the past year
percentages, and there’s its problem. (A) gives only half of
would absolutely explain the decrease. Eliminate.
the story, and tells us nothing about the relationship between
grads and dropouts. From (A), we can come to no conclusion (B) Since people only need to buy one pass per year, more
concerning the percentage of dropouts recruited by the army, frequent visitors would explain why hotel and restaurant
so (A) doesn’t tell us whether the army depends on recruiting revenues spiked while pass sales remained flat. Eliminate.
dropouts. (C) is a clear weakener. An increase in hotel and meal prices
(B) suggests that the army needs at least some graduates, but absolutely explains increased revenue. Eliminate.
it can need those graduates while still depending on dropouts (D) eliminates an alternative explanation for the discrepancy in
for the majority of its recruitment efforts. So (B) doesn’t speak revenues and therefore strengthens the argument. This is the
to the percentage of dropouts, either. correct answer. For the record:
(D) Encouraged? So what? What should we expect the army to (E) A photograph on the pass would make it very difficult
say? “Forget about school, kid! We’ll teach you how to shoot for people to sell their passes, and therefore weakens the
things!” The army can hype the value of school even while it argument.
depends on dropouts, so (D) is no weakener.
(E) The issue here is high school education, not college 86. (B)
education. Sure, you need to complete high school to attend A weakener will explain why, on its own terms, a plan or
college, but (E), if anything, suggests that the applicant pool is proposal is doomed to fail.
better educated in ‘86 than ‘80, so (E) is no weakener. The author proposes a novel way of detecting life on other
planets by looking for methane. Why would this work? Well,
84. (E) methane degrades in an atmosphere unless it is replenished
Here’s a stimulus that business travelers can relate to. The by life processes. But what if some living beings do not
question is “do electronic devices interfere with the low-power produce methane? Then this test would give a false negative in
circuitry of navigation systems?” Four of the choices will cases where a planet is populated by non-methane producing
strengthen the argument, and one won’t. life forms. And that’s just what (B) says.
(A) suggests that in at least this one case, circuitry was (A) There may be other ways to detect life, but that says
affected by the laptop. So (A) is a strengthener. nothing about the effectiveness of this particular plan.
(B) implies that all electronic devices interfere with circuitry, (C) The second line of the stimulus says that looking for
which certainly strengthens the argument. methane “would be” the most reliable means of detecting
(C) doesn’t prove the argument, but it does strengthen it. life. In other words, the author argues that the detection of
If navigational equipment only has problems when other methane would work in theory. Just because the technique is
electronic devices are on board, then it’s more reasonable to beyond current technology doesn’t mean it will never work.
infer a connection. (D) Given (D), the plan could still work in principle. How much
methane is in the atmosphere is a question of detection. While
this may present a practical concern, it doesn’t invalidate the Is there any other way to explain this trend? Well, if as (E)
logic of the plan. suggests, teachers with this special personality type are
(E) Just because the earth is the only planet known to have leaving the profession at a faster rate than the average
methane doesn’t have any bearing on the potential efficacy teacher, then that would explain the decline. This absolutely
of the method. After all, Earth is the only planet known to weakens the original explanation.
have life. (A) brings in the medical profession. Only the teaching
profession is relevant.
87. (C) (B) ’s connection to teachers is tenuous. Even if we assume
Invalidate the assumption of an argument and you’ve students pursuing a degree in education will become teachers,
seriously weakened it. the 5 percent only seems to confirm the argument. But really,
The representative’s argument just doesn’t put our mind this is outside the scope of the argument.
at ease. Even though federal law mandates the testing of (C) “Students of teachers” is completely irrelevant, as it’s
chemicals, leaking chemicals from a “long-established” dump never mentioned in the stimulus.
could still be a problem. Given (C), chemicals could have been (D) Being liked by teachers has no clear relationship with
placed at the dump site long before the law took effect. actually becoming a teacher.
(A) Pharmaceuticals were only mentioned by way of analogy,
and therefore are outside of the argument’s scope. 90. (C)
(B) would be cause for alarm if the leakage is hazardous, but The stimulus begins with “it is often said that beauty is
according to the representative the law ensures that it’s not. subjective,” so we can predict that the author will argue that
(D) , if anything, would help the argument by further allaying the opposite is true. Generally, GMAT authors mention that
people’s fears. a view is widespread only when they want to disagree with
it. So, this author thinks that there is an objective standard
(E) Silt from a construction company has nothing to do with a
of beauty, one that doesn’t depend on people’s opinions or
leaking chemical dump.
preferences. In support of this claim, the author points out
that the works of art considered beautiful in earlier cultures
88. (B)
isn’t that different from our own. But so what? Even if people
New evidence consistent with a theory will strengthen that tend to come to the same judgments, that doesn’t show
theory. that people who come to different conclusions must be
The argument here is simple: the psychologist posits that one wrong. From here, a precise pre-phrase would be tough, but
first must learn to copy curves before one can copy angles. (B) (C) should jump out when you see it. If (C) is true, and we
provides support for this theory: if all children who can copy come to the judgments we have in large part because of the
angles can also copy curves, then it seems that curve copying standards of earlier cultures, then our judgments sound less
is indeed connected to angle copying. independent, less objective. So (C) weakens the argument
(A) Straight lines are never discussed. by undercutting the evidence: Sure, we may all have the
same standards of beauty, but if we didn’t come to them
(C) introduces the concept of discriminating angles. Only
independently, then they seem less universal.
the skill of copying angles is relevant to the psychologist’s
hypothesis. (A) only mentions some artists, and we don’t know whether
those artists produce anything that anyone considers
(D) 180. If some children can copy angles but not copy curves,
beautiful, so (A) has no effect at all.
then the idea that curve copying is a necessary skill for angle
copying is seriously weakened. (B) and (D) are outside the scope. The importance of art is one
step removed from the issue of what constitutes a beautiful
(E) If young children automatically possess the ability to copy
work of art.
angles, then copying curves isn’t a prerequisite skill necessary
for copying angles. This weakens the argument. (E) Knowing which people tend to own art doesn’t tell us
anything about the qualities that the culture in general found
89. (E) beautiful.
When asked to weaken an explanation of a trend or statistic,
91. (E)
look for an alternative explanation.
We’re looking for the choice that would give us the most
Secondary school children liked a character type
help in evaluating the argument, so the best approach is to
underrepresented in secondary school teachers. The author’s
attack the stimulus on the lookout for any ambiguities that
conclusion? These people are not entering the profession.
could use clearing up. And ambiguities abound here. The
action of the do-gooders is supposedly undermining their graduates find work in journalism must be that the school has
own cause, because while they may slow the woodcutters a valuable program. Is there another possible explanation
from damaging the monarch’s habitat, they themselves for the placement rate? (A) offers a good one: over half of
are wreaking havoc on shrubs necessary for the survival of the school’s students had already held jobs in journalism
monarchs that fall out of the trees. Lots of questions should before enrolling in the program. If that’s true, then the link
jump out at you: Does the good done from slowing down the between the placement rate and the value of the program is
woodcutters outweigh the bad done by the trampling tourists? significantly weakened.
How much woodcutting is the group’s action deterring? Is it (B) goes beyond the scope of the stimulus. Even if some
worth the damage to the shrubs? How many klutzy monarchs editors do not regard journalism school as necessary, others
fall out of the trees anyway?—one would expect they’d be might; and even the ones who regard it as unnecessary
pretty good at just sitting in their chosen hibernation spot. may still find a graduate from a good school preferable to
The latter question is particularly relevant to the conclusion someone who has not attended school. The bottom line: what
that the groups themselves are actually endangering the some newspaper editors think has no effect on the logical
monarch population—if relatively few monarchs fall out of chain here.
the trees, then the conclusion would seem to be a bit of an
(C) is completely irrelevant. What does the number of cities
overstatement. Scanning with this specific issue in mind
with more than one major paper have to do with the quality of
should have turned you onto (E), and allowed you to quickly the Hyperion journalism program? Nothing.
cut past the other choices.
(D) fails to weaken the link between the placement rate
Once we remember that the conclusion, signaled by and the value of the program. If the Hyperion program is
“therefore,” is that the tour groups are actually endangering comparable in quality and content to those of peer schools,
the monarch population, it’s easier to see why the other then the placement rate at Hyperion may be comparable to the
choices are irrelevant. Many of them center on the effects of placement rate at other schools and thus still a direct function
the woodcutters, but that’s a different issue. We only care of quality, as the dean maintains.
about the effects of the tour groups.
(E) If anything, (E) suggests that the program may be more
(A) , (B) The amounts referred to in these choices really have selective than in the past, which could only support the notion
no bearing on the specific argument just cited: What if we that the program is considered useful. On the other hand, this
did know how much more land was available to monarchs, may mean nothing, but it certainly doesn’t weaken the dean’s
(A)? That still wouldn’t change the fact that the trampling argument.
tourists are damaging the shrubs, and we still don’t know just
how crucial to the monarch’s survival these shrubs are. The 93. (A)
amount in (B) is even further removed from the argument’s
Isn’t technology wonderful? The author wants to foil
main conclusion: This information tells us something about
counterfeiters by printing paper currency with a special
the extent of the woodcutter threat, but tells us nothing about
ink. The upside of the plan is that, with the new ink, nearly
the main issue: whether trampling the bushes endangers the
everyone could easily recognize counterfeit bills, whereas with
monarchs.
the current microprinting method, only experts can detect
(C) Plant life not necessary to monarch survival is outside counterfeits. The downside is that the new method is more
the scope. All the information in the world on the effect of expensive. At this point, we’re left wondering if the added cost
the tourists on these plants wouldn’t help us evaluate this is worth the added benefit. But what if the cost of relying on
argument. microprinting is really much higher than ordinary bookkeeping
(D) is even further removed from (B), which at least relegated indicates it is? We still don’t know, after reading (A), the
its amount to the forests in question, those used by the absolute relationship between the cost of each method, but
monarchs for hibernation. (D), on the other hand, deems to we DO know that if we consider the cost of paying expert
tell us the percentage that these trees represent of all the trees counterfeit detectors, the costs are closer together than
hacked down by the cutters. Who cares? What does this have we originally thought. Thus, by shrinking the downside, (A)
to do with the effects of the rampaging tourists? The question supports the author’s argument.
is still whether their actions are putting the monarchs in
(B) If (B) is true, then the author has hatched a terrible plan. It
danger, and only (E) would get us closer to the answer to that will be obvious to most people that currency changes colors in
question.
ordinary light, so the technique cannot be effective.
(C) The fact that printing currency with the special ink seems
92. (A)
to be more complicated can’t help the author’s argument—if
Do you see the cause-and-effect element buried here? anything, it may weaken the argument. But the real issue is
The dean argues that the reason why 65% of the school’s
still cost, so if you saw the number of steps involved in the 95. (D)
processes as being largely irrelevant, and crossed off (C) for The information in this one is quite precise, so we must read
that reason, then your reasoning was right on track. the second and third sentences closely: rhododendron leaves
(D) provides useless background information. This choice curl when the temperature of the air around them is below
tells us nothing about modern counterfeiting and gives no 0°C. Similarly, we know that mature crocus blossoms open
additional reason for changing detection methods. in temperatures above 2°C. So, the observation of uncurled
(E) So, what if many criminals don’t have access to advanced rhododendron leaves and unopened crocus blossoms should
photocopiers? All that means is that they’re not the criminals mean that the temperature is not below 0 and not above 2; in
producing the counterfeit bills that circulate widely. But other words, between 0 and 2. Then there’s the thermometer
someone’s producing these phonies. This information neither reading 1°C—halfway between the two key temperatures. So,
supports nor detracts from the author’s argument. the thermometer reading must be accurate to within a degree,
right? Perhaps not! Did you spot the assumption here? The
94. (E) author assumes that the two plants are experiencing the same
temperature; only then would the thermometer observation
Next up is a strengthener, so we need to identify the evidence
support the conclusion in the final sentence. You may not
and conclusion and figure out what would link them. The
have spotted this assumption, or been able to prephrase a
Keyword “Thus” indicates the conclusion: the specific
weakener, but hopefully the issue raised in (D) struck you as
ultraviolet light patterns reflected by Glomosus spiderwebs
relevant. If, as (D) points out, the air temperature around each
attract insects. Why? Because those insects can sense the
of the two types of plant is likely to differ by more than 2°C,
reflections, and they are attracted to the spiderwebs. Sounds
then an observation of the plants can’t help us pinpoint the
plausible so far, and it’s at least possible that insects are
accuracy of the thermometer within a degree. If (D) is true,
attracted by the UV patterns, but could there be another
it’s possible that the temperature around the rhododendron
explanation? As far as we know, some other characteristic,
leaves is 3°C, and that around the crocus plant is minus 3°C.
such as smell, could be responsible for the attraction. So, the
The observations described would be consistent with these
correct answer will probably rule out alternatives, or present
temperatures, and yet the thermometer would not necessarily
additional evidence that the UV pattern, and not something
be accurate to within 1 degree.
else, is the overriding factor here. It’s difficult to pre-phrase
further than this, so we have no option but to test the choices. (A) is irrelevant to the argument. The observation involves
If (E) were the case, and most flies given a choice between rhododendron leaves, not rhododendron blossoms.
two webs that differ only in their capacity for UV reflection (B) and (C) are also irrelevant. People’s distaste for cold
choose the one that reflects UV, then it sure sounds like weather or the conditions in which these plants thrive have
UV is responsible for the attraction. So (E) strengthens the nothing to do with the observations here or the conclusion
argument. drawn from them. There’s nothing in these two choices that
(A), (B) and (C) indicate that not all spiderwebs (or spider even approaches an issue relevant to the conclusion about the
silks) reflect UV light, but that doesn’t tell us anything thermometer’s accuracy.
about what attracts insects, so these choices don’t help. If (E) Was this type of thermometer used to take our
anything, (A) and (C) suggest that some other property would measurement? We don’t know. What constitutes the
be required to draw insects to webs that lack the reflective “moderate” temperature range? We don’t know. What does
capacity, so those choices can’t improve the likelihood that “extremely accurate” mean? We don’t know. The issue
reflecting UV is the only explanation. here is not the thermometer, it’s the observations made in
(D) could only suggest that something else is responsible for determining the thermometer’s accuracy.
the attraction. In this scenario, both webs reflect UV, so the
ability to reflect UV isn’t tested here. So (D) doesn’t rule out 96. (A)
alternative causes for the attraction, such as smell. The fruit We need to find the question most relevant to an evaluation of
flies might have chosen the Glomosus web because of the the author’s claim that there is some evidence that the rate of
specific pattern of reflection there, but this seems unlikely, extinctions is increasing. The author cited some data on North
since the webs have similar patterns. If the fact that they American fishes that seems to support the argument. But
have similar patterns means that they reflect UV light in the hold on, that data is only about fishes, not animal species in
same way, then (D) would actually weaken the argument, general. If the plight of these fishes is a good indicator of the
as we would expect a more equal distribution under these plight of animals in general, then the author has a strong case.
circumstances if the author’s theory is correct. Either way, (D) The fishes seem to be dropping off at an increased rate, and
is no strengthener. if animals in general do likewise, then they’re in big trouble.
But if North American fishes aren’t representative of animals
in general (in this respect), then the fish extinctions, however that the author believes ties in with the earliest possible origin
regrettable, don’t provide any support for the claim that there of the play.
will be an increase in the number of extinctions in animal (D) King Henry’s personal history outside of the year he died
species. (A) addresses this relevant issue; the answer to the is irrelevant. Whether he was in power every year during his
question in (A) would help to evaluate the argument. lifetime doesn’t matter—nothing in (D) is inconsistent with the
(B) Whether the extinct populations were large or small fact that the king died in 1471, nor does this info about the
doesn’t matter, since this does nothing to connect fish king impact on the relation between the year of his death and
extinctions to animal extinctions in general. The numbers the supposed time frame for the origin of the play.
by themselves tell us nothing about why they died or what
connection there might be to dangers to animal species in 98. (C)
general. As always, we need to read the question stem carefully. We’re
(C) is outside the scope. The relevant issue is the extinction, looking for a category of trees whose failure to show a drop
not the “origin of species” (no offense to Darwin). in fruit loss would support the official’s claim. In other words,
(D) is connected to the issue of the well-being of a species, for each choice, we need to ask what would happen to the
but this argument is limited to what the rate of extinction of a official’s argument if the trees in that choice did not show a
certain species says about the rate of extinctions in general. drop in fruit loss. What would the official like to point to in
order to demonstrate that the new pesticide is more effective?
(E) Commercial importance? What does this have to do with
Well, the official wouldn’t want to point to any insect-infested
the connection between North American fishes and animal
tree treated with the new pesticide. Instead, the official would
species in general?
like to point to a tree treated with the old pesticide but not the
new pesticide, (C). That would cast doubt on the effectiveness
97. (E)
of the old pesticide, but leave the new pesticide blameless.
This Weaken the Argument question revolves around a Such trees that did not show a reduction in fruit losses would
conclusion that Mankind must have been written between thus support the official’s conclusion.
certain dates: not before 1431 (because a coin mentioned
(A) Peach trees?
in the play was not circulated until 1431) and not after
1471 (because Henry VI is mentioned as living in the play’s (B) Peach trees again?
dedication, and he died in 1471). A weakener would, (D) If neither pesticide was used, how could we tell which is
ostensibly, open up the possibility that the play was written more effective?
before 1431 or after 1471. And the only way to do this would (E) 180; this would put the blame on the new pesticide and
be to somehow cast doubt on the relevance of the factors leave the old pesticide blameless. However, there’s another
relating to those dates—the coins or the King’s death. (E) problem with (E): the relevance of the results in other districts
provides just such a weakener—a scenario that, if true, is highly questionable.
damages the validity of using the circulation date of the rose
noble as an accurate marker for the play’s conception. If the 99. (D)
design of the coin was viewed and talked about before 1431,
The first purpose of reform is to increase the sum total
it’s possible that it could have been included in the play even
of happiness. The proposed reform would increase the
before it was circulated. If (E) is true, it’s reasonable to believe
happiness of the MP’s constituents, so the MP concludes that
that the play could have been written before 1431, and the
it therefore meets the requirements of “good social reform.”
argument would therefore be weakened.
Oh really? The sum total of happiness includes everyone’s
(A) is irrelevant to the argument. The fact that the play was happiness, and everyone’s potential unhappiness. What if, as
performed in 1480 does not mean that it was written that late, (D) suggests, that reform could cause so much unhappiness
so (A) poses no effective challenge to the author’s conclusion. for others that the benefit to the MP’s constituents would be
(B) is outside the scope. An earlier-minted coin might have outweighed? In that case, the sum total of happiness would
been mentioned, but this has no bearing on the connection not increase, and the reform in question may not in fact be
between the rose noble coin and the play’s presumed date of “good social reform” as the MP suggests.
origin. If the rose noble wasn’t heard of until 1431, it doesn’t (A) So, what if different things make different people
matter what other coins were around—the play could not have happy? We know that the proposed reform will make the
been written until the author was aware of the rose noble as MP’s constituents happier, and that’s all the “first
well. purpose of reform” requires.
(C) The date at which the rose noble went out of circulation is (B) The reform would still be a good idea if it had a positive
not relevant here—it’s the date when the coin first appeared effect on a few even if it failed to make others happier. Its
benefits may not be widespread, but such reform would are cheaper than installing the “smart highway” systems,
still increase the sum total of happiness if a few were made Eva’s potential counter would be defeated.
happier while the rest remained neutral. (A) doesn’t address Eva’s point concerning radio stations,
(C) 180; if the reform affects only the MP’s constituents and and ignores Eva’s description of the benefits of the “smart
makes them happy, then adopting the reform accords with the highway” system. She doesn’t claim that it controls the
first principle and the MP’s assertion that the reform is good weather; she only claims that it will allow motorists to adapt
social reform would be validated. better to potential traffic problems.
(E) The popular support for reform is outside the scope. (C) and (E) weaken Luis’s argument, and strengthen Eva’s
argument. If the “smart highway” system helps in preventing
100. (B) congestion (C) or eliminating bottlenecks (E), then it would
Curve ball: the argument starts off with one argument be a marked improvement over the current system of radio
(sentences 1 and 2: we must build a new post office because reports.
the old one can’t be expanded), but moves past that first (D) The privacy of drivers? Who cares?
decision to another, more problematic one: where can the
building be built cheaper? The conclusion that the outskirts 102. (B)
site is “clearly” cheaper is the one we seek to undermine. Most categorical assertions of cause & effect are relatively
Sure, land is cheaper on the outskirts than in town. But what easy to weaken, and this one is no exception. Note the
(you may well have asked yourself) about other factors—other considerable distance between the main evidence (that THC,
issues that might impact the cost? If (B) is true, then there’s a marijuana ingredient, can cause cancer by inactivating
a major cost of the outskirts site, building a parking lot, that bodily viruses) and the conclusion (that marijuana can cause
wouldn’t be incurred in town. Notice that (B) doesn’t prove cancer). Weakening this argument is just a matter of breaking
that the center of town would be the cheaper site. It simply this tenuous connection, and with any luck (B)’s assertion that
casts doubt on the certainty of the author’s judgment that other marijuana ingredients can knock out the effects of THC
building on the outskirts would be cheaper. Given (B), we’re jumped out at you readily.
left with a need for more evidence in order to choose between (A) at best strengthens the argument, by reinforcing the results
the center location with its land costs, and the outskirts site of the experiments cited.
with its parking lot costs. And the very need for more evidence
(C) , as well, could only strengthen the argument. If THC
proves that the argument has been undermined.
diminishes the body’s ability to fight viruses linked to cancer,
(A) Even assuming that that new building code would affect then perhaps THC enhances the effects of cancers, but that’s
the cost of construction at all, let alone in a major way, this not the same as causing cancer.
code inferably applies to both sites and hence cannot affect
(D) The positive effect of a possible modification of THC speaks
the site comparison.
not at all to whether marijuana causes cancer. (And that effect
(C), (D) Running extra buses to the outskirts site (C), or is even further removed in that it relates to herpes, not cancer
delivering more mail (D), would likely cost the city a few bucks. itself.)
But they’re not construction bucks—they’re not part of the (E) That marijuana might alleviate some of the pain of cancer
building costs—and the argument is specifically about which therapy doesn’t mean that marijuana doesn’t cause, or
site would be cheaper to build. Therefore, neither choice has contribute to, the disease.
any effect on the logic whatsoever.
(E), if anything, strengthens the logic. Adding yet another cost 103. (D)
(however much it may cost to work nights and weekends) to Don’t let the double negative (probably not true … caused by
the city center site makes the outskirts site seem that much the inability…) in the beginning of the sentence here fool you.
cheaper in comparison. Evidently, some may feel that the inability to tolerate some
ingredients in cow’s milk may cause colic in infants, but the
101. (B) author says that this is “probably not true.” In other words,
Luis doesn’t see why a “smart highway” system would be any the author is defending cow’s milk. But on what basis? The
better, since radio traffic reports are already available. Now evidence, signaled nicely by “since,” is that colic symptoms
we’re asked for a strengthener for Luis’s argument assuming show up often in infants fed only on breast milk. Since the
that Eva responded that current radio reports don’t provide author is defending cow’s milk, one way to weaken the
enough information. Well, current reports may be inadequate, argument would be to show that cow’s milk is in some way
but if (B) were true, and an “all traffic” radio station gave associated with infant colic. Maybe you were able to prephrase
continuous, in-depth traffic information, and those stations a general answer: “Cow’s milk may still play a part somehow in
infant colic, even in those infants fed breast milk exclusively.” Thus, it would not be true that garbage dumps do not harm
This notion is enough to lead us to (D): when the mothers wildlife.
eliminate all forms of cow’s milk from their diets, their infants’ (C) Dangerously high cholesterol is indicative of harm, so (C)
colic symptoms quickly disappear. This allows us to infer that also weakens the argument.
cow’s milk, contrary to the author’s argument, may still play a
(E) A higher rate of birth defects since the landfills opened
part in infant colic by way of transmission through the mother.
indicates that garbage dumps harm wildlife.
Does this mean that cow’s milk definitely causes all infant
colic? No, but it seriously weakens the author’s conclusion
105. (D)
that cow’s milk probably doesn’t cause colic.
The right answer for a Strengthen question will not
(A) completely ignores the main issue by not revealing the type
necessarily make the argument iron-clad. A consistent piece
of milk these infants were fed.
of evidence can be a strengthener.
(B) The argument discusses the causes of infant colic. How
As proof of the hypothesis that people prefer music, they
the condition develops or whether it disappears altogether
have heard on an earlier occasion to music they are hearing
at some point are issues that are outside the scope of the
for the first time, the author notes an experiment where 100
argument.
people listened to a piece of music in the morning. Later that
(C) , if anything, only helps the author’s argument by day, the people listened to the music again, as well as a
showing that most infants raised on cow’s milk exclusively second piece of music. A strong majority preferred the
did not show signs of colic. piece they’d heard twice.
(E) is too vague to weaken the argument because it doesn’t Answer choice (D) offers a complementary experiment: the
show the likelihood of a direct connection between cow’s same procedure was followed, except this time, the second
milk and infant colic. For all we know, the infant formulas piece was played twice. A substantial majority of the listeners
mentioned need not even contain cow’s milk. (E) may offer preferred the second piece (that they’d heard twice over the
an indirect link through an entirely different mechanism—the course of the day) in the new experiment. Neither experiment
development of a mature vs. immature digestive system—but is particularly strong evidence to support the conclusion (we
the effects of this, and its relation to the cow’s milk/colic don’t know whether the music pieces were equally attractive,
connection, are ambiguous at best. or whether the listeners for each experiment were from a
comparable pool of research subjects, for example), but the
104. (D) same strong preference for the piece that was heard twice
The credited response to this type of question is the during the course of the day, regardless of which piece was the
answer choice that either has no effect on the argument or one that was played twice, is another piece of evidence that
strengthens the argument. The four incorrect choices will makes the conclusion somewhat more plausible.
all weaken the argument. The author’s conclusion is that (A) weakens the argument. It describes an experiment where
garbage dumps do not harm wildlife. The evidence is about there was no group preference for the musical piece heard
one particular group of baboons on a reserve in Kenya and twice.
how they compare favorably to other baboons on the reserve
(B) suggests that the piece called “Study I” was inherently
who do not eat from the garbage dumps. Your best strategy for
more likable than “Study II,” and weakens the argument.
this type of question is process of elimination—prephrasing
an answer would be nearly impossible. We’re told in choice (C) The inclusion of professional music critics might give a
(D) that the hyenas live near the landfills, but nothing directly reason other than that proposed by the hypothesis to explain
about the effect the landfills have on the hyenas. We’re also the group’s preference, which would weaken the argument.
told, however, that the population of hyenas has doubled! If Moreover, all we know is that at least one (“some”) of the
(D) is true, it doesn’t seem that the landfill has had a negative listeners was a professional music critic. Unless a significant
effect. Thus, (D) does not weaken the argument. number of the listeners were professional music critics,
this piece of information has no effect on the weight of the
(A) provides an alternative explanation for the differences
argument.
between the baboons that eat garbage and those that don’t.
(A) casts doubt on the evidence by making it possible that the (E) Another weakener—if a substantial number of people who
garbage eating baboons live longer anyway (because they are listened to “Study I” in the morning were not able to hear well,
a different species) and therefore we don’t know if the garbage they would not necessarily be familiar with the piece when it
has a harmful effect. was played for them a second time.
(B) If the baboons who feed on the garbage have a lower life
expectancy, they are certainly being harmed in some way.
106. (B) per sedentary—i.e., not exercising, not active—citizen. The
An initial study of the question stem reveals your need to evidence, however, never brings up the “voluntary” nature of
decide what kind of statement would be countered by the sedentary living, and therein lies the argument’s flaw. If (C)
given evidence, which basically states that two different is true, then in fact the cause-and-effect to which the author
categories of infant have similar “precursors of speaking”: points is reversed. It’s not that sitting around raises the cost
hearing kids with hearing parents babble audibly, while deaf to society; it’s that the illnesses require one to sit around; the
kids with deaf parents’ “babble” in signs. Having noticed that, lack of exercise is hardly voluntary, and the costs are run up by
you probably could have prephrased nothing more complex the greater costs of care.
than Hearing and deaf kids develop speech differently and Any choice, like (A), that accuses people of voluntarily
come up with (B). The author’s argument—that both categories failing to exercise tends to strengthen this argument rather
of infant start to develop speech in much the same way— than weakening it. (B) seems to cast blame on doctors for
certainly does counter any claim that a characteristic allegedly not discussing exercise with patients, but that’s hardly a
necessary for language development is one that only hearing compelling argument for letting sedentary people off the hook.
kids possess. According to the author, the deaf kids don’t (D) deals with those who do exercise and (E) with the benefits
possess a mature vocal tract and are developing language they enjoy—but they’re not the author’s target.
mighty well, thank you.
(A)’s topic is “What are the simplest words in a language?” 109. (B)
something our author never takes up. Also, who says that The argument for a fifth universal force hinges on fairly
“babbling” = “names of persons or things”? What a bogus technical experiments and terminology, but even we
answer choice—no offense if you picked it, but still…. laypersons can appreciate that the argument will be
(C) All of the evidence is about how babies babble with adults strengthened if it can be proved that the theory doesn’t violate
present, so (C)’s hypothesis is outside our author’s scope. any of the other laws and scientific “truth” we already know of.
So (B) is what we’re looking for.
(D) We don’t get enough information about the babbling of
either hearing or deaf children, to counter (D)’s claim that (A) suggests that the fifth force couldn’t have been located
babies are unaware of how purposive their babbling is. And before 1970, but that doesn’t make the argument for its
anyway, it’s the alleged deaf/hearing distinction, and not existence any more plausible now that the technology is in
babies’ self-awareness, that our author is interested in. place. (C) is a 180: By suggesting a plausible alternative to a
fifth force, it tends to weaken the argument. So would a lack
(E) Given that the question asks “Which would the argument
of precision in the experimentation (D). As for (E), a theory’s
counter?” we might have expected a choice to provide
appearance during a time of discoveries doesn’t make that
the exact opposite, and this is it. (E) is supported by the
theory any more (or less) likely. Each discovery needs to be
stimulus text.
assessed on its own merits.

107. (B)
110. (C)
The hypothesis is “that for long dives, seals also store
Q’s objection to P’s proposal to introduce computers and
oxygenated blood in their spleens.” Since (B) ignores spleens
computer language to schoolkids is the “pointlessness” of
completely, and simply states that oxygen can be directly
teaching skills that will be obsolete when the kids grow up.
stored in muscles, it provides no support for the blood-in-the-
Perhaps you noticed that Q ignores any possibility that skills
spleen hypothesis.
learned by the young might turn out to be useful and relevant
(A) strengthens the hypothesis by showing that spleens later, and that’s what makes (C) correct: The early computer
are used by other organisms to store oxygenated blood. (C) training, suggests (C), will be extremely helpful and hardly
states that the oxygen in the lungs and bloodstream alone is “pointless” when the kids grow up and interact with new
inadequate. This would lend credence to a hypothesis that technology.
more oxygen must be stored somewhere else, like the spleen.
By suggesting that ongoing training is needed in the face of
If the spleen of this species of seal is noticeable larger (D), or
advancing technology, (A) implies that early training may not
contains many more blood vessels (E), the hypotheses that it
be especially relevant and hence is a point that Q might make,
stores oxygenated blood is further strengthened.
rather than a comeback that P might make. No one is arguing
that today’s kids and tomorrow’s adults are “incapable” of
108. (C)
“adaptation,” so (B) wouldn’t be a logical statement by either
“Thus” signals the conclusion: Society is burdened by speaker. (D) makes an analogy to automobile technology, but
people’s voluntary choice not to exercise. The evidence argument by analogy is by definition weak; Q could just reply
is the greater cost to society of almost $2,000 additional to (D) that “The car analogy doesn’t hold in this instance;
my argument is undamaged.” One responds effectively to an the board. So even if the improved care explains why the
opponent by dealing with his precise terms, not analogous elderly suffer less disability, it would leave the shrinking-
ones. (E) implies that the training is important in the lower proportion phenomenon untouched. The difference in the
grades because it would be difficult to obtain after graduation, payer of benefits, (B), also has nothing to do with the alleged
but that difficulty doesn’t speak to the efficacy of the training discrepancy. That science extends the lifespan, (C), is nice,
at any age. but so what? That says nothing about East Wendell’s payment
policies. (D), like (A), presents a fact that affects all of East
111. (E) Wendell’s disabled people across the board and hence cannot
The politician’s somewhat bombastic conclusion is that a be what this question is seeking.
nation can’t let its highest tax bracket rise above 30% if it
wants to maintain its values and way of life. It’s based on 113. (A)
a chain of logic that goes like so: High income taxes lead To weaken an argument means to drive a wedge between
to hampered innovation, which leads to a back seat in the evidence and conclusion.
world arms race (!); and strategic disadvantage loses a The critic makes a huge leap that damages his logic. He
nation its world voice. You go into the choices knowing—not asserts that the society-as-body metaphor is pervasive in
suspecting, but knowing—that four of them attack the logic at authoritarian societies, and then leaps to the conclusion that
its heart. And there are so many vulnerabilities here, so many that metaphor is the one that most promotes authoritarianism,
mismatched terms that can lead to problems. makes the populace most accepting of it. The mistaken
(A) attacks the conclusion about 30% directly, by asserting idea that pervasiveness must necessarily equal influence
that the top rate has to hit 45% before it can hamper is attacked by (A). If an alternative metaphor is equally
innovation. (B) attacks the assumption made in the politician’s pervasive, then by the author’s own logic it holds equal claim
very first clause, that something about retaining one’s income to the title of “greatest promoter of authoritarian repression.”
must impact technological innovation. (C) counters one blatant That every society uses metaphor to legitimize its government,
gap in the argument, between sentences 1 and 2—that is, (B), doesn’t affect the conclusion that one type of metaphor
between taking a back seat in the arms race and getting a promotes one type of government. And even if a particular
strategic disadvantage; and (D) counters the other gap, the metaphor is “sometimes” used in nonauthoritarian societies,
one between sentences 2 and 3—that is, between a nation’s (C), it still could promote repression in the societies in which it’s
loss of world voice and its maintenance of values and way of “pervasive.” Likewise, the constant search for new metaphors,
life. Each drives a wedge between all the grandiose claims and (D), could still leave the society-as-body metaphor the
the apocalyptic conclusion. preeminent one for promoting the acceptance of repression.
Only (E) fails to undermine the logic. The stimulus phrase (E) if anything strengthens the argument, by making the
“through historical accident or the foolishness of…political metaphor’s use by more liberal governments a rarity.
leadership” carries with it an implied “whether”; that is,
“no matter how a nation has lost its strategic disadvantage, 114. (C)
whether accidentally or foolishly…” The distinction, in other When arguments deal with percent/number comparisons,
words, is moot. By drawing a distinction between two moot study scope carefully.
terms, (E)’s effect on the logic is nil.
From the fact that single people today spend 50% less of their
income on food than they did 30 years ago, and the fact that
112. (E)
incomes rose over that period, the author draws a conclusion
“The explanation…is at best incomplete” signals a weakener in the form of a comparison: Incomes have risen faster than
(if “at best” it’s incomplete, then “at worst” it’s disproven). food prices. But this is only true if the nature and amount
The author sees a discrepancy in the fact that while each of of food that single people buy has stayed the same, (C). If
us runs a greater risk of disability as we age, the proportion of for instance, they are buying much more food now, or caviar
those receiving disability benefits is reduced with each older instead of corned beef hash, then it is quite possible that food
age group. He explains it by suggesting that the proportion of prices have risen slower, or faster, than incomes.
jobs offering such benefits is a relatively recent phenomenon. Those baffled by arguments involving economic data could
But if (E) is true—if most disability payments stop at age 65— have taken heart from the fact that all four wrong choices
then it would be small wonder that the older one gets, the less deviate markedly from the scope. Amount of food eaten is
likely it is that one receives such payments. (E) in and of itself irrelevant, which knocks out (A) and (B); (B)’s reference to
would explain the so-called discrepancy. “healthier food” is even further afield. Nonfood items, (D), and
(A)’s assertion that disabilities are better treated now than food purchased by families, (E), are in the same way not part
“in the past” affects all of East Wendell’s people across of the author’s equations.
115. (E) is specifically from the eruption of Mount Etna in Sicily. So,
“Strengthen” means “look for a way to tighten the connection we must correct that scope shift, by filling in the author’s
between evidence and conclusion.” assumption: no other, closer volcano could have erupted and
caused the colder weather in China. We would need to know if
The reason for conservationists’ belief that native Australian
this assumption was in fact correct, which leads us to (D).
ringtail opossums are endangered by “non-native predator
species” is that 75% of a group of opossums was killed by just (A) Modern equipment could help us monitor volcanic
such a non-native species, the fox. But there’s a scope shift eruptions today, but it won’t help us figure out what happened
occurring here that you might have caught: The opossums of in 44 BCE.
the evidence were first raised in captivity and then let loose in (B) The background information in the first sentence suggests
the wild, whereas the conclusion deals with native opossums that the ash from volcanic eruptions causes cold temperatures
generally. For the evidence to lead to the conclusion, this for “a year or more,” but this is not given as a minimum
scope shift must be papered over by (E), the assertion that number. We don’t need to know if the colder weather in China
there is no difference in the vulnerability to foxes between lasted that long.
opossums raised wild and opossums raised in captivity. If (C) The argument deals with colder temperatures in China
(E) were proved false, then the “study” would be invalidated that were potentially caused by an eruption in Sicily. Colder
(those opossums killed by foxes would have died because temperatures in Sicily are irrelevant.
they were vulnerable from having been raised in captivity) and
(E) Subsequent eruptions of Mount Etna are outside the scope
we’d be no closer to drawing a general conclusion from it.
of the argument.
The number of non-native vs. native predators, (A), has no
impact on whether the opossum is endangered from predators 117. (B)
or from some other cause. Foxes, (B), appear in the passage
You can strengthen an argument by providing evidence that
only as one example of any number of non-native species
shores up the author’s assumption.
that allegedly threaten the opossum. Whether the two types
of opossum shared the same kind of diet, (C), is irrelevant to It seems the GMAT has conspiracy theories on its mind.
the conclusion about opossums generally being threatened by Hopefully we won’t see a question about the Loch Ness
non-native predators; it’s also irrelevant to the food scarcity Monster next. Topic aside, the information in this question
issue, which is thrown in mostly to distract us. That’s also is actually pretty straightforward, even if the conclusion is
what makes (D) incorrect; the food issue plays no role in the a bit loony. The essayist suspects that there have, in fact,
evidence, and understandably, since the conclusion is about been extraterrestrial spacecraft sighted near Earth. Why?
predators. Because the British government hasn’t been forthcoming with
information about sightings of UFOs. Turning a UFO sighting
116. (D) into a real extraterrestrial spacecraft is a pretty big leap of
logic. It requires the equally large assumption that some
Be on the lookout for variations on a common question type.
of the UFOs the British government won’t talk about were
While this question stem may be unfamiliar, “evaluating the actually extraterrestrial spacecraft. If all of the UFOs were more
support given for the conclusion” is what we do when we are mundane objects, like meteors or large birds, the essayist’s
trying to weaken an argument. We can approach this question entire argument would fall apart. (B) provides some evidence
just like any other Weaken question. The stem even gives us for the author’s assumption: the only possible reason that the
some extra clues to make up for the fact that it’s a little more British government wouldn’t talk about UFOs is if they really
difficult to figure out what we have to do. We know that we’re were extraterrestrial.
looking specifically to weaken the connection between the
(A) actually weakens the essayist’s argument by giving us an
evidence and the conclusion given in the last sentence of the
alternative explanation for the British government’s secrecy,
passage.
making this choice a 180.
Keep a close eye on the scope in that sentence. The opening
(C) is a tempting wrong answer trap for those of us familiar
of the stimulus fills in some background, telling us how
with conspiracy theories. But note that the argument never
volcanic eruptions can affect weather. The evidence then hints
says that UFOs or extraterrestrial spacecraft are “something
that a year of cold weather in China may have been caused by
to hide.” Be careful not to choose an out-of-scope choice that
a volcanic eruption in Sicily in 44 BCE. The conclusion is an if/
sounds close to the terms of the argument.
then: if the weather in China was caused by volcanic ash in
the atmosphere, then the ash from Sicily must have spread (D) is a classic irrelevant comparison. It doesn’t really matter
all the way to China. But there’s a shift in the scope between whom the British government trusts less.
the trigger and the result. The trigger refers to volcanic ash (E) We don’t know whether the government has always tried
in the atmosphere, but the result refers to volcanic ash that to deny the existence of UFOs, or whether they just started
making such denials yesterday, but the timing involved in the that languages have words for elements of their culture that
release of information doesn’t help the essayist’s argument are present (e.g., snow) and don’t have words for elements
that the UFOs were really extraterrestrial. that are not present (e.g., sea). So, to most effectively weaken
this argument, just look for an answer choice that attacks
118. (A) this underlying assumption. (B) does precisely that by saying
You can strengthen an argument by finding an additional that some languages lack words for prominent elements of
piece of evidence that would make the conclusion more likely the environments of their speakers. In other words, you could
to be true. have an ocean (certainly a prominent element) without having
a word for it. If this were true, it would undermine the author’s
The recent proliferation of “me too” drugs (which mimic the
conclusion that the P-I-E people were isolated from the sea
function of existing drugs) is the subject of the editorial in
just because their language contained no word for it. Maybe
this question. The editorialist uses a structure that should be
they just never bothered to get around to making up the word.
familiar from your work on Reading Comprehension: he cites
the beliefs of some unnamed critics, then goes on to refute (A) might have looked tempting if you thought having the word
those beliefs. Here, the critics believe that “me too” drugs do for fish made it more likely that they lived near a sea. But the
not benefit the consumer, because they are redundant. But presence of fish doesn’t necessarily imply a sea. The fish could
the author cites a possible reduction in the price of drugs to have come from rivers or lakes.
conclude that consumers really do benefit from the presence (C) shifts the scope to modern languages, and fails on that
of “me too” drugs. Ultimately, it seems that the editorial account alone.
expanded the definition of what “benefits” means to include (D) The presence of the concept of “heat” tells us nothing
monetary benefits as well as health benefits. Any additional about where the P-I-E people lived. Anyone who knows about
benefit from “me too” drugs would strengthen its conclusion. fire knows about heat, so (D) tells us nothing.
We find an additional benefit of “me too” drugs in (A). If the
(E) tells us they roamed around, but doesn’t tell us where they
“me too” drugs are actually more effective than the drugs they
went. They could have been close to the sea, or not. So (E) is
mimic, then their presence in the market means that there are
no weakener, either.
better drugs available to consumers, a clear benefit.
(B) would actually weaken the argument by suggesting that 120. (E)
more money would be available to produce new, innovative
This particular question is a variation on Strengthening the
drugs if “me too” drugs were not allowed. This would mean
Argument with a strong whiff of Paradox, and you need to
that “me too” drugs actually harm the consumer by taking
understand it up front. You’re asked to find a statement that
money away from potentially helpful research. This choice is a
“would resolve the dispute . . . in favor of one party,” the
180.
problem being that you can’t be sure which party that’s going
(C) The amount of money pharmaceutical companies make to be until you (1) understand the two sides and (2) study
on different classes of drugs doesn’t make a difference to the the choices and assess each one’s impact. The topic of the
consumer. (C) is an irrelevant comparison. debate—prehistoric birds—and its scope (their bloodedness)
(D) is another 180. It takes the argument against “me too” is revealed in the first two sentences, and so are the two
drugs from (B) even further, suggesting that the development disputed views, though we have to read further to understand
of “me too” drugs cuts into the development of innovative each in full.
drugs. (D) also weakens the argument. One view, supported by a recent study, argues that prehistoric
(E) This choice deals exclusively with the profits made by birds were cold-blooded. This view sees the “growth rings” in
pharmaceutical companies, which are irrelevant to the prehistoric birds and says, Hey, those growth rings are present
benefits to consumers. only in cold-blooded creatures. Hence the conclusion that the
birds were cold-blooded.
119. (B) An alternate view, “widely-held,” is that prehistoric birds
Remember, on Weaken questions, the best strategy is were the warm-blooded ancestors of today’s birds. This view
usually to attack any underlying assumption that the author gets support from the second study, which sees the dense
is making. This author is concluding that the P-I-E people blood vessels in the prehistoric birds’ bones and says, Hey,
lived in a cold climate, isolated from the ocean or sea. What these dense blood vessels are evidence of activity. Hence the
evidence does the author use? That the P-I-E language did conclusion that the birds were warm-blooded.
not have a word for “sea,” but did have words for “winter,” This seems to be a real conundrum, with the growth rings
“snow,” and “wolf.” Can you identify an assumption that connoting cold blood, the dense blood vessels connoting
would link the evidence and conclusion? The author assumes warm blood, and no additional evidence at hand to resolve
the dispute—except, of course, the credited choice. We need (C) Given that traces of carbon 14 were found “throughout”
a choice that will favor one side or the other and, in so doing, the rocks and some parts of the rocks were never submerged,
resolve the conflicting evidence. And that’s what (E) does. The some of the carbon 14 had to come from land dwellers.
possibility of dense blood vessels in a cold-blooded species (D) weakens the argument by providing an alternative way for
severs the tentative connection in the second theory between carbon 14 to have ended up in the rocks. This is our answer.
dense blood vessels and warm-blooded creatures. According For the record:
to this theory, dense blood vessels merely suggest a warm-
(E) Independent confirmation of the age of the rocks reinforces
blooded animal, but the appearance of such vessels in what is
the paleontologist’s argument.
definitely a cold-blooded animal blows this connection away
(at least in this case) and therefore strongly supports theory 1
122. (A)
over theory 2.
The question stem asks you to justify the claim that the price
(A) is outside the scope. That other modern warm-bloods
of halibut will increase. In other words, you need to find the
have cold-blooded descendants merely makes it possible that
choice that strengthens the argument made in the stimulus.
the prehistoric birds, if cold-blooded, are nevertheless the
We’re told that since fishermen are not allowed to catch as
ancestors of today’s bird. That’s in line with the conclusion of
much halibut, the supply of the fish will decrease and the
study 1, but proves nothing, because examining the evidence
price will rise. Since the argument uses supply and demand
supporting each study is the only real way to support one over
as its main evidence, you need to think about the price in
the other.
terms of supply and demand. If demand remains constant, a
(B) That cold-blooded species have other physical traits only decreasing supply would lead to a higher price. But if demand
makes us hungrier to know (1) what they are and (2) whether decreases with supply, then the price very well may not rise.
the prehistoric birds had them. It hardly puts the lid on the So, to justify the conclusion regarding the price increase, it
dispute in question. must be assumed that demand will not decrease once the
(C) , if true, would suggest that both theories are for the lower ceiling is imposed, and choice (A) provides this missing
birds (so to speak): Study 1 says that modern warm-blooded piece.
birds descended from cold-blooded birds, as it challenges (B) is too vague: Even if we know that there is a connection,
the widely held view supported by study 2 that modern we don’t know what that connection is, so choice (B) doesn’t
birds descended from cold-blooded birds. (C) says that our help us to support the claim that the price of halibut will rise.
modern birds didn’t descend from prehistoric birds at all,
(C) What does compensating for the loss of halibut with
which suggests that both theories are off the mark.
other fish have to do with the price of halibut? We can’t
(D) Even though, as (D) suggests, some warm-blooded species tell—it means nothing, for all we know, and so (C) is not an
lack dense blood vessels, enough of them still may have them assumption that helps justify the conclusion here.
to render the dense blood vessels of prehistoric birds a sign of
(D) is outside the scope, since the stimulus is concerned only
warm-bloodedness. In other words, despite (D) the evidence
with the demand for halibut and not the demand for other
for warm-bloodedness is unchanged, and we are no closer to
fish. Try the Denial Test: What if the demand for other fish is
resolving this dispute.
unaffected? That doesn’t damage the conclusion here, so (D)
need not be assumed.
121. (D)
(E) places the total demand for halibut into perspective within
Four of the answer choices will strengthen the argument.
the entire fish market, but doesn’t address the potential price
Eliminate them. What’s left is the correct answer.
increase. (E) veers outside the limited scope of halibut supply,
This paleontologist is trying to disabuse people of the notion demand, and price, and thus does not help to justify the claim
that life didn’t exist on land until about a half billion years ago. in the argument.
She cites the fact that traces of carbon 14 have been found
in a 1.2-billion-year-old rock. Since carbon 14 is left by plants 123. (D)
and microbes, she reasons that life had to exist on land earlier
We’re looking to weaken an argument, so naturally the first
than half a billion years ago.
thing to do is to pinpoint the evidence and conclusion. The
(A) offers additional evidence that life existed earlier than half sociologist concludes (signaled by “Therefore”) that to be as
a billion years ago. Eliminate. happy as possible, one should not have a pet. Her support
(B) If the primordial oceans were inhospitable to life, it makes for this conclusion is research that most people with pets are
it that much more likely that life started on land, thereby less happy than most people without pets. The author draws
supporting the argument. from the research the unwarranted assumption that having
pets causes people to be unhappy; remove the cause, and
the greatest possible happiness will ensue. Attacking this unemploy them and leave them with injury compensation
assumption is the key to weakening the argument. The author inadequate to their needs. Once the plant closed down, of
fails to consider the possibility that those people with pets are course, this hesitation would be moot, but it would certainly
made happier by their pets, even though they are generally make an employed person pause before filing.
less happy than those without. It’s possible that if pet owners All of those choices imply that the workers were sitting on
didn’t have their pets, they would be even worse off, as claims for one legitimate reason or another. (E) takes a
opposed to being “as happy as possible.” Choice (D) points different, yet equally plausible, tack: The claims could be
this out, challenging the author’s assumption of causation, for injuries of recent vintage, brought on by the blues of
and thus weakens the argument. impending unemployment. Here again, quite the opposite of
(A) The existence of a few ecstatic pet owners doesn’t counter fraud and greed is suggested.
the research findings, or the conclusion derived from it. The We’re left with (C), which at worst tends to strengthen the
sociologist doesn’t claim that all pet owners are miserable, argument, and at best is irrelevant to the argument at hand.
just that they are worse off than their counterparts who don’t That most injured workers generally file right away either
have pets. throws greater suspicion on those who delay filing, or has
(B) An occasional desire to have pets isn’t relevant because nothing to do with the particular situation of the workers at
the argument is limited to actual ownership. Every person the Baerton plant. Either way, (C) fails to weaken the logic and
without a pet could desire a pet, and the author’s conclusion hence is correct.
regarding the relationship between actual happiness and
actual pet ownership would be just as strong. 125. (C)
(C) The author’s conclusion deals with wanting to be as happy There’s a controversy among music lovers about which
as possible. So, the fact that pet owners are “reasonably” recordings sound better, those played with vacuum-tube amps
happy does not undermine the argument. If (C) is true, it’s still or with solid-state amps. The author’s target is those who
possible that pet owners would be even happier without their argue for the former. Those folks’ sense that vacuum-tube
pets. (C) is simply too vague to have any real effect on the amps sound better must be in their minds (or ears), says the
argument. author, because there’s no measurable difference between
(E) Everyone gets the blues occasionally, right? Simply the two types of amps in terms of quality. Ah, we respond,
because those without pets sometimes feel unhappy doesn’t but what about the unmeasurable differences? This argument
damage the research findings at all (that overall this group is appeals to a lot of scientific jargon and findings, but if there
happier than the group of pet owners), or the conclusion that are relevant issues not subject to scientific analysis—and
the author draws from it. (C) argues that there are—then it would be wrong to say that
the vacuum-tube amp champs are merely imagining that
124. (C) superiority. They are hearing a difference, (C) implies—just not
one that can be confirmed in the lab.
The rather cynical conclusion here is that the sharp increase
in injury claims filed right after the plant closed is explained (A) covers too much ground, by making an assertion about
by the workers’ having filed for undeserved money that people in general when the argument is about music lovers.
would tide them over. It should be a pleasure to weaken this Besides which, (A) would be evidence for the equivalency
one, and we get four opportunities to do so. Any alternative of the two amp types, which in turn supports rather than
explanation as to why the number of claims increased after weakens the author’s point.
the plant shutdown will do, since the author of this letter (B) , (D) In plain English, (B) means that there’s more quality
offers no evidence of the workers’ fraud and greed except that variation when a vacuum-tube amp is used, which if anything
the claims followed the closing. would argue for the superiority of solid-state. And (D) even
(A) is a splendid weakener, in that it specifies that the end more obviously lists superior qualities of solid-state amps.
of one’s employment is a necessary condition for filing many (E) This argument is all about “the best” amps in each
injury claims. That would be the direct antithesis of the category (note the presence of that phrase in both evidence
author’s charges. and conclusion). Since the scope has been narrowed to “the
(B) rakes up past history. If the company had a history of firing best” amps, a tentative assertion about amps in general—
those who filed claims, it’s no wonder that other employees which might be shoddy ones, hence outside the scope of this
held off their own filings until such time as they had no jobs argument—won’t affect matters either way.
to lose.
(D) puts forward another reason why employees might
voluntarily postpone filing a claim, namely that filing would
126. (D) choice most weakened by the critic’s claim, assuming that
It’s Marianne’s view that her humming is involuntary because the answer choice was supported by the claims made by the
she didn’t notice it. But if (D) is true, then she’s left with no health association.
such defense. (D) confirms that Marianne is not in the thrall of The health association notes that certain studies showed that
her humming addiction, and can and should make a change. most volunteers who used to eat meat were able to adopt
(A) In one sense this is outside the scope, in that it deals with a vegetarian diet. The critic’s response points to the flaw of
actions outside of matches in a dispute about humming during representativeness: the volunteers were already interested
chess play. In another sense, it’s a 180 in that it undermines the in trying a vegetarian diet, and despite that interest, many of
chess authorities. Either way it’s a poor answer. them failed in their efforts. The critic’s point? Evidence that
some people were able to switch to a vegetarian diet doesn’t
(B) The stimulus provides no information on amateur chess’s
mean that anybody could do it. Answer choice (E) makes
position on humming, involuntary or otherwise, so we cannot
the claim that most people could succeed in adopting a
apply (B) to this case.
vegetarian diet, if they were told by their doctors to do so. That
(C) seems to think that the issue is whether Marianne is aware conclusion is undermined by evidence that the studies were
of all of her involuntary actions. (C)’s generalization has no not representative.
effect on whether Marianne has a case against this particular
(A) The critic does not dispute the potential health benefits of
order concerning this particular involuntary action.
switching to a vegetarian diet.
(E) As far as we know, disqualification doesn’t hinge on
(B) This is the critic’s point—people who are favorably
Marianne’s distracting some but not all opponents. And
disposed toward trying a vegetarian diet are more likely to
anyway, (E) would get Marianne off the hook, not “undermine
succeed than people who are not already favorably disposed.
her argument.”
(C) The critic is not concerned with the number of people who
127. (E) have adopted a vegetarian diet in recent years—his point is
that the study results are not necessarily representative of the
The objection argues that energy use decisions are best left to
general population.
the market, while the original statement argues that without
government intervention, improvements in using fossil energy (D) The critic does not address the feelings of those who tried
are unlikely. To undermine the objection, therefore, we simply to adopt a vegetarian diet yet failed to do so.
need an answer choice that states that without government
intervention, inefficient products and practices will continue 129. (D)
(that the market, by itself, will not improve efficiency). Choice When an argument is based on the results of an experiment,
(E) supports the notion that the market will not eliminate evidence that the experiment is unrepresentative of actual
inefficiency (strong likelihood inefficient products will still be conditions will weaken the argument.
used because the energy purchaser isn’t buying the product). Based on experiment in which smoke samples from a
(A) states that change won’t occur all at once. It does not forest fire were bottled and then analyzed to reveal high
preclude, however, the possibility that the market could make concentrations of nitrous oxide, industrial scientists (no
good energy use decisions. Thus, it does not undermine the bias there!) hypothesize that much of the nitrous oxide that
objection. contributes to air pollution comes from forest fires. Answer
(B) and (D) are 180 answers. Both hint that the market will, choice (D) suggests that perhaps the process of bottling the
in fact, promote energy efficiency, even without government smoke samples has artificially raised the concentration of
intervention. (B) claims that consumers will become aware of nitrous oxide within those samples, clearly weakening the
inefficiency (implying they might do something about it) and hypothesis.
(D) says that consumers will react to inefficiency. (A) The identification of other potential sources for up to 10
(C) is outside the scope of the argument. The argument deals percent of the nitrous oxide present in the atmosphere says
only with using fossil energy more efficiently. nothing about whether or not forest fires are responsible for
the majority of the nitrous oxide present in the atmosphere.
128. (E) (B) The fact that many of the “pollutants” (not just nitrous
When the question stem is long and/or unusual in its format, oxide) present in our air come from the burning of fossil fuels
slow down and look for Keywords that tell you what to do. (which themselves derive from wholesome organic matter)
says nothing about whether or not forest fires are a major
The most prominent clue in this question stem is the phrase
source of nitrous oxide in the atmosphere.
“call into question,” a typical indicator of a Weaken the
Argument question. Specifically, we are looking for the answer
(C) If anything, the existence of soil bacteria that feed on of net fishing, but to find out how many are killed under
ash and excrete small amounts of nitrous oxide into the current conditions.
atmosphere strengthens the argument by suggesting another
way that forest fires can contribute to the amounts of nitrous 131. (B)
oxide in the air. In this long question stem we’re asked to recognize a situation
(E) The existence of a different way of analyzing the forest in which the program is not likely to achieve its desired
fire smoke has no effect on the argument, unless we also result. In other words, we want to undermine the conclusion.
know the results of that alternative way of analyzing the The program relies on the fishing industry’s incentive for
smoke. We have no way of knowing whether on-site analysis cooperation. Under the program, the fishing industry must
of forest fire smoke would strengthen or weaken the hand over the dead seabirds, which yields the information that
industrial scientists’ hypothesis. the government needs—the total number of dead seabirds.
The fishing industry, in return, receives valuable information
130. (C) about toxin levels in fish. However, if (B) is true, then the
The stem tells us we’ll be looking for a statement that fishing industry can secure the info it wants without turning
strengthens the argument, so the first step, as always, is over all of the dead seabirds, which would stymie the intent of
to break down the argument into evidence, conclusion, the program to provide an accurate count of seabirds killed by
and assumptions. Smack dab in the middle of the net fishing.
stimulus, highlighted by the always reliable structural (A), (C), and (D) All of these choices deal with possible things
signal “therefore,” the author tells us the conclusion: “The the government may find once the seabirds are turned in, and
government should therefore institute a program” to test thus all three of these choices discuss things that are beyond
the toxin level in the fish eaten by the seabirds killed by net the scope of the argument. We’re looking for something that
fishing. The evidence appears in the other two sentences— will affect how many seabirds are turned in—remember that
The fishing industry currently has no incentive to report the the government is concerned with getting an accurate count of
number of seabirds actually killed by net fishing (the first the seabirds that died. In order to undermine the government
sentence), and, if the program is adopted, the industry will program, we need an answer choice that would make it likely
have a reason to turn in birds because the industry will get that the number of seabirds turned in won’t reflect the actual
desirable information in return (the third sentence). In other number of seabirds killed by net fishing. None of the factors
words, under the program, the author envisions a win-win stated in these choices impacts on that issue at all.
situation for the government and the fishing industry. Since (E), like (A), (C), and (D), has no bearing on the number of
the argument is essentially for the creation of the program, seabirds turned in. The only difference is that unlike the other
a strengthener must bolster the need for such a program of wrong choices, the irrelevancy in this one centers on what the
cooperation. (C) does this nicely: if the government can’t government may do in a specific situation, rather than what
reach its objective (an accurate count of seabirds killed by the government may learn about the nature of contamination.
net fishing) without the cooperation of the fishing industry,
a program providing an incentive for the fishing industry to 132. (D)
help out, such as the one outlined, is essential.
Whenever stimuli throw around percent increases or
(A) , if anything, weakens the argument, by working against the decreases, beware: percent increases and decreases are
fishing industry’s incentive to cooperate. (A) makes any data not the same as changes in the actual numbers or amount.
that the government gives to the industry less valuable. Here the author signals the conclusion with “it is clear”:
(B) is irrelevant; it really doesn’t matter what the government what is clear is that people are throwing away just as much
has done in the past regarding tissue sampling. You might plastic or even more than ever before. We know that because
have thought that (B) weakens the argument [similar to (A)], by waste management companies are reporting that an “ever-
reasoning that if (B) were true the government might, through increasing” percentage of the waste they handle is plastic. But
inexperience, have difficulty administering the program— percentages are deceiving; there may, for example, be a higher
However, this reasoning is not supported by the stimulus. In percentage of plastic in the waste, but if the total amount of
any case, it’s far from a strengthener. waste is decreasing, then the actual amount of plastic waste
(D) certainly doesn’t support the idea of the program; the only may have remained stable or even decreased. Choice (D) gets
effect it would have is to help destroy the incentive of the at this: waste handlers are receiving less of other types of
fishing industry to participate. waste; thus, there is less overall waste; thus, the increase in
percent of plastic waste does not necessarily mean that people
(E) brings up a hypothetical situation, and a result of it, that
are throwing out as much as or more plastic waste.
are totally irrelevant to the program itself; the point is not to
find out how many seabirds are killed by a reduced amount
Choice (A) sets up an opposition that doesn’t exist in the of humans 20,000 years ago and (C) seems to support that
stimulus. There’s no difference between plastic incinerated denial.
and that sent to a landfill; it’s all included in the plastic waste (D) merely speaks to the accuracy of the carbon dating, but
“handled” by the management companies. what is at issue are the inferences that can be drawn from it
The point (B) raises—that some plastic isn’t recyclable, and with regard to humanity.
therefore ends up in the garbage—would strengthen rather (E) is certainly a 180. This is in line with the skeptics’ view that
than weaken this argument. this Pennsylvania site’s early carbon sampling is an anomaly,
(C) is irrelevant; whatever people do with plastic at home is and not an effective challenge to earlier estimates of when
not the issue. The question is whether they are putting more of humans appeared in North America.
it in the trash.
Same thing with choice (D). The amount of plastic 134. (B)
manufactured, or even an increase in the amount of plastic The author concludes in the last sentence that people
packaging, isn’t necessarily connected to how that plastic are willing to pay more for the convenience of microwave
ends up in the trash. popcorn—presumably, the faster popping time compared
to conventional popcorn. But once again, we see an author
133. (A) overlooking the possibility of other factors. If, as (B) has it,
Before weakening the skeptics’ suggestion, we need to most people buy microwaveable popcorn because it’s less
know what they’re skeptical about, and it’s the timeline fattening, then it’s not necessarily true that they’re willing to
that has been posited between human activities and the pay more money for the convenience of having their popcorn
carbon dating at each level of the Pennsylvania rock shelter. quickly. They might pay the premium price only to avoid the fat
What’s controversial here is the deepest carbon sample. If in conventional popcorn.
this is a timeline of human activity, then the earliest (A) So what? Even if fewer than 50% of consumers buy
humans must have inhabited the site approximately microwave popcorn, that could still be plenty of people, and
20,000 years ago. the conclusion is stated in terms of “many people.”
It’s that that the skeptics take issue with: since that dating (C) The reason microwaveable popcorn costs so much is
conflicts with what has been believed about the arrival of irrelevant; the point is that many people pay that extra cost,
humans in North America, they suggest that contamination is and the issue to be addressed is why they do so.
to blame, that the carbon simply seeped out of nearby coal
(D) We’ve seen irrelevant distinctions in wrong answer choices;
deposits into the lowest level (and, thus, that the presence of
here we have an irrelevant similarity. But what does this do to
carbon that deep has nothing to do with human activity at all).
weaken the claim in the last sentence? Nothing. (If we were
Now, there is no evidence that carbon contamination occurred; told that conventional popcorn results in five times as many
it’s simply a plausible alternative theory that the skeptics have unpopped kernels compared to microwave popcorn, then we’d
cooked up because they don’t like the one at hand (i.e., that have a possible weakener, but even then, only as long as we
humans lived in North America 20,000 years ago). So, if we assume that consumers don’t like unpopped kernels.)
can render that alternative theory implausible, then we will
(E) tells us that microwaveable popcorn differs from
have shut the skeptics up—and that’s just what (A) does.
conventional popcorn in that it contains additives, but instead
According to (A), if such carbon contamination had of going on to tell us that people want the additives, it merely
occurred, it could only have done so by contaminating the tells us that microwaveable corn is heavier. Is this heaviness
upper levels too, which would fly in the face of the excellent
attractive to consumers? Will they pay more for it? We have
correlation between carbon samples and human activity no idea.
above. (After all, since the uppermost layers have been dated
to the present, they clearly have not been contaminated.) In
135. (A)
other words, (A) says that in the absence of contamination
Watch out for scope shifts in Weaken questions.
evidence up above, it’s totally unlikely that contamination
could have occurred below— which, again, effectively This stimulus documents the strides that have been made
silences the skeptics. against polio in North America—no naturally occurring cases
(B) ’s suggestion of an occasional weakness in the carbon- have popped up in the last few years. The author argues that
dating technique doesn’t necessarily affect this application of we can go even further by switching from OPV to IPV. His
the technique. evidence is the fact that OPV causes about a dozen cases
of polio a year, while IPV causes about half that many. But
(C) at best is irrelevant (human use of coal is not part of either
you may have noticed that “vaccination-caused polio” isn’t
the original theory or that of the skeptics), and at worst is a
the only type of polio around; the first sentence mentioned
180 choice, since the skeptics are trying to deny the presence
“naturally occurring polio.” If you noticed this scope shift, you 137. (E)
probably noticed the author’s assumption: using IPV won’t To weaken a causal allegation, look for the three classic
lead to more cases of naturally occurring polio. (A) contradicts alternatives: a reversal, a coincidence, or an alternative cause.
this assumption. (The third is always the most likely.)
(B) tells us why OPV causes some cases of polio, but it doesn’t The commissioner attributes the reduction in violent crime,
help us weaken the author’s conclusion that IPV would be down 15% last year, to the mandatory sentencing law. On its
better. face those things seem to be arguably separate, but (says
(C) shows just how rarely the problem with OPV crops up, but the chief) no other policy changes were made last year, so
we know from the stimulus that IPV would cause the same that must have been the reason. Ah, but what about previous
problem even more rarely. This choice doesn’t give us any years? If, as (E) says, two years ago it was decided to add
reason to reject IPV. 100 new cops every year, then those 100 additional men and
(D) The vaccination preferences of most countries aren’t a women in blue are just as likely (if not more so) an explanation
factor in the author’s decision—he just wants to prevent a few for the reduction in crime, as the sentencing law.
more polio cases. (A) seems to hint at an alternative cause—improved economic
(E) This choice is tempting. It does give us a reason why IPV conditions—but unless and until we know that conditions in
might not be ideal, but for all we know, OPV could have the the city in question have improved, (A)’s generalization is too
same problem—after all, most vaccines do. Besides, this far removed from this case.
choice doesn’t relate to the author’s assumption. (B) is trying to suggest that the mandatory sentencing law
didn’t change anything. But “unusually harsh” punishment
136. (C) “for some crimes” isn’t as sweeping or across-the-board as
The correct answer to a Weaken question will sometimes look the law as described by the commissioner, so (B) leaves open
as though it is outside the scope. the possibility that the causality did work as he alleges.
Anyone familiar with DDT has heard something like this (C) ’s distinguishing between the reduction in violent
argument before. The author tells us that the practice of crime and overall crime makes no mind, since the
manufacturing and exporting banned pesticides in the U.S. commissioner’s argument only concerns the effect of the
increases the health risk to people in the U.S. and others, law on the former.
because the banned pesticides are used on agricultural (D) Adding a couple of new crimes to the “violent” category
products in other countries which are then imported to the wouldn’t affect the alleged causality, since the mandatory
U.S. But that evidence means that it’s the use of the pesticides sentencing law applies to violent criminals; if anything, (D)
that’s actually dangerous, not just their manufacture. So, the strengthens the causal relation (if commission of those two
author must be assuming that the pesticides wouldn’t be new crimes was similarly affected by the law).
used unless they were manufactured in the U.S. (C) makes this
conclusion a lot less likely—if the pesticides are manufactured 138. (B)
in other countries, then it probably doesn’t matter whether the The conclusion (sentence 1) is that “a great number of”
U.S. manufactures them as well. readers were influenced by 1984, because Orwell’s novel
(A) The “trace amounts of pesticides in the soil” in this choice came in second, after the Bible, in a survey. What could be a
don’t necessarily mean that the use of those pesticides on clearer confusion of number and percentage than this? If the
agricultural products wouldn’t increase the risk to consumers. Bible was cited by—say—999 out of 1,000 readers as their
Even if there’s already a risk from these pesticides, it can still most influential book, the one remaining reader who cited
be increased. 1984 would hardly qualify as “a great number.” The claim can
(B) and (E) The pesticides that aren’t banned for use in the only be assessed if we know more about the numbers in the
U.S. are outside the scope, as is whether or not they are survey, and that’s (B).
banned in other countries. The survey, and the conclusion, deal with the influence of
(D) Even if there is a greater risk to people in other countries, books, so whether those surveyed read a great many books or
the banned pesticides can still increase the risk to U.S. few (A), and whether or not they actually read the books they
consumers. claim to have been influenced by (E), are outside the scope.
Even less relevant is the readership of the column (C). Of the
wrong choices perhaps (D) was most tempting, but the number
of books on the list is less important than the number of
readers per book—which is (B)’s domain.
139. (E) the author, a necessary condition of serious consideration is
“The proposal is pointless” should practically scream “I am that it not be trivial, and since (C) supports that point it’s the
the conclusion,” since such a value judgment cannot stand one that will strengthen the logic. (A) goes too far—there may
without supporting evidence. The proposal is to reduce local be some trivial claims that the author finds interesting, albeit
sewage so as to protect lobsters against gill diseases, and the not worth serious consideration. The practices of most people,
alleged “pointlessness” stems from the brief lobster lifespan, (B), fall outside the scope of the argument, which solely
too brief for the creatures “to be harmed by those diseases.” concerns the author’s view of how the claim is to be taken.
Of course, the proposal gains merit should one prove that a Only one claim is discussed, so more information about “every
positive benefit accrues from preventing lobster gill disease, claim” or “every interpretation,” (D) and (E) respectively,
and (E) does just that. If (E) is true, the removed sewage doesn’t bolster this limited argument.
should result in healthier lobsters who, in turn, will lead to
healthier (or at least less sickly) lobster eaters. 142. (A)
Other sources of harm to lobsters (A) can be addressed in “This conclusion” means that the conclusion is to be found
another proposal, but have nothing to do with the rationality immediately before.
of this one. That the open ocean leads to longer-lived The author seeks to rebut the idea of extending North
crustacea (B) is irrelevant to the health effects of “sewage Americans’ life spans through reduced-calorie diets. He
in the harbor where lobsters are caught.” By arguing that does so by ascribing the longer lives of lab animals to the
breeding success is irrelevant to pollution, (C) if anything lends readjusting effect of a reduced calorie diet, which (he says)
credence to the claim that the sewage removal is pointless. brings those lab animals back to their optimum life span in
And the detection of gill diseases (D) would be irrelevant to the wild. But if (A) is true and North Americans on average
the claim about harmfulness even if one’s everyday knowledge consume more than their optimal number of calories, then
didn’t tell us that people don’t tend to eat the shells but rather perhaps a reduced-calorie diet would have the same beneficial
the lobster meat inside them. effect: precisely the one originally asserted.
By putting the onus on fat, (B) begs the question of the
140. (A) effect that fewer calories might have on life span. That
The evidence for the conclusion that North America’s first there may be worthwhile ideas not based on lab animal
Eurasian settlers probably came from distant Eurasia is a experimentation, (C), doesn’t in and of itself speak to whether
single object: A human-made projectile, dissimilar to any this particular idea has merit. (D) tends to reinforce the
nearby-Eurasian artifact, dates back to the peak of the Ice doctors’ recommendation and hence is a 180 here, while (E)
Age; and Eurasians didn’t settle in North American prior to is a statement that both the author and the doctors would
that time. All well and good. But if that projectile bears no inferably agree upon.
resemblance to anything made anywhere in Eurasia, then
the evidence—the projectile—is wiped out, and we are left 143. (C)
with absolutely no evidence whatsoever as to where North Use the One-Sentence Test to locate the conclusion.
America’s first Eurasians came from. (A) does the job.
“Thus” is ordinarily a reliable signal of the conclusion, but
Of the wrong choices, only (C) would have any chance of being here the first sentence is the one that conveys the author’s
correct because it’s the only one that mentions the projectile… main point: Wood-burning stoves are more dangerous than
and remember, the projectile is the sole piece of evidence. open fireplaces. Why? Because their cooler smoke deposits
But by adding an additional similar piece of evidence, (C) if more dangerous creosote. The assumption being made is that
anything strengthens the logic. (D), too, tends to strengthen creosote deposits are the only, or main, source of indoor fire
the argument because it tends to connect remote Eurasia and danger, but if (C) is true, then we have significant evidence
North America via the artifacts. Of the others, the nomadic of a greater danger source posed by the open fireplaces. At
habits of the ancestors (B) and the relative North American/ the very least we’d then need more evidence to arbitrate the
Eurasian climates (E) are perhaps the easiest to reject on the debate as to which type poses greater danger.
grounds of scope.
We don’t know what proportion of wood-burning stoves are
among the “most efficient” (A); if it’s a small proportion,
141. (C)
then the vast majority of wood-burners is as dangerous as
To strengthen means to solidify the connection between the author says they are. (B) and (E) both deal with use, but
evidence and conclusion. neither one speaks to the wood vs. open comparison. Indeed,
All human discovery and invention is self-expressive, say (B) avoids it while (E) wipes it out. (D) is poor because we
“some people.” The author takes issue with this, because on don’t know whether creosote residue poses anywhere near
a trivial level “anything we do is self-expressive.” Evidently, to
the same threat as creosote itself, and if so, how “large” an (B) says that those species that have protection against the
amount of residue is enough to cast the conclusion in doubt. radiation are not declining. This bolsters the argument that
ozone depletion and the resulting exposure to UV-B radiation
144. (C) are what are causing the decline.
In essence we have to find four ways to weaken the argument (C) also strengthens the argument, because if you are arguing
against the predominant theory, meaning that we have to that A causes B, as the author is, you would expect that
weaken the view of the author. He says that the paintings wherever you saw A (ozone depletion) you would also see B
cannot have depicted the diets of their creators, because of (declining amphibians). And that’s what (C) tells us.
the absence of depictions of sea creatures that would’ve been (D) supports the argument by ruling out an alternative
necessary to the painters’ diets. He doesn’t say what creatures explanation for the declining population—a decrease in the
are depicted in the caves, but (C) does, and by asserting the amphibians’ natural habitat. By ruling out an alternative
predominance of land animals on the cave walls (C) leaves the explanation, (D) supports the author’s argument that the
author’s objection untouched at least, and bolsters it at most. depleted ozone layer is the actual cause.
So (C) is the “non-weakener.”
(E) also rules out a possible alternative explanation. Because
(A) explicitly asserts that the painters ate land animals, so the the decline has been continuous, just as the ozone depletion
absence of pictures of sea animals shouldn’t be troubling. has been, the decline can’t be explained by a sudden one-
(B) addresses the claim that no surviving paintings time catastrophe.
“unambiguously depict” sea creatures, by implying that some
of the depicted animals might well be seen as sea creatures 146. (B)
but for the damage done over time. We have to weaken the medical researcher’s argument, so as
(D) may seem outside the scope, but it addresses the problem usual we have to identify the components of the argument.
of feeding the sailors: if meats could be preserved, then the The conclusion is that adequate prenatal care significantly
need the author sees for ingesting sea creatures would be decreases the risk of low-birth-weight babies. The evidence for
moot. this is that hospital records show that mothers who had
And (E) ties the paintings closer to the “current diets of the received adequate prenatal care were less likely to have low
painters”; those painters ate land animals, so both (A) and (E) birth weight babies. The medical researcher also tells us
remove the author’s sole objection to the predominant theory. that records show that babies born prematurely were more
likely to have low birth weights than on-time babies. This is
145. (A) a classic correlation = causation argument: the author cites
a high correlation between inadequate prenatal care and low
The question stem says that all of the answer choices would
birth weight babies, and then concludes that one must have
strengthen the argument EXCEPT the correct one. Therefore,
caused the other. In most cases the best way to weaken these
the correct answer choice will either weaken the argument
arguments is to find an alternative explanation, or to find a
or have no effect at all. The conclusion of this argument is
situation where the two are not correlated (you see X, but you
helpfully introduced by the Keyword “Thus”: the primary cause
don’t see Y).
of the declining amphibian population is the depletion of
the ozone layer. The evidence is that populations have been (B) tells us that mothers giving birth prematurely are classified
declining at the same time the ozone layer has been depleted. by hospitals as having received inadequate prenatal care
Additionally, the ozone protects against UV-B radiation, when there is no actual record of the care they received. Well,
which can damage the genes of amphibians, who have no from the stimulus we know that babies born prematurely are
natural shields against the radiation. (A), which suggests that more likely to have low birth weight. (B) tells us that in at least
no other type of ozone-blocked radiation damages genes, some of these cases, a mother who gave birth prematurely
would strengthen the notion that UV-B is the primary cause was classified as having received inadequate prenatal care,
of the declining amphibian population. After all, it rules out even though her actual care was unknown. This means the
other causes. But that isn’t the conclusion here. The author’s number of premature births, including those resulting in low-
conclusion is that ozone depletion is the primary cause of the birth-weight babies, that are classified as having inadequate
declining amphibian population. That conclusion is unaffected prenatal care is most likely artificially high. This casts doubt
by (A). Whether the ozone-caused damage to amphibian genes then on the accuracy of the evidence that mothers who are
comes from one type of radiation or many doesn’t matter to recorded as having received adequate prenatal care are at less
the author. He’s only trying to show that it’s ozone depletion risk of giving birth to low weight babies. If (B) is true, then the
and not something else. (A) points the finger solely at UV-B correlation cited in the stimulus is not as strong as the author
radiation, but that doesn’t matter here. believes, and the author’s conclusion is in turn weakened. So
(B) is correct.
(A), (D) The medical researcher merely argues that adequate X, but this could only support the argument. Cats with large
care decreases the risk of low birth weight; so, the medical interstitial nuclei are quite rare, but every single cat with
researcher accepts that there will be exceptions to the general disease X has one. So (D) reinforces the correlation presented
rule. There will be some normal weight babies from mothers in the stimulus.
who had inadequate care, as (A) suggests, and there will be
some low-birth-weight babies from mothers with adequate 148. (A)
care, as (D) indicates. Neither of these choices impacts on Here’s a classic causation question, although it’s a
the medical researcher’s conclusion. If someone says that complicated one. For starters, it’s hard to keep the left and
eating right will help you live longer, they aren’t saying that right straight. Plus, this is an “all/EXCEPT” question. Here
no one with a bad diet lives to be 100, and they aren’t saying we go: Depressed people show less left frontal lobe activity,
that eating right will guarantee that you’ll live for a long time. which means they show more right activity. Good-natured
They’re just saying that a good diet will help. people (and you have to wonder how they measured THAT)
(C) sounds similar to (B) at first, but premature babies aren’t show more left lobe activity. So, we have a correlation between
the issue here. The connection between premature babies mood and lobe activity. But this author concludes, in the
and low birth weight babies does not affect the relationship last sentence, that the relationship is causal. So, what would
between prenatal care and low birth weight, and that’s the weaken an argument that claims that lobe activity causes
argument we want to weaken. changes in mood? Anything that suggests that the causal
(E) strengthens the argument. If adequate prenatal care makes relationship is reversed (i.e., that mood influences lobe
you less likely to give birth prematurely, then that should also activity) or that some other condition influences both would
reduce your chance of giving birth to a low-birth-weight baby. do the job. On to the choices:
(Remember the first piece of evidence tells us that babies born (A) is the one choice that doesn’t weaken the argument. If
prematurely are more likely to have low birth weights.) drugs that combat depression work by increasing left lobe
activity and thus make the patient’s lobe activity more like the
147. (E) lobe activity of happy people, then it sounds as if lobe activity
The argument here is a standard correlation/causation does influence mood in the way the author maintains. So (A) is
argument. Male cats in general have smaller interstitial nuclei a strengthener, which means it isn’t a weakener, which means
than female cats. But male cats that died of the dreaded and it’s correct here.
rare disease X had interstitial nuclei that are as large as those (B) and (E) suggest that one’s mood influences one’s lobe
found in female cats. So, the author concludes that the size activity, which suggests that the author has the causal
of a male cat’s interstitial nuclei determines whether that cat relationship backwards.
can get disease X. Possibly, but the causal relationship could (C) says that disposition is much more complicated than
be the other way around (disease X could cause an increase differences in lobe activity, which makes it harder to believe
in a male cat’s interstitial nucleus), or some other factor that lobe activity is the be-all end-all of moods. If disposition
might influence both. (E) breaks the causal connection. If the varies much more widely than lobe activity, we’d be less likely
hypothalamus (and therefore the interstitial nucleus, which is to believe the author’s theory that lobe activity influences
a subregion of the hypothalamus) has no causal relationship moods.
to Y, and X is just a subtype of Y, then the hypothalamus has
(D) says that some other factor affects both mood and lobe
no causal relationship to X.
activity, which again makes us more doubtful that changes in
(A) is outside the scope. The scope of the conclusion concerns lobe activity affect disposition in the way the author describes.
male cats only. Female cats were brought up only to place the
increase in size of the interstitial nucleus in perspective. 149. (B)
(B) Since we know nothing about Z, the connection here Nowadays, says the author, publishers care more about
makes no difference one way or another. making money than about putting out books of quality.
(C) says that the correlation that’s present among male cats The evidence for this depressing conclusion is the author’s
is also present among female cats. This is no weakener, since observation that fewer excellent yet uncommercial books are
female cats are (technically) outside the scope, but also being published these days. This argument seems to assume
because (C) just builds more of a correlation. There’s still no that the number of quality books written is constant; but if that
evidence of a causal connection. assumption is false, if (B) is true, then it’s just as likely that
(D) plays on the statistic in the stimulus reporting the the decline in overall book quality is to blame for the current
incidence of disease X. (D) points out that cats with large dearth of fine books as is the venality of publishers. When the
interstitial nuclei are just as common as cats with disease author argues that X is the cause of a phenomenon, and you
find a plausible alternative Y that could just as easily be the It’s possible that the reduced effectiveness was caused by
cause, you have successfully weakened that argument. resistance to the pesticide, but it’s up to you to reserve your
(A) The author’s point is that only recently has publishers’ judgment and search for other possibilities. A precise pre-
desire to make a buck compromised their simultaneous, past phrase is tough to formulate here, but you should have hit
desire to publish quality, uncommercial books for their sheer the choices looking for something that would provide some
merit. (A) leaves this point unscathed. other reason why the traps might be less effective over time.
(A) does just that, by posing an alternative explanation. If (A)
(C) The surprising commercial success of some quality
is true, then it looks as though it wasn’t the resistance of the
books in years past is a further indictment of publishers’
pests that was the key factor. Rather, it was their interest in
unwillingness to take a risk, but hardly undermines the
the bait. If pests that don’t like glucose are on the rise, and
author’s attack on those publishers.
the traps lure pests with the glucose, then it’s more likely that
(D) What authors do or do not want is of no relevance to this aversion to glucose, and not resistance to pesticides, is the
argument about the dearth of excellent published books and key issue. Thus, (A) weakens the hypothesis.
what that says about publishers’ motives.
(B) , if anything, strengthens the hypothesis, by pointing
(E) is a salient fact about the economics of the publishing out the strength of the mechanism by which resistance to
trade but has no effect on the allegations about publishers’ pesticides could be handed down over generations.
motives and book quality.
(C) also tends to strengthen the argument. If regular use
seems to make the glucose/pesticide mix less effective, then
150. (E)
the “resistance” theory hasn’t lost any power.
Here’s one more question based on a scope shift. The
(D) says that increased concentration didn’t make any
evidence indicates that the average price paid for a new car
difference, but we don’t know why it didn’t make any
has risen in relation to individual incomes. From this, the
difference. Was it due to resistance towards the higher
author concludes that individuals spend more of their money
concentration, or some other factor? We don’t know, so
on cars. Sounds OK, except that not all car sales are sales to
(D), which doesn’t speak to the issue of what happens over
individuals. Some of them must be sales to organizations,
successive generations, can’t help.
governments, etc. That’s subtle, but that’s the GMAT. So how
would that claim be weakened? By pointing to the sales, the (E) is irrelevant. Who cares whether the glucose is natural or
author has ignored. If (E) is true, and sales to individuals make not? The stimulus gives us nothing implying that the origin of
up a smaller proportion of new car sales now, then it’s the bait makes any difference, so (E) doesn’t matter.
possible that the increase in car prices isn’t reflected in the
spending of individuals. So (E) weakens the argument. 152. (D)

(A) If (A) is true, then we have more multiple-income This question is an all...EXCEPT question, and we need to
households, but (A) wouldn’t change the income of any find the choice that does NOT weaken the argument. This
individual, so (A) doesn’t weaken the argument. must be a pretty lousy argument, since there are at least four
ways to weaken it, so we already know to be quite suspicious
(B) is irrelevant, since the entire argument concerns rates. Had
of the reasoning. The stimulus says that a particular TV
the stimulus introduced a claim concerning the number of car
show’s record for recommending stocks is worse than overall
buyers, (B) would be relevant, but that would be a different
market performance, so no one should ever follow the
question.
recommendations of the so-called “experts” on this show.
(C) The stimulus speaks of car prices increasing relative to Perhaps a reason or two for why this conclusion may be
individual income, so inflation is already taken care of here. unwarranted jumped to mind, but we need to eliminate four
(D) The argument concerns only people who buy cars. Whether weakeners, so the best bet is to go right to the choices after
car buyers are a small or a large proportion of the population a moment’s reflection. The right answer will either have no
doesn’t matter. effect on the argument, or will strengthen it. (D), if anything,
falls into the latter category: If the statistics were verified by
151. (A) several independent sources, this can only support the notion
The question stem tells us that we need to undermine a of the relative inadequacy of the experts’ recommendations. It
hypothesis, so we need to attack the stimulus looking for may not be a great strengthener, but it certainly doesn’t do any
that hypothesis. It appears in the last sentence: the recently harm to the conclusion.
installed traps are less effective because the pests developed (A) If the show’s recs outperformed the market as a whole for
a resistance to the poison in the traps. This hypothesis seems the past year, then it would have been beneficial for investors,
plausible on its face. After all, the traps are the same, so at least last year, to follow the TV show’s advice.
something must be different if the new traps are less effective.
(B) weakens the argument by stating that although the (D) , if true, provides one more possible explanation for
show’s recommendations performed worse than the market the decline in fatalities. If the number of hospital
as a whole, it was still the most effective way of selecting emergency facilities has doubled in the past five years, it’s
investments. Even though the show’s advice didn’t perform very possible that some crash victims may have been
spectacularly, if (B) is true, it’s still the best system around, saved who might otherwise have died before the increase
and the conclusion that no one should follow the show’s in medical facilities.
advice would seem unreasonable.
(C) hurts the argument by showing a flaw in the methodology 154. (B)
of the study. By comparing dividends to change in share price, The fact that there are four weakeners among the five choices
the test is comparing apples to oranges, and without further is a hint that the argument must be pretty shaky. And it is fairly
information, the extreme nature of the conclusion would be sparse: studies indicate that different cultures differ in their
unsupported. moral codes (no big shock there). So, the author concludes
(E) is a weakener because it shows a case in which the show’s that as long as there are different cultures, there are no shared
guests gave advice which did outperform the market. Perhaps values across cultures. That seems a bit rash, doesn’t it?
the recommendations made on the TV show stunk, but if (E) Having different moral codes doesn’t mean that there are no
is true then their advice certainly benefited someone, and shared values, but the author has ruled out that possibility
it would therefore be unfair to conclude that no one should for all eternity (as long as there are different cultures). Four
follow any recommendations made by these “experts.” choices will weaken this claim. The one that doesn’t is (B). If
someday we will all share the same culture, then the author’s
153. (E) claim doesn’t apply. After all, the author only claimed that
as long as there are distinct cultures, there will be no shared
Something interesting has happened in Australia over the
values. But if there are no distinct cultures, then the author
past five years: the number of people of driving age has
is off the hook. Since (B) is talking about a different issue (a
increased, but traffic fatalities are down. What could explain
world where there are no distinct cultures), (B) doesn’t weaken
this? The stimulus concludes that drivers in Australia have
the argument.
become more skillful. Of course, you know by now that there
are always other possibilities, and this question gives us the (A) and (D) both provide reasons to doubt the value of the
opportunity to recognize a plethora of alternatives—four of the studies on which the argument is based, and both therefore
answer choices will discuss other possibilities, and will thus weaken the argument. If anthropologists are biased in favor
weaken the argument. (E) is the only choice that strengthens of differences, (D), and/or not able to adequately investigate
the original argument. If driver education became mandatory values, (A), then their findings of different moral codes are
five years ago, then it’s reasonable to conclude that drivers suspect.
are, overall, more skillful than they were before. (C) and (E) both highlight the gap between having different
(A) A mandatory seat-belt law imposed three years ago offers moral codes and having no shared values. If either (C) or (E) is
an alternative explanation for the decrease in fatalities, at true, then there are shared values, certain general principles,
least for the past three years, and thus weakens the argument that lie behind the different moral codes of distinct cultures.
in the stimulus. If (A) is true, then it’s less likely that fatalities
are down due to an increase in driving skill—perhaps fewer 155. (B)
people are dying because more people are wearing seat belts. The historians’ argument is an interpretation: probably,
(B) Major road repairs begun five years ago offers another they argue, narrow floorboards were a status symbol for the
alternative explanation for the decrease in fatalities: better 19th-century North American wealthy. This conclusion is
roads are safer roads. arrived at through a pair of correlations: bigger houses tended
to correlate better with narrow floorboards than did smaller
(C) If Australians are driving less on average each year,
houses; and owners of bigger houses tended to be richer than
then it’s possible that the overall number of miles driven is
owners of smaller houses.
declining also, even though there are more drivers. Think of
it this way: if 10 Australians drove an average of 10 miles The assumption being made is that there’s something about
apiece last year, then Australians drove a total of 100 miles. the cost of narrow floorboards that was outside the range
This year, if 12 Australians drove, but they averaged only eight of the less wealthy—something that made narrow boards a
miles apiece, then the total was 96 miles. If (C) is true, it’s status symbol. Strengthening that assumption goes a long
reasonable to attribute the decline in fatalities to an overall way towards justifying the conclusion, which is why (B) is
decrease in miles driven, which would weaken the “better what we need. If (B) were false—if narrow floorboards were a
drivers” argument. lot cheaper than wide ones—then the whole idea of narrow
boards as a status symbol for the rich would be absurd. By
removing the possibility that the wider floorboards were more (D) is true, but irrelevant. The argument is about how
expensive, (B) makes the historians’ interpretation more likely. businesses can retain their power as long as possible. We all
(A) compares the survival rate of floorboards from bigger and know that nothing lasts forever, but the philosophical question
smaller houses, which is interesting but irrelevant to what of immortality has no bearing on the argument. Businesses
floorboards meant to the rich back in the 19th century. keeping power as long as they can is the issue.
(C) The number of rooms that needed to be floored wouldn’t (E) makes the mistake of confusing something that is
necessarily have anything to do with what materials were necessary with something that is sufficient. We’re told that
used, or the status associated with them. for a business to retain its power it must act responsibly, but
we are not told that acting responsibly is the only thing that
(D) If anything, (D)’s assertion that narrower floorboards were
the business must do to keep its power. There may be a slew
generally covered with carpet tends to weaken the logic. Why
of other things a business must do to retain its power—stay
would someone hide away a status symbol like that? Isn’t the
competitive, for example, or offer a good product. This choice
whole point of status symbols is that they be conspicuous?
attacks the notion that social responsibility is sufficient to
In any event, (D) never distinguishes one type of house from
retain power, but sufficiency is something the author never
another, so it’s of little use to us either way.
claims.
(E)’s assertion that more big houses than little ones featured
expensive marble floors is, like (D), either irrelevant (the use of 157. (E)
marble needn’t take away from the status of whatever narrow
We are looking for an answer choice that will weaken the
floorboards were in use) or a weakener (wouldn’t it be marble,
spokesperson’s position that there would not be a net
rather than any wood at all, that was the true status symbol?),
spending increase in the province if the tax refund occurred.
depending on how you look at it.
To counter the spokesperson’s claim, the right answer will
demonstrate that the tax refund won’t necessarily be offset
156. (B)
by measures such as increasing other taxes or firing some
The conclusion in this argument is signaled by the word province employees. In other words, it will point out an
“therefore” in the last sentence: a business must act alternative possibility. It was fairly difficult to form a more
responsibly if it wishes to retain its power for as long as specific prephrase, but we hope you recognized it when you
possible. The evidence for this position is summarized in the saw it: (E) explains how the refunded money can be offset by
next-to-last sentence: society ultimately takes power away another method—using existing workers more effectively. In
from those businesses which it thinks are acting irresponsibly. this scenario, the people get a tax refund that they can use to
Did you catch the scope shift? The author doesn’t say that if stimulate economic activity without any new taxes eating away
a business acts irresponsibly, society will strip its power. It at their extra income and with no workers losing their jobs.
says that if society thinks a business is acting badly, it will If increased efficiency would save as much as the tax refund
strip its power. This leaves open the possibility that a business gives out, then the province’s budget would be balanced,
need not act responsibly, but needs only to convince society and all of the problems raised by the spokesperson would be
that it does. What if society doesn’t realize a business is solved.
acting irresponsibly? If society thinks a business is acting
(A) is a 180. In this situation even less money would be spent
responsibly when in fact it’s not, society would not know to
in the province. If taxpayers would spend half their tax refund
reduce the power of that business. Thus, a good PR campaign
outside the area, then the spokesperson’s claim that there
that convinced society that a business was acting responsibly
would be no net gain for the province sounds even more
when it really wasn’t would refute the speaker’s argument that
persuasive.
society will ultimately correct these abuses of power, so (B) is
correct. Society can’t correct what it doesn’t know. (B) is outside the scope. Whether the taxpayers receive the
money in installments or in one lump sum has no bearing on
(A) is an irrelevant comparison. It does not weaken or
how much they would spend.
strengthen this argument to learn that government institutions
are also subject to the Law of Responsibility. This argument is (C) is also outside the scope. The only thing that matters is
concerned with businesses and what causes them to lose their whether the taxpayers will be spending enough money in
power, so what happens to government institutions is outside the province to compensate for the tax refund. Whether the
the scope. measures required to compensate for the refund make the
taxpayers angry is irrelevant to the question of how much they
(C) is also an irrelevant comparison. How fast power erodes
would spend.
for some institutions as compared to others doesn’t impact
whether or not society ultimately reduces the power of (D) This proposal still leaves the province’s budget with a
businesses that act irresponsibly. deficit that must be recouped somehow. Simply using the
$600 million to stimulate the economy rather than as a refund
for the people changes the method of spending but wouldn’t The other four choices are wrong because whether their
counter the spokesperson’s argument. However, the money answer is “yes” or “no,” we’re no closer to being able to judge
is spent, it needs to be compensated for somehow, but (D) the plan.
fails to explain how the province will make up for the (A) ’s issue is irrelevant. The two tax rates need not be
increased spending. compared, since there’s no indication that the proposal’s
aim is for the luxury tax to bring in the same amount of
158. (C) revenue as, or more or less than, the current tax. And even
The question is How come the Akabe people drink such if such a comparison were apt, a lower luxury tax rate could
small amounts of tea in the early morning? Anthropologists bring in even more revenue, since luxuries are inferably more
figure that the problem is the tea’s high caffeine content; expensive than essentials.
Akabe people don’t want to get so shaky from caffeine that (B) is probably the most tempting wrong answer, since one
they can’t work. That’s plausible, but the author doesn’t give might assume that the officials making the proposal are
any real reason for the anthropologists to fix on caffeine anticipating some comparability in revenue. But (B) keys off
as the explanation, other than the observation that the tea the same irrelevant issue that (A) does—a bogus comparison
does in fact have a lot of it. (C) weakens that explanation by between the revenue generated now by the essentials tax
suggesting a plausible alternative explanation: if the leaves and that to be generated by the luxury tax. The goal of the
are loaded with a “soluble” narcotic, then tea made from the plan is simply to raise revenue “substantially,” without teeing
leaves probably has a strong narcotic effect. That suggests off the poor and middle class by raising a tax that mostly
that the Akabe people might not want to drink much tea in impacts them. That goal has nothing to do with the rate of the
the morning because they don’t want to be put right back to essentials tax or the revenue amount generated thereby.
sleep, not because the tea’s caffeine would turn them into (D) smacks of the “fallacy of denying the antecedent.” We’re
clumsy oafs. given to understand that if the tax on essentials is raised, then
(A) provides a reason why the Akabe would want to drink a low- and middle-income groups get mad. It doesn’t follow
great deal of the tea at dawn, but does nothing to weaken the that the goal of the luxury tax is to keep those groups happy.
author’s explanation of why they don’t. Whether the answer to (D) is yes or no, the proposal could
(B) fits in nicely with the anthropologists’ theory that the Akabe work as it’s meant to.
sure enjoy this drink, but in no way weakens the anthropologists’ (E) is what you learned in class as an “irrelevant distinction.”
conclusion that it must be the high caffeine content that The stimulus lumps rich people and corporations together as
prevents them from consuming much of it in the morning. purchasers of luxuries, so the proposal neither gains nor loses
(D) , if anything, supports the explanation. If adults avoid ground by any comparison of those groups separately.
drinking a lot of tea because of the powerful caffeine blast, it
stands to reason that they’d be especially careful with their 160. (D)
children. After all, how much coffee do we give to kids? We need to strengthen the argument that the advertising
(E) is consistent with the argument. It makes perfect sense campaign was responsible for the reduction of the number
that, when they’re celebrating and presumably not worrying of smokers—a pretty hasty conclusion, considering there’s
about “daily tasks,” the Akabe might cut loose and start an obvious competing explanation: cigarette prices are now
pounding down the tea. 20 cents higher. It’s not going out on a limb to speculate that
this factor might have played a role in the decline. In making
159. (C) her conclusion, the author assumes that the 20-centincrease
The “prediction” whose “accuracy” is in question is that played no part in the small decline in smoking, which would
imposing a luxury tax will generate big bucks for the be the answer if this were an Assumption question. But as
government, while affecting only fat cats and not us working you know from your preparation, a central assumption could
stiffs. Among the major assumptions here is that the rich will form the basis of a Strengthen or Weaken question as well.
continue to buy these luxuries even after such a tax is passed. Break down the assumption, and you have a weakener; shore
Of course, if that assumption proves to be false—if the tax up the assumption, and you have a strengthener. (D) does
turns out to deter the rich from making luxury purchases— the latter: if merchants responded to the tax by lowering the
then the scheme could end up generating little or no revenue price of cigarettes by the exact amount of the tax, then a price
whatsoever. That’s why (C)’s question has to be asked, and increase drops out as a candidate for the decline, the author’s
answered in the affirmative, if the plan is to work as it’s central assumption is validated, and the argument that the
meant to. advertising was the cause of the decline is strengthened.
(A) is outside the scope. The argument deals with smoking, Even if you couldn’t decide between (A) and (B), you should
not tobacco consumption in general. have found it easy to eliminate the others—if worst comes
(B) is also outside the scope. The author claims that the to worst be ready to eliminate obviously wrong choices to
number of smokers has been reduced, and so smokers that improve your odds of guessing correctly.
are merely cutting down don’t count.
(C) provides very weak evidence for the notion that the number
of smokers has declined, but does not address the cause of
the reduction.
(E) is mostly neutral, but one could argue that if smokers are
relatively poor, then they might be sensitive to the cigarette
price increase. The increase would then have more of an
impact than the author acknowledged, which would tend to
weaken the argument. In any case, whether this choice slightly
weakens or is neutral to the argument, you shouldn’t have
picked it as a strengthener.

161. (B)
We’re faced with a peculiar question stem here: we’re looking
for a proposal that would work together with the passage’s
proposal and improve its prospects for reducing fatalities. The
correct answer should help ensure that clearing away seats to
provide better exit access actually will decrease fatalities. (B)
does the job because it deals with the other factor mentioned
in the stimulus—that the increase in collisions was caused
by an increase in flights. If (B) weren’t to be implemented, if
airlines did increase the number of flights, then there would be
a danger that the number of collisions would increase as well,
and thus that the total number of fatalities would not decrease,
even though each individual collision might be safer. (B)
eliminates this possible reason for an increase in collisions, and
gives the stimulus proposal a better chance to work.
Of the other choices, only (A) is tempting—it says that any
new planes bought must already meet the proposal’s safety
requirements. However, (A) doesn’t really add anything to the
passage’s proposal. According to that proposal, whatever type
of airplane the airlines buy, they’ll have to clear access to the
exits before using the planes—it might save time and money to
buy airplanes in which this was already done, but it wouldn’t
help save lives. Also, we don’t know how many (if any) new
airplanes the airlines intend to buy—if they buy many, they
run the risk that (B) tries to avoid, that of overcrowding. (A)
may have been tempting, but you should have rejected it after
reading (B)—a good illustration of why you should always read
all the answer choices carefully in Logical Reasoning.
The other choices are wide misses. (C) and (E) talk about
check-in procedures and ticket prices, which have nothing to
do with passenger safety.
(D) talks about “security precautions,” a little nearer the mark,
but it only vaguely speaks of “defining” them more clearly,
not of making them more effective; in any case, security
precautions weren’t mentioned in the passage.
Flaw Questions
Directions: The questions in this section are based on the reasoning contained in brief statements or passages. For some questions, more
than one of the choices could conceivably answer the question. However, you are to choose the best answer; that is, the response that
most accurately and completely answers the question. You should not make assumptions that are by commonsense standards implausible,
superfluous, or incompatible with the passage.

1. Many corporations have begun decorating their halls 2. Editorial: Clearly, during the past two years, the
with motivational posters in hopes of boosting their unemployment situation in our city has been
employees’ motivation to work productively. However, improving. Studies show that the number of
almost all employees at these corporations are already unemployed people who are actively looking for
motivated to work productively. So, these jobs has steadily decreased during that period.
corporations’ use of motivational posters is unlikely to
The editorial’s reasoning is most vulnerable to criticism
achieve its intended purpose.
on the grounds that it
The reasoning in the argument is most vulnerable to
(A) presumes, without providing justification, that the
criticism on the grounds that the argument
government is at least partly responsible for the
(A) fails to consider whether corporations that do improvement in the employment situation
not currently use motivational posters would (B) relies on data from a period that is too short to
increase their employees’ motivation to work justify an inference about a general trend
productively if they began using the posters (C) fails to take into account the possibility that many
(B) takes for granted that, with respect to their unemployed workers who still desire jobs may
employees’ motivation to work productively, have stopped looking for jobs
corporations that decorate their halls with (D) fails to take into account that the sorts of
motivational posters are representative of governmental efforts that reduce unemployment
corporations in general may not be effective in creating more high-
(C) fails to consider that even if motivational posters paying jobs
do not have one particular beneficial effect for (E) ignores other economic indicators, which may not
corporations, they may have similar effects that have improved during the past two years
are equally beneficial
(D) does not adequately address the possibility that 3. Researcher: People with certain personality disorders
employee productivity is strongly affected by have more theta brain waves than those without
factors other than employees’ motivation to such disorders. But my data show that the amount
work productively of one’s theta brain waves increases while
(E) fails to consider that even if employees are watching TV. So, watching too much TV
already motivated to work productively, increases one’s risk of developing personality
motivational posters may increase that disorders.
motivation
A questionable aspect of the reasoning above is that it
(A) uses the phrase “personality disorders”
ambiguously
(B) fails to define the phrase “theta brain waves”
(C) takes correlation to imply a causal connection
(D) draws a conclusion from an unrepresentative
sample of data
(E) infers that watching TV is a consequence of a
personality disorder
4. Joan got A’s on all her homework assignments, so if she 6. Although it has been suggested that Arton’s plays have
had gotten an A on her term paper, she could pass the a strong patriotic flavor, we must recall that, at the time
course even without doing the class presentation. of their composition, her country was in anything but
Unfortunately, she did not get an A on her term paper,
so, it is obvious that she will have to do the class a patriotic mood. Unemployment was high, food was
presentation to pass the course. costly, and crime rates were soaring. As a result, the
general morale of her nation was at an especially low
The argument’s reasoning is questionable because the
point. Realizing this, we see clearly that any apparent
argument
patriotism in Arton’s work must have been intended
(A) ignores the possibility that Joan must either ironically.
have an A on her term paper or do the class
The reasoning above is questionable because it
presentation to pass the course
(B) presupposes without justification that Joan’s not (A) posits an unstated relationship between
getting an A on her term paper prevents her unemployment and crime
from passing the course without doing the class (B) takes for granted that straightforward patriotism
presentation is not possible for a serious writer
(C) overlooks the importance of class presentations to (C) takes for granted that Arton was attuned to the
a student’s overall course grade predominant national attitude of her time
(D) ignores the possibility that if Joan has to do the (D) overlooks the fact that some citizens prosper in
class presentation to pass the course, then she times of high unemployment
did not get an A on her term paper (E) confuses irony with a general decline in public
(E) fails to take into account the possibility that some morale
students get A’s on their term papers but do not
pass the course 7. Proponent: Irradiation of food by gamma rays would
keep it from spoiling before it reaches the
5. The consumer price index is a measure that detects consumer in food stores. The process leaves
monthly changes in the retail prices of goods and no radiation behind, and vitamin losses are
services. The payment of some government retirement comparable to those that occur in cooking,
benefits is based on the consumer price index so that so, there is no reason to reject irradiation on
those benefits reflect the change in the cost of living the grounds of nutrition or safety. Indeed, it
as the index changes. However, the consumer price kills harmful Salmonella bacteria, which in
index does not consider technological innovations that contaminated poultry have caused serious illness
may drastically reduce the cost of producing some to consumers.
goods. Therefore, the value of government benefits is
Opponent: The irradiation process has no effect on the
sometimes greater than is warranted by the true change
bacteria that cause botulism, a very serious form
in costs.
of food poisoning, while those that cause bad
The reasoning in the argument is most vulnerable to the odors that would warn consumers of botulism are
criticism that the argument killed. Moreover, Salmonella and the bacteria that
cause botulism can easily be killed in poultry by
(A) fails to consider the possibility that there are
using a safe chemical dip.
years in which there is no change in the
consumer price index Which one of the following could the opponent properly
(B) fails to make explicit which goods and services cite as indicating a flaw in the proponent’s reasoning
are included in the consumer price index concerning vitamin losses?
(C) presumes, without providing warrant, that
(A) After irradiation, food might still spoil if kept in
retirement benefits are not generally used to
storage for a long time after being purchased by
purchase unusual goods
the consumer.
(D) uncritically draws an inference from what has
(B) Irradiated food would still need cooking, or,
been true in the past to what will be true in the
if eaten raw, it would not have the vitamin
future
advantage of raw food.
(E) makes an irrelevant shift from discussing retail
(C) Vitamin loss is a separate issue from safety.
prices to discussing production costs
(D) Vitamins can be ingested in pill form as well as
in foods.
(E) That food does not spoil before it can be offered
to the consumer is primarily a benefit to the
seller, not to the consumer.
8. It is widely believed that by age 80, perception and 10. John: For 40 years, fluoride has been added to public
memory are each significantly reduced from their drinking water. According to a study, fluoridated
functioning levels at age 30. However, a recent study public drinking water when given to laboratory
showed no difference in the abilities of 80-year-olds and rats causes bone cancer. Ninety percent of all the
30-year-olds to play a card game devised to test male rats in the test sample were affected, but the
perception and memory. Therefore, the belief that female rats were unaffected. Even though I am
perception and memory are significantly reduced by age healthy now, I should nevertheless stop drinking
80 is false. fluoridated water; only then will I be sure not to
The reasoning above is most vulnerable to criticism on develop bone cancer.
the grounds that it fails to consider the possibility that Which one of the following is the strongest criticism of
John’s reasoning?
(A) the study’s card game does not test cognitive
abilities other than perception and memory (A) John does not consider how others besides
(B) card games are among the most difficult cognitive himself are affected by fluoridation of water.
tasks one can attempt to perform (B) John does not consider whether fluoridated water
(C) perception and memory are interrelated in ways causes other diseases.
of which we are not currently aware (C) John does not consider whether there were any
(D) the belief that 80-year-olds’ perception and brief periods during the 40 years in which
memory are reduced results from prejudice fluoride was not added to the water.
against senior citizens (D) John does not focus on the positive effects that
(E) playing the study’s card game perfectly requires fluoridated water has on people
fairly low levels of perception and memory (E) John does not consider the possibility of other
causes of bone cancer.
9. Brewer: All children should be given the opportunity to
participate in competitive sports; these activities 11. News item: The result of a recent public survey has
provide an unsurpassed opportunity to engage been called into question because one of the
children’s emotions and so stimulate them to pollsters admitted to falsifying data. The survey
put maximum effort into attaining high athletic originally concluded that most people in the
standards. country favor investing more money in
information technologies. Because falsified data
Polanski: I disagree. Competitive athletics does, over were included in the survey, its conclusion is not
time, produce a handful of highly motivated true; a majority does not favor more investment in
children with a desire to excel at an activity, but information technologies.
many children have no taste for competition, and
to make them participate would only cause them The news item’s argument is vulnerable to criticism
to develop an antipathy toward athletics. because it fails to consider the possibility that

Polanski’s response most strongly supports the (A) the conclusion of the survey would be verified if
contention that Polanski misunderstood Brewer to be the falsified data were excluded
asserting that (B) the conclusion of the survey will be accepted by
the public even though falsified data were used
(A) characteristics acquired by playing competitive (C) other pollsters in other surveys also may have
sports carry over into the rest of one’s life falsified data
(B) winning at competitive sports is essential to (D) some people who responded to the survey were
motivation to excel at athletics lying
(C) children should put more effort into athletic (E) people’s opinions about investing in information
activities than any other form of activity technologies can change as new technologies are
(D) children should be required to participate in developed
competitive sports regardless of their interests
(E) children cannot be motivated without their
emotions being engaged
12. Undoubtedly, one’s freedom is always worth the risk of 14. Formal performance evaluations in the professional
losing one’s life. Consider a person who is locked in a world are conducted using realistic situations.
bare cement room with no hope of escape. This person Physicians are allowed to consult medical texts freely,
is not really living and has nothing to lose. attorneys may refer to law books and case records, and
physicists and engineers have their manuals at hand for
A flaw in the argument’s reasoning is that the argument ready reference. Students, then, should likewise have
(A) presumes, without providing justification, that access to their textbooks whenever they take
nothing can have greater value than one’s own examinations.
freedom The reasoning in the argument is questionable because
(B) fails to consider that it is not always possible to the argument
rebel physically against an encroachment on
one’s freedom (A) cites examples that are insufficient to support the
(C) generalizes inappropriately from a single extreme generalization that performance evaluations in
case to a universal claim the professional world are conducted in realistic
(D) fails to establish that the freedom of others is situations
worth taking risks for (B) fails to consider the possibility that adopting its
(E) overlooks the possibility that some people do not recommendation will not significantly increase
have the courage to take risks for freedom most students’ test scores
(C) neglects to take into account the fact that
13. Researchers have developed a membrane that quickly professionals were once students who also
removes the oxygen from a refrigerator, thereby killing did not have access to textbooks during
bacteria and enabling food to be stored almost examinations
indefinitely. Since food need not be kept as cold to (D) neglects to take into account the fact that, unlike
prevent spoilage, energy costs will be greatly reduced. students, professionals have devoted many years
Thus, over the long run, oxygen-removing refrigerators of study to one subject
will prove far less expensive than current models. (E) fails to consider the possibility that the purposes
of evaluation in the professional world and in
The reasoning in the argument is most vulnerable to
school situations are quite dissimilar
criticism on the grounds that it does not
(A) address the expense of building or refitting a
refrigerator with the new technology
(B) address the possibility of consumer discomfort
with the new refrigerators
(C) explain the technology that enabled the oxygen-
removing membrane to be developed
(D) take into account the effectiveness of current
refrigerator technology in preventing food
spoilage
(E) take into account the inconvenience caused by
food spoilage in current refrigerators
15. Jane: Television programs and movies that depict 17. Large discount chains can make a profit even while
violence among teenagers are extremely popular. offering low prices, because they buy goods in large
Given how influential these media are, we have quantities at favorable cost. This creates a problem for
good reason to believe that these depictions cause small retailers. If they try to retain their customers by
young people to engage in violent behavior. lowering prices to match those of large discount chains,
Hence, depictions of violence among teenagers the result is a lower profit margin. But small retailers can
should be prohibited from movies and television retain their customer base without lowering prices if they
programs, if only in those programs and movies offer exceptional service. Hence, small retailers that are
promoted to young audiences. forced to compete with large discount chains must offer
exceptional service in order to retain their level of
Maurice: But you are recommending nothing short profitability.
of censorship! Besides which, your claim that
television and movie depictions of violence The reasoning is flawed because it fails to take into
cause violence is mistaken: violence among account the possibility that
young people predates movies and television by (A) not all large discount chains do in fact make a
centuries. profit
Maurice’s attempted refutation of Jane’s argument is (B) some large discount chains have lower profit
vulnerable to criticism on which one of the following margins than do some small retailers
grounds? (C) small retailers are often motivated by things other
than the desire for profit
(A) It presupposes that an unpopular policy cannot (D) not all small retailers are forced to compete with
possibly achieve its intended purpose. large discount chains
(B) It confuses a subjective judgment of private moral (E) exceptional service is not the only reason
permissibility with an objective description of customers prefer small retail stores
social fact.
(C) It rules out something as a cause of a current 18. Marion knows that the bridge on her usual route is
phenomenon solely on the ground that the closed and that, unless she takes the train instead of
phenomenon used to occur without that thing. driving, she can get to work on time only by leaving at
(D) It cites purported historical facts that cannot least 45 minutes early. She must go to her neighborhood
possibly be verified. bank before leaving for work, however, and the bank
(E) It relies on an ambiguity in the term “violence” to will not be open in time for her to do so if she leaves
justify a claim. 45 minutes early. Therefore, since she hates taking the
train, Marion cannot avoid being late for work.
16. Political commentators see recent policies of the The reasoning in the argument is flawed because the
government toward Country X as appeasement, pure argument
and simple. This view is fundamentally mistaken, for
polls show that most people disagree with the political (A) mistakes a situation that almost certainly affects
commentators’ assessment of government policies many people for one that affects a particular
toward Country X. person alone
(B) ignores the fact that people often know that
The reasoning in the argument is questionable because something is the case without considering all the
(A) the term “policies” is used ambiguously in the consequences that follow from its being the case
argument (C) assumes without justification that because people
(B) the political commentators discussed in the generally have an interest in avoiding a given
passage are not identified result, any particular person will have an interest
(C) a claim is inferred to be false merely because a in avoiding that result
majority of people believe it to be false (D) treats evidence that someone will adopt a
(D) the claim that the political commentators are particular course of action as though that
mistaken is both a premise and a conclusion in evidence excluded the possibility of an
the argument alternative course of action
(E) it is assumed that what is true of persons (E) overlooks the possibility that someone might
individually is true of a country as a whole occasionally adopt a given course of action
without having a good reason for doing so
19. Grasses and woody plants are planted on dirt 21. Consumer: I would like to have the features contained
embankments to keep the embankments from eroding. in the latest upgrade to your computer software
The embankments are mowed to keep the grasses from package, but I am leery of installing the upgrade
growing too tall; as a result, clippings pile up. These because a friend has told me he had a problem
piles of clippings smother the woody plants, causing with it.
their roots, which serve to keep the embankments from
Company representative: We have distributed nearly
eroding, to rot; they also attract rodents that burrow into
3,000 copies of the upgrade and we have received
the dirt and damage the woody plants’ roots. Therefore,
fewer than 100 calls saying that it has caused
bringing in predators to eradicate the rodents will
problems. So, it is very unlikely that you will
prevent erosion of the embankments.
experience any problems with the upgrade.
Which one of the following is an error of reasoning in
The reasoning in the company representative’s argument
the argument?
is most vulnerable to criticism because it fails to
(A) Two events that merely co-occur are treated as if consider the possibility that
one caused the other.
(A) the company will issue another upgrade that
(B) A highly general proposal is based only on an
corrects the problems with the current upgrade
unrepresentative set of facts.
(B) some of the problems people have experienced
(C) The conclusion is no more than a restatement
with the upgrade have been quite serious
of one of the pieces of evidence provided to
(C) a significant number of people have experienced
support it.
problems with the upgrade but have not reported
(D) One possible solution to a problem is claimed to
them
be the only possible solution to that problem.
(D) the consumer will experience software problems
(E) An action that would eliminate one cause of a
if the upgrade is not installed
problem is treated as if it would solve the entire
(E) some of the reported problems were a result of
problem.
users failing to follow instructions
20. Politician P: My opponent claims that the government
22. Insurance that was to become effective at 9 A.M. on
is obligated to raise taxes to increase funding for
a certain date was taken out on the life of a flight
schools and health care. Because raising taxes
attendant. He died on that date at 10 A.M. local time,
to increase funding for schools and health care
which was two hours before 9 A.M. in the time zone
would make taxpayers upset over their loss of
where the policy had been purchased. The insurance
buying power, my opponent is simply mistaken.
company contended that the policy had not become
Politician P’s reasoning is questionable because it effective; a representative of the flight attendant’s
involves beneficiary, his mother, countered by arguing that the
policy amount should be paid because the attendant had
(A) presupposing that a claim is mistaken on the
been his mother’s sole support, and she was ill.
grounds that the person defending it advocates
other unpopular views The representative’s argument is flawed as a counter to
(B) assuming that a claim is false on the grounds the insurance company’s contention because
that the person defending it is of questionable
(A) the conclusion is no more than a paraphrase of
character
the evidence offered in support of it
(C) concluding that a view is false on the grounds
(B) it appeals to the emotion of pity rather than
that its implementation would lead to
addressing the issue raised
unhappiness
(C) it makes an unwarranted distinction between
(D) appealing to wholly irrelevant issues to deflect
family obligations and business obligations
attention away from the real issue
(D) it substitutes an attack on a person for the giving
(E) insisting that an obligation exists without offering
of reasons
any evidence that it exists
(E) a cause and its effect are mistaken for each other
23. Consumer advocate: Last year’s worldwide alarm about 25. Sociologist: The claim that there is a large number
a computer “virus”—a surreptitiously introduced of violent crimes in our society is false, for
computer program that can destroy other this claim is based upon the large number of
programs and data—was a fraud. Companies stories in newspapers about violent crimes. But
selling programs to protect computers against since violent crimes are very rare occurrences,
such viruses raised worldwide concern about newspapers are likely to print stories about them.
the possibility that a destructive virus would
The sociologist’s argument is flawed because it
be activated on a certain date. There was more
smoke than fire, however, only about a thousand (A) presupposes that most newspaper stories are
cases of damage were reported around the world. about violent crime
Multitudes of antivirus programs were sold, so the (B) presupposes the truth of the conclusion it is
companies’ warning was clearly only an effort to attempting to establish
stimulate sales. (C) assumes without warrant that the newspaper
stories in question are not biased
The reasoning in the consumer advocate’s argument is
(D) mistakes a property of each member of a group
flawed because this argument
taken as an individual for a property of the
(A) restates its conclusion without attempting to offer group taken as a whole
a reason to accept it (E) uncritically draws an inference from what has
(B) fails to acknowledge that antivirus programs been true in the past to what will be true in the
might protect against viruses other than the future
particular one described
(C) asserts that the occurrence of one event after 26. Several excellent candidates have been proposed for
another shows that the earlier event was the the presidency of United Wire, and each candidate
cause of the later one would bring to the job different talents and experience.
(D) uses inflammatory language as a substitute for If the others are compared with Jones, however, it will
providing any evidence be apparent that none of them has her unique set of
(E) overlooks the possibility that the protective qualifications. Jones, therefore, is best qualified to be the
steps taken did work and, for many computers, new president of United Wire.
prevented the virus from causing damage
The argument is vulnerable to criticism on the ground
that it
24. Columnist: The country is presently debating legislation
that, if passed, would force manufacturers to (A) uses flattery to win over those who hold an
increase the number of paid vacation days for opposing position
employees, to pay higher overtime wages, and (B) refutes a distorted version of an opposing position
to pay all day-care expenses for children of each (C) seeks to distinguish one member of a group on
employee. This legislation is being supported by the basis of something that applies to all
members of groups that have resorted to violent (D) supports a universal claim on the basis of a single
tactics in the past, and by individuals who are example
facing indictment on tax-evasion charges. We (E) describes an individual in terms that appropriately
must defeat this legislation and what it stands for. refer only to the group as a whole
The columnist’s argument is flawed because it
(A) attacks legislation by calling into question the
integrity of the originators of the legislation
(B) assails legislation on the basis of the questionable
character of supporters of the legislation
(C) attempts to discredit legislation by appealing
to public sentiment for those who would be
adversely affected
(D) presupposes that legislation is bad legislation
whenever it has only a small number of
supporters outside the country’s national
legislative body
(E) rejects legislation on the grounds that its
supporters act inconsistently in seeking to place
burdens on manufacturers upon whose business
success the supporters depend
27. Television news coverage gives viewers a sense of direct 29. Astorga’s campaign promises are apparently just an
involvement with current events but does not provide the attempt to please voters. What she says she will do if
depth of coverage needed for the significance of those elected mayor is simply what she has learned from
events to be appreciated. Newspapers, on the other hand, opinion polls that voters want the new mayor to do.
provide depth of coverage but no sense of direct Therefore, voters are not being told what Astorga
involvement. Unfortunately, a full understanding of actually intends to do if she becomes mayor.
current events requires both an appreciation of their
significance and a sense of direct involvement with Which one of the following is a questionable assumption
them. Therefore, since few people seek out news sources on which the argument relies?
other than newspapers and television, few people ever (A) If she is elected mayor, Astorga will not be
fully understand current events. capable of carrying out the campaign promises
The reasoning in the argument is flawed because the she has made.
argument (B) The opinion polls on which Astorga’s promises
are based do not accurately reflect what voters
(A) treats two things, neither one of which can want the new mayor to do.
plausibly be seen as excluding the other, as (C) Most voters are unlikely to be persuaded by
though they were mutually exclusive Astorga’s campaign promises to vote for her in
(B) ignores the possibility that people read the mayoral election.
newspapers or watch television for reasons other (D) Astorga has no strong opinions of her own about
than gaining a full understanding of current what the new mayor ought to do in office.
events (E) Astorga does not actually intend, if elected, to do
(C) makes crucial use of the term “depth of coverage” what she has learned from the public opinion
without defining it polls that voters want the new mayor to do.
(D) fails to consider the possible disadvantages of
having a sense of direct involvement with tragic 30. Since Professor Smythe has been head of the
or violent events department, the most distinguished member of the
(E) mistakenly reasons that just because something faculty has resigned, fewer new courses have been
has the capacity to perform a given function it developed, student enrollment has dropped, and the
actually does so reputation of the department has gone down. These facts
provide conclusive evidence that Professor Smythe was
28. Advertisement: Among popular automobiles, appointed to undermine the department.
Sturdimades stand apart. Around the world,
The reasoning in the argument is flawed because the
hundreds of longtime Sturdimade owners have
argument
signed up for Sturdimade’s “long distance” club,
members of which must have a Sturdimade they (A) overlooks the fact that something can have the
have driven for a total of at least 100,000 miles reputation for being of poor quality without
or 160,000 kilometers. Some members boast of being of poor quality
having driven their Sturdimades for a total of (B) bases a general claim on a few exceptional
300,000 miles (480,000 kilometers)! Clearly, if instances
you buy a Sturdimade you can rely on being able (C) assumes that because an action was followed by a
to drive it for a very long distance. change, the action was undertaken to bring about
that change
Construed as an argument, the advertisement’s reasoning
(D) fails to distinguish between a decline in quantity
is most vulnerable to criticism on which one of the
and a decline in quality
following grounds?
(E) presupposes what it purports to establish
(A) It draws a general conclusion from cases selected
only on the basis of having a characteristic that
favors that conclusion.
(B) Its conclusion merely restates the evidence given
to support it.
(C) It fails to clarify in which of two possible
ways an ambiguous term is being used in the
premises.
(D) The evidence given to support the conclusion
actually undermines that conclusion.
(E) It treats popular opinion as if it constituted
conclusive evidence for a claim.
31. A report of a government survey concluded that Center 33. Safety inspector: The number of laboratory samples of
City was among the ten cities in the nation with the rabies virus sent through the university delivery
highest dropout rate from its schools. The survey data service has recently grown dangerously high. We
were obtained by asking all city residents over the age need to limit this use of the service.
of 19 whether they were high school graduates and
Biologist: There is no need for a limit. The university
computing the proportion who were not. A city school
delivery service has been handling rabies virus
of official objected that the result did not seem accurate
samples for 20 years with no accidents.
according to the schools’ figures.
As a rebuttal of the safety inspector’s claim, the
The school official can most properly criticize the
biologist’s reasoning is flawed in that it
reasoning by which the survey report reached its result
for failure to do which one of the following? (A) fails to explain why the number of samples sent
through the service has increased
(A) take into account instances of respondents’
(B) fails to focus specifically on the issue of rabies
dropping out that occurred before the
virus samples
respondents reached high school
(C) overlooks the possibility that there has been a
(B) ask residents whether they had completed their
change in the hazardousness of the rabies virus
high school work in fewer than the usual number
samples themselves
of years
(D) offers no defense of the university’s need for
(C) distinguish between residents who had attended
continued delivery of rabies virus samples
the city’s schools and those who had received
(E) does not address the potential for harm that is
their schooling elsewhere
posed by the recent increase in the number of
(D) predict the effect of the information contained in
samples sent through the service
the report on future high school dropout rates for
the city
34. Questions have arisen regarding the accuracy of the
(E) consider whether a diploma from the city’s high
reports the university’s archaeological museum issues
schools signaled the same level of achievement
on its sales and acquisitions for the year. To forestall
over time
controversy, this year’s report is being reviewed by
three archaeologists from other universities. Since these
32. The law firm of Sutherlin, Pérez, and Associates is archaeologists will be given full access to all documents
one of the most successful law firms whose primary on which the report is based, they will be able to
specialization is in criminal defense cases. In fact, determine whether it is indeed accurate.
the firm has a better than 90 percent acquittal rate in
such cases. Dalton is an attorney whose primary The reasoning in the argument is flawed because the
specialization is in divorce cases, so Dalton certainly argument
cannot be a member of Sutherlin, Pérez, and Associates. (A) does not specify whether the reviewers will have
The reasoning in the argument is flawed because the access to data about objects that have been in the
argument museum’s collection for many years
(B) provides no information regarding the size
(A) offers in support of its conclusion pieces of
or quality of the archaeological museum’s
evidence that are mutually contradictory
collection
(B) overlooks the possibility that a person can
(C) omits any mention of whether the museum’s
practice law without being a member of a law
collection is on display or is available only to
firm
researchers
(C) concludes that someone is not a member of a
(D) ignores the possibility that there might have been
group on the grounds that that person does not
some sales or acquisitions during the past year
have a characteristic that the group as a whole
that were not mentioned in the documents on
has
which the report was based
(D) takes a high rate of success among the members
(E) does not describe what will occur if the reviewers
of a group to indicate that the successes are
discover discrepancies between the report and
evenly spread among the members
the documents on which it was based
(E) states a generalization based on a selection that is
not representative of the group about which the
generalization is supposed to hold true
35. P: Because an elected official needs the support of a 37. Each of the elements of Girelli’s recently completed
political party to be effective, the independent design for a university library is copied from a different
candidate for the legislature cannot possibly be an one of several historic libraries. The design includes
effective legislator if she wins. various features from Classical Greek, Islamic, Mogul,
Q: I disagree. By your reasoning, our current legislator, and Romanesque structures. Since no one element in
who has the support of a political party, ought to the design is original, it follows that the design of the
have been effective, but he has not been. library cannot be considered original.
Which one of the following is the best criticism of Q’s Which one of the following is a reasoning error made in
statement? the argument?
(A) It simply contradicts P’s claim without offering (A) assuming that because something is true of each
evidence against it. of the parts of a whole it is true of the whole
(B) It does not consider the possibility that a political itself
party might decide to support an elected (B) generalizing illegitimately from a few instances
legislator even though he or she ran as an of a certain kind to all instances of that kind
independent. (C) concluding that an unknown instance of a
(C) It fails to prove a precise definition for a key phenomenon must have all the properties of the
term—the word “effective.” known instances
(D) It presupposes what is to be proved—that a (D) presupposing that alternatives that can be true
legislator must have the support of a political separately cannot be true together
party in order to be “effective.” (E) deriving a factual conclusion from evidence
(E) It mistakenly interprets P to be claiming that a derived from reports of aesthetic preferences
factor assures, rather than is necessary for, a
legislator’s effectiveness. 38. Broadcaster: Our radio station has a responsibility to
serve the public interest. Hence, when our critics
36. Novice bird-watcher: I don’t know much about animal contend that our recent exposé of events in the
tracks, but I do know that birds typically have private lives of local celebrities was excessively
four toes, and most birds have three toes pointing intrusive, we can only reply that the
forward and one toe pointing backward. Since overwhelming public interest in these matters
this track was made by an animal with four toes, makes it our responsibility to publicize them.
of which three point forward and one points
backward, we can conclude it was made by some Which one of the following is a flaw in the broadcaster’s
kind of bird. defense of the radio station’s practice?

The argument is flawed because it (A) assuming without argument that there is a right to
privacy
(A) relies on the vagueness of the term “track” (B) ignoring grounds for criticism of the exposé aside
(B) does not define birds as animals with four toes from intrusion into people’s private lives
(C) fails to identify what kind of bird might have (C) intentionally failing to specify what is meant by
made the track “excessively intrusive”
(D) does not establish that only a bird could have (D) confusing legal responsibility with moral
made the track obligation
(E) depends on evidence about an individual bird (E) improperly exploiting an ambiguity in the phrase
rather than about birds in general “public interest”
39. In the last election, 89 percent of reporters voted for the 41. The druid stones discovered in Ireland are very, very
incumbent. The content of news programs reveals that old. But this particular druid stone was discovered in
reporters allowed the personal biases reflected in this Scotland; hence, it must be of more recent vintage.
voting pattern to affect their news coverage: 54 percent The argument is flawed because it
of coverage concerning the challenger was negative,
compared with only 30 percent of that concerning the (A) allows a key term to shift in meaning from one
incumbent. use to the next
(B) takes the fact that most members of a group have
The argument is logically most vulnerable to criticism a certain property to constitute evidence that all
on the grounds that it members of the group have that property
(A) presumes, without providing justification, that (C) takes for granted the very claim that it sets out to
both candidates received equal amounts of establish
coverage overall (D) presumes without justification that what was
(B) ignores the possibility that there was more true of the members of a group in the past will
negative news worthy of reporting concerning continue to be true of them in the future
the challenger than there was concerning the (E) takes the fact that all members of a group have a
incumbent certain property to constitute evidence that the
(C) presumes, without providing justification, that members of the group are the only things with
allowing biases to influence reporting is always that property
detrimental to the resulting news coverage
(D) ignores the possibility that the electorate’s voting 42. While it was once believed that the sort of psychotherapy
behavior is not significantly affected by the appropriate for the treatment of neuroses caused by
environmental factors is also appropriate for schizophrenia
content of coverage of candidates and other psychoses, it is now known that these latter,
(E) ignores the possibility that reporters generally more serious forms of mental disturbance are best treated
fear losing access to incumbents more than they by biochemical—that is, medicinal—means. This is
fear losing access to challengers
conclusive evidence that psychoses, unlike neuroses, have
nothing to do with environmental factors but rather are
40. In determining the authenticity of a painting, caused by some sort of purely organic condition, such as
connoisseurs claim to be guided by the emotional impact abnormal brain chemistry or brain malformations.
the work has on them. For example, if a painting
purportedly by Rembrandt is expressive and emotionally The argument is vulnerable to criticism because it
moving in a certain way, then this is supposedly ignores the possibility that
evidence that the work was created by Rembrandt (A) the organic conditions that result in psychoses
himself, and not by one of his students. But the degree to can be caused or exacerbated by environmental
which an artwork has an emotional impact differs wildly factors
from person to person. So, a connoisseur’s assessment (B) the symptoms of mental disturbance caused by
cannot be given credence. purely organic factors can be alleviated with
The reasoning in the argument is most vulnerable to medicine
criticism on the grounds that the argument (C) organic illnesses that are non-
psychological in nature may be treatable
(A) ignores the fact that anybody, not just a without using biochemical methods
connoisseur, can give an assessment of the (D) the nature of any medical condition can be
emotional impact of a painting inferred from the nature of the treatment that
(B) is based on the consideration of the nature of just cures that condition
one painter’s works, even though the conclusion (E) organic factors having little to do with brain
is about paintings in general chemistry may be at least partially responsible
(C) neglects the possibility that there may be for neuroses
widespread agreement among connoisseurs
about emotional impact even when the public’s
assessment varies wildly
(D) presumes, without giving justification, that a
painting’s emotional impact is irrelevant to the
determination of that painting’s authenticity
(E) presumes, without offering evidence, that
Rembrandt was better at conveying emotions in
painting than were other painters
43. Mayor: The law prohibiting pedestrians from crossing 45. Restaurant manager: In response to requests from our
against red lights serves no useful purpose. patrons for vegetarian main dishes, we recently
After all, in order to serve a useful purpose, a introduced three: an eggplant and zucchini
law must deter the kind of behavior it prohibits. casserole with tomatoes, brown rice with
But pedestrians who invariably violate this law mushrooms, and potatoes baked with cheese. The
are clearly not dissuaded by it; and those who first two are frequently ordered, but no one orders
comply with the law do not need it, since they the potato dish, although it costs less than the
would never cross against red lights even if other two. Clearly, then, our patrons prefer not to
there were no law prohibiting pedestrians from eat potatoes.
crossing against red lights.
Which one of the following is an error of reasoning in
The mayor’s argument is flawed because it the restaurant manager’s argument?
(A) takes for granted that most automobile drivers (A) concluding that two things that occur at the same
will obey the law that prohibits them from time have a common cause
driving through red lights (B) drawing a conclusion that is inconsistent with one
(B) uses the word “law” in one sense in the premises premise of the argument
and in another sense in the conclusion (C) ignoring possible differences between what
(C) ignores the possibility that a law might not serve people say they want and what they actually
a useful purpose even if it does deter the kind of choose
behavior it prohibits (D) attempting to prove a claim on the basis of
(D) fails to consider whether the law ever dissuades evidence that a number of people hold that claim
people who sometimes but not always cross to be true
against red lights (E) treating one of several plausible explanations of a
(E) provides no evidence that crossing against red phenomenon as the only possible explanation
lights is more dangerous than crossing on green
lights 46. A group of 1,000 students was randomly selected from
three high schools in a medium-sized city and asked
44. In his new book on his complex scientific research, R the question, “Do you plan to finish your high school
frequently imputes bad faith to researchers disagreeing education?” More than 89 percent answered “Yes.” This
with him. A troubling aspect of R’s book is his stated shows that the overwhelming majority of students want
conviction that other investigators’ funding sources to finish high school, and that if the national dropout rate
often determine what “findings” those investigators among high school students is high, it cannot be due to a
report. Add to this that R has often shown himself to lack of desire on the part of the students.
be arrogant, overly ambitious, and sometimes plain
The reasoning of the argument above is questionable
nasty, and it becomes clear that R’s book does not merit
because the argument
attention from serious professionals.
(A) fails to justify its presumption that 89 percent is
The author of the book review commits which one of
an overwhelming majority
the following reasoning errors?
(B) attempts to draw two conflicting conclusions
(A) using an attack on the character of the writer from the results of one survey
of the book as evidence that his person is not (C) overlooks the possibility that there may in fact
competent on matters of scientific substance not be a high dropout rate among high school
(B) taking it for granted that an investigator is students
unlikely to report findings that are contrary to (D) contradicts itself by admitting that there may be a
the interests of those funding the investigation high dropout rate among students while claiming
(C) dismissing a scientific theory by giving a biased that most students want to finish high school
account of it (E) treats high school students from a particular
(D) presenting as facts several assertions about the medium-sized city as if they are representative
book under review that are based only on strong of high school students nationwide
conviction and would be impossible for others to
verify
(E) failing to distinguish between the criteria of being
true and of being sufficiently interesting to merit
attention
47. Philosopher: People are not intellectually well suited to 49. Scientists hoping to understand and eventually reverse
live in large bureaucratic societies. Therefore, damage to the fragile ozone layer in the Earth’s upper
people can find happiness, if at all, only in atmosphere used a spacecraft to conduct crucial
smaller political units such as villages. experiments. These experiments drew criticism from a
group of environmentalists who observed that a single
The reasoning in the philosopher’s argument is flawed
trip by the spacecraft did as much harm to the ozone
because the argument takes for granted that
layer as a year’s pollution by the average factory, and
(A) no one can ever be happy living in a society in that since the latter was unjustifiable so must be the
which she or he is not intellectually well suited former.
to live
The reasoning in the environmentalists’ criticism is
(B) the primary purpose of small political units such
questionable because it
as villages is to make people happy
(C) all societies that are plagued by excessive (A) treats as similar two cases that are different in a
bureaucracy are large critical respect
(D) anyone who lives in a village or other small (B) justifies a generalization on the basis of a single
political unit that is not excessively bureaucratic instance
can find happiness (C) fails to distinguish the goal of reversing harmful
(E) everyone is willing to live in villages or other effects from the goal of preventing those harmful
small political units effects
(D) attempts to compare two quantities that are not
48. A nationwide poll of students, parents, and teachers comparable in any way
showed that over 90 percent believe that an appropriate (E) presupposes that experiments always do harm to
percentage of their school’s budget is being spent on their subjects
student counseling programs. It seems, then, that any
significant increase in a school’s budget should be spent 50. Sid: The sign says “Keep off the grass.”
on something other than student counseling programs.
Micki: I know, but just one person walking across the
Which one of the following describes a flaw in the grass doesn’t hurt it.
reasoning of the argument above?
Sid: Your statement is false. If everyone believed as you
(A) The argument confuses a mere coincidence with a do, everyone would walk across the grass, and the
causal relationship. grass would die.
(B) The argument confuses the percentage of the
budget spent on a program with the overall Sid’s argument is questionable in that it
amount spent on that program. (A) attempts to use a statement about the
(C) The argument fails to justify its presumption that consequences of actions to disprove a statement
what is true of a part of the budget is also true of about the actions themselves
the total budget. (B) treats a statement about the consequences of
(D) The argument fails to consider the possibility that an action as though it were instead about
money could be saved by training students as the consequences of everyone believing the
peer counselors. statement
(E) The argument fails to consider that if more (C) contradicts itself by treating a statement that
money is spent on a program, then more money the arguer does not believe as though it were a
cannot also be used for other purposes. statement believed by everyone
(D) discounts the fact that there may be circumstances
under which hurting the grass is justified
(E) attempts to undermine a statement by calling into
question the character of the person making the
statement
51. In a recent poll of chief executive officers (CEOs) of 53. Plant Manager: We could greatly reduce the amount of
125 large corporations, the overwhelming majority sulfur dioxide our copper-smelting plant releases
claimed that employee training and welfare is of into the atmosphere by using a new process. The
the same high priority as customer satisfaction. So, new process requires replacing our open furnaces
the popular belief that the top management of large with closed ones and moving the copper from
corporations behaves indifferently to the needs and one furnace to the next in solid, not molten, form.
aspirations of employees is unfounded. However, not only is the new equipment
expensive to buy and install, but the new process
The argument is most vulnerable to criticism on the
also costs more to run than the current process,
grounds that it
because the copper must be reheated after it has
(A) fails to define adequately the term “top cooled. So overall, adopting the new process will
management” cost much but bring the company no profit.
(B) presumes, without giving justification, that one is
Supervisor: I agree with your overall conclusion, but
not indifferent to something that one considers a
disagree about one point you make, since the
top priority
latest closed furnaces are extremely fuel-efficient.
(C) presumes, without giving justification, that the
CEOs’ priorities tend to be misplaced The plant manager’s argument is most vulnerable to
(D) presumes, without giving justification, that the criticism on which one of the following grounds?
CEOs’ claims are reflected in actual practice
(A) The overall conclusion is about a net effect but is
(E) makes a generalization based on an
based solely on evidence about only some of the
unrepresentative sample
factors that contribute to the effect.
(B) The support for the overall conclusion is the
52. Faden: Most of our exercise machines are still in use
authority of the plant manager rather than any
after one year. A recent survey of our customers
independently verifiable evidence.
shows this.
(C) The overall conclusion reached merely repeats the
Greenwall: But many of those customers could easily evidence offered.
be lying because they are too embarrassed to (D) Evidence that is taken to be only probably true is
admit that they don’t exercise anymore. used as the basis for a claim that something is
definitely true.
Faden: You have no way of showing that customers
(E) Facts that are not directly relevant to the
were lying. Your objection is absurd.
argument are treated as if they supported the
Which one of the following most accurately describes a overall conclusion.
flaw in the reasoning above?
(A) Greenwall takes for granted that many customers
have stopped using the equipment but are too
embarrassed to admit it.
(B) Greenwall presumes, without giving justification,
that most people are dishonest about their
personal habits.
(C) Faden presumes, without providing justification,
that the more conclusive the evidence is for a
claim, the less believable the claim becomes.
(D) Faden presumes, without providing justification,
that the evidence for a claim has not been
undermined unless that evidence has been
proven false.
(E) Greenwall ignores the possibility that some
people stopped using the equipment but were not
embarrassed about it.
54. Prosecutor: Dr. Yuge has testified that, had the robbery 56. The consequences of surgical errors can be devastating,
occurred after 1:50 A.M., then, the moon having and no one would want to risk surgery unless it was
set at 1:45 A.M., it would have been too dark performed by someone highly competent to perform
for Klein to recognize the perpetrator. But Yuge surgery. General surgeons have special training and
acknowledged that the moon was full enough expertise that make them extremely competent to
to provide considerable light before it set. And perform surgery. Therefore, surgery at the hands of
we have conclusively shown that the robbery anyone other than a general surgeon involves highly
occurred between 1:15 and 1:30 A.M. So, there undesirable risks.
was enough light for Klein to make a reliable
The reasoning in the argument is flawed because the
identification.
argument fails to consider the possibility that
The prosecutor’s reasoning is most vulnerable to
(A) there are general surgeons who are incompetent
criticism because it overlooks which one of the
(B) general surgeons are not the only doctors
following possibilities?
competent to perform surgery
(A) Klein may be mistaken about the time of the (C) the competence of the doctor performing surgery
robbery and so it may have taken place after the does not guarantee a successful outcome
moon had set. (D) risk is not the only factor in deciding whether to
(B) The perpetrator may closely resemble someone have surgery
who was not involved in the robbery. (E) factors in addition to competence are relevant
(C) Klein may have been too upset to make a reliable when choosing a doctor
identification even in good light.
(D) Without having been there, Dr. Yuge has no way 57. Television allows us to transmit images of ourselves that
of knowing whether the light was sufficient. propagate into space. The earliest of these transmissions
(E) During the robbery the moon’s light may have have by now reached all of our neighboring star
been interfered with by conditions such as cloud systems. None of these transmissions, so far as we
cover. know, has been recognized; we have yet to receive any
messages of extraterrestrial origin. We must conclude
55. A number of Grandville’s wealthiest citizens have been that there is no extraterrestrial intelligence in any of our
criminals. So, since it is of utmost importance that the neighboring star systems.
Grandville Planning Committee be composed solely The reasoning in the argument is questionable because
of individuals whose personal standards of ethics are the argument
beyond reproach, no wealthy person should be appointed
to that committee. (A) fails to provide an adequate definition of the word
“messages”
The argument is most vulnerable to the criticism that it (B) infers that there is no extraterrestrial intelligence
(A) confuses a result with something that is sufficient in neighboring star systems from the lack of
for bringing about that result proof that there is
(B) mistakes a temporal relationship for a causal (C) assigns too little importance to the possibility that
relationship there is extraterrestrial intelligence beyond our
(C) assumes that because a certain action has a neighboring star systems
certain result the person taking that action (D) neglects to mention that some governments
intended that result have sent meticulously prepared messages and
(D) judges only by subjective standards something recordings on spacecraft
that can be readily evaluated according to (E) overlooks the immense probability that most star
objective standards systems are uninhabited
(E) generalizes on the basis of what could be
exceptional cases
58. President of Central Supply Company: Profits are at an 60. People in the tourist industry know that excessive
all-time low this fiscal year because of decreased development of seaside areas by the industry damages
demand for our products. If this situation the environment. Such development also hurts the
continues, the company may have to declare tourist industry by making these areas unattractive to
bankruptcy. So, it is important to prevent any tourists, a fact of which people in the tourist industry
further decrease in profits. Consequently, the only are well aware. People in the tourist industry would
options are to reduce planned expansion or to never knowingly do anything to damage the industry.
eliminate some less profitable existing operations. Therefore, they would never knowingly damage the
seaside environment, and the people who are concerned
Which one of the following most accurately describes a
about damage to the seaside environment thus have
flaw in the company president’s reasoning?
nothing to fear from the tourist industry.
(A) It presumes without giving justification that
The reasoning in the argument is most vulnerable to
survival of the company has been a good thing.
criticism on which one of the following grounds?
(B) It does not take into account that there are
alternatives to declaring bankruptcy. (A) No support is provided for the claim that
(C) It presumes without giving justification that only excessive development hurts the tourist industry.
decreased demand can ever be the cause of (B) That something is not the cause of a problem is
decreased profits. used as evidence that it never coexists with that
(D) It does not allow for the possibility that profits problem.
will decrease only slightly during the next fiscal (C) The argument shifts from applying a
year. characteristic to a few members of a group to
(E) It does not take into account that there may be applying the characteristic to all members of that
other ways to stop the decrease in profits. group.
(D) The possibility that the tourist industry would
59. Knowledge of an ancient language is essential for unintentionally harm the environment is ignored.
reading original ancient documents. Most ancient (E) The argument establishes that a certain state of
historical documents, however, have been translated into affairs is likely and then treats that as evidence
modern languages, so scholars of ancient history can that the state of affairs is inevitable.
read them for their research without learning ancient
languages. Therefore, aspirants to careers as ancient-
history scholars no longer need to take the time to learn
ancient languages.
The argument is vulnerable to criticism on which one of
the following grounds?
(A) It concludes that something is never necessary on
the grounds that it is not always necessary.
(B) A statement of fact is treated as if it were merely
a statement of opinion.
(C) The conclusion is no more than a restatement
of the evidence provided as support of that
conclusion.
(D) The judgment of experts is applied to a matter in
which their expertise is irrelevant.
(E) Some of the evidence presented in support of the
conclusion is inconsistent with other evidence
provided.
61. M: The Greek alphabet must have been invented by 62. Terry: Some actions considered to be bad by our society
some individual who knew the Phoenician writing have favorable consequences. But an action is
system and who wanted to have some way of good only if it has favorable consequences. So,
recording Homeric epics and thereby preserving some actions considered to be bad by our society
expressions of a highly developed tradition of oral are actually good.
poetry. Pat: I agree with your conclusion, but not with the
P: Your hypothesis is laughable! What would have been reasons you give for it. Some good actions
the point of such a person’s writing Homeric actually do not have favorable consequences. But
epics down? Surely a person who knew them no actions considered to be bad by our society
well enough to write them down would not need have favorable consequences, so your conclusion,
to read them; and no one else could read them, that some actions our society considers bad are
according to your hypothesis. actually good, still holds.

P’s argument is vulnerable to which one of the following Which one of the following correctly describes both
criticisms? an error in Terry’s reasoning and an error in Pat’s
reasoning?
(A) It fails to demonstrate that the Phoenician
alphabet alone could have provided the basis for (A) presupposing that if a certain property
the Greek alphabet. distinguishes one type of action from another
(B) It incorrectly assumes that the first text ever type of action, then that property is one of many
written in Greek was a Homeric poem. properties distinguishing the two types of action
(C) It confuses the requirements for a complex oral (B) presupposing that if most actions of a certain type
tradition with the requirements of a written share a certain property, then all actions of that
language. type share that property
(D) It attempts to demonstrate the truth of a (C) presupposing that if a certain property is shared
hypothesis merely by showing that it is possible. by actions of a certain type in a given society,
(E) It overlooks the possibility that the person who then that property is shared by actions of that
invented the Greek alphabet did so with the type in every society
intention of teaching it to others. (D) presupposing that if an action’s having a certain
property is necessary for its being a certain type
of action, then having that property is sufficient
for being that type of action
(E) presupposing that if a certain property is shared
by two types of action, then that property is the
only property distinguishing the two types of
action from actions of other types
63. Newsletter for community-center volunteers: Retired 65. Wirth: All efforts to identify a gene responsible for
persons who regularly volunteer their time to predisposing people to manic-depression have
help others generally display fewer and milder failed. In fact, nearly all researchers now agree
effects of aging than their nonvolunteering that there is no “manic-depression gene.”
contemporaries: in social resources, mental Therefore, if these researchers are right, any
outlook, physical health, economic resources, claim that some people are genetically
and overall functioning, they are found to predisposed to manic-depression is simply false.
be substantially stronger than non-volunteers. Chang: I do not dispute your evidence, but I take issue
Volunteering is often described as doing good with your conclusion. Many of the researchers
works to improve the lives of others. How good you refer to have found evidence that a set of
to know that there is evidence that it can equally several genes is involved and that complex
benefit your own well-being! interactions among these genes produce a
predisposition to manic-depression.
The inference drawn above is unwarranted because
(A) the center has a self-interested motive to attract Which one of the following most accurately expresses
new volunteers Chang’s criticism of Wirth’s argument?
(B) it interprets “well-being” as including the factors (A) It presupposes only one possibility where more
of social and economic resources, mental than one exists.
outlook, physical health, and overall functioning (B) It depends on separate pieces of evidence that
(C) some of those who do not volunteer might be contradict each other.
older than some volunteers and so could not be (C) It relies on the opinion of experts in an area
considered their peers outside the experts’ field of expertise.
(D) growing older might not necessarily result in a (D) It disallows in principle any evidence that would
change in mental outlook disconfirm its conclusion.
(E) those with better resources, health, outlook, and (E) It treats something that is merely unlikely as
functioning are more able to work as volunteers though it were impossible.

64. Premiums for automobile accident insurance are often 66. Letter to the editor: I was shocked to learn that Judge
higher for red cars than for cars of other colors. To Mosston was convicted of criminal assault, but I
justify these higher charges, insurance companies disagree with my fellow citizens who believe that
claim that, overall, a greater percentage of red cars are he should be forced to resign. I have played cards
involved in accidents than are cars of any other color. If with Judge Mosston for many years, and he has
this claim is true, then lives could undoubtedly be saved always shown himself to be fair toward me and
by banning red cars from the roads altogether. our fellow cardplayers. Our judicial system
simply cannot afford to lose any just and fair
The reasoning in the argument is flawed because the judges.
argument
The reasoning in the letter is most vulnerable to the
(A) accepts without question that insurance criticism that it
companies have the right to charge higher
premiums for higher-risk clients (A) confuses duties specific to legal professionals
(B) fails to consider whether red cars cost the same to with the responsibilities of private citizens
repair as cars of other colors (B) insists upon a distinction between “fair” and
(C) ignores the possibility that drivers who drive “just” which cannot plausibly be maintained
recklessly have a preference for red cars (C) makes a general claim about an individual’s
(D) does not specify precisely what percentage of red professional competence based on an
cars are involved in accidents unrepresentative sampling of the individual’s
(E) makes an unsupported assumption that every behavior
automobile accident results in some loss of life (D) treats the violent crime of criminal assault as
if its consequences were no more serious than
winning or losing at cards
(E) asserts a conclusion based on popular opinion
rather than on argumentation
67. By examining the fossilized leaves of any prehistoric 68. Many people do not understand themselves, nor do they
plant, it is possible to determine the climate in which try to gain self-understanding. These people might try to
that specimen grew because the size and shape of a leaf understand others, but these attempts are sure to fail,
are unique to a given climate. Since the climate at a because without self-understanding it is impossible to
given location depends on the altitude at that location, it understand others. It is clear from this that anyone who
follows that the size and shape of a fossilized leaf also lacks self-understanding will be incapable of
indicates the altitude at which the plant grew. understanding others.

The reasoning in the argument is vulnerable to criticism The reasoning in the argument is flawed because the
on the grounds that it argument

(A) fails to demonstrate that no species of plant can (A) mistakes something that is necessary to bring
long survive a violent change in its environment about a situation for something that in itself is
(B) overlooks the possibility that locations at different enough to bring about that situation
altitudes can have the same climate (B) fails to take into account the possibility that not
(C) treats the size and shape of a leaf as if they were everyone wants to gain a thorough understanding
the only physical characteristics of a leaf that of himself or herself
depend on climate (C) blames people for something for which they
(D) relies on a weak analogy between a leaf and cannot legitimately be held responsible
the fossil of a leaf as evidence for the claims (D) makes use of the inherently vague term “self-
advanced understanding” without defining that term
(E) ignores the possibility that the location at which (E) draws a conclusion that simply restates a claim
a fossilized leaf was found is not the location at given in support of that conclusion
which the original plant grew
69. Politician: Critics of the wetlands-protection bill are 70. Political advocate: Campaigns for elective office should
delaying passage of this important legislation be subsidized with public funds. One reason is
merely on the grounds that they disagree with its that this would allow politicians to devote less
new, more restrictive definition of the term time to fund-raising, thus giving campaigning
“wetlands.” But this bill will place stricter limits incumbents more time to serve the public. A
on the development of wetlands than the existing second reason is that such subsidies would make
regulations do. Therefore, in quibbling over it possible to set caps on individual campaign
semantics, critics of this bill show that they care contributions, thereby reducing the likelihood
little about what really happens to our wetlands. that elected officials will be working for the
benefit not of the public but of individual large
The politician’s reply to the opponents of the wetlands-
contributors.
protection bill is most vulnerable to which one of the
following criticisms? Critic: This argument is problematic: the more the caps
constrain contributions, the more time candidates
(A) It falsely identifies the motives of those who
have to spend finding more small contributors.
have criticized the wetlands-protection bill with
the motives of all those who are opposed to The critic objects that the advocate’s argument is flawed
conservation. because
(B) It does not adequately recognize the possibility
(A) any resourceful large contributor can circumvent
that the definition of the word “wetlands”
caps on individual contributions by sending in
determines the impact of the legislation.
smaller amounts under various names
(C) It assumes without justification that those who
(B) one of the projected results cited in support of
criticized the wetlands-protection bill stand to
the proposal made is entailed by the other
profit if the bill is defeated.
and therefore does not constitute independent
(D) It fails to provide a defense for a less restrictive
support of the proposal
definition of “wetlands.”
(C) of the two projected results cited in support of the
(E) It attempts to defend the credibility of the author
proposal made, one works against the other
of the bill rather than defending the bill itself.
(D) it overlooks the possibility that large contributors
will stop contributing if they cannot contribute at
will
(E) it overlooks the possibility that incumbents with
a few extremely generous contributors will be
hit harder by caps than incumbents with many
moderately generous contributors
71. On Saturday Melvin suggested that Jerome take the 72. In a learning experiment a researcher ran rats through a
following week off from work and accompany him on a maze. Some of the rats were blind, others deaf, others
trip to the mountains. Jerome refused, claiming that he lacked a sense of smell, and others had no sensory
could not afford the cost of the trip added to the wages deficiencies; yet all the rats learned the task in much the
he would forfeit by taking off without notice. It is clear, same amount of time. Of the senses other than sight,
however, that cost cannot be the real reason for Jerome’s hearing, and smell, only kinesthesia had not previously
unwillingness to go with Melvin to the mountains, since been shown to be irrelevant to maze-learning. The
he makes the same excuse every time Melvin asks him researcher concluded on the basis of these facts that
to take an unscheduled vacation regardless of where kinesthesia, the sensation of bodily movement, is
Melvin proposes to go. sufficient for maze-learning.

The reasoning is most vulnerable to which one of the The researcher’s reasoning is most vulnerable to which
following criticisms? one of the following criticisms?

(A) It attempts to forestall an attack on Melvin’s (A) The small differences in proficiency found by
behavior by focusing attention on the behavior the researcher did not appear to fall into a
of Jerome. systematic pattern by group.
(B) It fails to establish that Melvin could no more (B) The possibility that the interaction of kinesthesia
afford to take an unscheduled vacation trip to the with at least one other sense is required for
mountains than could Jerome. maze-learning cannot be ruled out on the basis
(C) It overlooks the possibility that Jerome, unlike of the data above.
Melvin, prefers vacations that have been planned (C) It can be determined from the data that rats who
far in advance. are deprived of one of their sources of sensory
(D) It assumes that if Jerome’s professed reason is not stimulation become more reliant on kinesthesia
his only reason, then it cannot be a real reason than they had been, but the data do not indicate
for Jerome at all. how such a transference takes place.
(E) It does not examine the possibility that Jerome’s (D) It can be determined from the data that rats can
behavior is adequately explained by the reason learn to run mazes by depending on kinesthesia
he gives for it. alone, but the possibility that rats respond to
non-kinesthetic stimulation is not ruled out.
(E) It can be determined from the data that
maze-learning in rats depends on at least two
sources of sensory stimulation, one of which is
kinesthesia, but which of the remaining sources
must also be employed is not determinable.
73. The people most likely to watch a televised debate 75. Pundit: People complain about how ineffectual their
between political candidates are the most committed legislative representatives are, but this apparent
members of the electorate and thus the most likely to ineffectuality is simply the manifestation of
have already made up their minds about whom to compromises these representatives must make
support. Furthermore, following a debate, uncommitted when they do what they were elected to do:
viewers are generally undecided about who won the compete for the government’s scarce funds. So,
debate. Hence, winning a televised debate does little to when people express dissatisfaction with their
bolster one’s chances of winning an election. legislative representatives, we can be assured that
The reasoning in the argument is most vulnerable to these representatives are simply doing what they
criticism because the argument fails to consider the were elected to do.
possibility that The pundit’s argument is flawed because it takes for
(A) watching an exciting debate makes people more granted that
likely to vote in an election (A) the apparent ineffectuality of legislative
(B) the voting behavior of people who do not watch representatives is the only source of popular
a televised debate is influenced by reports about dissatisfaction with those representatives
the debate (B) governmental resources that are currently scarce
(C) there are differences of opinion about what cannot become more abundant except by the
constitutes winning or losing a debate actions of politicians
(D) people’s voting behavior may be influenced in (C) constituents would continue to be dissatisfied
unpredictable ways by comments made by the with the effectuality of their legislative
participants in a televised debate representatives if constituents were aware of the
(E) people who are committed to a particular cause of this apparent ineffectuality
candidate will vote even if their candidate is (D) legislative compromise inevitably results in
perceived as having lost a televised debate popular dissatisfaction with politicians
(E) only elected public servants tend to elicit
74. In a car accident, air bags greatly reduce the risk of dissatisfaction among the public
serious injury. However, statistics show that cars without
air bags are less likely to be involved in accidents than 76. Cotrell is, at best, able to write magazine articles of
are cars with air bags. Thus, cars with air bags are no average quality. The most compelling pieces of evidence
safer than cars without air bags. for this are those few of the numerous articles submitted
by Cotrell that are superior, since Cotrell, who is
The argument is most vulnerable to criticism on the incapable of writing an article that is better than average,
grounds that it must obviously have plagiarized superior ones.
(A) assumes, without providing justification, that any The argument is most vulnerable to criticism on which
car with air bags will probably become involved one of the following grounds?
in an accident (A) It simply ignores the existence of potential
(B) denies the possibility that cars without air bags counterevidence.
have other safety features that reduce the risk of (B) It generalizes from atypical occurrences.
serious injury at least as much as do air bags (C) It presupposes what it seeks to establish.
(C) overlooks the possibility that some accidents (D) It relies on the judgment of experts in a matter to
involve both cars with air bags and cars without which their expertise is irrelevant.
air bags (E) It infers limits on ability from a few isolated
(D) assumes, without providing justification, that lapses in performance.
the likelihood of an accident’s occurring should
weigh at least as heavily as the seriousness
of any resulting injury in estimates of relative
safety
(E) takes for granted that all accidents would cause
air bags to be deployed
77. Anthropologist: All music is based on a few main 79. Philosopher: Scientists talk about the pursuit of truth,
systems of scale building. Clearly, if the but, like most people, they are self-interested.
popularity of a musical scale were a result of Accordingly, the professional activities of most
social conditioning, we would expect, given the scientists are directed toward personal career
diversity of social systems, a diverse mixture of
diatonic and non-diatonic scales in the world’s enhancement, and only incidentally toward the
music. Yet diatonic scales have always dominated pursuit of truth. Hence, the activities of the
the music of most of the world. Therefore, the scientific community are largely directed toward
popularity of diatonic music can be attributed enhancing the status of that community as a
only to innate dispositions of the human mind. whole, and only incidentally toward the pursuit of
truth.
The anthropologist’s argument is most vulnerable to
criticism on the grounds that it fails to The reasoning in the philosopher’s argument is flawed
because the argument
(A) consider the possibility that some people
appreciate non-diatonic music more than they (A) improperly infers that each and every scientist
do diatonic music has a certain characteristic from the premise that
(B) explain how innate dispositions increase most scientists have that characteristic
appreciation of non-diatonic music (B) improperly draws an inference about the scientific
(C) explain the existence of diatonic scales as well as community as a whole from a premise about
the existence of non-diatonic scales individual scientists
(D) consider that innate dispositions and social (C) presumes, without giving justification, that the
conditioning could jointly affect the popularity aim of personal career enhancement never
of a type of music advances the pursuit of truth
(E) consider whether any appreciation of non- (D) illicitly takes advantage of an ambiguity in the
diatonic music is demonstrated by some meaning of “self-interested”
nonhuman species of animals (E) improperly draws an inference about a cause
from premises about its effects
78. Attorney: I ask you to find Mr. Smith guilty of
assaulting Mr. Jackson. Regrettably, there were 80. Supervisor: Our next budget proposal will probably be
no eyewitnesses to the crime, but Mr. Smith has a approved, because normally about half of all
violent character: Ms. Lopez testified earlier that budget proposals that the vice president considers
Mr. Smith, shouting loudly, had threatened her. are approved, and our last five budget proposals
Smith never refuted this testimony. have all been turned down.
The attorney’s argument is fallacious because it reasons The supervisor’s reasoning is flawed because it
that presumes, without giving warrant, that
(A) aggressive behavior is not a sure indicator of a (A) the last five budget proposals’ having been turned
violent character down guarantees that the next five budget
(B) Smith’s testimony is unreliable since he is loud proposals will be approved
and aggressive (B) the vice president is required to approve at least
(C) since Smith never disproved the claim that he half of all budget proposals submitted
threatened Lopez, he did in fact threaten her (C) having the last five budget proposals turned
(D) Lopez’s testimony is reliable since she is neither down affects the likelihood that the next budget
loud nor aggressive proposal will be turned down
(E) having a violent character is not necessarily (D) the majority of the last five budget proposals
associated with the commission of violent deserved to be turned down
crimes (E) the likelihood that a budget proposal will be
approved is influenced by the amount of money
that budget proposal requests
81. A survey of historians shows that most believe written 83. All actions are motivated by self-interest, since any action
texts to be the best source for historical understanding. that is apparently altruistic can be described in terms of
None of the historians regarded painting, architecture, self-interest. For example, helping someone can be
music, dance, or culinary arts as the best source for described in terms of self-interest: the motivation is hope
historical understanding. So, these historians neglect for a reward or other personal benefit to be bestowed as a
many important repositories of historical knowledge. result of the helping action.

The reasoning in the argument is flawed because the Which one of the following most accurately describes an
argument takes for granted that error in the argument’s reasoning?

(A) there are no potential sources for historical (A) The term “self-interest” is allowed to shift in
understanding other than written texts and the meaning over the course of the argument.
arts (B) The argument takes evidence showing merely
(B) painting, architecture, music, dance, and culinary that its conclusion could be true to constitute
arts are important only as sources for historical evidence showing that the conclusion is in
understanding fact true.
(C) there are no sources for historical understanding (C) The argument does not explain what is meant by
that are neither considered best by historians nor “reward” and “personal benefit.”
neglected by them (D) The argument ignores the possibility that what is
(D) something other than written texts is the best taken to be necessary for a certain interest to be
source for historical understanding a motivation actually suffices to show that that
(E) the other sources for historical understanding interest is a motivation.
mentioned by the historians surveyed are not (E) The argument depends for its appeal only on the
important repositories of historical knowledge emotional content of the example cited.

82. In Debbie’s magic act, a volunteer supposedly selects a 84. When interviewing job candidates, personnel managers
card in a random fashion, looks at it without showing it not only evaluate a candidate’s work experience and
to her, and replaces it in the deck. After several shuffles, educational background but also inquire about hobbies.
Debbie cuts the deck and supposedly reveals the same Personnel managers try to justify these inquiries by
selected card, A skeptic conducted three trials. In the noting that the enthusiasm someone shows for a hobby
first, Debbie was videotaped, and no sleight of hand was may well carry over to enthusiasm for a job. But such
found. In the second, the skeptic instead supplied a enthusiasm may also indicate that the candidate is
standard deck of cards. For the third trial, the skeptic less concerned with work than with play.
selected the card. Each time, Debbie apparently revealed Therefore, personnel managers should not inquire
the selected card. The skeptic concluded that Debbie about a candidate’s hobbies.
uses neither sleight of hand, nor a trick deck, nor a The argument is flawed because it overlooks each of the
planted “volunteer” to achieve her effect. following possibilities EXCEPT:
Which one of the following most accurately describes a
(A) A candidate’s involvement in particular hobbies
flaw in the skeptic’s reasoning?
may indicate a capacity to make long-term
(A) The skeptic failed to consider the possibility that commitments.
Debbie did not always use the same method to (B) Candidates who have no hobbies may pretend
achieve her effect. that they have one when asked in an interview.
(B) The skeptic failed to consider the possibility that (C) Inquiries about a hobby may put candidates at
sleight of hand could also be detected by some ease, eliciting more honest responses about
means other than videotaping. important questions.
(C) The skeptic failed to consider the possibility that (D) Having certain kinds of hobbies may indicate that
Debbie requires both sleight of hand and a trick a candidate has good organizational skills.
deck to achieve her effect. (E) Personnel managers may make better choices
(D) The skeptic failed to consider the possibility that among candidates if they are not restricted from
Debbie used something other than sleight of asking particular types of questions.
hand, a trick deck, or a planted “volunteer” to
achieve her effect.
(E) The skeptic failed to consider the possibility
that Debbie’s success in the three trials was
something other than a coincidence.
85. Parents who wish to provide a strong foundation for the 87. Attorney for Ziegler: My client continued to do
musical ability of their children should provide them consulting work between the time of his arrest for
with a good musical education. Since formal instruction attempted murder and the start of this trial. But I
is often a part of a good musical education, parents who contend that Ziegler was insane at the time that he
wish to provide this strong foundation need to ensure fired the shot. This is the only reasonable
that their children receive formal instruction. conclusion to draw from the fact that the accusers
The reasoning is most vulnerable to criticism on the have submitted no evidence that he was sane at
grounds that it fails to consider that the time he pulled the trigger, only that he was
sane some time after he did so.
(A) parents might not be the only source of a child’s
musical education Which one of the following most accurately describes a
(B) some children might not be interested in receiving flaw in the reasoning of Ziegler’s attorney?
a strong foundation for their musical ability (A) It presumes that being a well-educated
(C) there are many examples of people with formal professional is relevant to being guilty or
instruction whose musical ability is poor innocent.
(D) formal instruction might not always be a part of a (B) It concludes on the basis of evidence against
good musical education Ziegler’s being sane that there is a lack of
(E) some children might become good musicians evidence for Ziegler’s being sane.
even if they have not had good musical (C) It fails to consider that Ziegler might have been
educations insane when he worked as a consultant.
(D) It presumes that whether one is sane is relevant
86. Tires may be either underinflated, overinflated, to whether one is morally responsible for one’s
or neither. We are pretty safe in assuming that actions.
underinflation or overinflation of tires harms their tread. (E) It fails to consider the possibility that Ziegler’s
After all, no one has been able to show that these do not being sane after the shooting is an indication
harm tire tread. that he was sane at the time of the shooting.
Which one of the following most accurately describes a
flaw in the argument’s reasoning? 88. Commissioner: I have been incorrectly criticized for
having made my decision on the power plant
(A) The argument assumes what it is attempting to issue prematurely. I based my decision on the
demonstrate. report prepared by the neighborhood association
(B) The argument overlooks that what is not in and, although I have not studied it thoroughly,
principle susceptible to proof might be false. I am sure that the information it contains
(C) The argument fails to specify how it is that is accurate. Moreover, you may recall that
underinflation or overinflation harms tire tread. when I received input from the neighborhood
(D) The argument rejects the possibility that what has association on jail relocation, I agreed with its
not been proven is nevertheless true. recommendation.
(E) The argument fails to precisely define the terms
“underinflation” and “overinflation.” The commissioner’s argument is LEAST vulnerable to
which one of the following criticisms?
(A) It takes for granted that the association’s
information is not distorted by bias.
(B) It draws a conclusion about the recommendations
of the association from incomplete recollections.
(C) It takes for granted that the association’s report
is the only direct evidence that needed to be
considered.
(D) It hastily concludes that the association’s report is
accurate, without having studied it in detail.
(E) It takes for granted that agreeing with the
association’s past recommendation
helps to justify agreeing with its current
recommendation.
89. Editorialist: Society is obligated to bestow the 91. Economist: In order to decide what to do about
privileges of adulthood upon its members once protecting the ozone layer, we must determine
they are mature enough to accept the the monetary amount of the economic resources
corresponding responsibilities. But science has that we would willingly expend to protect it.
established that physiological development is Such a determination amounts to a calculation
completed in most persons by age seventeen. of the monetary value of the ozone layer.
Since this maturing process has been completed Environmentalists argue that the ozone layer does
by most seventeen-year-olds, there is no reason not have a calculable monetary value. However,
not to grant these citizens all of the privileges of we would not willingly expend an amount
adulthood. equal to all of the world’s economic resources
to protect the ozone layer, so the ozone layer is
The editorialist’s argument is most vulnerable to
demonstrably worth less than that amount. Thus,
criticism on the ground that it
the ozone layer has a calculable monetary value.
(A) assumes what it is trying to prove
The reasoning in the economist’s argument is flawed in
(B) too hastily reaches a general conclusion on the
that the argument
basis of a few examples
(C) equivocates with respect to a central concept (A) uses evidence that the monetary value of a
(D) too readily accepts a claim by appeal to particular natural resource is less than a certain
inappropriate authority amount in order to establish that the monetary
(E) ignores the fact that some people are mature at value of any natural resource is less than that
age sixteen amount
(B) presupposes that the ozone layer should not be
90. Press release: A comprehensive review evaluating the protected and then argues to that claim as a
medical studies done up to the present time has conclusion
found no reason to think that drinking coffee in (C) takes advantage of an ambiguity in the term
normal amounts harms the coffee-drinker’s “value” to deflect the environmentalists’ charge
heart. So, coffee drinkers can relax and enjoy their (D) gives no reason for thinking that merely
beverage—it is safe to drink coffee. establishing an upper limit on a certain monetary
value would allow the calculation of that
Which one of the following points to a weakness in the
monetary value
reasoning in the press release’s argument?
(E) does not directly address the argument of the
(A) The review was only an evaluation of studies and environmentalists
did not itself undertake to study patients.
(B) The health of the heart is not identical with the 92. Music critic: Some people argue that, unlike certain
general health of the body. works of Handel, which set to music familiar
(C) Coffee drinkers might choose to eat, along with religious texts, the organ symphonies of Louis
their coffee, foods containing substances that Vieme are not religious music. Quite the contrary.
harm the heart. Sitting in Notre Dame cathedral in Paris and
(D) Other beverages besides coffee might contain hearing his organ symphonies demonstrates that
stimulants that have some effect on the heart. Vierne’s works are divinely inspired.
(E) Drinking unusually large amount of coffee could
The music critic’s reasoning is vulnerable to criticism on
be caused by stress that itself directly harms the
the ground that it
heart.
(A) takes for granted that all religious music is
inspiring
(B) confuses two different meanings of the term
“religious”
(C) overlooks the possibility that some organ music is
not divinely inspired
(D) confuses two different meanings of the term
“symphonies”
(E) takes for granted that all organ symphonies are
religious music
93. Mr. Nance: Ms. Chan said that she retired from Quad 95. In the first decade following the founding of the British
Cities Corporation, and had received a watch and Labour party, the number of people regularly voting
a wonderful party as thanks for her 40 years of for Labour increased fivefold. The number of committed
loyal service. But I overheard a colleague of hers Labour voters increased a further fivefold during the
say that Ms. Chan will be gone for much of the party’s second decade. Since the increase was thus the
next year on business trips and is now working same in the first as in the second decade, the often-made
harder than she ever did before; that does not claim that the Labour party gained more voters in the
sound like retirement to me. At least one of them party’s second decade than in its first is clearly false.
is not telling the truth. The reasoning in the argument is flawed because the
argument
Mr. Nance’s reasoning is flawed because it
(A) fails to specify dates necessary to evaluate
(A) is based in part on hearsay the truth of the conclusion, even though the
(B) criticizes Ms. Chan rather than the claims she argument depends on distinguishing between
made two time periods
(C) draws a conclusion based on equivocal language (B) draws a conclusion that cannot be true if all the
(D) fails to consider that Ms. Chan’s colleague may data advanced in its support are true
have been deceived by her (C) relies on statistical evidence that, strictly
(E) fails to infer that Ms. Chan must be a person of speaking, is irrelevant to establishing the
superior character, given her long loyal service conclusion drawn
(D) fails to allow for the possibility that the policy
94. Antinuclear activist: The closing of the nuclear power positions advocated by the Labour party changed
plant is a victory for the antinuclear cause. It during the period in question
also represents a belated acknowledgment by the (E) overlooks the possibility that more elections were
power industry that they cannot operate such held in one of the two decades than were held in
plants safely. the other
Nuclear power plant manager: It represents no such
thing. The availability of cheap power from 96. Yolanda: Gaining access to computers without
nonnuclear sources, together with the cost of authorization and manipulating the data and
mandated safety inspections and safety repairs, programs they contain is comparable to joyriding
made continued operation uneconomic. Thus in stolen cars; both involve breaking into private
it was not safety considerations but economic property and treating it recklessly. Joyriding,
considerations that dictated the plant’s closing. however, is the more dangerous crime because
it physically endangers people, whereas only
The reasoning in the manager’s argument is flawed intellectual property is harmed in the case of
because the argument computer crimes.
(A) fails to acknowledge that the power industry Arjun: I disagree! For example, unauthorized use of
might now believe nuclear power plants to be medical records systems in hospitals could
unsafe even though this plant was not closed for damage data systems on which human lives
safety reasons
depend, and therefore computer crimes also cause
(B) overlooks the possibility that the sources physical harm to people.
from which cheap power is available might
themselves be subject to safety concerns The reasoning in Arjun’s response is flawed because he
(C) mistakes the issue of what the closure of the plant (A) fails to maintain a distinction made in Yolanda’s
represents to the public for the issue of what the argument
managers’ reasons for the closure were (B) denies Yolanda’s conclusion without providing
(D) takes as one of its premises a view about the evidence against it
power industry’s attitude toward nuclear safety (C) relies on the actuality of a phenomenon that he
that contradicts the activist’s view has only shown to be possible
(E) counts as purely economic considerations some (D) mistakes something that leads to his conclusion
expenses that arise as a result of the need to take for something that is necessary for his
safety precautions conclusion
(E) uses as evidence a phenomenon that is
inconsistent with his own conclusion
97. Louis: People’s intentions cannot be, on the whole, 99. Joshua Smith’s new novel was criticized by the book
more bad than good. Were we to believe editor for The Daily Standard as implausible. That
otherwise, we would inevitably cease to trust criticism, like so many other criticisms from the same
each other, and no society can survive without source in the past, is completely unwarranted. As anyone
mutual trust among its members. who has actually read the novel would agree, each one
of the incidents in which Smith’s hero gets involved is
The argument is most vulnerable to which one of the
the kind of incident that could very well have happened
following criticisms?
to someone or other.
(A) It fails to rule out the possibility that a true belief
Which one of the following is the most serious error of
can have deleterious consequences.
reasoning in the argument?
(B) It mistakenly assumes that if two claims cannot at
the same time both be true, then they cannot at (A) It relies on the assumption that a criticism can
the same time both be false. legitimately be dismissed as unwarranted if it
(C) It challenges the truth of a claim merely by is offered by someone who had previously
calling into question the motives of those who displayed questionable judgment.
profess that they believe it to be true. (B) It ignores the fact that people can agree about
(D) It assumes without warrant that in any situation something even though what they agree about is
with two possible outcomes, the most negative not the case.
one will inevitably occur. (C) It calls into question the intellectual integrity of
(E) It provides no reason to believe that a statement the critic in order to avoid having to address the
that is true of a given group of individuals is also grounds on which the criticism is based.
true of any other group of individuals. (D) It takes for granted that a whole story will have a
given characteristic if each of its parts has that
98. Morton: In order to succeed in today’s society, one must characteristic.
have a college degree. Skeptics have objected (E) It attempts to justify its conclusion by citing
that there are many people who never completed reasons that most people would find plausible
any education beyond high school but who are only if they were already convinced that the
nevertheless quite successful. This success is only conclusion was true.
apparent, however, because without a college
degree a person does not have enough education 100. The senator has long held to the general principle that no
to be truly successful. true work of art is obscene, and thus that there is no
conflict between the need to encourage free artistic
Morton’s argument is flawed because it expression and the need to protect the sensibilities of the
(A) assumes what it sets out to conclude public from obscenity. When well-known works generally
(B) mistakes a correlation for a cause viewed as obscene are cited as possible counterexamples,
(C) draws a highly general conclusion from evidence the senator justifies accepting the principle by saying that
about individual cases if these works really are obscene then they cannot be
(D) fails to consider the status of alleged works of art.
counterexamples The senator’s reasoning contains which one of the
(E) bases its conclusion on the supposition that most following errors?
people believe in that conclusion
(A) It seeks to persuade by emotional rather than
intellectual means.
(B) It contains an implicit contradiction.
(C) It relies on an assertion of the senator’s authority.
(D) It assumes what it seeks to establish.
(E) It attempts to justify a position by appeal to an
irrelevant consideration.
101. The true scientific significance of a group of unusual
fossils discovered by the paleontologist Charles Walcott
is more likely to be reflected in a recent classification
than it was in Walcott’s own classification. Walcott was,
after all, a prominent member of the scientific
establishment. His classifications are thus unlikely to
have done anything but confirm what established science
had already taken to be true.
Which one of the following most accurately describes a
questionable technique used in the argument?
(A) It draws conclusions about the merit of a position
and about the content of that position from
evidence about the position’s source.
(B) It cites two pieces of evidence, each of which is
both questionable and unverifiable, and uses this
evidence to support its conclusions.
(C) It bases a conclusion on two premises that
contradict each other and minimizes this
contradiction by the vagueness of the terms
employed.
(D) It attempts to establish the validity of a claim,
which is otherwise unsupported, by denying the
truth of the opposite of that claim.
(E) It analyzes the past on the basis of social and
political categories that properly apply only to
the present and uses the results of this analysis
to support its conclusion.
102. Driver: My friends say I will one day have an accident 104. Commentator: Human behavior cannot be fully
because I drive my sports car recklessly. But understood without inquiring into nonphysical
I have done some research, and apparently aspects of persons. As evidence of this, I submit
minivans and larger sedans have very low the following: suppose that we had a complete
accident rates compared to sports cars. So, scientific account of the physical aspects of some
trading my sports car in for a minivan would particular human action—every neurological,
lower my risk of having an accident. physiological, and environmental event involved.
Even with all that we would obviously still not truly
The reasoning in the driver’s argument is most
comprehend the action or know why it occurred.
vulnerable to criticism on the grounds that this argument
Which one of the following most accurately describes a
(A) infers a cause from a mere correlation
flaw in the argument’s reasoning?
(B) relies on a sample that is too narrow
(C) misinterprets evidence that a result is likely as (A) No support is offered for its conclusion other than
evidence that the result is certain an analogy that relates only superficially to the
(D) mistakes a condition sufficient for bringing about issue at hand.
a result for a condition necessary for doing so (B) The purported evidence that it cites in support of its
(E) relies on a source that is probably not well- conclusion presumes that the conclusion is true.
informed (C) It concludes that a proposition must be true merely
on the grounds that it has not been proven false.
103. New Age philosopher: Nature evolves organically (D) It fails to indicate whether the speaker is aware of
and nonlinearly. Furthermore, it can best any evidence that could undermine the conclusion.
be understood as a whole; its parts are so (E) It presumes, without providing justification, that
interconnected that none could exist without science can provide a complete account of any
support from many others. Therefore, attaining physical phenomenon
the best possible understanding of nature requires
an organic, holistic, nonlinear way of reasoning 105. Counselor: Constantly comparing oneself to those one
rather than the traditional linear reasoning of sees as more able or more successful almost
science, which proceeds through experiments on invariably leads to self-disparagement. Conversely,
deliberately isolated parts of nature. constantly comparing oneself to those one sees as
less able or less successful almost invariably leads
The reasoning in the New Age philosopher’s argument
to being dismissive of others. So, those who for
is most vulnerable to criticism on the grounds that the
the most part refrain from comparing themselves
argument
to others will most likely be, on the whole, self-
(A) takes for granted that if a statement must be true accepting and accepting of others.
for the argument’s conclusion to be true, then
The counselor’s reasoning is most vulnerable to
that statement’s truth is sufficient for the truth of
criticism because it
the conclusion
(B) overlooks the possibility that the overall structure (A) overlooks the possibility that one can compare
of a phenomenon is not always identical to the oneself both to those one perceives to be more
overall structure of the reasoning that people do able and more successful than oneself and to
about that phenomenon those one perceives to be less able and less
(C) fails to distinguish adequately between the successful than oneself
characteristics of a phenomenon as a whole and (B) overlooks the possibility that constantly
those of the deliberately isolated parts of that comparing oneself to others may have beneficial
phenomenon effects that those who refrain from making such
(D) takes for granted that what is interconnected comparisons are deprived of
cannot, through abstraction, be thought of (C) takes for granted that if one is both dismissive of
as separate others and self-disparaging, one will not be self-
(E) takes for granted that a phenomenon that can best accepting and accepting of others
be understood as having certain properties can (D) overlooks the possibility that self-disparagement
best be understood only through reasoning that and being dismissive of others can result from
shares those properties something other than comparing oneself to others
(E) takes for granted that whenever one compares
oneself to others one sees them as more
successful and more able than oneself or less
successful and less able than oneself
106. Television network executive: Some scientists have 108. Media consultant: Electronic media are bound to bring
expressed concern about the numerous highly an end to the institution of the traditional school
popular television programs that emphasize in our culture. This is because the emergence of
the traditional school, characterized by a group of
paranormal incidents, warning that these students gathered with a teacher in a classroom,
programs will encourage superstition and thereby was facilitated by the availability of relatively
impede the public’s scientific understanding. inexpensive printed books. Currently, however,
But these predictions are baseless. Throughout the function of books in communicating
recorded history, dramatists have relied on ghosts information is gradually being taken over by
and spirits to enliven their stories, and yet the electronic media. So, it is inevitable that the
scientific understanding of the populace has traditional school will not survive in our culture.
steadily advanced.
The reasoning in the consultant’s argument is flawed
The television network executive’s argument is most because it
vulnerable to criticism on which one of the following
(A) presupposes as a premise what it is trying to
grounds?
establish
(A) It fails to consider that one phenomenon can (B) relies inappropriately on expert testimony
steadily advance even when it is being impeded (C) presupposes that just because something can
by another phenomenon. happen it will happen
(B) It takes for granted that if a correlation has been (D) mistakes something that enables an institution to
observed between two phenomena, they must be arise for something necessary to the institution
causally connected. (E) confuses the value of an institution with the
(C) It fails to consider that the occurrence of medium by which it operates
one phenomenon can indirectly affect the
pervasiveness of another even if the former does 109. A year ago, the government reduced the highway speed
not impede the latter. limit, and in the year since, there have been significantly
(D) It fails to consider that just because one fewer highway fatalities than there were in the previous
phenomenon is known to affect another, the year. Therefore, speed limit reduction can reduce traffic
latter does not also affect the former. fatalities.
(E) It takes for granted that the contention that one The argument is most vulnerable to the criticism that it
phenomenon causes another must be baseless takes for granted that
if the latter phenomenon has persisted despite
steady increases in the pervasiveness of the (A) highway traffic has not increased over the
former. past year
(B) the majority of drivers obeyed the new
107. If violations of any of a society’s explicit rules routinely speed limit
go unpunished, then that society’s people will be left (C) there is a relation between driving speed and the
F number of automobile accidents
L without moral guidance. Because people who lack moral
guidance will act in many different ways, chaos results. (D) the new speed limit was more strictly enforced
Thus, a society ought never to allow any of its explicit than the old
rules to be broken with impunity. (E) the number of traffic fatalities the year before
the new speed limit was introduced was not
The reasoning in the argument is most vulnerable to abnormally high
criticism on the grounds that the argument
(A) takes for granted that a society will avoid chaos
as long as none of its explicit rules are routinely
violated with impunity
(B) fails to consider that the violated rules might have
been made to prevent problems that would not
arise even if the rules were removed
(C) infers, from the claim that the violation of some
particular rules will lead to chaos, that the
violation of any rule will lead to chaos
(D) confuses the routine non-punishment of
violations of a rule with sometimes not
punishing violations of the rule
(E) takes for granted that all of a society’s explicit
rules result in equally serious consequences
when broken
110. Sandy: I play the Bigbucks lottery—that’s the one where 112. Some statisticians claim that the surest way to increase
you pick five numbers and all the players who the overall correctness of the total set of one’s beliefs is:
have picked the five numbers drawn at the end of never change that set, except by rejecting a belief when
the week share the money pot. But it’s best to play given adequate evidence against it. However, if this
only after there have been a few weeks with no were the only rule one followed, then whenever one
winners, because the money pot increases each were presented with any kind of evidence, one would
week that there is no winner. have to either reject some of one’s beliefs or else leave
one’s beliefs unchanged. But then, over time, one could
Alex: No, you’re more likely to win the lottery when
only have fewer and fewer beliefs. Since we need many
the money pot is small, because that’s when the
beliefs in order to survive, the statisticians’ claim must
fewest other people are playing.
be mistaken.
Which one of the following most accurately describes a
The argument is most vulnerable to criticism on the
mistake in the reasoning of one of the two speakers?
grounds that it
(A) Sandy holds that the chances of anyone’s winning
(A) presumes, without providing any justification,
are unaffected by the number of times that
that the surest way of increasing the overall
person plays.
correctness of the total set of one’s beliefs must
(B) Alex holds that the chances of Sandy’s winning
not hinder one’s ability to survive
are affected by the number of other people
(B) neglects the possibility that even while following
playing.
the statisticians’ rule, one might also accept
(C) Sandy holds that the chances of anyone’s winning
new beliefs when presented with some kinds of
are unaffected by the size of the pot.
evidence
(D) Alex holds that the chances of Sandy’s winning in
(C) overlooks the possibility that some large sets of
a given week are unaffected by whether anyone
beliefs are more correct overall than are some
has won the week before.
small sets of beliefs
(E) Sandy holds that the chances of there being a
(D) takes for granted that one should accept some
winner go up if no one has won the lottery for
beliefs related to survival even when given
quite a while.
adequate evidence against them
(E) takes for granted that the beliefs we need in order
111. On the basis of the available evidence, Antarctica has
generally been thought to have been covered by ice for to have many beliefs must all be correct beliefs
at least the past 14 million years. Recently, however,
three-million-year-old fossils of a kind previously found
only in ocean-floor sediments were discovered under the
ice sheet covering central Antarctica. About three
million years ago, therefore, the Antarctic ice sheet must
temporarily have melted. After all, either severe climatic
warming or volcanic activity in Antarctica’s mountains
could have melted the ice sheet, thus raising sea levels
and submerging the continent.
The reasoning in the argument is most vulnerable to
which one of the following criticisms?
(A) That a given position is widely believed to be true
is taken to show that the position in question
must, in fact, be true.
(B) That either of two things could independently
have produced a given effect is taken to show
that those two things could not have operated in
conjunction to produce that effect.
(C) Establishing that a certain event occurred is
confused with having established the cause of
that event.
(D) A claim that has a very general application is
based entirely on evidence from a narrowly
restricted range of cases.
(E) An inconsistency that, as presented, has more
than one possible resolution is treated as though
only one resolution is possible.
113. A recent study of 6,403 people showed that those treated 115. Some psychologists claim that, in theory, the best way
with the drug pravastatin, one of the effects of which is to understand another person would be through deep
to reduce cholesterol, had about one-third fewer nonfatal empathy, whereby one would gain a direct and complete
heart attacks and one-third fewer deaths from coronary
disease than did those not taking the drug. This result is grasp of that person’s motivations. But suppose they
consistent with other studies, which show that those who are right; then there would be no way at all to achieve
have heart disease often have higher than average understanding, since it is psychologically impossible
cholesterol levels. This shows that lowering cholesterol to gain a direct and complete grasp of another person’s
levels reduces the risk of heart disease. motivations. But obviously one can understand other
people; thus, these psychologists are wrong.
The argument’s reasoning is flawed because the
argument The argument is most vulnerable to the criticism that it
(A) neglects the possibility that pravastatin may have (A) fails to adequately define the key phrase “deep
severe side effects empathy”
(B) fails to consider that pravastatin may reduce the (B) assumes something that it later denies, resulting
risk of heart disease but not as a consequence of in a contradiction
its lowering cholesterol levels (C) confuses a theoretically best way of
(C) relies on past findings, rather than drawing its accomplishing something with the only way of
principal conclusion from the data found in the accomplishing it
specific study cited (D) accepts a claim on mere authority, without
(D) draws a conclusion regarding the effects of requiring sufficient justification
lowering cholesterol levels on heart disease, (E) fails to consider that other psychologists may
when in fact the conclusion should focus on the disagree with the psychologists cited
relation between pravastatin and cholesterol
levels 116. Lawyer: Did Congleton assign the best available
(E) fails to consider what percentage of the general graphic artist to the project?
population might be taking pravastatin
Witness: Yes.
114. A university study reported that between 1975 and 1983 Lawyer: And the best writer?
the length of the average workweek in a certain country Witness: Yes.
increased significantly. A governmental study, on the
other hand, shows a significant decline in the length of Lawyer: In fact, everyone she assigned to work on the
the average workweek for the same period. Examination project was top notch?
of the studies shows, however, that they used different Witness: That’s true.
methods of investigation; thus, there is no need to
look further for an explanation of the difference in the Lawyer: So, you lied to the court when you said, earlier,
studies’ results. that Congleton wanted the project to fail?

The argument’s reasoning is flawed because the Each of the following accurately describes a flaw in the
argument fails to lawyer’s reasoning displayed above EXCEPT:

(A) distinguish between a study produced for the (A) It takes for granted that Congleton was not forced
purposes of the operation of government and a to assign the people she did to the project.
study produced as part of university research (B) It takes for granted that the project could fail only
(B) distinguish between a method of investigation and if Congleton wanted it to fail.
the purpose of an investigation (C) It ignores the possibility that Congleton knew
(C) recognize that only one of the studies has been that the people assigned to the project would not
properly conducted work well together.
(D) recognize that two different methods of (D) It ignores the possibility that the witness failed to
investigation can yield identical results infer from known facts what should have been
(E) recognize that varying economic conditions result inferred and therefore was not lying.
in the average workweek changing in length (E) It ignores the possibility that Congleton failed
to allot enough time or resources to the project
team.
117. In a business whose owners and employees all belong to 119. McKinley: A double-blind study, in which neither the
one family, the employees can be paid exceptionally low patient nor the primary researcher knows whether
wages. Hence, general operating expenses are much the patient is being given the drug being tested
lower than they would be for other business ventures, or a placebo, is the most effective procedure for
making profits higher. So, a family business is a testing the efficacy of a drug. But we will not be
family’s surest road to financial prosperity. able to perform such a study on this new drug,
since the drug will have various effects on the
The reasoning in the argument is flawed because the
patients’ bodies, which will make us aware of
argument
whether the patients are getting the drug or a
(A) ignores the fact that businesses that achieve placebo.
high levels of customer satisfaction are often Engle: You cannot draw that conclusion at this point, for
profitable even if they pay high wages you are assuming you know what the outcome of
(B) presumes, without providing justification, that the study will be.
businesses that pay the lowest wages have the
Engle’s statement indicates that he is most likely
lowest general operating expenses and thus the
interpreting McKinley’s remarks to be
highest profits
(C) ignores the fact that in a family business, paying (A) presuming that a double-blind study is the only
family members low wages may itself reduce the effective way to test new drugs
family’s prosperity (B) denying that the drug will be effective
(D) presumes, without providing justification, that (C) presuming that the placebo will produce no
family members are willing to work for low effects whatever on the patients’ bodies
wages in a family business because they believe (D) referring to the drug’s therapeutic effects rather
that doing so promotes the family’s prosperity than to any known side effects
(E) presumes, without providing justification, that (E) based on a confusion about when a drug is
only businesses with low general operating efficacious

$$$$
expenses can succeed
120. A successful chess-playing computer would prove either
118. Columnist: It is impossible for there to be real evidence that a machine can think or that chess does not involve
that lax radiation standards that were once in thinking. In either case the conception of human
effect at nuclear reactors actually contributed to intelligence would certainly change.
the increase in cancer rates near such sites. The
The reasoning above is most vulnerable to criticism on
point is a familiar one: who can say if a particular
the grounds that it does not consider the possibility that
case of cancer is due to radiation, exposure to
environmental toxins, smoking, poor diet, or (A) the conception of intelligence is inextricably
genetic factors. linked to that of thought
(B) a truly successful chess program may never be
The argument’s reasoning is most vulnerable to criticism
invented
on which one of the following grounds?
(C) computer programs have been successfully
(A) The argument fails to recognize that there may be applied to games other than chess
convincing statistical evidence even if individual (D) a successful chess-playing computer would not
causes cannot be known. model a human approach to chess playing
(B) The argument inappropriately presupposes that (E) the inability to play chess has more to do with
what follows a certain phenomenon was caused lack of opportunity than with lack of intelligence
by that phenomenon.
(C) The argument inappropriately draws a conclusion
about causes of cancer in general from evidence
drawn from a particular case of cancer.
(D) The argument ignores other possible causes of the
increase in cancer rates near the nuclear reactor
complexes.
(E) The argument concludes that a claim about a
causal connection is false on the basis of a lack
of evidence for the claim.
121. All any reporter knows about the accident is what 123. Concerned citizen: The mayor, an outspoken critic of the
the press agent has said. Therefore, if the press agent proposed restoration of city hall, is right when he
told every reporter everything about the accident, then notes that the building is outdated, but that the
no reporter knows any more about it than any other restoration would be expensive at a time when
reporter. If no reporter knows any more about the the budget is already tight. We cannot afford such
accident than any other reporter, then no reporter can a luxury item in this time of financial restraint, he
scoop all of the other reporters. However, the press says. However, I respectfully disagree. The
agent did not tell every reporter everything about the building provides the last remaining link to
accident. It follows that some reporter can scoop all of the days of the city’s founding, and preserving
the other reporters. a sense of municipal history is crucial to
The argument’s reasoning is flawed because the maintaining respect for our city government and
argument fails to recognize that which one of the its authority. So, to the question, “Can we really
following is consistent with the facts the argument afford to?” I can only respond, “Can we afford
presents? not to?”
(A) The press agent did not tell everything about the Which one of the following most accurately
accident to any reporter. characterizes a flaw in the concerned citizen’s argument?
(B) Even if some reporter knows more about the (A) The argument is solely an emotional appeal to
accident than all of the other reporters, that history.
reporter need not scoop any other reporter. (B) The argument ambiguously uses the word
(C) Some reporter may have been told something “afford.”
about the accident that the reporter tells all of (C) The argument inappropriately appeals to the
the other reporters. authority of the mayor.
(D) The press agent may not know any more about (D) The argument incorrectly presumes that the
the accident than the most knowledgeable restoration would be expensive.
reporter. (E) The argument inappropriately relies on the
(E) No reporter knows any more about the accident emotional connotations of words such as
than any other reporter. “outdated” and “luxury.”
122. Research indicates that 90 percent of extreme 124. Only computer scientists understand the architecture of
insomniacs consume large amount of coffee. Since Tom personal computers, and only those who understand the
drinks a lot of coffee, it is quite likely that he is an architecture of personal computers appreciate the
extreme insomniac. advances in technology made in the last decade. It
Which one of the following most accurately describes a follows that only those who appreciate these advances
flaw in the argument’s reasoning? are computer scientists.
Which one of the following most accurately describes a
(A) It fails to acknowledge the possibility that Tom
flaw in the reasoning in the argument?
is among the 10 percent of people who drink
large amounts of coffee who are not extreme (A) The argument contains no stated or implied
insomniacs. relationship between computer scientists and
(B) It fails to consider the possible contribution to those who appreciate the advances in technology
extreme insomnia of other causes of insomnia in the last decade.
besides coffee. (B) The argument ignores the fact that some
(C) It relies on evidence that does not indicate the computer scientists may not appreciate the
frequency of extreme insomnia among people advances in technology made in the last decade.
who drink large amounts of coffee. (C) The argument ignores the fact that computer
(D) It draws an inference about one specific scientists may appreciate other things besides the
individual from evidence that describes only the advances in technology made in the last decade.
characteristics of a class of individuals (D) The premises of the argument are stated in such a
(E) It presumes without warrant that drinking coffee way that they exclude the possibility of drawing
always causes insomnia. any logical conclusion.
(E) The premises of the argument presuppose that
everyone understands the architecture of
personal computers.
125. Physicist: Determinism is the view that every event has 126. Essayist: The existence of a moral order in the
a preceding cause sufficient for its occurrence. universe—i.e., an order in which bad is always
That is, if determinism is true, then the events eventually punished and good rewarded—depends
upon human souls being immortal. In some
that are presently occurring could not have failed cultures this moral order is regarded as the result
to occur given the state of the universe a moment of a karma that controls how one is reincarnated,
ago. Determinism, however, is false because it in others it results from the actions of a supreme
is impossible to know the complete state of the being who metes out justice to people after their
universe at any given time since it is impossible death. But however a moral order is represented,
to measure accurately both the position and if human souls are immortal, then it follows that
velocity of any given subatomic particle at a the bad will be punished.
particular time.
Which one of the following most accurately describes a
The physicist’s reasoning is most vulnerable to criticism flaw in the essayist’s reasoning?
on which one of the following grounds?
(A) From the assertion that something is necessary to
(A) That it is impossible to measure accurately both a moral order, the argument concludes that that
the position and velocity of any given subatomic thing is sufficient for an element of the moral
particle does not imply that it is impossible order to be realized.
to know either the position or velocity of all (B) The argument takes mere beliefs to be established
subatomic particles. facts.
(B) That the complete state of the universe at any (C) From the claim that the immortality of human
given time is unknowable does not imply that the souls implies that there is a moral order in the
states at that time of the individual subatomic universe, the argument concludes that there
particles making it up are unknowable. being a moral order in the universe implies that
(C) That it is impossible to measure accurately both human souls are immortal.
the position and velocity of any given subatomic (D) The argument treats two fundamentally different
particle at a particular time does not imply that conceptions of a moral order as essentially
its position or velocity cannot be accurately the same.
measured separately. (E) The argument’s conclusion is presupposed in the
(D) That it is impossible to know the complete state definition it gives of a moral order.
of the universe at any given time does not imply
that there is no complete state of the universe at 127. A recent survey showed that 50 percent of people polled
that time. believe that elected officials should resign if indicted for
(E) That the position and velocity of any given a crime, whereas 35 percent believe that elected officials
subatomic particle cannot be jointly measured should resign only if they are convicted of a crime.
with accuracy does not imply that this is the case Therefore, more people believe that elected officials
for the position and velocity of all subatomic should resign if indicted than believe that they should
particles. resign if convicted.
The reasoning above is flawed because it
(A) draws a conclusion about the population
in general based only on a sample of that
population
(B) confuses a sufficient condition with a required
condition
(C) is based on an ambiguity of one of its terms
(D) draws a conclusion about a specific belief based
on responses to queries about two different
specific beliefs
(E) contains premises that cannot all be true
128. Statistician: Changes in the Sun’s luminosity correlate 129. To hold criminals responsible for their crimes involves a
exceedingly well with average land temperatures failure to recognize that criminal actions, like all actions,
on Earth. Clearly—and contrary to accepted are ultimately products of the environment that forged
opinion among meteorologists—the Sun’s the agent’s character. It is not criminals but people in the
luminosity essentially controls land temperatures law-abiding majority who by their actions do most to
on Earth. create and maintain this environment. Therefore, it is
law-abiding people whose actions, and nothing else,
Meteorologist: I disagree. Any professional
make them alone truly responsible for crime.
meteorologist will tell you that in a system as
complicated as that giving rise to the climate, no The reasoning in the argument is most vulnerable to
significant aspect can be controlled by a single criticism on the grounds that
variable.
(A) it exploits an ambiguity in the term
The reasoning in the meteorologist’s counterargument is “environment” by treating two different
questionable because that argument meanings of the word as though they were
equivalent
(A) rejects a partial explanation, not because it is
(B) it fails to distinguish between actions that are
incorrect, but only because it is not complete
socially acceptable and actions that are socially
(B) fails to distinguish phenomena that exist
unacceptable
independently of a particular system from
(C) the way it distinguishes criminal from crimes
phenomena that exist only as part of the system
implicitly denies that someone becomes a
(C) calls into question the existence of a correlation
criminal solely in virtue of having committed a
when the only real issue is that of how to
crime
interpret the correlation
(D) its conclusion is a generalization of statistical
(D) dismisses a hypothesis on the grounds that it
evidence drawn from only a small minority of
fails to deal with any matters of the scientific
the population
significance
(E) its conclusion contradicts an implicit principle on
(E) appeals to the authoritativeness of an opinion
which an earlier part of the argument is based
without evaluating the merit of a putative
counterexample
130. Dobson: Some historians claim that the people who 132. Magazine editor: I know that some of our regular advertisers
built a ring of stones thousands of years ago have been pressuring us to give favorable mention to their
in Britain were knowledgeable about celestial products in our articles, but they should realize that for us to
events. The ground for this claim is that two of yield to their wishes would actually be against their interests.
the stones determine a line pointing directly to To remain an effective advertising vehicle, we must have
the position of the sun at sunrise at the spring loyal readership, and we would soon lose that readership if
equinox. There are many stones in the ring, our readers suspect that our editorial integrity has been
however, so the chance that one pair will point compromised by pandering to advertisers.
in a celestially significant direction is large.
Advertising-sales director: You underestimate the
Therefore, the people who built the ring were not
sophistication of our readers. They recognize that the
knowledgeable about celestial events.
advertisements we carry are not articles, so their response to
Which one of the following is an error of reasoning in the advertisements has never depended on their opinion of the
Dobson’s argument? editorial integrity of the magazine as a whole.
(A) The failure of cited evidence to establish a Which one of the following is the most accurate
statement is taken as evidence that statement is assessment of the advertising-sales director’s argument
false. as a response to the magazine editor’s argument?
(B) Dobson’s conclusion logically contradicts some
(A) It succeeds because it shows that the editor’s
of the evidence presented in support of it.
argument depends on an unwarranted
(C) Statements that absolutely establish Dobson’s
assumption about factors affecting an
conclusion are treated as if they merely give
advertisement’s effectiveness.
some support to that conclusion.
(B) It succeeds because it exposes as mistaken the
(D) Something that is merely a matter of opinion is
editor’s estimation of the sophistication of the
treated as if it were subject to verification as a
magazine’s readers.
matter of fact.
(C) It succeeds because it undermines the editor’s
(E) Dobson’s drawing the conclusion relies on
claim about how the magazine’s editorial
interpreting a key term in two different ways.
integrity would be affected by allowing
advertisers to influence articles.
131. All orchid species that are pollinated solely by insects
have features that attract insects. The flower of a (D) It fails because the editor’s argument does not
recently discovered orchid species contains edible depend on any assumption about readers’
tendrils that invariably attract insects to the inside of the response to the advertisements they see in the
flower. Hence, it follows that this orchid species is one magazine.
that is pollinated solely by insects. (E) It fails because it is based on a misunderstanding
of the editor’s view about how readers respond
The argument is flawed because it to advertisements they see in the magazine.
(A) makes an unwarranted assumption that a
characteristic that is shared by two classes of
things is their only common feature
(B) assumes without warrant that a characteristic that
distinguishes one class of things from another
is the only characteristic that distinguishes that
class from the other
(C) mistakes a characteristic that is unique to one
particular class of things for a characteristic that
is unique to an unrelated class of things
(D) treats a characteristic known to be true of one
class of things as if that characteristic were
unique to that class
(E) makes broad generalizations about all members
of a particular class of things on the basis of
what is known about a member of an unrelated
class of things
Flaw 47. A 95. B
48. B 96. C
1. E 49. A 97. A
2. C 50. B 98. A
3. C 51. D 99. D
4. B 52. D 100. D
5. E 53. A 101. A
6. C 54. E 102. A
7. B 55. E 103. E
8. E 56. B 104. B
9. D 57. B 105. D
10. E 58. E 106. A
11. A 59. A 107. D
12. C 60. D 108. D
13. A 61. E 109. E
14. E 62. D 110. B
15. C 63. E 111. E
16. C 64. C 112. A
17. E 65. A 113. B
18. D 66. C 114. D
19. E 67. B 115. C
20. C 68. E 116. B
21. C 69. B 117. C
22. B 70. C 118. A
23. E 71. E 119. D
24. B 72. B 120. D
25. B 73. B 121. E
26. C 74. D 122. C
27. A 75. A 123. B
28. A 76. C 124. B
29. E 77. D 125. D
30. C 78. C 126. A
31. C 79. B 127. B
32. C 80. C 128. E
33. E 81. C 129. E
34. D 82. A 130. A
35. E 83. B 131. D
36. D 84. B 132. D
37. A 85. D
38. E 86. D
39. B 87. E
40. C 88. B
41. E 89. C
42. A 90. B
43. D 91. D
44. A 92. B
45. E 93. C
46. E 94. E
Flaw Questions (A) The government’s responsibility, or lack thereof, is entirely
outside the scope of the editor’s argument.
(B) The studies about the next two years may be too short to
justify a conclusion about a general trend, but the argument’s
1. (E) conclusion only deals with the period in the studies—no flaw
Boil down an argument to its true elements; don’t be there.
distracted by extraneous information. (D) Neither governmental efforts to reduce unemployment nor
As always in an argument-based question type, start by high-paying jobs are relevant in the editorial.
breaking the argument into its components. The crux of (E) Other economic indicators are also outside the scope of
this one is concentrated in the last two sentences of the the argument.
stimulus. The conclusion is that motivational posters at
these corporations won’t boost employee motivation to work 3. (C)
productively. Why not? The only evidence submitted is that
Correlation vs. Causation is a classic GMAT reasoning flaw
most are already motivated to work productively. But the goal
that appears over and over. Be ready to take a quick point by
was to “boost” motivation, not to create it. The fact that it
recognizing this classic flaw.
already exists doesn’t preclude an increase. That’s a perfect
match for (E). This question lends itself very well to prediction The researcher says the disorders and the higher theta waves
and you shouldn’t have to analyze answer choices, but for the have a tendency to happen together. This is correlation
record: evidence. Predictably, the researcher then concludes that one
of them (theta waves) causes the other (the disorders) when
(A) The argument addresses only those companies that are
she says that one will increase the risk of the other. That’s
using the posters, so (A) is outside the scope.
confusing correlation and causation. When scanning for this
(B) Again, outside the scope—since we’re only concerned with prediction, (C) leaps out. Take it, and move on to the next
whether THESE corporations will achieve their goal through the question. For the record:
use of these posters, representativeness is irrelevant.
(A) The author doesn’t use any phrases ambiguously—
(C) We’re concerned only with one beneficial effect: motivating ambiguous use in a flaw question means using a term
employees to work productively. with multiple possible definitions in a way that shifts the
(D) Also outside the scope; we’re not concerned with actual scope from one to another, or else assuming that the reader
employee productivity, but with motivation. understands which definition the author means when the
definition could change the argument. Remember that one
2. (C) wrong answer to a flaw question is often a different classic
Remember, Flaw questions are closely related to Assumption flaw from the one in the stimulus.
questions. (B) is true—the author does fail to define the phrase. But
Arguing over unemployment statistics is something of a the argument doesn’t depend on the reader understanding
pastime for economists and politicians alike. Now it seems the definition of that phrase. A flaw isn’t just something the
that the editors of a local paper have gotten into the act, argument does; it is something the argument does wrong.
concluding that unemployment in their city is getting better. (D) There is no “sample” of data; the author makes factual
Their reasoning is based on a single piece of evidence: studies statements using the word “data,” but does not identify a
showing that the number of unemployed people looking for sample; another classic flaw that does not appear in this
work has decreased. argument.
Notice the difference between the terms of the evidence (E) is clearly the opposite of what the author states in his
and the terms of the conclusion: the conclusion deals with conclusion; this answer is a classic 180.
unemployment in general, while the evidence only looks at
unemployed people who are looking for work. The editorialist 4. (B)
has assumed that all (or perhaps most) unemployed people The argument states that if Joan got an A on the term paper,
are actively looking for jobs—this would mean that the then she will be able to pass the course without doing the
decreasing number of unemployed looking for jobs must mean class presentation. Since she didn’t get an A on the paper, the
a decreasing total number of unemployed. (C) points out the author concludes that she needs to do the class presentation
flaw based on this assumption. If many of the unemployed to pass the class. Note, however, that the statement “If A on
workers have stopped looking for jobs, the assumption, and the paper, then no presentation” doesn’t tell us anything
the conclusion based on it, would fall apart. about what happens if Joan doesn’t get an A on the paper.
It could be that she won’t need to do the presentation as (E) describes exists; rather, the author finds sincerity (of
regardless of her paper grade. (B) sums up this objection. patriotic writing) to be incompatible with low citizen morale.
(A) introduces as a possibility the necessity of getting an A
on either the paper or on the presentation to pass the class. 7. (B)
There are no grounds for this assertion, and it doesn’t directly The question must hinge on the proponent’s second
affect this argument anyway. Since the argument deals sentence, where he supports the claim that irradiation
specifically with the necessity of the class presentation, we doesn’t compromise nutrition by saying that irradiation’s
certainly couldn’t say, as (C) does, that it ignores this factor. vitamin loss would be the same as in cooking. The right
(D) simply restates the conclusion of the argument, while (E) answer will put a wedge between the irradiation/cooking
discusses the possibility, irrelevant here, that some students analogy, and that’s what (B) does, by asserting that
get A’s on their papers and still fail the course. Since Joan irradiation would lessen vitamin content whether food was
didn’t get an A on her paper, we don’t really care about that eaten cooked or raw.
group of students. (A) and (E) each shift the scope from vitamins to spoilage.
(C) properly states that vitamin loss and safety are separate
5. (E) issues, but so what? The proponent is aware of that fact too.
The author draws his conclusion by drawing what he takes to The vitamin pill option (D) is, if anything, an alternative that
be a telling distinction between the CPI’s measure of the price the proponent might offer if confronted by (B), so it’s certainly
of goods sold, and the innovations that can reduce the cost of not an attack on his reasoning.
goods produced. But these are two different things, and so the
conclusion based on this distinction is utterly unwarranted. 8. (E)
Even if all the economic jargon blurred the author’s scope Here we’re told to look for a possibility that the argument
shift, perhaps reasoning backward from (E)’s wording would fails to consider. Memory and perception aren’t reduced from
have made that scope shift stand out. age 80 to age 30, as many believe. Why not? Because both
The author commits a scope shift, but the four wrong answers age groups play, equally well, a card game that tests memory
simply go beyond the scope. The conclusion has to do with and perception. But if that test isn’t very demanding—that is,
“sometime” differences in the value of government benefits, if the memory and perception one needs to succeed at the
so (A)’s suggestion that those differences might not occur is game are at a low level—then the equal card-playing abilities
useless. As described to us, the CPI isn’t measured, or used, would be no surprise, and the original “wide belief” about
any differently depending on which goods and services are younger people’s having sharper wits might well be true after
involved, so (B) is off topic too. (C) ventures into the area of all. So (E) drives a wedge between the study and the author’s
retiree purchasing behavior—it’s never mentioned, just as contrarian view.
(D)’s appeal to the future vs. the past never comes up in the Other abilities (A) are outside the scope of the specific
argument. 80-vs.-30 comparison at issue. By raising the bar in terms
of the savvy one needs to succeed at card games, (B) tends
6. (C) to strengthens the logic, not identify a flaw. The possible
A huge scope shift takes place between the evidence (the interrelationship of perception and memory (C), and the
patriotism of Arton’s nation was quite low) and the conclusion charge of ageism (D), deflect attention from the comparison
(any patriotism in Arton’s plays must have been insincere, at hand.
ironic). The author must be assuming that Arton had to have
personally shared the mood of her countrymen, but certainly 9. (D)
that need not be the case, as (C) points out; it’s just as likely Polanski argues that kids who don’t care for competition could
that she was genuinely patriotic and bucked the prevailing be driven to hate sports if forced to compete. So, he must have
trend. heard Brewer say (D), that a kid should be forced to compete
(A) draws an unwarranted comparison between two factors in whether s/he likes it or not. Of course, Brewer said nothing of
the nation that the author presents as co-equal. The author the kind; all she said was that all kids should have the chance
denies the patriotism of Arton’s work not, as (B) would have to compete. But Polanski failed to hear that, and that was his
it, because patriotism and serious writing are incompatible, mistake.
but because he thinks that patriotic writing can’t be done (A) is a view that Polanski holds, so how can (A) be the
when the nation’s mood is morose. Since his evidence is misunderstanding that would prompt Polanski to say “I
based on “general morale,” the author’s logic would allow for disagree”? Both speakers do allude to motivational issues, but
exceptions of the type that (D) cites. And no such confusion those are secondary to the main topic each considers, which
is whether kids should compete in sports, so both (B) and (E)
fall outside of the scope. (C) offers an irrelevant comparison— which the public favors investing in information technologies
irrelevant to each person’s comments—between sports and instead of changing the result overall. (A) picks up on this
other activities. possibility: perhaps the conclusion of the survey would still be
true even if the falsified data weren’t considered.
10. (E) (B) Whether or not the public accepts the survey results is
When an argument involves causation, always be on the irrelevant to whether or not the survey results were true.
lookout for alternative causes. (C) Other surveys wouldn’t have any effect on the data or the
John’s argument shows us what can happen when people conclusions of the survey in question.
take the results of scientific studies too far. According to the (D) Even if some respondents lied, that doesn’t guarantee that
study he mentions, fluoridated public drinking water causes their responses were the ones that were falsified or that the
bone cancer in laboratory rats. Of course, he also notes conclusions of the survey could still be true as a result.
that fluoride has been added to public drinking water for 40
(E) Perhaps if the survey were redone, this might have
years, so unless there’s a secret epidemic of bone cancer, it
some effect, but a later change in opinion wouldn’t make a
doesn’t appear to have the same effect in humans. But John
difference in a survey of recent opinions.
skips over this reassuring fact, and assumes that fluoridated
water causes bone cancer in humans. And that’s not all: John
12. (C)
concludes that he should stop drinking fluoridated water
altogether, and that in doing so, he will be sure not to develop Always consider how the evidence relates (or doesn’t relate)
bone cancer. to the conclusion.

Even if John were right in thinking that fluoridated water When reading through this stimulus, you probably shook your
causes bone cancer in humans, his conclusion shows that head and said to yourself, “That doesn’t make any sense!”
he takes yet another logical leap. He thinks that by removing Well, if you did, you were correct, but that’s exactly the point
one cause of bone cancer, he’ll remove all of them and so be of this question. The GMAT designed the argument to be
certain not to get it. This shows that he hasn’t considered all flawed, and your job is to figure out how. The conclusion is
the other possible causes of bone cancer, which is (E). All of the abstract principle that one’s freedom is worth the risk of
the wrong choices are outside of the scope of his argument: losing one’s life. But the only evidence given in support is an
example of a person locked in a room with no hope of escape,
(A) Maybe John would be a bit more reasonable if he
which the author does not consider living at all. If you had
considered the (lack of) effect of fluoridation on everybody
trouble figuring out how these two statements relate to each
else, but the fact that he doesn’t do so isn’t a flaw in his
other, that’s a clue that they actually aren’t at all related. And
argument.
that is the fundamental flaw in this argument: the author uses
(B) Diseases besides bone cancer are irrelevant to John’s a single, unrelated example to support his conclusion. (C)
argument. best expresses this flawed relationship between evidence and
(C) Perhaps there were periods over the last 40 years when conclusion.
public water was not fluoridated, but John is concerned with (A) The author compares the value placed on freedom with
the effects of fluoridated water. only one other thing, the risk of losing one’s life. This is not
(D) John is indeed only concerned with the negative effects of the same as presuming that freedom has a greater value than
fluoridation, but this is not a flaw in his argument. anything else.
(B) While the risk of losing one’s life might imply rebelling
11. (A) physically against an encroachment on one’s freedom, there is
Something that the argument “fails to consider” is an no guarantee that the two things are the same. That makes (B)
alternative possibility. outside the scope of the argument.
The stem of this question gives us an extra hint: this argument (D) and (E) also fall outside of the scope of the argument.
is flawed because the author fails to consider something that The freedom of other people and the courage to take risks for
she should have. We should be on the lookout for alternatives freedom are never mentioned by the author.
to the argument’s conclusion whenever we see language like
this in a question stem. 13. (A)
We know from the evidence that pollsters falsified data in a An argument is flawed when the evidence fails to establish
recent survey. This would of course call the conclusions of the conclusion.
that survey into question, but it doesn’t necessarily mean Someone is already trying to build a better mousetrap, or in
that the survey’s conclusions are false, as the author asserts. this case, a better refrigerator. The current version includes
The falsified data could simply have overstated the margin by
a membrane that kills bacteria by removing oxygen from the What difference does it make that today’s professionals
air. The author tells us that this new type of refrigerator will went through the old “closed book” exams? The question
reduce energy costs, since it does not need to be kept as here is whether closed book exams make sense. Similarly,
cold. This leads the author to conclude that running the new (D) points out a distinction (years of study) that doesn’t help
refrigerators will be cheaper in the long run. But we know that us determine whether closed book exams make sense for
something must be wrong with this argument, since the GMAT students. (C) and (D) invite you to make up a complicated
has asked us to find a flaw in it. If we look a little closer, we’ll story in their support, but that’s never the way to go on
see that the author’s evidence only deals with energy costs, the GMAT.
whereas the conclusion deals with the entire long-term cost of
running the refrigerator. Any other costs that the author failed 15. (C)
to consider—like the cost of the new membrane itself—would Here’s a familiar topic: whether media violence causes real
prove that this conclusion need not be true. (A) provides us violence, and whether minors should have access to violent
with that extra expense. images. It’s a real debate with many important points to be
(B) Whether or not consumers like the new refrigerators is made on both sides, but Maurice adds nothing to it. Maurice
outside the scope of the author’s argument, which is all about merely points out that violence existed long before movies and
cost. TV, but that doesn’t prove that movie and TV violence doesn’t
(C) The development of the membrane is likewise outside the cause real violence. After all, cancer existed for thousands
scope of the argument. of years before people cultivated tobacco, but that doesn’t
prove that cigarettes don’t cause cancer. Sure, TV and movies
(D) The argument does take into account the effectiveness of
cannot be the sole cause of violence, but Maurice’s claim does
current refrigerators; the author says that the new refrigerators
nothing to counter Jane’s claim that they play a causal role (so
are more effective, or at least do the same job using less
to speak) in violent acts. (C) nicely captures this criticism.
energy.
(A) Maurice doesn’t refer to popular opinion.
(E) The inconvenience of food spoilage is outside the scope of
the argument. (B) Maurice makes a judgment concerning what’s morally
permissible when he decries Jane’s call for censorship, and
14. (E) then he makes a claim about objective facts when he makes
the historical argument, but he doesn’t confuse the two. The
For Flaw questions, it’s usually possible to form a prephrase
word “besides” indicates that these are separate points.
of the answer, which should save you time in reviewing
answer choices. Put simply, the argument says that students (D) We have plenty of proof that violence has existed for
should be allowed to have open-book tests. Why? Because in centuries. That’s not Maurice’s problem.
performance evaluations in the professional world, doctors, (E) “Violence” is used in the same way throughout. No
lawyers, etc. are allowed to refer to their books. This is ambiguity there.
nothing more than an argument by analogy. The operating
assumption in all arguments by analogy is that the two things 16. (C)
that are compared are, in fact, comparable. Therefore, the best The first sentence gives the view with which the author
way to weaken them is to show how the two things are not disagrees, and the second sentence announces the author’s
comparable. Or, in this case, where you must determine why conclusion. The view in the first sentence is wrong, for
the reasoning is questionable, look for an answer choice that (note the Keyword) most people disagree with it. That’s an
says something along the lines of “the reasoning is flawed unpersuasive argument. The fact that most people disagree
because it tries to compare apples and oranges.” (E) does just with a point of view doesn’t make that view wrong. After all,
that by saying that the author hasn’t considered the possibility at one time most people believed that the earth was flat. (C)
that the purposes of the two tests mentioned in the stimulus points out this error.
are in fact quite dissimilar.
(A) There’s no ambiguity inherent in either mention of the word
(A) There is nothing insufficient about those examples. In fact, “policies”: In both cases, “government policies” simply means
they support that generalization quite nicely. the government’s position, in this case in regard to Country X.
(B) The author is not trying to make the argument that (B) Sure, the commentators aren’t identified, but so what? The
students’ scores will improve, only that they should have author need not tell us precisely who they are in order to argue
access to textbooks. By bringing up scores, this choice falls that their views are incorrect.
outside the scope.
(D) essentially says that the argument is circular, but it isn’t.
(C) , (D) It may be true that the author neglects to The argument may stink, but at least the evidence is different
take these into account, but that’s not why his argument is from the conclusion.
questionable.
(E) doesn’t describe this argument. The author points out that 19. (E)
(most) people individually have a point of view, but doesn’t The stem tells us we’re dealing with another logical flaw
then claim that the country in general has that point of view. question, and this time it’s a fairly complicated situation.
The key to finding the answer comes in recognizing that the
17. (E) stimulus describes two causes of a problem that results from
Flawed arguments with a lot of “can’s” and “must’s” will mowing, not one. Grasses and woody plants are planted on
confuse necessity and sufficiency. dirt embankments to prevent erosion. When the grass gets too
This long chain of reasoning rewards test takers whose ears high, someone mows it. The resultant clippings smother the
are tuned to words of necessity and sufficiency. Small retailers woody plants, causing the roots of the plants to rot. That’s the
“can” maintain customers with exceptional service. The author problem—the roots, which are the safeguard against erosion,
concludes with a flourish: “hence” small retailers “must” offer get damaged. But mowing has another effect: The clippings
exceptional service in order to compete with the big discount also attract rodents that damage the roots as well. So, the
chains. Correct answer (E) calls the author on this unwarranted problem—damaged roots—results from two different causes
switch. There may be other reasons customers prefer small associated with mowing. The proposed solution—bringing in
stores, making it possible that small retail stores don’t have to predators to eradicate the rodents—eliminates the second
offer exceptional service. cause of root damage, but ignores the first, as (E) correctly
points out.
(A) The conclusion is based on small retailers, not the large
discount chains. (A) is fairly ambiguous: What are the two co-occurring events?
If we assume that they are the smothering of woody plants
(B) The author points out that large retail stores can make a
by clippings and damage to the roots by rodents, then these
profit with large prices. Whether or not all large retailers realize
events are not causally related, but rather are both the result
this potential isn’t important to the conclusion, which is about
of a different event, mowing.
the profitability of small retailers.
(B) The proposal to introduce predators is relatively specific
(C) may be the case, but the scope of the argument deals only
and is based on representative information about rodents
with profitability.
damaging plant roots. Unfortunately, as specific and relevant
(D) The author’s argument only applies to “small retailers that as it may be, it only addresses part of the erosion problem.
are forced to compete with large discount chains.”
(C) No, the conclusion is not a restatement of evidence:
The conclusion is that introducing predators to eradicate
18. (D)
the rodents will stop the erosion, while predators are not
Like Assumption questions, Logical Flaws involve a gap mentioned in the evidence.
between evidence and conclusion.
(D) The author claims that this is a solution to the problem but
The last sentence alone provides a nice summary of the not that it is the only solution.
argument. Marian hates to take the train; therefore, she’ll be
late to work. It’s clear that the author assumes that Marion will 20. (C)
not take the train because she hates it. (D) correctly identifies
The politician’s argument is flawed. Why? Well, he concludes
the flaw. In other words, just because we have reason to think
that his opponent is mistaken in believing that tax increases
Marion won’t take the train doesn’t suggest that she will not
are necessary, and what evidence does he have to back
take the train.
this up? Solely the fact that tax increases would make some
Contrary to (A), the author never generalizes to all people. The taxpayers upset. Huh? The politician has neglected all other
argument only deals with Marion and her schedule. issues connected to taxation, education, and health care. (C)
(B) , like (A), generalizes to all people. Furthermore, Marion has sums up the problem well. It’s not valid to claim that a course
thought through all the consequences of the various possible of action shouldn’t be followed merely because it will lead to
ways to work. some unhappiness.
(C) Again, this choice suggests the argument is flawed because (A) What “other unpopular views”?
it generalizes from one person to all people. But the author (B) There are no personal insults here, just poor logic.
only makes a prediction about Marion.
(D) Taxpayer unhappiness is a relevant issue. It’s just that it
(E) Marion has good reasons for both taking the train and for isn’t the only issue.
driving, and so (E) can’t apply to the argument.
(E) Actually, the politician is claiming that an obligation
doesn’t exist.
21. (C) evidence has nothing to do with the technicalities of time-of-
The stem alerts us to another missed opportunity on the part death. Instead, we learn only that the attendant was his ill
of an author to see a different side of an argument. We’re mother’s sole means of support. Do you hear violins wailing in
presented with a common variation on the standard the background here? (B) pinpoints the flaw exactly: instead
assumption/find-the-flaw group of questions. The stem flat- of responding to the evidence about time technicalities, the
out states that the company representative fails to consider representative makes an appeal to emotion.
an alternative possibility. (A) clearly does not describe the representative’s argument.
What’s the alternative that the representative doesn’t The evidence involves an ill, dependent mother, and the
consider? conclusion involves something entirely different—a payout
The representative argues that it’s unlikely that the consumer on an insurance policy. The reasoning is clearly bad, but not
will have any problems with the software upgrade. Why? circular.
Because the company has received fewer than 100 telephone (C) also misses by a wide mark. The representative makes
calls about problems, even though they’ve distributed no attempt whatsoever to distinguish between family and
nearly 3000 copies of the upgrade. Did you notice the scope business obligations.
shift? The evidence is about the frequency of phone calls, (D) and (E) should both register as completely loopy. What
but the conclusion is about the likelihood of problems. The person (D) does the representative attack? And where is there
representative assumes that if you don’t make a phone call, any hint of an argument based on cause and effect (E)?
you don’t have a problem. Has the representative overlooked
another possibility? Sure, and (C) states the plausible 23. (E)
alternative scenario that you’ve probably already figured out.
The argument here can be stated quite succinctly: Because
(A) This hypothetical situation is irrelevant to the argument. there were few cases of damage from the computer virus, the
The representative doesn’t consider the possibility that aliens alarm raised by companies selling virus-protection programs
will someday land and fix the problem, either. The hypothetical must have been a fraud. But the argument contains a
doesn’t relate to the immediate possibility that uploading this classical GMAT flaw: It ignores alternative explanations for
upgrade now will cause problems for the consumer, so the rep the minimal damage. Choice (E) suggests a plausible
is justified in not considering the possibility cited in (A). alternative explanation: computer owners responded to the
(B) The representative’s conclusion concerns the likelihood alarms and bought antivirus programs in time to prevent
of having a problem, not how serious a problem will be if the widespread disaster.
consumer actually experiences it. (A) The author does provide a reason to accept the conclusion:
(D) The rep is merely reassuring the customer that installing the limited damage from the virus. The problem is that the
the upgrade will be safe, so the issue of what may happen conclusion doesn’t clearly follow from the evidence.
if the upgrade is not installed is beyond the scope of the (B) The conclusion involves the motivation of the companies
argument. In the context of the situation cited, the rep is not that raised the virus alarm. Even if the antivirus programs
obligated to consider the possibility in (D). protect against other viruses, the companies’ motivation for
(E) Whatever the cause of the problems, so far, those raising an alarm against this virus may still be suspect.
problems have generated fewer than 100 telephone calls, (C) completely turns around the author’s argument. The author
and that’s doesn’t claim that the sale of antivirus protection programs
the gist of the representative’s evidence. The conclusion caused the damage to be limited. On the contrary, the author
drawn from this evidence is about the likelihood of having claims that the limited damage proves there was no major
a problem—reasons for the reported problem are one step problem for the software to fix in the first place.
beyond the scope. (If anything, (E) would help the rep’s case
(D) Even though the author called the companies’ claims
by suggesting that it’s not the product, but the bonehead
a “fraud,” which might be construed as inflammatory, the
consumers who are to blame for the problems experienced
argument still derives its conclusion from evidence about the
with the software.)
few cases of damage.
22. (B)
24. (B)
Arguments are flawed when evidence does not support
The first step in finding a flaw is always to examine the
a conclusion. Here, the representative argues that an
connection between evidence and conclusion. The author here
insurance policy should be paid to the mother of a dead flight
doesn’t mince words: The new legislation is bad because its
attendant, even though the policy had not yet taken effect
supporters include hooligans and tax evaders. Thus, a classic
at the time of the attendant’s death. What evidence does
the representative rely upon to back up this conclusion? The
GMAT flaw—the ad hominem attack—rears its ugly head, 26. (C)
and choice (B) describes that flaw in satisfactory detail. What do you know about the various “excellent” candidates
(A) The testmakers set a deadly little trap here. If you didn’t for the presidency of the company? You know that each one
notice that the argument attacks the bill’s supporters instead possesses “different talents and experience.” The writer
of its originators, you might have immediately pounced on this argues that Jones is the best qualified. Why? Because Jones
choice. has a “unique” set of qualifications. But all the candidates
(C) The author does make an appeal on behalf of are unique—they all possess different talents and experience.
manufacturers who would be hurt if the bill passes, but this So, what sets Jones apart? Nothing, really. In the language
isn’t a flaw in the argument; in fact, it constitutes the only of choice (C), the argument tries to distinguish Jones on the
valid reasoning in sight. The problem, as seen above, arises basis of something that applies to all the candidates, personal
with the author’s support for her appeal, not from the appeal uniqueness.
itself. (A) The argument describes all the candidates as “excellent”
(D) The argument doesn’t tell us how many people outside and praises their different talents and experience. But those
the legislative body support the bill. We know only that some comments function to give us a basis for comparison with
supporters don’t meet with the author’s approval. Jones, not to win over opponents by flattery. And it’s the
comparison that ultimately fails.
(E) goes far beyond the scope of the argument. Do the bill’s
supporters act inconsistently? Do they somehow depend upon (B) The argument doesn’t work by refuting opposing positions.
manufacturers’ success? Nothing in the argument suggests What opposing positions are there to refute? The argument
they do. acknowledges that several excellent and uniquely qualified
candidates want the job. Then it makes a pitch for Jones.
25. (B) (D) The argument does not claim that all the candidates
The question stem tells us that the argument is flawed, which are excellent because Jones is excellent. It makes a specific
is handy, because without that hint the sociologist’s logical claim—that Jones is the best candidate for the presidency of
error is harder to spot. The conclusion is announced in the United Wire—on the basis of a characteristic, uniqueness.
first sentence: The number of violent crimes isn’t so huge. (E) The group would not bring to the job different talents and
Why should we believe that? The Keyword “for” signals that experience—the individual candidates would. The problem is
the evidence will follow: the belief that crimes are common is that every member of the group is unique, including Jones.
based on the number of crime stories, but crime stories are
more likely to be printed because they’re such rare events. 27. (A)
Wait a second. Crime is rare, so newspapers are likely to print A “full understanding” of current events requires two things:
stories about it, so therefore we can conclude that crime is an appreciation of their significance and a sense of direct
rare? The sociologist’s argument depends on crime being involvement with them. Television provides the second (a
rare in order to show that crime is rare. That’s circular: The sense of direct involvement) but not the first (it doesn’t have
supposed rarity of crime is both the evidence and conclusion enough depth to provide an appreciation of the events’
here. So, as (A) points out, the sociologist’s argument significance). Newspapers provide the first, but not the
assumes the truth of the conclusion it sets out to establish. second. Most people don’t seek out other news sources,
(A) There is a faulty “presupposition,” but then (A) goes and so the author concludes (note the keyword “therefore”)
astray. The sociologist only claims that stories about crime are that most people cannot fully understand current events.
common, not that they’re the majority of stories. We can tell where the author is going here. Anyone limited
(C) The fairness of the stories themselves isn’t an issue in the to only television or only newspapers couldn’t get a full
argument. The sociologist is only concerned with the decision understanding because one of the requirements would always
of which stories to publish. So, the sociologist doesn’t make be missing. But there’s no reason to believe that anyone is
the assumption in (C). limited to just television or just newspapers. Someone who
watched television and read the newspaper would fulfill both
(D) is an GMAT flaw, but isn’t present here. The sociologist
requirements, but the author ignores this possibility. (A)
doesn’t make a claim about every member of a group (the
points out this flaw in fancier language.
passage only mentions many stories) and the sociologist
doesn’t assign a characteristic to a group taken as a whole. (B) and (D) are not flaws. Sure, the author doesn’t address the
reason why people read newspapers and watch television (B),
(E) The sociologist’s argument is confined to what is the case
or potential disadvantages of direct involvement (D), but why
now; no predictions are made.
should she? The author is concerned only with what it takes to
get a “full understanding.” Other issues are outside the scope.
(C) “Depth of coverage” is adequately defined for the purposes like someone who just jumps on the bandwagon, but does
of this argument. There’s no ambiguity in the term that would that mean that she won’t deliver on her promises? Not
make any difference here. necessarily. The central issue here is the nature of Astorga’s
(E) The author never makes the distinction between what true intentions. If she does not intend to follow the popular
the media can do and what the media actually does. Rather, course of action, then the author’s claim that the voters aren’t
the author argues that anyone limited to just television (or being told the truth sounds valid. However, if (E) is false, and
just newspapers) cannot get a benefit not provided by that Astorga does intend to do what the polls have told her the
medium. people want, then the voters aren’t being misled, and the
argument falls apart. In other words, the author assumes that
28. (A) Astorga has some kind of hidden agenda, but she might not.
Therefore, (E) is the questionable assumption we seek.
The question stem alerts us that there’s a flaw in the ad’s
reasoning (which shouldn’t be too surprising), so in our (A) is too extreme. The author does not suggest that Astorga
reading of the stimulus we can focus on the issue of why is incapable of fulfilling her campaign promises. The author
the argument goes flat. The ad assures us that Sturdimades questions whether Astorga will actually fulfill them.
are, well, sturdily made (hey, it was the testmaker’s joke, not (B) The author claims that Astorga, basing her campaign
ours) and that we can count on being able to drive them long promises on the polls, is just telling the people what she
distances. Why? The ad notes that every member of the “long- thinks they want to hear but will do something different if
distance” club drove their Sturdimade 100,000+ miles, and elected. Whether the polls truly reflect the voters’ wishes is
some much further than that. But wait a second: the only way irrelevant. If the polls are false, then Astorga is promising to
to get into the “long-distance” club is to drive a Sturdimade do things that the people really do not want, but the author’s
for a long distance. As (A) indicates, the ad draws a general claim that Astorga is not telling the voters the truth about her
conclusion about all Sturdimade cars based solely on facts intentions could still be true.
about the cars that lasted for 100,000 miles. What do the (C) is outside the scope. This argument is about Astorga’s
experiences of the “long-distance” club members tell us about true intentions, her sincerity. Whether or not the voters are
our chances of driving a Sturdimade a long distance? Nothing. persuaded by her campaign promises is another issue.
For all we know, there are millions of Sturdimades that fell to
(D) is a 180: the author assumes that Astorga does have some
pieces on the drive home from the dealership, but you won’t
kind of hidden agenda.
see those in the ad.
(B) accuses the argument of employing circular reasoning, a 30. (C)
flaw that sometimes appears on the GMAT, but not here.
Wow—poor Smythe! Based solely on the fact that the
The evidence may not support the conclusion, but it is at
department’s disastrous decline followed Professor Smythe’s
least different from the conclusion.
appointment as its head, the author concludes not simply that
(C) No term is used in two distinct ways. This is the “error the decline is due to Smythe’s bumbling or incompetence,
of equivocation,” another common GMAT flaw that does but even worse, that Smythe was appointed specifically in
not appear here. order to undermine the department. But there’s no evidence
(D) The positive experiences of a few hundred owners may for this conspiracy theory; we simply have an event (Smythe’s
not provide much support for the argument, but they don’t ascension to the head of the department) followed by other
undermine the argument. events (three negative happenings in the department). Lacking
(E) may have been tempting in that the ad draws a strong actual proof, the argument must be assuming, as (C) has it,
conclusion based upon shaky evidence, but the experiences that simply because a change followed an action, the action
of a few hundred Sturdimade owners isn’t enough to qualify as was taken in order to bring about that change.
“popular opinion.” (A) concentrates on a minor detail in the argument. Moreover,
it doesn’t accurately criticize that detail. The argument says
29. (E) that the department’s “reputation” declined. This is, in itself,
We know from the question stem that the author is assuming a bad thing to happen to an academic department, and the
something questionable, so we know to focus in on the argument needn’t prove that this decline in reputation is
evidence and conclusion and look for a missing premise. The matched by a real decline in quality.
author explains that Astorga is simply responding to polls and (B) The argument makes no “general claim,”—it makes a
is telling the voters what they want to hear. The conclusion, specific claim about this one case.
found in the last sentence, is that Astorga is not telling voters (D) introduces an irrelevant distinction. All the developments
what she will really do as mayor. Granted, Astorga sounds cited in the argument (including “quantitative” developments,
like the drop in student enrollment), are indicative of an (A) There are no mutually contradictory premises here. It
overall decline in the quality of the department, which the is certainly possible for Sutherlin, Pèrez, and Associates
conclusion imputes to deliberate sabotage. to specialize in criminal defense, for them to have a 90%
(E) The argument doesn’t presuppose that Smythe was acquittal rate, and for Dalton to specialize in divorce; none of
appointed to wreck the department, it merely concludes this, these statements contradicts the others.
based on the changes that followed Smythe’s appointment. (B) is outside the scope of the argument. The author here is
The argument is faulty, but not because the reasoning is concerned only with excluding Dalton from the law firm of
“circular.” Sutherlin, Pèrez, and Associates based on his specialization.
The possibility that lawyers can go it alone plays no part in the
31. (C) faulty logic here.
Always read the question stem before tackling the stimulus. (D) The 90% acquittal rate does suggest a high rate of success
Here we see a golden example of why that’s so important. Just for the firm in general, but nowhere does the author use this
by reading the stem, we know that this question asks for the to indicate that the firm’s success is evenly spread among the
one choice that the school official could use to identify the attorneys.
flaw in the survey report. So, what’s wrong with that survey (E) Does the argument state a generalization based on a
report? The survey asked all Center City residents over the nonrepresentative sample? No. One could possibly argue that
age of 19 if they were high school graduates. Based on their there’s an implied generalization that Sutherlin, Pèrez, and
response, the survey report concluded that the city is among Associates consists only of criminal defense lawyers, but this
the ten worst cities with respect to the dropout rate from is somewhat of a stretch. No generalization is stated outright.
its schools. Do you see the problem? The survey asked all
residents over 19, whether or not they actually attended school 33. (E)
in the city, if they were grads. What if a significant number of
Subtle shifts in scope can lead to reasoning errors.
the dropouts moved to the city after dropping out from some
other city’s schools? This would change the findings. The The safety inspector is concerned about a recent increase in
results would be different if the methodology employed by the the number of laboratory samples of the rabies virus being
survey took the distinction in (C) into account, and the school sent through the university delivery service, and claims that
official can justifiably criticize the report for failing to do so. that use of the service must be limited. The biologist utterly
fails to address the recent change in circumstances. He argues
(A) Since the survey report concluded that the city has one of
that the lack of accidents over the past 20 years proves that
the worst dropout rates from its schools (and not just its high
this use of the university delivery service poses no danger. (E)
schools), the survey does take into account dropouts that
pinpoints the scope shift between the two speakers.
occurred before high school.
(A) brings up an irrelevant consideration. The reason why there
(B) The survey report concerned high school dropouts. The
has been an increase in the use of the university delivery
existence of over-achievers who finished school more quickly
service to ship laboratory samples of the rabies virus says
than usual is irrelevant.
nothing about the potential danger involved.
(D) and (E) The future effects of the report (D) and the value of
(B) is a 180. The biologist does focus on the issue of rabies
the city’s high school diplomas (E) are both outside the scope
virus samples.
of the argument.
(C) The safety inspector’s argument does not depend on a
32. (C) change in the “hazardousness” of the rabies samples being
sent through the university delivery system; he is concerned
The question stem alerts you to watch for a flaw, and
with the recent increase in the number of samples going
the extreme-sounding conclusion (Dalton certainly is not
through the system. You can’t commit flawed reasoning
a member of this law firm) should raise a red flag. The
by failing to consider something outside the scope of the
information we’re given about Sutherlin, Pèrez, and Associates
argument you are attempting to rebut.
states that their primary specialization (not necessarily their
only specialization) is criminal defense; there is nothing in the (D) misses the point at issue. The safety inspector does
stimulus to indicate that a lawyer in this firm cannot specialize not propose eliminating the transportation of rabies virus
in something other than criminal defense. It’s therefore unfair samples; he just wants there to be some limits on this use of
to conclude that Dalton is “certainly” not a member of the the university delivery system. There is no logical necessity for
firm solely because he doesn’t share the firm’s predominant the biologist to prove that the university needs to deliver the
characteristic (criminal defense as a primary specialization). rabies virus samples from one place to another.
Choice (C) states this flaw in more general terms.
34. (D) 36. (D)
The author’s conclusion—that the archaeologists will surely The bird-watcher proves that he knows no more about logic
be able to determine the accuracy of this year’s museum than he does about animal tracks when he identifies a
financial report—is based on their universal access to all typical trait of birds (three toes forward, one back), and then
relevant documents and, in turn, on the assumption that those concludes that a track with that trait must be that of a bird.
documents are going to list all relevant financial transactions. That’s like saying that since most sailors wear a white hat,
But if they do not, then the reviewers’ determination any particular person with a white hat must be a sailor. As if!
of accuracy may be way off. (D) points out the possibly Now, if we knew that sailors and only sailors wore a white hat,
unwarranted assumption here. then that conclusion would be correct; likewise, the stimulus
(A) The report examines only each year’s transactions; long- conclusion would work if we knew that only birds made tracks
held pieces are irrelevant. in that 3/1 pattern. But that evidence is absent. Answer choice
(D) points out the flaw.
(B) and (C) If the documents under review are complete, then
the accuracy of the report will be assured; if they’re not, then it (A) As used, “track” is adequately concrete.
won’t. Neither the size nor quality of the collection (B), nor its (B) The bird-watcher takes for granted that birds are animals,
availability to the public (C), will matter one way or the other. and that “typically” they have four toes. No further definition
(E) True, the author doesn’t discuss what the next steps is required.
would be if discrepancies were found between the report (C) Since the issue is simply whether the animal that made
and the documents, but he is not logically obligated to do the track in question was a bird or not, the type of bird is
so. The argument is simply that given the documents, the irrelevant.
archaeologists will be able to determine the accuracy of (E) has it exactly backwards. The evidence is about birds in
the report—what happens after this is one step beyond the general, and the conclusion drawn is about an individual
argument. critter.

35. (E) 37. (A)


Did you catch the word “needs” in P’s assertion, and did you Reading the stem first tells us that the reasoning is flawed,
then connect it to the concept of necessity? Q sure didn’t. so you should have been on the lookout for the reasoning
She takes P’s assertion—that an effective official needs error right out of the gate. The individual pieces, the elements
the support of a party—to mean that any official with party that make up the library, were all copied and are therefore
support will be effective. However, that which is necessary for unoriginal. Does it follow, then, that the completed library
effectiveness need not be sufficient for effectiveness, which is cannot be considered original? Not if there’s something else
exactly what answer choice (E) tartly points out. All P is saying that confers originality on such a design, such as the creative
is that an independent candidate—i.e., one without a party process involved in choosing the pieces, or in arranging them
backing him or her—can’t be effective. But Q screws that up. to form a new coherent whole. In other words, something can
(A) Q does indeed offer evidence: it’s evidence that she be true of the whole without being true of any of the parts, and
believes to be a counterexample to P’s argument. The problem so the author makes the error described in (A).
is that that evidence completely distorts what P has said about If it’s still unclear, try this analogous example: assume there
an independent candidate. are four musicians . . . oh, let’s call them John, Paul, George,
(B) focuses on a party’s possible decision to support an and Ringo. If none of them is considered a spectacular
independent after an election, but that has absolutely nothing musician individually, does it follow that if you put them
to do with Q’s statement and hence cannot explain its together, the result must be unspectacular as well? Nope; it’s
weakness. the same issue as the one in the stimulus—we can’t logically
(C) As long as P and Q agree on a definition of “effective,” assume that something true of parts of a whole must be true
and there’s no reason to believe that they do not, neither the of the whole itself.
precision nor the vagueness of that term has any impact on (B) The author does not claim that the specified features are
their disagreement. similar in kind to the design of a library in general, and so
(D) is accusing Q of circular reasoning (that’s what there is no such generalization.
“presupposing” one’s conclusion means). But far from (C) There’s no “unknown instance of a phenomenon”
presupposing P’s statement, Q is taking exception to it by mentioned, so we can stop there.
presenting a counterexample. (D) Nope; there is no false “combination of alternatives” made
up of alternatives that could be true separately. If you chose
this, you have to defend it—what are the alternatives? How
are they true individually, and not in combination? If you can’t (D) The author never goes beyond the coverage comparison to
answer these questions, you can’t select this choice. discuss the impact of his thesis on the electorate.
(E) It’s possible to make conclusions of fact based on reports (E) might make sense based on our outside knowledge:
of aesthetic preferences, and so (E) isn’t a reasoning error at of course, reporters who cover a candidate negatively
all. In any case, there are no such reports in the stimulus. will lose access, right? Not necessarily. In fact, we don’t
even
38. (E) know that these reporters have access to the incumbents or
The radio broadcaster claims that a responsibility to the challengers in the first place, nor that fear of losing access
“public interest” justifies the intrusions into the private to the candidates would curtail negative coverage, nor that
lives of celebrities. When the broadcaster speaks of “public candidates are capable of denying reporters access in the
interest” in this way, he or she is talking about “for the public first place.
welfare or betterment.” However, the broadcaster then goes
on to use the term “interest” in the context of “the public is 40. (C)
interested/curious in this matter.” The broadcaster’s fallacy is The author states that the emotional impact on people
in using the term “public interest” in two different ways. differs wildly, so we can’t trust the assessment given by a
(A) Quite the contrary, the broadcaster actually implies that connoisseur. Note the shift from people to connoisseurs. It
when the public is interested in a matter, no such right exists. may well be that people have all kinds of different reactions,
but connoisseurs pretty much agree with one another. If so, we
(B) There’s no logical flaw committed by disregarding other
can trust their assessments; this flaw is summed up by (C).
grounds for criticism of the expose. The argument centers on
the invasion of privacy issue; the flaw is in the manner of the The argument hinges on the fact that everyone has an
broadcaster’s defense, not that he or she ignored other issues. emotional response to a painting, so we can certainly rule
out (A), which states that the argument ignores this fact. (B)
(C) Defining the term “excessively intrusive” isn’t necessary to
and (E) both involve a misunderstanding of the purpose of
the broadcaster’s argument. The term belongs to the critics,
the Rembrandt example. The example is used to provide a
and the broadcaster was merely repeating it.
concrete example of a painter, but (B) views the example
(D) In that legality is not discussed and morality is only vaguely as the foundation of the argument, and (E) gets sidetracked
hinted at in the critics’ argument, there’s no confusion of by the irrelevant consideration of whether Rembrandt was
these types of responsibilities/obligations. more successful than other painters at conveying emotions.
The use of the phrase “without justification” rules out (D),
39. (B) since the entire argument is constructed to prove the point
Investigating an argument’s flaw is always rendered easier that emotional impact should not be used to determine
when you explore the assumptions underlying the logic. authenticity.
The author confidently cites statistics to support his view
that news coverage was influenced by reporters’ personal 41. (E)
views: the candidate for whom an overwhelming plurality of This is a good place to use your formal logic skills. Translate
reporters voted garnered much less negative coverage than the first sentence as “If a druid stone is discovered in Ireland,
the opponent. One key assumption, of course, is that there’s then it’s very, very old.” The next clause means “This stone
no other equally (or more) logical explanation for the disparity was not discovered in Ireland,” while the final clause means
in negative coverage, and (B) provides a stunning one: if the “Therefore it is not very, very old.” In if-then language, the
challenger did in fact offer more newsworthy negative stories, “trigger” of Ireland produces the “response” of oldness, but
who can be surprised that s/he got the lion’s share of the this doesn’t mean that some other “trigger” can’t also produce
coverage? (B) offers a sharp alternative to the author’s Q.E.D. the same result. Perhaps all druid stones are very, very old.
allegation of reporter bias. The flaw, then, is in supposing that only Irish stones can be
(A) The argument is only interested in the negative coverage old, or as stated in (E), that because the members of a group
afforded to each candidate. A comparison between total (Irish stones) have a quality (oldness), that only members of
amounts of coverage might open the door to a new bias that group (i.e., not Scottish stones) can have that property.
allegation but sheds no light on this one. (A) discusses “key terms,” but none of the terms in this
(C) The quality of the coverage—solid, detrimental, or argument shift meaning—Irish, Scottish, druid stones, and
indifferent—is irrelevant to an argument concerned with old all mean the same thing throughout the argument. If
amounts of negative coverage. anything, (B) is a 180 in that the argument supposes that
some of the members of a group (the Irish members of the
stones group) are the only members that can have a certain
quality (oldness). (C) describes a circular argument, a classic
wrong answer choice on Flaw questions. Here, for example, 44. (A)
the argument’s conclusion is that the Scottish stone cannot be “Reasoning error” means look for the flaw.
very old, but this is never used as support for the argument.
Why, according to the reviewer, is R’s book unworthy of
(D) discusses past and future, but the argument is only
serious consideration? Because R implies that his critics are
concerned with whether is the Scottish rock is old right now,
corrupt, and R is arrogant, too ambitious, and nasty to boot.
not whether it will continue to be so in the future.
Do the words ad hominem come to mind? When one attacks
a speaker’s character rather than his ideas, one is committing
42. (A)
an ad hominem attack, and (A) says so in English rather
The sweeping, two-part conclusion both denies the cause than Latin.
of psychoses (i.e., it’s not environmental) and asserts their
(B) is describing a flaw that the reviewer attributes, in his
cause (i.e., it’s purely organic), all because the best treatment
second sentence, to R, not one that the reviewer himself
of psychoses is medicinal, whereas the right treatment for
commits. The reviewer never gives an account of R’s theories,
neuroses—some of which are caused by environmental
(C), biased or otherwise; the whole point is that he attacks R
factors—is traditional psychotherapy. (A) essentially points
instead of R’s theories. No facts, (D), are given—just opinions,
out that the phenomena the author cites are not mutually
verifiable or otherwise. And (E)’s truth vs. interest distinction is
exclusive. If environmental factors can cause organic
just made up out of whole cloth; no evidence supplied.
conditions or make them worse, then it’s quite possible
that psychoses caused by organic conditions do have an
45. (E)
environmental cause.
“Error of reasoning” means look for the flaw. “Clearly then”
Far from ignoring (B), the author states that medicine can
signals conclusion.
alleviate psychoses that he believes are caused by organic
factors. Non-psychological illnesses (C) are outside the scope; One possible reason for the fact that the potato dish is never
this paragraph is completely focused on psychological ones. ordered is, as the manager believes, that the patrons don’t like
Extending the scope to all medical conditions (D) is even spuds. But any number of other possible reasons exist: Maybe
further removed from pinpointing the flaw in this paragraph. If patrons don’t think of potatoes with cheese as a main dish;
anything, (E) casts doubt on the cause of neuroses, but is far or maybe the dish looks bad; or it smells bad; or the other
removed from the conclusion about psychoses that the right two entrées are so great that the baked spuds have no chance
answer is designed to weaken. to compete. Since the manager fixes on the one explanation
without recognizing that many others are equally possible, (E)
43. (D) describes his flaw.

The mayor insists that jaywalking laws serve no useful The manager’s remarks mention neither cause and effect nor
purpose, stating that they don’t deter anyone. His evidence correlation, as (A) implies. An “inconsistency,” (B), is a pair of
involves two sets of people: those who “invariably violate statements that contradict each other, but no two statements
this law” and “those who comply.” But what about those contradict each other here. (C) doesn’t apply here because it
who don’t fit into either extreme of the spectrum? What about mentions what “people say they want,” but in this case the
people who might cross against a red light once in a while, patrons never said they wanted a potato dish per se in the first
but not “invariably”? If the law serves to persuade these place. And the initial request for vegetarian dishes was just
in-between folk who “sometimes but not always” disobey that, a request, not a claim, as (D) suggests.
the law, then by the mayor’s own logic it’s serving a useful
purpose. (D), therefore, is correct. 46. (E)

(A) takes on drivers where the stimulus deals only with “1,000 students . . . randomly selected . . . 89 percent” all
pedestrians. (B) suggests that the mayor equivocates with his suggest that you should look for representativeness.
use of the word “law,” which is simply not the case. “Law” The author draws a pair of sweeping conclusions from
means “statute” everywhere the mayor uses it. As for (C), the response of “Yes” from almost 90% of students to
if you proved to the mayor that the law was a deterrent, he the question about planning to finish high school. But
might retreat to (C) and argue that the law was nonetheless the students were all from “a medium-sized city,” yet the
useless. In other words, since the mayor might espouse such conclusions about dropout rate cut across the board of all
a possibility, it’s silly to say that his argument is “flawed” geographical areas and demographics, and (E) properly points
for “ignoring” it. (E) is a train wreck; it engages in a classic out the virtually certain lack of representativeness in the
irrelevant comparison (red vs. green), and brings up the issue survey sample.
of danger that isn’t part of the mayor’s argument. One would be hard-pressed to call “almost 89%” anything
less than an “overwhelming majority,” (A), and in any case this
argument is vulnerable not on its numbers but on its viability. 49. (A)
The twin conclusions are separate and not contradictory, The ozone layer again. This spacecraft and the factory may
contrary to both (B) and (D). The argument concedes a high produce equivalent harm to it, but the latter’s is part of the
dropout rate, (C), and even if the author were wrong to so everyday course of doing business, while the former’s was
concede the argument might still have validity (but for its specifically done in the service of finding a way to solve the
unrepresentative sample, that is). ozone layer problem. By ignoring the differing contexts of the
damage and the blatant moral difference between the two, the
47. (A) author has come up with a failed analogy, as (A) describes.
“The argument takes for granted that . . .” signals an The author’s conclusion isn’t a generalization (B), so the
unwarranted assumption at work. problem can’t be that he is using only one example to justify
The flaw here is a classic scope shift. The evidence concerns one. See (E), below for an example of a generalization. (C)
people’s being intellectually ill-suited to large bureaucracies, creates an irrelevant distinction between reversal of the
while the conclusion concerns happiness. That the author fails ozone layer problem (which is pertinent to the argument) and
to see that one could possibly be happy even in a society to prevention (which isn’t). Contrary to (D) the spacecraft and
which one was intellectually ill-suited is the argument’s fatal factory are comparable: Each is a technological creation that
error, and (A) describes it. causes harm to the ozone layer. The problem is that they’re
That happiness may be found only in small political units not completely comparable, which gets us back to (A). (E)’s
doesn’t necessarily imply that that is the purpose for generalization about experiments doesn’t match the specific
which such units were founded, (B). “Plagued by excessive analysis of one situation that the author presents.
bureaucracy,” (C), is really putting words into the author’s
mouth; you cannot properly attack logic that way. (D) is a 50. (B)
classic example of the fallacy known formally as “affirming the When you are looking for a flaw within a dialogue, consider
consequent,” and informally as “flipping the if and then.” That how the arguments relate to each other.
a small political unit is (to the author) a necessary condition We’re looking for a flaw within one of the two times Sid
for happiness doesn’t imply that simply finding such a small speaks. Since there really isn’t anything wrong with pointing
political unit will ensure happiness. People’s willingness, out what a sign says, the flaw must be within his second
(E), to live in one type of political unit or another is utterly argument. In order to determine what the flaw is, we’ll have to
irrelevant to the author’s point about finding happiness. see how Sid’s argument responds to Micki’s.
Micki’s argument is probably something we’ve all said to
48. (B)
ourselves at some point: “just once won’t hurt.” If only one
Why should any additional cash be applied to some area person walks across the grass, the grass isn’t harmed. But
other than student counseling? Because almost everyone Sid concludes that her statement is false, because if everyone
concerned believes that the right percentage of the budget walked across the grass, the grass would die. Could it still be
is already going there. But percentage and amount are two true that one person walking on the grass won’t hurt it? Of
different things, as (B) points out. If budget dollars were to go course, it could. And therein lies the flaw with Sid’s argument:
up but the percentage allotted to counseling were to remain he doesn’t really address what Micki said. Instead, he twists
the same, then fewer dollars would be spent on counseling, a her words so that he can make his response sound stronger.
state of affairs that might not please anybody. Unfortunately for Sid, that means he’s shifted the scope of
(A) No causality, confused or otherwise, is described in the the argument from Micki’s action to what would happen if
argument. everyone believed what Micki does. (B) is a perfect paraphrase
(C) The author makes no such claim; to argue that a part of a of the way in which Sid falls victim to a classic type of flaw, the
whole is adequate is not necessarily to commit a part-to-whole scope shift.
fallacy. (A) Sid’s statement isn’t about the consequences of Micki’s
(D) raises a couple of issues irrelevant to the argument, actions; it is about the consequences of everybody copying
namely saving money and the content of counseling programs. Micki’s action. (A) distorts Sid’s argument.

(E) The argument pretty much does consider that money spent (C) Sid does talk about what would happen if everyone
in one area takes money away from another, and anyway, that believed a statement, but he never implies that everyone
point isn’t central to the logical error. actually believes that statement. He uses the statement as a
hypothetical situation to show the potential consequences of
Micki’s statement.
(D) Any justification of hurting the grass is outside the scope might have lied to cover their embarrassment at the fact that
of both Micki’s argument and Sid’s argument. they’ve stopped exercising. Again, this argument is pretty
(E) “Calling into question the character of the person making reasonable. If Greenwall had tried to make a concrete claim
the statement” is a common argumentative flaw in real life, (such as, “half of the survey respondents were lying”) without
known as an “ad hominem” attack. But this flaw almost never any evidence to back it up, his argument would be obviously
appears on the GMAT, and Sid doesn’t use it in this question. flawed. But instead, he simply raises a possibility that could
reflect on the truth of Faden’s conclusion.
51. (D) Faden’s final response discounts Greenwall’s argument on
Look out for gaps between the evidence and the conclusion in the ground that Greenwall can’t prove the customers were
Flaw questions. lying. Now we can find a problem with the argument. Even if
Greenwall can’t prove that some of the customers were lying,
Contrary to popular belief, the author tells us, the top
they could still be lying. The absence of proof doesn’t prove
management of large corporations behaves as though
an argument false. This is the flaw in the argument, which is
employees’ needs are important to them. The reason?
outlined in (D).
The CEOs of several large corporations said that employee
training and welfare is of the same high priority as customer (A) Greenwall doesn’t take this idea for granted, he merely
satisfaction. But saying that is easy; acting like it is true is suggests that it is a possibility.
hard. The author makes a huge logical leap—an assumption— (B) Similarly, Greenwall doesn’t make any conclusion about
in going from what the CEOs say to what they actually do. (D) most people’s honesty, only that some of Faden’s customers
points out that this leap is a flaw in the argument. might be dishonest in one case.
(A) The argument defines “top management” as the CEOs (C) Faden makes no claims about the relationship between the
of the corporations in question, which sounds like a good conclusiveness of evidence and the believability of a claim.
definition. (E) is irrelevant to both arguments, which are only concerned
(B) The author does presume that being indifferent to with people who claimed to still use the equipment and
something and considering it a top priority are mutually whether they were truthful or not.
exclusive, but this assumption is not questionable, because
the definitions of the words “indifferent” and “top priority” are 53. (A)
opposites. Now we need to identify the flaw in the plant manager’s
(C) The author tries to disprove the idea that CEOs’ priorities reasoning. The plant manager concludes that the new process
are misplaced. is more expensive, but he’s only taken two factors into
(E) One hundred twenty-five CEOs of large corporations is a consideration. Other factors, such as fuel efficiency, might be
pretty representative sample of the top management of large important as well, but the plant manager ignores them. (A)
corporations. expresses this problem using different words.
(B) No, the plant manager does describe why the new process
52. (D) would be more expensive. His argument has a little more
It is crucial to understand the connections between arguments support than “because I said so.”
to find a flaw in a dialogue. (C) accuses the argument of being circular, but the evidence
The dispute between Faden and Greenwall takes a form that is here is different from the conclusion. The argument isn’t a
quite rare on the GMAT. Faden makes an argument, Greenwall great one, but it isn’t circular.
disputes his conclusion, then Faden responds to Greenwall’s (D) There’s no probably/certainly distinction made here.
contention. Usually, dialogues on the GMAT are limited to a (E) Actually, the facts cited by the plant manager are relevant.
single statement from each speaker, but this difference won’t They just don’t prove his case.
change our approach to the question. That approach is made
more difficult, however, by the fact that the stem doesn’t 54. (E)
tell us which argument is flawed. We have to look at each
Looks like we’re going to have to play criminal defense
argument separately to determine which one has a problem.
attorney in this one as we try to determine how the
Faden’s first argument is simple: a survey of customers shows prosecutor’s argument is vulnerable. The prosecutor concludes
that most of them still use their exercise machines after a in the last sentence that there was enough light for Klein
year. Nothing wrong there—if we knew more about the survey, to make a reliable identification. What does the prosecutor
perhaps we could find fault with it, but not yet. Greenwall, base this on? 1) The fact that they have shown that the
however, does take issue with the survey. He claims that the robbery took place between 1:15 and 1:30; 2) the moon
results may not be accurate, because some of the people
did not set until 1:45; and 3) the moon was full enough to 56. (B)
provide considerable light before it set. It’s kind of tough to Always try to predict the answer to Logical Flaw questions—
prephrase an answer for this question. But before you went doing so puts you in the driver’s seat.
on to the answer choices, you needed to remain focused
General surgeons are “extremely competent,” and we certainly
on the prosecutor’s precise conclusion and the evidence on
want someone “competent” to perform surgery. According to
which he bases that conclusion. (E) is correct because even
the argument, anyone else is going to be worse. As correct
if all the other facts are true (i.e., the time of the robbery, the
answer (B) points out, just because general surgeons are
time when the moon set, the condition of the moon on that
extremely competent doesn’t mean that others can’t be
night, and the amount of light a full moon provides), the
extremely competent, too.
prosecutor’s conclusion that there was sufficient light could
still be questionable, because he has not considered whether (A) contradicts the evidence. Besides, the conclusion of the
anything could have interfered with that light. argument is about people other than general surgeons.

(A) Klein just makes an identification, and never says when the (C) The argument never claims that even the extremely
robbery took place. So, the prosecutor isn’t relying on competent doctor will always get a successful outcome, only
Klein’s ability in that respect. that they present less risk.

(B) may be a reason to believe that some innocent dupe (D) may be true, but the argument’s scope only involves risk.
may take the fall, but it does not represent a flaw in the (E) The author only considers the risk involved in surgery, not
prosecutor’s reasoning that led him to conclude there was the larger issue of choosing a long-term doctor.
enough light.
(C) Same thing—(C) does not affect the conclusion that there 57. (B)
was sufficient light to make a reliable identification. Absence of proof is not proof of absence.
(D) What’s wrong with drawing a conclusion about the lighting TV transmissions have reached neighboring star systems.
conditions? Anyone that read a weather report should be able Since no response has been received, the author reasons,
to do so reliably. Since there’s no reason to doubt the doctor, there is no extraterrestrial life in any these star systems. Well,
the possibility in (D) isn’t a reasonable one, and so (D) doesn’t that assumes a lot. You can make a list of ways to explain how
describe a flaw in the argument. it could be that extraterrestrials have not responded. (B) nicely
sums them up.
55. (E) (A) is perhaps tempting, but the argument doesn’t hinge on a
Some of the wealthy have been criminals, and so none of specific definition of “messages.” Would it make a difference
the wealthy should be appointed to the committee. Doesn’t how extraterrestrial life communicated with us? No.
that seem a little harsh? It seems unfair to reject everyone (C) is beyond the scope of the conclusion. The author
in a class just because of the actions of some people in that concludes that there are no extraterrestrials in the neighboring
class. We’re not used to the wealthy being the victims of unfair star system.
treatment, but (E) has it right. This argument makes a broad
(D) would simply reinforce the conclusion, not weaken it.
generalization (no wealthy people on the committee) based on
Messages specifically addressing extraterrestrials would
what could be exceptional cases (the crooks).
presumably make it more likely for these extraterrestrials to
(A) uses the language of some GMAT flaws, but isn’t respond.
appropriate here. No one says that anything is sufficient for
(E), if anything, supports the conclusion.
anything else. Even the author doesn’t say all rich people are
crooks. He just wants to be sure that the rich crooks are off the
58. (E)
committee.
The stem tells us we’re dealing with a Flaw question, so we
(B) gets at a correlation/causation problem but there is none.
know the evidence somehow fails to support the conclusion.
The author never speaks to what causes people to be crooks.
The argument involves several steps, but ultimately reduces
(C) There’s no distinction between accidents and intentional to this: Since any further decrease in profits will lead to
acts here. bankruptcy, the company’s only options are to reduce planned
(D) There’s no subjective evaluation here. The author doesn’t expansion or to eliminate less-profitable operations. The
use his/her personal opinion in an inappropriate way. evidence doesn’t even provide grounds for evaluating the
effectiveness of the options mentioned in the conclusion,
let alone for ruling out any other possible solutions to the
problem. So (E) stands out as the flaw in the argument.
(A) True, the author doesn’t justify the company’s survival, but So, the seaside environment has nothing to fear from the
the issue of whether the company should survive goes beyond tourist industry. Sounds pretty good, except for the modifier
the scope of an argument about how the company can survive. “intentionally.” What if the tourist industry could cause harm
(B) The author does mention alternatives to declaring unintentionally? Then the seaside environment could be in big
bankruptcy in the last sentence, so (B) doesn’t accurately trouble. (D) describes this omission.
describe the problem with the logic here. (A) The author doesn’t provide additional support for the
(C) flat-out distorts the argument. Sure, decreased demand is evidence, but so what? You’re looking for a flaw in the author’s
cited as the culprit here, but the author makes no claim that reasoning, how the author moves from evidence to conclusion.
only decreased demand can ever be the cause of decreased Authors aren’t required to provide endless supporting
profits. evidence for every claim they make.

(D) Profits are already at an all-time low, and the author states (B) The author isn’t trying to show that the tourist industry
that it’s important to prevent any further decrease. couldn’t exist at the same time as environmental damage. The
only issue is whether the tourist industry will cause harm to
59. (A) the environment.

This Logical Flaw question revolves around one little word. (C) and (E) are both logical flaws that have appeared on the
Why do ancient-history scholars no longer need to know GMAT in the past, but neither is present here. The
an ancient language? Because most ancient historical author doesn’t discuss individual examples, (C), and
documents have been translated into modern languages. Not probability never comes up, (E).
all documents, most documents. The author mistakenly bases
a strict conclusion—aspirants DO NOT have to learn ancient 61. (E)
languages—on evidence that most of the time, they won’t need The problem with P’s rebuttal becomes evident if you follow up
it. The conclusion is too strong for the evidence presented; on his rhetorical question. There is no point to creating a new
choice (A) wisely points out the gap between “most” and “all,” alphabet if you’re the only one who will ever read it. But if you
between “not always” and “never.” expect that others will read it someday or if you’re planning
(B) The argument does not treat a statement of fact as if it to teach it to others, then there’s plenty of point. P fails to
were opinion. The argument begins with a statement of fact, see—as (E) points out—that that creator of the alphabet may
then introduces another fact that it claims renders the first fact well have had a use in mind over and above his own reading
unimportant. pleasure.
(C) The conclusion is that modern scholars do not need to (A) The whole issue of the Phoenician basis of the Greek
learn ancient languages. The evidence is that most ancient alphabet is unmentioned by P and irrelevant to his rebuttal.
historical documents have been translated into modern When you come right down to it, the purpose of M’s
languages. The evidence and conclusion clearly differ. There’s mentioning Phoenicia is mostly to inspire wrong choices (C)
a problem here, but circular reasoning is not it. and (A).
(D) The argument doesn’t involve the judgment of experts (B) It’s M rather than P who assumes, correctly or not, that the
in any context. The only judgment here is the judgment that first text written in Greek was a poem by Homer.
passes as the author’s conclusion. (C) P commits no such confusion, since he is not concerned
(E) The evidence consists of two statements: First, you can’t with the roots of either an oral or written language, but with a
read an ancient historical document in its original language theory concerning who created one particular alphabet.
unless you know the language; second, most ancient historical (D) , if anything, describes M’s method (and its weakness)
documents have been translated into modern languages. as she unveils her hypothesis, but has nothing to do with
These statements are not inconsistent or contradictory in any P’s rebuttal of a hypothesis.
way—the problem is that they aren’t strong enough to lead to
the author’s very definite conclusion. 62. (D)
Terry and Pat are having a logic contest, and nobody wins. The
60. (D) question stem gives us plenty of helpful information: Not only
We know from the stem that this argument is flawed, which are both arguments flawed, but both arguments contain the
is helpful, since it seems pretty reasonable at first glance. same flaw. It’s our job to find the choice that describes the
Excessive development by the tourist industry damages error that both speakers make. In that case, there’s no reason
the seaside environment which in turn damages the tourist to read both arguments. After all, if they’ve committed the
industry. The tourist industry knows this, and they’ll never same error, then the choice that describes one will describe
intentionally do anything to damage their own industry. the other.
So, let’s start with Terry. The first sentence tells us that therein. That means we should attack the stimulus looking for
some things considered bad have favorable a logical flaw. The first two-thirds of the stimulus tells us that
consequences. No problem so far. The second sentence, retired persons that volunteer have all sorts of advantages
“an action is good only if it has favorable consequences” over retired persons that do not volunteer. The newsletter
means that favorable consequences are necessary for then claims that volunteering benefits one’s well-being,
goodness. That is, if something is good, then it has which is another way of claiming that there is some kind of
favorable consequences. But remember the difference causal relationship between volunteering and having those
between necessary and sufficient benefits. But hang on: the fact that there is a connection, or
conditions: An action having favorable consequences will not correlation, between two things (in this case, volunteering and
guarantee that that action is good; favorable consequences the benefits) doesn’t mean that one caused the other. Maybe
are necessary for goodness, but are NOT sufficient for there is no causal relationship at all; perhaps some other
goodness. Unfortunately, Terry’s concluding sentence factor caused both; or, as seems likely in this case, perhaps
indicates that he does not understand this distinction. He the causal relationship is reversed. As (E) explains, those who
concludes that some things considered bad must actually be have better resources, health, etc. are the ones who are likely
good, and we can follow his mistaken line of reasoning: Those to serve as volunteers in the first place. It’s no surprise that
things considered bad from the first sentence have favorable those volunteers are better off, considering that anti-social,
consequences; and if having favorable consequences was sickly, poor, mentally unstable, or dysfunctional people are
sufficient for goodness, that is, would guarantee goodness less likely to volunteer.
(and this is his mistaken assumption), then these supposedly (A) While the center might have a self-interested motive to
bad things with favorable consequences from the first attract new volunteers, there’s no evidence that this played
sentence would actually be good. But as we saw, the second any role in this argument.
sentence does not denote sufficiency, but rather necessity.
(B) Sure, the author does draw a conclusion about improved
So, Terry has confused the two, and so he (and therefore Pat
“well-being” based on evidence about the factors in (B), but
as well) has committed the flaw in (D).
what’s wrong with that? Those factors are at least plausibly
There was no need to explore Pat’s argument (unless related to well-being.
Terry’s argument totally blew you away), but for the record,
(C) Peers? The comparison offered in (C) is irrelevant since the
here’s why Pat makes the same error: Pat claims that
author never stipulates that all volunteers must belong to the
“no actions considered to be bad by our society have
same peer group.
favorable consequences.” This means that “not favorable
consequences” are a necessary feature of those actions (D) scrambles some terms in the argument. The newsletter
considered bad; if an action is considered bad, then it never claims that growing older will result in a change in
does not have favorable consequences; “not favorable mental outlook. Rather, the argument claims that volunteering
consequences” are necessary for an action considered bad. will cause a change in mental outlook, as well as diminish the
But like Terry, Pat mistakes necessity for sufficiency, assuming effects of aging.
that “not favorable consequences” are sufficient for actions
considered bad. If that were true, then any action that does 64. (C)
not have favorable consequences would be considered Because a higher percentage of red cars are involved in
bad, including the good actions that do not have favorable accidents, insurance companies charge higher premiums
consequences mentioned in the beginning of her argument. to owners of red cars. Our well-intentioned author then
This is how she reaches her conclusion, in agreement with goes on to make the remarkable conclusion that lives could
Terry, that some actions considered bad are actually good. She “undoubtedly” be saved by totally banning red cars. So, cars
has confused necessity and sufficiency (in the same way as drive themselves, do they? The author makes the mistake of
Terry) in reaching her conclusion. assuming that driving red cars actually causes more accidents,
(A) Neither speaker talks about many properties distinguishing and ignores, as (C) has it, the possibility that reckless drivers
different types of actions. might just like red cars. If that’s true and red is the color of
choice for awful drivers, then banning red cars won’t do any
(B) Neither speaker jumps from most to all.
good. The bad drivers would still be out on the road (just
(C) Other societies? driving a different colored car).
(E) Neither speaker attempts to rule out the possibility of other (A) The argument isn’t concerned with whether insurance
means of distinguishing actions. companies are justified in charging the red car owners higher
premiums. The author simply uses the insurance companies’
63. (E) statistics to support her conclusion: that banning red cars will
If, as the stem asserts, the inference drawn in the newsletter is save lives.
unwarranted, then there must be a problem with the reasoning
(B) Repair costs are outside the scope of the argument. just card player, or the convicted thug? These two facts alone
(D) The author isn’t obligated to name the actual percentage give us reason to suspect that maybe there is more to Judge
because her argument is based on a hypothetical. The claim Mosston than how he plays cards. (C) makes that point quite
about red car crashes isn’t hers, it’s the insurance companies’, nicely.
and the author argues hypothetically: “if this claim is true . . .” (A) Neither refraining from criminally assaulting another
(E) The author doesn’t say that every crash involving a red car person nor refraining from cheating at cards are duties
results in loss of life. She just says that if all red cars were “specific to legal professionals,” and so there can be no
banned and all the resulting crashes were avoided, then some confusion of those particular duties with the responsibilities of
lives would “undoubtedly be saved.” private citizens.
(B) The problem with this argument is whether Judge
65. (A) Mosston’s behavior while playing cards is evidence that
Chang does not dispute the fact that researchers think there supports keeping this convicted criminal on the bench; there
is no “manic-depression gene.” But Chang does enter into a is no issue raised by any supposed distinction between “fair”
game of semantics. Whereas Wirth feels that this evidence and “just.”
leads directly to the conclusion that no one is genetically (D) Don’t be distracted by a mischaracterization of the
predisposed to manic depression, Chang cites evidence that issues. The writer mentions the card playing not as a way
may prove another possibility—that a group of genes are of minimizing the seriousness of the assault, but rather, in
responsible for predisposing people to the disease. Wirth feels support of the claim that Judge Mosston is the type of “just
that because there is no single manic-depression gene, there and fair” judge our judicial system cannot afford to lose.
cannot be a genetic component to manic depression. Chang, (E) For this answer choice to be correct, we’d need to see
who takes issue with this conclusion, criticizes Wirth’s single- some evidence of popular opinion supporting the author’s
mindedness. He feels that Wirth presupposes, or assumes, argument. Here, the apparent popular opinion is that Judge
that there is only one possible result of the evidence—no Mosston should be forced to resign, which is exactly the
single gene, then no genetic component—when in fact there opposite of what this writer concludes.
is another possibility: multiple genes interacting to form the
basis of a genetic component. 67. (B)
(B) and (C) can be eliminated immediately because they deal Arguments based on correlations are loaded with
with Wirth’s misuse of evidence. As we’ve seen, Chang does potential flaws.
not dispute Wirth’s evidence.
This argument presents a chain of correlations: the size and
(D) can’t be Chang’s criticism of Wirth’s argument, because shape of a plant’s leaves are correlated to (dependent on) the
Chang himself introduces evidence that might disconfirm climate in which the plant grew. Likewise, climate is correlated
Wirth’s conclusion. Therefore, Chang can’t say, and doesn’t to (dependent on) altitude. The open question is how tight
say, that Wirth’s argument is structured so as to be immune to are those correlations? Altitude may determine climate, but
disconfirmation. over what range of altitudes can one see the same climate
(E) Chang does not accuse Wirth of making the stretch from patterns? Nothing in the stimulus tells us. That is precisely the
“unlikely” to “impossible.” His argument would have to problem identified in answer choice (B).
read like this for (E) to be correct: “a genetic component to (A) The ability of plant species to survive violent changes in
manic depression is merely unlikely, but you argue that it’s environment is outside the scope of the argument.
impossible.” This is not the nature of Chang’s response, so
(C) The argument only requires the relation between leaf size
(E) is incorrect. You may have axed (E) simply on the grounds
and shape to be correlated to climate, not for that relation
that Chang does not seem to feel that a genetic component to
to be the only physical characteristic of the leaves that
manic depression is unlikely at all.
is determined by climate. If the intensity of leaf color, for
example, were also determined by climate, it would not affect
66. (C)
the argument in any way.
When a writer tries to support a generalization with a limited
(D) The author does not rely on any supposed “analogy”
sampling of data, think about the flaw of representativeness.
between leaves and fossilized leaves—there is no
Here, the writer bases his entire argument about Judge reason to believe that the fossilized leaves are somehow
Mosston’s fairness on his observations of the judge during unrepresentative of the size and shape of the actual plant
card games. But we are also told that the judge, despite leaves.
his exemplary card playing, has been convicted of criminal
(E) The conclusion of the argument relates to the place where
assault. So, which is the “real” Judge Mosston? The fair and
the plant grew, not where the leaf fossilized. That the leaf may
have fossilized some distance away from where the plant grew not caring, and we can’t infer from that alone that he therefore
would have no effect on the argument. assumes the critics will profit from the bill’s defeat.
(D) It’s the critics of the proposed legislation who seem to
68. (E) prefer the less restrictive definition, and the politician has no
This is a classic example of the reasoning error sometimes duty to argue their case.
called “circular reasoning.” The conclusion is signaled by (E) is out of the scope. The author of the bill is never
the Keyword phrase “It is clear from this that . . .,” and notice mentioned, and therefore his or her credibility is not an issue.
that what follows is just a rewrite of the previous sentence,
“without self-understanding it is impossible to understand 70. (C)
others.”
Since the question stem tells us that the critic believes that
(A) The flaw described in (A)—mistaking necessity for the advocate’s argument is flawed, match the critic’s language
sufficiency—is an error often committed in GMAT Logical and logic to the relevant portions of the advocate’s argument.
Reasoning arguments, but not in this one. We’re told a The critic suggests that contribution caps (the advocate’s
condition (self-understanding) that is necessary for a particular second reason) will prevent the candidate from spending more
result (understanding others). So, answer choice (A) would be time serving the public (the advocate’s first reason), because
correct if the argument went on to conclude that “Anyone who candidates will need to spend more time finding more
understands himself can understand others.” But that’s not small contributors. As (C) states, the two projected results
the conclusion, of course; as answer choice (E) recognizes, the (more time with the public and less time working for large
conclusion we get is just the evidence restated. contributors) cited in support of the proposal (public campaign
(B) First of all, the first sentence does suggest that there funding) work against each other.
are some people who don’t want self-understanding, but (A) is outside the scope; the critic’s objection never mentions
beyond that, this whole issue of wanting or not wanting self- large contributors or ways to circumvent caps. Don’t read more
awareness is irrelevant. The author’s concern is whether one into an answer choice than you’re allowed. That can lead to
can understand others without self-awareness. missed points.
(C) No blame is assigned, and even if it were, there’s no (B) is a tricky half-right half-wrong answer choice. It starts out
indication that the people in question “cannot . . . be held strong by mentioning both results in support of the proposal,
responsible” for not understanding themselves or others. but fades quickly when it states that the two reasons are
(D) There’s nothing “inherently vague” about the term “self- actually one. The critic believes that the two reasons are
understanding,” even though different people could define the inconsistent, not that one entails the other.
dimension of that understanding in different ways. (D) mentions something, the possibility that large competitors
will stop contributing, that neither the advocate nor the critic
69. (B) mention. Thus, (D) can’t be a flaw in the critic’s argument.
What’s wrong with the politician’s reply? The politician argues (E) sets up a false contrast/distinction that isn’t present in
that the dispute over the definition of “wetlands” is “quibbling the critic’s response. The critic’s response does not mention
over semantics,” meaning that the definition doesn’t make a generous contributors or moderately generous contributors
real difference one way or another. But this is just his opinion; and never compares the two.
no evidence is given to support the notion that how the
wetlands are defined is irrelevant—it may make a whole lot of 71. (E)
difference to the critics of the bill. For example, the politician
The reasoning to be criticized here is the author’s claim that
ignores the possibility that a restrictive definition might reduce
Jerome is being disingenuous when he, Jerome, cites cost
the scope of the legislation’s impact. The politician argues
as the reason for declining Melvin’s vacation invitation. Why
that the legislation would be more demanding in areas that
must Jerome be lying, or perhaps fudging? Because, says the
are designated “wetlands,” but unreasonably dismisses the
author, Jerome cites cost every time Melvin tenders the invite.
question of which areas will receive that protection.
Well come on, so what? Maybe the combined cost of the trip
(A) distorts the argument. The politician does claim that the plus the lost wages is always the one and only reason Jerome
critics don’t really care about wetlands, but doesn’t make the says no to these impromptu jaunts. The author simply fails
further claim that the critics have the same motives as all the to take Jerome at face value, as (E) points out. Once again,
opponents of conservation. the imputation of bad motives to a person without proper
(C) Does the politician assume the critics will profit if the bill is evidence is shown to be faulty.
defeated? No, that doesn’t show up anywhere in the argument. (A) No attack on Melvin is being planned, so none need be
In fact, the only thing the politician accuses the critics of is “forestalled.”
(B) Melvin’s the one making the offers. It’s Jerome’s refusals (C) The conclusion asserts what’s true of a clear-cut debate
that are the topic of the argument. winner (i.e., her election chances aren’t bettered). That which
(C) If Jerome does prefer vacations planned in advance but might be true about a debate that ended uncertainly has no
simply fails to mention that to Melvin, that actually tends to bearing.
support the author’s suspicion that Jerome is fudging things. (D) This choice suggests that there may be “unpredictable”
(D) Tempting, perhaps, but the author doesn’t say that cost exceptions to the generalization that watching a debate
can’t be Jerome’s only reason, she says it can’t be the real doesn’t affect one’s voting. Those exceptions could be few in
reason. It’s the adequacy of the explanation that’s in question, number and, at any rate, may not redound to the benefit of the
which brings us back to (E). perceived debate winner, just as the author asserts.
(E) Like (A), this choice deals with voting likelihood, not voting
72. (B) behavior.
In this experiment, the researcher made the error of
assuming that kinesthesia was sufficient for maze-learning 74. (D)
simply because there was no evidence to the contrary. The evidence indicates that cars without airbags are less likely
Since the researcher never tested this point, it’s possible to get into accidents, but this doesn’t mean that cars with air
that kinesthesia, by itself, is not sufficient while still being bags are no safer. After all, it could be that the accidents are
necessary. (B) illustrates this point by suggesting the often fatal for drivers and passenger of cars with no airbags,
possibility that kinesthesia needs to be combined with at least while most drivers and passengers of cars with airbags survive
one other sense. accidents. The term safe, then, should take into account the
(A) Any “small differences” observed by the researcher are nature of injuries sustained in accidents, as well as the simple
irrelevant to the reasoning employed. All that’s important is number of accidents. (D) addresses this point nicely.
that the researcher reached a general conclusion from fairly The argument never assumes that most cars with airbags
similar results. will be in accidents (A). If airbag-less cars have other safety
(C), (D) First of all, neither of these choices can be determined features comparable to airbags (B), this fact would actually
from the data. Secondly, if they could, they would lean more in strengthen the argument, while the possibility that some
support of the argument that kinesthesia is sufficient, rather accidents involve cars both with and without airbags (C)
than damage it. Thirdly, both contain aspects that are beyond doesn’t affect the conclusion at all. The author never mentions
the scope—the mechanisms of transference and response to whether airbags will be deployed in all accidents (E), and it’s
non-kinesthetic stimulation. not clear how this would affect the argument anyway.
(E) The biggest difference between this and (B) is that (B)
75. (A)
correctly offers the interaction with another sense as a
possibility. Like the previous two choices, what’s stated here The argument is that people complain about ineffectual
can’t actually be determined by the data, so it isn’t a strike legislators, but these legislators are forced to be ineffectual
against the researcher’s reasoning. because they were elected to compete for funds. So, when
people express dissatisfaction, the legislators are just doing
73. (B) what they were elected to do. It might be difficult to predict
the flaw here, but be on the lookout for an answer choice that
For many students, “fails to consider the possibility that”
identifies an unwarranted leap in logic or terminology. (A)
is one of the easiest choices to predict, because they enjoy
identifies just such a leap. It’s possible that people are not
coming up with alternative possibilities.
just complaining about ineffectuality, but also high salaries,
The author is fairly pessimistic about the effect of a TV debate corruption, ugly suits, or any number of offences committed by
win on a candidate’s election prospects because the likely legislators.
watchers are already committed to a candidate and undecided
(B) addresses the issue of whether scarce funds could become
viewers are rarely swayed. What, you might ask, about those
less scarce in the future because of legislators. This doesn’t
who only hear about the outcome? If (B) is true, then quite
affect whether the legislators are ineffective now, or whether
possibly the people who don’t watch the debate are heavily
that ineffectuality is the cause of complaints. The argument
influenced by hearing about who won. That’s the alternative
deals with why people are dissatisfied now, so whether they
possibility that could turn winning a TV debate into a huge
would be more or less under different circumstances (C) is
election win.
not important, and is certainly not a flaw in the argument. The
(A) The argument hinges on whom the watchers would vote for, “fact” that dissatisfaction with legislators is inevitable (D) or
not on their likelihood of voting. peculiar to legislators (E) doesn’t influence the cause of that
dissatisfaction, so (D) and (E) can also be dismissed.
76. (C) Smith ever testified at all (B); what the attorney decries is the
References to circular arguments, those that “presuppose absence of refutation, not the guy’s unreliable testimony. And
what they seek to establish,” are usually made in GMAT the attorney never opines as to Lopez’s character (D) one way
wrong answers, but once in a while a genuinely circular or the other; the only grounds he gives for believing her is
argument comes along. This argument assumes that Cotrell (again) that Smith didn’t deny her testimony.
is a lousy writer in order to prove that he’s a lousy writer,
and that’s what circular reasoning, choice (C), is. The author 79. (B)
provides no independent evidence whatsoever as to the The philosopher moves from the specific—indicting individual
quality of Cotrell’s writing. Indeed, there is actually scientists for their self-aggrandizement—to the general, in
counterevidence— the “superior” articles—that Cotrell is his conclusion (“Hence”) that the scientific community as a
better than the author gives him credit for, but the author whole works toward self-aggrandizement. That is “groupthink”
blithely ascribes them to someone else, again with no bigotry and overgeneralization, and (B) skewers the
independent support. philosopher on it.
(A) is a popular wrong answer, but since the author explicitly (A) gets the relationship exactly reversed. (C) is false because
acknowledges that the counterevidence of the superior Cotrell the author does believe that the scientific community pursues
articles exists, he cannot be said to “ignore” it. If anything, the truth—albeit “only incidentally.” There’s no ambiguity (D)
author takes the bad Cotrell writing from which he is reasoning in the term “self-interest”; indeed, the author specifies that
as typical rather than atypical (B). No outside expertise (D) in a scientific context it means “one’s self-enhancement.”
is cited or even alluded to, and (E) is a total 180: Rather, The only cause-and-effect (E) that the philosopher posits is
the author infers limited ability despite what he sees as that scientists’ self-interest causes them to work primarily
occasional examples of high performance. toward their own career, but the move to the conclusion
is not a cause-and-effect, but rather (as (B) notes) an
77. (D) overgeneralization.
Is a musical scale popular because of “social conditioning”
or “innate dispositions”? The author rejects the former and 80. (C)
says that the latter is the “only” attributable cause of diatonic The prediction that the next proposal will probably be
music’s popularity. But if it’s not an either/or situation—if both approved is based on two facts: the last five got thumbs down,
could be a contributor to a scale’s popularity—then that would and “normally about half” get thumbs up. You needn’t know
cast serious doubt on the conclusion. (D) sums up the flaw. the total number of proposals submitted to recognize that
“Appreciation” is never mentioned (we can’t infer that it is the (C) is the problem: the supervisor has no reason to believe
same as “popularity”), and in any case the argument doesn’t that the last five turndowns are relevant, in any way, to the
take up the popularity of non-diatonic music, so both (A) and next budget decision. If, contrary to (C), the V.P. considers
(B) can be rejected. The author is trying to explain why a type each proposal on its own merits only, then the probability of
of music is popular, not (C) why multiple types of music exist the next one’s success is unaffected, at least in the way the
at all. Nonhuman species (E) are probably about as far outside supervisor argues.
the scope as one can get. (A) ’s scope extends to “the next five” proposals, but the
reasoning here concerns only the next one.
78. (C) Nothing in the argument suggests that the requirement that
Convict Smith of assaulting Jackson, the attorney argues, (B) mentions is in place (indeed, the wording of the “because”
because Smith has a violent character, because Lopez says clause suggests that the 50%-approval result is a habit rather
he threatened her, and because Smith didn’t refute that. than a rule).
There are lots of assumptions being made here, and even The merit or lack of merit of the past failed proposals (D) is
some of the wrong answer choices may have tempted you by irrelevant, as is the reason for the previous turndowns (E).
seeming to connect pieces of this chain. The only one that
really hits the spot, though, is (C), which points out the flawed 81. (C)
reasoning that since Smith didn’t refute Lopez’s testimony,
Something that the argument “takes for granted” is an
he’s making the unwarranted assumption that her accusation
assumption.
is in fact true. Sheer common-sensedictates that the absence
of a rebuttal doesn’t in and of itself render a charge true. Here, we get an extra clue from the question stem. Not only
is this argument flawed, it is flawed because it “takes for
(A) and (E) each qualify as a 180: The attorney wants the jury
granted” something that it shouldn’t have—in other words,
to believe that Smith’s verbal aggression does in fact indicate
it made a faulty assumption. We can treat this Flaw question
his violent character (A), and that Smith’s violent character
does support the charge of assault (E). We don’t know that
exactly like an Assumption question, looking at evidence her effect, and so recognizes that some other trick might be
and conclusion to find the gap between them. The author effective, and that there’s no coincidence at work here.
concludes that the historians he mentions neglect important
repositories of historical knowledge. Why does he think so? 83. (B)
Because most of them believe that written texts are the best Getting the right answer to this question requires you to
sources, and don’t regard other areas (painting, music, dance, spot a scope shift. The author concludes that all actions are
etc.) as the best sources of historical understanding. Notice motivated by self-interest. Why? Because any action that is
the different ways that the author talks about these sources apparently altruistic can be described in terms of self-interest.
in the evidence and the conclusion: the conclusion says that As additional support, the author then gives an example of
the historians neglect certain sources, but the evidence says an altruistic act, and proceeds to describe it in terms of self-
that the historians don’t consider these sources to be the best. interest. Do you see how this is a subtle scope shift? The
Well, just because you don’t think something is the best in its author is using evidence of how an action might be described
class doesn’t necessarily mean that you neglect it altogether, to support a conclusion of what is actually motivating the
but that’s what the author assumes is true. We find this idea action. These two things are not identical. (B) gets at this
restated in (C). discrepancy. Just because it could be true that helping
(A) Sources other than written texts and the arts are outside someone was motivated by self-interest, (in other words, you
the scope of the argument. could describe it that way) doesn’t mean that it is in fact true
(B) There is no reason to think that the author assumes that the action was motivated by self-interest.
the arts are not important for things besides historical (A) is wrong because the meaning of the term “self-interest”
understanding. does not shift. In each instance “self-interest” means exactly
(D) Whether or not texts are the best source is not part of the what it says, in the interest of the person performing the
author’s argument; he just argues in favor of the arts as an apparently altruistic act.
additional source for historical understanding. (C) It’s true that the argument doesn’t explain these terms. But
(E) is a 180; the author is trying to argue that the other sources that’s not why the argument is flawed. Defining these terms
mentioned are important. would have no effect on the underlying logic of the argument.
They’re clear enough for these purposes.
82. (A) (D) suggests that there’s a necessary/sufficient problem at
Once you know that there is an error in the reasoning, you work here, but there isn’t. The stimulus doesn’t even describe
should read critically, looking to poke holes in the author’s what’s necessary for an interest to be a motivation, so the
argument. In this paragraph, the skeptic concludes that possibility that, in this context, necessary = sufficient doesn’t
Debbie uses neither sleight of hand, nor a trick deck, nor a matter a bit.
planted “volunteer” to achieve her effect. This conclusion is (E) The argument does not depend on emotional content for its
based on three separate trials of her card trick. In each trial appeal. There’s no sob story here.
though, the skeptic is really only controlling for one variable
(using videotape to control for sleight of hand in Trial 1, etc.). 84. (B)
But as correct answer choice (A) points out, the skeptic failed A pretty darn rotten argument, from the looks of the stem. Four
to consider the possibility that Debbie did not always use the of the choices point out alternative possibilities that would
same method to achieve her effect. In other words, when the show the argument to be flawed, while the right answer will
skeptic tried to control for a trick deck, perhaps Debbie used be the only choice that doesn’t point to something important
a planted “volunteer” or sleight of hand to make the trick that the author has overlooked. And the argument itself isn’t
work. Then, when the skeptic tried to control for the planted too complicated: Personnel managers are interested in job
“volunteer,” Debbie used one of the other methods to make it applicants’ hobbies because passion for a hobby could signify
work. the passion that the candidate will bring to the job. But there’s
(B) The skeptic needs only one reliable method of detecting a problem here, according to the author: Devotion to a hobby
sleight of hand. may indicate that the applicant is “less concerned with work
(C) The skeptic has no need to entertain the possibility in (C). than with play”—and from this he therefore concludes that
Requiring two methods would make it harder for Debbie to pull inquiring about hobbies is a bad idea. Perhaps some pre-
off the trick, and would only increase the accuracy of the tests. phrases came to mind, perhaps not. In any case, you should
have recognized that the wrong choices—that is, the ones that
(D) , (E) The skeptic concludes that Debbie uses
point out something relevant that the author has overlooked—
neither sleight of hand, nor a trick deck, nor a planted
will focus on alternative possibilities that would suggest that
“volunteer” to achieve
asking about hobbies does have benefits. The right answer will
be the one that doesn’t point out such an omission, or that children’s desires, but rather in what happens when parents
even supports the argument. With these clues in mind, let’s wish to musically educate their children.
evaluate the choices: (C) It doesn’t matter whether a child’s musical ability is poor
(A) , (D) Ability to make long-term commitments (A) and or brilliant. In either case, the author claims, children won’t
possessing good organizational skills (D) seem like good get a good musical education unless parents ensure formal
things to look for in a candidate. If information about hobbies instruction. This is an example of necessity vs. sufficiency: The
can indicate these things about an applicant, then inquiring author argues that formal instruction is necessary for a strong
may be a good idea after all. musical foundation. He doesn’t say that formal instruction will
(B) seems largely irrelevant, involving as it does candidates guarantee musical virtuosity.
with no hobbies. We could, however, even make the case that (E) This argument concerns the measures parents can take to
(B) supports the argument: If inquiring about hobbies may help their children develop a strong musical foundation. The
cause some candidates to lie, thus distorting the interviewing fact that these measures may not be necessary to provide that
process, then the author may be right that avoiding the foundation in all cases does not injure the argument.
topic is wise (unless, of course, the interviewer knows that
the candidate is lying, in which case valuable info about 86. (D)
the candidate would be revealed; but we have no way of Another simple—and bad!—argument for us to contemplate:
knowing that this would be the case). So, whether you saw Since no one has shown that underinflation or overinflation
(B) as irrelevant or as a possible strengthener, it fits the doesn’t harm tire tread, therefore they must harm tire tread.
bill as the one choice that doesn’t point out a possible flaw It’s like explaining to Columbus that the world must be flat
in the reasoning. because no one has proven that it’s round. (In fact, it even
(C) is the flip side of (B): Putting candidates at ease and thus hearkens back to the esteemed attorney in the last question
encouraging honesty are both good things, so (C) qualifies as who argued that the lack of evidence for his client’s sanity
another overlooked benefit that could result from inquiring during a shooting proves his client was insane.) Even if you
about hobbies. have little knowledge of formal logic, your intuition should
(E) comes at the issue from the other side; that is, from the tell you that this turkey doesn’t fly. Clearly the author ignores
angle of the interviewer. If allowing personnel managers full another simple yet plausible possibility—the possibility, stated
freedom in the interviewing process has positive side effects, in (D), that underinflation and overinflation don’t harm tire
then, once again, we can conclude that the author’s proposed tread, even though it has yet to be proven.
ban on hobby-talk may very well be unwarranted. (A) The testmakers enter yet another bid for circular reasoning;
once again, unsuccessfully. The argument attempts to
85. (D) demonstrate that underinflation and overinflation harm tire
More flawed logic: After reading the first sentence, we know tread. However, the author doesn’t simply restate evidence
that a good musical education provides children with a to this effect (which would make the argument circular),
strong foundation for their musical abilities. That much is but rather argues that an effect that hasn’t been shown not
certain. But then things become a little less definite. We’re to occur must in fact occur. Bad reasoning, but again, not
told that formal instruction is often a part of a good musical circular.
education. This strongly implies that formal instruction may (B) Since when is this issue “not in principle susceptible to
not always be a part of a good musical education; otherwise, proof”? We’re talking about tires here, not the Meaning of Life.
why would the author say “often” instead of “always”? Yet the The wording of (B) may sound impressive, but it’s meaningless
argument proceeds to treat formal instruction as if it were an in the context of the stimulus.
indispensable aspect of good musical education, the failing (C) and (E) This may be a lousy argument, but the problem
described by (D). isn’t that it fails to specify how underinflation or overinflation
(A) It seems that the author does consider the fact that parents harms tires, or that it fails to define those terms. Even if the
aren’t the only one involved in this process. They’re the ones argument did specify a harm—uneven wear on the treads,
who will ensure that the kids get formal instruction, but say—or gave the most exacting definition of underinflation and
presumably this instruction will come from music teachers. overinflation, the conclusion would be just as invalid.
(B) Perhaps not, but no matter how children feel about musical
education, parents can’t provide a strong foundation without 87. (E)
ensuring formal instruction—at least, not if the author’s The stem tells us that a flaw’s afoot, so keep your eyes
correct. Bluntly put, the author’s not really interested in the peeled. The attorney reasons this way: Since the accusers
had evidence that Ziegler was sane sometime after he pulled
the trigger, but produced no evidence that he was sane when can’t recall—there are no “incomplete recollections” here. This
he actually did the deed, he must have been insane at that concept is found nowhere in the stimulus, and (B) is therefore
moment. Well, this reasoning is downright silly: Arguing that the correct choice. As for the valid complaints against the
since there’s no evidence to prove he was sane, he must commissioner’s argument:
have been insane is akin to arguing that since nobody has (A) By providing no evidence to demonstrate a lack of bias,
proven that I was not Henry VIII in a previous life, I must have the commissioner’s argument does take for granted that
been Henry VIII in a previous life. And, in fact, the attorney the neighborhood association’s information is reliable and
ignores the likelihood that being found sane later on may be unbiased. In this matter, however, it is reasonable to suspect
relevant to his state of mind during the incident—if anything, that this is not the case.
it could just as easily attest to his sanity during the attempted
(C) By basing the decision entirely on the association’s report,
murder as the lack of evidence of his sanity during that time
the commissioner in effect is saying, “This is the only evidence
supposedly proves he was insane. (E) points out this plausible
worth considering.” Again, it is reasonable to suspect that
alternative explanation that the attorney overlooks (or
more evidence may be needed.
conveniently ignores).
(D) , (E) The commissioner admits that he didn’t study the
(A) doesn’t even come close. We don’t know that Ziegler is a
report “thoroughly,” and the last sentence of the stimulus
well-educated professional; the stimulus doesn’t mention his
refers to the association’s earlier recommendation, but gives
education or even his professionalism. (A) is merely a weak
us no reason for why that decision might be related to this
nod toward the word “consultant” in the stimulus.
one. Both valid criticisms.
(B) is a little tough to figure out, but look at it this way: The
lack of evidence for Ziegler’s being sane is not the conclusion 89. (C)
here, as (B) posits—it’s the evidence that the attorney uses to
This flaw question may seem trickier than most, so consider
try to prove that Ziegler was insane when he pulled the trigger.
the evidence and conclusion carefully. The editorialist argues
There is no real “evidence against Ziegler’s being sane”; the
that there is no reason to deny the privileges of adulthood
only “evidence” is the lack of evidence for his being sane.
to 17-year-olds. Why? Two pieces of evidence back up this
(C) Ziegler apparently worked as a consultant both before he conclusion: 1) because society is obligated to bestow those
pulled the trigger and after (he “continued” his consulting privileges upon persons who are mature enough to accept
work). We have evidence that he was sane at least part of the corresponding responsibilities; and 2) 17-year-olds are
time afterward; what happened beforehand is beyond the physiologically mature. Did you notice the scope shift at work
scope of the argument. here? Both pieces of evidence deal with maturity—but in
(D) Does the author mention moral responsibility? No. But each case, it’s a different kind of maturity. The first refers to
the presumption of a connection between sanity and legal behavioral maturity—the maturity to accept responsibility—
responsibility may be safely inferred, but even then, that while the second refers to physiological maturity. The
wouldn’t constitute a flaw in the argument. editorialist equivocates over maturity, choice (C). (If you’re
unfamiliar with the concept of “equivocation,” pay special
88. (B) attention to the bullet point below.)
Talk about lousy logic! The stem tells us that four of the five (A) The argument doesn’t assume that 17-year-olds should get
choices will contain legitimate criticisms of the commissioner’s all of the privileges of adulthood; there is evidence regarding
argument, and that we’re to find the one criticism that doesn’t maturity that’s presented in support of this conclusion. What
apply; that is, the one that’s irrelevant to the argument. the argument does assume is that physical maturity is the
Well, here’s what we know: The commissioner made a same as emotional maturity.
decision about a power plant. The decision was based solely (B) What examples? There are no examples. The author draws
on a report prepared by the neighborhood association. a general conclusion based upon general evidence.
The commissioner didn’t study the report carefully. In the
(D) The only claim accepted by the author is science’s claim
past, the commissioner also agreed with the neighborhood
that 17-year-olds are physically mature. There’s no reason
association’s report on jail relocation.
to believe that science is an inappropriate authority on this
It would be very difficult to pre-phrase the correct answer, matter, nor that there’s anything wrong with the use of this
so we should quickly work through the answer choices, evidence.
eliminating the valid critiques as we go. (B) is the one that
(E) True, the author doesn’t deal with sixteen-year-olds,
falls short: It criticizes the argument as based on “incomplete
but why should she? She says only that “most” seventeen-
recollections.” Now, the commissioner does many things here,
year-olds are physiologically mature, which leaves open the
but forgetting what the association said on the matter isn’t
possibility that some younger kids may be as well. In any
one of them. He does not base his conclusion on specifics he
case, whether she ignores sixteen-year-olds or not, she has no As (D) has it, the economist has erred by concluding that the
obligation to include them here, and (E) is not a valid critique ozone layer has a calculable value from the mere fact that it
of the argument. has an upper limit.
(A) starts off OK, but then goes south. The economist
90. (B) isn’t talking about establishing upper limits for all natural
The author of the press release argues that, because a resources.
comprehensive review of medical studies showed no link (B) accuses the economist of circular reasoning, but there is
between drinking normal amounts of coffee and heart disease, some independent evidence in the economist’s position (i.e.,
it is therefore safe to drink coffee, period. But the argument the upper limit). Besides, the conclusion in (B) doesn’t match
assumes that a connection to heart disease would be the only that of the stimulus—the economist never argues that the
thing that could make drinking coffee unsafe. Can you think of ozone layer shouldn’t be protected. His attempt to show that
other possibilities? Choice (B) suggests that you might have one potential obstacle to doing so is not an obstacle at all
a healthy heart while the rest of your body is a wreck. What seems to imply that he’s at least open to the idea of protecting
effect does drinking coffee have on general health? The press the ozone layer.
release doesn’t say. An argument that concludes that coffee
(C) is a common GMAT flaw that is not present here. The
is safe overall requires evidence about more than just heart
word “value” is used the same way throughout.
disease.
(E) The economist does directly address the environmentalists’
(A) No, the review did not study patients, but reviews are not
argument: they say there’s no way to calculate the value of the
designed to study patients. The review evaluated other studies
ozone layer, and the economist says there is. It’s his reason
of real patients. To whatever degree the other studies were
for saying so that’s faulty.
valid, the review would also be valid.
(C) What coffee drinkers might choose to eat with their coffee 92. (B)
has nothing to do with the safety of the coffee itself. People
“Vulnerable to criticism” means “containing a logical flaw.”
who drink anything might eat harmful foods at the same time.
Some people find certain of Handel’s music derived from
Even if some coffee drinkers eat foods that are bad for the
religious texts to be “religious music,” implying that for those
heart, the foods cause the harm, not the coffee, so (C) doesn’t
people, “religious music” means music with specific religious
weaken the argument that coffee is safe.
messages. Those same people believe that Vierne’s organ
(D) The press release doesn’t draw conclusions about symphonies are not “religious music,” presumably because
beverages other than coffee. The possible effects of other they are not accompanied by religious texts. The music
beverages are outside the scope of the argument. critic counters that Vierne’s works are religious, because the
(E) This choice involves a shifting of cause and effect. Stress experience of listening to them demonstrates that they are
harms the heart, and, by the way, also may cause people to “divinely inspired.” Huh? “Divinely inspired” doesn’t mean
drink coffee. In this scenario, drinking coffee might be an the same thing as “having a religious message,” but the critic
indicator of stress, but coffee doesn’t cause the stress or the treats them as the same. So, the critic confuses two different
harm associated with stress. So, as far as we know, it’s still meanings of the word “religious,” (B).
safe to drink coffee, at least as far as the heart’s concerned. (A), (E) The critic makes no claims about all religious music (A)
or all organ symphonies, (E).
91. (D)
(C) True, the critic does overlook this possibility, but her
The stem tells us we’re dealing with a Flaw question, so we argument does not depend on the notion that all organ music
need to see how the economist’s evidence fails to support is divinely inspired. Just like (A) and (E), the critic makes no
the conclusion. The first six lines basically boil down to this claims about all organ music.
notion: to decide what we should do about the ozone layer,
(D) What two meanings of “symphonies?” Organ symphonies
we have to know what the ozone layer is worth; we must
are mentioned twice, both times referring to the works of
calculate the monetary value of the ozone layer. Those pesky
Vierne.
environmentalists say such a calculation is impossible, but we
do know that it’s less than all the money in the world, since we
93. (C)
wouldn’t sacrifice all of the world’s economic resources just for
the ozone layer. Just because it’s less than that huge figure, Mr. Nance believes that someone is lying to him because
does that mean that we can precisely calculate how much the Ms. Chan said she retired from the company and another
ozone layer is worth? The economist has merely shown that person said that Ms. Chan was traveling around working
the dollar value of the ozone layer is less than some figure, harder than she ever did before. That didn’t sound like
but that isn’t the same as showing that we can know what it is. “retirement” to Mr. Nance, so, in his mind, someone must
not be telling the truth. Mr. Nance has made the mistake of manager is just trying to show that safety problems were not
interpreting the word “retirement” to mean “not working very the reason that this nuclear power plant was closed.
hard” when in this context it really means “not working for the (C) The plant manager’s response doesn’t touch on the
same employer anymore.” Ms. Chan has only “retired from issue of public perception; the public is alluded to only in
the company” which means she simply is not working for THAT the phrase “antinuclear cause” in the first sentence of the
company any more. Ms. Chan may be working for herself or activist’s argument. The manager simply debates the activist’s
someone else or just tending to her own personal business second claim regarding what the plant closing shows about
but still staying busier than she was before. (C) sums it up: the power industry’s attitudes toward the operation of such
this is a typical case of the error of equivocation. The author plants.
confuses two different meanings of a single term.
(D) In so far as the overall industry’s attitude can be said to be
(A) As they remind us frequently on Law & Order, the legal represented by the manager’s reasons for this plant’s closing,
term “hearsay” describes information that is heard second- the manager’s conclusion, not his premises, contradicts
hand and is not allowed into court as evidence in certain the activist’s view of the power industry’s attitude toward
circumstances. Often, however, hearsay information can be nuclear safety. Even so, that’s not a flaw in the argument; his
sound evidence upon which to base an argument. So, it conclusion is allowed to contradict the activist’s views—but as
is not necessarily logically unsound to base one’s we see in (E), it’s his reasoning that’s suspect.
reasoning on hearsay. In this particular case, does Mr.
Nance give too much credence to information heard 95. (B)
second-hand? No, he goes out of his way to express
The author’s mathematical reasoning is pretty crummy, and
skepticism about the report of Ms. Chan’s activities, so (A)
picking a few sample numbers demonstrates why. Suppose
doesn’t describe a flaw in his reasoning.
the Labour party was founded in 1815 and commanded 1,000
(B) Mr. Nance does not attack Ms. Chan personally. If he voters. By 1825, according to the author, they’d have five
did, he would say disparaging things about her as an times as many or 5,000 voters; and five times that, or 25,000
individual. Even though he has misinterpreted a key term, voters, by 1835—in short, a net gain of 4,000 voters over the
he does stick to the issue at hand. first decade, and 20,000 more over the second. Pick whatever
(D) Technically, we can’t be sure that Ms. Chan has even numbers you choose, but there’s no getting around it, the
spoken to the colleague. In any case, Mr. Nance is sufficiently second decade must have shown a greater voter gain than
suspicious of all the claims in this scenario. the first. The “often-made claim” is solid, and the author’s
(E) is not a logical flaw. There is nothing unsound in failing to conclusion disputing it is as wrong as wrong can be as (B)
infer that a person is of superior character just because they points out.
have worked loyally for years. (A) A decade is a decade, and it doesn’t matter which decades
we’re talking about when we’re told the rate of increase
94. (E) in each.
The manager denies the activist’s claim that the plant closing (C) Au contraire, the argument concerns Labour party voter
represents an acknowledgment on the part of the nuclear gains and all of the evidence does likewise.
power industry that it can’t operate such plants safely. Not (D) This argument is solely about voter numbers, not policies
at all, insists the plant manager, who believes that the plant or the reasons for voter support.
closed because of economic, not safety considerations; it
(E) might be relevant if the argument were about votes rather
was simply impossible to compete with other sources of
than voters. The data measuring numbers of supporters are
cheap power, especially given the cost of “mandated safety
irrelevant to the number of elections at any given time.
inspections and safety repairs.” But, as (E) points out, the
manager is ignoring the fact that the cost of “mandated safety
96. (C)
inspections and safety repairs” is part of running a plant
safely. Those costs arise as a result of safety considerations. Now here’s a classic scope shift: Arjun says that hacking at
Thus, if such costs contributed significantly to the plant hospital computers could damage data systems, and then
closing, the power plant manager’s argument actually plays arrives at the unwavering conclusion that computer crimes
right into the activist’s hands. definitely do cause physical harm to people. In one breath,
a particular result of computer crime is only possible; in the
(A) isn’t a flaw, since the manager and activist are only
next breath the result from this “possibility” (physical harm to
discussing the significance of this particular plant’s closing.
people) is a foregone conclusion. (C) nicely picks up on this
(B) isn’t an oversight. It doesn’t matter to the manager’s subtle shift that Arjun employs for the sake of his argument.
argument if other sources of power have safety problems. The
(A) Arjun disagrees with Yolanda’s assessment of the (B) An example of mistaking correlation for cause-and-effect:
distinction between the level of danger associated with “All of the students at Drake Middle School wear uniforms, and
joyriding and computer crime. However, he doesn’t break they’re all well-behaved; it must be the uniforms that make
down the distinction or argue that there’s no difference them behave.” It’s a common error, but it’s not committed
between the two kinds of crime. here.
(B) If there’s one thing Arjun does, it’s provide evidence—in (C) The argument, such as it is, is general in nature. That
this case in the form of an example. The problem is how he Morton ends by mentioning “a person” doesn’t change
uses his evidence, not a total lack of evidence to support his things—he’s not speaking about one specific person.
stance. (D) Morton does consider the counterexamples: he looks
(D) Huh? What “something”? No such mistake is made: Arjun at them and finds them wanting; they are only “apparent”
simply supplies evidence in the form of one example that is counterexamples.
intended to lead to his conclusion. The problem is that the (E) Morton’s evidence and conclusion are his own beliefs, and
way the example is used to lead to the conclusion is flawed in do not imply that which “most people believe” in the slightest.
the way described in correct choice (C). As for the concept of
necessity, this plays no role in Arjun’s short argument. 99. (D)
(E) Arjun’s evidence is not strong enough to lead to his There are lots of characters here, so make sure you keep them
conclusion. However, this doesn’t mean his evidence is straight. First, there’s Joshua Smith, who wrote a book. Then
inconsistent with his conclusion; that is, the evidence in no there’s a book editor who criticizes the book as “implausible.”
way conflicts with or contradicts the conclusion. Finally, there’s the author of the stimulus, who criticizes the
editor’s criticism, stating that it is “unwarranted.” Got all
97. (A) that? Now, on what grounds does the stimulus author feel the
Frequently the flaw of an argument is a lack of support for its editor’s criticism of the book is off base? Each of the incidents
central assumption. in the novel “could very well have happened”—in other words,
Louis concludes that people’s intentions cannot be, on the each incident is “plausible.” But just because the individual
whole, more bad than good, because were we to believe events ring true doesn’t mean that they will do likewise when
otherwise, a series of consequences would follow, ending in strung together. The author is committing what is sometimes
the destruction of society. Louis assumes (without support) called the “part to whole” fallacy, the assumption that that
that something that would result in the destruction of society which is true of the parts must be true of the whole. (D) states
cannot occur. That’s answer choice (A). the flaw; the “given characteristic” mentioned is “plausibility.”

(B) Louis does not assume that we must either believe or (A) and (C) both key off the sideswipe at the editor in sentence
not believe his conclusion because the two beliefs must be 2, but that catty reference to the editor’s previous off-base
mutually exclusive. Rather, he assumes that we must believe criticisms is just background information, not central to the
his conclusion because the alternative is a path that leads to author’s rebuttal. (Note that that entire phrase could be
the destruction of society. dropped from the paragraph with no damage to the logic.)
(C) There is no personal attack made on the opponents of (B) criticizes the author for relying on people’s judgment
Louis’s argument, let alone any mention of those who would concerning whether an individual incident is plausible.
oppose his argument. Granted, we might be mistaken concerning what is plausible.
But the fact remains that even if each event in the book is
(D) 180. Louis actually assumes that where there are two
plausible by itself, the book as a whole could be implausible.
possible outcomes, the most negative will not occur.
So (B) fails to address the central issue here.
(E) The argument is based on claims about what is true of
(E) The evidence isn’t necessarily relevant only to those who
people’s intentions as a whole and society as a whole—
already believe the conclusion. One could accept the evidence
there is no distinction being made between two groups of
that each of the novel’s incidents is plausible, yet reject the
individuals.
conclusion that the entire novel is likewise.
98. (A)
100. (D)
Compare the first two lines of Morton’s speech (his
The senator’s position—that no work of art can be
conclusion) to the last two (the evidence, signaled by the
obscene, and hence that the world of art has nothing to
Keyword “since”). All he does is repeat his claim, that success
do with protecting the public—is attacked by means of
requires a college degree, but he provides no independent
counterexamples, works of art that people call obscene. His
evidence for same.
response? “[I]f [they] really are obscene then they cannot be
works of art.” But that statement is just the contrapositive elite. That “category” (that of the elite) could just as readily
of the meaning of his original remark. “No work of art can be apply to Walcott then as now.
obscene” means “IF a work of art, THEN not obscene.” Reverse
and negate both terms and you get the senator’s statement 102. (A)
in the last sentence. So, what’s wrong with the reasoning? Certain common flaws appear regularly on the GMAT.
The senator provides no independent evidence as to the
A simple Flaw question that employs one of the most common
nature of obscenity and the nature of art—he defends the
logical fallacies on the GMAT has to be refreshing. As is often
original principle by invoking its contrapositive, that is, its
the case with arguments involving drivers, this author sounds
logical equivalent. So, he is guilty of “circular reasoning” or,
like he’s not far out of high school. His friends are all worried
as (D) puts it, assuming the truth of that which he is trying to
that he’ll get into an accident because he’s such a reckless
prove.
driver. Yet he’s convinced that all he has to do is trade in
(A) The senator uses no emotional language or appeal. He his sports car for a minivan, and he’ll be less likely to have
is trying to persuade by logic, but he fails because he begs an accident, since minivans and sedans have low accident
the question by restating his conclusion as if it offered more rates compared to sports cars. What he doesn’t realize (and
support. That’s what (D) is saying. what should be obvious to us) is that accidents are caused
(B) Au contraire, his points are logically equivalent (as by drivers, not by cars. Drivers of minivans tend to be more
mentioned above, contrapositives of one another)—not cautious than drivers of sports cars, so minivans have a lower
contradictory. accident rate than sports cars. The author won’t be able to
(C) would be true if he alleged (however absurdly) “Art is not take advantage of that accident rate unless he starts driving
obscene because I am a senator and what I say goes.” There like the typical minivan driver instead of the typical reckless
is, of course, no such assertion of authority here. sports car nut. He’s fallen into a classic logical flaw: mistaking
a correlation (minivans have low accident rates) for causation
(E) Again, his appeal is to his own conclusion; he veers off not
(driving a minivan causes fewer accidents). That’s (A).
into the irrelevant, but into the redundant.
(B) Even though there is a mention of a statistic, there’s
101. (A) no sample here—the rate of accidents for different types of
vehicles comes from data about all accidents.
The conclusion, first of all, is a comparison. On the issue of
which classification will better reflect the scientific significance (C) is a pretty good definition of the flaw of Possibility
of some fossils, the author votes for a recent one over the one vs. Certainty, but that flaw isn’t present here.
proposed by the discoverer of the fossils, Walcott. But the (D) also mentions a common flaw that isn’t present in the
author doesn’t (as she should) discuss the superiority of this argument: Necessity vs. Sufficiency.
recent classification over Walcott’s; she simply argues that as (E) No source, well-informed or otherwise, is cited in the
a member of the science elite, Walcott’s view would simply argument.
mimic the views of the establishment. As choice (A) has it, this
is drawing a conclusion about a position based on the source 103. (E)
of the position rather than on the position’s actual merit.
A complex, verbose stimulus needs to be reduced to its bare
(B) The first piece of evidence cited is that Walcott was a big bones.
shot in the scientific elite, which is more “verifiable” than (B)
This peroration is awash with adverbs and adjectives and
lets on. The second piece is a conclusion (sentence 3) that
repetition that can set your head spinning if you don’t
follows from the first, and it too could be verified: One could
take pains to reduce the logic to its essence. Starting with
verify whether Walcott simply confirmed what established
the Conclusion Keyword “therefore,” you might formulate
science deemed true.
something like this: The best understanding of nature requires
(C) See the analysis of the evidence in (B), above. These nontraditional (holistic) reasoning, rather than traditional
premises don’t contradict each other; the second follows from (scientific) reasoning. That’s because nature evolves in a
the first. holistic way and is itself holistic and interconnected.
(D) Denying that “not X” can be true in order to prove that
This kind of distillation may reveal the author’s unwarranted
“X” is true is questionable logic, yes, but not the issue here. scope shift between the holistic characteristics of nature
The evidence concerns Walcott’s standing in the scientific mentioned in the evidence, and the holistic reasoning for
community and ignores his science; that’s the problem. which the author argues in the conclusion. And that should
(E) jumbles past and present. That Walcott’s classification was lead you to (E). It’s an unwarranted assumption that to best
made in the past is not relevant to the main objection, which understand a holistic phenomenon (like nature) one must
is that his views were simply mimicking those of the scientific
apply holistic reasoning; for all we know, a traditionally linear (C) No, the circularity here is that it concludes that a
reasoning approach might be equally adequate to the task. proposition (the commentator’s first sentence) is true
(A) accuses the argument of confusing necessity and because it’s true, not because it’s been tested and no one has
sufficiency. While that’s a flaw plaguing many GMAT LR disproved it.
arguments, this isn’t one of them, as our distillation above (D) A speaker need not be so open-minded as to be “aware
demonstrates. The problem is not the sufficiency of evidence of any evidence that could undermine” his argument. He
but the inappropriateness of evidence: that is, a conclusion would be wise to consider it and defend against it in advance,
about reasoning needs evidence about reasoning, not but he’s not required to do so, so (D) is not describing the
generalizations about the essence of nature. commentator’s logical flaw.
(B) The philosopher doesn’t argue that linear reasoning cannot (E) The commentator asks us to “suppose” a complete
come to grips with nature, simply that it falls short of the best scientific cataloguing of a human action. This is not unlike
way to do so because nature is organic rather than linear. (B) a Logic Games question’s “if” clause, in which we are to
is on the right track, but since the author could concede a suppose that G goes to the picnic, at least for that question
possible difference between the structure of nature and the only. In both cases, we are not to argue with the supposition,
structure of thinking, without compromising his insistence that as (E) would have us do; we’re supposed to accept it as
holistic thinking is the best approach, (B) doesn’t point to the true and proceed from there. So, the commentator is not
argument’s vulnerability. “presuming” that the supposition is true; he is asking us to
(C) The argument cuts right past any distinction between parts presume that it’s true for the moment, in order to press his
and wholes—almost ruthlessly so. One can only attack an case.
argument on its own terms, and the terms of this argument are
the leap from the holistic traits of nature to a holistic approach 105. (D)
of analysis. Don’t be fooled by (C)’s use of terms from the Certain flaws can be phrased more than one way—for
argument! Just because the author mentions something example, overlooking an alternative possibility is really the
doesn’t mean it will be in the right answer. same thing as confusing necessity and sufficiency.
(D) , like (C), appeals to a specious and irrelevant If only things were really as simple as this counselor claims,
distinction between the whole and the parts. we could all learn to be more accepting of ourselves and
others. Unfortunately, the question stem tells us that her
104. (B) argument doesn’t hold water, and our job is to figure out why.
When an author “presumes the conclusion to be true,” he is The counselor gives us the results of comparing ourselves to
offering no independent evidence for it. others. This is a pretty simple cause-and-effect argument, and
To prove that human behavior requires consideration of as such can be phrased as formal logic: if the other people
people’s nonphysical elements, the commentator argues that are more able or more successful, we’ll end up disparaging
even if every physical element of an action was catalogued ourselves; if the other people are less able or less successful,
and understood, we still wouldn’t fully comprehend the action. we’ll end up being dismissive of others. But this only tells us
On what evidence? The commentator is essentially arguing what happens when we compare oneself to others. As in any
that “we can’t fully understand people on a totally physical Formal Logic statement, we can’t be certain what happens if
level, because we can’t fully understand people on a solely we don’t pull the trigger—if we don’t compare ourselves to
physical level.” He provides no independent evidence for others. Unfortunately, the counselor bases her conclusion
that lack of understanding (all he says is “obviously” we still on exactly that, and therein lies the flaw in her argument. We
wouldn’t understand), and that’s what makes the argument know from our work with Formal Logic that you can always
feel like it’s going in a circle. get the result of a Formal Logic statement without pulling
the trigger. In this case, that would mean that it’s possible
Indeed, this is a classic example of “circular reasoning,” in
to become self-disparaging or dismissive of others without
which the author presumes his conclusion to be true in order
comparing oneself to others. The counselor overlooks that
to prove it. You’re not responsible for the term, but you should
alternative possibility, mistaking one sufficient cause for a
spot the flaw when it appears. (B) describes it in appropriate,
result as the necessary cause, as (D) points out.
abstract terms.
(A) The results of comparing oneself to both groups would
(A) There’s no analogy here. An analogy is a comparison of two
almost certainly be a combination of the results of comparing
essentially different things. Here the body is analyzed from the
oneself to each group separately. As long as these results
perspective of its two components: physical and nonphysical.
aren’t mutually exclusive (and they aren’t here), the author’s
No analogy.
reasoning allows for that possibility, so this isn’t our flaw.
(B) The beneficial effects of making comparisons between 107. (D)
oneself and others are outside the scope of the argument; Be on the lookout for scope shifts in Logical Flaw questions.
the counselor is only concerned with avoiding the detrimental
Routinely failing to punish rule breakers may very well lead
effects of the comparisons.
to chaos, but that’s not an argument for never allowing an
(C) just doesn’t make much sense. If one is dismissive exception. (D) expresses that sentiment almost verbatim.
of others, one isn’t accepting of them, and if one is self-
(A) The argument does no such thing. Chaos, according to the
disparaging, one isn’t self-accepting. Even if the author
stimulus, results when violators routinely go unpunished. The
does take this for granted, it’s common sense, not the
stimulus never discusses the effects of eliminating violators
argument’s flaw.
altogether.
(E) Although the argument does not acknowledge those who
(B) The reasons the rules were put in place are irrelevant.
are neither more nor less successful than oneself, filling this
in as something the author “takes for granted” does nothing (C) Like (A), this choice irrelevantly deals with violators rather
to rectify this flawed conclusion, as a correct assumption (no than with the punishment of same.
matter how (E) No, the author takes for granted that routinely overlooking
every transgression results in chaos. That’s very different
106. (A) from (E).
Always try to predict an argument’s logical flaw, especially if
its topic is particularly familiar or easy to relate to. 108. (D)

The X-Files and Medium, anyone? This TV executive urges us The argument boils down to this: books allowed traditional
not to take seriously the claim that such shows will make schools to develop. Books are being phased out by electronic
people more superstitious and scientifically ignorant, on media. Therefore, traditional schools will also be phased out.
historical grounds: ghost stories have always prevailed even The problem with this argument, as summed up in (D), is
as people have gotten steadily smarter, scientifically assuming that books are necessary for the continued existence
speaking. What might have occurred to you, as you looked for of traditional schools, just because books were instrumental in
an alternative explanation, is that “maybe the public would the development of those schools. (It’s possible, for example,
have been even more scientifically aware if those ghost stories that electronic media could replace books within traditional
hadn’t been around.” Correct choice (A) puts it more formally: schools, allowing those schools to continue to exist.)
the historical evidence doesn’t categorically support the idea Circular reasoning as described in (A) is a common wrong
that ghost stories can’t hold back the public’s understanding answer choice on Logical Flaw questions; here, for example,
of science, even as that understanding does advance over the conclusion that electronic media will bring an end to
time. traditional schools is not used as evidence. There is no expert
(B) The executive isn’t taking for granted any causal testimony cited here, so we can rule out (B). (C) states that
connection. Indeed, he’s trying to sever one, by showing that the argument “presupposes that because something can
paranormal stories don’t have to lead to a dumbing-down of happen it will happen,” but the argument is not constructed
the public. to establish that something “can” happen. Instead, the media
consultant argues that that electronic media will definitely
(C) “The former does not impede the latter,” eh? Well, in
mean the end of books, which will definitely mean the end
the context of the argument, the former = ghost stories
of traditional schools. Since the argument never establishes
and the latter = public understanding of science. But the
that something “can” happen, we can’t say that the author
executive isn’t arguing that ghost stories affect the public’s
then makes the logical error of “presupposing that it must
scientific knowledge, directly or indirectly, so (C) can only be
happen.” The media consultant never discusses the value of
understood as gobbledygook.
the institution of school, so (E) is way off base.
(D) even more than (C) ventures into the realm of the mutual
effects of phenomena, when in fact all the executive wants 109. (E)
to do is separate paranormal TV shows from the public’s
This one boils down to a very simple argument: the speed limit
command of science.
was reduced a year ago; there have been fewer fatalities since
(E) More Hijinx with “former” and “latter.” If you substitute then; therefore, speed limit reductions can reduce fatalities.
the actual terms for the abstract ones, (E) becomes “the Remember that correlation doesn’t imply causality. In other
contention that ghost stories cause scientific ignorance must words, something else may be responsible for the reduction in
be baseless if the scientific ignorance has persisted despite fatalities, and (E) provides just such an alternate explanation.
steady advances in the pervasiveness of ghost stories.” Say If the number of fatalities the previous year was abnormally
what?! high for some strange reason, then the number this year
might have been lower even without the lowered speed limit— (A) implies that the author is attempting to go along with
maybe the speed limit had nothing to do with the reduction in the “general belief” when in fact he’s seeking to prove an
fatalities. exception to it. (B) comes out of the “After all” sentence,
We would expect an increase in traffic to result in an and misreads it; either/or’s are not mutually exclusive, so
increase—not a decrease—in fatalities, so (A) would actually sentence 4 cannot be read as if warming and volcanic activity
strengthen the argument. (B) and (D) both move outside the must be inconsistent, as (B) charges. The author establishes
scope of the argument by discussing how many people obeyed nothing—that’s what (E) points out—so the confusion between
the law (B) and how well it was enforced (D). Since we only event and cause mentioned by (C) has no connection with the
care if the law had an effect by whatever means, we don’t care argument as written. And the problem with the reasoning isn’t
if people are driving more carefully because of the law but that the fossil counterevidence is too narrow (D), it’s that the
still speeding, or driving more slowly even without the police, counterevidence can be used to draw many other conclusions
or any other set of circumstances, as long as the new law than the one drawn.
somehow decreased the number of fatalities. (C) is interesting
because it must be true for the argument to be true, but 112. (A)
it’s not something that the argument takes for granted. Not until the last sentence do we really find out what the
(Remember, that’s what the question is asking for.) Instead, author’s getting at. “Since” (evidence) we need many beliefs
it’s something the entire argument is constructed to prove. to survive, the statisticians’ claim (implied “therefore”)
must be false. That claim, described at length earlier, is that
110. (B) one makes one’s beliefs-set correct by eliminating the bad
In this unusual twist on the standard logical flaw question, you ones without taking on new ones. The author disputes this,
first have to decide who has committed the logical misstep, implying that such a method could lead to death. But that
and then describe it. Sandy is mentioned in three choices, but doesn’t make the statistician’s claim wrong; it only suggests
actually hers is the sounder reasoning. There’s no problem that it could have bad consequences. Consider the following
with her suggestion that “it’s best to play” Bigbucks after a parallel: “Bob says the surest way to make a lot of money is to
few winner-less weeks, because she never defines “best” as work 20-hour days. But that would mean you’d be unhappy,
“most likely to win.” Her point is that after a few winner-less so Bob is wrong.” No, Bob may very well be right about how to
weeks there’s a bigger prize, and there’s no arguing with make money; the evidence hasn’t countered the claim at all.
that. (A), (C), and (E) are all wrong because Sandy makes no The speaker has assumed, with no evidence, that making
judgment call on the chances of winning. money shouldn’t conflict with one’s happiness. In the same
way our author has assumed, with no evidence, that having
It’s Alex who makes the boo-boo, because his suggestion
mostly correct beliefs shouldn’t conflict with survival (A).
makes no sense. In a game that’s won by picking five
numbers, how could your chances of winning be affected by Note that (B), (C), and (E) cannot possibly be correct because
the number of other players? (B) points out the fallacy in Alex’s none of them mentions survival, and the right answer must
arguing that Sandy is “more likely to win” when the player do so: The argument’s flaw is that it introduces the idea of
pool is small. As for (D), it points at Alex, all right, but Alex “survival” at the very end, with no connection to the rest of the
makes no reference to past week’s winners. logic. Beyond that, the statisticians’ rule says not to take on
new beliefs, so the author hasn’t neglected the possibility that
111. (E) one could do so (B). (C) is an irrelevant comparison; it might
be true that the author overlooks this possibility, but it has no
The only evidence that the ice sheet must have melted
direct bearing on the conclusion. (D) discusses “beliefs related
3,000,000 years ago is that a bunch of fossils of that age,
to survival,” a distortion of the two very different stimulus
usually only found on the ocean floor, were found in the sheet.
terms. (E) goes against the grain of the conclusion; the author
But you need not be a paleontologist to figure that there are
never suggests, or takes for granted, that correct beliefs are
a lot of ways, short of the ice sheet melting, in which those
better for survival than incorrect ones.
fossils could have appeared there. That such fossils were
“previously found only in” one place doesn’t mean that they
113. (B)
can’t be found all over the earth. The creatures could have
slipped through a hole in the otherwise solid ice sheet and Pravastatin reduces heart attacks and reduces cholesterol.
frozen as fossils there. All you have to predict is that the right But the author of this stimulus assumes that the drug reduces
answer will say, Not enough evidence to support the cause heart attacks by reducing cholesterol. In other words, it
and effect, and you can choose (E) with confidence. (E) rightly confuses correlation with causation, and (B), by focusing on
deems the fossils an “inconsistency” and condemns the the issue of consequence, gets this exactly right.
reasoning on the proper grounds.
(A) deals with side effects and (E) with whether the drug relied upon. (E) distorts the expert issue in a different way.
is used widely or narrowly, both of which are well outside An argument need not be faulty simply because some experts
the scope. (C) is not a flaw, since the past findings provide disagree with it.
additional evidence for the conclusion. (D) suggests the
conclusion should focus on pravastatin and cholesterol levels, 116. (B)
but that relationship is declared as part of the evidence The biggest single flaw—and it appears as three flaws among
(lines 2–3); why should the author make any more effort to the wrong choices—is the lawyer’s failure to recognize that,
draw a conclusion about that relationship? notwithstanding all the indications of Congleton’s support,
Congleton still could have wanted the project to fail and the
114. (D) witness is not lying.
“Fails to” signals that there is something lacking in the logic; The possibility that Congleton tried to sabotage the project by
“thus” signals the conclusion. hiring incompatible people, (C), or by underbudgeting for it (E)
Two studies come up with diametrically different conclusions certainly exists, as does the possibility that Congleton didn’t
about the length of the average workweek over 8 years, and have the freedom to choose people who might sabotage the
the author’s conclusion is that the different methodologies project (A).
explain the different results. But why couldn’t different The remaining wrong choice deals with the lawyer’s inference
methodologies come up with the very same results? (D) points about the witness. It’s not the witness’s fault if he looks at
out that indeed, they could, and so the reason for the different all of the listed facts and for whatever reason fails to infer
results may lie elsewhere. that Congleton wasn’t rooting for the project’s failure. In that
Contrary to (A), the author does distinguish between the two case, the witness wasn’t lying; he simply does believe that
studies, at least insofar as he acknowledges fundamental Congleton wanted the project doomed (D).
differences between them, and in any case, we cannot assume (B) is no flaw in the lawyer’s logic because it misses the point.
that the nature of each study’s sponsor has anything to do The issue here is what Congleton wanted and what the witness
with the results. The purposes, (B), of the study are irrelevant believed, not whether the project did or did not fail.
to the scope of the argument, which deals solely with methods
and results. There’s no evidence that only one of the two 117. (C)
studies was properly run, (C); for all we know, both were. (E)
The alleged chain of phenomena (low family wages → lower
deals with the substance of the studies, but the dilemma
operating expenses → higher profits → greater family
here is methodological only. After all, both studies looked
prosperity) hits a snag early on. Since a reasonable definition
at the very same “economic conditions” and came up with
of “a family’s financial prosperity” would have to touch on, at
very different conclusions, and so (E) comes nowhere near
least, how much money the members of the family possess,
explaining the argument’s flaw.
there is a strong likelihood that running a family business
in the way the author suggests would weaken the family’s
115. (C)
prosperity. That is what we get in (C).
The stem tells you, going in, that the argument is faulty.
(A) drags in the utterly irrelevant issue of customer
The complaint that psychologists are wrong about “deep satisfaction. The argument sticks to dealing with one type of
empathy” is that the impossibility of grasping someone business, so it can’t be accused of (B)’s generalizations about
else’s motivation would seem to make understanding people the effects of the “lowest” or “highest” commodities. The
impossible as well, and yet we do understand people. The motivations behind the family members’ willingness to work
problem with that reasoning is the scope shift that the author at low pay (D) are as unmentioned as they are irrelevant to the
unwittingly commits. The psychologists have deemed “deep conclusion. (E) suffers from the same problem as (B), namely
empathy” the best way to understand people in theory. overgeneralization from one specific case.
That such depth of understanding might be impossible in
practice doesn’t contradict the psychologists’ view of what is 118. (A)
theoretically best. (C) has a handle on this.
Once again, an GMAT argument author is using faulty
“Deep empathy” is given a solid definition, (A), and there’s logic; no big surprise there. The columnist concludes
no reason to believe that we need a better one. (B) describes that it is “impossible” for there to be evidence that lax
an argument that contradicts itself, whereas the stimulus radiation standards at nuclear reactors actually
author is describing what she sees as a contradiction within contributed to the
his opponent’s argument. (D) is what happens when you take increase in cancer rates near those sites. And the columnist’s
an expert’s word on something just because s/he is an expert, evidence that it is impossible for there to be such evidence?
but the only experts in this argument are being rebutted, not Simply the fact that no one can say if a particular case of
cancer is due to radiation, smoking, or other factors. Did you So (D) is correct. Engle thinks that McKinley is referring to the
notice the scope shift there? Many flawed arguments turn on therapeutic effects of the drug, but she could be referring to
such subtle scope shifts. In this instance the columnist takes some known side effects.
evidence about what may have caused a single case of cancer (A) If (A) were correct, Engle would suggest that some
and uses it to support a conclusion about whether there is alternative test exists. But he doesn’t.
evidence that radiation contributed to an increase in cancer
(B) Au contraire, Engle suggests that McKinley assumes the
rates. Therefore, you should have been looking for an answer
drug will be effective.
choice that addresses this scope shift. (A) does the job. If, for
instance, there were statistical evidence that cancer rates had (C) Engle’s statements don’t suggest that the placebo will
increased by 80% during the relevant time period, that would have some effect, and besides, the entire point of a placebo
be evidence that the standards had made a contribution, is that it’s a substance that has no effect on the patient. Engle
even if you couldn’t prove conclusively that any one of the doesn’t challenge McKinley by calling into question the very
individual cases was due to radiation. possibility of administering a placebo. If he did, he would say
something like “every substance administered to a patient has
(B) says that the argument incorrectly assumes that where
some effect.”
Y follows X (cancer follows lax radiation standards), X must
have caused Y (the radiation standards caused the cancer). (E) is tough, but Engle doesn’t say that McKinley doesn’t know
Upon rereading the argument, you see that the columnist did whether the drug worked. He just says that McKinley shouldn’t
nothing of the sort. The columnist argues that there isn’t proof assume that it works.
of a causal link here.
120. (D)
(C) is incorrect because the author specifically does not draw
any conclusions about the causes of cancer. His conclusion is Sometimes there’s more than one flaw in a Flaw question.
that we will never be able to find evidence that one particular Don’t worry if you don’t have a clear-cut prediction.
factor (radiation exposure) caused cancer. This one is terse, and yet, there’s a lot wrong with it, so don’t
(D) The author does not ignore other possible causes. In fact, worry if an answer doesn’t jump off the page. According to the
the author specifically considers other possible causes of author, a chess-playing computer would show us either that
cancer in the last sentence. computers can think or that chess doesn’t involve thinking.
In either case, our notions of human intelligence will never be
(E) is tricky, but not if we remember the terms of this argument.
the same. Let’s go through the answers:
The author hasn’t concluded that a causal connection between
lax radiation standards and cancer doesn’t exist; he merely (A) is a position that our author would surely agree with,
concludes that it will be impossible to find hard evidence given the last statement of the conclusion. A flaw isn’t ever
for it. something the author is going to agree with.
(B) The argument is hypothetical: “a successful chess-playing
119. (D) computer would prove.... ” The fact that such a computer will
The question stem says that Engle is interpreting McKinley’s never be designed is therefore irrelevant.
remarks in a peculiar way, so we have good reason to suspect (C) That may be the case, but the argument at hand only
that some misinterpretation is going on. Let’s start with what applies to chess.
McKinley actually says. McKinley says that a double-blindtest (D) pokes a hole in the argument. Given (D), a computer can
won’t work in this case, since the drug will certainly affect play chess without thinking, and the argument falls apart. For
the patients in a way that the doctors will recognize. So, the the record:
doctors will know whether the drug or the placebo had been
(E) is referring to humans, but the argument is focusing on
administered. Here’s where Engle comes in. Engle says one
computer chess players.
cannot be sure whether a double-blind test is possible, since
we don’t know whether the test will be effective. In other
121. (E)
words, Engle is saying that we can’t be sure that the drug
will have any effect at all, and so it may be possible for the The argument is bogus because it doesn’t recognize that some
researchers to administer the test without being able to tell set of circumstances is consistent with the facts as provided
whether the drug or a placebo was administered. Fair enough, that would demonstrate that the author’s conclusion is too
but the real question is what McKinley meant when she hasty. So, we’ll hit the stimulus looking for that possible
referred to the known effects on the patient’s bodies. If she situation. All the reporters get all their info from the press
meant the therapeutic effects, then Engle has a valid point. agent, and if the press agent tells them all the entire story,
But McKinley could have been referring to known side effects, then no one reporter can scoop all the others. Sounds
which would be noticeable whether or not the drug worked. reasonable so far. But the logic goes astray from there.
If everything’s told to all, then the reporters know equal the reverse correlation from which we can infer nothing about
amounts. But we find out that everything’s not told, and the coffee-guzzling Tom’s sleeping habits. The author mistakenly
author assumes from this that the reporters therefore don’t reverses the direction of the correlation.
know equal amounts. That’s the only way she can conclude (A), (B) and (E) all focus in some way on the issue of
that some reporter can scoop the others. But this assumption causation, but neither the research nor the author gives us a
is faulty, and for a familiar reason: “If X (agent tells everything reason to conclude that coffee drinking causes insomnia, only
to all), then Y (reporters know equal amounts)” is not the that the two are correlated, in a specific way: Among extreme
same as “if NOT X (agent doesn’t tell everything to all), then insomniacs, lots are coffee drinkers. (A), in fact, refers to a
NOT Y (reporters don’t know equal amounts).” The reasoning is group that doesn’t even exist in the stimulus: The 10% left
flawed because despite the agent not revealing everything to over from the 90% figure cited in the research would have to
all, it’s still possible for the reporters to know equal amounts refer to the 10% of extreme insomniacs who don’t drink large
(perhaps the agent said nothing to anyone). If that’s the case, amounts of coffee. Since coffee as a causal agent isn’t the
then no scooping is possible. (E) is perfectly consistent with issue, other so-called “causes” are irrelevant here as well (B);
the argument, and would invalidate the conclusion if true. and (E) not only wrongly focuses on causation, it’s also way
(A) is tough, but technically, it doesn’t point to a counter too extreme in its use of the word “always.”
example. In the situation described in (A), all the reporters (D) isn’t a flaw: It’s perfectly valid to infer something about
have at most partial information. But it’s still possible that one an individual based on evidence about the characteristics
reporter knows more of the picture than the others, and could of a class of individuals—if you get the terms straight. As
therefore scoop the others. So (A) doesn’t point out a situation mentioned above, if this author concluded that “since Tom is
where no one can scoop the others. an extreme insomniac, he probably consumes a large amount
(B) is a quicker kill. The argument concerns whether some of coffee,” we wouldn’t be looking for a flaw at all.
reporter can scoop the others. Whether any reporter will scoop
the others is one step removed from that issue. 123. (B)
(C) If there’s an overlap between the facts known by the The question stem tells us there’s a flaw in this argument.
different reporters, then no one reporter can scoop all the This is a helpful hint, as the error in the stimulus is pretty
others with respect to those commonly-known facts. But it’s subtle. The mayor says that we cannot afford the restoration
still possible for one reporter to scoop all the others if they given the requirement of financial restraint. The concerned
have greater access to all the other facts, so (C) doesn’t point citizen counters with a description of the building’s historical
to a “no scoop” situation, either. importance. There’s nothing wrong with that, but the
(D) indicates that one reporter knows everything the press concerned citizen’s parting words contain a faulty comparison.
agent knows. But then this reporter could scoop anyone who The concerned citizen asks “can we afford not to” in response
knows less, so (D), rather than demonstrate that no one can to the mayor’s question “can we afford to.” However, the
scoop anyone else, tends to suggest that some reporters could two are using different meanings of the word “afford.” The
scoop other reporters. So (D), if anything, goes in the opposite mayor refers to financial affordability, whereas the sense of
direction. affordability used by the concerned citizen is more general.
The concerned citizen isn’t referring to money in her use of
122. (C) “afford,” and so she doesn’t really respond to the mayor’s
argument. (B) describes this problem.
The author of this short argument wins the prize for speed
of misinterpretation: No sooner does she finish citing a Picking up on this flaw wasn’t easy, but you could have
statistic than she applies it in bogus fashion to an actual made an educated guess after eliminating choices that don’t
example. The conclusion in the second sentence would seem describe a flaw in the concerned citizen’s argument.
reasonable if it were established that a large percentage of (A) is too strong. There is an appeal to the value of history, but
heavy coffee drinkers were extreme insomniacs. But has what’s wrong with that? In any case, the concerned citizen’s
that been established? No. The research cited involves a argument isn’t merely an appeal to the value of history,
correlation that goes the other way: Among those who are since preserving a link to the days of the city’s founding is
extreme insomniacs, most drink a lot of coffee. From this we connected to “maintaining respect for our city government and
could validly conclude that if Tom is an extreme insomniac, authority.”
he probably consumes a large amount of coffee—after all, 9 (C) The concerned citizen disagrees with the mayor, so she
out of 10 of them do. But the fact that he drinks a lot of coffee doesn’t appeal to his authority.
doesn’t mean beans (sorry): The flaw, as (C) points out, is
that the evidence doesn’t tell us what percentage of heavy
coffee drinkers are extreme insomniacs, but instead offers
(D) If anything, the mayor assumes that the restoration would Determinists don’t claim that one can know the complete state
be expensive, and the concerned citizen merely agrees but of the universe; they only claim that the complete state of the
argues that the expense is worth it. universe fully determines future events, which only requires
(E) The mayor uses those terms, not the concerned citizen. that there be a complete state of the universe. The author,
however, misses this nuance, as (D) points out, and in so
124. (B) doing constructs a faulty argument against determinism.

The word “only” in the stimulus should alert you that your (A) and (C) are true statements, but don’t point out a flaw
formal logic skills will be tested here. “Only computer in the reasoning. Yes, the fact that one cannot know both
scientists understand the architecture of personal computers” features of subatomic particles doesn’t imply that one cannot
means that if you understand the architecture, then you know either individual feature. But the issue of “knowing”
must be a computer scientist. Similarly, “only those that is brought up by the author, and is not, as far as we’re told,
understand the architecture of personal computers appreciate a component of determinism, which is concerned with the
the advances in technology” means that if you appreciate relationship between what occurs and what previously existed
the advances in technology, then you must understand the that prompted that occurrence.
architecture. Combining these statements, it follows that (B) mixes up the language of the stimulus. The author claims
anyone who appreciates the advances must be a computer that since the states of subatomic particles are unknowable,
scientist. But the author concludes something different: it is impossible to know the current state of the universe. But
“only those who appreciate these advances are computer (B) gets this backwards. Simply switching around the author’s
scientists” means that all computer scientists appreciate the premises doesn’t reveal the flaw here.
new advances. But we can’t conclude that. As mentioned (E)’s criticism is itself flawed. If something is true of any
above, we can deduce that all those who appreciate the individual in a group, then it must be true of all members
new advances are computer scientists, but that still leaves of that group. So far from pointing out a flaw in the author’s
open the possibility that some computer scientists do not logic, (E) adds to the confusion.
appreciate the advances. (B) describes this omission.
(A) and (D) are too extreme. There is an implied relationship, 126. (A)
a valid conclusion to be drawn here (see above). It’s just that This stimulus is abstract and difficult to navigate, but it’s
the author draws a different conclusion, and that’s the flaw. more manageable when you distill it down to its essence.
(C) is outside the scope. The question is whether computer The existence of a moral order (defined as: good rewarded,
scientists appreciate the advances in technology, not whether bad punished) depends on immortal souls. That means that
they appreciate other developments. if a moral order exists, then souls are immortal. The next
(E) No such presupposition is made. In fact, the author sentence (more than half the stimulus) is basically filler—
states flat out that only computer scientists understand the fodder for the wrong answer choices. The last sentence
architecture of personal computers, so that familiarity is announces the conclusion: if souls are immortal, then the
relatively rare—unless for some bizarre reason everyone in the bad will be punished. But hang on, immortal souls were
world is a computer scientist (what a world that would be . . .). just a requirement for a moral order to exist; the existence of
immortal souls doesn’t guarantee that a moral order exists,
125. (D) but the essayist concludes that it does. So, behind all the
abstract language, the essayist commits a classic GMAT
It’s difficult to imagine a topic that could be more abstract
flaw. Immortal souls are required for the existence of a
than this one. But even though the writing can be intimidating,
moral order in which the bad are punished, but their
we know from the stem that this author’s argument is weak.
existence isn’t sufficient to show that such an order exists.
First, we get a definition of determinism. Determinists believe
(A) describes this flaw.
that every event has a cause. So, the complete state of the
universe at one time fully “determines” the complete state of (B) The essayist does cite different beliefs about immortality
the universe at later times. Rubbish, says the author, since related to various forms of moral orders, but he doesn’t parade
we cannot know the complete state of the universe (e.g., we these beliefs as facts. He merely cites them as examples
can’t know everything about particular subatomic particles). without commenting on the merits of such beliefs.
Notice the scope shift here: The author doesn’t argue that (C) mixes up the evidence and the conclusion. The author’s
the reason determinism is false is because there can be no conclusion is that if souls are immortal, then the bad will be
complete state of the universe at any given moment, but punished, and supports it with the claim that if a moral order
rather because this totality can never be known. But the exists, then souls are immortal, not the other way around.
argument for determinism is based only on the existence of
such a state, not on man’s complete knowledge of that state.
(D) Yes, the author does make a generalization about different But the statistician presents evidence—the correlation in the
senses of moral order (they’re both based on some form of first sentence—that appears to provide a counterexample
immortality), but this isn’t a flaw. There’s nothing logically to the accepted opinions of meteorologists. As (E) says, the
faulty about claiming that different moral orders have common meteorologist simply falls back on the authority of the rule—
features. the accepted opinion—without taking a fair look at evidence
(E) The evidence and conclusion are different, it’s just that the that might prove the case otherwise.
one doesn’t adequately support the other. (A) First of all, the statistician’s explanation isn’t intended
to be “partial”; he says the Sun’s luminosity “essentially
127. (B) controls” the Earth’s land temperatures. And secondly, the
Reading the question stem first alerts us to the fact that there meteorologist doesn’t reject the explanation as incomplete,
is a flaw in this argument, so we know to look carefully for it as but says it can’t be correct based on the rule that a single
we read. 50% of people believe that an official should resign variable can’t control anything so complex as the Earth’s
if indicted for a crime. This means that half the people believe temperature.
that every indicted official should resign. In other words, (B) introduces an irrelevant distinction. The meteorologist’s
for these people, an official’s indictment is sufficient to call point is that the Earth’s climate is a complex system, and no
for that official’s resignation. 35% of people believe that an significant aspect of such a system can be controlled by a
official should resign only if they are convicted, which means single variable, whether that variable should be considered
that these people believe that only convicted officials should “part of the system” or not.
resign. In other words, for these people, conviction is required (C) is simply wrong. The meteorologist never denies the
for resignation. The author then concludes that more people correlation between changes in the Sun’s luminosity and
believe that elected officials should resign if indicted than changes in the Earth’s land temperatures.
believe that they should resign if convicted.
(D) The meteorologist never belittles the importance of the
Granted, 50% is more than 35%, but the author has neglected statistician’s hypothesis.
an important distinction, the difference between “if” and “only
if.” 50% of people believe that every indicted official should 129. (E)
resign, but we aren’t told what percentage of people believe
Here’s some looniness to give us a good chuckle: our legal-
that every convicted official should resign. We only know the
beagle author here concludes that criminals cannot be held
percentage of people that believe that conviction is necessary
responsible for their crimes; rather, law-abiding citizens
for resignation—a different issue. For all we know, 100% of
are to blame. How does she reach this bizarre conclusion?
people believe that every convicted official should resign, it’s
She absolves criminals from responsibility by arguing that
just that 35% of people believe that no other factor should
their actions are products of the environment that form their
lead an official to resign. Thus, the author has confused
characters. She then maintains that this “environment”
necessity with sufficiency, and committed the flaw in (B).
is created solely by the good guys, and therefore it’s their
(A) No, the author does not generalize to a whole population actions, and nothing else, that make them alone responsible
based on a sample. The author makes no generalizations for crime. Hold on a minute: on one hand, the author invokes
at all. “environment” to forgive criminals for their actions, but this
(C) What ambiguity? “environment” issue disappears when it comes to the actions
(D) The author does draw conclusions about the two beliefs of law-abiding citizens. She states early on that “criminal
cited in the poll, but there is not a separate, third belief that is actions, like all actions, are products of environment . . .”
the basis of the author’s conclusion. If this is true, then the law-abiding citizens can’t be held
responsible for their actions either; the author can’t have it
(E) It is possible that the premises can all be true, as described
both ways. There’s a definite contradiction here, and hopefully
above.
that was enough to prompt you to choose (E), despite its
seeming complexity. But (E) does characterize the flaw: the
128. (E)
implicit principle the argument is based on is that all actions
When faced with the case described by the statistician, the are products of the environment that influenced the agent
meteorologist cites a general rule, which she says is the performing the action. If this is to hold for criminals, then it
universal opinion of professional meteorologists. That’s an must hold for law-abiding citizens as well. The conclusion
appeal to authority. In many cases, an appeal to a legitimate in the last sentence pins all the blame on the actions of the
authority (like professional meteorologists) carries some good guys, and nothing else, and thus violates the spirit of the
weight, but in a case like this it’s not appropriate. Presumably, principle stated earlier in the passage.
meteorologists have based their opinion on past observations.
(A) The concept of “environment” is introduced and then (E) Dobson equivocates on no term, key or otherwise
referred to later on as “this environment.” Contrary to (A), the (“equivocation” means using the same word in two different
author is consistent in her use of this word. meanings or senses). He changes no meanings en route.
(B) The concept of “social acceptability” is at least one step
removed from the argument. One might argue that there is a 131. (D)
distinction made (a somewhat nutty one) between criminal The GMAT writers have many ways of expressing the
behavior and the behavior of law-abiding citizens, but it’s logical flaw of confusing what is sufficient with what is
vague at best how and if this distinction relates in any way to necessary.
“social acceptability.” The stimulus tells us that if an orchid species is pollinated
(C) is wordy and tricky, but maybe you saw why it’s wrong: the solely by insects, then it has features that attract insects.
author doesn’t distinguish between criminals and crimes. A The argument concludes by stating that an orchid that
criminal is someone who commits crimes—the author wouldn’t possesses features that attract insects must be pollinated
argue with that. Her point is that criminals shouldn’t be held solely by insects, a classic reversal of sufficient and necessary
responsible for their crimes, not that people who commit conditions.
crimes shouldn’t be thought of as criminals. This may sound (D) is one way to express that concept: the argument treats
strange, but this is a strange passage, isn’t it? a characteristic (insect-attracting features) known to be true
(D) What statistical evidence? of one class of things (orchids that are pollinated solely by
insects) as if that characteristic were unique to that class
130. (A) (assuming that if an orchid has insect-attracting features, then
The claim that Dobson tries to rebut—that the ancient builders it must be pollinated solely by insects).
of this ring of stones must have known about celestial (A) The argument does not attempt to prove that two kinds of
events—is based on the provocative fact that a line drawn orchid species have only one characteristic in common.
between two of the stones points to a significant phenomenon (B) The argument is not about two classes of things, nor is
in the sky. Phooey, says Dobson—there are many stones it about there being only one characteristic that is different
in the ring, so many that the odds of any two pointing in between two classes of things.
a provocative way are huge. But not content to leave well
(C) The problem with the argument is that it does assume that
enough alone, Dobson announces that he has proved that the
a characteristic (insect-attracting features) is unique to the
ancients didn’t know about celestial events. Whoa! Maybe the
class of orchids that are pollinated solely by insects.
evidence for the original claim is skimpy, but Dobson goes too
far in claiming that that skimpiness proves the claim false. (E) The argument is not based on a generalization based
That’s what (A) is getting at. Let’s try an analogy—a parallel on what is known about one member of an unrelated class.
argument, as it were. Suppose you argue that the players Rather, the author is trying to argue that one member that
on the Columbus Kings football team must be churchgoers, possesses a characteristic known to be possessed by all
because every Sunday I see some of them walking toward a members of a certain class must belong to that class.
church. Well, in response, I could argue that that evidence
is insufficient, that it doesn’t prove your point, that any of 132. (D)
them could be seen walking towards some house of worship; The first task in this one is to make our way through the
but I cannot say that I have therefore proved that the players complicated stem. Boiled down to its basics, the question
aren’t churchgoers! This is essentially the same error that the simply asks us to assess the second argument as a response
stimulus author commits. to the first. In other words, does it work or not, and why?
(B) Dobson’s evidence contradicts the original claim. That’s Notice that the first three choices begin with “it succeeds” and
what a rebuttal does. But he doesn’t contradict himself. the last two start with “it fails.” That makes it easier; we can
narrow the choices down first by a gut reaction to the validity
(C) As described above, the evidence falls far short of
of the response, and then go on to pinpoint the choice with
“absolutely establishing” Dobson’s conclusion.
the correct reason.
(D) Whether the ancients knew about celestial events is a
Well, did you think the response worked? Hopefully not; the
question subject to factual verification and not a matter
Advertising-Sales director misses the point. The Magazine
of opinion. Of course, the right kind of evidence must be
editor is worried that readers will perceive a decline in
assembled, which is why (A) is correct. But it is a fact-based
editorial integrity on the part of the magazine if the magazine
dispute.
favorably mentions products in articles. The Advertising-Sales
director counters, “Don’t worry about it: people know the
difference between articles and ads, so the response to the
ads has nothing to do with the magazine’s editorial integrity.”
Here’s where you should have begun scratching your head and
muttering, “Huh?! What does that have to do with anything?”
That one quick gut feeling is enough to kill choices (A), (B),
and (C)—no way does this response succeed, it’s barely talking
about the same thing. The editor is talking about a reader
response to articles and the director is talking about a reader
response to ads.
Now to narrow down the precise reason why the response
fails. The editor doesn’t care, at least in this argument, about
readers’ response to ads—she’s worried about their response
to the articles written to glorify the advertisers’ products.
(D) cites this as the correct reason why the response fails as a
counter to the editor’s concern.
As for (E), the editor doesn’t state a view about reader
response to advertisements, so obviously the advertising-
sales director can’t misunderstand it.
Evaluate

1. It is illegal to advertise prescription medications in Hedland except in professional medical journals or by mail
directly to physicians. A proposed law would allow general advertising of prescription medications. Opponents
object that, in general, laypersons lack the specialized knowledge to evaluate such advertisements and might ask
their physicians for inappropriate medications. But since physicians have the final say as to whether to prescribe a
medication for a patient, the objection provides no grounds for concern. Which of the following would it be
most useful to establish in order to evaluate the argument?
A. Whether nonprescription medications can interact with and block the action of any prescription medications that
could be advertised to the general public
B. Whether most prescription medication advertisements directed at the general public would be advertisements for
recently developed medications newly available by prescription
C. Whether prescription medication advertisements directed at the general public would appear on television and
radio as well as in print
D. Whether physicians are more likely to pay attention to advertising directed to the general public than to
advertising directed to physicians
E. Whether physicians are likely to succumb to pressure from patients to prescribe inappropriate medications

2. Community activist: If Morganville wants to keep its central shopping district healthy, it should prevent the
opening of a huge SaveAll discount department store on the outskirts of Morganville. Records from other small
towns show that whenever SaveAll has opened a store outside the central shopping district of a small town,
within five years the town has experienced the bankruptcies of more than a quarter of the stores in the shopping
district. The answer to which of the following would be most useful for evaluating the community
activist’s reasoning?
A. Have community activists in other towns successfully campaigned against the opening of a SaveAll store on the
outskirts of their towns?
B. Do a large percentage of the residents of Morganville currently do almost all of their shopping at stores in
Morganville?
C. In towns with healthy central shopping districts, what proportion of the stores in those districts suffer bankruptcy
during a typical five-year period?
D. What proportion of the employees at the SaveAll store on the outskirts of Morganville will be drawn from
Morganville?
E. Do newly opened SaveAll stores ever lose money during their first five years of operation?

3. Journalist: Well-known businessman Arnold Bergeron has long been popular in the state, and he has often talked
about running for governor, but he has never run. However, we have just learned that Bergeron has fulfilled the
financial disclosure requirement for candidacy by submitting a detailed list of his current financial holdings to the
election commission. So, it is very likely that Bergeron will be a candidate for governor this year. The answer to
which of the following questions would be most useful in evaluating the journalist’s argument?
A. Has anybody else who has fulfilled the financial disclosure requirement for the upcoming election reported greater
financial holdings than Bergeron?
B. Is submitting a list of holdings the only way to fulfill the election commission’s financial disclosure requirements?
C. Did the information recently obtained by the journalists come directly from the election commission?
D. Have Bergeron’s financial holdings increased in value in recent years?
E. Had Bergeron also fulfilled the financial disclosure requirements for candidacy before any previous gubernatorial
elections?

4. Magazine Publisher: Our magazine does not have a liberal bias. It is true that when a book review we had
commissioned last year turned out to express distinctly conservative views, we did not publish it until we had also
obtained a second review that took a strongly liberal position. Clearly, however, our actions demonstrate not a
bias in favor of liberal views but rather a commitment to a balanced presentation of diverse opinions.
Determining which of the following would be most useful in evaluating the cogency of the magazine
publisher’s response?
A. Whether any other magazines in which the book was reviewed carried more than one review of the book
B. Whether the magazine publishes unsolicited book reviews as well as those that it has commissioned
C. Whether in the event that a first review commissioned by the magazine takes a clearly liberal position the
magazine would make any efforts to obtain further reviews
D. Whether the book that was the subject of the two reviews was itself written from a clearly conservative or a
clearly liberal point of view
E. Whether most of the readers of the magazine regularly read the book reviews that the magazine publishes
5. A certain automaker aims to increase its market share by deeply discounting its vehicles’ prices for the next
several months. The discounts will cut into profits, but because they will be heavily advertised the manufacturer
hopes that they will attract buyers away from rival manufacturers’ cars. In the longer term, the automaker
envisions that customers initially attracted by the discounts may become loyal customers. In assessing the
plan’s chances of achieving its aim, it would be most useful to know which of the following?
A. Whether the automaker’s competitors are likely to respond by offering deep discounts on their own products
B. Whether the advertisements will be created by the manufacturer’s current advertising agency
C. Whether some of the automaker’s models will be more deeply discounted than others
D. Whether the automaker will be able to cut costs sufficiently to maintain profit margins even when the discounts
are in effect
E. Whether an alternative strategy might enable the automaker to enhance its profitability while holding a constant
or diminishing share of the market

6. The growing popularity of computer-based activities was widely expected to result in a decline in television
viewing, since it had been assumed that people lack sufficient free time to maintain current television-viewing
levels while spending increasing amounts of free time on the computer. That assumption, however, is evidently
false: in a recent mail survey concerning media use, a very large majority of respondents who report increasing
time spent per week using computers report no change in time spent watching television. Which of the
following would it be most useful to determine in order to evaluate the argument?
A. Whether a large majority of the survey respondents reported watching television regularly
B. Whether the amount of time spent watching television is declining among people who report that they rarely or
never use computers
C. Whether the type of television programs a person watches tends to change as the amount of time spent per week
using computers increases
D. Whether a large majority of the computer owners in the survey reported spending increasing amounts of time per
week using computers
E. Whether the survey respondents’ reports of time spent using computers included time spent using computers at
work

7. Business Analyst: National Motors began selling the Luxora—its new model of sedan—in June. Last week, National
released sales figures for the summer months of June, July, and August that showed that by the end of August
only 80,000 Luxoras had been sold. Therefore, National will probably not meet its target of selling 500,000
Luxoras in the model’s first twelve months. Which of the following would be most useful to establish in
order to evaluate the analyst’s prediction?
A. Whether new-car sales are typically lower in the summer months than at any other time of the year
B. Whether National Motors currently produces more cars than any other automaker
C. Whether the Luxora is significantly more expensive than other models produced by National Motors
D. Whether National Motors has introduced a new model in June in any previous year
E. Whether National Motors will suffer serious financial losses if it fails to meet its sales goal for the Luxora

8. Scientists have made genetic modifications to cotton to increase its resistance to insect pests. According to
farmers’ reports, the amount of insecticide needed per acre to control insect pests was only slightly lower for
those who tried the modified seed than for those who did not. Therefore, since the modified seed costs more than
ordinary seed without producing yields of higher market value, switching to the modified seed would be unlikely
to benefit most cotton farmers economically. Which of the following would it be most useful to know in
order to evaluate the argument?
A. Whether farmers who tried the modified cotton seed had ever tried growing other crops from genetically modified
seed
B. Whether the insecticides typically used on ordinary cotton tend to be more expensive than insecticides typically
used on other crops
C. Whether for most farmers who grow cotton it is their primary crop
D. Whether the farmers who have tried the modified seed planted as many acres of cotton, on average, as farmers
using the ordinary seed did
E. Whether most of the farmers who tried the modified seed did so because they had previously had to use
exceptionally large quantities of insecticide
9. The emission of sulfur dioxide when high-sulfur coal is burned is restricted by law. New coal-burning plants
usually comply with the law by installing expensive equipment to filter sulfur dioxide from their emissions. These
new plants could save money by installing instead less expensive cleaning equipment that chemically removes
most sulfur from coal before combustion. Which of the following, if known, would be most relevant to
evaluating the claim above about how new coal-burning plants could save money?
A. Whether existing oil-burning plants are required to filter sulfur dioxide from their emissions
B. Whether the expense of installing the cleaning equipment in a new plant is less than the expense of installing the
cleaning equipment in an older plant
C. Whether the process of cleaning the coal is more expensive than the process of filtering the emissions
D. Whether lawful emissions of sulfur dioxide from coal-burning plants are damaging the environment
E. Whether existing plants that use the filtering equipment could replace this equipment with the cleaning
equipment and still compete with new plants that install the cleaning equipment

10. Certain genetically modified strains of maize produce a powerful natural insecticide. The insecticide occurs
throughout the plant, including its pollen. Maize pollen is dispersed by the wind and frequently blows onto
milkweed plants that grow near maize fields. Caterpillars of monarch butterflies feed exclusively on milkweed
leaves. When these caterpillars are fed milkweed leaves dusted with pollen from modified maize plants, they die.
Therefore, by using genetically modified maize, farmers put monarch butterflies at risk. Which of the following
would it be most useful to determine in order to evaluate the argument?
A. Whether the natural insecticide is as effective against maize-eating insects as commercial insecticides typically
used on maize are
B. Whether the pollen of genetically modified maize contains as much insecticide as other parts of these plants
C. Whether monarch butterfly caterpillars are actively feeding during the part of the growing season when maize is
releasing pollen
D. Whether insects that feed on genetically modified maize plants are likely to be killed by insecticide from the
plant’s pollen
E. Whether any maize-eating insects compete with monarch caterpillars for the leaves of milkweed plants growing
near maize fields

11. TrueSave is a mail-order company that ships electronic products from its warehouses to customers worldwide.
The company’s shipping manager is proposing that customer orders be packed with newer, more expensive
packing materials that virtually eliminate damage during shipping. The manager argues that overall costs would
essentially remain unaffected, since the extra cost of the new packing materials roughly equals the current cost of
replacing products returned by customers because they arrived in damaged condition. Which of the following
would it be most important to ascertain in determining whether implementing the shipping
manager’s proposal would have the argued-for effect on costs?
A. Whether the products shipped by TrueSave are more vulnerable to incurring damage during shipping than are
typical electronic products
B. Whether electronic products are damaged more frequently in transit than are most other products shipped by
mail-order companies
C. Whether a sizable proportion of returned items are returned because of damage already present when those
items were packed for shipping
D. Whether there are cases in which customers blame themselves for product damage that, though present on
arrival of the product, is not discovered until later
E. Whether TrueSave continually monitors the performance of the shipping companies it uses to ship products to its
customers

12. Regulations will not allow a pesticide that is toxic to humans to be used inside houses unless the pesticide will
dissipate completely from the air within eight hours after its application. One test that pesticide manufacturers
standardly use to determine how quickly anti-termite pesticides dissipate involves spraying the pesticides on the
walls of room-sized plywood boxes and then timing its dissipation. Which of the following would it be most
useful to know in order to evaluate whether a dissipation time of just under eight hours on the
manufacturers’ test indicates that an anti-termite pesticide that is toxic to humans obeys
regulations for use in houses?
A. Whether anti-termite pesticides dissipate more slowly in furnished rooms than in plywood boxes
B. Whether people who apply anti-termite pesticide standardly wear protective equipment that prevents them from
being exposed to the pesticide
C. Whether people whose house is being treated with anti-termite pesticide generally know that they should remain
out of their house during the hours immediately after the pesticide’s application
D. Whether there are anti-termite pesticides that are toxic to humans that, when subjected to the manufacturers’
test, dissipate completely from the air in the boxes in well under eight hours
E. Whether anti-termite pesticides that are not toxic to humans tend to take longer to dissipate than those that are
toxic
13. Plant scientists have been able to genetically engineer vegetable seeds to produce crops that are highly resistant
to insect damage. Although these seeds currently cost more than conventional seeds, their cost is likely to
decline. Moreover, farmers planting them can use far less pesticide, and most consumers prefer vegetables grown
with less pesticide, therefore, for crops for which these seeds can be developed, their use is likely to become the
norm. Which of the following would be most useful to know in evaluating the argument above?
A. Whether plant scientists have developed insect-resistant seeds for every crop that is currently grown commercially
B. Whether farmers typically use agricultural pesticides in larger amounts than is necessary to prevent crop damage.
C. Whether plants grown from the new genetically engineered seeds can be kept completely free of insect damage.
D. Whether seeds genetically engineered to produce insect-resistant crops generate significantly lower per acre crop
yields than do currently used seeds.
E. Whether most varieties of crops currently grown commercially have greater natural resistance to insect damage
than did similar varieties in the past.

14. Fish currently costs about the same at seafood stores throughout Eastville and its surrounding suburbs. Seafood
stores buy fish from the same wholesalers and at the same prices, and other business expenses have also been
about the same. But new tax breaks will substantially lower the cost of doing business within the city. Therefore,
in the future, profit margins will be higher at seafood stores within the city than at suburban seafood stores. For
the purposes of evaluating the argument, it would be most useful to know whether
A. more fish wholesalers are located within the city than in the surrounding suburbs.
B. any people who currently own seafood stores in the suburbs surrounding Eastville will relocate their businesses
nearer to the city
C. the wholesale price of fish is likely to fall in the future
D. fish has always cost about the same at seafood stores throughout Eastville and its surrounding suburbs.
E. seafood stores within the city will in the future set prices that are lower than those at suburban seafood stores.

15. Goronian lawmaker: Goronia’s Cheese Importation Board, the agency responsible for inspecting all wholesale
shipments of cheese entering Goronia from abroad and rejecting shipments that fail to meet specified standards,
rejects about one percent of the cheese that it inspects. Since the health consequences and associated costs of
not rejecting that one percent would be negligible, whereas the cost of maintaining the agency is not, the
agency’s cost clearly outweighs the benefits it provides. Knowing the answer to which of the following
would be most useful in evaluating the lawmaker’s argument?
A. Are any of the types of cheeses that are imported into Goronia also produced in Goronia?
B. Has the Cheese Importation Board, over the last several years, reduced its operating costs by eliminating
inefficiencies within the agency itself?
C. Does the possibility of having merchandise rejected by the Cheese Importation Board deter many cheese
exporters from shipping substandard cheese to Goronia?
D. Are there any exporters of cheese to Goronia whose merchandise is never rejected by the Cheese Importation
Board?
E. How is the cheese rejected by the Cheese Importation Board disposed of?

16. The population of desert tortoises in Targland’s Red Desert has declined, partly because they are captured for
sale as pets and partly because people riding all-terrain vehicles have damaged their habitat. Targland plans to
halt this population decline by blocking the current access routes into the desert and announcing new regulations
to allow access only on foot. Targland’s officials predict that these measures will be adequate, since it is difficult
to collect the tortoises without a vehicle. Which of the following would it be most important to establish
in order to evaluate the officials’ prediction?
A. Whether possessing the tortoises as pets remains legally permissible in Targland
B. Whether Targland is able to enforce the regulations with respect to all-terrain vehicle entry at points other than
the current access routes
C. Whether the Red Desert tortoises are most active during the day or at night
D. Whether people who travel on foot in the Red Desert often encounter the tortoises
E. Whether the Targland authorities held public hearings before restricting entry by vehicle into the Red Desert

17. Though sucking zinc lozenges has been promoted as a treatment for the common cold, research has revealed no
consistent effect. Recently, however, a zinc gel applied nasally has been shown to greatly reduce the duration of
colds. Since the gel contains zinc in the same form and concentration as the lozenges, the greater effectiveness of
the gel must be due to the fact that cold viruses tend to concentrate in the nose, not the mouth. In order to
evaluate the argument, it would be most helpful to determine which of the following?
A. Whether zinc is effective only against colds, or also has an effect on other virally caused diseases
B. Whether there are remedies that do not contain zinc but that, when taken orally, can reduce the duration of colds
C. Whether people who frequently catch colds have a zinc deficiency
D. Whether either the zinc gel or the lozenges contain ingredients that have an impact on the activity of the zinc
E. Whether the zinc gel has an effect on the severity of cold symptoms, as well as on their duration
18. Trancorp currently transports all its goods to Burland Island by truck. The only bridge over the channel separating
Burland from the mainland is congested, and trucks typically spend hours in traffic. Trains can reach the channel
more quickly than trucks, and freight cars can be transported to Burland by barges that typically cross the
channel in an hour. Therefore, to reduce shipping time, Trancorp plans to switch to trains and barges to transport
goods to Burland. Which of the following would be most important to know in determining whether
Trancorp’s plan, if implemented, is likely to achieve its goal?
A. Whether transportation by train and barge would be substantially less expensive than transportation by truck.
B. Whether there are boats that can make the trip between the mainland and Burland faster than barges can
C. Whether loading the freight cars onto barges is very time consuming
D. Whether the average number of vehicles traveling over the bridge into Burland has been relatively constant in
recent years
E. Whether most trucks transporting goods into Burland return to the mainland empty

19. Urban air contains more sulfur dioxide than does rural air, and plants in cities typically grow more slowly than do
plants in rural areas. In an experiment to see how much of the difference in growth is due to sulfur dioxide,
classes in an urban and a rural school grew plants in greenhouses at their schools and filtered the greenhouse air
to eliminate sulfur dioxide. Plants in the urban greenhouse grew more slowly than those in the rural greenhouse.
Which of the following, if true, would it be most important to take into account in evaluating the
result?
A. The urban school was located in a part of the city in which levels of sulfur dioxide in the air were usually far lower
than is typical for urban areas.
B. At both schools, the plants in the greenhouses grew much more quickly than did plants planted outdoors in plots
near the greenhouses.
C. The urban class conducting the experiment was larger than the rural class conducting the experiment.
D. Heavy vehicular traffic such as is found in cities constantly deposits grime on greenhouse windows, reducing the
amount of light that reaches the plants inside.
E. Because of the higher levels of sulfur dioxide in the air at the urban school, the air filters for the urban school's
greenhouse were changed more frequently than were those at the rural school.

20. Following several years of declining advertising sales, the Greenville Times reorganized its advertising sales force
two years ago. Before the reorganization, the sales force was organized geographically, with some sales
representatives concentrating on city-center businesses and others concentrating on different outlying regions.
The reorganization attempted to increase the sales representatives' knowledge of clients' businesses by having
each sales representative deal with only one type of industry or of retailing. After the reorganization, advertising
sales increased. In assessing whether the improvement in advertising sales can properly be attributed
to the reorganization, it would be helpful to find out each of the following EXCEPT:
A. Two years ago, what proportion of the Greenville Times’ total revenue was generated by advertising sales?
B. Has the circulation of the Greenville Times increased substantially in the last two years?
C. Has there been a substantial turnover in personnel in the advertising sales force over the last two years?
D. Before the reorganization, had sales representatives found it difficult to keep up with relevant developments in all
types of businesses to which they are assigned?
E. Has the economy in Greenville and the surrounding regions been growing rapidly over the last two years?

21. In a certain wildlife park, park rangers are able to track the movements of many rhinoceroses because those
animals wear radio collars. When, as often happens, a collar slips off, it is put back on. Putting a collar on a
rhinoceros involves immobilizing the animal by shooting it with a tranquilizer dart. Female rhinoceroses that have
been frequently re-collared have significant lower fertility rate than un-collared females. Probably, therefore, some
substances in the tranquilizer inhibit fertility. In evaluating the argument, it would be most useful to
determine which of the following?
A. Whether there are more collared female rhinoceroses than un-collared female rhinoceroses in the park.
B. How the tranquilizer that is used for immobilizing rhinoceroses differs, if at all, from tranquilizers used in working
with other large mammals.
C. How often park rangers need to use tranquilizer dart to immobilize rhinoceroses for reasons other than attaching
radio collars.
D. Whether male rhinoceroses in the wild park lose their collar any more often than the park's female rhinoceroses
do
E. Whether radio collar is the only practical means that park rangers have for tracking the movements of
rhinoceroses in the park.
22. Capuchin monkeys in Venezuela often rub a certain type of millipede into their fur. Secretions of these millipedes
have been shown to contain two chemicals that are potent mosquito repellents, and mosquitoes carry parasites
that debilitate the capuchins. The rubbing behavior is rare except during the rainy season, when mosquito
populations are at their peak. Therefore monkeys probably rub millipedes into their fur only because doing so
helps protect them against mosquitoes. Which of the following would be most useful to determine in
order to evaluate the argument?
A. Whether the two chemicals provide any protection for millipedes against their own repellents.
B. Whether the type of millipede used by the capuchin monkeys in Venezuela is found in other parts of the world.
C. Whether animals other than capuchins rub insects of any kind into their fur
D. Whether the only time millipedes are readily available to capuchins is during rainy season
E. Whether secretions of any other insects accessible to capuchins contain chemicals that repel the mosquitoes

23. Columnist: People should avoid using a certain artificial fat that has been touted as a resource for those whose
medical advisers have advised them to reduce their fat intake. Although the artificial fat, which can be used in
place of fat in food preparation, has none of the negative health effects of fat, it does have a serious drawback: it
absorbs certain essential vitamins, thereby preventing them from being used by the body. In evaluating the
columnist's position, it would be most useful to determine which of the following?
A. Whether increasing one's intake of the vitamins can compensate for the effects of the artificial fat
B. Whether the vitamins that the artificial fat absorbs are present in foods that contain the fat
C. Whether having an extremely low fat intake for an extended period can endanger the health
D. Whether there are any foods that cannot be prepared using the artificial fat as a substitute for other fats
E. Whether people are generally able to detect differences in taste between foods prepared using the artificial fat
and foods that are similar except for the use of other fats

24. In many scientific disciplines, scientists generally do not do highly creative work beyond the age of forty, a
tendency that has normally been taken to show that aging carries with it a loss of creative capacity. However, by
the age of forty most scientists have been working in their chosen field for at least fifteen years, so an alternative
explanation is that spending too long in a single field reduces the opportunity for creative thought.
Investigating which of the following would be most useful in choosing between the competing
explanations described above?
A. Whether among those scientists who do highly creative work beyond age forty a large proportion entered
their field at a considerably later age than is common
B. Whether scientists' choice of research projects tends to be influenced by their own belief that their most
creative work will be done relatively early in their career
C. Whether scientists who are older than forty tend to find more satisfaction in other activities, such as teaching
and mentoring, than they do in pursuing their own research
D. Whether funding agencies are more inclined to award research grants to scientists who are veterans in their
field than to scientists who are relative newcomers
E. Whether there is significant variation among scientific fields in the average age at which scientists working in
those fields are at their most productive

25. Lyme disease is caused by a bacterium transmitted to humans by deer ticks. Generally, deer ticks pick up the
bacterium while in the larval stage by feeding on infected white-footed mice. However, certain other species on
which the larvae feed do not harbor the bacterium. If the population of these other species were increased, more
of the larvae would be feeding on uninfected hosts, so the number of ticks acquiring the bacterium would likely
decline. Which of the following would it be most important to ascertain in evaluating the argument?
A. Whether populations of the other species on which deer tick larvae feed are found only in areas also inhabited
by white-footed mice
B. Whether the size of the deer tick population is currently limited by the availability of animals for the tick's
larval stage to feed on
C. Whether the infected deer tick population could be controlled by increasing the number of animals that prey
on white-footed mice
D. Whether deer ticks that were not infected as larvae can become infected as adults by feeding on deer on
which infected deer ticks have fed
E. Whether the other species on which deer tick larvae feed harbor any other bacteria that ticks transmit to
humans
26. Fedora Convertibles began selling the Freewheeler – its newest convertible sports car – in May of this year.
Fedora sent out a press release last month indicating that the Freewheeler’s sales for May, June, July, and August
totaled over 50,000. As a result, Fedora will in all likelihood easily meet its stated sales goal of 120,000 for
Freewheeler's first year of release. Which of the following would be most useful in order to evaluate this
prediction?
A. Fedora is the market leader in this category of automobile.
B. Freewheeler is significantly more expensive than similar models produced by Fedora’s competitors.
C. Fedora released a similar model in May of the previous year.
D. Sales of similar models of cars are typically much higher in the summer months than at any other time of the
year.
E. Freewheeler sales are important to meeting the sales goals of Fedora as a company.

27. Most of the world's supply of uranium currently comes from mines. It is possible to extract uranium from
seawater, but the cost of doing so is greater than the price that uranium fetches on the world market.
Therefore, until the cost of extracting uranium from seawater can somehow be reduced, this method of
obtaining uranium is unlikely to be commercially viable. Which of the following would it be most useful to
determine in evaluating the argument?
A. Whether the uranium in deposits on land is rapidly being depleted
B. Whether most uranium is used near where it is mined
C. Whether there are any technological advances that show promise of reducing the cost of extracting uranium
from seawater
D. Whether the total amount of uranium in seawater is significantly greater than the total amount of uranium on
land
E. Whether uranium can be extracted from freshwater at a cost similar to the cost of extracting it from seawater

28. Over the last five years, demand for hotel rooms in Cenopolis has increased significantly, as has the average
price Cenopolis hotels charge for rooms. These trends are projected to continue for the next several years. In
response to this economic forecast, Centennial Commercial, a real estate developer, is considering a plan to
convert several unoccupied office buildings it owns in Cenopolis into hotels in order to maximize its revenues
from these properties. Which of the following would it be most useful for Centennial Commercial to
know in evaluating the plan it is considering?
A. Whether the population of Cenopolis is expected to grow in the next several years
B. Whether demand for office space in Cenopolis is projected to increase in the near future
C. Whether the increased demand for hotel rooms, if met, is likely to lead to an increase in the demand for other
travel-related services
D. Whether demand for hotel rooms has also increased in other cities where Centennial owns office buildings
E. Whether, on average, hotels that have been created by converting office buildings have fewer guest rooms than
do hotels that were built as hotels

29. Guitar strings often go "dead"—become less responsive and bright in tone—after a few weeks of intense use. A
researcher whose son is a classical guitarist hypothesized that dirt and oil, rather than changes in the material
properties of the string, were responsible. Which of the following investigations is most likely to yield
significant information that would help to evaluate the researcher's hypothesis?
A. Determining if a metal alloy is used to make the strings used by classical guitarists
B. Determining whether classical guitarists make their strings go dead faster than do folk guitarists
C. Determining whether identical lengths of string, of the same gauge, go dead at different rates when strung on
various brands of guitars
D. Determining whether a dead string and a new string produce different qualities of sound
E. Determining whether smearing various substances on new guitar strings causes them to go dead

30. Because of steep increases in the average price per box of cereal over the last 10 years, overall sales of
cereal have recently begun to drop. In an attempt to improve sales, one major cereal manufacturer reduced
the wholesale prices of its cereals by 20 percent. Since most other cereal manufacturers have announced
that they will follow suit, it is likely that the level of overall sales of cereal will rise significantly. Which of the
following would it be most useful to establish in evaluating the argument?
A. Whether the high marketing expenses of the highly competitive cereal market led to the increase in cereal
prices
B. Whether cereal manufacturers use marketing techniques that encourage brand loyalty among consumers
C. Whether the variety of cereals available on the market has significantly increased over the last 10 years
D. Whether the prices that supermarkets charge for these cereals will reflect the lower prices the supermarkets
will be paying the manufacturers
E. Whether the sales of certain types of cereal have declined disproportionately over the last 10 years
31. A certain pharmaceutical firm claims that its dietary supplement, Dietol, is highly effective in helping obese
people lose weight and improve physical endurance. The company supports this claim by providing the results
of a recent experiment that involved over 800 subjects suffering from obesity. During the 9-week experiment,
each of the subjects was required to take a regular dose of Dietol every day immediately after their daily 2-
hour workouts supervised by a professional fitness instructor. As a result of the experiment, the subjects lost
an average of 12 pounds of weight per person, and over 95% of all participants demonstrated higher physical
endurance. The answer to which of the following questions would be most helpful in evaluating the
conclusion of the pharmaceutical company?
 What was the daily dose of Dietol that the subjects were required to take?
 What was the maximum weight lost by any participant during the 9-week program?
 What would be the average weight loss and the improvement in endurance in a group of subjects with similar
characteristics involved in the same physical fitness program but not taking Dietol?
 What was the average age among the subjects participating in the experiment?
 Did the majority of subjects experience a significant improvement in physical strength?

32. The recording industry is fighting a losing battle: it simply does not have the resources to prosecute all of the
individuals who illegally download music from the Internet. Because the number of individuals who will be
charged with a crime is so limited, the actions of the recording industry will have a minimal impact on the
number of people who illegally download music. The answer to which of the following questions would best
help evaluate the accuracy of the conclusion above?
 Will recording industry lawyers dedicate the majority of their time to prosecuting those who illegally download
music?
 Is a small minority of individuals responsible for the majority of illegal song downloads?
 Do many individuals who illegally download songs share their music files with other Internet users?
 Will new Internet security technology permit the recording industry to more quickly and easily identify
individuals who illegally download music?
 Will the threat of prosecution alter the behavior of those who illegally download music?

33. While political discourse and the media in the United States have focused on the rise of job outsourcing, few
have mentioned the sharp fall of talent ―insourcing,‖ or the drop in enrollment of foreign-born graduate
students since 2001, and its dire results. The decrease in such insourcing will hurt America’s competitiveness
in basic research and applied technology, with serious consequences for years to come. The de-
internationalization of graduate programs across the country will also negatively affect the global outlook and
experience of the American students remaining in those programs; they will not have the opportunity to learn
about foreign cultures directly from members of those cultures. What distinguishes the decline of talent
insourcing from the rise of job outsourcing is that the former can be easily rectified by a policy change of the
United States government. The answer to which of the following questions would be most useful in evaluating
the author’s claim regarding the impact of decreased insourcing in America?
 What is the cost to reverse the trend of insourcing in America?
 How does insourcing replace domestic jobs lost from outsourcing?
 Since 2001, what has been the decrease in the number of foreign-born students in America?
 What opportunities do American graduate students have to interact regularly with foreigners who are not
students?
 What effect would a government policy have on the number of foreign graduate students?

34. Profits for one of Company X's flagship products have been declining slowly for several years. The CFO
investigated and determined that inflation has raised the cost of producing the product but consumers who
were surveyed reported that they weren't willing to pay more than the current price. As a result, the CFO
recommended that the company stop producing this product because the CEO only wants products whose
profit margins are increasing. The answer to which of the following questions would be most useful in
evaluating whether the CFO's decision to divest the company of its flagship product is warranted?
 Does the company have new and profitable products available with which to replace the flagship product?
 Will the rest of Company X's management team agree with the CFO's recommendation?
 Are there additional features which could be added to the product and for which consumers might be willing
to pay a higher price?
 Is there a way to alter the manufacturing or distribution processes in order to reduce the cost to produce the
flagship product?
 What percentage of Company X's revenues is represented by sales of the flagship product in question?
35. The Civil Service Act of 1883, also known as the Pendleton Act, which created a professional corps of
administrators, was passed after a disappointed office-seeker assassinated President James A. Garfield. For a
hundred years, this system has anchored American government service to competence rather than
corruption. The best way to preserve this state in the new millennium is to maintain the Pendleton Act as it is.
Which of the following would be most useful to evaluate the argument’s conclusion?
 The methods that the Swiss and British governments have used to prevent corruption in government service
for the past one hundred years
 The current level of job satisfaction among government office-seekers and office-holders
 The levels of competence and corruption in American government service between 1950 and the present
 The number of Presidents assassinated since the passage of the Pendleton Act
 The percentage of office-holders fired or convicted on charges stemming from corruption during the first
hundred years of the Pendleton Act

36. Zoologists warn of an imminent surge in the number of bird species that will become extinct within this
century. Nevertheless, these zoologists are wrong. One need only consider the information gathered on native
North American raptors, such as bald eagles and peregrine falcons. Several of these species came close to
vanishing between 1900 and 1970, but since 1970, the local populations of these raptors have rebounded.
The answer to which of the following questions provides information that would be most helpful in evaluating
the argument above?
 How many species of non-native raptors have been introduced into North America since 1970?
 What special efforts, if any, have been made to rescue native North American raptors since 1970?
 How many years' experience do the zoologists have in evaluating patterns of extinction among animals?
 To what degree have native North American raptors migrated to other parts of the world?
 How many acres of woodland are set aside each year as bird refuges?

37. Most cable television companies currently require customers to subscribe to packages of channels, but
consumer groups have recently proposed legislation that would force the companies to offer a la carte
pricing. Subscribers would pay less, argue the consumer groups, because they could purchase only the
desired channels. However, the cable industry argues that under the current package pricing, popular
channels subsidize less-popular ones, providing more options for viewers. For this reason, the industry claims
that it is always cheaper for the consumer to purchase many bundled channels than to buy them individually.
Which of the following would be most important for the government to determine before deciding whether to
require cable television companies to offer a la carte pricing in order to reduce consumer costs?
 Whether the total number of channels offered to consumers would decrease, along with programming
diversity, as a result of the a la carte pricing structure
 Whether advertising revenue for the cable television companies would decrease as a result of the a la carte
pricing structure
 Whether the vast majority of consumers would greatly reduce the number of channels purchased if given the
option of purchasing them individually
 Whether cable and satellite companies currently have the ability to buy channels individually from
programmers and content providers
 Whether a la carte subscribers would be required to have new television set-top boxes

38. The director of programming at NNN, the National News Network, proclaimed that, despite charges to the
contrary, the Network does not have a conservative bias. The director acknowledged that, when a liberal
news personality was recently added to the Network, an additional conservative news program was also
established. However, the director argued that far from demonstrating any bias, these actions reflect a
commitment to presenting a balanced perspective in interpreting current events. Which of the following
pieces of information would be most useful in evaluating the validity of the programming director’s argument?
 The Network has established an additional liberal news program whenever an additional conservative news
personality has been added.
 The Network airs a program focused largely on the opinions of its viewers on contemporary issues.
 The newly added liberal news personality came from a rival network with an acknowledged liberal bias.
 The newly added liberal news personality has indicated that he has not felt any editorial pressure in his new
environment.
 The viewer ratings of the Network have been lower this past year than in previous years.
39. Child development specialists have observed that adolescents who receive large weekly allowances tend to
spend money on items considered frivolous by their parents whereas adolescents who receive small weekly
allowances do not. Thus, in order to ensure that their children do not spend money on frivolous items,
parents should not give their children large weekly allowances. Which of the following pieces of information
would be most useful in evaluating the validity of the conclusion above?
 The average amount of money received by adolescents who receive large weekly allowances
 Any differences among parents in the standard used to judge an item as frivolous
 The educational background of the child development specialists who made this observation
 The difference between the average annual income of families in which the parents give their children large
weekly allowances and that of families in which the parents give their children small weekly allowances
 The percentage of adolescents who receive no weekly allowance

40. The downturn in the economy last year has prompted many companies to make widely publicized layoffs,
resulting in thousands of lost jobs. Economists predicted that these layoffs would cause people generally to
cut back on their discretionary spending even if their jobs were secure, in anticipation of coming hard times.
However, this prediction has not come to pass, since there has been no increase in the amount of money set
aside by the general public in savings accounts. The answer to which of the following questions would be
most useful in evaluating the significance of the savings patterns described above?
 What business sectors were most affected by the layoffs?
 How much of their savings, on average, do laid-off employees deplete before finding new employment?
 What has been the percent increase in the cost of necessities such as food, housing, and utilities during the
period since the layoffs?
 What percentage of people laid off have savings accounts?
 What has been the average salary during the period since the layoffs?

41. Because of a rare type of fungus that killed off many cacao trees in Brazil, there was an unusually meager
harvest of cocoa beans this year. The wholesale price of cocoa solids and cocoa butter has increased
significantly and does not look likely to fall in the foreseeable future. As a result, the retail price of chocolate
is certain to increase within six months. The answer to which of the following questions would
provide information relevant to evaluating the claims made in the argument above?
• Has the price of cocoa remained steady during other period of poor harvest?
• Are consumers willing to spend more for chocolate?
• Have the prices of other ingredients in chocolate decreased recently?
• What percentage of cacao trees in Brazil were affected by the fungus?
• Can the fungus be eliminated within the next six months?

42. Scientists have determined that an effective way to lower cholesterol is to eat three servings of whole grains
every day. Studies have shown that the cholesterol levels of people who did so were significantly lower after
six months than were those of people who did not, even though the cholesterol levels of the two groups were
the same before the studies began. Clearly, eating whole grains can have an appreciable effect on cholesterol
levels. The answer to which of the following questions, if true, would be most useful in evaluating
the claim about whole grains above?
 Is it realistic to expect people to eat three servings of whole grains per day?
 Were the two groups of people in the study involved in the same exercise program?
 Can the same drop in cholesterol be achieved through medication?
 Did the study continue to track the subjects beyond six months?
 Are most consumers aware of the different between whole grains and processed grains?
1.

The argument concludes that even if the public were to ask for inappropriate medicines “But since
physicians have the final say as to whether to prescribe a medication for a patient, inappropriate
prescriptions would not become more common”
IS there an option that tells us that this may not be the case?

A. Even if this were true, it does not indicate whether inappropriate prescriptions would become more
common or not.

B. Even if this were true, physicians have the final say as to whether to prescribe a medication, according
to the passage.

C. Same as above.

D. The argument says talks of general public not having the specialized knowledge to evaluate such
advertisements. It does not discuss the physician's knowledge.

E. CORRECT. This indicates a possible scenario when the despite having the final say on prescribing
medication, the physicians may give into the patient's demand.

2.

A. Irrelevant. What is applicable to other towns need not necessarily be applicable to Morganville.

B. Irrelevant. Even if they did not do all their shopping at Morganville, this does not help us answer
whether opening Saveall will bankrupt the other smaller shops

C. CORRECT.
The answer to this will tell us whether stores are going bankrupt due to Saveall or due to other external
factors.

D. The answer to this would tell us of the job opportunities Saveall may create. It would however tell us
nothing about

Saveall's effect on the smaller shops.

E. Irrelevant. Saveall's profits are not irrelevant here.

3.
The argument assumes that since Bergeron has submitted a detailed list of his current financial holdings
to the election commission, it means that he is running for governor.

A. This is irrelevant. We are not concerned with who the richest candidate is. This does not shed further
light on whether Bergeron is running for Governor or not.

B. There may be other ways. But we already know that Bergeron has done so by submitting a detailed
list of his current financial holdings. So the answer to this question will tell us nothing new.

C. Even if it did not come from the election commission that is not sufficient grounds for us to assume
that the report is untrue.

D. Irrelevant. His wealth is irrelevant to evaluate whether he is running for governor or not.

E. CORRECT. This will help us decide if Bergeron submitting a list of his financial holdings is false alarm.
If Bergeron has done the same last year, then there is a chance that he might not stand for elections
despite having fulfilled the financial disclosure requirement.

4.

Here’s a simplified version:

Magazine’s got a conservative review, but then waited until they got a liberal review.

They say they were being balanced.

They say that they DON'T have a bias in favor of liberal views.

We need to pick an answer choice that will DISTINGUISH BETWEEN THESE TWO POSSIBILITIES.

A. What other magazines do is immaterial. The option needs to help evaluate the position of the
magazine in question.

B. Irrelevant. What do unsolicited reviews have to do with whether the magazine is liberal biased or not?

C. CORRECT. In case the first review was liberal and the magazine did not bother to find a second, more
conservative review then we can conclude that the magazine is liberal biased.

D. This is irrelevant. We are strictly dealing with reviews here.

E. Irrelevant. The question isn't about readership. Rather we are evaluating the claim that the magazine
is unbiased.
5.

A. CORRECT. If the answer to this question were yes, the plan would probably not achieve its aim of
increasing market share. If the answer to this were no, the plan would have a good chance of
succeeding.

B. Irrelevant. We do not have any information about the automaker's current advertising strategy and
hence cannot determine the effectiveness of this option.

C. Irrelevant. Even if the answer to this were yes, it does not tell us whether the overall share of the
automaker in the market will increase.

D. The passage already tells us that the discount will cut into profits. The automaker hopes his plan will
yield profits in the long term

E. Irrelevant. The idea is to increase the market share. Not hold it constant or diminish it.

6.
We need to evaluate whether there has been a decline in television since the popularity of computers has
increased viewing despite the survey which states otherwise.

The thing to keep in mind is, the passage states due to increased amount of FREE time spent on the
computer, there isn't sufficient FREE time to watch TV.

The report however says that the persons in the survey spent an increasing amount of time in TOTAL on
the computer and managed to watch the same amount of TV.

A. This is irrelevant. We are concerned with whether there has been a decline in the time spent viewing
TV. Even if the answer to this option was “No” it would still mean that there was no change in the TV
viewing pattern before and after computers.

B. Irrelevant. Once again we are concerned with decline in TV viewing due to increased use of Computer
in FREE time.

C. Irrelevant. The content of TV viewing is irrelevant to us.

D. the report already states this to be true.

E. CORRECT.
A-ha! If the answer to this is yes, it would tell us that the people in the survey spent increasing amount
of time on the computer at work. Since we wouldn't know if the use of computers has increased during
people's free time, the report would become meaningless.
If the answer is no, then report would be accurate and would have found a surprising result indeed.

7.
A. CORRECT.

If the answer to this is yes, then it could mean that the sales will pick up in the coming months allowing
National Motors to achieve its target. If the answer is no, then by extrapolation, the business analyst's
prediction may come true.

B. Irrelevant. We already have a clear target of 800,000 to achieve.

C. Same as above. Despite pricing the target has been set.

D. Even if this were true, it does not help us decide whether Luxora can achieve the mentioned target.

E. We are evaluating the ability to meet the target and not the repercussions of not meeting it.

8.

This argument assumes that the fields in which insecticide was used do not differ from the others in
terms of insect infestation. E, the correct answer, addresses this crucial assumption. If E is true, that
farmers who tried the modified seed would have had to use SUBSTANTIALLY more insecticide to deal
with the insects. B is irrelevant, as we are talking only about cotton. The information is given on a per
acre basis, so D is irrelevant, too.

9.
A. Irrelevant. We are trying to evaluate the most cost effective way for new plants to deal with sulfur
emissions.

B. Same as above. We are not interested in making a comparison between the old and the new plants.

C. CORRECT.

If the answer to this were yes, it would mean that cleaning equipment is not a good idea as filtering
would save more money and vice-versa.

D. Irrelevant. We are trying to evaluate the most cost effective way for new plants to deal with sulfur
emissions not what is best for the environment.

E. Same as B.

10.
A. Irrelevant. We are concerned with the effect of natural insecticides on monarch butterflies only and
not on other maize eating insects.

B. Since the monarch butterflies are affected only by the pollens and these pollens contain enough
insecticide to cause their death, this option is irrelevant.
C. CORRECT.

If the answer to this is no, then there is no risk from using genetically modified maize as no pollens will
blow on the milkweed during the feeding season of the butterflies.
If the answer is yes, then the butterflies are indeed at risk.

D. Same as A.

E. The passage is concerned with the death of butterflies caused due to pollens with insecticides and not
due to competition for food. Also, even if this were true we do not know if this competition is sufficient to
threaten the existence of the butterflies.

11.
The manager states that the added cost of the new fancy packing materials will make up for no more
damage claims.

The assumption is that IF we use the advanced packaging, then there WILL BE no more damage claims.

A.

This doesn't matter. The comparison is not outside Truesave. Irrespective of whether this was true or not
it doesn't help evaluate the above.

B. Since Truesave only ships electronic products, this is irrelevant.

C. CORRECT.
IF this were true, then despite using advanced packing, the items will still be returned. Thus the company
will spend both on packing as well as replacing damaged goods.

If this were incorrect, then the manager's plan will work.

D. There may be such cases. But how we are only concerned with products that are returned.

E. Since this does not tell us that the damage is caused by shipping companies, it is irrelevant.

12.
A. CORRECT.

This is pretty straight forward. IF the answer to this is yes, then the test clearly does not give us accurate
time for dissipation.

If the answer is no, then we can take assume the test to be accurate and that the pesticide obeys the
standard time of 8 hours for dissipation.

B. Irrelevant. This does not affect the dissipation time.


C. same as above.

D. This is irrelevant. We already know that pesticides harmful to humans that do not dissipate less than 8
hours are banned. The existence of such pesticides is not the question. Rather we are discussing if the
means to determine the safety is accurate.

E. Irrelevant. Either way we know it needs to be less than 8 hours.

13.
The passage suggests that since the price of the genetically engineered seeds will decline and there will
be a high demand for plants grown from these seeds, the farmers will soon switch from the conventional
seeds.

A. Irrelevant. The passage simply discusses the “crops for which these seeds can be developed”.

B. Irrelevant. Even if the answer to this were yes, we still do not know how much pesticide is required to
prevent crop damage. Whatever the amount, we know that farmers using the new seeds will need far
less pesticides making this irrelevant.

C. The plants need not be pesticides free. Lowered use of pesticides will decrease cost for farmers and
are preferred by consumers as well. This would achieve the purpose.

D. CORRECT. IF this were true then despite the lowered pesticides cost and increasing consumer
demand, the farmers may not be able to switch from conventional seeds.

If this were false, then the plan will succeed.

E. Irrelevant

The correct answer is choice D.

For relevant information questions, you should figure out the assumption. Then, you should look for a
choice where if it goes one way it will strengthen the argument, and if it goes the other way it will
weaken the argument. So, one way to approach this type of question is to treat it as a hybrid
strengthen/weaken. Another way is to look for the choice that falls flatly within the scope of the
argument.

The author argues that genetically engineered seeds will replace conventional seeds because they are
highly resistant to insect damage, because their cost is likely to decline, and because they require less
pesticide.

When figuring out the assumption, you should ask "what else must be true for the argument to work?"
What else must be true in order for the genetically engineered seeds to replace conventional ones? well,
they would have to deliver at least almost as much yield as the conventional ones. (If, they didn't, then it
is unlikely that farmers will turn to them.)

Choice D matches this prediction. Let's apply the hybrid strengthen/weaken test to choice D:

What if the genetically engineered seeds generated significantly lower yield? Then, farmers are unlikely to
turn to them, and the argument is weakened. And, what if genetically engineered seeds generated just
as much yield as the conventional seeds? Then, farmers are more likely to turn to them (for all the
reasons the author cites), and the argument is strengthened.

Let's consider choices B and E.

Choice B is irrelevant. If farmers right now are using more pesticides than are necessary, then they can
simply use less pesticide; they don't have to turn to genetically engineered seeds.

Choice E offers an irrelevant comparison. Even if current crops are more insect-resistant than those used
in the past, we still know from the passage that the genetically engineered seeds are far more insect-
resistant than the ones being currently used. (Choice E would be wrong even if it said upcoming
conventional seeds are more insect-resistant than current ones, because we still wouldn't know whether
the upcoming ones are as insect-resistant as the genetically engineered ones).

As an aside, it shouldn't necessarily be surprising that the correct answer brings up an idea (yield) that
did not make an appearance in the stimulus. The right answer to a relevant information question is
closely related to the assumption, and assumptions are always unstated.

14.

There are two kinds of seafood stores → suburban and city. Now the cost of doing business for both has
been the same. Since the taxes within the city will be lowered soon, the passage indicates that the city
stores may make more profit.

A. This may mean that city stores overall have more business. But it does not help us evaluate whether
lowered taxes will lead to increased profits when compared to the suburban stores.

B. While this looks tempting, it does not tell us whether the suburban stores will relocate to the city due
to increased profit margins.

C. this would affect the city and suburbs equally.

D. We already know this to be true.

E. CORRECT. IF this were true then the city stores may attract all the customers due to lowered rates
and make considerable profit.
15.

A. Irrelevant. We are only concerned with the usability of the importation agency

B. This is irrelevant. Even if inefficiencies have been reduced it still costs more to keep an agency open
than to close it down.

C. CORRECT.

If this were true, then the reason why the board does not receive substandard cheese is because of the
presence of such a board. Removal of the agency may then cause amount of substandard cheese
imported
to rise and this would in turn cause the health costs to go up.

If this were untrue, then the evaluation is correct.

D. This does not help evaluate the usefulness of the agency. The number of such exporters could range
from 0.1% to 100%

E. Irrelevant. This does not help evaluate the usefulness of the Cheese Importation Board in anyway.

16.

A. This does not help us evaluate whether blocking current access routes and allowing access only on
foot will help protect the tortoise. Also we already know that the tortoises are captured for sale as pets

B. CORRECT.

If this answer to this is no, then the regulations may not help protect the tortoise as people will simply
access different routes to hunt them and damage the habitat.
If the answer is yes, then the officer’s prediction about the measures being adequate will be true.

C. Irrelevant. Since the regulations are not separate for day and night, this is irrelevant to us.

D. Irrelevant. We know people travelling on foot can't catch them even if they encountered them.

E. Irrelevant. This does not affect the effectiveness of the plan.

17.
A. This is irrelevant. We are trying to evaluate whether cold virus concentrate in the mouth or the nose.
This doesn't help the evaluation in anyway.

B. This seems tempting. But, unlike with zinc medicines, we do not have a comparison of what happens
when such medicines are applied on the nose. Is there a further reduction in the duration of cold?

C. Irrelevant. How does this tell us whether the virus is in the nose or the mouth?
D. CORRECT. If this is true then we know that the reason gel worked better than lozenges was because
of the zinc activity (which was different even though the zinc concentration was same in both) and not
due to the concentration of the virus in the mouth.

E. Irrelevant. We are not concerned with the effect in symptoms.

18.
A. The goal of Trancorp's plan is to reduce shipping time. This might lower costs – but whether or not it
does so is not relevant to whether or not the plan's goal is achieved.

B. Maybe they can but it has nothing to do with whether the current plan will reduce shipping time.

C. CORRECT. This properly identifies something that would be important in determining whether
TranCorp's plan for reducing shipping time will achieve its goal – that is whether loading the freight cars
onto barges will use up all the time saved by not using trucks

D. Regardless of variation in traffic we know that the bridge is typically congested and causes delays.

E. The state of trucks returning to mainland is irrelevant to our evaluation.

19.
The argument states that even in greenhouse environments where sulfur dioxide was entirely eliminate
the urban plants grew more slowly than the rural plants. This seems to indicate that some other factor
appears to account for the difference in growth rates between urban and rural plants, at least within the
greenhouse environment.

A. This choice is irrelevant, since both the urban and rural greenhouses are free of sulfur dioxide.

B. This choice doesn't account for any difference in growth; instead, it indicates that the greenhouse
environments in both urban and rural areas are more conducive to plant growth than are the outside
environments.

C. Irrelevant. Size of class does not matter.

D. CORRECT
This answer choice provides a factor other than sulfur dioxide that may account for the slow growth in
urban environments. This is important because it undermines certain potential conclusions that the
classes might make concerning the relationship between sulfur dioxide and plant growth in urban and
rural environments.
E. Irrelevant. Both environments still remained sulfur dioxide free.

20.

A. CORRECT.

doesn't matter: we don't care about any other revenue, besides advertising-sales revenue.

and even if we did care about those types of revenue, the proportion still wouldn't tell us anything at all
about whether, or by how much, advertising sales revenue has gone up or down.

either of these two reasons is enough to eliminate this choice; the combination of both is absolutely
beyond all doubt.

B. If the circulation has increased this could lead attract more advertisers and thereby lead to increased
advertisement sales. In this case, the increased sales will not be because of reorganization.

Therefore B is definitely relevant.

C. If the number of advertising employees increased, this means that there were more people involved in
advertising and this could have increased the ad sales. This is relevant.

D. This is clearly relevant. If this were true, then reorganization would have helped them cope better.

E. Relevant. If the economy has been booming than the increased sales can be attributed to the
economic growth rather than reorganization.

21.
A. Irrelevant. This does not tell us anything about whether the darts affect the fertility rate of a female.

B. irrelevant. We do not know if the same behavior is observed in other mammals.

C. CORRECT.

If this were true then there is a chance that the un collared rhinos have been tranquilized too. Since the
fertility rate of an un-collared rhino is higher, this would destroy the assumption that a substance in the
tranquilizer inhibits fertility
if it were false, then the conclusion would be right.

D. We are only concerned with females.

E. Irrelevant. Does help evaluate if the tranquilizer is in fact causing a decrease in fertility.

22.
if you see "Which of the following would be most useful to determine in order to evaluate the argument",
then you should think of the same sorts of things you'd consider in a STRENGTHEN/WEAKEN problem.

so, let's think about this as a STRENGTHEN/WEAKEN QUESTION.

when you consider STRENGTHEN/WEAKEN problems, you should SIMPLIFY THE ARGUMENT as much as
possible.

in this problem, the argument basically boils down to they only rub the millipedes during the rainy
season.

during the rainy season, there are more mosquitoes than usual. Therefore, the rubbing must have to do
with mosquitoes.

the weak link in this argument is the assumption that there's no OTHER connection between the rubbing
and the rainy season.

in other words, the argument assumes that because both of these ideas are connected to the
mosquitoes, the mosquitoes MUST be responsible for the overall connection.

therefore, if an answer choice gives ANOTHER connection between the rubbing and the rainy season,
then it weakens the argument.

A. talks about the millipedes protecting themselves against their own predators. Does that address the
hypothesis? No. The hypothesis is specifically about the monkeys protecting themselves against the
mosquitoes. This one is out of scope. Eliminate A.

B. talks about the location of the millipedes – whether they can be found in other places. Does that
address the hypothesis? No. The hypothesis is specifically about certain monkeys located in Venezuela.
This one is also out of scope. Eliminate B.

C. talks about whether other animals besides the capuchins exhibit the same type of behavior. Does that
address the hypothesis? No. The hypothesis is specifically about certain capuchins exhibiting this
behavior. This one is also out of scope. Eliminate C.

D. CORRECT
This option provides another reason why the rubbing might be connected to the rainy season, a reason
that has nothing at all to do with mosquitoes. IT is possible the only reason the monkey run millipedes
during the rainy seasons is because this is the only time they are available.

E. This doesn't really help unless there's some connection to the rainy season (which is the crux of the
argument). whether or not there are such alternate insects, it doesn't really matter unless they would
have differential effects during the rainy season vs. non-rainy season.

23.
A. CORRECT.

If this were true then there is no need to reduce the artificial fat intake. One just needs to increase the
amount of vitamin they consume.

B. Irrelevant. We know essential vitamins are being absorbed. Whether these vitamins are present in the
same food as artificial fat or not, is irrelevant.

C. The columnist does not advise people to stay away from fat. Also we are discussing the cons of
artificial fat. This makes the option irrelevant.

D. Irrelevant. IF there were such food items, it does not answer whether use of artificial fat will absorb
essential vitamins.

E. This would be slightly relevant if we were told that people had no other way of knowing the difference
between artificial fat and actual fat than by taste.

Since we do not know if this is true, even if people were unable to detect the difference in taste, it is
irrelevant to our evaluation.

24.

the problem gives two possible explanations for the scientists' loss of creativity: (1) they're too old, (2)
they've been working in the same field for too long.

therefore, we need some sort of thing that DIFFERENTIATES between these two -- in other words, a
situation in which one of them would be true, but not the other.

A. CORRECT.

This option will tell us two things:


1) Whether it is possible for scientists to do creative work after 40.

2) Whether scientists who do creative work after 40 entered the field at a later age than common - this
means that they may not have the average 15 years’ experience. This indicates that since they have
spent less time in that field, they have not exhausted all their creative avenues as yet.

B. This makes a connection between a scientist's expectation and creativity. This is not supported by the
passage. Nor does it address the explanation that scientists burn out creative avenues after 15 years in
the same field.

In fact this option offers a third, completely different explanation for why scientists do their most creative
work before age forty.

C. This seems to indicate that scientists lose interest in research post 40 and that they cultivate other
interests. This does not help us choose between one application or another.

D. While this tells us that scientists over 40 maybe more favored for grants, it does not address the
question of "creative capacity". Has the creative capacity of these scientists reduced? Is creative work a
criterion for receiving the grant? We do not know.

E. The passage already tells us that the finding is true in many scientific fields. The variations in the
average age at which scientists are most productive will not help us chose between the two options.

25.

Here is the essence of the argument.

1) Lyme disease -> caused by a bacterium contained deer ticks.

2) deer ticks pick up this bacterium when they feed on infected white-footed mice in the larval stage.

3) The other species that the larvae of deer ticks feed on do not contain this bacterium.
Conclusion: If the other species were increased, thereby providing more food for the larvae to feed on
that is not infected, of ticks acquiring the bacterium will decline.

what helps evaluate the above conclusion?

A. Even if this were true, since we plan to increase the population of the other species, the number of
deer ticks feeding on infected white-foot mice should decrease.

B. CORRECT.

If this were true then it tells us that the current population of ticks is limited due to the lack on animals to
feed on and hence they feed on infected mice.

If the presence of uninfected hosts were to increase, the ticks feeding on such hosts would also increase
thereby increasing not just the tick population but also the population of ticks that do not carry the
bacterium.

C. The plan is to increase the animals the ticks feed on rather than increasing the animals that feed on
white-foot mice.

D. Even if this were true, if the number of infected larvae itself were few, then the number of adults
contracting the disease from such larvae would also decrease.

E. We are concerned only with curbing the bacterium that causes Lyme disease. Other diseases are
irrelevant to the argument.

26.
D. CORRECT.

IF true, this would mean that the sales recorded for May, June, July and August were not the average
and there is a chance that the sales might dip so Fedora might not be able to the target.
IF false, then by extrapolation Fedora will meet its target
27.

Let's breakdown the argument.

Fact1: Most of the uranium comes from mines. i.e. deposit on land.

Fact 2: cost of extracting uranium from seawater is higher than the market price for uranium,

Conclusion: until the cost of extracting uranium from seawater can somehow be reduced, this method of
obtaining uranium is unlikely to be commercially viable.

Loophole: The above conclusion depends on the argument that uranium will continue to be available on
land

A) CORRECT. The answer to this question will us if the prices of uranium will continue to remain the
same. If uranium on land decreases, the prices are likely to go up making extracting uranium from
seawater a commercially viable process.

B) Irrelevant. Irrespective of whether uranium is used close to the land where it’s being mined, the cost
of extraction will remain constant. The argument does not take into account transportation costs and
such.

C) Inconclusive. Only tells us that there will be a reduction in the cost of extracting uranium from the sea.
We do not know whether that reduction would even be enough to make up for the current cost disparity,
much less make such a venture profitable.

Note: the important qualifier in the original argument:

until the cost of extracting uranium from seawater can somehow be reduced, this method of
obtaining uranium is unlikely to be commercially viable.

in other words, the argument already acknowledges the fact that reducing the cost of extracting uranium
from seawater (if possible) would make the method commercially viable. so choice (c) basically just
repeats that consideration, and therefore neither helps nor hurts the argument.

on the other hand, if that choice contained any other method of cost reduction, then it would be correct

D) Irrelevant. Even if the amount of uranium is higher in seawater as long as it can be mined on land,
this becomes immaterial to the cost of uranium and ultimately, the argument.
E) Irrelevant. The comparison is between the cost of extracting uranium from mines and extracting it
from sea water.

Fresh water extraction and its costs become irrelevant here.

28.
Key : Centennial Commercial, a real estate developer, is considering a plan to convert several
unoccupied office buildings it owns in Cenopolis into hotels in order to maximize its revenues from
these properties.

Now, this plan would only work if we can be sure that converting office buildings to hotels will indeed
maximize the revenue. Let’s see which option addresses that.

A. This is irrelevant. We are only concerned with maximizing revenues.

B. CORRECT. What if the demand for office space were to exceed hotel revenues in the next few years?
The plan would then fail.

C. Irrelevant. We are interested in “maximizing its revenues from these properties” and do not care for
other travel services.

D. Irrelevant. We are only discussing Cenopolis.

E. Irrelevant. The question is will converting office buildings to hotels increase revenue for the developer
than if he were to continue to let them exist as office buildings.

29.

this argument is presented quite simply, with only one key issue -- is the “dead” quality of guitar strings
produced by lots of playing, or by the presence of foreign substances such as dirt and oil?

According to researcher dirt and oil rather than changes in material are responsible for "dead" Guitar
strings.

We need to evaluate i.e. using some extra info, we either support or weaken the researcher position.

A. IRRELEVANT. Knowing that Metal alloys are used to make the strings used by the classical guitarists
does not affect researcher's position as it does not tell us whether it is fault of these alloys or dirt and oil
that the strings go dead.

B. IRRELEVANT. That classical guitarists can make their strings go dead faster than folk guitarists does
not affect researcher position.

C. IRRELEVANT. Whether identical length strings go dead faster at different rates depending of the
various brand is not relevant to identifying whether it is the fault of the metal or oil and dirt
D. IRRELEVANT. We already know that the quality of sound produced is different from the passage.
“become less responsive and bright in tone”

E. CORRECT. will contribute to resolving this issue, because it will shed light on whether foreign
substances cause this problem.

30.

A. It doesn't matter what lead to the increase in price. We already know now that a cereal manufacturer
reduced the wholesale prices of its cereals by 20 percent and most other manufacturers will follow suit.

B. This is irrelevant too. We are told that the sales of cereals have dropped due to increase in price. Now
the manufacturers have reduced the prices so the sales must go up again. The brand loyalty among
consumers is irrelevant to evaluating this argument.

C. Once again, this is out of scope. Even if more varieties are available, so what? We are concerned only
with its price.

D. CORRECT.

If the answer to this were no, then despite the manufactures slashing their prices, the supermarkets
would still charge the same price as before and the sales would continue to fall.

E. We are concerned with the overall sales of cereals and not a specific type.

31.

The pharmaceutical company claims that Dietol is highly effective in helping people lose weight
and increase their physical endurance. However, since the regular intake of Dietol was
accompanied by daily workouts under professional supervision, it is unclear whether the results
of the experiment should be credited to the effects of Dietol or the rigorous fitness program in
which the subjects participated. Thus, in order to evaluate the conclusion about the
effectiveness of Dietol, it would be useful to separate these two effects and to see what result,
if any, is attributable to Dietol alone.
(A) While the answer to this question could be helpful in estimating the required doses of
Dietol, the actual amount of the supplement taken by participants provides little information
about its effectiveness.

(B) Knowing the maximum weight loss is unlikely to be helpful in evaluating the effectiveness of
the supplement, as the maximum weight loss among the 800 participants is unlikely to be
representative of the typical result and provides little information about the effectiveness of
Dietol.

(C) CORRECT. This answer choice underscores the importance of separating the effects of the
physical program from those resulting from Dietol in evaluating the overall result of the
experiment. For example, if a group of subjects with similar characteristics would be able to
achieve comparable results just by repeating the fitness program over the 9-week experiment,
this outcome would cast serious doubts on the claim of the pharmaceutical company about the
effectiveness of Dietol.

(D) Since the company does not make a claim about Dietol’s effectiveness for a specific age
group, the information about the average age is beyond the scope of this argument.

(E) Note that the pharmaceutical company claims that Dietol will help obese people lose weight
and improve endurance. No claim is made about the supplement’s effect on physical strength.
Therefore, information about the improvement in physical strength is unrelated to the
conclusion.

32.

The argument concludes that the prosecution of a small number of people who download music
illegally will have a minimal impact on the overall number of people who engage in illegal
downloading. The correct answer must relate specifically to this issue and provide additional
insight as to whether it seems reasonable.

(A) One premise of the argument states that the recording industry does not have the
resources to prosecute all individuals who download music illegally, while a second premise
states the number of people who will be charged with a crime is limited. These statements
indicate that the legal resources of the recording industry are too limited to have a major
impact on the overall number of people who engage in illegal downloading, no matter how
these lawyers dedicate their time.

(B) If a small minority of individuals were responsible for the majority of illegal song downloads,
the actions of the recording industry could have a significant impact on the number of
downloaded songs. The conclusion of the argument, however, was about the number of people
who download songs illegally; this number would remain unaffected.

(C) Whether songs are downloaded illegally and then shared with other Internet users is not
relevant to the conclusion.

(D) Similar to answer choice A, this choice is limited by the premises of the argument. If new
Internet security technology permits the recording industry to more quickly and easily identify
individuals who illegally download music, then the recording industry will know who is breaking
the law. However, the lack of industry resources still restricts the industry’s ability to prosecute
a large number of people, even if they are identified as individuals who illegally download
music.

(E) CORRECT. The argument concludes that the prosecution of a small number of people who
download music illegally will have a minimal impact on the overall number of people who
engage in illegal downloading. However, if the threat of prosecution were enough to “alter the
behavior” of others (i.e., deter them from illegally downloading music), the actions of the
recording industry could have a significant impact on the number of people who illegally
download music.

33.

The author claims that the "decrease in such insourcing will hurt America's competitiveness in
basic research and applied technology, with serious consequences for years to come." In
addition, the author claims that the decline in insourcing will "negatively affect the global
outlook and experience of American students" because they will "not have the opportunity to
learn about foreign cultures directly from members of those cultures." We are asked to find a
question whose answer would provide information useful to evaluating the claims of the
argument.
(A) The focus of the argument is not on the financial costs of insourcing.

(B) The focus of the argument is not on how insourcing can compensate for outsourcing.

(C) The focus of the argument is not on the specific numbers of foreign-born students, but on
the decline in their number instead.

(D) CORRECT. The argument assumes that the students will not have contact with foreigners
through channels other than school. This choice asks whether the students have such contact
elsewhere. If the answer to this question is "yes", the author's claims carry less weight.

(E) A government policy might have an effect on insourcing, but it would not necessarily have
an effect on the specific claims of the argument.

34.

Profits for a particular product have been going down and the CFO has determined that this is
because, on the one hand, the cost to make the product has increased and, on the other,
consumers won't pay more than the current price (recall that Profit = Revenues – Costs). The
CEO only wants Company X to sell products with increasing profit margins; as a result, the CFO
decides the solution is to stop making this product. This decision would make sense only if we
can be assured that there is no way to have an increasing profit margin for the product in
question.

(A) Whether there are new, profitable products does not address the issue of whether there is a
way to achieve increasing profit margins for the flagship product.

(B) Whether the management team agrees with the CFO's recommendation does not address
the issue of whether there is a way to achieve increasing profit margins for the flagship
product.
(C) This may increase the revenues earned by the product, but this choice does not address the
additional cost associated with new features, so we still do not know whether we can achieve
increasing profit margins for the product. We may be able to, but we may not: the features
may cost more than the increased price that consumers would be willing to pay.

(D) CORRECT. If the costs for the existing product can be reduced, then the profit margin will
increase (again, recall that Profits = Revenues – Costs). If the costs cannot be reduced, then
the profit margins will not increase.

(E) The flagship product's revenues as a percentage of total revenues does not address the
issue of whether there is a way to achieve increasing profit margins for the flagship product
(although it does highlight why the company might find itself with a big problem if it follows the
CEO's advice!).

35.

This argument concludes that the best way to have good governance in the future is to
preserve the Pendleton Act. It bases that claim on the record of the first hundred years under
that Act. The argument assumes that American government service is still uncorrupted and that
no better way exists to preserve that state.

(A) The methods used by other governments in other countries are irrelevant to this argument,
which only concerns America. Furthermore, one cannot assume that the Swiss and British
governments are free of corruption.

(B) The opinions, wishes, and satisfaction levels of government applicants and employees are
irrelevant to the level of corruption.

(C) CORRECT. This would confirm or deny the assumption that American government service is
currently not corrupt. The argument only claims that this was so for the first hundred years
after the passage of the Pendleton Act. There is no information about the level of corruption for
the past twenty-some years.
(D) The number of Presidents assassinated is utterly irrelevant to an argument about corruption
in government service.

(E) This percentage of corrupt office-holders in the first hundred years of the Pendleton Act
would provide no information about the level of corruption in government service for the past
twenty years, which is the gap in the evidence. Thus, investigating this percentage would not
be that useful for the evaluation of a conclusion that involves the immediate present and the
future.

36.

The conclusion of the argument is that the rate of extinction of bird species will not surge in this
century. Evidence from native North American raptors (birds of prey) is cited: namely, that even
though several such species nearly vanished between 1900 and 1970, the populations of these
birds have since rebounded. We are asked to select a question which would, when answered,
provide information that would help us evaluate this claim.

(A) This answer choice inquires about the introduction of non-native raptors. However, the
argument cites as evidence the resurgence in native raptor populations. Thus, this question
does not shed light on the relevance of the evidence.

(B) CORRECT. This question points to an unwarranted assumption on the author's part. The
author assumes that the rebound in native North American raptor populations since 1970 was
not due to any special effort to save these species. If special efforts have been made to save
these raptor species, then one might not be able to generalize this evidence to other bird
species or to make predictions about population trends in the next century, using this evidence.

(C) This question is irrelevant: the argument does not depend in any way on the experience of
the zoologists. It hinges instead on the trend in raptor populations that the author uses to
justify his or her point.

(D) This question is irrelevant. Note that the given evidence states that "local populations of
these raptors have rebounded." Thus, population increases of native North American raptors
outside of North America are not pertinent.
(E) This question is irrelevant: the argument does not depend in any way on the number of
acres of woodland set aside as bird refuges.

37.

Two points of view are expressed in the argument: consumer groups claim that a la carte
pricing will reduce consumer costs, while the cable television industry claims that the current
package pricing structure is most cost effective for consumers. If the goal of the government is
to reduce the cost of cable television for consumers, it is critical for the government to
determine which point of view is correct. The cable television industry favorably compares the
cost of buying bundled channels to the cost of buying those same channels individually, but
what if most consumers would choose to greatly reduce the number of channels purchased?
That could mean that a la carte pricing would result in lower cable bills for those consumers.

(A) The government's decision is based only on the costs to consumers, not the number of
channels available to them.

(B) The government's decision is based only on the costs to consumers, not the advertising
profits of the cable television companies.

(C) CORRECT. If consumers would not choose to order all of the channels they currently buy as
part of a package subscription, then the cable television industries' claim that a la carte costs
always would increase is faulty. If most consumers only watch and wish to pay for a few of
their favorite channels, a la carte pricing could very well result in lower cable bills for those
consumers. Consumers who want to purchase more channels could still choose the package
subscriptions, so a la carte pricing would not harm them.

(D) The government's decision is based on only the costs to consumers, not the current
purchasing agreements that exist between the cable television companies and content
providers.

(E) The government's decision is based on only the costs to consumers, not the technical
equipment requirements a change in cost structure would require.
38.

The director of programming argues that the addition of a conservative news program in
response to the hiring of a liberal news personality represents an institutional attempt at
balancing different perspectives, rather than any conservative bias. The evidence provided by
the director is outside of the context of any past actions on the part of the Network; it would be
useful to have more information about actions the Network has taken in response to the hiring
of conservative or liberal news personalities in the past to determine a pattern of behavior.

(A) CORRECT. If the Network responds to the addition of a liberal news personality in the same
way that it does the addition of a conservative news personality, then the argument presented
by the director is valid. An identical response to the hiring of a personality from either side of
the political spectrum suggests that the Network does act in a fair and balanced manner, at
least in this regard.

(B) Whether the Network presents a program airing the opinions of its viewers on contemporary
issues does not indicate either a balanced approach or a conservative bias. This statement is
irrelevant.

(C) The nature of the former employer of the newly-hired liberal news personality is not
relevant in determining whether or not there is a systematic conservative bias regarding the
overall programming of the Network.

(D) Whether the new liberal news personality has indicated that he has felt any editorial
pressure is not relevant; he might not want to admit to such pressure even if it did exist.
Moreover, the experience of one employee does not address the overall programming of the
Network.

(E) That the Network has suffered from lower ratings in the past year is not relevant to
determining the bias or lack thereof of the Network, though it may explain why new
personalities and programming are being added.
39.

The conclusion of the passage is that parents can ensure that their children will not spend
money on frivolous items by limiting their children's allowances. This claim is based on the
observed difference between the spending habits of children who receive large allowances and
those of children who receive small allowances. The argument assumes that the high dollar
amount of the allowance – as opposed to some other unobserved factor – is directly linked to
the fact that children spend the money on items their parents consider frivolous. Information
that provides data about any other factor that might be the cause of the children's spending
behavior would help to evaluate the validity of the conclusion.

(A) The actual dollar amount received by adolescents who receive large weekly allowances is
not related to the conclusion of the passage. Whether this amount is $40 or $10, the key is that
it is considered by the child development specialists to be a "large" weekly allowance.

(B) CORRECT. One alternative to the conclusion of the passage is that the standard used to
judge an item as frivolous was much lower for parents who gave their children large weekly
allowances than for parents who gave their children small weekly allowances. If for example,
the former group of parents considered all movie tickets to be frivolous, while the latter did not,
then this fact (and not the difference in allowance money) might explain the difference
observed by the child development specialists. Thus, information about any differences among
parents in the standard used to judge an item as frivolous would be extremely relevant in
evaluating the validity of the conclusion of the passage.

(C) The background of the child development specialists who made the observation has no
bearing on the conclusion. The conclusion is based on the observation, not on the credentials of
those making the observation.

(D) Family income differences have no clear relevance to the link posited between high
allowances and spending on frivolous items.

(E) Adolescents who receive no weekly allowance play no role in the argument. Thus,
information about this group of adolescents has no relationship to the conclusion of the
passage.
40.

The conclusion of the argument is that the prediction of decreased consumer spending has not
come to pass. The evidence for this is that there has been no corresponding increase in the
amount of money set aside in savings accounts by the general public.

This question asks us to choose a question whose answer will provide information relevant to
evaluating the significance of the fact that the amount of money placed in savings accounts has
not increased. The author mentions this savings pattern to support the claim that people have
not been curtailing their discretionary spending. So we need to find a question whose answer
will relate to savings and spending patterns.

(A) Information about which business sectors were most affected by the layoffs will not help us
establish the relevance of the savings pattern to the claim.

(B) The savings used by those who are laid off has no bearing on the savings patterns of those
who are not laid off, which is the group this argument addresses ("even if their jobs were
secure").

(C) CORRECT. This question asks about the prices of necessities such as food and utilities. If
people have not been saving their money, perhaps they have been spending it on necessities
(as opposed to spending it on discretionary items). If these items have become more
expensive, then perhaps people have not been able to save money despite curtailing their
discretionary spending.

(D) The percentage of people with savings accounts is not related to the question of whether
people have curtailed their discretionary spending or increased the amount they save.

(E) Knowing the statistics for average salaries during the period since the layoffs will not help
us evaluate the significance of the spending and savings patterns during that time. It might
help to be able to compare the statistics for the periods before and after the layoffs, but
knowing just one of those statistics tells us nothing about the trends.
41.

The correct answer is C. The conclusion of the argument is that the price of chocolate will
increase within six months. The basis for that claim is that the wholesale price of cocoa has
increased. However, if the price of other ingredients in the chocolate has dropped, the decrease
could offset the higher price of cocoa and render the argument invalid. The only choice to
address this is C.

42.

The correct answer is B. The conclusion of the argument is that "eating whole grains can have
an appreciable effect on cholesterol levels." This assertion is based on the fact that some
people who ate three servings of whole grains every day for six months had lower cholesterol
than did people who did not, even though their cholesterol levels were the same before the
study began. The argument does not take into account, however, other factors, such as
exercise, that may have contributed to the difference in cholesterol levels. Choice B asks
whether there is indeed another factor – exercise – that should be taken into account.
www.top-one-percent.com

Assumptions Latest Questions

1. Until now, only injectable vaccines against influenza have been available. Parents are reluctant to
subject children to the pain of injections, but adults, who are at risk of serious complications from
influenza, are commonly vaccinated. A new influenza vaccine, administered painlessly in a nasal
spray, is effective for children. However, since children seldom develop serious complications from
influenza, no significant public health benefit would result from widespread vaccination of
children using the nasal spray. Which of the following is an assumption on which the
argument depends?
A. Any person who has received the injectable vaccine can safely receive the nasal-spray vaccine as
well.
B. The new vaccine uses the same mechanism to ward off influenza as jnjectable vaccines do.
C. The injectable vaccine is affordable for all adults.
D. Adults do not contract influenza primarily from children who have influenza.
E. The nasal spray vaccine is not effective when administered to adults.

2. Last year all refuse collected by Shelbyville city services was incinerated. This incineration
generated a large quantity of residue ash. In order to reduce the amount of residue ash Shelbyville
generates this year to half of last year’s total, the city has revamped its collection program. This
year city services will separate for recycling enough refuse to reduce the number of truckloads of
refuse to be incinerated to half of last year’s number. Which of the following is required for the
revamped collection program to achieve its aim?
A. This year, no materials that city services could separate for recycling will be incinerated.
B. Separating recyclable materials from materials to be incinerated will cost Shelbyville less than half
what it cost last year to dispose of the residue ash.
C. Refuse collected by city services will contain a larger proportion of recyclable materials this year
than it did last year.
D. The refuse incinerated this year will generate no more residue ash per truckload incinerated than
did the refuse incinerated last year.
E. The total quantity of refuse collected by Shelbyville city services this year will be no greater than
that collected last year.

3. Newspaper editorial: In an attempt to reduce the crime rate, the governor is getting tough on
criminals and making prison conditions harsher. Part of this effort has been to deny inmates the
access they formerly had to college-level courses. However, this action is clearly counter to the
governor’s ultimate goal, since after being released from prison, inmates who had taken such
courses committed far fewer crimes overall than other inmates. Which of the following is an
assumption on which the argument depends?
A. Not being able to take college-level courses while in prison is unlikely to deter anyone from a
crime that he or she might otherwise have committed.
B. Former inmates are no more likely to commit crimes than are members of the general population.
C. The group of inmates who chose to take college-level courses were not already less likely than
other inmates to commit crimes after being released.
D. Taking high school level courses in prison has less effect on an inmate’s subsequent behavior than
taking college-level courses does.
E. The governor’s ultimate goal actually is to gain popularity by convincing people that something
effective is being done about crime.
4. Two centuries ago, Tufe Peninsula became separated from the mainland, isolating on the newly
formed Tufe Island a population of Turfil sunflowers. This population’s descendants grow to be, on
average, 40 centimeters shorter than Turfil sunflowers found on the mainland. Tufe Island is
significantly drier than Tufe Peninsula was. So the current average height of Tufe’s Turfil
sunflowers is undoubtedly at least partially attributable to changes in Tufe’s environmental
conditions. Which of the following is an assumption on which the argument depends?
A. There are no types of vegetation on Tufe Island that are known to benefit from dry conditions.
B. There were about as many Turfil sunflowers on Tufe Peninsula two centuries ago as there are on
Tufe Island today.
C. The mainland’s environment has not changed in ways that have resulted in Turfil sunflowers on
the mainland growing to be 40 centimeters taller than they did two centuries ago.
D. The soil on Tufe Island, unlike that on the mainland, lacks important nutrients that help Turfil
sunflowers survive and grow tall in a dry environment.
E. The 40-centimeter height difference between the Turfil sunflowers on Tufe Island and those on the
mainland is the only difference between the two populations.

5. The average hourly wage of television assemblers in Vernland has long been significantly lower
than that in neighboring Borodia. Since Borodia dropped all tariffs on Vernlandian televisions
three years ago, the number of televisions sold annually in Borodia has not changed. However,
recent statistics show a drop in the number of television assemblers in Borodia. Therefore,
updated trade statistics will probably indicate that the number of televisions Borodia imports
annually from Vernland has increased. Which of the following is an assumption on which the
argument depends?
A. The number of television assemblers in Vernland has increased by at least as much as the number
of television assemblers in Borodia has decreased.
B. Televisions assembled in Vernland have features that televisions assembled in Borodia do not
have.
C. The average number of hours it takes a Borodian television assembler to assemble a television has
not decreased significantly during the past three years.
D. The number of televisions assembled annually in Vernland has increased significantly during the
past three years.
E. The difference between the hourly wage of television assemblers in Vernland and the hourly wage
of television assemblers in Borodia is likely to decrease in the next few years.

6. In Teruvia, the quantity of rice produced per year is currently just large enough to satisfy domestic
demand. Teruvia’s total rice acreage will not be expanded in the foreseeable future, nor will rice
yields per acre increase appreciably. Teruvia’s population, however, will be increasing
significantly for years to come. Clearly, therefore, Teruvia will soon have to begin importing rice.
Which of the following is an assumption on which the argument depends?
A. No pronounced trend of decreasing per capita demand for rice is imminent in Teruvia.
B. Not all of the acreage in Teruvia currently planted with rice is well suited to the cultivation of rice.
C. None of the strains of rice grown in Teruvia are exceptionally high-yielding.
D. There are no populated regions in Teruvia in which the population will not increase.
E. There are no major crops other than rice for which domestic production and domestic demand are
currently in balance in Teruvia.
7. In the past the country of Siduria has relied heavily on imported oil. Siduria recently implemented
a program to convert heating systems from oil to natural gas. Siduria already produces more
natural gas each year than it burns, and oil production in Sidurian oil fields is increasing at a
steady pace. If these trends in fuel production and usage continue, therefore, Sidurian reliance on
foreign sources for fuel should decline soon. Which of the following is an assumption on which
the argument depends?
A. In Siduria the rate of fuel consumption is rising no more quickly than the rate of fuel production.
B. Domestic production of natural gas is rising faster than is domestic production of oil in Siduria.
C. No fuel other than natural gas is expected to be used as a replacement for oil in Siduria.
D. Buildings cannot be heated by solar energy rather than by oil or natural gas.
E. All new homes that are being built will have natural-gas-burning heating systems.

8. Three large companies and seven small companies currently manufacture a product with potential
military applications. If the government regulates the industry, it will institute a single set of
manufacturing specifications to which all ten companies will have to adhere. In this case,
therefore, since none of the seven small companies can afford to convert their production lines to a
new set of manufacturing specifications, only the three large companies will be able to remain in
business. Which of the following is an assumption on which the author’s argument relies?
A. None of the three large companies will go out of business if the government does not regulate the
manufacture of the product.
B. It would cost more to convert the production lines of the small companies to a new set of
manufacturing specifications than it would to convert the production lines of the large companies.
C. Industry lobbyists will be unable to dissuade the government from regulating the industry.
D. Assembly of the product produced according to government manufacturing specifications would
be more complex than current assembly procedures.
E. None of the seven small companies currently manufactures the product to a set of specifications
that would match those the government would institute if the industry were to be regulated.

9. A significant number of complex repair jobs carried out by Ace Repairs have to be reworked under
the company’s warranty. The reworked jobs are invariably satisfactory. When initial repairs are
inadequate, therefore, it is not because the mechanics lack competence; rather, there is clearly a
level of focused concentration that complex repairs require that is elicited more reliably by rework
jobs than by first-time jobs. The argument above assumes which of the following?
A. There is no systematic difference in membership between the group of mechanics who do first-
time jobs and the group of those who do rework jobs.
B. There is no company that successfully competes with Ace Repairs for complex repair jobs.
C. Ace Repairs’ warranty is good on first-time jobs but does not cover rework jobs.
D. Ace Repairs does not in any way penalize mechanics who have worked on complex repair jobs that
later had to be reworked.
E. There is no category of repair jobs in which Ace Repairs invariably carries out first-time jobs
satisfactorily.
10. With a record number of new companies starting up in Derderia, and with previously established
companies adding many new jobs, a record number of new jobs were created last year in the
Derderian economy. This year, previously established companies will not be adding as many new
jobs overall as such companies added last year. Therefore, unless a record number of companies
start up this year, Derderia will not break its record for new jobs created. Which of the following
is an assumption on which the argument relies?
A. In a given year, new companies starting up create more new jobs on average than do previously
established companies.
B. The number of people seeking employment is no larger this year than it was last year.
C. This year, the new companies starting up will not provide substantially more jobs per company
than did new companies last year.
D. Previously established companies in Derderia will be less profitable this year than such companies
were last year.
E. The number of jobs created in the Derderian economy last year was substantially larger than the
number of jobs lost.

11. When storing Renaissance oil paintings, museums conform to standards that call for careful
control of the surrounding temperature and humidity, with variations confined within narrow
margins. Maintaining this environment is very costly, and recent research shows that even old oil
paint is unaffected by wide fluctuations in temperature and humidity. Therefore, museums could
relax their standards and save money without endangering their Renaissance oil paintings. Which
of the following is an assumption on which the argument depends?
A. Renaissance paintings were created in conditions involving far greater fluctuations in temperature
and humidity than those permitted by current standards.
B. Under the current standards that museums use when storing Renaissance oil paintings, those
paintings do not deteriorate at all.
C. Museum collections typically do not contain items that are more likely to be vulnerable to
fluctuations in temperature and humidity than Renaissance oil paintings.
D. None of the materials in Renaissance oil paintings other than the paint are vulnerable enough to
relatively wide fluctuations in temperature and humidity to cause damage to the paintings.
E. Most Renaissance oil paintings are stored in museums located in regions near the regions where
the paintings were created.

12. For similar cars and drivers, automobile insurance for collision damage has always cost more in
Greatport than in Fairmont. Police studies, however, show that cars owned by Greatport residents
are, on average, slightly less likely to be involved in a collision than cars in Fairmont. Clearly,
therefore, insurance companies are making a greater profit on collision-damage insurance in
Greatport than in Fairmont.
Which of the following is an assumption on which the argument depends?
A. Repairing typical collision damage does not cost more in Greatport than in Fairmont.
B. There are no more motorists in Greatport than in Fairmont.
C. Greatport residents who have been in a collision are more likely to report it to their insurance
company than Fairmont residents are.
D. Fairmont and Greatport are the cities with the highest collision-damage insurance rates.
E. The insurance companies were already aware of the difference in the likelihood of collisions
before the publication of the police reports.
13. Agricultural societies cannot exist without staple crops. Several food plants, such as kola and okra,
are known to have been domesticated in western Africa, but they are all supplemental, not staple,
foods. All the recorded staple crops grown in western Africa were introduced from elsewhere,
beginning, at some unknown date, with rice and yams. Therefore, discovering when rice and yams
were introduced into western Africa would establish the earliest date at which agricultural
societies could have arisen there. Which of the following is an assumption on which the
argument depends?
A. People in western Africa did not develop staple crops that they stopped cultivating once rice and
yams were introduced.
B. There are no plants native to western Africa that, if domesticated, could serve as staple food crops.
C. Rice and yams were grown as staple crops by the earliest agricultural societies outside of western
Africa.
D. Kola and okra are better suited to growing conditions in western Africa than domesticated rice
and yams are.
E. Kola and okra were domesticated in western Africa before rice and yams were introduced there.

14. In the year following an eight-cent increase in the federal tax on a pack of cigarettes, sales of
cigarettes fell ten percent. In contrast, in the year prior to the tax increase, sales had fallen one
percent. The volume of cigarette sales is therefore strongly related to the after-tax price of a pack
of cigarettes. The argument above requires which of following assumptions?
A. During the year following the tax increase, the pretax price of a pack of cigarettes did not increase
by as much as it had during the year prior to the tax increase.
B. The one percent fall in cigarette sales in the year prior to tax increase was due to a smaller tax
increase.
C. The pretax price of a pack of cigarettes gradually decreased throughout the year before and the
year after the tax increase.
D. For the year following the tax increase, the pretax price of a pack of cigarettes was not eight or
more cents lower than it had been the previous year.
E. As the after-tax price of a pack of cigarettes rises, the pretax price also rises.

15. In Kravonia, the average salary for jobs requiring a college degree has always been higher than the
average salary for jobs that do not require a degree. Current enrollments in Kravonia’s colleges
indicate that over the next four years the percentage of the Kravonian workforce with college
degrees will increase dramatically. Therefore, the average salary for all workers in Kravonia is
likely to increase over the next four years. Which of the following is an assumption on which
the argument depends?
A. Kravonians with more than one college degree earn more, on average, than do Kravonians with
only one college degree.
B. The percentage of Kravonians who attend college in order to earn higher salaries is higher now
than it was several years ago.
C. The higher average salary for jobs requiring a college degree is not due largely to a scarcity among
the Kravonian workforce of people with a college degree.
D. The average salary in Kravonia for jobs that do not require a college degree will not increase over
the next four years.
E. Few members of the Kravonian workforce earned their degrees in other countries.
16. For most people, the left half of the brain controls linguistic capabilities, but some people have
their language centers in the right half. When a language center of the brain is damaged, for
example by a stroke, linguistic capabilities are impaired in some way. Therefore, people who have
suffered a serious stroke on the left side of the brain without suffering any such impairment must
have their language centers in the right half. Which of the following is an assumption on which
the reasoning in the argument above depends?
A. No part of a person’s brain that is damaged by a stroke ever recovers.
B. Impairment of linguistic capabilities does not occur in people who have not suffered any damage
to any language center of the brain.
C. Strokes tend to impair linguistic capabilities more severely than does any other cause of damage to
language centers in the brain.
D. If there are language centers on the left side of the brain, any serious stroke affecting that side of
the brain damages at least one of them.
E. It is impossible to determine which side of the brain contains a person’s language centers if the
person has not suffered damage to either side of the brain.

17. The Earth’s rivers constantly carry dissolved salts into its oceans. Clearly, therefore, by taking the
resulting increase in salt levels in the oceans over the past hundred years and then determining
how many centuries of such increases it would have taken the oceans to reach current salt levels
from a hypothetical initial salt-free state, the maximum age of the Earth’s oceans can be accurately
estimated. Which of the following is an assumption on which the argument depends?
A. The quantities of dissolved salts deposited by rivers in the Earth’s oceans have not been unusually
large during the past hundred years.
B. At any given time, all the Earth’s rivers have about the same salt levels.
C. There are salts that leach into the Earth’s oceans directly from the ocean floor.
D. There is no method superior to that based on salt levels for estimating the maximum age of the
Earth’s oceans.
E. None of the salts carried into the Earth’s oceans by rivers are used up by biological activity in the
oceans.

18. In a certain wildlife park, park rangers are able to track the movements of many rhinoceroses
because those animals wear radio collars. When, as often happens, a collar slips off, it is put back
on. Putting a collar on a rhinoceros involves immobilizing the animal by shooting it with a
tranquilizer dart. Female rhinoceroses that have been frequently recollared have significantly
lower fertility rates than uncollared females. Probably, therefore, some substance in the
tranquilizer inhibits fertility. Which of the following is an assumption on which the argument
depends?
A. The dose of tranquilizer delivered by a tranquilizer dart is large enough to give the rangers putting
collars on rhinoceroses a generous margin of safety.
B. The fertility rate of uncollared female rhinoceroses in the park has been increasing in the past few
decades.
C. Any stress that female rhinoceroses may suffer as a result of being immobilized and handled has
little or no negative effect on their fertility.
D. The male rhinoceroses in the wildlife park do net lose their collars as often as the park’s female
rhinoceroses do.
E. The tranquilizer used in immobilizing rhinoceroses is the same as the tranquilizer used in working
with other large mammals.
19. Frobisher, a sixteenth-century English explorer, had soil samples from Canada’s Kodlunarn Island
examined for gold content. Because high gold content was reported, Elizabeth I funded two mining
expeditions. Neither expedition found any gold there. Modern analysis of the island’s soil indicates
a very low gold content. Thus the methods used to determine the gold content of Frobisher’s
samples must have been inaccurate. Which of the following is an assumption on which the
argument depends?
A. The gold content of the soil on Kodlunarn Island is much lower today than it was in the sixteenth
century.
B. The two mining expeditions funded by Elizabeth I did not mine the same part of Kodlunarn Island.
C. The methods used to assess gold content of the soil samples provided by Frobisher were different
from those generally used in the sixteenth century.
D. Frobisher did not have soil samples from any other Canadian island examined for gold content.
E. Gold was not added to the soil samples collected by Frobisher before the samples were examined.

20. In response to mounting public concern, an airplane manufacturer implemented a program with
the well-publicized goal of reducing by half the total yearly amount of hazardous waste generated
by its passenger-jet division. When the program began in 1994, the division’s hazardous waste
output was 90 pounds per production worker; last year it was 40 pounds per production worker.
Clearly, therefore, charges that the manufacturer’s program has not met its goal are false. Which
of the following is an assumption on which the argument depends?
A. The amount of nonhazardous waste generated each year by the passenger-jet division has not
increased significantly since 1994.
B. At least as many passenger jets were produced by the division last year as had been produced in
1994.
C. Since 1994, other divisions in the company have achieved reductions in hazardous waste output
that are at least equal to that achieved in the passenger-jet division.
D. The average number of weekly hours per production worker in the passenger-jet division was not
significantly greater last year than it was in 1994.
E. The number of production workers assigned to the passenger-jet division was not significantly
less in 1994 than it was last year.

21. The milk of many mammals contains cannabinoids, substances that are known to stimulate certain
receptors in the brain. To investigate the function of cannabinoids, researchers injected newborn
mice with a chemical that is known to block cannabinoids from reaching their receptors in the
brain. The injected mice showed far less interest in feeding than normal newborn mice do.
Therefore, cannabinoids probably function to stimulate the appetite. Which of the following is an
assumption on which the argument depends?
A. Newborn mice do not normally ingest any substance other than their mothers’ milk.
B. Cannabinoids are the only substances in mammals’ milk that stimulate the appetite.
C. The mothers of newborn mice do not normally make any effort to encourage their babies to feed.
D. The milk of mammals would be less nutritious if it did not contain cannabinoids.
E. The chemical that blocks cannabinoids from stimulating their brain receptors does not
independently inhibit the appetite.
22. In 1992 outlaw fishing boats began illegally harvesting lobsters from the territorial waters of the
country of Belukia. Soon after, the annual tonnage of lobster legally harvested in Belukian waters
began declining; in 1996, despite there being no reduction in the level of legal lobster fishing
activity, the local catch was 9,000 tons below pre-1992 levels. It is therefore highly likely that the
outlaw fishing boats harvested about 9,000 tons of lobster illegally that year. Which of the
following is an assumption on which the argument depends?
A. The illegal lobster harvesting was not so extensive that the population of catchable lobsters in
Belukia’s territorial waters had sharply declined by 1996.
B. The average annual lobster catch, in tons, of an outlaw fishing boat has increased steadily since
1992.
C. Outlaw fishing boats do not, as a group, harvest more lobsters than do licensed lobster-fishing
boats.
D. The annual legal lobster harvest in Belukia in 1996 was not significantly less than 9,000 tons.
E. A significant proportion of Belukia’s operators of licensed lobster-fishing boats went out of
business between 1992 and 1996.

23. Tourists have long complained that hotel accommodations in Midville are too expensive. Starting
last year, the city council, hoping to attract more tourists, lowered the hotel tax rate to 5 percent of
room charges. By the end of last year, Midville had taken in no less money from hotel taxes than it
did the year before, so an examination of the hotel records will show that more tourists stayed in
city hotels last year than the year before. Which of the following is an assumption on which the
argument depends?
A. The tourists who stayed in Midville hotels last year were aware that the hotel tax rate had been
lowered.
B. The average price of hotel accommodations in Midville was not significantly higher than in hotels
in other cities either last year or the year before.
C. The average length of a tourist’s stay in Midville hotels was not longer last year than it had been
the year before.
D. There were significantly more efforts to publicize Midville as a tourist destination last year than
there had been the year before.
E. On average, tourists in Midville did not spend significantly more on meals last year than they did
on hotels accommodations.

24. Because it was long thought that few people would watch lengthy televised political messages,
most televised political advertisements, like commercial advertisements, took the form of short
messages. Last year, however, one candidate produced a half-hour-long advertisement. During the
half hour the advertisement was aired, a substantial portion of the viewing public tuned into the
advertisement. Clearly, then, many more people are interested in watching lengthy televised
political messages than was previously thought. Which of the following is an assumption on
which the argument depends?
A. The candidate’s ratings improved significantly as a result of the half-hour-long political
advertisement.
B. Political advertisements have become increasingly influential in determining voters’ decisions at
the polls.
C. Many people would appreciate the opportunity to become better acquainted with political
candidates’ views on current political issues.
D. Most people who are interested in political issues watch television regularly.
E. Most of the viewers who tuned in to the candidate’s half-hour-long advertisement last year did not
change channels after the first few minutes.
25. Because ethylene dibromide, a chemical used to fumigate grain, was blamed for the high rate of
nerve damage suffered by people who work in grain-processing plants, many such plants switched
to other chemical fumigants two years ago. Since then, however, the percentage of workers at
these plants who were newly diagnosed with nerve damage has not dropped significantly.
Therefore, either ethylene dibromide was wrongly blamed or else the new chemicals also cause
nerve damage. Which of the following is an assumption on which the argument depends?
A. If the new chemicals cause nerve damage, the nerve damage caused would be different from any
nerve damage that ethylene dibromide may cause.
B. There are no chemical fumigants that are completely safe for workers in grain-processing plants.
C. If ethylene dibromide causes nerve damage, it does not take two years or longer for that damage to
become detectable.
D. Workers at grain-processing plants typically continue to work there even after being diagnosed
with nerve damage.
E. Workers at grain-processing plants that still use ethylene dibromide continue to have a high rate
of nerve damage.

26. Many people suffer an allergic reaction to certain sulfites, including those that are commonly
added to wine as preservatives. However, since there are several wine makers who add sulfites to
none of the wines they produce, people who would like to drink wine but are allergic to sulfites
can drink wines produced by these wine makers without risking an allergic reaction to sulfites.
Which of the following is an assumption on which the argument depends?
A. These wine makers have been able to duplicate the preservative effect produced by adding sulfites
by means that do not involve adding any potentially allergenic substances to their wine.
B. Not all forms of sulfite are equally likely to produce the allergic reactions.
C. Wine is the only beverage to which sulfites are commonly added.
D. Apart from sulfites, there are no substances commonly present in wine that give rise to an allergic
reaction.
E. Sulfites are not naturally present in the wines produced by these wine makers in amounts large
enough to produce an allergic reaction in someone who drinks these wines.

27. Most household appliances use electricity only when in use. Many microwave ovens, however,
have built-in clocks and so use some electricity even when they are not in use. The clocks each
consume about 45 kilowatt-hours per year. Therefore, households whose microwave oven has no
built-in clock use 45 kilowatt-hours per year less, on average, than do comparable households
whose microwave oven is otherwise similar but has a built-in clock. Which of the following is an
assumption on which the argument depends?
A. Households that do not have a microwave oven use less energy per year, on average, than do
households that have a microwave oven.
B. Microwave ovens with a built-in clock do not generally cost more to buy than microwave ovens
without a built-in clock.
C. All households that have a microwave oven also have either a gas oven or a conventional electric
oven.
D. Households whose microwave oven does not have a built-in clock are no more likely to have a
separate electric clock plugged in than households whose microwave oven has one.
E. There are more households that have a microwave oven with a built-in clock than there are
households that have a microwave oven without a built-in clock.
28. Lawmaker: Raising taxes is not the only means of reducing government debt. The government’s
stockpile of helium is worth 25 percent more, at current market prices, than the debt accumulated
in acquiring and storing it. Therefore, by selling the helium, the government can not only pay off
that debt but reduce its overall debt as well. Which of the following is an assumption on which
the argument depends?
A. The government has no current need for helium.
B. Twenty-five percent of the debt the government has accumulated in stockpiling helium is not an
insignificant portion of the government’s total debt.
C. It is not in the lawmaker’s interest to advocate raising taxes as a means of reducing government
debt.
D. Attempts to sell the government’s helium will not depress the market price of helium by more than
25 percent.
E. The government will not incur any costs in closing its facilities for stockpiling helium.

29. Early in the twentieth century, Lake Konfa became very polluted. Recently fish populations have
recovered as release of industrial pollutants has declined and the lake’s waters have become
cleaner. Fears are now being voiced that the planned construction of an oil pipeline across the
lake’s bottom might revive pollution and cause the fish population to decline again. However, a
technology for preventing leaks is being installed. Therefore, provided this technology is effective,
those fears are groundless. The argument depends on assuming which of the following?
A. Apart from development related to the pipeline, there will be no new industrial development
around the lake that will create renewed pollution in its waters.
B. Other than the possibility of a leak, there is no realistic pollution threat posed to the lake by the
pipeline’s construction.
C. There is no reason to believe that the leak-preventing technology would be ineffective when
installed in the pipeline in Lake Konfa.
D. Damage to the lake’s fish populations would be the only harm that a leak of oil from the pipeline
would cause.
E. The species of fish that are present in Lake Konfa now are the same as those that were in the lake
before it was affected by pollution.

30. In the past, every ten-percentage-point increase in cigarette prices in the country of Coponia has
decreased per capita sales of cigarettes by four percent. Coponia is about to raise taxes on
cigarettes by 9 cents per pack. The average price of cigarettes in Coponia is and has been for more
than a year 90 cents per pack. So the tax hike stands an excellent chance of reducing per capita
sales of cigarettes by four percent. Which of the following is an assumption on which the
argument depends?
A. Tobacco companies are unlikely to reduce their profit per pack of cigarettes to avoid an increase in
the cost per pack to consumers in Coponia.
B. Previous increases in cigarette prices in Coponia have generally been due to increases in taxes on
cigarettes.
C. Any decrease in per capita sales of cigarettes in Coponia will result mainly from an increase in the
number of people who quit smoking entirely.
D. At present, the price of a pack of cigarettes in Coponia includes taxes that amount to less than ten
percent of the total selling price.
E. The number of people in Coponia who smoke cigarettes has remained relatively constant for the
past several years.
31. Personnel officer: The exorbitant cost of our health-insurance benefits reflects the high dollar
amount of medical expenses incurred by our employees. Employees who are out of shape, as a
group, have higher doctor bills and longer hospital stays than do their colleagues who are fit.
Therefore, since we must reduce our health-insurance costs, we should offer a rigorous fitness
program of jogging and weight lifting to all employees, and require employees who are out of
shape to participate. The conclusion reached by the personnel officer depends on which of
the following assumptions?
A. A person who is fit would receive a routine physical checkup by a doctor less regularly than would
a person who is out of shape.
B. The medical expenses incurred by employees who are required to participate in the fitness
program would be less than those incurred by employees who are not required to participate.
C. The strenuous activities required of out-of-shape employees by the program would not by
themselves generate medical expenses greater than any reduction achieved by the program.
D. The fitness program would serve more employees who are out of shape than it would employees
who are fit.
E. The employees who participate in the fitness program would be away from work because of illness
less than would the employees who do not participate.

32. The ancient Nubians inhabited an area in which typhus occurred, yet surprisingly few of their
skeletons show the usual evidence of this disease. The skeletons do show deposits of tetracycline,
an antibiotic produced by a bacterium common in Nubian soil. This bacterium can flourish on the
dried grain used for making two staples of the Nubian diet, beer and bread. Thus, tetracycline in
their food probably explains the low incidence of typhus among ancient Nubians. Which of the
following is an assumption on which the argument relies?
A. The tetracycline deposits did not form after the bodies were buried.
B. The diseases other than typhus to which the ancient Nubians were exposed would not be affected
by tetracycline.
C. Typhus is generally fatal.
D. Nubian grain became contaminated with tetracycline-producing bacteria prior to being harvested.
E. Bread and beer were the only foods eaten by the ancient Nubians which could have contained
tetracycline.

Variation of the above question asked:


The ancient Nubians inhabited an area in which typhus occurs, yet surprisingly few of their
skeletons show the usual evidence of this disease. The skeletons do show deposits of tetracycline,
an antibiotic produced by a bacterium common in Nubian soil. This bacterium can flourish on the
dried grain used for making two staples of the Nubian diet, beer and bread. Thus, tetracycline in
their food probably explains the low incidence of typhus among ancient Nubians. Which of the
following is an assumption on which the argument relies?
A. Infectious diseases other than typhus to which the ancient Nubians were exposed are unaffected
by tetracycline.
B. Tetracycline is not rendered ineffective as an antibiotic by exposure to the processes involved in
making bread and beer.
C. Typhus cannot be transmitted by ingesting bread or beer contaminated with the infectious agents
of this disease.
D. Bread and beer were the only items in the diet of the ancient Nubians which could have contained
tetracycline.
E. Typhus is generally fatal.
33. In Brindon County, virtually all of the fasteners—such as nuts, bolts, and screws—used by
workshops and manufacturing firms have for several years been supplied by the Brindon Bolt
Barn, a specialist wholesaler. In recent months many of Brindon County’s workshops and
manufacturing firms have closed down, and no new ones have opened. Therefore, the Brindon Bolt
Barn will undoubtedly show a sharp decline in sales volume and revenue for this year as compared
to last year. The argument depends on assuming which of the following?
A. Last year the Brindon Bolt Barn’s sales volume and revenue were significantly higher than they
had been the previous year.
B. The workshops and manufacturing firms that have remained open have a smaller volume of work
to do this year than they did last year.
C. Soon the Brindon Bolt Barn will no longer be the only significant supplier of fasteners to Brindon
County’s workshops.
D. The Brindon Bolt Barn’s operating expenses have not increased this year.
E. The Brindon Bolt Barn is not a company that gets the great majority of its business from customers
outside Brindon County.

34. Educational Theorist: Recent editorials have called for limits on the amount of homework
assigned to children. They point out that free-time activities play an important role in childhood
development and that large amounts of homework reduce children’s free time, hindering their
development. But the average homework time for a ten year old, for example, is little more than 30
minutes per night. Clearly, therefore, there is no need to impose the limits these editorials are
calling for. Which of the following is an assumption on which the educational theorist’s
argument relies?
A. The free-time activities that ten year olds engage in most are all approximately equally
effective at fostering development
B. Regularly doing homework assignments improves children’s academic performance.
C. Individual teachers are not the best judges of how much homework to assign the children they
teach
D. In most schools, if not all, the homework assignments given are of a length that does not
diverge widely from the average.
E. Free-time activities rarely teach children skills or information that they can use in their
academic work.

35. Political advocacy groups have begun to use information services to disseminate information that
is then accessed by the public via personal computer. Since many groups are thus able to bypass
traditional news sources, whose reporting is selective, and to present their political views directly
to the public, information services present a more balanced picture of the complexities of political
issues than any traditional news source presents. Which of the following is an assumption on
which the argument above depends?
A. Information services are accessible to enough people to ensure that political advocacy groups can
use these services to reach as large a percentage of the public as they could through traditional
news sources.
B. People could get a thorough understanding of a particular political issue by sorting through
information provided by several traditional news sources, each with differing editorial biases.
C. Information on political issues disseminated through information services does not come almost
entirely from advocacy groups that share a single bias.
D. Traditional news sources seldom report the views of political advocacy groups accurately.
E. Most people who get information on political issues from newspapers and other traditional news
sources can readily identify the editorial biases of those sources.
36. Radio stations with radio data system (RDS) technology broadcast special program information
that only radios with an RDS feature can receive. Between 1994 and 1996, the number of RDS
radio stations in Verdland increased from 250 to 600. However, since the number of RDS-
equipped radios in Verdland was about the same in 1996 as in 1994, the number of Verdlanders
receiving the special program information probably did not increase significantly. Which of the
following is an assumption on which the argument depends?
A. Few if any of the RDS radio stations that began broadcasting in Verdland after 1994 broadcast to
people with RDS-equipped radios living in areas not previously reached by RDS stations.
B. In 1996 most Verdlanders who lived within the listening area of an RDS station already had a
radio equipped to receive RDS.
C. Equipping a radio station with RDS technology does not decrease the station’s listening area.
D. In 1996 Verlanders who did not own radios equipped to receive RDS could not receive any
programming from the RDS radio stations that began broadcasting in Verdland after 1994.
E. The RDS radio stations in Verdland in 1996 did not all offer the same type of programming.

37. Excavations of the Roman city of Sepphoris have uncovered numerous detailed mosaics depicting
several readily identifiable animal species: a hare, a partridge, and various Mediterranean fish.
Oddly, most of the species represented did not live in the Sepphoris region when these mosaics
were created. Since identical motifs appear in mosaics found in other Roman cities, however, the
mosaics of Sepphoris were very likely created by traveling artisans from some other part of the
Roman Empire. Which of the following is an assumption on which the argument depends?
A. The Sepphoris mosaics are not composed exclusively of types of stones found naturally in the
Sepphoris area
B. There is no single region to which all the species depicted in the Sepphoris mosaics are native
C. No motifs appear in the Sepphoris mosaics that do not also appear in the mosaics of some other
Roman city
D. All of the animal figures in the Sepphoris mosaics are readily identifiable as representation of
known species
E. There was not a common repertory of mosaic designs with which artisans who lived in various
parts of the Roman empire were familiar

38. The higher the level of certain vitamins and minerals in the bloodstream, the better a person's
lungs function, as measured by the amount of air the person can expel in one second. The lung
function of smokers is significantly worse, on average, than that of nonsmokers. Clearly, therefore,
one way for smokers to improve their lung function is for them to increase their intake of foods
that are rich in these helpful vitamins and minerals. Which of the following is an assumption on
which this argument depends?
A. Smokers are less likely than nonsmokers to have diets that are rich in vitamins and minerals
B. The lung function of smokers whose diet are rich in those vitamins and minerals is generally
better than that of nonsmokers with comparable diets
C. People whose diets are deficient in those vitamins and minerals do not typically have other health
problems in addition to diminished lung function
D. Stopping smoking will not typically improve lung function more than any diet changes can
E. Smoking does not introduce into the body chemicals that prevent the helpful vitamins and
minerals from entering the bloodstream
39. In a study conducted in Pennsylvania, servers in various restaurants wrote “Thank you” on
randomly selected bills before presenting the bills to their customers. Tips on these bills were an
average of three percentage points higher than tips on bills without the message. Therefore, if
servers in Pennsylvania regularly wrote “Thank you” on restaurant bills, their average income
from tips would be significantly higher than it otherwise would have been. Which of the
following is an assumption on which the argument relies?
A. The “Thank you” messages would have the same impact on regular patrons of a restaurant as they
would on occasional patrons of the same restaurant.
B. Regularly seeing “Thank you” written on their bills would not lead restaurant patrons to revert to
their earlier tipping habits.
C. The written “Thank you” reminds restaurant patrons that tips constitute a significant part of the
income of many food servers.
D. The rate at which people tip food servers in Pennsylvania does not vary with how expensive a
restaurant is.
E. Virtually all patrons of the Pennsylvania restaurants in the study who were given a bill with
“Thank you” written on it left a larger tip than they otherwise would have.

40. Generally scientists enter their field with the goal of doing important new research and accept as
their colleagues those with similar motivation. Therefore, when any scientist wins renown as an
expounder of science to general audiences, most other scientists conclude that this popularizer
should no longer be regarded as a true colleague. The explanation offered above for the low
esteem in which scientific popularizers are held by research scientists assumes that
A. serious scientific research is not a solitary activity, but relies on active cooperation among a
group of colleagues
B. research scientists tend not to regard as colleagues those scientists whose renown they envy
C. a scientist can become a famous popularizer without having completed any important research
D. research scientists believe that those who are well known as popularizers of science are not
motivated to do important new research
E. no important new research can be accessible to or accurately assessed by those who are not
themselves scientists

41. The technological conservatism of bicycle manufacturers is a reflection of the kinds of demand
they are trying to meet. The only cyclists seriously interested in innovation and willing to pay for
it are bicycle racers. Therefore, innovation in bicycle technology is limited by what authorities will
accept as standard for purposes of competition in bicycle races. Which of the following is an
assumption made in drawing the conclusion above?
A. The market for cheap, traditional bicycles cannot expand unless the market for high-
performance competition bicycles expands.
B. High-performance bicycles are likely to be improved more as a result of technological
innovations developed in small workshops than as a result of technological innovations
developed in major manufacturing concerns.
C. Bicycle racers do not generate a strong demand for innovations that fall outside what is officially
recognized as standard for purposes of competition.
D. The technological conservatism of bicycle manufacturers results primarily from their desire to
manufacture a product that can be sold without being altered to suit different national markets.
E. The authorities who set standards for high-performance bicycle racing do not keep informed
about innovative bicycle design.
42. Roland: The alarming fact is that 90 percent of the people in this country now report that they
know someone who is unemployed.
Sharon: But a normal, moderate level of unemployment is 5 percent, with 1 out of 20 workers
unemployed. So at any given time if a person knows approximately 50 workers, 1 or more will
very likely be unemployed.
Sharon's argument relies on the assumption that
A. normal levels of unemployment are rarely exceeded
B. unemployment is not normally concentrated in geographically isolated segments of the
population
C. the number of people who each know someone who is unemployed is always higher than 90% of
the population
D. Roland is not consciously distorting the statistics he presents
E. knowledge that a personal acquaintance is unemployed generates more fear of losing one's job
than does knowledge of unemployment statistics

43. Because no employee wants to be associated with bad news in the eyes of a superior, information
about serious problems at lower levels is progressively softened and distorted as it goes up each
step in the management hierarchy. The chief executive is, therefore, less well informed about
problems at lower levels than are his or her subordinates at those levels. The conclusion drawn
above is based on the assumption that
A. problems should be solved at the level in the management hierarchy at which they occur
B. employees should be rewarded for accurately reporting problems to their superiors
C. problem-solving ability is more important at higher levels than it is at lower levels of the
management hierarchy
D. chief executives obtain information about problems at lower levels from no source other than
their subordinates
E. some employees are more concerned about truth than about the way they are perceived by their
superiors

44. The interview is an essential part of a successful hiring program because, with it, job applicants
who have personalities that are unsuited to the requirements of the job will be eliminated from
consideration. The argument above logically depends on which of the following
assumptions?
A. A hiring program will be successful if it includes interviews.
B. The interview is a more important part of a successful hiring program than is the development of
a job description.
C. Interviewers can accurately identify applicants whose personalities are unsuited to the
requirements of the job.
D. The only purpose of an interview is to evaluate whether job applicants' personalities are suited
to the requirements of the job.
E. The fit of job applicants' personalities to the requirements of the job was once the most
important factor in making hiring decisions.
45. Bank depositors in the United States are all financially protected against bank failure because the
government insures all individuals' bank deposits. An economist argues that this insurance is
partly responsible for the high rate of bank failures, since it removes from depositors any financial
incentive to find out whether the bank that holds their money is secure against failure. If
depositors were more selective, then banks would need to be secure in order to compete for
depositors' money. The economist's argument makes which of the following assumptions?
A. Bank failures are caused when big borrowers default on loan repayments.
B. A significant proportion of depositors maintain accounts at several different banks.
C. The more a depositor has to deposit, the more careful he or she tends to be in selecting a bank.
D. The difference in the interest rates paid to depositors by different banks is not a significant
factor in bank failures.
E. Potential depositors are able to determine which banks are secure against failure.

46. To prevent harbor porpoises from getting tangled in its nets and suffocating, a fishing company
installed acoustic alarms on all its boats that fish in waters off Massachusetts. The sound emitted
temporarily disorients the porpoises and frightens them away. Since the installation of the alarms,
the average number of porpoises caught in the company's nets has dropped from eight to one per
month. The alarms, therefore, are saving the lives of harbor porpoises in those waters. Which of
the following is an assumption on which the argument depends?
A. The use of acoustic alarms increases the number of commercial fish caught by the fishing
company's boats.
B. When disoriented, harbor porpoises are not significantly more likely to be killed by other boats.
C. Environmentalists advocate the use of acoustic alarms as a means of protecting the harbor
porpoise population.
D. The alarms were installed at the time of year when harbor porpoises are most plentiful in the
Massachusetts waters.
E. The cost of installing acoustic alarms on fishing boats is less than the cost of repairing nets
damaged by harbor porpoises.

47. A recent report determined that although only three percent of drivers on Maryland highways
equipped their vehicles with radar detectors, thirty-three percent of all vehicles ticketed for
exceeding the speed limit were equipped with them. Clearly, drivers who equip their vehicles with
radar detectors are more likely to exceed the speed limit regularly than are drivers who do not.
The conclusion drawn above depends on which of the following assumptions?
A. Drivers who equip their vehicles with radar detectors are less likely to be ticketed for exceeding
the speed limit than are drivers who do not.
B. Drivers who are ticketed for exceeding the speed limit are more likely to exceed the speed limit
regularly than are drivers who are not ticketed.
C. The number of vehicles that were ticketed for exceeding the speed limit was greater than the
number of vehicles that were equipped with radar detectors.
D. Many of the vehicles that were ticketed for exceeding the speed limit were ticketed more than
once in the time period covered by the report.
E. Drivers on Maryland highways exceeded the speed limit more often than did drivers on other
state highways not covered in the report.
48. Researchers have found that when very overweight people, who tend to have relatively low
metabolic rates, lose weight primarily through dieting, their metabolisms generally remain
unchanged. They will thus burn significantly fewer calories at the new weight than do people
whose weight is normally at that level. Such newly thin persons will, therefore, ultimately regain
weight until their body size again matches their metabolic rate. The conclusion of the argument
above depends on which of the following assumptions?
A. Relatively few very overweight people who have dieted down to a new weight tend to continue
to consume substantially fewer calories than do people whose normal weight is at that level.
B. The metabolisms of people who are usually not overweight are much more able to vary than the
metabolisms of people who have been very overweight.
C. The amount of calories that a person usually burns in a day is determined more by the amount
that is consumed that day than by the current weight of the individual.
D. Researchers have not yet determined whether the metabolic rates of formerly very overweight
individuals can be accelerated by means of chemical agents.
E. Because of the constancy of their metabolic rates, people who are at their usual weight normally
have as much difficulty gaining weight as they do losing it.

49. Traditionally, decision-making by managers that is reasoned step-by-step has been considered
preferable to intuitive decision-making. However, a recent study found that top managers used
intuition significantly more than did most middle- or lower-level managers. This confirms the
alternative view that intuition is actually more effective than careful, methodical reasoning. The
conclusion above is based on which of the following assumptions?
A. Methodical, step-by-step reasoning is inappropriate for making many real-life management
decisions.
B. Top managers have the ability to use either intuitive reasoning or methodical, step-by-step
reasoning in making decisions.
C. The decisions made by middle- and lower-level managers can be made as easily by using
methodical reasoning as by using intuitive reasoning.
D. Top managers use intuitive reasoning in making the majority of their decisions.
E. Top managers are more effective at decision-making than middle- or lower-level managers.

50. Kitchen magazine plans to license the use of its name by a line of cookware. For a magazine,
licensing the use of its name for products involves some danger, since if the products disappoint
consumers, the magazine's reputation suffers, with consequent reductions in circulation and
advertising. However, experts have evaluated the cookware and found it superior to all other
cookware advertised in Kitchen. Therefore, Kitchen can collect its licensing fee without
endangering its other revenues. The argument above assumes which of the following?
A. No other line of cookware is superior to that which will carry the Kitchen name.
B. Kitchen will not license the use of its name for any products other than the line of cookware.
C. Makers of cookware will not find Kitchen a less attractive advertising vehicle because the
magazine's name is associated with a competing product.
D. Consumers who are not regular readers of Kitchen magazine will be attracted to the cookware by
the Kitchen name.
E. Kitchen is one of the most prestigious cooking-related magazines.
51. The general availability of high-quality electronic scanners and color printers for computers has
made the counterfeiting of checks much easier. In order to deter such counterfeiting, several
banks plan to issue to their corporate customers checks that contain dots too small to be
accurately duplicated by any electronic scanner currently available; when such checks are scanned
and printed, the dots seem to blend together in such a way that the word “VOID” appears on the
check. A questionable assumption of the plan is that
A. in the territory served by the banks the proportion of counterfeit checks that are made using
electronic scanners has remained approximately constant over the past few years
B. most counterfeiters who use electronic scanners counterfeit checks only for relatively large
amounts of money
C. the smallest dots on the proposed checks cannot be distinguished visually except under strong
magnification
D. most corporations served by these banks will not have to pay more for the new checks than for
traditional checks
E. the size of the smallest dots that generally available electronic scanners are able to reproduce
accurately will not decrease significantly in the near future
Part 2

1. To decrease the number of crimes in city Y, the city's Police Commissioner proposed taking
some police officers from low-crime districts of the city and moving them to high-crime
districts of the city. His proposal is based on city Y crime data that show that the number of
crimes in any district of the city decreases when additional police officers are moved into that
district. The Police Commissioner's proposal depends on which of the following assumptions?
 City X experienced a drastic reduction in crime after implementing a proposal similar to that
proposed by the Police Commissioner of city Y.
 The severity of crimes committed in any district of the city decreases when additional police
officers are moved into that district.
 The number of crimes committed in all high-crime districts of city Y is more than triple the
number of crimes committed in all low-crime districts of city Y.
 There are more low-crime districts than high-crime districts in city Y.
 Districts of the city from which police officers are removed do not experience significant crime
increases shortly after the removal of those officers.

2. Although there has been great scientific debate for decades over global warming, most
scientists now agree that human activity is causing the Earth’s temperature to rise. Though
predictions vary, many global warming experts believe that average global temperatures will
rise between three and eight degrees Fahrenheit during the next century. Such an increase
would cause an alarming rise in sea levels, displacing millions of people by destroying major
population centers along the world’s coastlines. Which of the following is an assumption in
support of the argument’s conclusion?
 New technological developments in the next century will not divert rising seas from the
world’s coastal cities.
 Individuals will not become more aware of the steps they can take to reduce the emission of
greenhouse gases.
 Rising sea levels similarly affect all coastal population centers.
 Some global warming experts predict a greater than eight degree Fahrenheit increase in global
temperatures during the next century.
 Human activity is the sole cause of increasing global temperatures.

3. "The new HitItFar driver is the only golf club that uses space-age Titanium-Kryptonium alloy to
strengthen its head. This driver is SO good, the last 12 winners of the major tour
championships have all recently switched to it! Isn't it time for you to add power to your swing
and distance to your drives? Trade in your old driver today -- and HitItFar tomorrow!" All of
the following claims are either implied or made explicitly in the above advertisement EXCEPT:
 Switching to the HitItFar driver will improve your play.
 The HitItFar driver helped the last 12 major championship winners achieve their victory.
 Major championship winners are experts and know what constitutes a great golf club.
 Your existing driver is inferior to the HitItFar driver.
 Only HitItFar golf clubs have Titanium-Kryptonium alloy in the head of their drivers.
4. Researchers studying the spread of the Black Plague in sixteenth-century England claim that
certain people survived the epidemic because they carried a genetic mutation, known as Delta-
32, that is known to prevent the bacteria that causes the Plague from overtaking the immune
system. To support this hypothesis, the researchers tested the direct descendants of the
residents of an English town where an unusually large proportion of people survived the
Plague. More than half of these descendants tested positive for the mutation Delta-32, a figure
nearly three times higher than that found in other locations. The researchers' hypothesis is
based on which of the following assumptions?
 Delta-32 does not prevent a carrier from contracting any disease other than the Plague.
 The Plague is not similar to other diseases caused by bacteria.
 Delta-32 did not exist in its current form until the sixteenth century.
 No one who tested positive for Delta-32 has ever contracted a disease caused by bacteria.
 The Plague does not cause genetic mutations such as Delta-32.

5. The popular notion that a tree's age can be determined by counting the number of internal
rings in its trunk is generally true. However, to help regulate the internal temperature of the
tree, the outermost layers of wood of the Brazilian ash often peel away when the temperature
exceeds 95 degrees Fahrenheit, leaving the tree with fewer rings than it would otherwise have.
So only if the temperature in the Brazilian ash's environment never exceeds 95 degrees
Fahrenheit will its rings be a reliable measure of the tree's age. Which of the following is an
assumption on which the argument above depends?
 The growth of new rings in a tree is not a function of levels of precipitation.
 Only the Brazilian ash loses rings because of excessive heat.
 Only one day of temperatures above 95 degrees Fahrenheit is needed to cause the Brazilian ash
to lose a ring.
 The internal rings of all trees are of uniform thickness.
 The number of rings that will be lost when the temperature exceeds 95 degrees Fahrenheit is
not predictable.

6. The Department of Homeland Security has proposed new federal requirements for driver’s
licenses that would allow them to be used as part of a national identification system. Using
licenses for purposes not directly related to operating a motor vehicle is un-American because
it would require U.S. citizens to carry the equivalent of “papers.” Such a requirement would
allow the government to restrict their movements and activities in the manner of totalitarian
regimes. In time, this could make other limits on freedom acceptable. The author assumes which
of the following?
 The next presidential election will be dishonest, as has happened in eastern European
countries.
 The government will soon start curtailing the activities of those it considers “dissidents.”
 Blanket restrictions on law-abiding individuals are contrary to the traditions of American
culture and law.
 The majority of Americans are not willing to give up their right to travel and move about
without identification.
 Americans should resist all government regulation of their lives.
7. Since the new publisher took control, a news magazine’s covers have featured only models and
movie stars. Previously, the covers had displayed only politicians, soldiers, and business
leaders. A leading gossip columnist claimed that the changes made the magazine relevant
again. However, many newspaper editorials disagreed and suggested that the new publisher is
more interested in boosting sales than in reporting important news events. Which of the
following is an assumption necessary for the argument made by the gossip columnist’s opponents?
 The charitable activities of models and movie stars often focus public attention on pressing
problems.
 Final authority for choosing the cover subject of the magazine lies with the publisher.
 A magazine can boost sales while highlighting the coverage of important world leaders.
 Some of the movie stars featured are now running for political office.
 Magazine issues with models or movie stars on the covers are purchased at a rate more than
three times greater than is the case with issues featuring politicians on the covers.

8. In response to the increasing cost of producing energy through traditional means, such as
combustion, many utility companies have begun investing in renewable energy sources, chiefly
wind and solar power, hoping someday to rely on them completely and thus lower energy
costs. The utility companies claim that although these sources require significant initial capital
investment, they will provide stable energy supplies at low cost. As a result, these sources will
be less risky for the utilities than nonrenewable sources, such as gas, oil, and coal, whose prices
can fluctuate dramatically according to availability. The claim of the utility companies
presupposes which of the following?
 The public will embrace the development of wind and solar power.
 No new deposits of gas, oil, and coal will be discovered in the near future.
 Weather patterns are consistent and predictable.
 The necessary technology for conversion to wind and solar power is not more expensive than
the technology needed to create energy through combustion.
 Obtaining energy from nonrenewable sources, such as gas, oil and coal, cannot be made less
risky.

9. Surveys consistently show that the best-selling ice cream flavor is vanilla, although those who
prefer chocolate rarely order vanilla. Vanilla-flavored candy, then, probably sells better than
chocolate-flavored candy. Which of the following is an assumption upon which the author of the
argument relies?
 Because someone prefers vanilla ice cream does not mean he prefers vanilla-flavored candy.
 Children who prefer vanilla ice cream also tend to like chocolate candy.
 Those who prefer neither vanilla nor chocolate ice cream also prefer other flavors of candy.
 Someone who prefers vanilla ice cream may still order chocolate on occasion.
 Preferences for certain ice cream flavors are similar to preferences for candy flavors.

10. The media claim that the economy is entering a phase of growth and prosperity. They point to
lower unemployment rates and increased productivity. This analysis is false, though. The
number of people filing for bankruptcy has increased every month for the last six months, and
bankruptcy lawyers report that they are busier than they have been in years. Which of the
following is an assumption on which the argument depends?
 Unemployment rates are not useful indicators of growth and prosperity.
 Economic growth cannot be measured in terms of productivity.
 Legislation has not been recently passed to make legal bankruptcy easier to obtain.
 There has not been an increase in the number of bankruptcy lawyers.
 The media often misrepresent the current state of economic affairs.
11. A newly discovered painting on wooden panel by Michelangelo must have been completed
after 1507 but before 1509. It cannot have been painted earlier than 1507 because one of its
central figures carries a coin that was not minted until that year. It cannot have been painted
after 1509 because it contains a pigment that Michelangelo is known to have abandoned when
a cheaper alternative became available in that year. Which of the following is an assumption on
which the argument depends?
 No stocks of the abandoned pigment existed after 1509.
 Michelangelo did not work on the painting over the course of several years.
 The coin depicted in the painting was known to general public in 1507.
 The wooden panel on which the painting was executed cannot be tested accurately for age.
 Michelangelo's painting style did not change between 1507 and 1509.

12. In order to save money, some of Company X's manufacturing plants converted from oil fuel to
natural gas last year, when the cost of oil was more than the cost of natural gas. Because of a
sudden, unexpected shortage, however, natural gas now costs more than oil, the price of which
has fallen steeply over the past year. The cost of conversion back to oil would more than negate
any cost savings in fuel. So Company X's fuel costs this year will be significantly higher than
they were last year. Which of the following is an assumption on which the argument above
depends?
 Company X does not have money set aside for the increased costs of fuel.
 The increase in the cost of fuel cannot be offset by reductions in other operating expenses.
 The price of natural gas will never again fall below that of oil.
 The cost of fuel needed by those of Company X's plants that converted to natural gas is not less
than the cost of fuel needed by those plants still using oil.
 The price of oil will not experience a sudden and steep increase.

13. Advertisement: Avian Oculars are the newest in binocular technology for the professional bird
watcher. For a price comparable to that of traditional binoculars, Avian Oculars are specially
designed with the features demanded by birding enthusiasts: they are lightweight, compact
and extremely durable. So, use Avian Oculars on your next bird watching excursion, and see
some of the world’s rarest species in all of their beautiful plumage. Which of the following is an
assumption on which the author of the advertisement relies?
 Avian Oculars weigh less than traditional binoculars.
 Avian Oculars should be used only by bird watchers.
 The reader will travel through the habitats of rare species during his or her next bird watching
trip.
 Avian Oculars are similar in cost to traditional binoculars.
 Birding enthusiasts often determine the specifications of new products.

14. Every year many people become ill because of airborne mold spores in their homes. After
someone becomes ill, specialists are often hired to eradicate the mold. These specialists look in
damp areas of the house, since mold is almost always found in places where there is substantial
moisture. If one wishes to avoid mold poisoning, then, one should make sure to keep all
internal plumbing in good condition to prevent leakage that could serve as a breeding ground
for mold. Which of the following is an assumption on which the argument depends?
 Mold itself does not create moisture.
 Most homeowners know enough about plumbing to determine whether theirs is in good
condition.
 Mold cannot grow in dry areas.
 No varieties of mold are harmless.
 Mold spores cannot be filtered from the air.
15. The legislature of the Philippines voted recently to abolish the death penalty. In contrast, the
death penalty remains legal in the United States. The difference in the legality of capital
punishment demonstrates that the majority of American citizens believe in the death penalty,
while the majority of Filipino citizens do not. Which of the following is an assumption upon
which the above argument depends?
 There are more murders per year in the United States than in the Philippines.
 The legal status of capital punishment in the United States and the Philippines aligns with how
the majority of citizens in those respective countries view the death penalty.
 There are not strong voices opposing the death penalty in the United States.
 Most American citizens who believe in the death penalty think that it acts as a deterrent for
potential criminals, while most Filipino citizens do not.
 The legal standard used to determine whether a criminal should be sentenced to the death
penalty in the United States is similar to the legal standard used in the Philippines before
capital punishment was abolished there.

16. Smoking is a known cause of certain serious health problems, including emphysema and lung
cancer. Now, an additional concern can be added to the list of maladies caused by smoking. A
recent study surveyed both smokers and nonsmokers, and found that smokers are significantly
more anxious and nervous than nonsmokers. Which of the following is an assumption on which
the argument rests?
 Anxiety and nervousness can lead to serious health problems.
 Anxiety and nervousness do not make individuals more likely to start smoking.
 Equivalent numbers of smokers and nonsmokers were surveyed for the study.
 Smokers are aware of the various health problems attributed to smoking, including lung cancer
and emphysema.
 Smokers who had smoked a cigarette immediately before responding to the survey were more
anxious and nervous than smokers who had not smoked for several hours.

17. Over the past 5 years, Company X has posted double-digit growth in annual revenues,
combined with a substantial improvement in operating margins. Since this growth is likely to
persist in the future, the stock of Company X will soon experience dramatic appreciation. The
argument above is based on which of the following assumptions?
 Company X has a large market share in its industry.
 Prior to the last 5 years, Company X had experienced similarly dramatic growth in sales
associated with stable or improving operating margins.
 The growth of Company X is likely to persist in the future.
 The current price of the stock of Company X does not fully reflect the promising growth
prospects of the firm.
 The stock of Company X will outperform other stocks in the same industry.
18. Antoine: The alarming fact is that among children aged 19 years and younger, the number
taking antipsychotic medicines soared 73 percent in the last four years. That is greater than the
increase in the number of adults taking antipsychotic medicines during the same period.
Lucy: But the use of antipsychotic drugs by adults is considered normal at the current rate of
11 adults per 1,000 taking the drugs. In contrast, the number of children on antipsychotic
medication last year was 6.6 per 1,000 children. Lucy’s argument relies on the assumption that
______.
 normal levels of antipsychotic drug use are rarely exceeded.
 the percentage of adults taking antipsychotic medication is always higher than the percentage
of children on such medication.
 the use of antipsychotic medication in children is no different from the use of such medications
in adults.
 Antoine is not consciously distorting the statistics he presents.
 a rapid increase in the number of children taking antipsychotic drugs generates more fear of
random violence by adolescents than does knowledge of the absolute number of children on
such medications.

19. A recent article stated that only 5.5% of American colleges grant the majority of their degrees
in the liberal arts. Citing this, a reader wrote to lament that this was further evidence of the
decline of academic rigor in American post high school education. Which of the following is an
assumption on the part of the reader?
 The percentage of American colleges granting liberal arts degrees would continue to drop.
 All colleges should grant the majority of their degrees in the liberal arts.
 Most post-secondary scientific, engineering, and vocational training does not involve as much
academic rigor as liberal arts training.
 Academic rigor is the most important aspect of post high school education.
 Of the colleges that do not grant the majority of their degrees in the liberal arts, many granted
fewer than a quarter of their degrees in the liberal arts.

20. Doctor: Research shows that adolescents who play video games on a regular basis are three
times as likely to develop carpal tunnel syndrome as are adolescents who do not play video
games. Federal legislation that prohibits the sale of video games to minors would help curb this
painful wrist condition among adolescents. The doctor’s conclusion depends on which of the
following assumptions?
 The majority of federal legislators would vote for a bill that prohibits the sale of video games to
minors.
 Not all adolescents who play video games on a regular basis suffer from carpal tunnel
syndrome.
 Playing video games is the only way an adolescent can develop carpal tunnel syndrome.
 Most parents would refuse to purchase video games for their adolescent children.
 The regular playing of video games by adolescents does not produce such beneficial effects as
better hand-eye coordination and improved reaction time.
21. Company X receives most of its revenues from the sale of gasoline through a network of gas
stations that it owns across the country. The company purchases ready-for-sale gasoline from
several oil refineries at wholesale prices and sells it to the final consumer at its gas stations.
Over the next quarter, the management of Company X expects that the market price of gasoline
will rise by approximately 10 percent. Therefore, the management projects that the next
quarter’s revenues from the sale of gasoline will also increase by approximately 10 percent.
The management's projection is based on which of the following assumptions?
 Consumption of gasoline at the company’s gas stations will not drop in response to higher
prices.
 Company profits will not decline below their current level.
 Higher gasoline prices will not reduce the company’s revenues from other business lines.
 The costs of gasoline purchased by the company for subsequent sale at its gas stations will
remain relatively constant.
 The supply of gasoline is likely to decline over the next quarter.

22. An oil field prospector and developer reported a large oil deposit in southwestern Texas. As a
result, a large oil and gas company purchased the field with the intention of drilling oil wells in
the area soon afterwards. However, the company found that what had been reported to be a
large oil deposit was actually much smaller than had been indicated. Thus, the methods that
the prospector had used to determine the size of the oil deposit must have been inaccurate.
Which of the following is an assumption on which the argument depends?
 The company's methods of measuring the size of the oil deposit were determined by a third
party to be more accurate than those used by the prospector.
 The prospector did not purposefully fabricate or misrepresent the size of the oil deposit.
 Though smaller than originally thought, the oil deposit contained enough oil to make drilling
commercially feasible.
 The prospector did not explore other oil fields and use the same methods to determine the
magnitude of the oil present, if any.
 The company had successfully drilled for oil in other large oil fields in Texas throughout the
early twentieth century.

23. Parent: The city education department is unable to distinguish between annoyances and
important problems. For instance, prohibiting students from having cell phones is an
overreaction. If a student uses one and thus interferes with instruction, confiscate it. All in all,
we need educational leadership that can solve problems, not create them. Which of the
following is an assumption made by the parent?
 Students having cell phones does not constitute an important problem for the city schools.
 Students have no need for cell phones in school.
 Faculty and staff should be allowed to possess cell phones.
 Students need to have cell phones because some of them have no stay-at-home parent.
 An interest in solving problems is the most important attribute of an educational leader.
24. The Ergonomic Society conducted a study that indicated that many people develop severe back
problems during adulthood, and that virtually all such people who received chiropractic
treatment showed great improvement. Therefore, in order to minimize the proportion of the
population that suffers from back pain, the Ergonomic Society recommended that chiropractic
treatment be directed toward those adults who suffer from severe back problems. Which of the
following is an assumption on which the argument depends?
 Any person who receives chiropractic treatment for back pain may also benefit from other
forms of treatment.
 Large insurance carriers cover chiropractic care for back problems to a lesser degree than they
do other medical treatments.
 Individuals who receive chiropractic or other treatment prior to developing severe back
problems are not less likely to develop back pain than those who do not.
 Chiropractic treatment is more effective in treating severe back problems when utilized over a
long period of time, as opposed to sporadically.
 Severe back pain and other problems often cause individuals to miss workdays.

25. Since the new publisher took control, a news magazine’s covers have featured only models and
movie stars. Previously, the covers had displayed only politicians, soldiers, and business
leaders. A leading gossip columnist claimed that the changes made the magazine relevant
again. However, many newspaper editorials disagreed and suggested that the new publisher is
more interested in boosting sales than in reporting important news events. Which of the
following is an assumption necessary for the argument made by the gossip columnist’s opponents?
 The charitable activities of models and movie stars often focus public attention on pressing
problems.
 Final authority for choosing the cover subject of the magazine lies with the publisher.
 A magazine can boost sales while highlighting the coverage of important world leaders.
 Some of the movie stars featured are now running for political office.
 Magazine issues with models or movie stars on the covers are purchased at a rate more than
three times greater than is the case with issues featuring politicians on the covers.

26. For-profit colleges serve far fewer students than either public or private non-profit colleges. At
the same time, relative to non-profit colleges, for-profit colleges draw a disproportionate share
of federal and state financial aid, such as tuition grants and guaranteed loans, for their
students. It must be, then, that for-profit colleges enroll a greater proportion of financially
disadvantaged students than do non-profit colleges. The conclusion above depends on which of
the following assumptions?
 Public non-profit colleges and private non-profit colleges enroll a similar proportion of
financially disadvantaged students.
 For-profit colleges do not engage in fraudulent practices in helping their students obtain
unneeded federal and state financial aid.
 The number of students receiving federal and state financial aid at for-profit colleges is greater
than the number of students receiving federal and state financial aid at non-profit colleges.
 For-profit colleges are of similar educational quality as non-profit colleges.
 The majority of students at for-profit colleges do not default on repayment of their loans after
they complete college.
27. Recent research has indicated that married people are not only happier than unmarried
people, but also healthier. This study has been widely reported by the media, with most
commentators concluding that being married is good for one’s health and attitude. The
conclusion of the media commentators depends on which of the following assumptions?
 The longer people are married, the happier and healthier they become.
 Married couples who had a large, extravagant wedding are happier than those who had a small,
simple ceremony.
 Married people cannot get depressed.
 Single people with depression or health problems are just as likely to get married as are other
single people.
 Some marriages are more harmonious than others.

28. Country X imposes heavy tariffs on imported manufactured goods. Company Y has determined
that it could increase its profits in the long term by opening a factory in Country X to
manufacture the goods that it currently produces in its home country for sale in Country X. For
Company Y's determination to be true, which of the following assumptions must also be true?
 Company Y will be able to obtain all the necessary permits to open a factory in Country X.
 Company Y currently produces no goods outside its home country.
 A sustainable market for Company Y's goods currently exists in Country X.
 Company Y's home country does not impose tariffs on imported goods.
 Labor costs in Country X are lower than those in Company Y's home country.

29. Country X imposes heavy tariffs on imported manufactured goods. Company Y has determined
that it could increase its profits in the long term by opening a factory in Country X to
manufacture the goods that it currently produces in its home country for sale in Country X. For
Company Y's determination to be true, which of the following assumptions must also be true?
 Company Y will be able to obtain all the necessary permits to open a factory in Country X.
 Company Y currently produces no goods outside its home country.
 A sustainable market for Company Y's goods currently exists in Country X.
 Company Y's home country does not impose tariffs on imported goods.
 Labor costs in Country X are lower than those in Company Y's home country.

30. When a company refuses to allow other companies to produce patented technology, the
consumer invariably loses. The company that holds the patent can charge exorbitant prices
because there is no direct competition. When the patent expires, other companies are free to
manufacture the technology and prices fall. Companies should therefore allow other
manufacturers to license patented technology. The argument above presupposes which of the
following?
 Companies cannot find legal ways to produce technology similar to patented technology.
 Companies have an obligation to act in the best interest of the consumer.
 Too many patents are granted to companies that are unwilling to share them.
 The consumer can tell the difference between patented technology and inferior imitations.
 Consumers care more about price than about quality.
31. Inorganic pesticides remain active on the surfaces of fruits and vegetables for several days
after spraying, while organic pesticides dissipate within a few hours after application, leaving
the surface of the sprayed produce free of pesticide residue. Therefore, when purchasing from
a farm that uses inorganic pesticides, one must be careful to wash the produce thoroughly
before eating it to prevent the ingestion of toxins. But one need not worry about ingesting
pesticides when purchasing from farms that use only organic pesticides. The argument above
assumes that
 Consumers are aware of the origins of the produce they purchase.
 Produce from farms that use organic pesticides reaches the consumer within hours after it is
picked or harvested.
 No farm uses both organic and inorganic pesticides.
 No pesticide is capable of penetrating the skin of a fruit or vegetable.
 The use of either type of pesticide does not increase the cost of produce.

32. The downturn in the economy last year has prompted many companies to make widely
publicized layoffs, resulting in thousands of lost jobs. Economists predicted that these layoffs
would cause people generally to cut back on their discretionary spending, even if their jobs
were secure, in anticipation of coming hard times. However, this prediction has not come to
pass, since there has been no increase in the amount of money set aside by the general public
in savings accounts. Which one of the following is an assumption on which the argument
depends?
 The economy has not improved in recent months.
 There has been no increase in the amount of money invested in stocks, certificates of deposit,
or other savings vehicles.
 Salaries have decreased as a result of the economic downturn.
 No business sectors have seen growth in recent months.
 Those who were laid off have been able to find other employment.

33. Because most hospitals suffer a chronic undersupply of physicians, patients must sometimes
wait hours in the emergency room to see a doctor. Nurses should therefore perform initial
examinations in hospital emergency rooms to determine which patients merit immediate
treatment and which can wait until the emergency physicians have more time to see them.
Which of the following is an assumption on which the argument above is based?
• Hospitals should expand their medical staffs.
• Physicians cannot be trained to perform initial examinations themselves.
• Emergency rooms will run more smoothly if initial examinations are performed.
• Hospitals are always fully staffed with nurses.
• Nurses are competent to judge the severity of patients' conditions.

34. Scientists have discovered a new species of butterfly that lives only in a small region of Central
America and is active only at night. During the day, it rests in treetops, where its green color
matches the foliage perfectly. Therefore, the scientists must have discovered the butterfly at
night. The argument depends on which of the following assumptions?
 The newly discovered butterfly is not related to any other species of butterfly.
 There is no way for the scientists to detect the butterfly during the day.
 No other butterfly species lives in this region of Central America.
 The foliage in the butterfly's habitat is completely green.
 The butterfly cannot survive in areas outside of Central America.
Assumptions Latest Questions

1. We know the following:


a)Parents are reluctant to subject children to the pain of injections
b)adults, who are at risk of serious complications from influenza, are commonly vaccinated

c)A new influenza vaccine, administered painlessly in a nasal spray, is effective for children

From the above the author concludes that since nasal spray can be used effectively only on children and
since children do not develop any serious complications from influenza, the widespread use of nasal
spray vaccine does not have any serious health benefit.

This conclusion will hold true ONLY IF we can be sure that adults do not contract influenza from children.
Only then will there be no significant health benefit in using the nasal spray.

A. Irrelevant. We know that this vaccine is effective only on children. Therefore even if this weren't true, it
would not make any difference to the conclusion.

B. Irrelevant. Once again, assumptions about the working of the vaccine is irrelevant to the conclusion.

C. Irrelevant. We are discussing the benefits of using nasal spray vaccine. The price of the injectable
vaccine does not concern us.

D. CORRECT. Let us try the negation of this. Adults contract influenza primarily from children than have
influenza. IF the negation were true,then the conclusion would become incorrect as there would then be a
huge benefit to suing nasal sprays for kids.

E. We already know this to be true in the passage. “A new influenza vaccine, administered painlessly in a
nasal spray, is effective for children.” Therefore this becomes a fact and not an assumption.

2.
Goal This Year: To reduce the number of truckloads of refuse(garbage) to be incinerated to half of last
year's number.

Which of the options are essential to achieve this goal?

Before we begin, let us say 1 ton of ash was generated per truckload of refuse(Garbage) last year.

A. This does not matter. As long sufficient refuse has been separated for recycling to ensure the amount
of ash generated is half of last year, it does not matter if some materials that could be recycled are incin-
erated.

B. The cost of the process is irrelevant to the conclusion.

C. Please note, we only need to seperate enough refuse for recycling to reduce the number of truckloads
of refuse to be incinerated to half of last year's number. So if 60% percent of the refuse collected last year
was recyclable and only 55% of the refuse collected this year can be recycled, our goal can still be
reached.
D. CORRECT. The amount of refuse that will be recycled is being decided based on the amount of ash
generated last year. Once the separation has been made, if the non-recyclable refuse produces 2 tons of
ash per truckload(say) then our goal will not be met despite recycling
last year 1 truck load of residue generated 1 ton of ash. This year (after u pick out recyclable waste), 1
truck-load generates 2 or 3 or even more tons of residue ash, then the city will not achieve its objective.

E. As long as we are able to separate out enough refuse to recycle it, this will not affect the goal. It is
possible to have waste in excess of last year and still achieve our goal.

3.

Governor's Goal: to reduce crime rate


Governor's Plan: be harsher with the inmates by denying them access to college-level courses.
Argument Against the Plan: the people who took college level courses in prison committed fewer crimes
once they were released from prison. So Governor's plan will backfire.

Assumption : it is clear that the argument assumes the prisoners committed fewer crimes BECAUSE they
took college level courses in prison. This argument would crumble if we were told that the prisoners who
are less likely to commit crimes are the ones who opt for the college-level courses.

A. Incorrect. If this were true, then the governor's plan for removing college-level courses would not have
met with any resistance.

B. Irrelevant .The argument discusses whether inmates who have taken the courses are less likely to
commit a crime than inmates who have not taken these courses. The comparison is not with the general
population.

C. CORRECT. Let us negate this.


“ The group of inmates who chose to take college-level courses are already less likely than other inmates
to commit crimes after being released”
If the above is true, then implementing the governor's plan will not have any counter effect,as the prison-
ers taking up the college level courses that are being scrapped are already less likely to commit crimes.
Therefore the necessary assumption we need to make for the argument to hold is that all prisoners have
an equal tendency to commit a crime before they take on the college level courses.

D. Irrelevant, We are not discussing High school courses here.

E. Irrelevant to the argument.

4.

Conclusion- the current average height of Tufe’s Turfil sunflowers is undoubtedly at least partially attribut-
able to changes in Tufe’s environmental conditions

This means that the changes in the newly formed island's climate caused the sunflower plants to get
shorter.
Assumption: Here we are assuming the sunflowers on Tufe's island are affected in comparison with the
sunflowers on the mainland(kept as a standard for comparison).

What if the sunflowers on the mainland are in fact different from the original species (because of the
weather conditions on the mainland) while the sunflowers on Tufe have grown to the normal height( and
therefore are unaffected by the climate on Tufe island)?

A. Irrelevant. We are not discussing if the weather is beneficial. We want to know if it affects the natural
growth of Turfil sunflowers.

B. Irrelevant. Once again, we are wondering about the height of sunflowers and not about the numbers.

C. CORRECT. Let us negate this.

“The mainland’s environment has changed in ways that have resulted in Turfil sunflowers on the mainland
growing to be 40 centimeters taller than they did two centuries ago”.
This means the climate on the mainland changed too. How will the conclusion or evidence hold true in
that case? Mainland could have become wetter which made the plants on the main island grow more and
not the ones on the island grow less.

D. Irrelevant. We want to know if the weather conditions were “partially” responsible or not.

E. Irrelevant. This does not tell as anything new. Even if this were the only difference,this option neither
strengthens nor weakens the conclusion.

5.
this is a good problem to which to apply the REVERSAL METHOD for assumptions:

* REVERSE the assumption

* the argument should be DESTROYED

the argument assumes that not as many televisions are being made domestically in borodia. (this is why
they will have to import more.)

if we reverse (c), then the assemblers can put together TV's faster than before.

this means that FEWER of them will be able to make the SAME TOTAL NUMBER of televisions.

this severs the connection between the drop in the number of assemblers and the drop in the number of
televisions produced, so the argument falls apart.

(c) is CORRECT.

6.
Fact 1: the quantity of rice produced per year is currently just large enough to satisfy domestic demand.
Fact 2. Teruvia’s total rice acreage will not be expanded in the foreseeable future nor will rice yields per
acre increase appreciably
Fact 3: Teruvia’s population, however, will be increasing significantly for years to come

Conclusion: Teruvia will soon have to begin importing rice.

Now since the production of rice is not going to increase and the population will continue to rise, it only
makes sense that Teruvia will soon run out of rice UNLESS as the population increases the people begin
to consume less rice.

A. CORRECT. Negate this

“There is a pronounced trend of decreasing per capita demand for rice in Teruvia”
this would automatically destroy our conclusion and is therefore the assumption on which the argument
depends.

B. Irrelevant. Since this is already the case, the amount of rice produced wont decrease and hence wont
affect out conclusion.

C. Irrelevant. We know that the yield will remain constant.

D. We are concerned with the general population growth and not region wise growth.

E. Irrelevant.

7.
Conclusion: If these trends in fuel production and usage continue, therefore,

Sidurian reliance on foreign sources for fuel should decline soon.

Assumption: the only way the above conclusion can be true if is the rate of fuel consumption is either less
than or equal to the rate of fuel production in Siduria. If the fuel production increased as stated in the pas-
sage but the rate of consumption were to exceed the rate of production, Siduria will still have to import
fuel.

The only option that addresses this is A. A is CORRECT.

8.
Conclusion: In this case, therefore, since none of the seven small companies can afford to convert their
production lines to a new set of manufacturing specifications, only the three large companies will be able
to remain in business.

Assumption: We are told clearly that none of the small companies can afford the cost of conversion and
therefore WILL have to go out of business. This is a fact and cannot be disputed. Is there any other way
these small companies could stay in business? What if either one or all the seven small companies were
already following the standards the government plans to implement?

A. Irrelevant. If conclusion discusses the state of the companies once the products are regulated. What
will happen to the large/small companies if the products are not regulated will not affect the conclusion in
anyway.

B. Irrelevant. We already know the small companies are unable to absorb the cost of conversion. Wheth-
er this cost is greater for smaller companies is irrelevant to our conclusion.

C. Irrelevant.

D. We are discussing costs and not complexity.

E. CORRECT. Let us negate this:


All/few of the seven small companies currently manufactures the product to a set of specifications that
would match those the government would institute if the industry were to be regulated.
This means that for all/few companies there will be zero cost of conversion and they can continue to stay
in business. Without the assumption that none of the smaller companies currently follow government
specification,the conclusion will not hold good.

9.
A. CORRECT. Let us negate the option
Let us assume that every time a job has to be reworked, the company sends in more reliable and comep-
tent mechanics. Then the reason that rework is successful is because of the increased skill of the me-
chanic rather than complex nature of the job.

B. Irrelevant. We are discussing if complex jobs require rework due to the nature of the job or other exter-
nal factors. This option is irrelevant to the conclusion.

C. We already know this statement to be untrue. Passage states:


“Ace Repairs have to be reworked under the company’s warranty”

D. The negation of this option would help negate the conclusion ONLY if the passage stated that the
same mechanics who were penalised were sent for the rework. Since we do not know this to be true, this
option is incorrect.

E. Irrelevant We are only discussing complex jobs that have required repair. Even though there exists a
category of jobs which carries out first time jobs satisfactorily, it does not necessarily negate the fact that
the other jobs failures needs concentration and requires rework.

10.
Conclusion- Derderia will not break its record for the new jobs created unless a record number of compa-
nies start up this year

Let us consider a situation where the above conclusion will not hold water:
let's say no. of companies started last year: 10

No. of jobs rolled out/company: 5

Total jobs last year: 50

No. of comapnies stared this year: 6 (= less than 10)

No. of jobs rolled out/company: 20

Total jobs this year: 120 (= way more than 50)

looks like we've broken the jobs record, without breaking the record for the number of companies started.
so we don't need to have a record number of startup companies.

A. New startups may create more jobs. But this does not answer why the startups need to be more than
the number last year. What if a lesser number of startups began but added more jobs per startup than last
year?

Also here is what negation of A tells us:

start up companies do not create more jobs than established companies.

Even if the negation were true, the conclusion is STILL valid.

B. choice (b) deals only with companies established last year, whereas the stated premise deals with
companies established at any time before this year. the companies dealt with in choice (b), then, are only
a small subset of the companies dealt with in the stated premise.

however, we don't need to make the assumption in choice (b), as it's irrelevant one way or the other: the
companies founded last year are merely a subset of "previously established companies". therefore, since
the stated premise already gives us the overall result for ALL previously founded companies, we have no
need to be concerned with the results of smaller components of that population (such as (b)).

C. CORRECT. Fits the above scenario perfectly

D. it's dead out-of-scope. Any consideration of jobs lost is irrelevant.

E. Same as D :)

11.
A. Irrelevant. Clearly we are not concerned with the conditions that existed while the paintings were cre-
ated but rather with the conditions required to save those paintings.

B. Weakens,This weakens the conclusion slightly and does not provide any information about what would
happen the paintings were the museums to relax their standards.

C. Irrelevant. The passage clearly discusses the cost involved in storing renaissance paintings alone and
not other objects.

D. CORRECT. Let us negate this:

Other materials in Renaissance oil paintings other than the paint are vulnerable enough to relatively wide
fluctuations in temperature and humidity to cause damage to the paintings.

This would destroy the conclusion that “museums could relax their standards and save money without
endangering their Renaissance oil paintings” as it states that despite oil paint not being affected by tem-
perature fluctuations (like the study proves) other materials in the painting maybe.

E. Irrelevant.

12.

A. CORRECT. Conclusion states that “Clearly, therefore, insurance companies are making a greater prof-
it on collision-damage insurance in Greatport than in Fairmont” If it costs more at Greatport to repair
damages than it does at fairmont,then despite there being a lower number of collisions at Greatport,the
companies may not make greater profit.

B. Let us consider the negation of this:


There are more motorist in Greatport than in Fairmont.

We know the number of collisions in Greatport are fewer. Therefore if there were an increased number of
motorist and all other factors remained contant between Greatport and Fairmont, then this would
STREGTHEN the conclusion rather than weaken it.

C. This actually weakens the conclusion as more people reporting the accident = more money the insur-
ance company has to pay.

D. Irrelevant. The comparison is between G and F.

E. Irrelevant. This assumption neither weakens nor strengthens the conclusion.

13.

This is a no brainer. But let us start by negating options.


A. CORRECT.
People in western Africa developed staple crops that they stopped cultivating once rice

and yams were introduced.

This at once destroys the conclusion that by establishing when rice and yams were introduced in Africa
we could determine when the agricultural societies began.

B. Let us negate this

There are plants native to western Africa that, if domesticated, could serve as staple food

crops.

Great. But do we know if these native plants were cultivated by the people in Western Africa? Since we
do not know the answer to this, the conclusion could still be valid.

C. This is good information but neither strengthens or weakens the conclusion as we are specifically con-
cerned with the Western African socities.

D. ok but how does it help with arriving at the conclusion?irrelevant.

E. We already know this. But we also know that these are supplementary food and that no agricultural
society can exist without staple crops.

14.
year x: tax price went up by eight cents. sales fell by 10%
Year before x: tax price constant. Sales fell by 1%

Conclusion: the reason for the sales to fall from 1% to 10% is the 8 cent increase in the tax price.

A.Irrelevant.

year before x : pre-tax price went up. Tax price constant. Sales fell by 1%
year x: pre tax price same as previous yeatt.tax price went up by eight cents. sales fell by 10%
year after x: pre tax price increased slightly but not as much as year before x.

Clearly this does not provide any correlation between increased tax price and decrease in sales.

B. Let us negate this:

The one percent fall in cigarette sales in the year prior to tax increase was not due to a smaller tax in-
crease.

We know that the sales have decreased by 10% this year.

Even if the sales in the previous year HAD NOT decreased due to a tax increase, the conclusion “he rea-
son for the sales to fall from 1% to 10% is the 8 cent increase in the tax price.” still holds. Incorrect.
C. But why did the sales fall to 10% DURING the year of tax increase? Option C does not discuss this at
all and only talks of years before and after tax increase.

D. CORRECT.

Let us negate this:

For the year following the tax increase, the pretax price of a pack of cigarettes was eight or more cents
lower than it had been the previous year.

This would mean that the increased tax price was cancelled out by the decrease in the pretax price. So
the overall price of a pack of cigarettes was lower or the same as before. This would destroy any relation
that exists between increased tax price and the decrease in sales (as the total price for the customer re-
mained the same)

E. Even if the pretax price did not rise and was constant, the overall price would have gone up and this
could be sufficient reason for the decrease in sales.

15.
A. Irrelevant. The comparison is between jobs that require a degree and jobs that don't. Double. triple de-
gree holders and their income is not relevant here :)

B. the comparison is between now and four years later. Not several years ago and the present.

C. CORRECT.

If the negation of this were to be true then once the scarcity no longer exists the high paying jobs wont
exist either and the average salary may not increase.

D. let's say the average salary for non-degree jobs will increase over the next 4 years.

this would actually STRENGTHEN the argument (!), because it would provide another reason that the
average salary would increase.

this is the complete opposite of the effect you're looking for; reversing the assumption should destroy the
argument.

E. Irrelevant.

16.
Conclusion: people who have suffered a serious stroke on the left side of the brain without suffering any
such impairment must have their language centres in the right half.

A. This maybe. But we are only concerned with whether a stroke affects the linguistic capabilities of a
person.

B. Irrelevant. Once again, we are concerned with people suffering from stroke and that affecting their lin-
guistic capabilities.

C. Irrelevant to the conclusion.


D. CORRECT. Let us negate this:

If there are language centres on the left side of the brain, any serious stroke affecting that

side of the brain damages does not damage any of them.


This would destroy the conclusion. It would now be possible for a person to have the language centre in
the left half, suffer a stroke that affected the left half of the brain and still retain linguistic capabilities

E. Since we are discussing people who have suffered a stroke which has damaged the brain, this option
does strengthen the conclusion to an extent.

However, It is NOT A NECESSARY assumption. The negation of the option would still cause the conclu-
sion to hold.

17.
A. CORRECT.

Let us negate A.

The quantities of dissolved salts deposited by rivers in the Earth’s oceans have been unusually large dur-
ing the past hundred years.

The conclusion says that by determining the increased salt levels in the ocean in the past 100 years and
extrapolating it to how many centuries how such deposition would have caused the salt level in the ocean
to rise to the current level, we can determine the age of the planet.

However, if there has been unusually large amounts of salt deposited in the last 100 years, this extrapola-
tion would be inaccurate and the resulting age of the earth maybe far younger.

B. This is not relevant to the conclusion. We are concerned with how much salt these rivers deposit and
not how much they contain. All rivers may have similar salt content, But have they deposited a constant
amount in the ocean?

C. This would weaken the conclusion by indicating that there are other ways in which the salt levels of the
ocean can increase.

D. This is irrelevant. We are discussing the effectiveness of the method at hand. Even if there were supe-
rior methods,our conclusion will remain unchanged.

E. To estimate the age of the oceans one will divide "ocean's current salt levels" A by "the resulting in-
crease in salt levels in the oceans over the past hundred years" a.

If the biological activities are using up salt and have been doing so from the beginning at a constant rate
then we have:

If biological activities use some of the salt, both quantities A and a will decrease correspondingly and the
calculation should lead to the same result.
Whether or not "the salts carried into the Earth’s oceans by rivers are used up by biological activity in the
oceans" will not change the reasoning.

18.
A. Irrelevant to the conclusion.

B. Increasing fertility rates are once again,irrelevant. We want to know if the fertility rates are affected by
the material in the tranquiliser.

C. CORRECT. Let us Negate this:

Any stress that female rhinoceroses may suffer as a result of being immobilized and handled

has negative effect on their fertility.

This would give us an alternate reason for the fertility of the female rhinos decreasing on being immobi-
lised. As you can see, with the negation, the conclusion “Probably, therefore, some substance in the tran-
quilizer inhibits fertility” instantly crumbles.

D. Since we are discussing fertility, we can safely ignore the male rhinos!

E. Since we do not have any information about the fertility rates in these mammals,this option does not
help/harm the conclusion.

19.
The conclusion states that because the modern day analysis indicates low gold content and because the
mining expeditions of the 16 century could not find any gold, the “methods used to determine the gold

content of Frobisher’s samples must have been inaccurate”

A. This weakens the conclusion to an extent by indicating that there was more gold on the island in 16th
century than presently.

B. This weakens the conclusion to an extent by indicating that since the same parts of the island were not
mined, they perhaps missed the gold rich soil.

C. Different does not necessarily mean inaccurate or less effective.

D. Irrelevant. We are discussing Kodlunarn Island only.

E. let us negate this


Gold was added to the soil samples collected by Frobisher before the samples were examined.

This would destroy the conclusion that the technique used by Frobisher was inaccurate and infact indi-
cate that while his technique was correct,the soil had been tampered with to indicate high gold content.

20.
A. This maybe. But the goal was to decrease the hazardous waste by half. This option does not tell us
anything about hazardous waste.

B. is irrelevant, as we have no information about how much waste is produced PER JET - not now, not
then.

C. once again , this is irrelevant. The goal was to “reduce by half the total yearly amount of hazardous
waste generated by its passenger-jet division”.

The other divisions are irrelevant to our argument.

D. We are discussing the waste output per pound per worker. The amount of time they spend working is
irrelevant

E. CORRECT.
the production company's GOAL was to lower the TOTAL AMOUNT of hazardous waste produced.

the EVIDENCE is stated in terms of the amount PER WORKER, not the total amount.

Let us negate this option


“ The number of production workers assigned to the passenger-jet division was significantly less in 1994
than it was last year”

This tells us why the amount of waste per worker may have gone down despite the amount of hazardous
waste being produced having remained the same or increased. This would cause the conclusion to crum-
ble.

21.
This one is pretty simple. Let us try our favourite method of negation here:

A. Irrelevant. This does not answer whether cannabinoids help stimulate the appetite or not.

B. The passage does not require us to assume that cannabinoids are the ONLY substance that stimulate
the appetite. Even if there were other substance that do so, the passage goes on to tell us that blocking
cannabinoids caused the appetite to decrease. This option does to give us any reason why this may have
happened.

C. Irrelevant. This was true both before and after the mice were injected with a chemical that blocked
cannabinoids.

D. Irrelevant. Does it help stimulate appetite? Thats all we care to know.

E. CORRECT.
Let us negate this.
“The chemical that blocks cannabinoids from stimulating their brain receptors independently inhibits the
appetite”

This clearly suggests that the loss of appetite was caused due to factors other than absence of canna-
binods. This destroys the conclusion.
22.
Let us assume that before 1992 there were 100,000 catchable lobsters in the sea.
Of 100,000 catchable lobsters, let us assume all 100,000 were legally harvested.

In 1996,

we know that the number of legally harvested lobsters were 91,000.

Therefore the passage states that that 9000 of the remaining lobesters (assuming there are still 100,000
catchable lobsters) were caught by illegal activities.

A. CORRECT.

let us negate this


The illegal lobster harvesting was so extensive that the population of catchable lobsters in

Belukia’s territorial waters had sharply declined by 1996.

This tells us that the catchable population is no longer 100,000.

Let us assume its come down to 95,000. So going by this assumption, the number of lobsters illegally
harvested comes to 95,000 – 91,000 = 4000. This destroys the conclusion.

B. Maybe. But do we know if it has increased to 9000 tons?

C. Irrelevant. We already know this to be true from the passage.

D. Since the annual legal catch was 9000 tons less than the pre 1992, we already know this statement to
be true.

E. This one may seem tempting. But we know that there “no reduction in the level of legal lobster fishing
activity”. Thus this option does not affect our conclusion.

23.
conclusion: Since the hotel taxes were lowered and yet the govt collected the same amount of money
from thee hotels,it follows that more number of tourists stayed in Midville that year.

A. Irrelevant. While the tourists maybe aware of lowered hotel rates, the option does not tell us that more
tourists came in because of it.

B. The prices in other cities is irrelevant.

C. CORRECT. Let us negate this:


“The average length of a tourist’s stay in Midville hotels was longer last year than it had been the year
before.”
This tells us why the despite the number of tourists having remained the same or lesser than previous
year,the hotels could have made more money thereby destroying the conclusion.
D. Irrelevant. The argument states that lowered taxes led to more number of tourists staying at Midville.
We are not discussing Midville's marketing stratergy here.

E. Tempting but please note that we are discussing lowered tax on hotel accommodation here. The price
on the meals is irrelevant.

24.
This is pretty straightforward.

Premise: one candidate produced a half-hour-long advertisement. During the half hour the advertisement
was aired, a substantial portion of the viewing public tuned into the advertisement.

Conclusion:many more people are interested in watching lengthy televised political messages than was
previously thought.

A. we are not interested in the effect the advertisement had on the people. Our only concern is do people
want to watch the long advertisement or not. If we negate this statement “ The candidate’s ratings did not
improve significantly as a result of the half-hour-long political advertisement” the conclusion can still be
valid. Maybe people did tune in to watch the campaign but did not like what they saw.

B. Irrelevant. We are concerned with the length of the political message.

C. Thats good. But do they want to do so in the form of lengthy televised political message?Thats the
crux of our argument that this option does not address.

D. Irrelevant. We already know a good amount of the population tuned in to watch the ad.

E. CORRECT.
Let us negate this:

Most of the viewers who tuned in to the candidate’s half-hour-long advertisement last year changed
channels after the first few minutes.

This would immediately destroy the conclusion that more people are interested in watching lengthy politi-
cal messages.

25.

This one is quite easy. Once again, let us refer back to our old friend-negation

A. This actually weakens the conclusion stating that the nerve damage caused by other chemicals would
be different from that caused by EB, indicating that we would be able to distinguish the reason for the
nerve damage.

B. Irrelevant. Does EB cause nerve damage or do the new chemicals being used also cause it?This op-
tion does not help us confirm either of the questions.
C. CORRECT. Let us negate this.
If ethylene dibromide causes nerve damage, it takes two years or longer for that

damage to become detectable.

If this were true then we know that EB was NOT wrongly blamed and also that new chemicals do not
cause nerve damage. This would collapse the conclusion.

D. we are talking about people who were “newly diagnosed”

E. strengthens the argument to an extent. But we will still have to assume that these plants do not have
any external factors that could cause nerve damage.

26.
This problem may seem tough only due the way its been worded.
The summary, simple put is this: There are some people who are allergic to sulphites. Several wine mak-
ers do not add sulphites as preservatives to their wines.

Conclusion: therefore people suffering from allergic reactions can drink wine from such makers to avoid
allergic reactions.

A. We are only concerned with sulphites and the allergy induced by them. We are not discussing any oth-
er “potentially allergic” substances.

B. We clearly know that people are allergic to sulphites that are added to preserve wine. That makes this
option irrelevant.

C. This information is irrelevant.

D. Again,we are discussing allergic reactions to sulphites in particular. Not relevant to the conclusion.

E. CORRECT. Let us negate this.

Sulfites are naturally present in the wines produced by these wine makers in amounts large enough to
produce an allergic reaction in someone who drinks these wines.
This clearly destroys the conclusion that such wines can be consumed by people with allergic reaction to
sulphites.

Therefore option E is a critical assumption we need to make to arrive at this conclusion.

27.
A. Irrelevant. The comparison is between households with a built-clock in their microwave oven and
household that have just the oven. The comparison is not between households with ovens and those
without.

B. This information is irrelevant to the discussion. We are discussing the electricity consumption.
C. Additional information to the stimulus - not helpful in addressing the issue at hand. Even if the nega-
tion of this option were true, the conclusion will hold.

D. CORRECT. - stating that if an oven don't have a clock, the household will not install one is essential to
the conclusion that houses without built in clocks in the microwave consume less electricity. The negation
of this option would destroy the conclusion ―Therefore, households whose microwave oven has no built-in
clock use 45 kilowatt-hours per year less, on average, than do comparable households whose microwave
oven has one‖

Negation:

Households whose microwave oven does not have a built-in clock are more likely to have a separate elec-
tric

clock plugged in than households whose microwave oven has one.

E. - Additional information to the stimulus - Not quite relevant. True otr not, this option does not affect
the conclusion.

28.

The conclusion is : “Therefore, by selling the helium, the government can not only pay off that debt but
reduce its overall debt as well”

Also its important to note that currently the govt can make 25% more money by selling heilum than the
current debt incurred due to it

Let us see which options support this conclusion.

A. This maybe. But does it mean that by selling the helium the govt will be able to pay off debts?We are
not told.

B. Whether it is a significant portion of the total debt is irrelevant. We know that by selling of helium, the
government can reduce its helium debts as well as a portion (whatever that maybe) of its total debts

C. This is completely out of scope.

D. CORRECT.

Let us negate:
Attempts to sell the government’s helium will depress the market price of helium by more than 25 percent.
This would mean that the prices of helium would fall by the 25% margin and not allow the government to
recover the debts it has incurred in purchasing and storing helium.

E. For the conclusion to fall apart, the loss incurred should be 25% more than the money spent in acquir-
ing and storing helium. This seems highly unlikely and moreover is an assumption thats outside the pas-
sage.
29

Premise: a technology for preventing leaks of oil pipelines is being installed


Conclusion: as long as the above technology is effective,there will be no leaks and hence no danger to
the fish.

Assumption: Are the leaks the only thing that may endanger the fish population when oil pipeline is con-
structed?

A. The conclusion talks of the dangers of constructing oilpipelines only. This is out of scope.

B. CORRECT.
Negation: Other than the possibility of a leak, there is a realistic pollution threat posed to the lake by the
pipeline’s construction.

This would mean that even if the anti leak technology was successful it would still destroy the fish popula-
tion thereby causing the conclusion to fall apart.

C. This is already stated in the conclusion “provided the technology is effective” so we know that the con-
clusion is subject to the condition that the anti leak technology is successful. This neither strengthens nor
weakens the conclusion.

D. Irrelevant. We are not concerned about other damages.

E. The passage discusses fish population in general and this is not specific to a certain species.

30.
Summary:

Coponia plans to increase the tax by 9 cents. This would inturn cause a 10 percentage point increase in
the price of cigarettes.

A 10 percentage point increase in price has always caused the sales to Dip by 4%.

conclusion: therefore,rising the tax by 9 cents on a 90cent packet of cigarettes will also cause the sales to
decrease by 4%.

Note: it is essential that the overall price of the cigarettes increase to cause a decrese in sales. What if
the cigarette companies decide to cut down their profit margin and absorb the extra 9 cent costs on the
tax to prevent reduction in sales? In this case the overall price of the cigarettes will not rise and the sales
will not fall.

Therefore option A is correct.

31.
The main conclusion here is by offering a rigorous fitness program of jogging and weight lifting to all em-
ployees, and requiring employees who are out of shape to participate, the company will save doctor bills
and longer hospital stays that less fir employees usually incur.
Therefore its more profitable for the company to invest in a fitness program.

A. This is irrelevant to the conclusion. We know already that less fit people incur higher medical bills.

B. This weakens the conclusion by stating that somehow the people asked not to participate in the pro-
gram incur heftier medical bills. This is a complete contradiction of what's stated in the passage.

C.CORRECT.
Let us negate it:
The strenuous activities required of out-of-shape employees by the program would by themselves gen-
erate medical expenses greater than any reduction achieved by the program.

This means that the fitness activity for our of shape employees will cost the company a whole lot more
interms of medically bills and more importantly,would be “greater than any reduction achieved by the
program”
This completely destorys the conclusion

D. We already know this to be true.

E. While this a good thing, it says the lesser fit employess will be at work more than the more fit employ-
ees(not required to participate in the program) This does not directly explain why the health insurance
cost for the company will go down.

32.
If this is solved based on the negation technique A is a clear choice
Negation of A :The tetracycline deposits formed after the bodies were buried.

This destroys the conclusion that “tetracycline in their food probably explains the low incidence of typhus
among ancient Nubians”

33.

Since the number of shops and workshops in Brindon County have reduced, the passage states that the
output of Brindon Bolt Barn will also reduce.

Let us see the negation of which option would destroy this conclusion/

A. This is irrelevant. The passage tells us that the workshops were shut down in the recent months and
the sales and revenue for next year will decrease.

B. Although this tells us to some extent that the amount of work for the workshops in Brindon County is
not very high,therefore sales may not be much, we do not know if the Brindon Bolts operates only in Brin-
don County or outside of it.

C. We are concerned with the sales and revenue of Brindon Bolts Barn only.

D. This is out of scope. We need information about its sales and revenue.
E. CORRECT. Let us negate this:
The Brindon Bolt Barn is a company that gets the great majority of its business from customers outside
Brindon County.

This means that although shops have shut in the Brindon county and its sales in the county maybe low,
the overall sales of Brindon Bolt Barn will continue to be high as compared to last year.

34.

Conclusion: Since average homework time is 30 minutes, there is no need to impose limits on the amount
as it does not eat into much of children's free time.

A. Even if this weren't true, the theorist's argument is that the children will have sufficient time to complete
their homework as well as engage in free time activities.(irrespective of what those activities are)

B. This argument isn't about the effectiveness of homework. Rather its about whether a child can do
homework as well as have time for other activities.

C. This would actually weaken the theorist's argument to an extent indicating that we need someone else
other than teachers to decide the amount of homework (Exactly what he is arguing against)

D. CORRECT. Let us negate this:

In most schools, if not all, the homework assignments given are of a length that diverges widely from the
average.
This would mean that children may have to spend more than 30 mins which might cut into their free time
and thereby proving the point made by the editorials. This would destroy the conclusion.

E. The theorist does not state that free time activities are useless. This is out of scope.

35.

The crux of this argument is as below:

Public advocacy groups can directly pass on information to the public. This means that information ser-
vices(the internet) present a more balanced picture of the complexities of political issues than any tradi-
tional news source presents

A. the argument does not discuss the reach of information services or newspapers. It simply says that
through information services a more balanced picture can be given to the public.

B. This actually weakens the conclusion by stating that an unbiased view can also be obtained by sorting
through several papers.

C. Correct. Let us negate this:


Information on political issues disseminated through information services comes almost entirely from ad-
vocacy groups that share a single bias.

This would mean that despite using information services the information will still be as (or more) biased
that the ones got in the papers,defeating the purpose. This would effectively destroy the conclusion.

D. We already know this to be true.

E. Once again this weakens the conclusion by indicating that there is no need for an unbiased source.

36.

A. CORRECT.

Here's a diagram for this argument:

(# of rds-equipped stations increased 250-->600)

BUT

(apprx same # of rds radios in Vland)

THEREFORE

(apprx same # of people receiving rds signals in Vland)

if you make this diagram, it should be clear that there's a logical leap between the latter two statements:
the author is assuming a direct correlation between the # of rds-equipped radios and the # of people
who actually receive rds signals with those radios.

in order to make this connection, you need to assume that nobody, or almost nobody**, with an rds-
equipped radio is now (in '96, that is) able to receive a signal but wasn't able to receive a signal back in
'94. that's pretty much what A. says.

B. Irrelevant. As it does not directly discuss whether these people already had a RDS radio before the
station was built , it does not fill the gap.

C. C is out of scope. It says that equipping a radio with the RDS feature does not decrease the range of
the radio. But who cares? Even if it did, this would have been the case for 1994 as well as 1996. We
need something that connects the data about radios and 94-96 with the claim that THE SAME NUMBER
OF PEOPLE received the programming, even though no one bought new radios.

D. D is not an assumption - it is a fact. We are told that you cannot get this type of signal without the
right equipment. An assumption must be something that is unstated.
E. Let’s negate this:

The RDS radio stations in Verdland in 1996 all offered the same type of programming.
This does not affect our conclusion in any way and hence is irrelevant.

37.

a) Reverse this assumption:

The Sepphoris mosaics are composed exclusively of types of stones found naturally in teh Sepphoris ar-
ea
This is consistent with the conclusion. Note that the passage says the mosaics were “created” by travel-
ling artisans which means that they could have used the locally available stones at Sepphoris to create
the mosaics.

B.

reverse this assumption: let's say all the species are indigenous to some common region.

this doesn't destroy the argument; it's perfectly consistent with the idea of traveling artisans (who presum-
ably would have come from that common region).

wrong answer.

C. reverse this
motifs that appear in the Sepphoris mosaics that also appear in the mosaics of some

other Roman city

This actually strengthens the conclusion rather than destroying it.

D.

reverse this assumption: let's say that there are some animal figures that are not readily identifiable.

this has no effect whatsoever on the argument, which is concerned only with some of the animal figures
(i.e., the ones that weren't native to the local area).

wrong answer.

E. CORRECT

reverse this assumption: let's say there was a common repertory of mosaic designs.
in this case, that repertory - since it was a common repertory - would have included animal figures from all
over the place. (at the very least, it would be quite unreasonable to expect a common repertory to have
been restricted to animal figures from the sepphoris area in particular.)

this destroys the argument, because, were there such a repertory, then artists local to sepphoris would
have followed it as well, creating the exotic designs despite their status as natives in the area.

38.

the first premise states, as fact, the following correlation:

higher vitamin/minerals in bloodstream <--> better lung function

The loophole:

if smokers get higher levels of the vitamins/minerals into their bloodstreams, then based on the premise
they will also have better lung function. The argument is therefore airtight, provided the smokers can get
the vitamins and minerals into their bloodstreams to begin with.(absorption)

That's the only missing link here - all that's mentioned in the argument is intake, which is an entirely sep-
arate matter from absorption into the bloodstream. We need to look for an option that can close this gap.

A)Irrelevant. it speaks of vitamins and minerals in general, not the special vitamins and minerals men-
tioned in the passage.

B)Strengthens the premise which we already know to be true

C)Irrelevant as we are only discussing how lung capacity of a smoker can be improved. People suffering
from vitamin deficiency (smokers or non smokers) who have other health problems are not relevant here.

D)irrelevant, as cessation of smoking doesn't affect the issue (the effect of the vitamins/minerals) at all.

E)CORRECT because it takes care of the connection between intake and absorption, by establishing that
the smokers' intake of vitamins and minerals will actually make it into the bloodstream.

Let’s negate this option:

Smoking introduces into the body chemicals that prevent the helpful vitamins and minerals from entering
the bloodstream.
This would destroy the conclusion by stating that despite smokers increasing their vitamin intake it will
not help improve their lung function.

39.

A. Let’s negate this:

"The ―Thank you‖ messages would not have the same impact on regular patrons of a restaurant as they
would on occasional patrons of the same restaurant".

This means that the message must have a positive effect on group and a negative effect on the other.

Even if the messages do not have the same impact on regular patrons as they do on occasional patrons,
the average income from tips would still increase. The group on which the messages have had a positive
effect will continue to tip more.

B. CORRECT.

Let’s negate this:

"Regularly seeing ―Thank you‖ written on their bills would lead restaurant patrons to revert to their earlier
tipping habits."

This means that the reason the patrons tipped more was because the messages were a one off thing.
Seeing the message with the bill regularly will not have the same impact anymore.

Therefore the assumption that seeing the message regularly will not cause the patrons to revert to their
previous tipping habit is essential to the conclusion.

C. We already know that the patrons are tipping more. The reason that they are doing so is not relevant
to the conclusion and is definitely not the assumption on which the argument relies.

D. The argument says that the "Tips on these bills were an average of three percentage points higher
than tips on bills without the message."

Since the comparison is within the same hotel, between bills with messages and bills without, even if rate
at which people tip food servers in Pennsylvania varied with how expensive a hotel was, it still does not
destroy the conclusion.
E. We know that there was an an average three point increase in the tips. Even if this option weren't true
i.e "Not all patrons of the Pennsylvania restaurants in the study who were given a bill with ―Thank you‖
written on it left a larger tip than they otherwise would have."

the conclusion still holds.

40.

A. Many scientists make this assumption, of course – but its not an assumption on which the explanation
depends. The explanation concerns the scientists' motivation, not their style of doing research.

B. This statement gives another reason that scientists may reject a populariser, but because it is not the
reason implied in the passage, it is not assumed.

C. Even if this is true, it does not address the core issue of the argument, what scientists believe about
the motivation of popularisers.

D. CORRECT. This statement properly identifies an assumption on which the explanation for scientists'
rejection of popularisers depends.

E. The passage is not concerned with whether non-scientists can understand new research, but rather
with the beliefs and motivations of scientists who reject popularisers as colleagues.

41.
A. The argument concerns innovation in bicycle technology. It is not about the entire market for all bicy-
cles, so this claim about traditional bicycles is not assumed.

B. The passage does not discuss where the best innovation are likely to be created, so no assumption
about small workshops versus large manufacturers is made

C. CORRECT. This statement identifies information that appropriately fills the gap in the reasoning as
stated in the passage.

D. This claim provides an explanation of manufacturers' technological conservatism that is quite different
from the explanation indicated in the passage.

E. The passage does not indicate what the racing authorities do or do not know about bicycle innovation
– even though it suggests that they may be reluctant to approve every possible innovation for racing
purposes.
42.
Sharon makes a general statement claiming that if a person knows 50 workers,it is likely that at least one
of them is unemployed. This generalization is not likely to be true if unemployment were concentrated in
certain geographic isolated areas.

A. Sharon's argument is about normal level of unemployment; how rarely or frequently that level is ex-
ceeded is outside the scope of her argument.

B. CORRECT. This statement properly identifies an assumption that underlies Sharon's argument If un-
employment were concentrated in geographically isolated segments of populations, then randomly se-
lected individuals would not have an equivalent chance of knowing an unemployed person. Instead,
based on geographic factors, some individuals would be much more likely to know unemployed workers
than would other individuals. In this case, Sharon's conclusion crumbles.

C. Although Sharon's argument is compatible with saying than even more than 90% of the population is
unemployed, nothing suggests that she assumes this is true.

D. Sharon's argument is not based on the figure Roland cites and does not assume its accuracy or inac-
curacy; her argument merely points out that his figure is not consistent with a normal rate of unemploy-
ment.

E. Irrelevant. We do not know if such a fear has any consequence. Maybe the person will work harder to
keep his job or maybe he will bungle it up. The negation of this option would still cause Sharon's conclu-
sion to hold.

43.
Premise: Because no employee wants to be associated with bad news in the eyes of a superior, infor-
mation about serious problems at lower levels is progressively softened and distorted as it goes up each
step in the

management hierarchy.
Conclusion: The chief executive is, therefore, less well informed about problems at lower levels than are
his or her subordinates at those levels.

Assumption: The chief executive is less well informed about problems because his subordinates soften
and distort the information at each level( based on the premise) indicating that problems have no other
way of reaching the chief executive other than through his subordinates.

A. is tempting to choose because it seems like a pretty good assumption to make in the real world; An-
swer A is wrong, though, because how the problems ―should‖ be solved doesn’t tell me anything about
how well-informed the chief executive is about those problems.

B. is tempting to choose because it sounds like a good way to resolve the problem described in the ar-
gument. Answer B is wrong, though, because we weren’t asked to resolve the problem; we were asked
to articulate a belief (an assumption) of the author who is pointing out the problem.
C. is tempting to choose because it sounds like a pretty good assumption to make in the real world. An-
swer C is wrong, though, because the ability to solve a problem still doesn’t tell me anything about how
well-informed the chief executive is about those problems.

D. CORRECT. because this is exactly the assumption that the author makes to draw his conclusion. If
CEOs really can’t get info from anyone other than their subordinates, and if those subordinates don’t
want to tell them any bad news, then those CEOs are not going to be well-informed about problems.

E. wrong, because it weakens the argument: if some subordinates are speaking up, then the bosses
aren’t less well-informed. We were asked to find an assumption, and an assumption is something the au-
thor must believe to be true in order to draw that conclusion. If the answer choice actually weakens the
conclusion, then that answer can’t be a valid assumption

44.
Conclusion: Personalities unsuited to the requirements of the job will be eliminated from consideration
during the interview.

Assumption: such personalities can be identified during the interview process.

We need to look for an option that addresses this assumption.

A. This is simply restating the premise. We already know that ―interview is an essential part of a success-
ful hiring program‖.

B. There is nothing in the passage that makes the comparison between job development and interview.

Option C is CORRECT.

―Interviewers can accurately identify applicants whose personalities are unsuited to the requirements of
the job‖
Clearly addresses the assumption made by the argument.

Let’s try negating this option:

Interviewers cannot accurately identify applicants whose personalities are unsuited to the requirements
of the job‖
Now the entire conclusion made above crumbles, if the interviewers cannot identify suited/unsuited per-
sonalities for the job then the purpose of an interview is lost. This proves option C is CORRECT.
D. It can be the only factor or it can be one of the many factors assessed in an interview. All we know is
that it is a factor. The conclusion will remain strong irrespective of whether the negation of this statement
is true.

E. We are not concerned with what was important in the past.

45.
Fact:Bank depositors in the United States are all financially protected against bank failure because the
government insures all individuals' bank deposits
Premise: Since depositors do not lose any money of the banks fail ,they do not care about the soundness
of the investment the bank makes.
Conclusion: If the government did not insure the depositors money, the depositors would take more care
in understanding the bank polices, thereby making the banks more secure.

This conclusion is clearly based on the assumption that ―the depositors would take more care in under-
standing the bank polices and hence will be able to determine which is a safer bank ‖.

Let’s look for an option that strengthens this assumption.

A. How bank failures are caused is irrelevant.

B. Irrelevant. The passage does not assume the number of accounts per bank per depositor anywhere.
C. Once again, the passage does not assume this. There is no connection made to the amount deposited
versus the care taken by the depositor to pick a bank.

D. Irrelevant. Even if the opposite were true I.e ―The difference in the interest rates paid to depositors by
different banks is a significant factor in bank failures‖ it would still make no difference to the economist's
argument which indicates that depositors become complacent on being assured of their money back de-
spite bank failures.

E .CORRECT. This is the basis of the conclusion made by the economist. If potential depositors are una-
ble to determine which banks are secure against failure, then the entire conclusion that removal of gov-
ernment insurance will make banks safer would crumble.

46.
A. This is irrelevant. We do not care about commercial fish . Our concern is the porpoise alone.

B. CORRECT

negating it , we have

― When disoriented, harbor porpoises are significantly more likely to be killed by other boats.‖
This would destroy the conclusion that ―The alarms, therefore, are saving the

lives of harbor porpoises in those waters.‖

C. Although this might strengthen the use of alarms, its not an assumption. This is simply a fact stated iin
support of the plan i.e the premise.

D. Irrelevant. Even if the harbor porpoises were not the largest in number at the time the alarm was in-
stalled, it does not affect the conclusion.

E. We talking about the safety of the porpoises and not the costs involved. This is irrelevant.

47.
An assumption that supports a conclusion, though unstated, is NECESSARILY TRUE. In other words, the
conclusion must directly depend on the information from the assumption. In this argument, the conclu-
sion states that drivers who equip their vehicles with radar detectors are more likely to exceed the speed
limit regularly than are driver who do not. The only factual basis (i.e. premise) for this conclusion is that
33% of vehicles ticketed for exceeding the speed limit were equipped with radar detectors, while only
3% of drivers on Maryland highways equip their vehicles with radar detectors.

The only information provided is about the percentage of vehicles equipped with radar and the percent-
age of ticketed vehicles equipped with radar; the conclusion, however, makes a sweeping generalization
about the regular driving behavior of individuals with radar-equipped vehicles. When the GMAT creates
such a "jump" or "gap" in the logic of an argument, the assumption will often "fill in" the missing infor-
mation.

In order for the conclusion to be valid, it must be true that these ticketed drivers exceed the speed limit
on a regular basis. This is satisfied by option B.

48.
The argument very clearly assumes that the newly thin people will continue to consume the same num-
ber of calories that the people whose weight is normally at that level do. Thereby they are unable to burn
as many calories as the people whose weight is normally at that level and regain their original weight.

A. CORRECT. Negating this we find ―Large number of very overweight people who have dieted down to a
new weight tend to continue to consume substantially fewer calories than do people whose normal
weight is at that level.‖ this would make our assumptions untrue and would allow the very over weight
people to maintain their new weight.

B. Negation of B
the metabolism of people who are usually not overweight are not able to vary than the metabolism of
people who have been very overweight.

Since the argument has already established that the metabolic rates of the usually overweight don't much
vary with dieting, the real force of negating B is the metabolic rates of those who are not usually over-
weight don't much vary with dieting But this doesn't weaken the conclusion that the usually overweight
will regain weight. It doesn't say anything whatsoever about the usually overweight.

C. Negation ―The amount of calories that a person usually burns in a day is not determined more by the
amount that is consumed that day than by the current weight of the individual.‖

This simply tells us that the amount of calories burned depends on the current weight of the individual I.e
the metabolism or equally on the amount eaten as well as the calorie intake.
However,the premise already tells us that the metabolism of overweight individual that has lost weight
through dieting is lower than that of an individual who is normally at that weight.

D. Irrelevant. Whether such a chemical exists or not, it will in no way affect the conclusion which dis-
cusses people who have lost weight primarily through dieting.

E. Irrelevant. The conclusion talks about people who are overweight and have lost weight,not people with
normal weight.

49.

the entire argument can be summed up as follows:

top managers use intuition more than do midlevel or lower-level managers

-->

intuition is MORE EFFECTIVE

there's a huge, glaring hole there: the argument has connected "top managers use intuition more" with
"it's a more effective way of decision making". these 2 specifics are entirely different from one another,
and NEED to be bridged for the argument to hold up.

in fact, you should get into the habit of regarding ANY two specifics that are even remotely different
(such as "drivers who speed" and "drivers who receive speeding tickets") as completely different things.
under that criterion, it's a no-brainer to choose the assumption that bridges the gap.

A. Irrelevant. The passage already suggests that Methodical, step-by-step reasoning is employed by
managers for decision making.
B. Irrelevant. ―top managers used intuition significantly more than did most middle- or lower-level man-
agers ― indicates that both top level managers and middle level managers have the ability to use either
intuitive reasoning or methodical, step-by-step reasoning in making decisions.

C. Irrelevant. Even if this were true, it wouldn't explain why ―intuition is actually more effective than care-
ful, methodical reasoning‖

D. Irrelevant. The comparison is between the success of the methods used by top managers versus mid-
dle level and low level managers.

E. CORRECT. This explain why intuition is considered a more successful way of decision making than
step-by-step reasoning by indicating that the managers who employ the former (Top managers) are
more successful.

50.

The argument concludes that since the evaluated cookware is superior to all other cookware advertised in
Kitchen, lending the magazine name to the line of cookware will DECREASE the revenue of the magazine
as the reputation of the magazine will not suffer.

The argument assumes that reputation alone is responsible for generating revenue.

Let’s use the negation technique to find which option destroys the conclusion.

A. Negation:

Other line of cookware is superior to that which will carry the Kitchen name.

This does not affect the conclusion. We are clearly told that among all the cookware advertised in Kitch-
en, the one Kitchen is lending it's name to is of the highest quality. Therefore it is unlikely that the read-
ers of Kitchen will be put off by a product that is not advertised in Kitchen if it's of a superior quality.

B. Negation:

Kitchen will license the use of its name for any products other than the line of cookware.

Unless we are explicitly told that the quality of these products is below the expected standard, it does not
affect the conclusion.

C. CORRECT

Negation:

Makers of cookware will find Kitchen a less attractive advertising vehicle because the magazine's name is
associated with a competing product.
This gives us a reason why the revenue of Kitchen might decrease despite the line of cookware it is lend-
ing it's name to being of superior quality.

If the competing cookware lines refuse to advertise in the Kitchen, it will lose it's advertisement revenues
from those companies.

D. Negation:

Consumers who are regular readers of Kitchen magazine will be attracted to the cookware by the Kitchen
name.

This would further strengthen the conclusion rather than weaken i.

E. Kitchen is not one of the most prestigious cooking-related magazines.

This neither strengthens nor weakens the conclusion. We are simply concerned with how Kitchen's plan
of lending it's name will affect revenues.

51.

A. This is irrelevant. The question is whether writing VOID on the check will deter the production of coun-
terfeit checks .

B. Once again, this is out of scope. Irrespective of the denomination for which the fake checks are made,
will issuing checks with dots too small to be picked up by the electronic scanner used by counterfeit
checks,help?

C. Irrelevant.

D. This strengthens the plan slightly by telling us that the cost of such new checks will be no more than
the old ones.

E. This strengthens the plan by telling us that the electronic scanners in the near future will not be able
to duplicate such tiny dots. If we negate this, the argument falls apart. Correct.

Assumptions Part 2

1.
The Police Commissioner's proposal hopes to decrease the number of crimes in city Y by shift-
ing police officers from low-crime to high-crime districts. His proposal is based on data that
demonstrate that crime decreases when additional police officers are moved into a district.
However, the data do not mention anything about the effect on the districts from which the po-
lice officers were removed. The commissioner's plan is based on the assumption that the
movement of police officers will not have any adverse effects on the low-crime districts.

(A) While it is encouraging that a similar plan worked successfully in City X, this fact is certainly
not essential for the success of the plan in City Y. The cities may be so different as to make the
comparison meaningless.

(B) The police commissioner's proposal is focused solely on decreasing the number of crimes in
city Y. The severity of the crimes has no bearing on whether the commissioner's proposal will
succeed or not.

(C) The actual numerical distinction between high and low-crime areas of the city is immaterial
to the commissioner's proposal. For instance, if the number of crimes committed in all high
crime districts was only double (instead of more than triple) the number of crimes committed in
low crime districts, the proposal could still be valid.

(D) It would be practically beneficial to the commissioner's plan if there were more low crime
than high crime districts in city Y. This would enable the movement of police officers to every
high crime district. However, this is not necessary to achieve the commissioner's goal of de-
creasing the total number of crimes in city Y. Even if there were more high-crime districts than
low-crime districts in city Y, police officers could still be shifted to some (though not all) high-
crime districts, and thereby possibly reduce the total number of crimes in city Y.

(E) CORRECT. The police commissioner's proposal would not make sense if districts of the city
from which police officers are removed experience significant crime increases shortly after the
removal of those officers. This would at least partially, if not fully, negate the reduction in the
number of crimes in the high-crime districts. This choice establishes that, in fact, the low-crime
districts do NOT suffer from significant crime increases after the removal of some officers--an
essential assumption upon which the commissioner's proposal depends.
2.

The argument concludes that rising sea levels caused by global warming will destroy major
coastal population centers and displace millions of people. Any assumption in support of this
conclusion would have to corroborate that these events will definitively take place.

(A) CORRECT. If new technological developments in the next century allow people to divert
rising seas from the world’s cities (i.e., population centers), cities will not be destroyed and mil-
lions of people will not be displaced. Thus, a necessary assumption is that these technologies
will not be developed.

(B) A simple awareness of the steps to reduce emissions in no way undermines the argument’s
conclusion, as this answer choice does not describe any action being taken by individuals. Addi-
tionally, greenhouse gases are never mentioned as the primary by-product of human activity
that causes global warming, and are therefore not sufficient to address the argument.

(C) The argument never suggests that all coastal population centers are similarly affected; this
choice is too extreme and overreaching for the argument’s conclusion.

(D) This might be true, but it is not an assumption on which the conclusion rests. Instead, this
answer choice is simply an inference that might be drawn from the premises.

(E) The idea that human activity is the sole cause of global warming is neither suggested nor
assumed by the argument. In addition, the wording "sole cause" is too extreme.

3.

As an advertisement, this passage attempts to entice the reader into purchasing a new HitItFar
driver by touting its benefits, both implied or explicit. The key to answering this question is to
be able to analyze each claim to determine whether it is implied, explicitly stated, or neither.
(A) By asking the reader rhetorically "isn't it time you added power ... and distance ... [by
switching to the HitItFar driver]?", the advertisement implies that the use of the club will add
"power ... and distance" and, hence, will improve one's play.

(B) CORRECT. The advertisement states that the 12 major championships winner have recently
switched to the new driver. There is nothing in the passage to imply that any of them were us-
ing the driver at the time of their victories; hence, this claim is neither implied, nor made explic-
itly, in the passage.

(C) It is a reasonable assumption that professional golfers, particularly those skilled enough to
win a major championship, are experts and know what constitutes a great club. The advertise-
ment makes this implication and reinforces it by citing the recent decision of these golfers to
switch to the HitItFar driver; this provides an implicit expert endorsement for the HitItFar driver.

(D) The point of the advertisement is to prompt the reader into purchasing a new driver to re-
place his or her old driver in order to "add power and distance"; this implies that the new driver
is superior to the reader's existing driver.

(E) This claim is explicitly stated in the first sentence of the passage.

4.

The researchers claim that Delta-32 prevents its carriers from contracting the Plague. They
support this claim by noting that a strikingly large percentage of descendants of Plague survi-
vors carry the mutation. We are asked to find an assumption underlying the claim.

(A) The argument is specific to the relationship between Delta-32 and resistance to the Plague.
Other diseases are irrelevant.

(B) Again, the argument is specific to the relationship between Delta-32 and resistance to the
Plague. Other diseases are irrelevant.
(C) Delta-32 may have existed in its current form before the sixteenth century and the merit of
the argument would not change.

(D) The argument does not claim that Delta-32 prevents all bacteria-caused disease.

(E) CORRECT. The researchers claim that Delta-32 prevented its carriers from contracting the
Plague on the basis of its presence in descendants of Plague survivors. But it is theoretically
possible that these descendants carry the mutation Delta-32 because the Plague mutated the
genes of their ancestors. In order to claim that the mutation prevented the Plague, we must
assume that the Plague did not cause the mutation Delta-32.

5.

The author concludes that one will only be able to determine the age of a Brazilian ash by
counting its rings if the temperature in the tree's environment never exceeds 95 degrees Fahr-
enheit. The author bases this conclusion on the fact that the tree loses rings when the tempera-
ture exceeds that level. However, if the number of rings lost by a Brazilian ash at high tempera-
tures can be predicted, it may be possible to determine the age of a tree even if the tempera-
ture exceeds 95 degrees.

(A) The argument says nothing about precipitation. This answer choice is out of scope since it
would require a number of other assumptions to make it relevant to the argument's conclusion.

(B) Whether other trees share this feature is irrelevant; the argument focuses only on the Bra-
zilian ash.

(C) The number of days of excessive heat needed to cause the tree to lose rings is irrelevant.

(D) The thickness of the rings is irrelevant.

(E) CORRECT. The conclusion is that the rings will be a reliable measure only if the temperature
never exceeds 95 degrees. This is true only if there is no way to predict how many rings would
be lost when the temperature does exceed 95 degrees. (If it were possible to predict this, one
might be able to assess the age of a tree using its rings even if the temperature had exceeded
95 degrees.)

6.

The conclusion of this argument is that the national identification system (―using licenses for
purposes not directly related to operating a motor vehicle‖) is un-American. The basis for this
claim is that such a system would allow the government to restrict the liberty of its people. The
necessary assumption is one that connects restrictions on liberties to the concept of ―un-
American‖ policies.

(A) The author never mentions future presidential elections, or the role of the president in such
a national identification system. Therefore, the conclusion that the national identification system
is un-American does not depend on this assumption.

(B) Whether the government will soon, or will ever, start curtailing the activities of dissidents is
irrelevant to this argument: that the national identification system is un-American simply be-
cause it restricts the liberties of U.S. citizens. Even if the government does not abuse the power
the national identification system provides, the system could still be considered un-American.

(C) CORRECT. This choice connects the concept of "un-American" policies to restrictions on lib-
erties, essentially defining blanket restrictions on citizens as un-American.

(D) Whether Americans are willing to give up their right to travel freely is irrelevant to this ar-
gument: that the national identification system is un-American simply because it restricts the
liberties of U.S. citizens. Even if Americans were willing to give up their right to move about
without identification, the system could still be considered un-American.

(E) While the author may be inclined to agree that Americans should resist the government
regulation of their lives that the national identification system represents, this argument does
not depend on such an assumption. In fact, the author makes a distinction between the nation-
al identification system and ―licenses for purposes…directly related to operating a motor vehi-
cle,‖ so it is possible that the author considers some government regulation reasonable.

7.

The argument presents the facts of an apparent change in a magazine's cover features since
the new publisher took control. While a gossip columnist hailed the change, newspaper editori-
als disagreed and concluded that the publisher favored profit over reporting. The editorials are
the opponents of the gossip columnist; since their conclusion is about the publisher’s desires,
there must be an assumption connecting the publisher to the covers.

(A) This choice is irrelevant, as it is not connected to the conclusion. The activities of celebrities
have nothing to do with the publisher’s interests.

(B) CORRECT. Since the conclusion concerns the publisher’s desires based on the content of the
magazine covers, the editorials have to assume that the publisher decides who is to be a cover
subject. If not, there is no connection between the covers and the publisher’s interests.

(C) This choice is the opposite of a necessary assumption. For the editorials to conclude that
the publisher prefers profits to reporting, they have to assume that the two are mutually exclu-
sive.

(D) ―Some‖ means ―at least one,‖ so this is not a powerful statement in any direction. Further-
more, even if several such stars were running for political office, it is not at all necessary to as-
sume that to conclude that the publisher was more interested in profits.

(E) This choice is not correct. While it is true that the editorials must assume model and movie
star covers are likely to sell more copies, it does not have to be assumed that such covers will
result in the sale of triple the number of copies, or any other specific number.

8.
The conclusion of the argument is that renewable sources of energy, chiefly solar and wind, will
be less risky for certain utilities than nonrenewable sources, such as oil and gas. The basis for
this claim is that the renewable sources will provide stable, low-cost supplies of energy, where-
as the prices for nonrenewable sources will fluctuate according to availability. We are asked to
find an assumption underlying this argument. In order for this argument to be valid, it must in
fact be true that these renewable sources of energy will provide stable, low-cost supplies.

(A) The utility companies' claim has to do with the supply risk of the new energy sources, not
with how these sources are received by the public.

(B) If no new supplies of traditional energy sources are found, then it is true that perhaps these
nonrenewable supplies will continue to fluctuate in price in a risky manner. However, the argu-
ment does not depend upon any assumption about the future discovery of oil and gas supplies.

(C) CORRECT. If we assume that weather patterns are consistent and predictable, then with the
stated premises, we can conclude that solar and wind power will be less risky than oil and gas.
If, on the other hand, weather patterns are not consistent and predictable, then solar and wind
power are not reliable and thus will not provide "stable energy supplies at low cost." Thus, the
argument's conclusion directly depends on this assumption.

(D) To reach the required conclusion, it is not necessary to assume that the conversion technol-
ogy for new sources is not more expensive than the present technology.

(E) This choice does not directly affect the argument. Whether or not energy produced through
combustion can be made less risky, the new energy sources might still be less risky than the
older sources.

9.

We are given two premises based on survey results: first, vanilla is the best-selling flavor of ice
cream and, second, those who prefer chocolate usually don't order vanilla. The author con-
cludes that vanilla-flavored candy should sell better than chocolate-flavored candy. The author
bases this conclusion on the assumption that it is valid to extend the survey's results beyond ice
cream to include candy.

(A) As a stand-alone, this choice makes common sense but, in the argument, it would under-
mine the author's conclusion. We are asked to find an assumption upon which the author re-
lies, which means the correct assumption should support the author's conclusion.

(B) This choice is either irrelevant at best (the survey does not address children specifically) or
would undermine the author's conclusion, at worst. We are asked to find an assumption upon
which the author relies, which means the correct assumption should support the author's con-
clusion.

(C) Preferences for flavors neither vanilla nor chocolate are outside of the scope of this argu-
ment.

(D) This choice addresses only ice cream preferences; it does not provide any information to tie
ice cream preferences to candy preferences.

(E) CORRECT. This assumption supports the author's conclusion by tying ice cream preferences
directly to candy preferences.

10.

The conclusion of the argument is that the media are wrong in saying that the economy is en-
tering a phase of growth and prosperity. The basis for that claim is that the number of people
filing for bankruptcy has increased every month for the last six months and that bankruptcy
lawyers are busier than they have been in years. In order for this argument to be valid, howev-
er, the author has to assume that the increase in the number of bankruptcies is a result of the
state of the economy and not the result of something unrelated.
(A) This statement does not have to be true for the claim that the media are wrong about the
economy to hold. Even if unemployment rates are useful indicators of growth and prosperity,
the media could still be wrong about the economy (e.g., if there are other indicators that show
problems in other areas).

(B) This does not have to be true for the conclusion to hold. Productivity could be a good
measure of economic growth, but the media could still be wrong about the economy (e.g., if
there are other indicators that show problems in other areas).

(C) CORRECT. This has to be true for the conclusion to hold. If legislation has recently been
passed that makes it easier to obtain bankruptcy, then the evidence cited would be less rele-
vant. The increased number of bankruptcies could have been the result of the easier process
rather than of a poor economy.

(D) This does not have to be true for the conclusion to hold. An increase in the number of
bankruptcy lawyers would not explain the increase in the number of bankruptcy filings.

(E) This does not have to be true for the claim that the media are wrong about the economy to
hold. Even if the media did not often misrepresent the current state of economic affairs, the
argument that the media are wrong might still hold.

12.

The correct answer is B.

The conclusion of the argument is that Michelangelo must have completed the painting be-
tween 1507 and 1509. The basis for that claim is that the painting depicts a coin that did not
exist before 1507 and that it contains a pigment that Michelangelo ceased using in 1509. We
are asked to find an assumption that completes the logic of this argument.

Choice A is incorrect. We do not need to assume that no stocks of the pigment existed after
1509. The argument is concerned only with the year in which Michelangelo stopped using the
pigment.
Choice B is correct. In order to conclude that the painting must have been completed before
1509 on the basis of the pigment, we must assume that he did not begin the painting before
1509 using the old pigment and complete the painting after 1509 with the new pigment.

Choice C is incorrect. The fact that the general public knew of the coin in 1507 is irrelevant to
the conclusion.

Choice D is incorrect. The fact that the panel cannot be tested for age does not relate to either
the coin or the pigment, the two bases for the conclusion.

Choice E is incorrect. Whether Michelangelo's painting style changed during this period does not
relate to either the coin or the pigment.

12.

The conclusion of the argument is that "Company X's fuel costs this year will be significantly
higher than they were last year. Why? Because some of the company's plants switched from oil
to natural gas when the price of gas was lower, and now the price of gas has outstripped the
price of oil. We are asked to find an assumption that is necessary for the argument to work.

(A) Whether Company X has the money to cover its costs does not affect the amount of those
costs.

(B) We do not need to assume that the costs cannot be offset by reducing expenditures in oth-
er areas in order for Company X's costs to be higher.

(C) We do not need to assume that gas will never be cheaper than oil in order for Company X's
costs to be higher.
(D) CORRECT. The author does not take into account the fact that only "some" of the compa-
ny's plants converted to natural gas. Some of the plants, then, still use oil, which is now cheap-
er. So in order to conclude that the company will have to spend more on fuel, the author must
assume that the extra cost of the natural gas for the plants that converted is at least as much
as the cost of the oil for the plants that did not.

(E) We do not need to assume that the price of oil will not suddenly rise in order for the argu-
ment to work.

13.

The advertisement discusses the merits of Avian Oculars, explaining that they are reasonably
priced and contain several popular features. The advertisement concludes, however, by encour-
aging readers to use Avian Oculars in order to see some of the world’s rarest bird species. A
proper assumption must reasonably connect these diverse ideas.

(A) The argument states that Avian Oculars are lightweight, but makes no direct comparison to
traditional binoculars.

(B) Although the advertisement is geared toward bird watchers, nothing indicates that Avian
Oculars should be used only by bird watchers. Even if Avian Oculars were used by individuals
other than bird watchers, no information in the advertisement would be undermined.

(C) CORRECT. The conclusion of the advertisement is that, by using Avian Oculars, the reader
will see some of the world’s rarest bird species on his or her next bird watching trip. In order for
this to be true, the reader would have to have access to these rare bird species. Even the best
bird watching technology would prove useless to view a particular species of bird if the species
were not present.

(D) This choice is true, but it is not an assumption. Remember, an assumption is an unstated
piece of information that ties a conclusion to its premises. This particular answer choice is simp-
ly a stated premise from the second sentence of the argument.
(E) This argument presents one specific product, Avian Oculars, which is specifically designed
with birding enthusiasts in mind. From this information, we cannot assume that birding enthusi-
asts themselves determined the specifications of Avian Oculars, nor can we make any assump-
tions about who determines the specifications of other products.

14.

The basic structure of this argument is fact that ―mold is almost always found in places where
there is substantial moisture,‖ so therefore, to avoid mold and the resultant mold poisoning,
then people should take steps to prevent wet areas. This argument assumes that wet areas oc-
cur first, causing mold to grow. Conversely, this assumption requires that the mold growth itself
does not occur first, creating wet areas as a result.

(A) CORRECT. The argument depends on the assumption that the reason mold and wetness are
observed together is that wet areas cause mold growth. If the reverse causation (mold causes
wetness) were true, then keeping all plumbing in good condition to prevent leakage would do
little to prevent the growth of mold. This choice eliminates the alternate causation.

(B) If most homeowners know enough about plumbing to determine whether theirs is in good
condition, then the recommendation made in this argument would be more useful. However,
this is not an assumption on which the argument depends.

(C) Even if mold could grow in dry areas, the fact that mold is almost always found in wet areas
is still valid. This is the fact upon which the argument is based, so the argument does not de-
pend on the unnecessarily absolute assertion that mold cannot grow in dry areas.

(D) Even if some varieties of mold are harmless, the conclusion of this argument, that ―one
should make sure to keep all internal plumbing in good condition to prevent leakage‖ and mini-
mize mold growth, could still be valid. Therefore, this argument does not depend on the unnec-
essarily absolute assertion that no varieties of mold are harmless.

(E) Whether mold spores can be filtered from the air may be relevant to a conclusion about the
health effects of mold in the home, but it is not directly relevant to this conclusion, that ―one
should make sure to keep all internal plumbing in good condition to prevent leakage‖ and mini-
mize mold growth.
15.

The conclusion of the argument is that the majority of American citizens believe in the death
penalty while the majority of Filipino citizens do not. This conclusion is based solely on the fact
that the death penalty is legal in the United States while it is now illegal in the Philippines. The
argument assumes that there is a link between the legal status of capital punishment and the
beliefs of the majority of citizens.

(A) While the number of murders per year might influence beliefs about capital punishment,
there is certainly no necessary correlation. Moreover, the difference in the number of murders
per year in the United States and the Philippines has little relevance without knowing the rela-
tive populations of the two countries.

(B) CORRECT. It is possible that the legal status of capital punishment in the United States and
the Philippines does NOT align with how the majority of citizens in those countries view the
death penalty. For example, it may be that the governing bodies of one or both countries are
out of touch with the views of the populace. The argument assumes that this is NOT the case.

(C) Even if there were strong voices opposing the death penalty in the United States, the argu-
ment might still hold. The argument is based on the views of the majority of citizens in the
United States, not on all US citizens.

(D) The argument addresses whether or not citizens of the United States and the Philippines
believe in the death penalty. The reasons behind those beliefs, as presented in this choice, are
irrelevant to the logic of the argument.

(E) The argument is centered on whether or not citizens of the United States and the Philip-
pines believe in the death penalty. While the legal standard used to sentence criminals to the
death penalty might impact why individuals hold certain beliefs, it has no impact on the logic of
the argument.

16.
Although the premises of this argument suggest only a correlation between smoking and anxie-
ty or nervousness, the argument has a causal conclusion: it concludes that smoking causes in-
dividuals to be anxious and nervous (i.e., that A causes B). Any assumption in a causal argu-
ment must support the causal ―direction‖ of the conclusion, that A causes B as opposed to some
other explanation. Often, assumptions support a causal conclusion either by eliminating an al-
ternate cause for the conclusion (that C did not cause B) or by demonstrating that the causa-
tion, if one exists, is in the proper direction (that B did not cause A).

(A) The argument concludes that smoking causes anxiety and nervousness. Whether these
maladies lead to more serious health problems is not relevant to the conclusion.

(B) CORRECT. For smoking to be the cause of anxiety and nervousness (i.e., that A caused B)
it must be true that these individuals were not more likely to be anxious and nervous before
they started smoking. If smokers had these preconditions, which contributed to their decision
to begin smoking (i.e., that B caused A), our conclusion – that smoking causes these maladies –
would be incorrect.

(C) The argument concludes that smoking causes anxiety and nervousness. The number of sur-
vey respondents is not relevant to the conclusion.

(D) The argument concludes that smoking causes anxiety and nervousness. The awareness of
the health problems related to smoking is not relevant to the conclusion.

(E) The argument is not based on the immediate impact that smoking has on anxiety and nerv-
ousness. Moreover, the argument never compares some smokers to other smokers.

17.

The argument concludes that the stock of the firm will experience rapid growth. The basis for
this claim is that the firm has shown strong historical performance that is likely to continue in
the future. The stock will appreciate dramatically in the future as a result only if it has not al-
ready appreciated in anticipation of the company's expected growth.
(A) The argument focused on the potential for stock appreciation rather than company weight
in the industry. A company with a large market share may well experience poor stock perfor-
mance, while a company with a small market share may continue to grow and increase in val-
ue.

(B) Since the conclusion of the argument is made regarding the future outlook, it is not neces-
sary to assume that the company had been growing, or had even existed, prior to the past 5
years. A new firm that has been in existence for only 5 years may well present an excellent in-
vestment opportunity.

(C) This statement is explicitly stated in the argument and therefore does not have to be as-
sumed.

(D) CORRECT. If this assumption were not true, i.e. if the current stock price already reflects
future growth prospects, then the premise that the company will experience high growth is cer-
tainly insufficient to warrant future stock price appreciation, since all of this growth would al-
ready be reflected in the current price. It is necessary to assume that the current price of Com-
pany X stock does not yet reflect the promising growth prospects of the firm, allowing the pos-
sibility that the stock price will rise further.

(E) Note that the argument makes a claim about the absolute return of stock X rather than its
return relative to the industry. Therefore, to justify the rapid growth in the stock price, it is not
necessary to assume that the company will outperform its competitors. For example, if the in-
dustry itself is growing very rapidly, other companies in the industry can experience just as rap-
id appreciation in stock prices.

18.

Antoine is alarmed that the number of children on antipsychotic medication has increased by 73
percent. Lucy begins her reply with ―but,‖ indicating that she is about to counter either An-
toine’s facts or his alarm; she accepts his facts but addresses his alarm. If the number of chil-
dren taking antipsychotic medication is still within the normal range, the rate at which the total
number has increased is not cause for alarm. Lucy uses information about adult use of such
drugs to imply that the lower rate of antipsychotic drug use in children must also be normal.
(A) Lucy’s argument is about a normal level of antipsychotic drug use; how rarely or frequently
that level is exceeded is outside the scope of her argument.

(B) Lucy uses the percentage of adults taking antipsychotic medication to illustrate normal lev-
els of the use of such drugs. It happens that the percentage of children taking such medication
last year was lower than the percentage of adults, but her argument does not require the as-
sumption that that will always be the case.

(C) CORRECT. If there is no difference between children and adults on the matter of antipsy-
chotic drug use, then Lucy can legitimately use information about adult use of such drugs to
imply that the lower rate of antipsychotic drug use by children must also be normal. On the
other hand, if this assumption were not valid – for example, if children responded differently to
the drugs, or if the rate of the drug use by adults is considered too high for children – then Lu-
cy’s statement would not be enough to address Antoine’s alarm.

(D) Lucy’s argument is not based on the figure Antoine cites and does not assume its accuracy
or inaccuracy. Rather, her argument uses the relative adult and child rates of antipsychotic drug
use to point out that Antoine’s statistic is not inconsistent with a normal rate of such use in chil-
dren.

(E) The fear of random violence by adolescents is not part of Lucy’s argument; this statement is
irrelevant.

19.

The question asks for an assumption made by the reader. The reader’s conclusion is that aca-
demic rigor is in decline, based on the percentage of colleges granting a majority of their de-
grees in the liberal arts mentioned in the article. To draw this conclusion, the reader must as-
sume that degrees not in the liberal arts were not as academically rigorous.

(A) It is not necessary for the reader to assume that the percentage will continue to drop. The
reader's conclusion concerns the present. Assumptions must be both unstated and necessary.
(B) This extreme statement is not a necessary assumption. The reader does not have to as-
sume that all colleges should do so; the conclusion only relies on an assumption that 5.5% is
too low.

(C) CORRECT. To conclude that the low percentage of colleges granting the majority of their
degrees in the liberal arts indicates a decline in academic rigor, the reader must assume that
other degree programs required less academic rigor. If not, this evidence would not indicate a
decline in academic rigor.

(D) This is not a necessary assumption. The relative importance of academic rigor is irrelevant
to the reader’s claim. That claim only asserts that academic rigor, in isolation, is in decline. The
claim has nothing to do with its importance relative to other attributes.

(E) It is not necessary to assume anything specific about the schools that do not grant a majori-
ty of their degrees in the liberal arts, as they are not the subject of the evidence or the conclu-
sion. The reader feels that the low percentage mentioned is evidence enough; it is not neces-
sary to assume any arbitrary level below the 50% of degrees standard that the article and the
reader use.

20.

The doctor concludes that federal legislation prohibiting the sale of video games to minors
would help reduce the incidence of carpal tunnel syndrome. This conclusion hinges on the as-
sumption that the only way for adolescents to access video games is to purchase the games
themeslves.

(A) Majority consensus in the legislature has no bearing on whether the recommended legisla-
tion would actually help to curb carpal tunnel syndrome.

(B) This argument states that "adolescents who play video games on a regular basis are three
times as likely to develop carpal tunnel syndrome." Thus, the argument directly indicates that
carpal tunnel syndrome does not affect all adolescents who play video games. Rather than an
assumption, this answer choice is simply an inference drawn from the text.
(C) The fact that adolescents can develop carpal tunnel syndrome by means other than playing
video games has no bearing on whether the recommended legislation would help to curb carpal
tunnel syndrome.

(D) CORRECT. In order for the doctor's recommended legislation to reduce the incidence of
carpal tunnel syndrome among adolescents, the prohibition from the purchase of video games
must result in the actual possession of fewer video games. Thus, it must be assumed that par-
ents will not simply purchase video games for their children.

(E) The fact that video games can benefit adolescents in other ways has no bearing on whether
the recommended legislation would help to curb carpal tunnel syndrome.

21.

The management concludes that a 10% increase in gasoline prices will result in a 10% increase
in revenues from the sale of gasoline. In order to reach this conclusion, we need to assume that
the amount of gasoline sold will not drop despite the higher prices.

(A) CORRECT. This assumption is critical to justify the projection that a 10% increase in gas
prices will result in a 10% increase in revenues from gasoline sales. Note that if this assumption
does not hold, the management’s projection will collapse. For example, if consumers switch to
public transportation or simply start to drive less in response to the higher prices, the revenues
of the company will not increase by the same amount as the increase in the sales price. In fact,
if the decline in gasoline consumption is substantial (e.g. 20%) the company will experience
lower rather than higher revenues.

(B) The issue of profits is irrelevant to the management’s conclusion about revenues from the
sale of gasoline.

(C) Since the management’s projection concerns only the sales of gasoline, revenues of other
business lines are beyond the scope of the argument.
(D) Since the management’s conclusion concentrates on revenues, the issue of costs is beyond
the scope of the argument.

(E) It is not necessary to assume that the supply of gasoline will decline, since the price in-
crease can be driven by a variety of other factors, such as production costs, market environ-
ment, and others.

22.

The conclusion of the argument is contained in the last sentence, that "the methods that the
prospector had used to determine the size of the oil deposit must have been inaccurate." The
evidence provided is that the prospector reported a large oil deposit that was later determined
to be much smaller in size. We are asked to find an unstated assumption that makes the con-
clusion valid based upon this evidence. In order to do this, we need to assume that there is not
another reason why the prospector might have reported a larger oil deposit than actually exist-
ed.

(A) It is not necessary to the conclusion that a third party affirmed the company's determina-
tion that the oil deposit turned out to be small. The conclusion accepts that the oil deposit was
indeed smaller than indicated by the prospector, and focuses on the cause of the discrepancy
as opposed to the discrepancy itself.

(B) CORRECT. The argument concludes that the prospector's methods resulted in inaccurate
measurements of the size of the oil deposit. This assumes that the prospector did not simply
misreport or misrepresent the measurements, presumably for personal gain. This answer
choice addresses the most plausible alternative explanation to the one given, and is necessary
for the conclusion to stand based upon the evidence presented.

(C) The commercial feasibility or profitability of the oil deposit is not integral to the argument,
or its conclusion. This statement is not an assumption that would support the conclusion.

(D) Whether or not the prospector utilized the same methods in regards to measuring the oil
deposits in other locations is not relevant to the argument, or the conclusion.
(E) The fact that the company had a long operating history and experience in drilling oil wells
is not relevant, in that the company's measurements of the size of the oil field are accepted as
given in the argument.

The correct answer is B.

23.

This argument concludes that the city needs educational leadership that can solve problems,
not create them. It illustrates this claim by discussing the prohibition on cell phones. This prohi-
bition is given as an example of the leadership creating problems where none exist. The neces-
sary assumption is one that bridges the logic gap between the premise and the conclusion.

(A) CORRECT. In order to use the issue of students having cells phones as an example of how
the educational leadership creates problems where none exist, the author must assume that
students having cell phones is not an important problem.

(B) The argument and its conclusion are not about students’ needs. They concern the quality of
educational leadership. Thus, this choice is irrelevant; it is not necessary for the argument to
assume anything about students’ needs.

(C) This choice is irrelevant; the argument does not concern faculty and staff cell phones, and
thus no assumption about them is necessary. Assumptions must be both unstated and neces-
sary to the conclusion.

(D) The argument and its conclusion are not about students’ needs. They concern the quality of
educational leadership. Thus, this choice is irrelevant; it is not necessary for the argument to
assume anything about students’ needs.

(E) The argument does not rank the various attributes of good educational leadership. It only
discusses one quality. Thus, no assumption about the relative importance of attributes is neces-
sary, and this choice is irrelevant.
24.

The argument proposes that directing chiropractic treatment toward adults with severe back
problems is the best way to minimize the proportion of the population that suffers from back
pain. The argument assumes a high degree of overlap between those adults who suffer from
severe back problems and all people who suffer from back pain, where the former group may
comprise only a subset of the latter group. It also assumes that there is not another recom-
mendation that could better accomplish the goal of minimizing back pain for a larger segment
of the population than treating just those adults with severe back problems.

(A) The argument does not depend upon whether or not chiropractic treatment can or cannot
be used in conjunction with other medical treatments. The fact that people could benefit from
other forms of treatment aside from chiropractic care weakens the argument to a slight degree,
and does not act as a supportive assumption.

(B) The relative degree of insurance coverage of chiropractic care compared with other medical
treatments is not vital to the argument. Also, that insurance carriers cover chiropractic care to a
lesser degree weakens the argument marginally, and does not act as a supportive assumption.

(C) CORRECT. This statement rules out the possibility that chiropractic care or other medical
treatments could effectively prevent or lessen back pain among those people who have not yet
developed severe back problems.

(D) That chiropractic treatment is more effective over time is irrelevant to the argument.

(E) The economic impact of severe back pain and other problems is not addressed in the argu-
ment. Instead, addressing back pain is treated as an intrinsic goal, with no mention of economic
externalities.

25.

The argument presents the facts of an apparent change in a magazine's cover features since
the new publisher took control. While a gossip columnist hailed the change, newspaper editori-
als disagreed and concluded that the publisher favored profit over reporting. The editorials are
the opponents of the gossip columnist; since their conclusion is about the publisher’s desires,
there must be an assumption connecting the publisher to the covers.

(A) This choice is irrelevant, as it is not connected to the conclusion. The activities of celebrities
have nothing to do with the publisher’s interests.

(B) CORRECT. Since the conclusion concerns the publisher’s desires based on the content of the
magazine covers, the editorials have to assume that the publisher decides who is to be a cover
subject. If not, there is no connection between the covers and the publisher’s interests.

(C) This choice is the opposite of a necessary assumption. For the editorials to conclude that
the publisher prefers profits to reporting, they have to assume that the two are mutually exclu-
sive.

(D) ―Some‖ means ―at least one,‖ so this is not a powerful statement in any direction. Further-
more, even if several such stars were running for political office, it is not at all necessary to as-
sume that to conclude that the publisher was more interested in profits.

(E) This choice is not correct. While it is true that the editorials must assume model and movie
star covers are likely to sell more copies, it does not have to be assumed that such covers will
result in the sale of triple the number of copies, or any other specific number.

26.

The argument concludes that for-profit colleges enroll a greater proportion of financially disad-
vantaged students than do non-profit colleges. This conclusion is based on the fact that stu-
dents at for-profit colleges draw a disproportionate share of federal and state financial aid. The
argument assumes a link between the proportion of aid received and the proportion of finan-
cially disadvantaged students enrolled. In so doing, it assumes that there are not other possible
reasons for the disproportionate aid distribution.
(A) The conclusion makes a claim about the differences between for-profit and non-profit col-
leges. Differences among non-profit colleges – such as public vs. private – are irrelevant to the
argument.

(B) CORRECT. One alternative reason that might explain the disproportionate aid distribution is
that for-profit colleges engaged in fraudulent practices to obtain unneeded financial assistance
for their students. If this were true, then much of the aid was distributed based not on the ac-
tual financial situation of the students but on the ability of colleges to defraud federal and state
governments. This answer choice asserts that this was NOT in fact the case, thereby eliminating
this alternative explanation and highlighting a key assumption upon which the argument rests.

(C) The argument's claim is centered on proportions. The actual number of students receiving
aid at for-profit vs. non-profit colleges is irrelevant to the conclusion.

(D) The relative educational quality of for-profit vs. non-profit colleges lies outside the scope of
the argument, which is focused solely on differences in financial aid distribution.

(E) The issue addressed by the argument is the amount of financial aid distributed to students
at two types of institutions. Whether students successfully repay their loans after college is im-
material to the claim made in the argument.

27.

Research indicates that there is a connection between being married and being happy and
healthy. Media commentators have concluded that marriage causes happiness and health.
However, one could reasonably conclude from the research that the cause and effect are the
reverse: being happy and healthy makes a person more likely to get married.

(A) The research compared married people to unmarried people. Neither the researchers nor
the media commentators made any distinction between newlyweds and those who had been
married a long time, so this assumption is not necessary.
(B) The type of wedding is outside the scope of this argument. The research compared married
people to unmarried people, but made no distinction based upon the type of wedding. Thus,
this assumption is unnecessary.

(C) At first, this statement may seem necessary—after all, if the commentators conclude that
marriage causes happiness, a lack of depression in married people would certainly support that
conclusion. However, the statement is too extreme. One depressed married person does not
invalidate the research indicating that, on average, married people are healthier and happier
than non-married people.

(D) CORRECT. This statement eliminates the alternative interpretation of the research find-
ings—that being happy and healthy makes a person more likely to get married.

(E) The research compared married people to unmarried people. Neither the researchers nor
the media commentators made any distinction between harmonious marriages and combative
marriages, so this assumption is not necessary.

28.

The text tells us only that Country X imposes heavy tariffs on imported goods and that Compa-
ny Y believes it can increase long-term profits by opening a factory in Country X so it can avoid
having to import its goods into Country X. We are asked to select an answer choice that is an
assumption required for Company Y's belief to be valid.

(A) While this is a tempting answer, it is not necessary to assume that Company Y will be able
to obtain all necessary permits. The text does not indicate whether Company Y will actually be
able to implement the plan, only that the plan could increase profits if implemented.

(B) We are given no information about Company Y's activities in other countries.

(C) CORRECT. In order for Company Y to conclude that it can increase long-term profits by
opening a factory in Country X, it must believe that a sustainable market exists for its products
in that country. Otherwise, the new factory would not generate revenue and the company could
not recoup the cost of the new factory.

(D) We are given no information about tariffs in Company Y's home country.

(E) We need not assume that labor costs are lower in Country X. It could be that labor costs in
Country X are higher than those in Company Y's home country but the increased cost of labor is
still less than the tariffs. This would result in a net savings for Company Y in Country X.

29.

The text tells us only that Country X imposes heavy tariffs on imported goods and that Compa-
ny Y believes it can increase long-term profits by opening a factory in Country X so it can avoid
having to import its goods into Country X. We are asked to select an answer choice that is an
assumption required for Company Y's belief to be valid.

(A) While this is a tempting answer, it is not necessary to assume that Company Y will be able
to obtain all necessary permits. The text does not indicate whether Company Y will actually be
able to implement the plan, only that the plan could increase profits if implemented.

(B) We are given no information about Company Y's activities in other countries.

(C) CORRECT. In order for Company Y to conclude that it can increase long-term profits by
opening a factory in Country X, it must believe that a sustainable market exists for its products
in that country. Otherwise, the new factory would not generate revenue and the company could
not recoup the cost of the new factory.

(D) We are given no information about tariffs in Company Y's home country.
(E) We need not assume that labor costs are lower in Country X. It could be that labor costs in
Country X are higher than those in Company Y's home country but the increased cost of labor is
still less than the tariffs. This would result in a net savings for Company Y in Country X.

30.

The conclusion of the argument is that companies should allow other manufacturers to license
patented technology. The basis for that claim is that not doing so keeps prices high and harms
the consumer. We're asked what the argument assumes ("presupposes") in drawing its conclu-
sion. The correct answer will fill the logic gap between the idea that keeping prices high harms
the consumer and that companies should allow other manufacturers to license patented tech-
nology. The conclusion is based on the assumption that companies have an obligation of some
kind to do what's best for the consumer.

(A) This does not address the moral obligation to the consumers (i.e. ―should‖) of the compa-
nies who produced the patented technology, the main point of the conclusion. Furthremore,
even if companies could find legal ways to produce similar technologies, the patented technolo-
gy could still command exorbitant prices, thereby harming the consumer.

(B) CORRECT. The conclusion only makes sense if companies have an obligation to act in the
best interest of the customer, as this choice states.

(C) This generally follows along with the author's claim, but we are not required to assume this
in order to reach the conclusion that companies who are granted patents are obligated to look
out for the best interests of their customers.

(D) This addresses a tangential issue of whether or not consumers could notice the difference
between a new patented technology and a possible imitation. This does not address the core
issue of the obligation to the consumer.
(E) This does not address the obligation of the companies toward the consumers, or indeed the
companies at all.

31.

The conclusion of the argument is that one need not worry about ingesting pesticides when
purchasing produce from farms that use only organic pesticides. The basis for that claim is the
fact that organic pesticides leave the surface of produce within a few hours of spraying. In or-
der for this argument to be valid, we must assume that the organic pesticides do not harm the
produce in any lasting way.

(A) This is unrelated to the argument since the conclusion speaks about not having to worry
about ingesting produce on which only organic pesticides were known to be used.

(B) If anything, this statement runs counter to what the argument is saying. If produce that has
been sprayed with organic pesticide reaches the final consumer within hours after it is picked, it
is possible that the consumer does need be concerned about the pesticides.

(C) The conclusion of the argument is already limited to those farms which use "only organic
pesticides."

(D) CORRECT. If a pesticide is capable of penetrating the skin of a fruit or vegetable then, while
the organic pesticide will dissipate from the surface of the fruit in a few hours, it may remain
inside the fruit. The author of this argument assumes that the pesticides cannot penetrate the
skin.

(E) The issue of cost is unrelated to the argument's conclusion about pesticide residues.

32.
The conclusion of the argument is that the prediction of decreased consumer spending has not
come to pass. The evidence for this is that there has been no corresponding increase in the
amount of money set aside in savings accounts by the general public. This question asks us to
find an assumption on which this argument is based. The author assumes that the mere fact
that people generally have not been adding more money to their savings accounts means they
have not cut down on their discretionary purchases (i.e., non-essential items). In order for this
to be valid, we must assume that a savings account is the only mechanism by which someone
would save any extra money when reducing discretionary spending.

(A) The state of the economy in the last few months has no bearing on the claim that a lack of
extra money deposited in consumer savings accounts is evidence that consumer spending has
not decreased.

(B) CORRECT. If there was an alternate explanation for the lack of increase in savings accounts,
the claim that the decrease in spending has not taken place would be greatly weakened. This
assumption guarantees us that at least one other possible explanation for the lack of increase in
savings (i.e. an increase in stocks, certificates of deposit, or other savings vehicles) is NOT true.

(C) Assuming that salaries decreased would actually weaken this argument. If salaries de-
creased, that may explain the decrease in the amount of money being put aside in savings
without necessarily implying a decrease in spending. People could be making less, spending the
same, and therefore saving less.

(D) The lack of growth in business sectors has nothing to do with what consumers are doing
with their money during the same time period.

(E) The argument says the prediction was made "even [for those whose] jobs were secure."
People who find other employment, therefore, would still fall under the economists' prediction
and would, if the prediction were true, be expected to cut discretionary spending and save
more money.

33.
The conclusion is that nurses should examine patients to determine which deserve to be seen
first by the doctors. The basis for this claim is that hospitals lack adequate numbers of physi-
cians.

(A) The idea of having nurses make the initial examination does not depend on increasing the
medical staff.

(B) The main premise for the conclusion was that patients ended up waiting due to an under-
supply of doctors. There weren't enough doctors to perform the initial examination. If the doc-
tors perform the initial examinations there will be no time saved.

(C) The conclusions rests on whether or not the nurses would be able to perform the examina-
tions, not on what the result of them doing the examinations would be.

(D) The hospitals don't need to be fully staffed with nurses for the nurses to perform the initial
examination.

(E) CORRECT. This argument is valid only if we assume that nurses are competent to determine
which patients merit immediate treatment.

The correct answer is E.

34.

The correct answer is B. The conclusion of the argument is that "the scientists must have dis-
covered the butterfly at night." Why? Because the butterfly's color matches the green of the
foliage, and the butterfly is active only at night. In order for this argument to work, one has to
assume that there is no way for the scientists to detect the butterfly during the day, despite its
camouflage. If there is a way for the scientists to detect the butterfly during the day, the con-
clusion is no longer logical.
THE MOST AUTHORITATIVE GUIDE EVER WRITTEN ON GMAT CRITICAL REASONING

The Most Authoritative Guide Ever Written on

Critical
Reasoning
Sandeep Gupta

BY SANDEEP GUPTA, GMAT PERFECT 800 | WWW.TOP-ONE-PERCENT.COM 2


THE MOST AUTHORITATIVE GUIDE EVER WRITTEN ON GMAT CRITICAL REASONING

Chapter 1
Boldface Questions
Some quick facts:

1. Most students believe that ‘boldface’ (BF) questions are the toughest type of CR
questions.
2. The surprise: if you master the approach given in this chapter thoroughly, you
will find ‘boldface’ questions the easiest type of CR questions.
3. Using the approach taught in this book, you can solve every single real-GMAT
‘boldface’ question in under 30 seconds.

Let’s dive in.

What is tested in BF questions?

Boldface questions test your skills of identifying the structure of an argument.

Before you understand the structure of an argument, first you need to understand
what we mean by the word argument in Critical Reasoning.

In day-to-day life, we think of an argument as a verbal fight.

But in critical reasoning, the word argument is defined much more precisely.

Suppose in a board meeting, the CEO wants to make the point:

Ours is the most customer-centric company in the country as we received


the fewest number of customer complaints last year among all the players
in our industry.

This may not look like an argument in real life, but in CR, this sentence will definitely
qualify as an argument. This is so because It contains a conclusion (ours is the most
customer-centric company in the country) and a support for the conclusion – called
the premise (we received the fewest number of customer complaints last year
among all the players in our industry).

Notice that each statement by itself can’t be called an argument.

By Sandeep Gupta, GMAT Perfect 800 | www.top-one-percent.com 6


THE MOST AUTHORITATIVE GUIDE EVER WRITTEN ON GMAT CRITICAL REASONING

“Ours is the most customer-centric company in the country” is not an argument.

“We received the fewest number of customer complaints last year among all the
players in our industry” is not an argument.

So, any argument is composed of at least one premise (supporting statement for the
conclusion) and at least one conclusion.

The conclusion is THE ANSWER TO “WHAT” one wants to show, prove, manifest,
or validate. So, when we ask: “WHAT does the CEO want to prove?”, the answer must
be the conclusion. In this case he wants to prove: “Ours is the most customer-centric
company in the country.”

The premise is THE ANSWER TO “WHY” one feels that the conclusion is true / valid.
So, if we ask WHY the CEO feels so (that “Ours is the most customer-centric company
in the country”), we can surely say BECAUSE he feels that “the company received the
fewest number of customer complaints last year among all the players in our
industry.”

• The answer to WHAT one wants to prove is the CONCLUSION.


• The answer to WHY one feels that the conclusion is valid is the PREMISE.

So, in the simplest possible terms, an argument can be visualized as:

P→C
From this point forward, we will refer to this simple relationship as the argument
core, and we will diagram the argument core using a "therefore" arrow.

Argument Core: A premise, or set of premises, used to arrive at a conclusion.

P→C
Arguments that appear on the GMAT are unsound arguments―we can find some
fault with the reasoning upon close inspection. Let’s evaluate the given argument for
its soundness:

By Sandeep Gupta, GMAT Perfect 800 | www.top-one-percent.com 7


THE MOST AUTHORITATIVE GUIDE EVER WRITTEN ON GMAT CRITICAL REASONING

“Ours is the most customer-centric company in the country as we received


the fewest number of customer complaints last year among all the players
in our industry.”

• What if the only way to register a complaint was through a phone call to a
particular number but the company’s phone line was dead for 9 out of the 12
months?
• What if this company received 5 complaints out of 10 customers whereas its
rivals received 100 or fewer complaints out of 10000 or more customers?
• What if the customers are extremely annoyed with the company? This may be
so because whenever a complaint is made, it is not paid any heed. Seeing this
trend, probably people didn’t even bother to make a call to register a complaint
last year even if they were extremely frustrated.
• What if last year the company paid millions of dollars in customer grievances
through out-of-court settlements?
• What if the complaints are registered only when they escalate to the highest
level?
• What if receiving the fewest number of complaints is not a measure of being
customer-centric?

Can you see that the argument given above is unsound?

Let’s look at a sound argument:

The sun rises only on Mondays. → The sun does not rise on Fridays.

Here are two ways to think about it:

The real-world approach: "No way! Terrible argument! We all know that the sun rises
every day, not just on Mondays." This approach is recipe for disaster on the GMAT.

The logical approach: "Well, if we take the premise given (that the sun rises ONLY on
Mondays) as truth, is this enough to substantiate the claim that the sun does NOT rise
on Fridays? Yes. Logically speaking, this argument is sound." This is the only
approach that we need.

By Sandeep Gupta, GMAT Perfect 800 | www.top-one-percent.com 8


THE MOST AUTHORITATIVE GUIDE EVER WRITTEN ON GMAT CRITICAL REASONING

Role of Premise:
• A premise is a statement that supports a conclusion.
o A premise is also called Evidence, Fact, Support, Explanation, or
Consideration.
• The final conclusion of an argument doesn’t support any statement in the entire
argument. The conclusion is also called Position, Opinion, Claim, Hypothesis, or
Prediction.
• Background information is important to give us some context, but isn’t part of the
argument core (P → C).
• Additional premises are statements that also support the conclusion.
• Intermediate conclusion (or sub-conclusion) is a conclusion that is drawn before
the final conclusion (in the logical sequence of events).
• One or more premises can support the intermediate conclusion, which in turn can
support the main conclusion.
• Always remember: in both CR and RC, a fact can be neither supported nor
challenged.
o I repeat: a fact can’t be challenged. A fact can’t be supported either.

In CR arguments, the facts given are taken at the face value.


We are never supposed to judge the validity of the facts at
all. We can never challenge or support a fact, either.

By Sandeep Gupta, GMAT Perfect 800 | www.top-one-percent.com 9


THE MOST AUTHORITATIVE GUIDE EVER WRITTEN ON GMAT CRITICAL REASONING

Argument structure:
Now that you have understood what an argument means, it is time to understand the
structure of an argument, the basis for solving more than 90 percent questions in
Critical Reasoning. If there is one thing that you need to understand well, it is
argument structure.

Identifying the structure of an argument comprises the following:

1. How to identify the various parts in an argument—premise(s), conclusion(s),


intermediate conclusion(s) etc.
2. How to establish the relationship of various parts in the argument (the flow of
the argument).

The two most important words in CR are FLOW and FLAW.

Boldface questions are based on the FLOW of an argument. Almost all the other
question types are based on the FLAW in an argument.

To understand what we mean by FLOW, let’s take one of the most complex
arguments ever tested on the real GMAT:

By Sandeep Gupta, GMAT Perfect 800 | www.top-one-percent.com 10


THE MOST AUTHORITATIVE GUIDE EVER WRITTEN ON GMAT CRITICAL REASONING

Historian: Newton developed mathematical concepts and techniques that are


fundamental to modern calculus. Leibniz developed closely analogous concepts
and techniques. It has traditionally been thought that these discoveries were
independent. Researchers have, however, recently discovered notes of Leibniz’
that discuss one of Newton’s books on mathematics. Several scholars have argued
that since the book includes a presentation of Newton’s calculus concepts and
techniques, and since the notes were written before Leibniz’ own development of
calculus concepts and techniques, it is virtually certain that the traditional view
is false. A more cautious conclusion than this is called for, however. Leibniz’ notes
are limited to early sections of Newton’s book, sections that precede the ones in
which Newton’s calculus concepts and techniques are presented.

Let’s understand this extremely complex argument in detail.

First of all, there are two sides—the historian (who is also the author of the passage)
and the scholars—both putting forward their opinions (conclusions), which are in
opposition to each other.

1. Newton developed mathematical concepts and techniques that are fundamental to


modern calculus: Background information
2. Leibniz developed closely analogous concepts and techniques: Background
information
3. It has traditionally been thought (indicator that this will be challenged) that these
discoveries were independent. (More details on this later)
4. Researchers have, however, recently discovered (fact) notes of Leibniz’ that
discuss one of Newton’s books on mathematics: the word however indicates that
this fact will be used to challenge something
5. Several scholars have argued that (signals that scholars are going to have an
opinion / conclusion)

By Sandeep Gupta, GMAT Perfect 800 | www.top-one-percent.com 11


THE MOST AUTHORITATIVE GUIDE EVER WRITTEN ON GMAT CRITICAL REASONING

6. since (the word since is a support / fact / premise indicator) the book includes a
presentation of Newton’s calculus concepts and techniques: this statement is a
premise used to support the conclusion of the scholars
7. and since (the word since is a support / fact / premise indicator) the notes were
written before Leibniz’ own development of calculus concepts and techniques: this
statement is also a premise used to support the conclusion of the scholars
8. it is virtually certain that the traditional view is false: the opinion / the conclusion
of the scholars
9. A more cautious conclusion than this is called for, however: author’s (historian’s)
opinion / conclusion—the word however indicates that the author (historian) is
challenging the previous conclusion drawn by the scholars.
10. Premise: Leibniz’ notes are limited to early sections of Newton’s book, sections that
precede the ones in which Newton’s calculus concepts and techniques are
presented: this statement is a premise / fact supporting the author’s (historian’s)
conclusion.

By Sandeep Gupta, GMAT Perfect 800 | www.top-one-percent.com 12


THE MOST AUTHORITATIVE GUIDE EVER WRITTEN ON GMAT CRITICAL REASONING

Summary of the Core:

1. (P1 + P2) Premises supporting the scholars’ conclusion: the book includes a
presentation of Newton’s calculus concepts and techniques … and … the notes
were written before Leibniz’ own development of calculus concepts and
techniques
2. (C1) Conclusion of the scholars (that the author challenges): it is virtually certain
that the traditional view is false
3. (C2) Author’s conclusion: A more cautious conclusion than this is called for,
however
4. (P3) Premise supporting the author’s conclusion: Leibniz’ notes are limited to
early sections of Newton’s book, sections that precede the ones in which
Newton’s calculus concepts and techniques are presented

So, the final structure is: P1 + P2 Support C1, which is challenged by C2, which is
supported by P3.

The diagram above is called the FLOW of the argument.


If you can get the flow of any argument, you can solve every
single Boldface question correctly extremely quickly.

Now let’s see the original question asked on the GMAT as a Boldface question:

By Sandeep Gupta, GMAT Perfect 800 | www.top-one-percent.com 13


THE MOST AUTHORITATIVE GUIDE EVER WRITTEN ON GMAT CRITICAL REASONING

Historian: Newton developed mathematical concepts and techniques that are fundamental
to modern calculus. Leibniz developed closely analogous concepts and techniques. It has
traditionally been thought that these discoveries were independent. Researchers have,
however, recently discovered notes of Leibniz’ that discuss one of Newton’s books on
mathematics. Several scholars have argued that since the book includes a presentation of
Newton’s calculus concepts and techniques, and since the notes were written before
Leibniz’ own development of calculus concepts and techniques, it is virtually
certain that the traditional view is false. A more cautious conclusion than this is called for,
however. Leibniz’ notes are limited to early sections of Newton’s book, sections
that precede the ones in which Newton’s calculus concepts and techniques are
presented.

In the historian’s reasoning, the two boldfaced portions play which of the following roles?

A. The first provides evidence in support of the overall position that the historian defends;
the second is evidence that has been used to support an opposing position.
B. The first provides evidence in support of the overall position that the historian defends;
the second is that position.
C. The first provides evidence in support of an intermediate conclusion that is drawn to
provide support for the overall position that the historian defends; the second provides
evidence against that intermediate conclusion.
D. The first is evidence that has been used to support a conclusion that the historian
criticizes; the second is evidence offered in support of the historian’s own position.
E. The first is evidence that has been used to support a conclusion that the historian
criticizes; the second is further information that substantiates that evidence.

By Sandeep Gupta, GMAT Perfect 800 | www.top-one-percent.com 14


THE MOST AUTHORITATIVE GUIDE EVER WRITTEN ON GMAT CRITICAL REASONING

Focus on the word however in “a more cautious conclusion than this is called for,
however”

This signifies that the two Boldface statements are in opposition.

The first Boldface statement is:


since the notes were written before Leibniz’ own development of calculus concepts and
techniques

As per the analysis above, this is a premise (also called evidence) that supports C1 (the
conclusion of the scholars), which is opposed by C2 (the conclusion of the historian,
who is also the author of the passage)

The second Boldface statement is:


Leibniz’ notes are limited to early sections of Newton’s book, sections that precede the ones in
which Newton’s calculus concepts and techniques are presented.

As per the analysis above, this is a premise (also called evidence) that supports C2 (the
conclusion of the historian, who is also the author of the passage), which opposes C1
(the conclusion of the scholars)

You are asked about the roles played by P2 and P3.

By Sandeep Gupta, GMAT Perfect 800 | www.top-one-percent.com 15


THE MOST AUTHORITATIVE GUIDE EVER WRITTEN ON GMAT CRITICAL REASONING

Let’s see the options one by one and eliminate:

A. The first provides evidence in support of the overall position that the historian
defends (WRONG); the second is evidence that has been used to support an
opposing position.
B. The first provides evidence in support of the overall position that the historian
defends (WRONG); the second is that position.
C. The first provides evidence in support of an intermediate conclusion that is drawn
to provide support for the overall position that the historian defends (WRONG);
the second provides evidence against that intermediate conclusion.
D. The first is evidence that has been used to support a conclusion that the historian
criticizes; the second is evidence offered in support of the historian’s own
position.
E. The first is evidence that has been used to support a conclusion that the historian
criticizes; the second is further information (fact) that substantiates that
evidence (means a fact supports a fact—something that is just not allowed in CR
and RC, so this is automatically WRONG). Please remember that we can’t
challenge or support any fact in RC and CR.

Because of the word however, it is absolutely clear that the historian is challenging
the conclusion drawn by the scholars, not defending it. This instantly eliminates A, B,
and C.

In option E, we have “substantiates that evidence” (means supports a fact—


something that is just not allowed in CR and RC, so this is automatically WRONG).

So, by elimination, the answer is D.

By Sandeep Gupta, GMAT Perfect 800 | www.top-one-percent.com 16


THE MOST AUTHORITATIVE GUIDE EVER WRITTEN ON GMAT CRITICAL REASONING

Still, for better understanding, let’s evaluate option D, (this step won’t be required on
the test day).

• The first is evidence (P2) that has been used to support a conclusion (C1) that the
historian criticizes; the second is evidence (P3) offered in support of the
historian’s own position (C2)

(position means conclusion; please check the terms given below).

BINGO!

How to solve this question in 30 seconds flat?

1. Label Boldface 1 and Boldface 2 as Facts or Conclusions. Here it is clear that both
of them are facts.
2. Always check the presence of a contradiction word (such as however) between the
two boldface statements. Here we see the word however; this will signify that the
two Boldface statements are in opposition. So, eliminate A, B, and C (defends
means supports, not opposes).
Note: the presence of a contradiction word anywhere else in the argument doesn’t
matter.
3. While reading E, check information that substantiates that evidence, means a fact
that supports another fact—definitely wrong
4. D is your answer in 30 seconds flat.

By Sandeep Gupta, GMAT Perfect 800 | www.top-one-percent.com 17


THE MOST AUTHORITATIVE GUIDE EVER WRITTEN ON GMAT CRITICAL REASONING

The 30-second
approach (and the
steps) to any
Boldface question,
howsoever complex!

Step 1: There is no need to read the entire argument. Your first step is to directly label
each boldface statement as Premise or Conclusion

Step 2: You must remember the other terms used for Premise and Conclusion

➢ Premise: (two types – challengeable and non-challengeable)

NON-CHALLENGEABLE PREMISES: Fact, Truth, Data, Evidence,


Information, Situation, Circumstance, Research, Proof, Observation, Example,
Finding, Phenomenon, Illustration, Acknowledgement, Pattern (in the past or
present)

CHALLENGEABLE PREMISES: Reason, Justification, Reasoning,


Explanation, Line of Reasoning, Support, Statement of support, Supposition,
Consideration, Policy, Practice, Generalization (about past or present),
Judgment in support, Acknowledgement

➢ Conclusion: Judgment reached, Opinion, Suggestion, View, Idea, Belief,


Proposal, Warning, Forecast, Claim, Stand, Prediction, Hypothesis, Position
(to posit), Stance, Point, Main Point, Generalization (about future), Contention,
Consequence, Pattern (guessed / predicted for the future)

By Sandeep Gupta, GMAT Perfect 800 | www.top-one-percent.com 18


THE MOST AUTHORITATIVE GUIDE EVER WRITTEN ON GMAT CRITICAL REASONING

Step 3: Check the presence of a contradiction word between the two Boldface
statements. The common contradiction words are:

Contradiction Cues: But, However, Nonetheless, Nevertheless,


Notwithstanding, Even so, Despite, Rather, Yet, On the other hand, Admittedly,
In contrast, By contrast, Contrary to, Although, Even though, Still, Whereas, In
spite of, After all, Alternatively, Apart from, Conversely, Regardless, Then again,
Unfortunately, Ironically etc.

This step helps us eliminate a few choices.

Step 4: Never forget:

➢ The role of a premise is to support a conclusion

➢ We can never challenge or support a fact

STEP 5: Go to the options and eliminate

What if things get confusing?


In some arguments, initially it may be difficult for you to differentiate between the
premise and the conclusion or between the intermediate conclusion and the main
conclusion. In such a case, use:

By Sandeep Gupta, GMAT Perfect 800 | www.top-one-percent.com 19


THE MOST AUTHORITATIVE GUIDE EVER WRITTEN ON GMAT CRITICAL REASONING

The Therefore Test


This test instantly tells you which statement is the first in the order of logic (and hence
is the premise or the intermediate conclusion) and which statement is the last in the
order of logic (and hence is the final conclusion).

The test is:


If we can logically write

Because X, therefore Y
Then X is the premise / intermediate conclusion and Y is the final conclusion.

Here the most important word is logically. Means the order must make sense.

Consider the example below:

A new lemonade stand has just opened for business in the town square. The stand
will surely fail. A popular juice store already sells lemonade in the town square,
so the new lemonade stand will not be able to attract customers.

In this case, there seem to be two possible conclusions (or opinions):

(1) The stand will surely fail (PREDICTION, which is always a conclusion),
and
(2) The new lemonade stand will not be able to attract customers.
(PREDICTION, which is always a conclusion)

By Sandeep Gupta, GMAT Perfect 800 | www.top-one-percent.com 20


THE MOST AUTHORITATIVE GUIDE EVER WRITTEN ON GMAT CRITICAL REASONING

Remember this: before we can answer any question related to such an argument, we
MUST know what the main point, or the final conclusion, is. There can be only one
final conclusion. Let's use what we call "The Therefore Test" to identify the final
conclusion.

Try to write: Because X therefore Y


and
Because Y therefore X
Only one of these will make sense. Suppose “Because X therefore Y” makes sense, then
X is the supporting statement (either premise or intermediate conclusion) and Y is the
main conclusion.

We'll propose two possible P → C relationships between our two candidates:

Case #1: The new lemonade stand will surely fail. THEREFORE, the new lemonade
stand will not be able to attract customers.

Case #2: The new lemonade stand will not be able to attract customers.
THEREFORE, the new lemonade stand will surely fail.

The first case doesn't make sense.


In the second case, however, the first part of the statement clearly supports, or leads
into, the second part of the statement. Because the stand will not be able to attract
customers, it will surely fail. (If you're having trouble, try thinking about it in terms of
chronology—what happens first?
The stand doesn't attract new customers, and this leads to the failure of the stand.)
Thus, the final conclusion, the main conclusion, is that "The stand will surely fail."
Any conclusion that supports the final conclusion is called an intermediate
conclusion. Intermediate conclusions are always supported by a premise.

Let's break this argument down:

By Sandeep Gupta, GMAT Perfect 800 | www.top-one-percent.com 21


THE MOST AUTHORITATIVE GUIDE EVER WRITTEN ON GMAT CRITICAL REASONING

➢ BACKGROUND INFORMATION: A new lemonade stand has just opened for


business in the town square.
➢ CONCLUSION (final opinion): The stand will surely fail.
➢ SUPPORTING PREMISE (fact): A popular juice store already sells lemonade in the
town square
➢ INTERMEDIATE CONCLUSION (opinion): so, the new lemonade stand will not
be able to attract customers.

Here it is in argument core form: (P) premise ➔ (IC) intermediate conclusion


➔ (C) conclusion.

Popular juice store already there ➔ new store won't be able to attract customers ➔
new store will fail

Let’s take another example:

The best way of eliminating traffic congestion will not be easily found. There are so
many competing possibilities that it will take millions of dollars to study every option,
and implementation of most options carries an exorbitant price tag.

Case 1: Because the best way of eliminating traffic congestion will not be easily
found, we can conclude that there are so many competing possibilities that it will
take millions of dollars to study every option, and implementation of most options
carries an exorbitant price tag.

Does that sound right? No!

Let us try again, this time making the first sentence the conclusion and the second
sentence the premise:

Because there are so many competing possibilities that it will take millions of dollars
to study every option, and implementation of most options carries an exorbitant price
tag, we can conclude that the best way of eliminating traffic congestion will not be
easily found.

By Sandeep Gupta, GMAT Perfect 800 | www.top-one-percent.com 22


THE MOST AUTHORITATIVE GUIDE EVER WRITTEN ON GMAT CRITICAL REASONING

This makes sense. In most cases when you have the conclusion and premise
backward, the arrangement will be confusing. The correct arrangement always sounds
more logical.

Some Language Cues


Conclusion Cues: So, Thus, Therefore, Thereby, Consequently, Clearly, As a result,
For this reason, This demonstrates that, They conclude that, Hence, Accordingly, It
must be that, It shows that, It follows that, It is likely that

Premise Cues: Since, The reason is, Because, For (when it means ‘because’), For
example, For the reason that, In that, Given that, As indicated by, Due to, Owing to,
This can be seen from, We know this by

Contradiction Cues: But, However, Nonetheless, Nevertheless, Notwithstanding,


Even so, Despite, Rather, Yet, On the other hand, Admittedly, In contrast, By contrast,
Contrary to, Although, Even though, Still, Whereas, In spite of, After all, Alternatively,
Apart from, Conversely, Regardless, Then again, Unfortunately, Ironically etc.

A special cue: Some people believe: One of the most frequently used constructions
is to raise a viewpoint at the beginning of the stimulus and then disagree with it
immediately thereafter. This efficiently raises two opposing views in a very short
paragraph. These stimuli are recognizable because they often begin with the phrase,
“Some people claim...” or one of the many variations on this theme, including but not
limited to the following:

“Some people propose...” “Many people believe...”

“Some people argue that...” “Some critics claim...”

“Some critics maintain...” “Some scientists believe...

All these imply that the author will challenge whatever is said by others.

By Sandeep Gupta, GMAT Perfect 800 | www.top-one-percent.com 23


THE MOST AUTHORITATIVE GUIDE EVER WRITTEN ON GMAT CRITICAL REASONING

Remember, these cues are just guidelines, not the


final verdict.

For example, every time the word thus may not signal a conclusion. The test maker
knows this: the moment you see the word “thus”, you will automatically be thinking
"conclusion!" So, sometimes the GMAT will attempt to fool you.

Focus on the sentence below:

Countries that invest heavily in technology and thus become highly advanced
automatically start seeing themselves as global superpowers.

Here the word thus doesn’t signal a conclusion.

The building blocks of CR – the most crucial Exercise in this entire book:
if you genuinely wish to ace CR, you must strengthen your grip on identifying
argument core (P → C). Keeping this in mind, please analyze the argument
structure in each of the questions below. Some questions ask you to identify the
conclusion, intermediate conclusion, and counter conclusion; some others ask you to
identify the argument core; and some ask you to break the argument down, statement
by statement. Please follow the directions and attempt these questions sincerely.
Solving BOLDFACE questions will become a cakewalk if you solve each of the
questions sincerely.

Total 35 questions
In Q. 1 to 9: Identify conclusion(s), intermediate conclusion(s), and counter
conclusion(s) in the arguments given below.

By Sandeep Gupta, GMAT Perfect 800 | www.top-one-percent.com 24


THE MOST AUTHORITATIVE GUIDE EVER WRITTEN ON GMAT CRITICAL REASONING

1. Environmental organizations want to preserve the land surrounding the Wilgrinn


Wilderness Area from residential development. They plan to do this by purchasing
that land from the farmers who own it. That plan is ill-conceived: if the farmers did
sell their land, they would sell it to the highest bidder, and developers would outbid
any other bidders. On the other hand, these farmers will never actually sell any of
the land, provided that farming it remains viable. But farming will not remain
viable if the farms are left unmodernized, and most of the farmers lack the financial
resources modernization requires. And that is exactly why a more sensible
preservation strategy would be to assist the farmers to modernize their farms to
the extent needed to maintain viability.

2. Local authorities are considering an amendment to the litter law that would raise
the fine for littering in the community picnic area to $1,000. Since the inception of
the litter law, incremental increases in the littering fine have proven to be
consistently effective at further reducing the amount of litter in the community
picnic area. However, raising the fine to $1,000 would actually have the
unintended effect of increasing the amount of litter in the picnic area. Picnic area
users would perceive this fine to be unreasonable and unenforceable, and would
disregard the litter law altogether.

3. Political analyst: A party that temporarily positions itself in the negligible crack
between the American right and left will do little to expand the public debate. What
America needs is a permanent third party. Some claim that America's success
stems from the two-party system. These people say that a third party would make
the passage of legislation and thus governance impossible. Furthermore, they point
to the current sluggish pace of government as proof that the country cannot bear
the burden of a third party. Yet, most European countries have multi-party systems
and few complain about any inability to govern there.

By Sandeep Gupta, GMAT Perfect 800 | www.top-one-percent.com 25


THE MOST AUTHORITATIVE GUIDE EVER WRITTEN ON GMAT CRITICAL REASONING

4. Traditionally, video game manufacturers have been most strongly influenced by


serious video gamers. Because devoted gamers have historically purchased the
majority of video games, companies react to the desires of this market segment.
Normally, devoted gamers crave speed and action; thus, most manufacturers
continue to produce games with faster chips and flashier graphics. Unfortunately,
faster chips and flashier graphics are no longer in the industry's best interest. The
devoted gaming market is deeply stagnant, and it won't soon expand. To infuse new
life into the video game market, manufacturers must simplify the functionality of
their games. By doing so, current non-gamers will be attracted to join the ranks of
video game fans.

5. Musical composers have generally made their most significant contributions to the
musical canon before they reached the age of thirty. It is commonly believed that
this is the case because aging brings about a loss of cognitive ability and creative
capacity. However, a study pointed out that a disproportionately large number of
those composers who made their most significant contributions to the canon after
the age of thirty became musicians at an older age than is generally the case. Since
by the age of thirty many composers have been engaged as musicians for a decade
or more, these findings suggest that the real reason why musicians over thirty
rarely make significant contributions to the musical canon is not that they have
aged but rather that they have spent too much time as musicians.

6. The city government should invest surplus funds in improving the city's
transportation network. Most of the network was put in place at a time when the
city was much smaller in both area and population. The subway system is outdated
and understaffed. The buses rarely run on schedule and their routes are
inconvenient. If the city does not make changes soon to the network, it will see
many of its prized industries relocate to more convenient cities and, as a result, the
city's financial health will be jeopardized.

By Sandeep Gupta, GMAT Perfect 800 | www.top-one-percent.com 26


THE MOST AUTHORITATIVE GUIDE EVER WRITTEN ON GMAT CRITICAL REASONING

7. Economist: Tropicorp, which constantly seeks profitable investment opportunities,


has been buying and clearing sections of tropical forest for cattle ranching,
although pastures newly created there become useless for grazing after just a few
years. The company has not gone into rubber tapping, even though greater profits
can be made from rubber tapping, which leaves the forest intact. Thus, some
environmentalists conclude that Tropicorp has not acted wholly out of economic
self-interest. However, these environmentalists are probably wrong. The initial
investment required for a successful rubber-tapping operation is larger than that
needed for a cattle ranch. Furthermore, there is a shortage of workers employable
in rubber-tapping operations, and finally, taxes are higher on profits from rubber
tapping than on profits from cattle ranching.

8. Rain-soaked soil contains less oxygen than does drier soil. The roots of melon
plants perform less efficiently under the low-oxygen conditions present in rain-
soaked soil. When the efficiency of melon roots is impaired, the roots do not supply
sufficient amounts of the proper nutrients for the plants to perform photosynthesis
at their usual levels. It follows that melon plants have a lower-than-usual rate of
photosynthesis when their roots are in rain-soaked soil. When the photosynthesis
of the plants slows, sugar stored in the fruits is drawn off to supply the plants with
energy. Therefore, ripe melons harvested after a prolonged period of heavy rain
should be less sweet than other ripe melons.

9. Every year many people become ill because of airborne mold spores in their homes.
After someone becomes ill, specialists are often hired to eradicate the mold. These
specialists look in damp areas of the house, since mold is almost always found in
places where there is substantial moisture. If one wishes to avoid mold poisoning,
then, one should make sure to keep all internal plumbing in good condition to
prevent leakage that could serve as a breeding ground for mold.

In Q. 10 to 24: break down the given arguments into its subparts and identify the
argument core (P → C).

By Sandeep Gupta, GMAT Perfect 800 | www.top-one-percent.com 27


THE MOST AUTHORITATIVE GUIDE EVER WRITTEN ON GMAT CRITICAL REASONING

10. Coherent solutions for the problem of reducing health care costs cannot be found
within the current piecemeal system of paying these costs. The reason is that this
system gives health care providers and insurers every incentive to shift, wherever
possible, the costs of treating illness onto each other or any other party, including
the patient. That clearly is the lesson of the various reforms of the 1980s: push in
on one part of this pliable spending balloon and an equally expensive bulge pops
up elsewhere. For example, when the government health care insurance program
for the poor cut costs by disallowing payments for some visits to physicians,
patients with advanced illness later presented themselves at hospital emergency
rooms in increased numbers.

11. A large group of hyperactive children whose regular diets included food containing
large amounts of additives was observed by researchers trained to assess the
presence or absence of behavior problems. The children were then placed on a low
additive diet for several weeks, after which they were observed again. Originally
nearly 60 percent of the children exhibited behavior problems; after the change in
diet, only 30 percent did so. On the basis of these data, it can be concluded that
food additives can contribute to behavior problems in hyperactive children.

12. In 1974 the speed limit on highways in the United States was reduced to 55 miles
per hour in order to save fuel. In the first 12 months after the change, the rate of
highway fatalities dropped 15 percent, the sharpest one year drop in history. Over
the next 10 years, the fatality rate declined by another 25 percent. It follows that
the 1974 reduction in the speed limit saved many lives.

13. Some legislators refuse to commit public funds for new scientific research if they
cannot be assured that the research will contribute to the public welfare. Such a
position ignores the lessons of experience. Many important contributions to the
public welfare that resulted from scientific research were never predicted as
potential outcomes of that research. Suppose that a scientist in the early twentieth
century had applied for public funds to study molds: who would have predicted
that such research would lead to the discovery of antibiotics―one of the greatest
contributions ever made to the public welfare?

By Sandeep Gupta, GMAT Perfect 800 | www.top-one-percent.com 28


THE MOST AUTHORITATIVE GUIDE EVER WRITTEN ON GMAT CRITICAL REASONING

14. When workers do not find their assignments challenging, they become bored and
so achieve less than their abilities would allow. On the other hand, when workers
find their assignments too difficult, they give up and so again achieve less than what
they are capable of achieving. It is, therefore, clear that no worker's full potential
will ever be realized.

15. The National Association of Fire Fighters says that 45 percent of homes now have
smoke detectors, whereas only 30 percent of homes had them 10 years ago. This
makes early detection of house fires no more likely, however, because over half of
the domestic smoke detectors are either without batteries or else inoperative for
some other reason.

16. Graphologists claim that it is possible to detect permanent character traits by


examining people's handwriting. For example, a strong cross on the "t" is supposed
to denote enthusiasm. Obviously, however, with practice and perseverance people
can alter their handwriting to include this feature. So, it seems that graphologists
must hold that permanent character traits can be changed.

17. Waste management companies, which collect waste for disposal in landfills and
incineration plants, report that disposable plastics make up an ever-increasing
percentage of the waste they handle. It is clear that attempts to decrease the
amount of plastic that people throwaway in the garbage are failing.

18. Crimes in which handguns are used are more likely than other crimes to result in
fatalities. However, the majority of crimes in which handguns are used do not
result in fatalities. Therefore, there is no need to enact laws that address crimes
involving handguns as distinct from other crimes.

19. Data from satellite photographs of the tropical rain forest in Melonia show that last
year the deforestation rate of this environmentally sensitive zone was significantly
lower than in previous years. The Melonian government, which spent millions of
dollars last year to enforce laws against burning and cutting of the forest, is
claiming that the satellite data indicate that its increased efforts to halt the
destruction are proving effective.

By Sandeep Gupta, GMAT Perfect 800 | www.top-one-percent.com 29


THE MOST AUTHORITATIVE GUIDE EVER WRITTEN ON GMAT CRITICAL REASONING

20. Many major scientific discoveries of the past were the product of serendipity, the
chance discovery of valuable findings that investigators had not purposely sought.
Now, however, scientific research tends to be so costly that investigators are heavily
dependent on large grants to fund their research. Because such grants require
investigators to provide the grant sponsors with clear projections of the outcome
of the proposed research, investigators ignore anything that does not directly bear
on the funded research. Therefore, under the prevailing circumstances, serendipity
can no longer play a role in scientific discovery.

21. George: Some scientists say that global warming will occur because people are
releasing large amounts of carbon dioxide into the atmosphere by burning trees
and fossil fuels. We can see, though, that the predicted warming is occurring
already. In the middle of last winter, we had a month of spring-like weather in our
area, and this fall, because of unusually mild temperatures, the leaves on our town's
trees were three weeks late in turning color.

22. A university should not be entitled to patent the inventions of its faculty members.
Universities, as guarantors of intellectual freedom, should encourage the free flow
of ideas and the general dissemination of knowledge. Yet a university that retains
the right to patent the inventions of its faculty members has a motive to suppress
information about a potentially valuable discovery until the patent for it has been
secured. Clearly, suppressing information concerning such discoveries is
incompatible with the university's obligation to promote the free flow of ideas.

23. Balance is particularly important when reporting the background of civil wars and
conflicts. Facts must not be deliberately manipulated to show one party in a
favorable light, and the views of each side should be fairly represented. This
concept of balance, however, does not justify concealing or glossing over basic
injustices in an effort to be even-handed. If all the media were to adopt such a
perverse interpretation of balanced reporting, the public would be given a picture
of a world where each party in every conflict had an equal measure of justice on its
side, contrary to our experience of life and, indeed, our common sense.

By Sandeep Gupta, GMAT Perfect 800 | www.top-one-percent.com 30


THE MOST AUTHORITATIVE GUIDE EVER WRITTEN ON GMAT CRITICAL REASONING

24. Some of my friends say that skiing is the best way to burn calories, but this is
ridiculous. Since the act of skiing down a mountain is primarily driven by the pull
of gravity, skiing requires very little physical exertion. Thus, skiing doesn't burn
many calories.

In Q. 25 to 35: Identify premise(s) and conclusion(s) in each of the following


arguments:

25. Every year, new reports appear concerning the health risks posed by certain
substances, such as coffee and sugar. One year an article claimed that coffee is
dangerous to one’s health. The next year, another article argued that coffee has
some benefits for one’s health. From these contradictory opinions, we see that
experts are useless for guiding one’s decisions about one’s health.

26. Some teachers claim that students would not learn curricular content without the
incentive of grades. But students with intense interest in the material would learn
it without this incentive, while the behavior of students lacking all interest in the
material is unaffected by such an incentive. The incentive of grades, therefore,
serves no essential academic purpose.

27. Damming the Merv River would provide irrigation for the dry land in its upstream
areas; unfortunately, a dam would reduce agricultural productivity in the fertile
land downstream by reducing the availability and quality of water there. The
productivity loss in the downstream area would be greater than the productivity
gain upstream, so building a dam would yield no overall gain in agricultural
productivity in the region as a whole.

28. While it was once believed that the sort of psychotherapy appropriate for the
treatment of neuroses caused by environmental factors is also appropriate for
schizophrenia and other psychoses, it is now known that these latter, more serious
forms of mental disturbance are best treated by biochemical—that is, medicinal—
means. This is conclusive evidence that psychoses, unlike neuroses, have nothing
to do with environmental factors but rather are caused by some sort of purely
organic condition, such as abnormal brain chemistry or brain malformations.

By Sandeep Gupta, GMAT Perfect 800 | www.top-one-percent.com 31


THE MOST AUTHORITATIVE GUIDE EVER WRITTEN ON GMAT CRITICAL REASONING

29. If relativity theory is correct, no object can travel forward in time at a speed greater
than the speed of light. Yet quantum mechanics predicts that the tachyon, a
hypothetical subatomic particle, travels faster than light. Thus, if relativity theory
is correct, either quantum mechanics’ prediction about tachyons is erroneous or
tachyons travel backwards in time.

30. Any course that teaches students how to write is one that will serve them well later
in life. Therefore, since some philosophy courses teach students how to write, any
student, whatever his or her major, will be served well in later life by taking any
philosophy course.

31. Because the Vikings have the best wide receiver in football, they therefore have the
best offense in football. Because they have the best offense in football, they will win
the Super Bowl next year.
32. It is well known that many species adapt to their environment, but it is usually
assumed that only the most highly evolved species alter their environment in ways
that aid their own survival. However, this characteristic is actually quite common.
Certain species of plankton, for example, generate a gas that is converted in the
atmosphere into particles of sulfate. These particles cause water vapor to condense,
thus forming clouds. Indeed, the formation of clouds over the ocean largely
depends on the presence of these particles. More cloud cover means more sunlight
is reflected, and so the Earth absorbs less heat. Thus, plankton cause the surface of
the Earth to be cooler and this benefits the plankton.

33. Wine is made by crushing grapes and eventually separating the juice from the grape
skins. However, the separated juice contains impurities and many wineries do not
filter the juice. These wineries claim the unfiltered juice ultimately produces a more
flavorful and intense wine. Since these wine makers are experts, we should trust
their judgment and not shy away from unfiltered wine.

By Sandeep Gupta, GMAT Perfect 800 | www.top-one-percent.com 32


THE MOST AUTHORITATIVE GUIDE EVER WRITTEN ON GMAT CRITICAL REASONING

34. Phenylketonurics are people who cannot metabolize the amino acid phenylalanine.
There are dangers associated with phenylketonuria, and products containing
phenylalanine must carry a warning label that states, “Phenylketonurics: contains
phenylalanine.” In addition, all children in developed societies receive a
phenylketonuria test at birth. Hence, at the moment, we are doing as much as
possible to protect against this condition.
35. During last night’s robbery, the thief was unable to open the safe. Thus, last night’s
robbery was unsuccessful despite the fact that the thief stole several documents.
After all, nothing in those documents was as valuable as the money in the safe.

By Sandeep Gupta, GMAT Perfect 800 | www.top-one-percent.com 33


THE MOST AUTHORITATIVE GUIDE EVER WRITTEN ON GMAT CRITICAL REASONING

Solutions
1.

Sub-Conclusion: That plan is ill-conceived.

Main Conclusion: A more sensible preservation strategy would be to assist the farmers to
modernize their farms to the extent needed to maintain viability.

Use the Therefore Test to decide between the sub-conclusion and the main conclusion:

Because a more sensible preservation strategy would be to assist the farmers to modernize their
farms to the extent needed to maintain viability therefore that plan is ill-conceived.

This makes no sense.

Because that plan is ill-conceived therefore a more sensible preservation strategy would be to
assist the farmers to modernize their farms to the extent needed to maintain viability.

This makes sense. If we can write “because X so Y”, then X is the supporting premise or
supporting reason or the sub-conclusion and Y is the main conclusion.

2.

Main Conclusion: However, raising the fine to $1,000 would actually have the unintended effect
of increasing the amount of litter in the picnic area.

Supporting Reason: Picnic area users would perceive this fine to be unreasonable and
unenforceable, and would disregard the litter law altogether.

Use the Therefore Test:

Because raising the fine to $1,000 would actually have the unintended effect of increasing the
amount of litter in the picnic area therefore picnic area users would perceive this fine to be
unreasonable and unenforceable, and would disregard the litter law altogether.

This makes no sense.

Because picnic area users would perceive this fine to be unreasonable and unenforceable, and
would disregard the litter law altogether therefore raising the fine to $1,000 would actually have
the unintended effect of increasing the amount of litter in the picnic area.

This makes sense. If we can write “because X so Y”, then X is the supporting premise or
supporting reason or the sub-conclusion and Y is the main conclusion.

By Sandeep Gupta, GMAT Perfect 800 | www.top-one-percent.com 34


THE MOST AUTHORITATIVE GUIDE EVER WRITTEN ON GMAT CRITICAL REASONING

3.

Sub-conclusion: A party that temporarily positions itself in the negligible crack between the
American right and left will do little (sort of prediction / opinion / view) to expand the public
debate.

Main Conclusion: What America needs is a permanent third party.

Conclusion of the other side (usually the author opposes this conclusion): America's
success stems from the two-party system.

4.

Main-conclusion: To infuse new life into the video game market, manufacturers must simplify
the functionality of their games.

Sub-conclusion: faster chips and flashier graphics are no longer in the industry's best interest.”

Use the Therefore Test:

Because to infuse new life into the video game market, manufacturers must simplify the
functionality of their games, therefore faster chips and flashier graphics are no longer in the
industry's best interest.

This makes no sense.

Because faster chips and flashier graphics are no longer in the industry's best interest therefore
to infuse new life into the video game market, manufacturers must simplify the functionality of their
games.

This makes sense.

Beware: “thus, most manufacturers continue to produce games with faster chips and flashier
graphics” is not a conclusion; rather it is a fact.

Beware: “The devoted gaming market is deeply stagnant, and it won't soon expand.” is the
supporting reason.

5.

Common explanation given: It is commonly believed that this is the case because aging brings
about a loss of cognitive ability and creative capacity.

Main conclusion of the argument: These findings suggest that the real reason why musicians
over thirty rarely make significant contributions to the musical canon is not that they have aged but
rather that they have spent too much time as musicians. (This is not the explicit opinion of the author
but the author implicitly agrees with the findings’ suggestion – so this statement serves as author’s
conclusion).

By Sandeep Gupta, GMAT Perfect 800 | www.top-one-percent.com 35


THE MOST AUTHORITATIVE GUIDE EVER WRITTEN ON GMAT CRITICAL REASONING

6.

Main Conclusion: The city government should invest surplus funds in improving the city's
transportation network.

Sub-conclusion: If the city does not make changes soon to the network, it will see many of its
prized industries relocate to more convenient cities and, as a result, the city's financial health will be
jeopardized.

Use the Therefore Test:

Because the city government should invest surplus funds in improving the city's transportation
network therefore if the city does not make changes soon to the network, it will see many of its
prized industries relocate to more convenient cities and, as a result, the city's financial health will be
jeopardized.

This makes no sense.

Because if the city does not make changes soon to the network, it will see many of its prized
industries relocate to more convenient cities and, as a result, the city's financial health will be
jeopardized therefore the city government should invest surplus funds in improving the city's
transportation network.

This makes sense.

7.
Conclusion of the other side (Environmentalists): Tropicorp has not acted wholly out of
economic self-interest.

Main Conclusion: However, these environmentalists are probably wrong.

8.

Sub-Conclusion: It follows that melon plants have a lower-than-usual rate of photosynthesis


when their roots are in rain-soaked soil.

Main Conclusion: Therefore, ripe melons harvested after a prolonged period of heavy rain should
be less sweet than other ripe melons.

9.

Main Conclusion: If one wishes to avoid mold poisoning, then, one should make sure to keep all
internal plumbing in good condition to prevent leakage that could serve as a breeding ground for
mold.

Implied Conclusion: Moisture causes mold.

By Sandeep Gupta, GMAT Perfect 800 | www.top-one-percent.com 36


THE MOST AUTHORITATIVE GUIDE EVER WRITTEN ON GMAT CRITICAL REASONING

10.

Coherent solutions for the problem of reducing health care costs cannot be found within the current
piecemeal system of paying these costs.

This sounds like an opinion. Could it be the final conclusion?

The reason is that this system gives health care providers and insurers every incentive to shift,
wherever possible, the costs of treating illness onto each other or any other party, including the
patient.

“The reason is that ...” is a big language cue. This must be support for the first sentence! So, it seems
we got the conclusion first, immediately followed by a supporting premise.

That clearly is the lesson of the various reforms of the 1980s: push in on one part of this pliable
spending balloon and an equally expensive bulge pops up elsewhere. For example, when the
government health care insurance program for the poor cut costs by disallowing payments for some
visits to physicians, patients with advanced illness later presented themselves at hospital emergency
rooms in increased numbers.

Wow, lots of information, but all of this is simply illustrating, or providing an example for, the
shifting costs described in the premise above it. We can think of all this as duplicate information.

Our core is: P → C

System gives incentive to shift costs to others → solutions for reducing costs cannot be found in
current system

11.

A large group of hyperactive children whose regular diets included food containing large amounts
of additives was observed by researchers trained to assess the presence or absence of behavior
problems.

Definitely background information: This is setting us up to receive research findings of some kind.

The children were then placed on a low additive diet for several weeks, after which they were
observed again.

More setup. (By the way, this is very common on arguments that make conclusions from research
studies. They generally start by giving background information on the way the study was
administered.)

Originally nearly 60 percent of the children exhibited behavior problems;

One of the findings from the study

After the change in diet, only 30 percent did so.

By Sandeep Gupta, GMAT Perfect 800 | www.top-one-percent.com 37


THE MOST AUTHORITATIVE GUIDE EVER WRITTEN ON GMAT CRITICAL REASONING

The other finding from the study

On the basis of these data, it can be concluded that food additives can contribute to behavior
problems in hyperactive children.

Okay, so these two data points (complementary premises) are being used to support the
conclusion.

CORE:

60% originally had behavior problems + 30% had behavior problems after decreasing additives in
diet → food additives can contribute to behavior problems in hyperactive children

12.

In 1974 the speed limit on highways in the United States was reduced to 55 miles per hour in order
to save fuel.

This is a historical fact. It's probably just background information.

In the first 12 months after the change, the rate of highway fatalities dropped 15 percent, the
sharpest one year drop in history.

Support for something. (Statistics will generally be used as supporting premise.)

Over the next 10 years, the fatality rate declined by another 25 percent.

Another statistic. These two stats will probably complement each other to come up with a final
claim.

It follows that the 1974 reduction in the speed limit saved many lives.

And there's the claim (conclusion).

The core of the argument is:

In first year after reduction, 15% drop in deaths + another 25% drop over next 10 years → reduction
has saved many lives

By Sandeep Gupta, GMAT Perfect 800 | www.top-one-percent.com 38


THE MOST AUTHORITATIVE GUIDE EVER WRITTEN ON GMAT CRITICAL REASONING

13.

Some legislators refuse to commit public funds for new scientific research if they cannot be assured
that the research will contribute to the public welfare

"Some legislators ...” This has the tone of an opposing point that is about to be countered.

Such a position ignores the lessons of experience.

This seems to be the main conclusion, a counter to the legislators view above. "Such a position"
borrows language from the first sentence. The author is claiming that the legislators' refusal to
commit public funds because of a lack of assurance of results is a position that ignores the lessons
of experience. We can anticipate that the "lessons of experience" are forthcoming!

Many important contributions to the public welfare that resulted from scientific research were never
predicted as potential outcomes of that research.

And here's the support—lessons of experience.

Suppose that a scientist in the early twentieth century had applied for public funds to study molds:
who would have predicted that such research would lead to the discovery of antibiotics―one of the
greatest contributions ever made to the public welfare?
Lots of information here, but it's duplicate information. It's a specific example of the premise
above, an example of a case in which contributions to the public welfare (discovery of antibiotics)
were not predicted.

The core is:

Many important contributions came from research but were never predicted as potential outcomes
→ legislators' position to refuse to commit to research unless outcomes are assured is a position that
ignores lessons of experience

14.

When workers do not find their assignments challenging, they become bored and so achieve less
than their abilities would allow. On the other hand, when workers find their assignments too
difficult, they give up and so again achieve less than what they are capable of achieving. It is,
therefore, clear that no worker's full potential will ever be realized.

Straightforward argument that uses two complementary premises to arrive at an easy-to-spot


conclusion:

Workers underachieve when assignments are not challenging enough + workers underachieve when
assignments are too challenging → no worker's full potential will ever be realized

By Sandeep Gupta, GMAT Perfect 800 | www.top-one-percent.com 39


THE MOST AUTHORITATIVE GUIDE EVER WRITTEN ON GMAT CRITICAL REASONING

15.

The National Association of Fire Fighters says that 45 percent of homes now have smoke detectors,
whereas only 30 percent of homes had them 10 years ago.

"The National Association of Firefighters says ... " Seems like it'll be opposing information of some
sort.

This makes early detection of house fires no more likely, however,

The word "however" indicates a pivot, or transition away from the first sentence. The information
given by the firefighters, that more homes now have detectors, would seem to indicate that
detection of home fires WO ULD be more likely, but the author is saying that the detection of fires
would NOT be any more likely. This might be the author's conclusion, or is it just a fact being used
for something else?

because over half of the domestic smoke detectors are either without batteries or else inoperative for
some other reason.

"Because" indicates that this is support for the author's claim above.

The core is:

Over half of domestic detectors are without batteries or are inoperative → increase in detectors from
30% to 45% does not make home fires any less likely

16.

Graphologists claim that it is possible to detect permanent character traits by examining people's
handwriting.

This sort of opposing point ("Graphologists claim ... ") is starting to get easy to recognize!

For example, a strong cross on the "t" is supposed to denote enthusiasm.

Simply an example to help explain the graphologists' claim.

Obviously, however, with practice and perseverance people can alter their handwriting to include
this feature.

The word "however" indicates that this statement counters the graphologists' claim. Is this the final
claim or just a factual statement that will support something else? Hard to tell for now.

So, it seems that graphologists must hold that permanent character traits can be changed.

Ah. The word "so" indicates that this is the main conclusion, and the part before is simply support
for this conclusion.

By Sandeep Gupta, GMAT Perfect 800 | www.top-one-percent.com 40


THE MOST AUTHORITATIVE GUIDE EVER WRITTEN ON GMAT CRITICAL REASONING

Core:

People can change their handwriting characteristics → graphologists must hold that people can
change their permanent character traits

17.

Waste management companies, which collect waste for disposal in landfills and incineration plants,
report that disposable plastics make up an ever-increasing percentage of the waste they handle.

"Waste management companies report ... " This seems to be another example of an opposing point
that will be refitted or countered somehow.

It is clear that attempts to decrease the amount of plastic that people throwaway in the garbage are
failing.

Oh, wait. We get no counter point. In fact, "it is clear" indicates that this is the conclusion. The
waste management company report is actually support for this conclusion.

Core:

Waste management reports increasing percentage of disposable plastics for disposal in landfills and
incinerators → attempts to decrease amount of plastic people throwaway in garbage are failing

18.

Crimes in which handguns are used are more likely than other crimes to result in fatalities.

Seems like a statement of fact. Hard to say exactly how it will function at this point.

However, the majority of crimes in which handguns are used do not result in fatalities.

This is tricky. The "however" doesn't seem to refute the first statement. It just introduces a related
fact.

Therefore, there is no need to enact laws that address crimes involving handguns as distinct from
other crimes.

This is obviously the conclusion, and now we can see that the two points made earlier ARE in fact
in opposition to each other. The conclusion is that we don't need laws for handgun crimes in
particular. The first statement (that handgun crimes are more likely to result in deaths) would
seem to suggest that we DO need special laws, but the second statement (the majority of handgun
crimes do not result in deaths) would be premise to suggest that we DON'T need the special laws.
50, the second statement supports the conclusion:

By Sandeep Gupta, GMAT Perfect 800 | www.top-one-percent.com 41


THE MOST AUTHORITATIVE GUIDE EVER WRITTEN ON GMAT CRITICAL REASONING

Core:

Most crimes in which handguns are used do not result in deaths → no need to enact special laws for
crimes involving handguns

19.

Data from satellite photographs of the tropical rain forest in Melonia show that last year the
deforestation rate of this environmentally sensitive zone was significantly lower than in previous
years.

Factual information. Maybe just background information? Hard to say just yet.

The Melonian government, which spent millions of dollars last year to enforce laws against burning
and cutting of the forest,

Another fact. The government spent millions of dollars to stop burning and cutting of forests.

The Melonian Government is claiming that the satellite data indicate that its increased efforts to halt
the destruction are proving effective.

The conclusion! The word "claiming" gives it away. Notice that the conclusion ties together
information about the deforestation rate and the efforts made by the government to curb burning
and cutting. This is a case where two complementary premises are used to support the final claim:

Core:

Deforestation rate decreasing + government spent millions to curb cutting and burning →
government efforts are proving effective

20.

Many major scientific discoveries of the past were the product of serendipity, the chance discovery
of valuable findings that investigators had not purposely sought.

Statement of fact. Not yet sure how it will be used.

Now, however, scientific research tends to be so costly that investigators are heavily dependent on
large grants to fund their research.

Another statement of fact that provides a contrast between then and now.

Because such grants require investigators to provide the grant sponsors with clear projections of the
outcome of the proposed research,

"Because" indicates support for something, and that something must be coming up...

investigators ignore anything that does not directly bear on the funded research.

Could this be the final claim, then?

By Sandeep Gupta, GMAT Perfect 800 | www.top-one-percent.com 42


THE MOST AUTHORITATIVE GUIDE EVER WRITTEN ON GMAT CRITICAL REASONING

Therefore, under the prevailing circumstances, serendipity can no longer play a role in scientific
discovery.

Ah. This is the final claim. So, we actually have a 3-part argument core with an intermediate
conclusion in the middle:

Core:

Grants require investigators to provide clear outcomes projections → investigators ignore anything
that does not directly bear on the funded research → serendipity can no longer play a role

21.

George: Some scientists say that global warming will occur because people are releasing large
amounts of carbon dioxide into the atmosphere by burning trees and fossil fuels.

"Some scientists say ... " Sounds like an opposing point that's about to be refuted!

We can see, though, that the predicted warming is occurring already.

Yes, "though" provides a transition into the author's claim: the warming is happening already. We
can anticipate that we'll get some support for this next.

In the middle of last winter, we had a month of spring-like weather in our area,

One piece of premise to support the claim that the warming is already happening.

and this fall, because of unusually mild temperatures, the leaves on our town's trees were three
weeks late in turning color.

And another piece of complementary premise. Two premises support the author's claim:

Core:

Spring-like weather in winter + mild fall weather delayed color change on leaves → predicted
warming already happening

22.

A university should not be entitled to patent the inventions of its faculty members.

Strong opinion. Could be the conclusion.

Universities, as guarantors of intellectual freedom, should encourage the free flow of ideas and the
general dissemination of knowledge.

Another opinion that seems to support the first!

Yet a university that retains the right to patent the inventions of its faculty members has a motive to
suppress information about a potentially valuable discovery until the patent for it has been secured.

By Sandeep Gupta, GMAT Perfect 800 | www.top-one-percent.com 43


THE MOST AUTHORITATIVE GUIDE EVER WRITTEN ON GMAT CRITICAL REASONING

Tricky. The word "yet" seems to indicate that a change in direction/opinion is afoot, but this
statement actually seems to support the notion that universities shouldn't be allowed to patent
inventions. There's not really any transition here at all.

Clearly, suppressing information concerning such discoveries is incompatible with the university's
obligation to promote the free flow of ideas.

This seems to be more support for the first statement. The core is complex. It uses three pieces of
complementary information to support its final claim.

Core:

Universities should promote free flow and dissemination of ideas + universities with right to patent
have incentive to suppress information + suppressing information is incompatible with obligation
to promote free flow of ideas → university should not be entitled to patent inventions by faculty

23.

Balance is particularly important when reporting the background of civil wars and conflicts.

Seems like an opinion. Could it be the conclusion?

Facts must not be deliberately manipulated to show one party in a favorable light, and the views of
each side should be fairly represented.

This seems like a duplicate claim! It's really just saying that balance is important.

This concept of balance, however, does not justify concealing or glossing over basic injustices in an
effort to be evenhanded.

Oh. This is a transition. Okay, balance is important, but not important enough to conceal injustices.
Now this feels like the conclusion. Will we get support?

If all the media were to adopt such a perverse interpretation of balanced reporting, the public would
be given a picture of a world where each party in every conflict had an equal measure of justice on
its side, contrary to our experience of life and, indeed, our common sense.

Yes, this is a reason why we can't have balance trumping everything else. This is the support.

Core:

If all media were to adopt balanced reporting, public would be given inaccurate representation of
justice → concept of balance does not justify concealing injustices

By Sandeep Gupta, GMAT Perfect 800 | www.top-one-percent.com 44


THE MOST AUTHORITATIVE GUIDE EVER WRITTEN ON GMAT CRITICAL REASONING

24.

We start off with an opposing point ("Some of my friends say ... "), and then we encounter a big
transition word ("but") that indicates a change in direction. Sure enough, we get the author's
opinion/conclusion next ("this is ridiculous"). The word "this" serves to borrow language from the
opposing point. "This" refers to the claim that skiing is the best way to burn calories. Essentially, the
author is saying "skiing is NOT the best way to burn calories." At this point, we should expect some
supporting reasoning. We encounter a supporting premise cue ("since"), which leads into the
supporting fact: gravity is the primary driver. What does it support? It supports the intermediate
conclusion ("skiing requires very little physical exertion"). In this case, "skiing requires very little
physical exertion" supports the intermediate conclusion that "skiing doesn't burn many calories,"
which supports the final conclusion that skiing is NOT the best way to burn calories.

So, here's the argument core:

P → IC → IC → C.

Skiing primarily driven by gravity → requires little physical exertion → doesn't burn many calories
→ not best way to burn calories

25.

Conclusion: From these contradictory opinions, we see that experts are useless for guiding one’s
decisions about one’s health.

Premise: Every year, new reports appear concerning the health risks posed by certain substances,
such as coffee and sugar.

Premise: One year an article claimed that coffee is dangerous to one’s health.

Premise: The next year, another article argued that coffee has some benefits for one’s health.

26.

Conclusion: The incentive of grades, therefore, serves no essential academic purpose.

Premise: Some teachers claim that students would not learn curricular content without the incentive
of grades.

Premise: But students with intense interest in the material would learn it without this incentive,
while the behavior of students lacking all interest in the material is unaffected by such an incentive.

By Sandeep Gupta, GMAT Perfect 800 | www.top-one-percent.com 45


THE MOST AUTHORITATIVE GUIDE EVER WRITTEN ON GMAT CRITICAL REASONING

27.

Conclusion: Building a dam would yield no overall gain in agricultural productivity in the region as
a whole.

Premise: Damming the Merv River would provide irrigation for the dry land in its upstream areas.

Premise: Unfortunately, a dam would reduce agricultural productivity in the fertile land
downstream by reducing the availability and quality of water there.

Premise: The productivity loss in the downstream area would be greater than the productivity gain
upstream.

28.

Conclusion: This is conclusive evidence that psychoses, unlike neuroses, have nothing to do with
environmental factors but rather are caused by some sort of purely organic condition, such as
abnormal brain chemistry or brain malformations.

Premise: While it was once believed that the sort of psychotherapy appropriate for the treatment of
neuroses caused by environmental factors is also appropriate for schizophrenia and other psychoses,
it is now known that these latter, more serious forms of mental disturbance are best treated by
biochemical—that is, medicinal—means.

29.

Conclusion: Thus, if relativity theory is correct, either quantum mechanics’ prediction about
tachyons is erroneous or tachyons travel backwards in time.

Premise: If relativity theory is correct, no object can travel forward in time at a speed greater than
the speed of light.

Premise: Yet quantum mechanics predicts that the tachyon, a hypothetical subatomic particle,
travels faster than light.

30.

Conclusion: Any student, whatever his or her major, will be served well in later life by taking any
philosophy course.

Premise: Any course that teaches students how to write is one that will serve them well later in life.

Premise: Some philosophy courses teach students how to write.

By Sandeep Gupta, GMAT Perfect 800 | www.top-one-percent.com 46


THE MOST AUTHORITATIVE GUIDE EVER WRITTEN ON GMAT CRITICAL REASONING

31.

Premise: “Because the Vikings have the best wide receiver in football,”

Sub-Conclusion (conclusion of the previous premise / Premise for the following conclusion): “they
therefore have the best offense in football.”

Main Conclusion: “they will win the Super Bowl next year.”

32.

Conclusion: This characteristic [altering the environment] is actually quite common

Premise: It is well known that many species adapt to their environment, but it is usually assumed
that only the most highly evolved species alter their environment in ways that aid their own survival.

Premise: Certain species of plankton, for example, generate a gas that is converted in the atmosphere
into particles of sulfate.

Premise: These particles cause water vapor to condense, thus forming clouds.

Premise: Indeed, the formation of clouds over the ocean largely depends on the presence of these
particles.

Premise: More cloud cover means more sunlight is reflected, and so the Earth absorbs less heat.

Sub-conclusion: Thus, plankton cause the surface of the Earth to be cooler and this benefits the
plankton.

The main conclusion is actually the second sentence. There is another conclusion in the argument,
in the last sentence, but this is a sub-conclusion. This sub-conclusion appears in the plankton
example, and like all examples, it is used to support the main conclusion.

33.

Premise: “Wine is made by crushing grapes and eventually separating the juice from the grape
skins.”

Counter-premise: “However, the separated juice contains impurities and many wineries do not filter
the juice.”

Premise: “These wineries claim the unfiltered juice ultimately produces a more flavorful and intense
wine.”

Premise: “Since these wine makers are experts,”

Conclusion: “we should trust their judgment and not shy away from unfiltered wine.”

By Sandeep Gupta, GMAT Perfect 800 | www.top-one-percent.com 47


THE MOST AUTHORITATIVE GUIDE EVER WRITTEN ON GMAT CRITICAL REASONING

34.

Premise: “Phenylketonurics are people who cannot metabolize the amino acid phenylalanine.”

Premise: “There are dangers associated with phenylketonuria, and products containing
phenylalanine must carry a warning label that states, ‘Phenylketonurics: contains phenylalanine.’”

Additional Premise: “In addition, all children in developed societies received a phenylketonuria test
at birth.”

Conclusion: “Hence, at the moment, we are doing as much as possible to protect against this
condition.”

35.

Premise: “During last night’s robbery, the thief was unable to open the safe.”

Counter-premise: “despite the fact that the thief stole several documents.”

Additional Premise: “After all, nothing in those documents was as valuable as the money in the safe.”

Conclusion: “Thus, last night’s robbery was unsuccessful.”

_______

With the mastery of all the component parts of an argument and of argument
structure, you are ready to deep-dive into solving Boldface questions in 30 seconds
flat. Let’s once recall the five steps:

By Sandeep Gupta, GMAT Perfect 800 | www.top-one-percent.com 48


THE MOST AUTHORITATIVE GUIDE EVER WRITTEN ON GMAT CRITICAL REASONING

Step 1: There is no need to read the entire argument. Your first step is to directly label
each boldface statement as Premise or Conclusion

Step 2: You must remember the other terms used for Premise and Conclusion

➢ Premise: (two types – challengeable and non-challengeable)

NON-CHALLENGEABLE PREMISES: Fact, Truth, Data, Evidence,


Information, Situation, Circumstance, Research, Proof, Observation, Example,
Finding, Phenomenon, Illustration, Acknowledgement, Pattern (in the past or
present)

CHALLENGEABLE PREMISES: Reason, Justification, Reasoning,


Explanation, Line of Reasoning, Support, Statement of support, Supposition,
Consideration, Policy, Practice, Generalization (about past or present),
Judgment in support, Acknowledgement

➢ Conclusion: Judgment reached, Opinion, Suggestion, View, Idea, Belief,


Proposal, Warning, Forecast, Claim, Stand, Prediction, Hypothesis, Position
(to posit), Stance, Point, Main Point, Generalization (about future), Contention,
Consequence, Pattern (guessed / predicted for the future)

Step 3: Check the presence of a contradiction word between the two Boldface
statements. The common contradiction words are:

Contradiction Cues: But, However, Nonetheless, Nevertheless,


Notwithstanding, Even so, Despite, Rather, Yet, On the other hand, Admittedly,
In contrast, By contrast, Contrary to, Although, Even though, Still, Whereas, In
spite of, After all, Alternatively, Apart from, Conversely, Regardless, Then again,
Unfortunately, Ironically etc.

This step helps us eliminate a few choices.

Step 4: Never forget:

➢ The role of a premise is to support a conclusion

➢ We can never challenge or support a fact

STEP 5: Go to the options and eliminate

By Sandeep Gupta, GMAT Perfect 800 | www.top-one-percent.com 49


THE MOST AUTHORITATIVE GUIDE EVER WRITTEN ON GMAT CRITICAL REASONING

Solve the questions that follow using the five steps. You will be (positively)
shocked to see your speed and accuracy.

1. Historian: In the Drindian Empire, censuses were conducted annually to


determine the population of each village. Village census records for the last
half of the 1600’s are remarkably complete. This very completeness makes
one point stand out; in five different years, villages overwhelmingly reported
significant population declines. Tellingly, each of those five years immediately
followed an increase in a certain Drindian tax. This tax, which was assessed on
villages, was computed by the central government using the annual census figures.
Obviously, whenever the tax went up, villages had an especially powerful economic
incentive to minimize the number of people they recorded; and concealing the size
of a village’s population from government census takers would have been easy.
Therefore, it is reasonable to think that the reported declines did not happen.
In the historian’s argument, the two portions in boldface play which of the
following roles?
A. The first supplies a context for the historian’s argument; the second acknowledges
a consideration that has been used to argue against the position the historian seeks
to establish.
B. The first presents evidence to support the position that the historian seeks to
establish; the second acknowledges a consideration that has been used to argue
against that position.
C. The first provides a context for certain evidence that supports the position that the
historian seeks to establish; the second is that position.
D. The first is a position for which the historian argues; the second is an assumption
that serves as the basis of that argument.
E. The first is an assumption that the historian explicitly makes in arguing for a
certain position; the second acknowledges a consideration that calls that
assumption into question.

By Sandeep Gupta, GMAT Perfect 800 | www.top-one-percent.com 50


THE MOST AUTHORITATIVE GUIDE EVER WRITTEN ON GMAT CRITICAL REASONING

My 30-second solution:

Boldface 1: Village census records for the last half of the 1600’s ARE (the word are
denotes a current reality, and denotes a fact) remarkably complete. This is a
Premise (FACT).

Boldface 2: (Therefore, it is reasonable to think that) the reported declines did not
happen. This is the CONCLUSION. The word therefore denotes a conclusion.

Now as per the terms we studied earlier, we know that “consideration” / “to
acknowledge” are terms for premise (fact). We also know that “position” is a term for
the conclusion. Check the options for this question now:

Options A, B, E say that ‘second (boldface)’ acknowledges a consideration (fact):


WRONG as per the above analysis. The second boldface is a ‘conclusion’.

Option D says that ‘first (boldface)’ is a position (conclusion): WRONG as per the
above analysis. The first boldface is a ‘fact’.

Option C is correct by elimination.

Once you become proficient, the above question will take no more than 30 seconds
(for you to get to its final answer).

Let’s solve one more question on the next page:

By Sandeep Gupta, GMAT Perfect 800 | www.top-one-percent.com 51


THE MOST AUTHORITATIVE GUIDE EVER WRITTEN ON GMAT CRITICAL REASONING

Astronomer: Observations of the Shoemaker-Levi comet on its collision course with


Jupiter showed that the comet broke into fragments before entering Jupiter’s
atmosphere in 1994, but they did not show how big those fragments were. In hopes of
gaining some indication of the fragments’ size, astronomers studied spectrographic
analyses of Jupiter’s outer atmosphere. These analyses revealed unprecedented traces
of sulfur after the fragments’ entry. Surprisingly, the fragments themselves
certainly contained no sulfur. Many astronomers consequently believe that the
cloud layer below Jupiter’s outer atmosphere does contain sulfur. Since sulfur would
have seeped into the outer atmosphere if comet fragments had penetrated this cloud
layer, it is likely that some of the fragments were at least large enough to
have passed through Jupiter’s outer atmosphere without being burned
up.

In the astronomer’s argument, the two portions in boldface play which of the following
roles?

A. The first presents a circumstance for which the astronomer offers an explanation;
the second part is that explanation.
B. The first acknowledges a consideration that weighs against the conclusion of the
argument; the second is that conclusion.
C. The first acknowledges a consideration that weights against the conclusion of the
argument; the second provides evidence in support of that conclusion.
D. The first provides evidence in support of the conclusion of the argument; the
second acknowledges a consideration that weighs against that conclusion.
E. The first is a judgment advanced in support of the conclusion of the argument; the
second is that conclusion.

By Sandeep Gupta, GMAT Perfect 800 | www.top-one-percent.com 52


THE MOST AUTHORITATIVE GUIDE EVER WRITTEN ON GMAT CRITICAL REASONING

My 30-second solution:
The first boldface, “The fragments themselves certainly contained no sulfur” is a FACT
(premise or evidence or supporting statement) | the word contained denotes past
tense – clear indicator of fact.

The second boldface, “it is likely that some of the fragments were at least large enough
to have passed through Jupiter’s outer atmosphere without being burned up” is a
conclusion. It is likely signals conclusion (is a conclusion indicator).

Also, we must note that there is no opposition in this argument by anyone else. This
is evident from the absence of any contradiction word between Boldface 1 and
Boldface 2.

A says second is the explanation (explanation is definitely not a conclusion): WRONG.

B, C, and D use the words ‘weigh against’ (but, as per the analysis above, there is no
opposition in this argument): WRONG.

‘Judgement in support’ means premise (refer to the terms). Ans. E

Let’s solve one more question on the next page:

By Sandeep Gupta, GMAT Perfect 800 | www.top-one-percent.com 53


THE MOST AUTHORITATIVE GUIDE EVER WRITTEN ON GMAT CRITICAL REASONING

Economist: Economists have long argued over the best way to measure the net wealth of a
nation's economy. On one side are those that favor the use of Gross Domestic
Product, or GDP, the market value of all goods and services produced within
the borders of a country within one year. On the other side are supporters of the use
of the GNP, a similar measure that, unlike GDP, takes into account foreign assets that are
owned by residents of a country. The debate over the accuracy of the GDP versus the GNP as
a measure of net wealth continues, but in reality, the strong population bias associated with
both statistics likely renders both inaccurate. In truth, per-capita GDP offers the clearest
picture, as it divides the total wealth by the number of individuals, which tells us the mean
income and average wealth and lifestyle of a nation's people. The difference between GDP /
GNP and per-capita GDP is stark. According to the former measures, China has recently
overtaken Japan as the second wealthiest country. If we look at per-capita GDP,
however, Japan, China, and even the United States do not make the top ten,
while small wealthy nations like Denmark, Switzerland, and Luxembourg
routinely rank higher.

In the economist's argument, the two portions in boldface play which of the following roles?
A. The first introduces one side of a dichotomy; the second offers a concrete example of the
dichotomy.
B. The first expresses an opinion about one perspective on a dichotomy; the second
describes premise undermining the dichotomy itself.
C. The first describes a general topic; the second offers an example of an alternative
approach to that topic.
D. The first is a common argument about a subject; the second is the author's conclusion.
E. The first identifies one side of a dichotomy; the second offers an example showing the
dichotomy to be false.

By Sandeep Gupta, GMAT Perfect 800 | www.top-one-percent.com 54


THE MOST AUTHORITATIVE GUIDE EVER WRITTEN ON GMAT CRITICAL REASONING

My 30-second solution:

We see that both the boldface statements are FACTS. Here is how:
BF1: On one side are those that favor the use of Gross Domestic Product, or GDP, the market
value of all goods and services produced within the borders of a country within one year.

The word are above denotes reality, and hence denotes a fact.

BF 2: If we look at per-capita GDP, however, Japan, China, and even the United States do not
make the top ten, while small wealthy nations like Denmark, Switzerland, and Luxembourg
routinely rank higher.

All the italicized portions denote a current reality, and hence denote a fact.

Since both BF1 and BF2 are facts, option B (opinion means a conclusion), and D (an
argument must contain a conclusion) are immediately eliminated.

C says Boldface 1 is a general topic. Terribly wrong! A general topic is something


that most people can discuss / have an opinion on. The passage presents a topic
related to international economics (not a general topic), making C instantly wrong.
Also, C is wrong because the second boldface doesn’t provide an approach. An
approach will always show how to do something, which is not the case here. The
second boldface just presents a simple fact.

This leaves us with A and E.

Please notice the words “but in reality, the strong population bias associated with
both statistics likely renders both inaccurate. In truth, per-capita GDP offers the
clearest picture (the opinion / conclusion of the author)

The word but tells us that the two Boldface statements are in opposition to each other.

Option A says that the second boldface is a concrete example of the dichotomy, which,
in a way, indicates that the second Boldface is supporting the first Boldface.
Absolutely wrong!

Option E says ‘showing the dichotomy to be false’, clearly establishing that the two
boldface statements are supporting two different conclusions, and hence are in
opposition to each other. E it is!

By Sandeep Gupta, GMAT Perfect 800 | www.top-one-percent.com 55


THE MOST AUTHORITATIVE GUIDE EVER WRITTEN ON GMAT CRITICAL REASONING

Sentence 1 (of the argument) outlines a debate going on among economists, and sentence two (BF-1),
our first boldface portion, presents us the first side of that debate on a platter, starting with the
words on one side. If we look at the second, we need to decide whether it 1) presents an example and
2) goes against the earlier dichotomy presented in the passage. Is the second boldface exemplifying
something? Sure, it is. There is a comparison of per-capita GDP, in general terms, between three
countries that rank high under traditional measures of the wealth of an economy and three other
countries that do not. This part checks out, then. What about whether the example flies in the face of
the dichotomy? Again, this checks out. The transition however cues us in on the opposition of this line
to the one that preceded it, the line that was framed, according to the former measures. In other
words, the old measures inaccurately describe the wealth of nations. The new view espoused by the
economist ought to be adopted instead. There is nothing to argue against here.

Let’s solve one more question:

By Sandeep Gupta, GMAT Perfect 800 | www.top-one-percent.com 56


THE MOST AUTHORITATIVE GUIDE EVER WRITTEN ON GMAT CRITICAL REASONING

Economist: Scholars in economics and public policy have long argued over the best way to
measure emotions such as love, anger, and jealousy in quantifiable terms that can both
enhance businesses' capacity to fulfill customer needs and allow the government to
implement policies that best address issues of national concern. On one side are those
who believe scholars can conduct research on emotions to such an extent that
it is possible to discover the financial value an individual places on such
principles as health, the safety of one's children, and the worth of higher
education. Sophisticated preference studies assessing a population's decisions on a very
atomic level are used to conduct these investigations. On the other side are those that believe
such studies, however perfected, are best only vague estimates of how emotions factor into
decisions. Those who support this belief cite numerous examples of situations
(an expected trend or pop culture success) in which a population's actual
behavior disrupted scholars' meticulously constructed mathematical models. It
remains to be seen whether mathematical models regarding decision sciences will ever reach
a higher degree of accuracy.
In the economist's argument, the two highlighted portions play which of the following roles?
A. The first introduces one side of a dichotomy; the second offers a concrete example of that
dichotomy.
B. The first discusses a common viewpoint about a dichotomy; the second describes premise
undermining the dichotomy itself.
C. The first describes a general topic; the second offers an example of an alternative
approach to that topic.
D. The first is a common argument about a subject; the second is the author's conclusion.
E. The first identifies one side of a dichotomy; the second indicates how one side supports
its argument.

By Sandeep Gupta, GMAT Perfect 800 | www.top-one-percent.com 57


THE MOST AUTHORITATIVE GUIDE EVER WRITTEN ON GMAT CRITICAL REASONING

My 30-second solution:

We can surely say that this isn’t a general topic / common viewpoint / common
argument; this instantly eliminates B, C, and D.

Focus on the portion: “On the other side are those that believe (means this is an
opinion / conclusion) such studies, however (contradiction word) perfected, are
best only vague estimates of how emotions factor into decisions – clearly an opinion

The word however confirms that the two Boldface statements aren’t supporting each
other.

Using the logic of the previous example, option A can’t be correct.

Option A says that the second boldface is a concrete example of the dichotomy, which,
in a way, indicates that the second Boldface is supporting the first Boldface.
Absolutely wrong!

By the process of elimination, choice E is correct.

Let’s take another example:

By Sandeep Gupta, GMAT Perfect 800 | www.top-one-percent.com 58


THE MOST AUTHORITATIVE GUIDE EVER WRITTEN ON GMAT CRITICAL REASONING

It has been commonly accepted for some time now that certain scenes in
Shakespeare’s Macbeth are interpolations from the writing of another author: act III,
scene 5, and parts of act IV, scene 1, have been determined to be the
writing of one of his contemporaries, Thomas Middleton. This can be
regarded as both illuminating and problematic, depending upon how the play is being
studied. It allows us to infer a great deal about the conventions and practices of writing
for the stage at the time. For example, playwriting may have been more collaborative
than previously thought, or perhaps Elizabethan notions of plagiarism were different
from ours. This does complicate our interpretation of the characters in the
play; it is more difficult to assess authorial intention with regard to a
character’s motives if the text has been redacted by multiple authors.

In the passage, the two boldface statements play which of the following roles?

A. The first explains a concept, and the second presents an example of that concept.
B. The first presents an example of the main subject of the passage, and the second is
a conclusion based on that example.
C. The first states the conclusion of the argument as a whole, and the second provides
support for that conclusion.
D. The first provides evidence for a conclusion that the passage as a whole opposes,
and the second presents the objection to that conclusion.
E. The first states the primary conclusion of the passage, and the second states the
secondary conclusion.

By Sandeep Gupta, GMAT Perfect 800 | www.top-one-percent.com 59


THE MOST AUTHORITATIVE GUIDE EVER WRITTEN ON GMAT CRITICAL REASONING

It has been commonly accepted for some time now that certain scenes in
Shakespeare’s Macbeth are interpolations from the writing of another author: (the
colon suggests that the first boldface is an example of what is written before the
colon) act III, scene 5, and parts of act IV, scene 1, have been determined
to be the writing of one of his contemporaries, Thomas Middleton.

So, the first boldface statement is used as an example of the interpolations that the
first part of the sentence mentions (that parts of Shakespeare’s work were in fact
written by his peer Middleton).

BF 2: This does complicate our interpretation of the characters in the play;


it is more difficult to assess authorial intention with regard to a
character’s motives if the text has been redacted by multiple authors.

This is surely an opinion. According to this statement, because others wrote certain
parts of Shakespeare’s work, it is more difficult to determine a character’s motives.
So, the first boldface is an example, and the second boldface is an opinion / conclusion
(opinion is the same as conclusion). There is no contradiction word between the first
and the second boldface.

Correct! Option (B) matches our analysis perfectly.

The other choices miss the mark completely.

• Choice (A) incorrectly states that the second boldface portion is the example.
• Similarly, choice (C) indicates that the first boldface is the opinion and the second is the
evidence, the exact opposite of our analysis.
• (D) states that the passage opposes an argument, but there is no conflict addressed in the
passage.
• Choice (E) identifies both statements as conclusions, which is not correct.

Single boldface questions:


Sometimes only one part of the argument is made BOLD. Let’s solve one such
question:

By Sandeep Gupta, GMAT Perfect 800 | www.top-one-percent.com 60


THE MOST AUTHORITATIVE GUIDE EVER WRITTEN ON GMAT CRITICAL REASONING

Government official: A satisfactory way of eliminating chronic food shortages in our


country is not easily achievable. Direct aid from other countries in the form of food
shipments tends to undermine our prospects for long-term agricultural self-
sufficiency. If external sources of food are delivered effectively by external
institutions, local food producers and suppliers are forced out of business. On the
other hand, foreign capital funneled to long-term development projects
would inject so much cash into our economy that inflation would drive
the price of food beyond the reach of most of our citizens.

The boldface part plays which one of the following roles in the government official's
argument?

A. It supports the claim that the official's country must someday be agriculturally self-
sufficient.

B. It supports the claim that there is no easy solution to the problem of chronic food
shortages in the official's country.

C. It is supported by the claim that the official's country must someday be


agriculturally self-sufficient.

D. It supports the claim that donations of food from other countries will not end the
chronic food shortages in the official's country.

E. It is supported by the claim that food producers and suppliers in the official's
country may be forced out of business by donations of food from other countries.

Here, the first statement (A satisfactory way of eliminating chronic food shortages
in our country is not easily achievable) is the conclusion of the argument and all the
other statements that follow are premises that support this conclusion.

If it is not clear, use The Therefore Test.

Which relationship makes more sense?

By Sandeep Gupta, GMAT Perfect 800 | www.top-one-percent.com 61


THE MOST AUTHORITATIVE GUIDE EVER WRITTEN ON GMAT CRITICAL REASONING

Scenario #1:

Eliminating chronic food shortages in our country is not easily achievable.


THEREFORE, foreign capital would drive the price of food beyond the reach of most
of our citizens.

Scenario #2:

Foreign capital would drive the price of food beyond the reach of most of our citizens.
THEREFORE, eliminating chronic food shortages in our country is not easily
achievable.

In terms of an order of reasoning, scenario #2 makes sense.

So, the Boldface statement is a premise supporting the main conclusion (claim) given
in statement 1. Please make sure you remember all the terms. If you recall, the word
claim is used for conclusion.

Options C and E say that the Boldface statement is supported. We know that the any
premise can never be supported, so C and E are eliminated.

Options A and D present the wrong claim. Out! The correct claim is stated in the first
boldface.

B it is!

Let’s take another example:

By Sandeep Gupta, GMAT Perfect 800 | www.top-one-percent.com 62


THE MOST AUTHORITATIVE GUIDE EVER WRITTEN ON GMAT CRITICAL REASONING

It is well documented that people have positive responses to some words, such as
“kind” and “wonderful” and negative responses to others, such as “evil” and “nausea.”
Recently, psychological experiments have revealed that people also have positive
or negative responses to many nonsense words. This shows that people’s
responses to words are conditioned not only by what the words mean, but also by how
they sound.

The boldface portion plays which one of the following roles in the argument?

A. It is a premise offered in support of the conclusion that people have either a positive
or a negative response to any word.
B. It is a conclusion for which the only support provided is the claim that people’s
responses to words are conditioned both by what the words mean and by how they
sound.
C. It is a generalization partially supported by the claim that meaningful words can
trigger positive or negative responses in people.
D. It is a premise offered in support of the conclusion that people’s responses to words
are engendered not only by what the words mean, but also by how they sound.
E. It is a conclusion supported by the claim that people’s responses under
experimental conditions are essentially different from their responses in ordinary
situations.

Premise: It is well documented (fact) that people have positive responses to some
words, such as “kind” and “wonderful” and negative responses to others, such as “evil”
and “nausea.”

Premise: Recently, psychological experiments have revealed that (fact) people


also have positive or negative responses to many nonsense words.

Conclusion: This shows that (conclusion) people’s responses to words are


conditioned not only by what the words mean, but also by how they sound.

The boldface statement is a fact supporting the conclusion. The argument is pretty
straightforward. No contradiction / no opposition / no other conclusion.

By Sandeep Gupta, GMAT Perfect 800 | www.top-one-percent.com 63


THE MOST AUTHORITATIVE GUIDE EVER WRITTEN ON GMAT CRITICAL REASONING

A. It is a premise offered in support of the conclusion that people have either a positive or
a negative response to any word (this is not the conclusion of the argument; the
conclusion is as identified above)
B. It is a conclusion (OUT) for which the only support provided is the claim that people’s
responses to words are conditioned both by what the words mean and by how they sound.
C. It is a generalization (this is the result of just one study; can’t be called a
generalization) partially supported by the claim (wrong) that meaningful words can
trigger positive or negative responses in people. (this part is given as a fact)
D. Correct: It is a premise offered in support of the conclusion that people’s responses to
words are engendered not only by what the words mean, but also by how they sound.
Bingo!
E. It is a conclusion (wrong) supported by the claim (claim means a conclusion; as we
know conclusions never support anything) that people’s responses under experimental
conditions are essentially different from their responses in ordinary situations.

Let’s take another example:

Psychologist: The obligation to express gratitude cannot be fulfilled


anonymously. However much society may have changed over the centuries, human
psychology is still driven primarily by personal interaction. Thus, the important social
function of positively reinforcing those behaviors that have beneficial consequences
for others can be served only if the benefactor knows the source of the gratitude.

The boldface portion plays which one of the following roles in the argument?

A. It is an illustration of a premise that is used to support the argument’s conclusion.


B. It is used to counter a consideration that might be taken to undermine the
argument’s conclusion.
C. It is used to support indirectly a claim that the argument in turn uses to support
directly the conclusion.
D. It is used to identify the social benefit with which the argument is concerned.
E. It is the conclusion that the argument is intended to support.

By Sandeep Gupta, GMAT Perfect 800 | www.top-one-percent.com 64


THE MOST AUTHORITATIVE GUIDE EVER WRITTEN ON GMAT CRITICAL REASONING

Premise: However much society may have changed over the centuries, human
psychology is still driven primarily by personal interaction.

Sub-conclusion / Premise: Thus, the important social function of positively


reinforcing those behaviors [of expressing gratitude] that have beneficial
consequences for others can be served only if the benefactor knows the source of the
gratitude.

Main conclusion: The obligation to express gratitude cannot be fulfilled


anonymously.

If you are not sure which statement is the main conclusion and which statement is the
sub-conclusion, use the therefore test.

BECAUSE The obligation to express gratitude cannot be fulfilled anonymously. SO,


the important social function of positively reinforcing those behaviors that have
beneficial consequences for others can be served only if the benefactor knows the
source of the gratitude.

Makes no sense!

BECAUSE the important social function of positively reinforcing those behaviors that
have beneficial consequences for others can be served only if the benefactor knows
the source of the gratitude. SO, the obligation to express gratitude cannot be fulfilled
anonymously.

Makes complete sense.

So, the boldface statement is the conclusion. E is the straightforward answer.

Let’s still eliminate all the others.

A. It is an illustration of a premise (wrong) that is used to support (wrong) the


argument’s conclusion.
B. It is used to counter a consideration (means challenge a fact – never correct in RC
and CR, this choice is wrong) that might be taken to undermine the argument’s
conclusion.

By Sandeep Gupta, GMAT Perfect 800 | www.top-one-percent.com 65


THE MOST AUTHORITATIVE GUIDE EVER WRITTEN ON GMAT CRITICAL REASONING

C. It is used to support (wrong – supporting statements are never the final


conclusions) indirectly a claim that the argument in turn uses to support directly
the conclusion.
D. It is used to identify the social benefit with which the argument is concerned. This
sentence better describes the last sentence. Junk answer!
E. Correct! It is the conclusion that the argument is intended to support.
The psychologist’s argument is structured as follows: Premise: However much society may have
changed over the centuries, human psychology is still driven primarily by personal interaction. Sub-
conclusion / Premise: Thus, the important social function of positively reinforcing those
behaviours [of expressing gratitude] that have beneficial consequences for others can be served only if
the benefactor knows the source of the gratitude. Main conclusion: The obligation to express gratitude
cannot be fulfilled anonymously.

Often, the identifiers used before the subsidiary conclusions are dramatic and somewhat misleading,
such as “clearly” and “obviously.” In these cases, the conclusion is neither clear nor obvious, and those
words are used to lead the reader into thinking that the conclusion should simply be accepted without
further analysis. The statement referenced in the question stem is the main conclusion of the argument,
and as the answer choice correctly describes, the conclusion that the argument is designed to support.
The argument, when considered in terms of order, is “out of order”: First sentence = main conclusion,
Second sentence = basic premise, Third sentence = sub-conclusion. The last sentence is a sub-
conclusion, and in an answer choice the author can describe a sub-conclusion in a variety of ways:
subsidiary conclusion / secondary conclusion / intermediate conclusion / supporting conclusion.

Note that as predicted, the main conclusion is not modified by a conclusion indicator but the sub-
conclusion is.

Answer choice (A): The statement in question is not an example of an idea raised in a premise.
Answer choice (B): The answer choice describes a premise that is used to defend the argument from
attack. This would better describe the second sentence of the argument.
Answer choice (C): This answer choice describes a premise supporting a sub-conclusion. Again, this
would better describe the second sentence of the argument.
Answer choice (D): This answer choice better describes the last sentence.
Answer choice (E): This is the correct answer. By consistently breaking down the structure of the
argument before reading the answer choices, these problems become very easy to solve.

Let’s see another question:

By Sandeep Gupta, GMAT Perfect 800 | www.top-one-percent.com 66


THE MOST AUTHORITATIVE GUIDE EVER WRITTEN ON GMAT CRITICAL REASONING

Nutritionist: Because humans have evolved very little since the development of
agriculture, it is clear that humans are still biologically adapted to a diet of
wild foods, consisting mainly of raw fruits and vegetables, nuts and seeds, lean meat,
and seafood. Straying from this diet has often resulted in chronic illness and other
physical problems. Thus, the more our diet consists of wild foods, the healthier we will
be.

The boldface portion plays which one of the following roles in the argument?

A. It is a conclusion for which the only support offered is the claim that straying from
a diet of wild foods has often resulted in chronic illness and other physical
problems.
B. It is a premise for which no justification is provided, but which is used to support
the argument’s main conclusion.
C. It is a phenomenon for which the main conclusion of the nutritionist’s argument is
cited as an explanation.
D. It is an intermediate conclusion for which one claim is offered as support, and
which is used in turn to support the argument’s main conclusion.
E. It is a premise offered in support of the claim that humans have evolved very little
since the development of agriculture.

Premise: Humans have evolved very little (fact) since the development of
agriculture.

Sub-conclusion: It is clear (denotes some kind of conclusion) that humans are


still biologically adapted to a diet of wild foods, consisting mainly of raw fruits and
vegetables, nuts and seeds, lean meat, and seafood.

Premise: Straying from this diet has often resulted (fact) in chronic illness and
other physical problems.

Conclusion: Thus, the more our diet consists of wild foods, the healthier we will be
– this is a prediction based on all the preceding statements … or the final logical
conclusion of the passage. The boldface statement is a sub-conclusion.

By Sandeep Gupta, GMAT Perfect 800 | www.top-one-percent.com 67


THE MOST AUTHORITATIVE GUIDE EVER WRITTEN ON GMAT CRITICAL REASONING

A. It is a conclusion (wrong) for which the only support (wrong) offered is the
claim (claim means conclusion and conclusion never supports anything) that
straying from a diet of wild foods has often resulted in chronic illness and other
physical problems.
B. It is a premise (wrong) for which no justification is provided (wrong: there is
support for the boldface portion), but which is used to support the argument’s main
conclusion.
C. It is a phenomenon for which the main conclusion of the nutritionist’s argument
is cited as an explanation.
The main conclusion does not explain anything, so this answer choice is incorrect.
The answer would be much improved if it said: “It is a phenomenon that helps
explain the main conclusion of the nutritionist’s argument.”
D. BINGO! It is an intermediate conclusion for which one claim is offered as
support, and which is used in turn to support the argument’s main conclusion.
E. It is a premise (wrong) offered in support of the claim that humans have evolved
very little since the development of agriculture.
This is a Reverse answer. The claim that humans have evolved very little since the
development of agriculture is a premise offered in support of the statement
referenced in the question stem.
Nutritionist view:(Sub-conclusion/Premise): It is clear that humans are still biologically adapted to a
diet of wild foods, consisting mainly of raw fruits and vegetables, nuts and seeds, lean meat, and
seafood.
Nutritionist reason for this view: Humans have evolved very little since the development of
agriculture.
Nutritionist observation: Premise: Straying from this diet has often resulted in chronic illness and
other physical problems.
Nutritionist conclusion based on his view and observation: Main Conclusion: Thus, the more our diet
consists of wild foods, the healthier we will be.
The statement referenced in the question stem is a sub-conclusion/intermediate conclusion.
(D) It is an intermediate conclusion for which one claim is offered as support, and which is used in
turn to support the argument’s main conclusion.
The boldfaced statement is supported by the statement/reason "humans have evolved very little since
the development of agriculture".
One claim/reasoning is offered as support- "humans have evolved very little since the development of
agriculture"
Also the boldfaced statement, which is an intermediate conclusion, is used to support (draw) the final
conclusion of the argument.

By Sandeep Gupta, GMAT Perfect 800 | www.top-one-percent.com 68


THE MOST AUTHORITATIVE GUIDE EVER WRITTEN ON GMAT CRITICAL REASONING

Exercise:
1. City Official: At City Hospital, uninsured patients tend to have shorter stays and fewer
procedures performed than do insured patients, even though insured patients, on average, have
slightly less serious medical problems at the time of admission to the hospital than uninsured
patients have. Critics of the hospital have concluded that the uninsured patients are not
receiving proper medical care. However, this conclusion is almost certainly false.
Careful investigation has recently shown two things: insured patients have much longer stays in
the hospital than necessary, and they tend to have more procedures performed than are
medically necessary. In the city official’s argument, the two boldface portions play which of the
following roles?

A. The first states the conclusion of the city official’s argument; the second provides support for
that conclusion.

B. The first is used to support the conclusion of the city official’s argument; the second states
that conclusion.

C. The first was used to support the conclusion drawn by hospital critics; the second states the
position that the city official’s argument opposes.

D. The first was used to support the conclusion drawn by hospital critics; the second provides
support for the conclusion of the city official’s argument.

E. The first states the position that the city official’s argument opposes; the second states the
conclusion of the city official’s argument.

By Sandeep Gupta, GMAT Perfect 800 | www.top-one-percent.com 69


THE MOST AUTHORITATIVE GUIDE EVER WRITTEN ON GMAT CRITICAL REASONING

2. Delta products, Inc., has recently switched at least partly from older technologies
using fossil fuels to new technologies powered by electricity. The question has been
raised whether it can be concluded that for a given level of output, Delta’s operation now causes
less fossil fuel to be consumed than it did formerly. The answer, clearly, is yes, since the amount
of fossil fuel used to generate the electricity needed to power the new technologies
is less than the amount needed to power the older technologies, provided that the
level of output is held constant. In the argument given, the two boldface portions play
which of the following roles?

A. The first is conclusion of the argument; the second provides support for that conclusion.

B. The first provides support for the conclusion of the argument; the second is the conclusion
of the argument.

C. The first states the position that the argument opposes; the second states the conclusion of
the argument.

D. The first states the position that the argument challenges; the second states the evidence in
support of that challenge.

E. Each provides support for the conclusion of the argument.

By Sandeep Gupta, GMAT Perfect 800 | www.top-one-percent.com 70


THE MOST AUTHORITATIVE GUIDE EVER WRITTEN ON GMAT CRITICAL REASONING

3. Ecologist: The Scottish Highlands were once the site of extensive forests, but these forests have
mostly disappeared and been replaced by peat bogs. The common view is that the Highlands’
deforestation was caused by human activity, especially agriculture. However, agriculture
began in the Highlands less than 2,000 years ago. Peat bogs, which consist of compressed
decayed vegetable matter, build up by only about one foot per 1,000 years and, throughout the
Highlands, remains of trees in peat bogs are almost all at depths great than four
feet. Since climate changes that occurred between 7,000 and 4,000 years ago favored the
development of peat bogs rather than the survival of forests, the deforestation was more likely
the result of natural processes than of human activity. In the ecologist’s argument the two
portions in boldface play which of the following roles?

A. The first is evidence that has been used in support of a position that the ecologist rejects; the
second is a finding that the ecologist uses to counter that evidence.

B. The first is evidence that, in light of the evidence provided in the second, serves as grounds
for the ecologist’s rejection of a certain position.

C. The first is a position that the ecologist rejects; the second is evidence that has been used in
support of that position.

D. The first is a position that the ecologist rejects; the second provides evidence in support of
that rejection.

E. The first is a position for which the ecologist argues; the second provides evidence to support
that position.

By Sandeep Gupta, GMAT Perfect 800 | www.top-one-percent.com 71


THE MOST AUTHORITATIVE GUIDE EVER WRITTEN ON GMAT CRITICAL REASONING

4. Scientists typically do their most creative work before the age of forty. It is commonly thought
that this happens because aging by itself brings about a loss of creative capacity.
However, studies show that a disproportionately large number of the scientists who
produce highly creative work beyond the age of forty entered their field at an older
age than is usual. Since by the age of forty the large majority of scientists have been working
in their field for at least fifteen years, the studies’ finding strongly suggests that the real reason
why scientists over forty rarely produce highly creative work is not that they have simply aged
but rather that they generally have spent too long in a given field. In the argument given, the
two portions in boldface play which of the following roles?
A. The first is the position that the argument as a whole opposes; the second is an objection that has
been raised against a position defended – wrong – there is no one defending any position
(conclusion) in the argument.
B. The first is a claim that has been advanced in support of a position that the argument opposes;
the second is a finding that has been used in support of that position.
C. The first is an explanation that the argument challenges; the second is a finding that has been
used in support of explanation presented in the first.
D. The first is an explanation that the argument challenges; the second is a finding on which that
challenge is based.
E. The first is an explanation that the argument defends; the second is a finding that has been used
to challenge that explanation.

By Sandeep Gupta, GMAT Perfect 800 | www.top-one-percent.com 72


THE MOST AUTHORITATIVE GUIDE EVER WRITTEN ON GMAT CRITICAL REASONING

5. Several of a certain bank’s top executives have recently been purchasing shares in
their own bank. This activity has occasioned some surprise, since it is widely believed that the
bank, carrying a large number of bad loans, is on the brink of collapse. Since the executives are
well placed to know their bank’s true condition, it might seem that their share purchases show
that the danger of collapse is exaggerated. The available information about the bank’s condition
is from reliable and informed sources, and corporate executives do sometimes buy shares in their
own company in a calculated attempt to calm worries about their company’s condition. On
balance, therefore, it is likely that the executives of the bank are following this
example. In the argument given, the two boldfaced portions play which of the following roles?

A. The first describes the circumstance the explanation of which is the issue that the argument
addresses; the second states the main conclusion of the argument.

B. The first describes the circumstance the explanation of which is the issue the argument
addresses; the second states a conclusion that is drawn in order to support the main
conclusion of the argument.

C. The first provides evidence to defend the position that the argument seeks to establish
against opposing positions; the second states the evidence that challenges the main
conclusion of the argument.

D. The first provides evidence to support the position that the argument seeks to establish; the
second states a conclusion that is drawn in order to support the argument’s main conclusion.

E. Each provides evidence to support the position that the argument seeks to establish.

By Sandeep Gupta, GMAT Perfect 800 | www.top-one-percent.com 73


THE MOST AUTHORITATIVE GUIDE EVER WRITTEN ON GMAT CRITICAL REASONING

6. As a large corporation in a small country, Hachnut wants its managers to have international
experience, so each year it sponsors management education abroad for its
management trainees. Hachnut has found, however, that the attrition rate of graduates from
this program is very high, with many of them leaving Hachnut to join competing firms soon after
completing the program. Hachnut does use performance during the program as a criterion in
deciding among candidates for management positions, but both this function and the goal
of providing international experience could be achieved in other ways. Therefore, if
the attrition problem cannot be successfully addressed, Hachnut should discontinue the
sponsorship program. In the argument given, the two boldfaced portions play which of the
following roles?

A. The first describes a practice that the argument seeks to justify; the second states a judgment
that is used in support of a justification for that practice.

B. The first describes a practice that the argument seeks to explain; the second presents part of
the argument’s explanation of that practice.

C. The first introduces a practice that the argument seeks to evaluate; the second provides
grounds for holding that the practice can never achieve its objective.

D. The first introduces a policy that the argument seeks to evaluate; the second provides
grounds for holding that the policy is not needed.

E. The first introduces a consideration supporting a policy that the argument seeks to evaluate;
the second provides evidence for concluding that the policy should be abandoned.

By Sandeep Gupta, GMAT Perfect 800 | www.top-one-percent.com 74


THE MOST AUTHORITATIVE GUIDE EVER WRITTEN ON GMAT CRITICAL REASONING

7. Plant scientists have used genetic engineering on seeds to produce crop plants that
are highly resistant to insect damage. Unfortunately, the seeds themselves are quite
expensive, and the plants require more fertilizer and water to grow well than normal ones.
Consequently, for most farmers the savings on pesticides cannot compensate for the
higher seed costs and the cost of additional fertilizer. However, since consumer demand
for grains, fruits, and vegetables grown without the use of pesticides continues to rise, the use of
genetically engineered seeds of this kind is likely to become widespread.
In the argument given, the two portions in boldface play which of the following roles?
A. The first supplies a context for the argument; the second is the argument's main conclusion.
B. The first introduces a development that the argument predicts will have a certain outcome; the
second is a state of affairs that, according to the argument, contributes to bringing about that
outcome.
C. The first presents a development that the argument predicts will have a certain outcome; the
second acknowledges a consideration that tends to weigh against that prediction.
D. The first provides evidence to support a prediction that the argument seeks to defend; the second
is that prediction.
E. The first and the second each provide evidence to support the argument's main conclusion.

By Sandeep Gupta, GMAT Perfect 800 | www.top-one-percent.com 75


THE MOST AUTHORITATIVE GUIDE EVER WRITTEN ON GMAT CRITICAL REASONING

8. Chaco Canyon, a settlement of the ancient Anasazi culture in North America, had massive
buildings. It must have been a major Anasazi center. Analysis of wood samples shows that
some of the timber for the buildings came from the Chuska and San Mateo mountains, 50 miles
from Chaco Canyon. Only a major cultural center would have the organizational
power to import timber from 50 miles away.
In the argument given, the two portions in boldface play which of the following roles?
A. The first is a premise used to support the argument's main conclusion; the second is the
argument's main conclusion.
B. The first is the argument's main conclusion; the second is a premise used to support that
conclusion.
C. The first is one of two premises used to support the argument's main conclusion; the second is
the other of those two premises.
D. The first is a premise used to support the argument's main conclusion; the second is a premise
used to support another conclusion drawn in the argument.
E. The first is inferred from another statement in the argument; the second is inferred from the
first.

By Sandeep Gupta, GMAT Perfect 800 | www.top-one-percent.com 76


THE MOST AUTHORITATIVE GUIDE EVER WRITTEN ON GMAT CRITICAL REASONING

9. Economist: Tropicorp, which constantly seeks profitable investment opportunities, has been
buying and clearing sections of tropical forest for cattle ranching, although pastures newly
created there become useless for grazing after just a few years. The company has not gone into
rubber tapping, even though greater profits can be made from rubber tapping, which leaves the
forest intact. Thus, some environmentalists argue that Tropicorp's actions do not serve
even its own economic interest. However, the initial investment required for a successful
rubber-tapping operation is larger than that needed for a cattle ranch; there is a shortage of
workers employable in rubber-tapping operations; and taxes are higher on profits from rubber
tapping than on profits from cattle ranching. Consequently, the environmentalists'
conclusion is probably wrong.
In the economist's argument, the two boldface portions play which of the following roles?
A. The first supports the conclusion of the economist's argument; the second calls that conclusion
into question.
B. The first states the conclusion of the economist's argument; the second supports that conclusion.
C. The first supports the conclusion of the environmentalists' argument; the second states that
conclusion.
D. The first states the conclusion of the environmentalists' argument; the second states the
conclusion of the economist's argument.
E. Each supports the conclusion of the economist's argument.

By Sandeep Gupta, GMAT Perfect 800 | www.top-one-percent.com 77


THE MOST AUTHORITATIVE GUIDE EVER WRITTEN ON GMAT CRITICAL REASONING

10. Economist: The price of tap water in our region should be raised drastically. Supplies in local
freshwater reservoirs have been declining for years because water is being used faster
than it can be replenished. Since the price of tap water has been low, few users have bothered
to adopt even easy conservation measures.
The two sections in boldface play which of the following roles in the economist's argument?
A. The first is a conclusion for which support is provided, and which in turn supports the main
conclusion; the second is the main conclusion.
B. The first is an observation for which the second provides an explanation; the second is the main
conclusion but not the only conclusion.
C. The first is a premise supporting the argument's main conclusion; so is the second.
D. The first is the only conclusion; the second provides an explanation for the first.
E. The first is the main conclusion; the second is a conclusion for which support is provided, and
which in turn supports the first.

By Sandeep Gupta, GMAT Perfect 800 | www.top-one-percent.com 78


THE MOST AUTHORITATIVE GUIDE EVER WRITTEN ON GMAT CRITICAL REASONING

Solutions:
1.

My 10-second solution:

BF 1: Critics of the hospital have concluded that the uninsured patients are not receiving proper
medical care: conclusion of the critics

BF 2: However, this conclusion is almost certainly false: opinion (conclusion) of the author (City
Official)

Both the boldface statements are conclusions.

The two conclusions are in opposition (notice the word however)

The word support isn’t used for a conclusion. So, A, B, C, and D are eliminated in a flash. Ans. E

A. The first states the conclusion of the city official’s argument (wrong); the second provides
support (wrong) for that conclusion.

B. The first is used to support (wrong) the conclusion of the city official’s argument; the second
states that conclusion.

C. The first was used to support (wrong) the conclusion drawn by hospital critics; the second
states the position that the city official’s argument opposes (wrong).

D. The first was used to support (wrong) the conclusion drawn by hospital critics; the second
provides support (wrong) for the conclusion of the city official’s argument.

E. The first states the position that the city official’s argument opposes; the second states the
conclusion of the city official’s argument. Correct!

By Sandeep Gupta, GMAT Perfect 800 | www.top-one-percent.com 79


THE MOST AUTHORITATIVE GUIDE EVER WRITTEN ON GMAT CRITICAL REASONING

2.

My 10-second solution:

BF 1: Delta products, Inc., has recently switched at least partly from older technologies using fossil
fuels to new technologies powered by electricity: FACT

BF 2: Reason for the conclusion: since (denotes a premise / a fact / a supporting statement / a
reason) the amount of fossil fuel used to generate the electricity needed to power the new
technologies is less than the amount needed to power the older technologies, provided that the level
of output is held constant: FACT

Conclusion: The answer, clearly, is yes

There is no contradiction word between the first and the second boldface. So, there no opposition in
the argument.

The role of the facts is to support the conclusion … Answer E

A. The first is conclusion (wrong) of the argument; the second provides support for that
conclusion.

B. The first provides support for the conclusion of the argument; the second is the conclusion
(wrong) of the argument.

C. The first states the position (wrong) that the argument opposes (wrong); the second states
the conclusion (wrong) of the argument.

D. The first states the position (wrong) that the argument challenges (wrong); the second
states the evidence in support of that challenge (wrong).

E. Each provides support for the conclusion of the argument.

By Sandeep Gupta, GMAT Perfect 800 | www.top-one-percent.com 80


THE MOST AUTHORITATIVE GUIDE EVER WRITTEN ON GMAT CRITICAL REASONING

3.

My 10-second solution:

BF 1: FACT: However, agriculture began in the Highlands less than 2,000 years ago. Began denotes
past tense, which is obviously a fact.

Remember, the word however is not between the two BFs, so it doesn’t count.

BF 2: FACT: throughout the Highlands, remains of trees in peat bogs are almost all at depths great
than four feet. Are denotes that the statement is a fact.

Options C, D, E say that the first BF is a position (means conclusion) – WRONG

Option A says counter that evidence (means challenge a fact, automatically wrong – challenging a
fact is not allowed in CR and RC)

Ans. B (in 10 seconds flat!)

A. The first is evidence that has been used in support of a position that the ecologist rejects; the
second is a finding that the ecologist uses to counter that evidence (WRONG).

B. The first is evidence that, in light of the evidence provided in the second, serves as grounds
for the ecologist’s rejection of a certain position. Correct!

C. The first is a position (WRONG) that the ecologist rejects; the second is evidence that has
been used in support of that position.

D. The first is a position (WRONG) that the ecologist rejects; the second provides evidence in
support of that rejection.

E. The first is a position (WRONG) for which the ecologist argues; the second provides
evidence to support that position.

By Sandeep Gupta, GMAT Perfect 800 | www.top-one-percent.com 81


THE MOST AUTHORITATIVE GUIDE EVER WRITTEN ON GMAT CRITICAL REASONING

4.

My 30-second solution:

It is commonly thought (indicates that this will be challenged)

that this happens because (denotes explanation)

aging by itself brings about a loss of creative capacity.

However (shows opposition)

studies show that (this is a finding) a disproportionately large number of the scientists who
produce highly creative work beyond the age of forty entered their field at an older
age than is usual.

Remember: BF 1 and BF 2 are in opposition.

A. The first is the position (wrong – position is used for a conclusion, but the first boldface is an
explanation) that the argument as a whole opposes; the second is an objection (wrong –
objection means opinion, but the second boldface is a finding of a study – a fact) that has been
raised against a position defended – wrong – there is no one defending any position
(conclusion) in the argument.
B. The first is a claim (wrong – claim is used for a conclusion, but the first boldface is an
explanation) that has been advanced in support of a position that the argument opposes; the
second is a finding that has been used in support of that position (wrong – the second boldface
is a finding but is challenging the first Boldface, not supporting it).
C. The first is an explanation that the argument challenges; the second is a finding that has been
used in support of explanation presented in the first (wrong – as we saw, the first and the second
boldface statements are opposing each other; this option implies that the second boldface
supports the first boldface – terribly wrong – 180 degrees).
D. The first is an explanation that the argument challenges; the second is a finding on which that
challenge is based. Correct!
E. The first is an explanation that the argument defends (wrong – the author challenges the
explanation – notice the word however); the second is a finding that has been used to challenge
that explanation.

By Sandeep Gupta, GMAT Perfect 800 | www.top-one-percent.com 82


THE MOST AUTHORITATIVE GUIDE EVER WRITTEN ON GMAT CRITICAL REASONING

5.

My 30-second solution:

BF 1: Fact / situation: Several of a certain bank’s top executives have recently been purchasing
shares in their own bank.

BF 2: Final conclusion: On balance, therefore, it is likely that the executives of the bank are following
this example. The words “On balance, therefore” indicate the final conclusion. On balance
means when everything has been considered, when all factors have been accounted for.

In the entire argument, there is only one conclusion; there is no intermediate conclusion;
there is no opposition

A. The first describes the circumstance the explanation of which is the issue that the argument
addresses; the second states the main conclusion of the argument.

B. The first describes the circumstance the explanation of which is the issue the argument
addresses; the second states a conclusion that is drawn in order to support (wrong – there
is no intermediate conclusion) the main conclusion of the argument.

C. The first provides evidence to defend the position that the argument seeks to establish
against opposing positions (“opposing positions” indicates there are two conclusions in
opposition to each other, but as per our analysis, there is only one conclusion); the second
states the evidence (wrong – the second is a conclusion) that challenges the main
conclusion of the argument.

D. The first provides evidence to support the position that the argument seeks to establish; the
second states a conclusion that is drawn in order to support (wrong – there is no
intermediate conclusion) the argument’s main conclusion.

E. Each provides evidence to support (wrong – the second is a conclusion) the position that
the argument seeks to establish.

By Sandeep Gupta, GMAT Perfect 800 | www.top-one-percent.com 83


THE MOST AUTHORITATIVE GUIDE EVER WRITTEN ON GMAT CRITICAL REASONING

6.

My 30-second solution:

BF 1: Fact: each year it sponsors management education abroad for its management trainees –
this could be company policy or practice.

BF 2: Intermediate Conclusion:

but both this function and the goal of providing international experience could be achieved in
other ways: this is not a fact; this is an implied suggestion / opinion by the author; the author says
there is an alternative, more effective, way to achieve the same desired result

BUT denotes contradiction, means that the author is not entirely in support of the policy or the
practice mentioned in the first boldface.

Final conclusion: Therefore, if the attrition problem cannot be successfully addressed, Hachnut
should discontinue the sponsorship program.

A. The first describes a practice that the argument seeks to justify (WRONG – the argument says
that there is a better practice possible, so the argument is definitely not justifying the
practice – WRONG); the second states a judgment that is used in support of a justification
(the second boldface is saying that the practice is not needed as there is a better practice possible,
so absolutely no justification is being made for the current practice; rather the author says the
current practice can be totally abandoned – WRONG) for that practice.

B. The first describes a practice that the argument seeks to explain (WRONG – the argument says
that there is a better practice possible, so the argument is definitely not explaining the
practice – WRONG); the second presents part of the argument’s explanation of that practice
(nothing could be farther from this answer: the second boldface is saying that the
practice is not needed as there is a better practice possible, so absolutely no explanation is being
made for the current practice; rather the author says the current practice can be replaced by
another better idea as suggested by the author – WRONG).

C. The first introduces a practice that the argument seeks to evaluate; the second provides grounds
for holding that the practice can never achieve its objective (the author says that “both this
function and the goal of providing international experience could be achieved in other ways
– means the current practice can also achieve its objective” so saying that the practice can never
achieve its objective is absolutely wrong).

D. The first introduces a policy that the argument seeks to evaluate; the second provides grounds
for holding that the policy is not needed.

By Sandeep Gupta, GMAT Perfect 800 | www.top-one-percent.com 84


THE MOST AUTHORITATIVE GUIDE EVER WRITTEN ON GMAT CRITICAL REASONING

E. The first introduces a consideration (means a fact) supporting a policy (means a fact) that
the argument seeks to evaluate; the second provides evidence (wrong) for concluding that the
policy should be abandoned.

Further explanation for E: fact supporting a fact is terrible; we know that a fact can’t be
supported. Also, the second Boldface isn’t an Evidence (fact). It is a suggestion made by the
author, or an intermediate conclusion.

7. Many students find this argument really difficult.

My 30-second solution (you may not get this in 30 seconds initially, no matter how hard you try,
but you will definitely get there, soon)

BF 1: FACT: Plant scientists have used (denotes fact) genetic engineering on seeds to produce crop
plants that are highly resistant to insect damage. Unfortunately (contradiction), the seeds
themselves are quite expensive, and the plants require more fertilizer and water to grow well than
normal ones.

BF 2: the reason why the first boldface might not be as easily realized: for most farmers the
savings on pesticides cannot compensate for the higher seed costs and the cost of
additional fertilizer.

Contradiction: However, (fact) since (since introduces a fact) consumer demand for grains,
fruits, and vegetables grown without the use of pesticides continues to rise

Final conclusion: the use of genetically engineered seeds of this kind is likely to become
widespread (prediction).

A. The first supplies a context for the argument; the second is the argument's main conclusion
(WRONG).
B. The first introduces a development that the argument predicts will have a certain outcome; the
second is a state of affairs (NO – the second is a prediction) that, according to the
argument, contributes to bringing about that outcome.
C. Correct: The first presents a development that the argument predicts will have a certain
outcome; the second acknowledges a consideration that tends to weigh against that prediction.
D. The first provides evidence to support a prediction that the argument seeks to defend; the second
is that prediction (wrong).
E. The first and the second each provide evidence to support the argument's main conclusion
(wrong – the second boldface isn’t supporting the main conclusion; rather it describes an
obstacle why the main conclusion might not be temporarily realized).

By Sandeep Gupta, GMAT Perfect 800 | www.top-one-percent.com 85


THE MOST AUTHORITATIVE GUIDE EVER WRITTEN ON GMAT CRITICAL REASONING

(Background information) Plant scientists have used genetic engineering on seeds to produce crop
plants that are highly resistant to insect damage. (Counter premise) Unfortunately, the seeds
themselves are quite expensive, and the plants require more fertilizer and water to grow well than
normal ones. (Counter-conclusion) Thus, for most farmers the savings on pesticides would not
compensate for the higher seed costs and the cost of additional fertilizer. (Premise - "However,"
changes direction of argument, everything prior to this statement is now the Counter
argument) However, since consumer demand for grains, fruits, and vegetables grown without the use
of pesticides continues to rise, (Main Conclusion) the use of genetically engineered seeds of this kind
is likely to become widespread

Conclusion: The use of genetically engineered seeds of this kind is likely to become widespread.
Evidence: Consumer demand for grains, fruits, and vegetables grown without the use of pesticides
continues to rise.

Define the role of each statement, and the relationship between the two. Statement 1 tells us about
an advance in seed tech to resist insect damage. Statement 2 reveals that most farmers won’t be
financially advantaged by the technology.

Choice C is correct. The development introduced by the statement is the use of genetic engineering. The
prediction is the conclusion: this use of genetically engineered seeds will become widespread. The
second statement is that the farmers would not be compensated enough to use the genetically
engineered seeds, which is a consideration that weighs AGAINST the prediction that the seeds will
become widespread.

A. The first statement does provide context, but the second is not the conclusion, as shown in the
analysis above.
B. The description of the first statement matches the correct choice well, but the second statement goes
AGAINST the conclusion, as described in the explanation to the correct choice.
D. The first statement does not directly support a prediction. It is just the background information.
E. The statements do not support the conclusion that the use of genetically engineered seeds is likely to
become widespread. This is particularly true for the second statement, which supports the idea that the
seeds might NOT become widespread.

By Sandeep Gupta, GMAT Perfect 800 | www.top-one-percent.com 86


THE MOST AUTHORITATIVE GUIDE EVER WRITTEN ON GMAT CRITICAL REASONING

8.

My 10-second solution:

The first thing to notice in this argument is that there is no contradiction word, and hence no
opposition.

BF 1: looks like the main conclusion: It must have been a major Anasazi center.

BF 2: Supports BF 1: Only a major cultural center would have the organizational power to import
timber from 50 miles away.

If you aren’t able to see this instantly, use the therefore test.

BECAUSE It must have been a major Anasazi center. SO Only a major cultural center would have
the organizational power to import timber from 50 miles away.

Makes no sense!

BECAUSE Analysis of wood samples shows that some of the timber for the buildings came from the
Chuska and San Mateo mountains, 50 miles from Chaco Canyon. AND BECAUSE Only a major
cultural center would have the organizational power to import timber from 50 miles away. SO, it
must have been a major Anasazi center.

Makes complete sense.

So, the first boldface is the conclusion of the argument.

A. The first is a premise (wrong) used to support the argument's main conclusion; the second is
the argument's main conclusion (wrong).
B. The first is the argument's main conclusion; the second is a premise used to support that
conclusion. Correct
C. The first is one of two premises (wrong) used to support the argument's main conclusion; the
second is the other of those two premises.
D. The first is a premise (wrong) used to support the argument's main conclusion; the second is a
premise used to support another conclusion drawn in the argument.
E. The first is inferred (wrong) from another statement in the argument; the second is inferred
from the first (wrong). We can’t anything from the final conclusion of an argument; rather,
the final conclusion is inferred from all the other statements.

By Sandeep Gupta, GMAT Perfect 800 | www.top-one-percent.com 87


THE MOST AUTHORITATIVE GUIDE EVER WRITTEN ON GMAT CRITICAL REASONING

9.

My 10-second solution:

BF 1: conclusion of the environmentalists

BF 2: conclusion of the economist

The word “However” signifies opposition.

A. The first supports (wrong) the conclusion of the economist's argument; the second calls that
conclusion into question.
B. The first states the conclusion of the economist's argument (wrong); the second supports that
conclusion.
C. The first supports (wrong) the conclusion of the environmentalists' argument; the second
states that conclusion.
D. The first states the conclusion of the environmentalists' argument; the second states the
conclusion of the economist's argument.
E. Each supports (wrong) the conclusion of the economist's argument.

10.

My 10-second solution:

Economist: The price of tap water in our region should be raised drastically (conclusion).

BF 1: Supplies in local freshwater reservoirs have been declining (FACT) for years because water
is being used faster than it can be replenished.

Since the price of tap water has been (FACT) low

BF 2: few users have bothered (FACT) to adopt even easy conservation measures.

So, there is only one conclusion. No opposition!

A. The first is a conclusion for which support is provided, and which in turn supports the main
conclusion; the second is the main conclusion (wrong).
B. The first is an observation (wrong) for which the second provides an explanation; the second is
the main conclusion but not the only conclusion (wrong).
C. The first is a premise supporting the argument's main conclusion; so is the second. Correct
D. The first is the only conclusion (wrong; the first is not a conclusion); the second provides an
explanation for the first.
E. The first is the main conclusion (wrong); the second is a conclusion (wrong) for which support
is provided, and which in turn supports the first.

By Sandeep Gupta, GMAT Perfect 800 | www.top-one-percent.com 88


THE MOST AUTHORITATIVE GUIDE EVER WRITTEN ON GMAT CRITICAL REASONING

Mega Exercise – Boldface questions:

Another variation of an already discussed question:


11. Delta Products Inc. has recently switched at least partly from older technologies
using fossil fuels to new technologies powered by electricity. The question has been
raised whether it can be concluded that for a given level of output Delta's operation now
causes less fossil fuel to be consumed than it did formerly. The answer, clearly, is yes,
since the amount of fossil fuel used to generate the electricity needed to power the new
technologies is less than the amount needed to power the older technologies, provided level of
output is held constant.
In the argument given, the two boldfaced portions play which of the following roles?
A. The first identifies the content of the conclusion of the argument; the second provides support
for that conclusion.
B. The first provides support for the conclusion of the argument; the second identifies the
content of that conclusion.
C. The first states the conclusion of the argument; the second calls that conclusion into question.
D. The first provides support for the conclusion of the argument; the second calls that conclusion
into question.
E. Each provides support for the conclusion of the argument.

By Sandeep Gupta, GMAT Perfect 800 | www.top-one-percent.com 89


THE MOST AUTHORITATIVE GUIDE EVER WRITTEN ON GMAT CRITICAL REASONING

Another variation of an already discussed question:


12. Scientists typically do their most creative work before the age of forty. It is commonly thought that
this happens because aging by itself brings about a loss of creative capacity. However, a
study has found that almost all scientists who produce highly creative work beyond the age of
forty entered their fields late and less than a dozen years before their creative breakthroughs.
Since creative breakthroughs by scientists under forty also generally occur within a
dozen years of the scientist’s entry into the field, the study’s finding strongly suggests
that the real reason why scientists over forty rarely produce highly creative work is not due to
age but rather because most have spent too long in their fields.
In the argument given, the two portions in boldface play which of the following roles?
A. The first is the position that the argument as a whole defends; the second is evidence that is
advanced as part of that defense.
B. The first and second are both claims that have been advanced in support of a position that the
argument as a whole opposes.
C. The first is an explanation that the argument challenges; the second provides evidence in support
of a competing explanation that the argument defends.
D. The first is an explanation that the argument challenges; the second is evidence that has been
used against an alternative explanation that the argument defends.
E. The first is an explanation that the argument defends; the second is evidence that has been used
to challenge that explanation.

By Sandeep Gupta, GMAT Perfect 800 | www.top-one-percent.com 90


THE MOST AUTHORITATIVE GUIDE EVER WRITTEN ON GMAT CRITICAL REASONING

Another variation of an already discussed question:


13. Scientists typically do their most creative work before the age of forty. It is commonly thought that
this happens because aging by itself brings about a loss of creative capacity. Studies show that of
scientists who produce highly creative work beyond the age of forty, a
disproportionately large number entered their field at an older age than is usual.
Since by the age of forty the large majority of scientists have been working in their field for at
least fifteen years, the studies' finding strongly suggests that the real reason why scientists over
forty rarely produce highly creative work is not that they have aged but rather that scientists
over forty have generally spent too long in their field.
In the argument given, the two portions in boldface play which of the following roles?
A. The first is a claim, the accuracy of which is at issue in the argument; the second is a
conclusion drawn on the basis of that claim.
B. The first is an objection that has been raised against a position defended in the argument; the
second is that position.
C. The first is evidence that has been used to support an explanation that the argument
challenges; the second is that explanation.
D. The first is evidence that has been used to support an explanation that the argument
challenges; the second is a competing explanation that the argument favors.
E. The first provides evidence to support an explanation that the argument favors; the second is
that explanation.

By Sandeep Gupta, GMAT Perfect 800 | www.top-one-percent.com 91


THE MOST AUTHORITATIVE GUIDE EVER WRITTEN ON GMAT CRITICAL REASONING

Another variation of an already discussed question:

14. Musical composers have generally made their most significant contributions to the musical
canon before they reached the age of thirty. It is commonly believed that this is the case because
aging brings about a loss of cognitive ability and creative capacity. However, a study
pointed out that a disproportionately large number of those composers who made
their most significant contributions to the canon after the age of thirty became
musicians at an older age than is generally the case. Since by the age of thirty many
composers have been engaged as musicians for a decade or more, these findings suggest that the
real reason why musicians over thirty rarely make significant contributions to the musical canon
is not that they have aged but rather that they have spent too much time as musicians.

In the argument above, the two portions in boldface play which of the following roles?

A. The first is an explanation that is challenged by the argument; the second is a finding used to
support that explanation.
B. The first is an explanation that the argument opposes; the second is an objection raised
against the alternative explanation advocated by the argument.
C. The first is a claim advanced in support of a position; the second is a finding that clarifies that
position.
D. The first is an explanation advocated by the argument; the second is a finding used to
challenge that explanation.
E. The first is an explanation that the argument challenges; the second is a finding on which that
challenge is based.

By Sandeep Gupta, GMAT Perfect 800 | www.top-one-percent.com 92


THE MOST AUTHORITATIVE GUIDE EVER WRITTEN ON GMAT CRITICAL REASONING

Another variation of an already discussed question:

15. Since it has become known that several of a bank's top executives have been buying
shares in their own bank, the bank's depositors, who had been worried by rumors that the
bank faced impending financial collapse, have been greatly relieved. They reason that, since top
executives evidently have faith in the bank's financial soundness, those worrisome rumors must
be false. Such reasoning might well be overoptimistic, however, since corporate executives
have been known to buy shares in their own company in a calculated attempt to
dispel negative rumors about the company's health.
In the argument given, the two boldfaced portions play which of the following roles?
A. The first describes evidence that has been taken as supporting a conclusion; the second gives
a reason for questioning that support.
B. The first describes evidence that has been taken as supporting a conclusion; the second states
a contrary conclusion that is the main conclusion of the argument.
C. The first provides evidence in support of the main conclusion of the argument; the second
states that conclusion.
D. The first describes the circumstance that the argument as a whole seeks to explain; the second
gives the explanation that the argument seeks to establish.
E. The first describes the circumstance that the argument as a whole seeks to explain; the second
provides evidence in support of the explanation that the argument seeks to establish.

By Sandeep Gupta, GMAT Perfect 800 | www.top-one-percent.com 93


THE MOST AUTHORITATIVE GUIDE EVER WRITTEN ON GMAT CRITICAL REASONING

Another variation of an already discussed question:

16. Since it has become known that several of a bank’s top executives have been buying shares in
their own bank, the bank’s depositors, who had been worried by rumors that the bank faced
impending financial collapse, have been greatly relieved. They reason that since top executives
evidently have faith in the bank’s financial soundness, those worrisome rumors must be
false. They might well be overoptimistic, however since corporate executives have
sometimes bought shares in their own company in a calculated attempt to dispel negative rumors
about the company’s health.
In the argument given, the two boldfaced portions play which of the following roles?
A. The first summarizes the evidence used in the reasoning called into question by the argument;
the second states the counterevidence on which the argument relies.
B. The first summarizes the evidence used in the reasoning called into question by the argument;
the second is an intermediate conclusion supported by the evidence.
C. The first is an intermediate conclusion that forms part of the reasoning called into question
by the argument; the second is evidence that undermines the support for this intermediate
conclusion.
D. The first is an intermediate conclusion that forms part of the reasoning called into question
by the argument; the second is the main conclusion of the argument.
E. The first is an intermediate conclusion that forms part of the reasoning called into question
by the argument; the second states a further conclusion supported by this intermediate
conclusion.

By Sandeep Gupta, GMAT Perfect 800 | www.top-one-percent.com 94


THE MOST AUTHORITATIVE GUIDE EVER WRITTEN ON GMAT CRITICAL REASONING

Another variation of an already discussed question:

17. Plant scientists have used genetic engineering on seeds to produce crop plants that are highly
resistant to insect damage. Unfortunately, the seeds themselves are quite expensive,
and the plants require more fertilizer and water to grow well than normal ones.
Thus, for most farmers the savings on pesticides would not compensate for the higher seed costs
and the cost of additional fertilizer. However, since consumer demand for grains, fruits,
and vegetables grown without the use of pesticides continues to rise, the use of
genetically engineered seeds of this kind is likely to become widespread.
In the argument given, the two portions in boldface play which of the following roles?
A. The first supplies a context for the argument; the second is the argument's main conclusion.
B. The first introduces a development that the argument predicts will have a certain outcome; the
second is a state of affairs that the argument denies will be part of that outcome.
C. The first presents a development that the argument predicts will have a certain outcome; the
second acknowledges a consideration that weighs against that prediction.
D. The first provides evidence to support a prediction that the argument seeks to defend; the second
is that prediction.
E. The first and the second each provide evidence to support the argument's main conclusion.

By Sandeep Gupta, GMAT Perfect 800 | www.top-one-percent.com 95


THE MOST AUTHORITATIVE GUIDE EVER WRITTEN ON GMAT CRITICAL REASONING

Another variation of an already discussed question:

18. Economist: Tropicorp, which constantly seeks profitable investment opportunities, has been
buying and clearing sections of tropical forest for cattle ranching, although pastures newly
created there become useless for grazing after just a few years. The company has not gone into
rubber tapping, even though greater profits can be made from rubber tapping, which
leaves the forest intact. Thus, some environmentalists conclude that Tropicorp has not acted
wholly out of economic self-interest. However, these environmentalists are probably
wrong. The initial investment required for a successful rubber-tapping operation is larger than
that needed for a cattle ranch. Furthermore, there is a shortage of workers employable in rubber-
tapping operations, and finally, taxes are higher on profits from rubber tapping than on profits
from cattle ranching.

In the economist’s argument, the two BOLDFACED portions play which of the following roles?
A. The first supports the conclusion of the economist’s argument; the second calls that
conclusion into question.

B. The first states the conclusion of the economist’s argument; the second supports that
conclusion.

C. The first supports the environmentalists’ conclusion; the second states that conclusion.

D. The first states the environmentalists’ conclusion; the second states the conclusion of the
economist’s argument.

E. Each supports the conclusion of the economist’s argument.

By Sandeep Gupta, GMAT Perfect 800 | www.top-one-percent.com 96


THE MOST AUTHORITATIVE GUIDE EVER WRITTEN ON GMAT CRITICAL REASONING

19. Boreal owls range over a much larger area than do other owls of similar size. Scientists have
hypothesized that it is scarcity of prey that leads the owls to range so widely. This
hypothesis would be hard to confirm directly, since it is not possible to produce a sufficiently
accurate count of the populations of small mammals inhabiting the forests where boreal owls
live. Careful study of owl behavior has shown that boreal owls do range over larger areas
when they live in regions where food of the sort eaten by small mammals is
comparatively sparse. This indicates that the scientists' hypothesis is not sheer speculation.
In the argument given, the two boldfaced portions play which of the following roles?
A. The first presents an explanatory hypothesis; the second states the main conclusion of the
argument.
B. The first presents an explanatory hypothesis; the second presents evidence tending to support
this hypothesis.
C. The first presents an explanatory hypothesis; the second presents evidence to support an
alternative explanation.
D. The first describes a position that the argument opposes; the second presents evidence to
undermine the support for the position being opposed.
E. The first describes a position that the argument opposes; the second states the main conclusion
of the argument.

By Sandeep Gupta, GMAT Perfect 800 | www.top-one-percent.com 97


THE MOST AUTHORITATIVE GUIDE EVER WRITTEN ON GMAT CRITICAL REASONING

20. A prominent investor who holds a large stake in the Burton Tool Company has recently claimed
that the company is mismanaged, citing as evidence the company's failure to slow down
production in response to a recent rise in its inventory of finished products. It is doubtful whether
an investor's sniping at management can ever be anything other than counterproductive, but in
this case, it is clearly not justified. It is true that an increased inventory of finished
products often indicates that production is outstripping demand, but in Burton's case
it indicates no such thing. Rather, the increase in inventory is entirely attributable to
products that have already been assigned to orders received from customers.
In the argument given, the two boldfaced portions play which of the following roles?
A. The first states a generalization that underlies the position that the argument as a whole opposes;
the second provides evidence to show that the generalization does not apply in the case at issue.
B. The first states a generalization that underlies the position that the argument as a whole opposes;
the second clarifies the meaning of a specific phrase as it is used in that generalization.
C. The first provides evidence to support the conclusion of the argument as a whole; the second is
evidence that has been used to support the position that the argument as a whole opposes.
D. The first provides evidence to support the conclusion of the argument as a whole; the second
states that conclusion.
E. The first and the second each provide evidence against the position that the argument as a whole
opposes.

By Sandeep Gupta, GMAT Perfect 800 | www.top-one-percent.com 98


THE MOST AUTHORITATIVE GUIDE EVER WRITTEN ON GMAT CRITICAL REASONING

Another variation of the question above:

20. A

A prominent investor who holds a large stake in the Burton Tool company has recently claimed that
the company is mismanaged, citing as evidence the company's failure to slow production in
response to a recent rise in its inventory of finished products. It is doubtful whether an investor's
sniping at management can ever be anything other than counterproductive, but, in this case, it is
clearly not justified. It is true that an increased inventory of finished products often indicates
that production is outstripping demand, but in Burton's case it indicates no such thing. Rather, the
increase in inventory is entirely attributable to products that have already been assigned to orders
received from customers.

In the argument given, the two boldfaced portions play which of the following roles?

A. The first states the position that the argument as a whole opposes; the second provides evidence
to undermine the support for the position being opposed.
B. The first states the position that the argument as a whole opposes; the second is evidence that
has been used to support the position being opposed.
C. The first states the position that the argument as a whole opposes; the second states the
conclusion of the argument as a whole.
D. The first is evidence that has been used to support a position that the argument as a whole
opposes; the second provides information to undermine the force of that evidence.
E. The first is evidence that has been used to support a position that the argument as a whole
opposes; the second states the conclusion of the argument as a whole.

By Sandeep Gupta, GMAT Perfect 800 | www.top-one-percent.com 99


THE MOST AUTHORITATIVE GUIDE EVER WRITTEN ON GMAT CRITICAL REASONING

Another variation of the question above:

21. A prominent investor who holds a large stake in the Burton Tool Company has recently claimed
that the company is mismanaged. As evidence for this claim, the investor cited the company's
failure to slow production in response to a recent rise in its inventory of finished products. It is
doubtful whether an investor's sniping at management can ever be anything other than
counterproductive, but in this case, it is clearly not justified. It is true that an increased
inventory of finished products often indicates that production is outstripping demand. In
Burton's case it indicates no such thing, however: the increase in inventory is entirely
attributable to products that have already been assigned to orders received from customers.
In the argument given, the two boldfaced portions play which of the following roles?
A. The first provides evidence to support the conclusion of the argument as a whole; the second
states that conclusion.
B. The first states the conclusion of the argument as a whole; the second states an intermediate
conclusion that is drawn in order to support that conclusion.
C. The first is the position that the argument as a whole opposes; the second provides evidence
against the position being opposed.
D. The first states an intermediate conclusion that is drawn in order to support the conclusion of
the argument as a whole; the second states the conclusion of the argument as a whole.
E. The first and the second both state intermediate conclusions that are drawn in order to support
jointly the conclusion of the argument as a whole.

By Sandeep Gupta, GMAT Perfect 800 | www.top-one-percent.com 100


THE MOST AUTHORITATIVE GUIDE EVER WRITTEN ON GMAT CRITICAL REASONING

22. Last year a record number of new manufacturing jobs were created. Will this year bring another
record? Well, any new manufacturing job is created either within an existing company or by the
start-up of a new company. Within existing firms, new jobs have been created this year
at well below last year's record pace. At the same time, there is considerable evidence that
the number of new companies starting up will be no higher this year than it was last year and
there is no reason to think that the new companies starting up this year will create
more jobs per company than did last year's start-ups. So clearly, the number of new jobs
created this year will fall short of last year's record.
In the argument given, the two portions in boldface play which of the following roles?
A. The first is a claim that the argument challenges; the second is an explicit assumption on which
that challenge is based.
B. The first is a claim that the argument challenges; the second is a judgment advanced in support
of the main conclusion of the argument.
C. The first provides evidence in support of the main conclusion of the argument; the second is an
objection that has been raised against that main conclusion.
D. The first provides evidence in support of the main conclusion of the argument; the second is a
judgment advanced in support of that main conclusion.
E. The first and the second are each claims that have been advanced in support of a position that
the argument opposes.

By Sandeep Gupta, GMAT Perfect 800 | www.top-one-percent.com 101


THE MOST AUTHORITATIVE GUIDE EVER WRITTEN ON GMAT CRITICAL REASONING

Another variation of the question above:

23. Last year a record number of new manufacturing jobs were created. Will this year bring another
record? Well, any new manufacturing job is created either within an existing company or by the
start-up of a new company. Within existing firms, new jobs have been created this year
at well below last year's record pace. At the same time, there is considerable evidence that
the number of new companies starting up this year will be no higher than it was last year and
there is no reason to think that the new companies starting up this year will create
more jobs per company than did last year's start-ups. So clearly, the number of new jobs
created this year will fall short of last year's record.
In the argument given, the two portions in boldface play which of the following roles?
A. The first provides evidence in support of the main conclusion of the argument; the second is a
claim that the argument challenges.
B. The first is a generalization that the argument seeks to establish; the second is a conclusion that
the argument draws in order to support that generalization.
C. The first is a generalization that the argument seeks to establish; the second is a judgment that
has been advanced in order to challenge that generalization.
D. The first is presented as an obvious truth on which the argument is based; the second is a claim
that has been advanced in support of a position that the argument opposes.
E. The first is presented as an obvious truth on which the argument is based; the second is a
judgment advanced in support of the main conclusion of the argument.

By Sandeep Gupta, GMAT Perfect 800 | www.top-one-percent.com 102


THE MOST AUTHORITATIVE GUIDE EVER WRITTEN ON GMAT CRITICAL REASONING

Another variation of the question above:

24. Last year a record number of new manufacturing jobs were created. Will this year bring another
record? Well, any new manufacturing job is created either within an existing
company or by the start-up of a new company. Within existing firms, new jobs have been
created this year at well below last year's record pace. At the same time, there is considerable
evidence that the number of new companies starting up will be no higher this year than it was
last year and there is no reason to think that the new companies starting up this year will create
more jobs per company than did last year's start-ups. So clearly, the number of new jobs
created this year will fall short of last year's record.
In the argument given, the two portions in boldface play which of the following roles?
A. The first is presented as an obvious truth on which the argument is based; the second is the
main conclusion of the argument.
B. The first is presented as an obvious truth on which the argument is based; the second is a
conclusion drawn in order to support the main conclusion of the argument.
C. The first and the second each provide evidence in support of the main conclusion of the
argument.
D. The first is a generalization that the argument seeks to establish; the second is the main
conclusion of the argument.
E. The first is a generalization that the argument seeks to establish; the second is a conclusion
that has been drawn in order to challenge that generalization.

By Sandeep Gupta, GMAT Perfect 800 | www.top-one-percent.com 103


THE MOST AUTHORITATIVE GUIDE EVER WRITTEN ON GMAT CRITICAL REASONING

Another variation of the question above:

25. Last year a record number of new manufacturing jobs were created. Will this year bring another
record? Well, any new manufacturing job is created either within an existing
company or by the start-up of a new company. Within existing firms, new jobs have been
created this year at well below last year’s record pace. At the same time, there is considerable
evidence that the number of new companies starting up this year will be no higher than it was
last year and there is no reason to think that the new companies starting up this year
will create more jobs per company than did last year’s start-ups. So clearly, the
number of new jobs created this year will fall short of last year’s record.

In the argument given, the two portions in boldface play which of the following roles?

A. The first provides evidence in support of the main conclusion of the argument; the second is
a claim that argument challenges.
B. The first is a generalization that the argument seeks to establish; the second is a conclusion
that the argument draws in order to support that generalization.
C. The first is a generalization that the argument seeks to establish; the second is a judgment
that has been advanced in order to challenge that generalization.
D. The first is presented as obvious truth on which the argument is based; the second is a claim
that has been advanced in support of a position that the argument opposes.
E. The first is presented as obvious truth on which the argument is based; the second is a
judgment advanced in support of the main conclusion of the argument.

By Sandeep Gupta, GMAT Perfect 800 | www.top-one-percent.com 104


THE MOST AUTHORITATIVE GUIDE EVER WRITTEN ON GMAT CRITICAL REASONING

Another variation of the question above:

26. Last year a record number of new manufacturing jobs were created. Will this year bring another
record? Well, any new manufacturing job is created either within an existing
company or by the start-up of a new company. Within existing firms, new jobs have been
created this year at well below last year’s record pace. At the same time, there is
considerable evidence that the number of new companies starting up will be no
higher this year than it was last year and surely the new companies starting up this
year will create no more jobs per company than did last year’s start-ups. So clearly,
the number of new jobs created this year will fall short of last year’s record.
In the argument given, the two portions in boldface play which of the following roles?
A. The first is presented as an obvious truth on which the argument is based; the second is a
prediction advanced in support of the main conclusion of the argument.
B. The first is presented as an obvious truth on which the argument is based; the second is an
objection that the argument rejects.
C. The first is presented as an obvious truth on which the argument is based; the second is the main
conclusion of the argument.
D. The first is a generalization that the argument seeks to establish; the second is a claim that has
been advanced in support of a position that the argument opposes.
E. The first is a generalization that the argument seeks to establish; the second is a claim that has
been advanced in order to challenge that generalization.

By Sandeep Gupta, GMAT Perfect 800 | www.top-one-percent.com 105


THE MOST AUTHORITATIVE GUIDE EVER WRITTEN ON GMAT CRITICAL REASONING

27. Consultant: Ace Repairs ends up having to redo a significant number of the complex
repair jobs it undertakes, but when those repairs are redone, they are invariably
done right. Since we have established that there is no systematic difference between the
mechanics who are assigned to do the initial repairs and those who are assigned to redo
unsatisfactory jobs, we must reject the hypothesis that mistakes made in the initial repairs are
due to the mechanics' lack of competence. Rather, it is likely that complex repairs require a
level of focused attention that the company's mechanics apply consistently only to
repair jobs that have not been done right on the first try.
In the consultant's reasoning, the two portions in boldface play which of the following roles?
A. The first is the consultant's main conclusion; the second provides evidence in support of that
main conclusion.
B. The first is evidence that serves as the basis for rejecting one explanation of a certain finding;
the second is the consultant's own explanation of that finding.
C. The first is a claim whose truth is at issue in the reasoning; the second provides evidence to
show that the claim is true.
D. The first presents a contrast whose explanation is at issue in the reasoning; the second is the
consultant's explanation of that contrast.
E. The first presents a contrast whose explanation is at issue in the reasoning; the second is
evidence that has been used to challenge the consultant's explanation of that contrast.

By Sandeep Gupta, GMAT Perfect 800 | www.top-one-percent.com 106


THE MOST AUTHORITATIVE GUIDE EVER WRITTEN ON GMAT CRITICAL REASONING

Another variation of the question above:

28. Consultant: A significant number of complex repair jobs carried out by Ace Repairs
have to be redone under the company's warranty, but when those repairs are
redone, they are invariably successful. Since we have definitely established that there is
no systematic difference between the mechanics who are assigned to do the initial
repairs and those who are assigned to redo unsatisfactory jobs, it is clear that
inadequacies in the initial repairs cannot be attributed to the mechanics' lack of competence.
Rather, it is likely that complex repairs require a level of focused attention that the company's
mechanics apply consistently only to repair jobs that have been inadequately done on the first
try.
In the consultant's reasoning, the two portions in boldface play which of the following roles?
A. The first is a claim that the consultant rejects as false; the second is evidence that forms the
basis for that rejection.
B. The first is part of an explanation that the consultant offers for a certain finding; the second
is that finding.
C. The first presents a pattern whose explanation is at issue in the reasoning; the second
provides evidence to rule out one possible explanation of that pattern.
D. The first presents a pattern whose explanation is at issue in the reasoning; the second is
evidence that has been used to challenge the explanation presented by the consultant.
E. The first is the position the consultant seeks to establish; the second is offered as evidence for
that position.

By Sandeep Gupta, GMAT Perfect 800 | www.top-one-percent.com 107


THE MOST AUTHORITATIVE GUIDE EVER WRITTEN ON GMAT CRITICAL REASONING

29. Public health expert: Increasing the urgency of a public health message may be
counterproductive. In addition to irritating the majority who already behave responsibly, it
may undermine all government pronouncements on health by convincing people
that such messages are overly cautious. And there is no reason to believe that those who
ignore measured voices will listen to shouting.
The two sections in boldface play which of the following roles in the public health expert's
argument?
A. The first is a conclusion for which support is provided, but is not the argument's main
conclusion; the second is an unsupported premise supporting the argument's main
conclusion.
B. The first is a premise supporting the only explicit conclusion; so is the second.
C. The first is the argument's main conclusion; the second supports that conclusion and is itself
a conclusion for which support is provided.
D. The first is a premise supporting the argument's only conclusion; the second is that
conclusion.
E. The first is the argument's only explicit conclusion; the second is a premise supporting that
conclusion.

By Sandeep Gupta, GMAT Perfect 800 | www.top-one-percent.com 108


THE MOST AUTHORITATIVE GUIDE EVER WRITTEN ON GMAT CRITICAL REASONING

30. In countries where automobile insurance includes compensation for whiplash


injuries sustained in automobile accidents, reports of having suffered such injuries
are twice as frequent as they are in countries where whiplash is not covered.
Presently, no objective test for whiplash exists, so it is true that spurious reports of whiplash
injuries cannot be readily identified. Nevertheless, these facts do not warrant the conclusion
drawn by some commentators that in the countries with the higher rates of reported whiplash
injuries, half of the reported cases are spurious. Clearly, in countries where automobile
insurance does not include compensation for whiplash, people often have little
incentive to report whiplash injuries that they actually have suffered.
In the argument given, the two boldfaced portions play which of the following roles?
A. The first is a claim that the argument disputes; the second is a conclusion that has been based
on that claim.
B. The first is a claim that has been used to support a conclusion that the argument accepts; the
second is that conclusion.
C. The first is evidence that has been used to support a conclusion for which the argument
provides further evidence; the second is the main conclusion of the argument.
D. The first is a finding whose implications are at issue in the argument; the second is a claim
presented in order to argue against deriving certain implications from that finding.
E. The first is a finding whose accuracy is evaluated in the argument; the second is evidence
presented to establish that the finding is accurate.

By Sandeep Gupta, GMAT Perfect 800 | www.top-one-percent.com 109


THE MOST AUTHORITATIVE GUIDE EVER WRITTEN ON GMAT CRITICAL REASONING

Another variation of the question above:

31. In countries where automobile insurance includes compensation for whiplash


injuries sustained in automobile accidents, reports of having suffered such injuries
are twice as frequent as they are in countries where whiplash is not covered. Some
commentators have argued, correctly, that since there is presently no objective test for whiplash,
spurious reports of whiplash injuries cannot be readily identified. These
commentators are, however, wrong to draw the further conclusion that in the countries with the
higher rates of reported whiplash injuries, half of the reported cases are spurious: clearly, in
countries where automobile insurance does not include compensation for whiplash, people often
have little incentive to report whiplash injuries that they actually have suffered.
In the argument given, the two boldfaced portions play which of the following roles?

A. The first is a finding whose accuracy is evaluated in the argument; the second is an
intermediate conclusion drawn to support the judgment reached by the argument on the
accuracy of that finding.
B. The first is a finding whose accuracy is evaluated in the argument; the second is evidence that
has been used to challenge the accuracy of that finding.
C. The first is a finding whose implications are at issue in the argument; the second is an
intermediate conclusion that has been used to support a conclusion that the argument
criticizes.
D. The first is a claim that the argument disputes; the second is a narrower claim that the
argument accepts.
E. The first is a claim that has been used to support a conclusion that the argument accepts; the
second is that conclusion.

By Sandeep Gupta, GMAT Perfect 800 | www.top-one-percent.com 110


THE MOST AUTHORITATIVE GUIDE EVER WRITTEN ON GMAT CRITICAL REASONING

Another variation of the question above:

32. In countries where automobile insurance includes compensation for whiplash


injuries sustained in automobile accidents, reports of having suffered such injuries
are twice as frequent as they are in countries where whiplash is not covered.
Presently, no objective test for whiplash exists, so it is true that spurious reports of whiplash
injuries cannot be readily identified. Nevertheless, these facts do not warrant a conclusion
that has been drawn by some commentators: that in the countries with the higher rates
of reported whiplash injuries, half of the reported cases are spurious. Clearly, in countries where
automobile insurance does not include compensation for whiplash, people often have little
incentive to report whiplash injuries that they actually have suffered.
In the argument given, the two boldfaced portions play which of the following roles?
A. The first is a claim that the argument disputes; the second is a conclusion that has been based
on that claim.
B. The first is claim that has been used to support a position that the argument accepts; the
second is a position that the argument rejects.
C. The first is a finding whose accuracy is evaluated in the argument; the second is the judgment
reached by the argument concerning the accuracy of the finding.
D. The first is a finding whose implications are at issue in the argument; the second is the
judgment reached by the argument concerning one alleged implication.
E. The first is a finding, the explanation of which is at issue in the argument; the second is an
objection that has been raised against the explanation that the argument defends.

By Sandeep Gupta, GMAT Perfect 800 | www.top-one-percent.com 111


THE MOST AUTHORITATIVE GUIDE EVER WRITTEN ON GMAT CRITICAL REASONING

33. Country X's recent stock-trading scandal should not diminish investors' confidence in the
country's stock market. For one thing, the discovery of the scandal confirms that Country
X has a strong regulatory system, as the following considerations show. In any stock
market, some fraudulent activity is inevitable. If a stock market is well regulated, any significant
stock-trading fraud in it will very likely be discovered. This deters potential perpetrators and
facilitates improvement in regulatory processes.
In the argument, the portion in boldface plays which of the following roles?
A. It is the argument's only conclusion.
B. It is a conclusion for which the argument provides support and which itself is used to support
the argument's main conclusion.
C. It is the argument's main conclusion and is supported by another explicitly stated conclusion
for which further support is provided.
D. It is an assumption for which no explicit support is provided and is used to support the
argument's only conclusion.
E. It is a compound statement containing both the argument's main conclusion and an
assumption used to support that conclusion.

By Sandeep Gupta, GMAT Perfect 800 | www.top-one-percent.com 112


THE MOST AUTHORITATIVE GUIDE EVER WRITTEN ON GMAT CRITICAL REASONING

34. A product that represents a clear technological advance over competing products can generally
command a high price. Because technological advances tend to be quickly surpassed and
companies want to make large profits while they still can, many companies charge the greatest
price the market will bear when they have such a product. But large profits on the new
product will give competitors a strong incentive to quickly match the new product's
capabilities. Consequently, the strategy to maximize overall profit from a new product is to
charge less than the greatest possible price.
In the argument above, the two portions in boldface play which of the following roles?
A. The first is an assumption that forms the basis for a course of action that the argument
criticizes; the second presents the course of action endorsed by the argument.
B. The first is a consideration raised to explain the appeal of a certain strategy; the second is a
consideration raised to call into question the wisdom of adopting that strategy.
C. The first is an assumption that has been used to justify a certain strategy; the second is a
consideration that is used to cast doubt on that assumption.
D. The first is a consideration raised in support of a strategy the argument endorses; the second
presents grounds in support of that consideration.
E. The first is a consideration raised to show that adopting a certain strategy is unlikely to
achieve the intended effect; the second is presented to explain the appeal of that strategy.

By Sandeep Gupta, GMAT Perfect 800 | www.top-one-percent.com 113


THE MOST AUTHORITATIVE GUIDE EVER WRITTEN ON GMAT CRITICAL REASONING

Another variation of the question above:

35. A product that represents a clear technological advance over competing products can generally
command a high price. Because technological advances tend to be quickly surpassed and
companies want to make large profits while they still can, many companies charge the
maximum possible price for such a product. But large profits on the new product will give
competitors a strong incentive to quickly match the new product's capabilities. Consequently,
the strategy to maximize overall profit from a new product is to charge less than the greatest
possible price.
In the argument above, the two portions in boldface play which of the following roles?
A. The first is a consideration raised to argue that a certain strategy is counterproductive; the
second presents that strategy.
B. The first is a consideration raised to support the strategy that the argument recommends; the
second presents that strategy.
C. The first is a consideration raised to help explain the popularity of a certain strategy; the
second presents that strategy.
D. The first is an assumption, rejected by the argument, that has been used to justify a course of
action; the second presents that course of action.
E. The first is a consideration that has been used to justify adopting a certain strategy; the second
presents the intended outcome of that strategy.

By Sandeep Gupta, GMAT Perfect 800 | www.top-one-percent.com 114


THE MOST AUTHORITATIVE GUIDE EVER WRITTEN ON GMAT CRITICAL REASONING

Another variation of the question above:

36. A product that represents a clear technological advance over competing products can generally
command a high price. Surprisingly, perhaps, the strategy to maximize overall profits
from a new product is to charge less than the maximum price the market will bear.
Many companies charge the maximum possible price for such a product, because they want to
make as much profit as they can and technological advances tend to be quickly surpassed. The
drawback is that large profits on the new product give competitors an incentive to
quickly develop a product to match the rival product's capabilities.
In the argument above, the two portions in boldface play which of the following roles?
A. The first is the position that the argument advocates; the second presents grounds for rejecting
an alternate position.
B. The first is the position that the argument advocates; the second is an alternative position that
the argument rejects.
C. The first presents a strategy for achieving a certain goal, the second presents a drawback to that
strategy.
D. The first presents a strategy for achieving a certain goal, the second presents grounds for
preferring a different goal.
E. The first presents a strategy that, according to the argument, is ineffective; the second presents
a way of improving the effectiveness of that strategy.

By Sandeep Gupta, GMAT Perfect 800 | www.top-one-percent.com 115


THE MOST AUTHORITATIVE GUIDE EVER WRITTEN ON GMAT CRITICAL REASONING

37. Newsletter: A condominium generally offers more value for its cost than an
individual house because of economies of scale. The homeowners in a condominium
association can collectively buy products and services that they could not afford on their own.
And since a professional management company handles maintenance of common areas,
condominium owners spend less time and money on maintenance than individual
homeowners do.
The two portions in boldface play which of the following roles in the newsletter's argument?
A. The first is the argument's main conclusion; the second is another conclusion supporting the
first.
B. The first is a premise, for which no evidence is provided; the second is the argument's only
conclusion.
C. The first is a conclusion supporting the second; the second is the argument's main conclusion.
D. The first is the argument's only conclusion; the second is a premise, for which no evidence is
provided.
E. Both are premises, for which no evidence is provided, and both support the argument's only
conclusion.

By Sandeep Gupta, GMAT Perfect 800 | www.top-one-percent.com 116


THE MOST AUTHORITATIVE GUIDE EVER WRITTEN ON GMAT CRITICAL REASONING

38. Aroca City currently funds its public schools through taxes on property. In place of this
system, the city plans to introduce a sales tax of 3 percent on all retail sales in the
city. Critics protest that 3 percent of current retail sales falls short of the amount raised for
schools by property taxes. The critics are correct on this point. Nevertheless, implementing
the plan will probably not reduce the money going to Aroca's schools. Several large
retailers have selected Aroca City as the site for huge new stores, and these are certain to draw
large numbers of shoppers from neighboring municipalities, where sales are taxed at rates of 6
percent and more. In consequence, retail sales in Aroca City are bound to increase substantially.
In the argument given, the two portions in boldface play which of the following roles?
A. The first presents a plan that the argument concludes is unlikely to achieve its goal; the second
expresses that conclusion.
B. The first presents a plan that the argument concludes is unlikely to achieve its goal; the second
presents evidence in support of that conclusion.
C. The first presents a plan that the argument contends is the best available; the second is a
conclusion drawn by the argument to justify that contention.
D. The first presents a plan one of whose consequences is at issue in the argument; the second
is the argument's conclusion about that consequence.
E. The first presents a plan that the argument seeks to defend against a certain criticism; the
second is that criticism.

By Sandeep Gupta, GMAT Perfect 800 | www.top-one-percent.com 117


THE MOST AUTHORITATIVE GUIDE EVER WRITTEN ON GMAT CRITICAL REASONING

Another variation of the question above:

39. Aroca City currently funds its public schools through taxes on property. In place of this system,
the city plans to introduce a sales tax of three percent on all retail sales in the city. Critics protest
that three percent of current retail sales falls short of the amount raised for schools
by property taxes. The critics are correct on this point. Nevertheless, implementing the plan
will probably not reduce the money going to Aroca’s schools. Several large retailers have selected
Aroca City as the site for huge new stores, and these are certain to draw large numbers of
shoppers from neighboring municipalities, where sales are taxed at rates of six percent and more.
In consequence, retail sales in Aroca City are bound to increase substantially.
In the argument given, the two potions in boldface play which of the following roles?

A. The first is an objection that has been raised against a certain plan; the second is a prediction
that, if accurate, undermines the force of that objection.
B. The first is a criticism, endorsed by the argument, of a funding plan; the second is a point the
argument makes in favor of adopting an alternative plan.
C. The first is a criticism, endorsed by the argument, of a funding plan; the second is the main
reason cited by the argument for its endorsement of the criticism.
D. The first is a claim that the argument seeks to refute; the second is the main point used by the
argument to show that the claim is false.
E. The first is a claim that the argument accepts with certain reservations; the second presents
that claim in a rewarding that is not subject to those reservations.

By Sandeep Gupta, GMAT Perfect 800 | www.top-one-percent.com 118


THE MOST AUTHORITATIVE GUIDE EVER WRITTEN ON GMAT CRITICAL REASONING

40. In order to achieve self-sufficiency in electricity production, the Hasarian government


proposes to construct eleven huge hydroelectric power plants. Although this is a
massive project, it is probably not massive enough to achieve the goal. It is true that adding the
projected output of the new hydroelectric plants to the output that Hasaria can
achieve now would be enough to meet the forecast demand for electricity. It will,
however, take at least fifteen years to complete the project and by then the majority of Hasaria's
current power plants will be too old to function at full capacity.

In the argument given, the two portions in boldface play which of the following roles?
A. The first introduces a proposed course of action for which the argument provides support;
the second gives evidence in support of that course of action.
B. The first introduces a proposed course of action for which the argument provides support;
the second gives a reason for not adopting a possible alternative course of action.
C. The first introduces a plan that the argument evaluates; the second provides evidence that is
used to support that plan against possible alternatives.
D. The first introduces a proposed plan for achieving a certain goal; the second is a claim that
has been used in support of the plan but that the argument maintains is inaccurate.
E. The first introduces a proposed plan for achieving a certain goal; the second provides
evidence that is used to support the argument's evaluation of that plan.

By Sandeep Gupta, GMAT Perfect 800 | www.top-one-percent.com 119


THE MOST AUTHORITATIVE GUIDE EVER WRITTEN ON GMAT CRITICAL REASONING

Variation on an already discussed question:

41. Historian: Newton developed mathematical concepts and techniques that are fundamental to
modern calculus. Leibniz developed closely analogous concepts and techniques. It has
traditionally been thought that these discoveries were independent. Researchers have, however,
recently discovered notes of Leibniz's that discuss one of Newton's books on mathematics.
Several scholars have argued that since the book includes a presentation of Newton's
calculus concepts and techniques, and since the notes were written before Leibniz's own
development of calculus concepts and techniques, it is virtually certain that the traditional
view is false. A more cautious conclusion than this is called for, however. Leibniz's notes are
limited to early sections of Newton's book, sections that precede the ones in which Newton's
calculus concepts and techniques are presented.

In the historian’s reasoning, the two portions in boldface play which of the following roles?
A. The first is a claim that the historian rejects; the second is a position that that claim has been
used to support.
B. The first is evidence that has been used to support a conclusion about which the historian
expresses reservations; the second is that conclusion.
C. The first provides evidence in support of a position that the historian defends; the second is
that position.
D. The first and the second each provide evidence in support of a position that the historian
defends.
E. The first has been used in support of a position that the historian rejects; the second is a
conclusion that the historian draws from that position.

By Sandeep Gupta, GMAT Perfect 800 | www.top-one-percent.com 120


THE MOST AUTHORITATIVE GUIDE EVER WRITTEN ON GMAT CRITICAL REASONING

42. Most of Western music since the Renaissance has been based on a seven-note scale known as the
diatonic scale, but when did the scale originate? A fragment of a bone flute excavated at a
Neanderthal campsite has four holes, which are spaced in exactly the right way for playing the
third through sixth notes of a diatonic scale. The entire flute must surely have had more
holes, and the flute was made from a bone that was long enough for these additional holes to
have allowed a complete diatonic scale to be played. Therefore, the Neanderthals who made
the flute probably used a diatonic musical scale.

In the argument given, the two portions in boldface play which of the following roles?
A. The first is presented as evidence that is confirmed by data presented elsewhere in the argument
given; the second states a hypothesis that this evidence is used to undermine.
B. The first is an opinion, for which no supporting evidence is presented in the argument given, that
is used to support the main conclusion of the argument; the second is that main conclusion.
C. The first describes a discovery as undermining the position against which the argument is
directed; the second states the main conclusion of the argument.
D. The first is a preliminary conclusion drawn on the basis of evidence presented elsewhere in the
argument given; the second is the main conclusion that this preliminary conclusion supports.
E. The first provides evidence to support the main conclusion of the argument; the second states a
subsidiary conclusion that is drawn in order to support the main conclusion stated earlier in the
argument.

By Sandeep Gupta, GMAT Perfect 800 | www.top-one-percent.com 121


THE MOST AUTHORITATIVE GUIDE EVER WRITTEN ON GMAT CRITICAL REASONING

Another variation of the question above:

43. Most of Western music since the Renaissance has been based on a seven-note scale known as the
diatonic scale, but when did the scale originate? A fragment of a bone flute excavated at a
Neanderthal campsite has four holes, which are spaced in exactly the right way for
playing the third through sixth notes of a diatonic scale. The entire flute must surely
have had more holes, and the flute was made from a bone that was long enough for these
additional holes to have allowed a complete diatonic scale to be played. Therefore, the
Neanderthals who made the flute probably used a diatonic musical scale.
In the argument given, the two portions in boldface play which of the following roles?
A. The first introduces evidence to support the main conclusion of the argument; the second is the
main conclusion stated in the argument.
B. The first introduces evidence to support the main conclusion of the argument; the second
presents a position to which the argument is opposed.
C. The first describes a discovery as undermining the position against which the argument as a
whole is directed; the second states the main conclusion of the argument.
D. The first introduces the phenomenon that the argument as a whole seeks to explain; the second
presents a position to which the argument is opposed.
E. The first introduces the phenomenon that the argument as a whole seeks to explain; the second
gives a reason to rule out one possible explanation.

By Sandeep Gupta, GMAT Perfect 800 | www.top-one-percent.com 122


THE MOST AUTHORITATIVE GUIDE EVER WRITTEN ON GMAT CRITICAL REASONING

44. Local authorities are considering an amendment to the litter law that would raise the fine for
littering in the community picnic area to $1,000. Since the inception of the litter law,
incremental increases in the littering fine have proven to be consistently effective
at further reducing the amount of litter in the community picnic area. However,
raising the fine to $1,000 would actually have the unintended effect of increasing the amount of
litter in the picnic area. Picnic area users would perceive this fine to be unreasonable
and unenforceable, and would disregard the litter law altogether.
In the argument, the two portions in boldface play which of the following roles?
A. The first is irrefutable evidence that the author offers in support of a prediction; the second
is that prediction.
B. The first is a statement of causation that the author predicts will be repeated in the case at
hand; the second raises evidence against this prediction.
C. The first is a statement of fact that the author accepts to be true; the second is presented as a
consequence of this fact.
D. The first is evidence that weakens the main position that the author defends; the second is
that position.
E. The first is a statement of causation that the author predicts will not hold in the case at hand;
the second offers a line of reasoning to support this prediction.

By Sandeep Gupta, GMAT Perfect 800 | www.top-one-percent.com 123


THE MOST AUTHORITATIVE GUIDE EVER WRITTEN ON GMAT CRITICAL REASONING

45. Political analyst: A party that temporarily positions itself in the negligible crack between the
American right and left will do little to expand the public debate. What America needs is a
permanent third party. Some claim that America’s success stems from the two-party
system. These people say that a third party would make the passage of legislation and thus
governance impossible. Furthermore, they point to the current sluggish pace of government as
proof that the country cannot bear the burden of a third party. Yet, most European countries
have multi-party systems and few complain about any inability to govern there.
Which of the following best describes the functions of the two sections in boldface in the
argument above?

A. The first is the main point of the argument; the second is a premise that supports that point.
B. The first opposes the premises of the argument; the second is the claim that the argument
supports.
C. The first supports the main position held by opponents of the main point; the second is a
premise that argues against that position.
D. The first is the primary claim made by opponents of the main point of the argument; the
second is evidence proposed in opposition to the first.
E. The first is a claim made by opponents of the main point of the argument; the second is the
claim that the first opposes.

By Sandeep Gupta, GMAT Perfect 800 | www.top-one-percent.com 124


THE MOST AUTHORITATIVE GUIDE EVER WRITTEN ON GMAT CRITICAL REASONING

46. Letter to the editor: Proposition Q, a controversial measure on this year’s ballot, would prohibit
the ownership of handguns within the city’s limits. Under the plan, gun owners would have a 90-
day grace period to turn in their weapons to authorities. Proponents of the proposition argue that
fewer handguns on the streets would lead to less violent crime, making the city
safer for all of its citizens. Unfortunately, the ban would actually have the opposite effect.
Since only law-abiding citizens would honor the ban, armed criminals would not only keep
their weapons but would also have the confidence to act with impunity on a
population that could no longer defend itself. In the letter to the editor, the two portions
in boldface play which of the following roles?
A. The first is an observation that the author uses to support a particular position; the second is
that position.
B. The first is a pattern of cause and effect that the author believes to be true; the second offers
evidence to contradict this pattern.
C. The first is a position that the author argues will not hold in this case; the second is the
author’s position.
D. The first is a prediction that the author believes to be untrue. The second is a statement of
fact that undermines the author’s position.
E. The first is a direct relationship that the author believes will not hold in this case; the second
offers evidence in support of the author’s position.

By Sandeep Gupta, GMAT Perfect 800 | www.top-one-percent.com 125


THE MOST AUTHORITATIVE GUIDE EVER WRITTEN ON GMAT CRITICAL REASONING

47. Scientist: Evolutionary biology has long held that the most attractive males of a species, defined
as those with the highest quality physical traits that have no Darwinian survival value, will draw
the most female mates. The resulting male offspring will inherit that attractiveness
and themselves have more children as a result, thus ensuring widespread dissemination
of the grandparents' genes. Recently, however, scientists have found that the sons of "high
quality" male flycatchers failed to inherit the father's mating status. Further, the most attractive
males were so busy mating that they neglected their offspring; as a result, the sons of
homelier birds, who took better care of their offspring, had more success
at propagating the species. The two portions in boldface play which of the following roles
in the scientist's argument?

A. The first is the conclusion of a theory disputed by the scientist; the second is the scientist's
new contention based upon the latest evidence.
B. The first is a premise of a long-held biological theory; the second is an example of how this
theory works.
C. The first is an explanation of how a biological theory is thought to work; the second is an
example of research results that do not support this theory.
D. The first is an example of a theory that used to be prevalent; the second is the new theory that
is now considered predominant by scientists.
E. The first introduces a long-held theory that the scientist is going to disprove; the second is
the scientist's new theory to replace the one she disproved.

By Sandeep Gupta, GMAT Perfect 800 | www.top-one-percent.com 126


THE MOST AUTHORITATIVE GUIDE EVER WRITTEN ON GMAT CRITICAL REASONING

48. Traditionally, video game manufacturers have been most strongly influenced by serious video
gamers. Because devoted gamers have historically purchased the majority of video games,
companies react to the desires of this market segment. Normally, devoted gamers crave speed
and action; thus, most manufacturers continue to produce games with faster chips
and flashier graphics. Unfortunately, faster chips and flashier graphics are no longer in the
industry’s best interest. The devoted gaming market is deeply stagnant, and it won’t
soon expand. To infuse new life into the video game market, manufacturers must simplify the
functionality of their games. By doing so, current non-gamers will be attracted to join the ranks
of video game fans. In the argument, the two portions in boldface play which of the following
roles?
A. The first is a situation that the author believes to be true; the second offers evidence to explain
this situation.
B. The first is a situation that the author argues should not continue; the second provides
evidence that supports the author’s position.
C. The first is a statement of fact that contradicts the author’s position; the second is the author’s
position.
D. The first is a statement of fact that supports the author’s position; the second is a
consideration that weighs against the author’s position.
E. The first is a prediction that the author believes should not hold in this case; the second is an
assumption that weighs against the author’s position.

By Sandeep Gupta, GMAT Perfect 800 | www.top-one-percent.com 127


THE MOST AUTHORITATIVE GUIDE EVER WRITTEN ON GMAT CRITICAL REASONING

49. The city government should invest surplus funds in improving the city's transportation network.
Most of the network was put in place at a time when the city was much smaller in
both area and population. The subway system is outdated and understaffed. The buses rarely
run on schedule and their routes are inconvenient. If the city does not make changes soon
to the network, it will see many of its prized industries relocate to more convenient
cities and, as a result, the city's financial health will be jeopardized.
In the argument above, the two portions in boldface play which of the following roles?

A. The first is an explanation of a current state of affairs; the second is a prediction based on that
state of affairs.
B. The first is a statement of fact in opposition to the author's conclusion; the second is that
conclusion.
C. The first emphasizes an existing problem; the second offers a proposal to solve that problem.
D. The first is information the author suggests has been overlooked in the situation at hand; the
second describes that situation.
E. The first is a justification of an impending problem; the second describes the consequences
of that problem.

By Sandeep Gupta, GMAT Perfect 800 | www.top-one-percent.com 128


THE MOST AUTHORITATIVE GUIDE EVER WRITTEN ON GMAT CRITICAL REASONING

50. Chief Economist: Usually, the release of economic data about higher-than-expected
growth in the Gross Domestic Product (GDP) results in an increase in stock prices.
However, this quarter, the release of data about strong GDP growth is most likely to result in a
decrease rather than an increase in stock prices. Robust GDP growth will lead to higher
interest rates, increasing the attractiveness of bonds and causing a shift of capital
from equity to debt securities.
In the above argument, the statements in boldface play which of the following roles?
A. The first acknowledges a consideration against the main conclusion of the chief economist;
the second is that conclusion.
B. The first is a pattern of cause and effect that the chief economist predicts will not hold in the
case at issue; the second offers a consideration in support of that prediction.
C. The first is a generalization that the chief economist accepts as true; the second is a
consequence that follows from that generalization.
D. The first is evidence that the chief economist provides in support of a certain prediction; the
second is that prediction.
E. The first is a pattern of cause and effect that the chief economist predicts will be repeated in
the case at issue; the second acknowledges a circumstance in which that pattern would not
hold.

By Sandeep Gupta, GMAT Perfect 800 | www.top-one-percent.com 129


THE MOST AUTHORITATIVE GUIDE EVER WRITTEN ON GMAT CRITICAL REASONING

51. Political Analyst: After the Soviet Union collapsed, some hoped that freedom would encourage
Russians to multiply, but as a result of dislocation and insecurity, the Russian population
continues to dwindle at the rate of 700,000 a year. The government proposes to address the
problem with a wide range of financial incentives, along with investments in improved health
care, road safety and the like. These are positive measures, but they have been tried before,
to little avail. A better plan to reverse the population decline is to improve the country's
governance in both the public and the private sphere. If a greater part of the population
participated in important decisions and shared in the country's wealth, then larger
families would result. In addition, if corruption and greed among the elite were curbed,
public health would improve and average life expectancy would increase. The two boldfaced
statements serve what function in the argument above?
A. The first is the main point of the analyst's argument; the second is a premise that supports
the first.
B. The first is a premise that undermines an alternative to the analyst's proposal; the second is
a premise that supports the analyst's main claim.
C. The first is a premise that contradicts the main point made by the analyst; the second is the
main point of the argument.
D. The first is a premise that supports a proposal; the second is that proposal.
E. The first is a conclusion that the argument endorses; the second is a premise that opposes
that conclusion.

By Sandeep Gupta, GMAT Perfect 800 | www.top-one-percent.com 130


THE MOST AUTHORITATIVE GUIDE EVER WRITTEN ON GMAT CRITICAL REASONING

52. United Energy recently invested in a series of large windmills which are able to produce
renewable energy with minimal negative effect to the environment. The company has not drilled
oil wells in the same area, even though greater revenues and profits could be generated
from oil wells. Because any drilling would disrupt the native habitat of certain marine species
in the area, some environmentalists assert that, by foregoing this drilling, United Energy has
established that it places environmental impact over financial returns. However,
United Energy may be acting in a manner consistent with its financial goals. Recent patterns of
increasing annual hurricane activity have some experts questioning the long-term viability and
profitability of oil wells in the area. The two boldfaced portions play which of the following
roles?

A. The first supports the conclusion of the argument; the second calls that conclusion into
question.
B. The first states the conclusion of the argument; the second supports that conclusion.
C. The first supports the environmentalists’ conclusion; the second states that conclusion.
D. The first states the environmentalists’ conclusion; the second provides a consideration in
support of that conclusion.
E. The first supports the conclusion of the argument; the second also supports the conclusion of
the argument.

By Sandeep Gupta, GMAT Perfect 800 | www.top-one-percent.com 131


THE MOST AUTHORITATIVE GUIDE EVER WRITTEN ON GMAT CRITICAL REASONING

53. Marketing Analyst: Traditionally, introducing a new consumer product contributes to


the diversity of a company’s product mix and results in an overall increase in
revenues and profits. However, our launch of a new brand of soft drinks, Cool Breeze,
planned for the end of this fiscal year, will almost certainly reduce rather than increase the
company’s profits. Since we already offer a line of chilled refreshments that are very similar to
Cool Breeze, the launch of the new drink is likely to cannibalize our current sales rather than
attract new customers. Additionally, since the costs of producing Cool Breeze are higher
than those of the other chilled refreshments, a large part of our current sales will
be replaced with lower-margin revenues.
In the argument above, which of the following statements best describes the role played by each
portion in boldface?
A. The first is a pattern of cause and effect that the marketing analyst predicts will be repeated
in the case at issue; the second acknowledges a circumstance in which that pattern would not
hold.
B. The first is a generalization that the marketing analyst accepts as true; the second is a
consequence that follows from that generalization.
C. The first acknowledges a consideration against the main conclusion of the marketing analyst;
the second is that conclusion.
D. The first is a pattern of cause and effect that the marketing analyst predicts will not hold in
the case at issue; the second offers a consideration in support of that prediction.
E. The first is evidence that the marketing analyst provides in support of a certain prediction;
the second is that prediction.

By Sandeep Gupta, GMAT Perfect 800 | www.top-one-percent.com 132


THE MOST AUTHORITATIVE GUIDE EVER WRITTEN ON GMAT CRITICAL REASONING

54. Letter writer: Illegal drug use is often associated with other serious problems, such as armed
robbery and other violent crimes. Statistics indicate that each time police increase their
enforcement of anti-drug laws in the city, the number of violent crimes committed
in the city declines as a result. However, eliminating criminal penalties for drug use would
almost certainly decrease rather than increase the incidence of armed robbery and other violent
crime. If drugs were no longer illegal, the price would drop precipitously, and drug
users would no longer need to commit crimes to acquire the money necessary to
support their drug habits.
In the letter writer’s argument, the two portions in boldface play which of the following roles?
A. The first is support offered by the letter writer for a certain forecast; the second is that
forecast.
B. The first acknowledges an observation that refutes the main position that the letter writer
takes; the second is that position.
C. The first is a direct relationship between two activities that the letter writer argues is an
infallible predictor of future events; the second acknowledges a circumstance in which that
relationship would not apply.
D. The first is a direct relationship between two activities that the letter writer predicts will not
hold in the future; the second offers information that, if true, would support that prediction.
E. The first is a statement that the letter writer believes is true; the second is presented as a
logical inference drawn from the truth of that statement.

By Sandeep Gupta, GMAT Perfect 800 | www.top-one-percent.com 133


THE MOST AUTHORITATIVE GUIDE EVER WRITTEN ON GMAT CRITICAL REASONING

55. Real Estate Agent: Next year, the occupancy rate for residential rental properties in
Riverton will increase despite projections of decreasing population in the city.
Generally, when a city’s population decreases, so does the occupancy rate for residential rental
properties. However, in the continuing climate of increasing interest rates on home
mortgages, an unusually large number of Riverton residents who would typically
buy homes will opt to rent instead. The boldfaced statements in the argument above play
which of the following roles?
A. The first is a conclusion drawn by the agent; the second is evidence that weighs against the
agent’s conclusion.
B. The first is a consideration that weighs against the agent’s prediction; the second is that
prediction.
C. The first describes a pattern of cause and effect; the second describes a situation for which
this pattern will not hold.
D. The first is the agent’s prediction; the second is reasoning offered in support of this
prediction.
E. The first is an undisputed statement of fact; the second is a position that is supported by this
fact.

By Sandeep Gupta, GMAT Perfect 800 | www.top-one-percent.com 134


THE MOST AUTHORITATIVE GUIDE EVER WRITTEN ON GMAT CRITICAL REASONING

56. Policy analyst: Most government agencies become less effective over time. Some
experts go so far as to recommend that every agency be eliminated after 10 years and created
anew by replacing all of its existing personnel and revamping its bureaucratic structure.
However, this policy would be impractical since certain government agencies perform
vital functions, such as protecting national security, and therefore cannot afford
even temporary upheaval.
In the policy analyst's argument above, the two portions in boldface play which of the following
roles?

A. The first is evidence offered in support of an opinion that the policy analyst rejects; the second
offers information that contradicts that evidence.
B. The first is a premise that the policy analyst accepts but argues against; the second offers
evidence that supports the analyst's position.
C. The first is a position that the policy analyst argues against; the second is the position that
the analyst defends.
D. The first is a generalization that the policy analyst accepts as accurate and is used as the basis
for an opinion that the analyst rejects; the second is a consideration used to defend the
analyst's position.
E. The first is a generalization that the policy analyst accepts as accurate and is used as the basis
for the analyst's position; the second offers another consideration used to defend that
position.

By Sandeep Gupta, GMAT Perfect 800 | www.top-one-percent.com 135


THE MOST AUTHORITATIVE GUIDE EVER WRITTEN ON GMAT CRITICAL REASONING

57. Political Candidate: Government subsidized prescription drug plans that would allow
individuals significant choice in determining their benefits and costs are
deceptively appealing to numerous stakeholders. However, buying prescription drug
coverage, like buying health insurance coverage, is not like buying a car. The consumer
cannot predict his or her future health needs. Moreover, the administrators of the choice-
based drug plans under consideration are allowed to change the drugs they cover and the prices
they charge at any time; this renders informed consumer choice meaningless and makes securing
appropriate coverage a crap shoot. Older and disabled individuals, the predominant consumers
of government subsidized prescription drug plans, should be offered drug coverage alternatives
that do not force them to gamble with their health.
In the argument above, the two portions in boldface play which of the following roles?

A. The first is a fact that the candidate argues against; the second is the ultimate claim that the
candidate supports.
B. The first is an observation which the candidate acknowledges as true but to which he is
ultimately opposed; the second is a claim that the candidate uses as evidence to support his
ultimate position.
C. The first is an observation that the candidate acknowledges as true but unfortunate; the
second is an assertion that the candidate makes to support his ultimate position.
D. The first is an observation that the candidate argues against; the second is an observation that
the candidate supports.
E. The first is an observation made by the candidate; the second is an assertion that the
candidate ultimately opposes.

By Sandeep Gupta, GMAT Perfect 800 | www.top-one-percent.com 136


THE MOST AUTHORITATIVE GUIDE EVER WRITTEN ON GMAT CRITICAL REASONING

58. Ambiguity inspires interpretation. The saying, "We are the measure of all things," for
instance, has been interpreted by some people to imply that humans are centrally important in
the universe, while others have interpreted it to mean simply that, since all knowledge is human
knowledge, humans must rely on themselves to find the truth.
The boldfaced part figures in the argument in which one of the following ways?
A. It is used to support the argument's conclusion.

B. It is an illustration of the claim that we are the measure of all things.

C. It is compatible with either accepting or rejecting the argument's conclusion.

D. It is a view that other statements in the argument are intended to support.

E. It sets out a difficulty the argument is intended to solve.

59. Economist: To the extent that homelessness arises from a lack of available housing,
it should not be assumed that the profit motive is at fault. Private investors will, in general,
provide housing if the market allows them to make a profit; it is unrealistic to expect investors to
take risks with their property unless they get some benefit in return.
Which one of the following most accurately describes the role played in the economist's
argument by the boldface portion?
A. It limits the application of the argument to a part of the problem.

B. It suggests that the primary cause of homelessness is lack of available housing.

C. It is offered as evidence crucial to the conclusion.

D. It expresses the conclusion to be argued for.

E. It suggests a possible solution to the problem of homelessness.

By Sandeep Gupta, GMAT Perfect 800 | www.top-one-percent.com 137


THE MOST AUTHORITATIVE GUIDE EVER WRITTEN ON GMAT CRITICAL REASONING

60. People's political behavior frequently does not match their rhetoric. Although many complain
about government intervention in their lives, people tend not to reelect inactive
politicians. But a politician's activity consists largely in the passage of laws whose enforcement
affects voters' lives. Thus, voters often reelect politicians whose behavior they resent.
Which one of the following most accurately describes the role played in the argument by the
boldfaced portion?
A. It describes a phenomenon for which the argument's conclusion is offered as an explanation.

B. It is a premise offered in support of the conclusion that voters often reelect politicians whose
behavior they resent.

C. It is offered as an example of how a politician's activity consists largely in the passage of laws
whose enforcement interferes with voters' lives.

D. It is a generalization based on the claim that people complain about government intervention
in their lives.

E. It is cited as evidence that people's behavior never matches their political beliefs.

By Sandeep Gupta, GMAT Perfect 800 | www.top-one-percent.com 138


THE MOST AUTHORITATIVE GUIDE EVER WRITTEN ON GMAT CRITICAL REASONING

61. Some vegetarians have argued that there are two individually sufficient reasons for not eating
meat―one based on health considerations, and the other based on the aversion to living at the
expense of other conscious creatures. But suppose that eating meat were essential to
good health for humans. Then it would be less clear that an aversion to living at the expense of
other conscious creatures is enough of a reason to stop eating meat.
Which one of the following most accurately describes the role played in the argument by the
boldfaced part?
A. It is used to disprove the vegetarian position that we should not eat meat.

B. It is used to show that the two types of reasons cited in favor of vegetarianism are
independent.

C. It is used to disprove the claim that a vegetarian diet is healthy.

D. It is used to weaken the claim that the consciousness of animals is a sufficient reason for not
eating meat.

E. It is used to show that there is no sufficient reason for not eating meat.

By Sandeep Gupta, GMAT Perfect 800 | www.top-one-percent.com 139


THE MOST AUTHORITATIVE GUIDE EVER WRITTEN ON GMAT CRITICAL REASONING

62. Sociologist: Some people argue that capital punishment for theft was an essential part of the
labor discipline of British capitalism. Critics of such a view argue that more people were executed
for theft in preindustrial England than were executed in England after industrialization. But such
a criticism overlooks the fact that industrialization and capitalism are two very different
social phenomena, and that the latter predated the former by several centuries.
Which one of the following most accurately describes the role played in the passage by the
boldfaced portion?
A. It is cited as some evidence against the claim that capital punishment for theft was an
essential part of the labor discipline of British capitalism.

B. It is cited as a direct contradiction of the claim that capital punishment for theft was an
essential part of the labor discipline of British capitalism.

C. It is an attempt to conclusively prove the claim that capital punishment for theft was an
essential part of the labor discipline of British capitalism.

D. It is cited as a fact supporting the critics of the view that capital punishment for theft was an
essential part of the labor discipline of British capitalism.

E. It is an attempt to undermine the criticism cited against the claim that capital punishment
for theft was an essential part of the labor discipline of British capitalism.

By Sandeep Gupta, GMAT Perfect 800 | www.top-one-percent.com 140


THE MOST AUTHORITATIVE GUIDE EVER WRITTEN ON GMAT CRITICAL REASONING

63.Environmental organizations want to preserve the land surrounding the Wilgrinn


Wilderness Area from residential development. They plan to do this by purchasing that
land from the farmers who own it. That plan is ill-conceived: if the farmers did sell their land,
they would sell it to the highest bidder, and developers would outbid any other bidders. On the
other hand, these farmers will never actually sell any of the land, provided that
farming it remains viable. But farming will not remain viable if the farms are left
unmodernized, and most of the farmers lack the financial resources modernization requires. And
that is exactly why a more sensible preservation strategy would be to assist the farmers to
modernize their farms to the extent needed to maintain viability.
In the argument as a whole, the two boldface proportions play which of the
following roles?

A. The first presents a goal that the argument rejects as ill-conceived; the second is evidence that
is presented as grounds for that rejection.
B. The first presents a goal that the argument concludes cannot be attained; the second is a
reason offered in support of that conclusion.
C. The first presents a goal that the argument concludes can be attained; the second is a
judgment disputing that conclusion.
D. The first presents a goal, strategies for achieving which are being evaluated in the argument;
the second is a judgment providing a basis for the argument’s advocacy of a particular
strategy.
E. The first presents a goal that the argument endorses; the second presents a situation that the
argument contends must be changed if that goal is to be met in the foreseeable future.

By Sandeep Gupta, GMAT Perfect 800 | www.top-one-percent.com 141


THE MOST AUTHORITATIVE GUIDE EVER WRITTEN ON GMAT CRITICAL REASONING

64. Although the earliest surviving Greek inscriptions written in an alphabet date from the eighth
century B.C., a strong case can be made that the Greeks actually adopted alphabetic
writing at least two centuries earlier. Significantly, the text of these earliest surviving Greek
inscriptions sometimes runs from right to left and sometimes from left to right. Now, the Greeks
learned alphabetic writing from the Phoenicians, and in the process, they would surely have
adopted whatever convention the Phoenicians were then using with respect to the
direction of writing. Originally, Phoenician writing ran in either direction, but by the eighth
century B.C. it had been consistently written from right to left for about two centuries.
In the argument given, the two portions in boldface play which of the following roles?

A. The first is the position that the argument seeks to establish; the second reports a discovery
that has been used to support a position that the argument opposes.
B. The first is the position that the argument seeks to establish; the second presents an
assumption on which the argument relies.
C. The first presents evidence that is used in support of the position that the argument seeks to
establish; the second presents an assumption on which the argument relies.
D. The first is an objection raised against a position that the argument opposes; the second is
the position that the argument seeks to establish.
E. The first is an objection raised against a position that the argument opposes; the second is
evidence that has been used to support that position.

By Sandeep Gupta, GMAT Perfect 800 | www.top-one-percent.com 142


THE MOST AUTHORITATIVE GUIDE EVER WRITTEN ON GMAT CRITICAL REASONING

Another variation of the question above:

65. Although the earliest surviving Greek inscriptions written in an alphabet date from the eighth
century B.C., a strong case can be made that the Greeks actually adopted alphabetic writing at
least two centuries earlier. Significantly, the text of these earliest surviving Greek
inscriptions sometimes runs from right to left and sometimes from left to right.
Now, the Greeks learned alphabetic writing from the Phoenicians, and in the process, they
would surely have adopted whatever convention the Phoenicians were then using
with respect to the direction of writing. Originally, Phoenicians writing ran in either
direction, but by the eighth century B.C. it had been consistently written from right to left for
about two centuries.
In the argument given, the two portions in boldface play which of the following roles?
The first provides evidence in support of the position that the argument seeks to establish; the
second is that position.
The first provides evidence in support of the position that the argument seeks to establish; the
second presents an assumption on which that argument relies.
The first is an assumption that the argument concludes is unjustified; the second presents part
of the grounds for that conclusion.
The first is evidence that forms the basis for an objection to the position that the argument seeks
to establish; the second is that position.
The first is evidence that forms the basis for an objection to the position that the argument seeks
to establish; the second is a judgment that is introduced in order to call into question the
relevance of that evidence.

By Sandeep Gupta, GMAT Perfect 800 | www.top-one-percent.com 143


THE MOST AUTHORITATIVE GUIDE EVER WRITTEN ON GMAT CRITICAL REASONING

66.Criminologist: Some legislators advocate mandating a sentence of life in prison for anyone
who, having twice served sentences for serious crimes, is subsequently convicted of a third
serious crime. These legislators argue that such a policy would reduce crime
dramatically, since it would take people with a proven tendency to commit crimes off the
streets permanently. What this reasoning overlooks, however, is that people old enough to have
served two prison sentences for serious crimes rarely commit more than one subsequent crime.
Filling our prisons with such individuals would have exactly the opposite of the
desired effect, since it would limit our ability to incarcerate younger criminals, who commit a
far greater proportion of serious crimes.
In the argument as a whole, the two boldfaced portions play which of the following roles?

A. The first is a conclusion that the argument as a whole seeks to refute; the second is a claim
that has been advanced in support of that conclusion.
B. The first is a conclusion that the argument as a whole seeks to refute; the second is the main
conclusion of the argument.
C. The first is the main conclusion of the argument; the second is an objection that has been
raised against that conclusion.
D. The first is the main conclusion of the argument; the second is a prediction made on the basis
of that conclusion.
E. The first is a generalization about the likely effect of a policy under consideration in the
argument; the second points out a group of exceptional cases to which that generalization
does not apply.

By Sandeep Gupta, GMAT Perfect 800 | www.top-one-percent.com 144


THE MOST AUTHORITATIVE GUIDE EVER WRITTEN ON GMAT CRITICAL REASONING

67.Editorial: An arrest made by a Midville police officer is provisional until the officer has taken
the suspect to the police station and the watch commander has officially approved the arrest.
Such approval is denied if the commander judges that the evidence on which the provisional
arrest is based is insufficient. A government efficiency expert has found that almost all
provisional arrests meet standards for adequacy of evidence that watch commanders enforce.
The expert therefore recommends that the watch commander’s approval should no
longer be required since the officers’ time spent obtaining approval is largely wasted. This
recommendation should be rejected as dangerous, however, since there is no assurance
that the watch commanders’ standards will continue to be observed once approval is no longer
required.
In the editorial, the two portions in boldface play which of the following roles?

A. The first is a recommendation made by the editorial; the second acknowledges a potential
objection against that recommendation.
B. The first is a proposal against which the editorial is directed; the second is a judgment reached
by the editorial concerning that proposal.
C. The first provides evidence in support of a recommendation that the editorial supports; the
second is the conclusion reached by the editorial.
D. The first is a position that the editorial challenges; the second is a judgment that was made in
support of that challenged position.
E. The first is a recommendation that the editorial questions; the second provides evidence
against that recommendation.

By Sandeep Gupta, GMAT Perfect 800 | www.top-one-percent.com 145


THE MOST AUTHORITATIVE GUIDE EVER WRITTEN ON GMAT CRITICAL REASONING

Another variation of the question above:

68. Editorial: An arrest made by a Midville police officer is provisional until the officer has taken
the suspect to the police station and the watch commander has officially approved the arrest.
Such approval is denied if the commander judges that the evidence on which the provisional
arrest is based is insufficient. A government efficiency expert has observed that almost all
provisional arrests meet the standards for adequacy of evidence that the watch
commanders enforce. The expert has therefore recommended that because the officers’
time spent obtaining approval is largely wasted, the watch commander’s approval no
longer be required. This recommendation should be rejected as dangerous, however, since there
is no assurance that the watch commanders’ standards will continue to be observed once
approval is no longer required.
In the editorial, the two portions in boldface play which of the following roles?
A. The first is a claim, the accuracy of which is disputed by the editorial; the second is a conclusion
drawn in order to support the main conclusion of the editorial.
B. The first is an observation that the editorial disputes; the second is a conclusion that was drawn
from that observation.
C. The first is a finding that was used in support of a proposal that the editorial opposes; the second
is a judgment that was based on that finding and in turn was used to support the proposal.
D. The first is a finding introduced to support the main conclusion of the editorial; the second is
that main conclusion.
E. The first is a conclusion, the evidence for which the editorial evaluates; the second is part of the
evidence cited in favor of that conclusion.

By Sandeep Gupta, GMAT Perfect 800 | www.top-one-percent.com 146


THE MOST AUTHORITATIVE GUIDE EVER WRITTEN ON GMAT CRITICAL REASONING

69. Business Consultant: Some corporations shun the use of executive titles because they fear
that the use of titles indicating position in the corporation tends to inhibit
communication up and down the corporate hierarchy. Since an executive who uses a
title is treated with more respect by outsiders, however, use of a title can facilitate an
executive’s dealings with external businesses. The obvious compromise is for these
executives to use their corporate titles externally but not internally, since even if it is widely
known that the corporation’s executives use executive titles outside their organization, this
knowledge does not by itself inhibit communication within the corporation.
In the consultant’s reasoning, the two portions in boldface play which of the following roles?

A. The first presents an obstacle to achieving a certain goal; the second presents a reason for
considering that goal to be undesirable.
B. The first is a consideration that has led to the adoption of a certain strategy; the second
presents a reason against adopting that strategy.
C. The first describes a concern that the consultant dismisses as insignificant; the second is a
consideration that serves as the basis for that dismissal.
D. The first is a belief for which the consultant offers support; the second is part of that support.
E. The first is a belief against which evidence is offered; the second is part of the evidence offered
against that belief.

By Sandeep Gupta, GMAT Perfect 800 | www.top-one-percent.com 147


THE MOST AUTHORITATIVE GUIDE EVER WRITTEN ON GMAT CRITICAL REASONING

Another variation of the question above:

70. Business Consultant: Some corporations shun the use of executive titles because they
fear that the use of titles indicating position in the corporation tends to inhibit communication
up and down the corporate hierarchy. Since an executive who uses a title is treated with more
respect by outsiders, however, use of a title can facilitate an executive’s dealings with
external businesses. Clearly, corporations should adopt the compromise of encouraging their
executives to use their corporate titles externally but not internally, since even if it is widely
known that the corporation’s executives use titles outside their organization, this knowledge does
not by itself inhibit communication within the corporation.
In the consultant’s reasoning, the two portions in boldface play which of the following roles?
A. The first describes a strategy that has been adopted to avoid a certain problem; the second
presents a drawback to that strategy.
B. The first describes a strategy that has been adopted to avoid a certain problem; the second is a
consideration raised to call into question the effectiveness of that strategy as a means of achieving
that goal.
C. The first describes a strategy that has been adopted to avoid a certain problem; the second is a
consideration the consultant raises in questioning the significance of that problem.
D. The first is part of an explanation that the consultant offers for a certain phenomenon; the second
is that phenomenon.
E. The first describes a policy for which the consultant seeks to provide a justification; the second
is a consideration the consultant raises as part of that justification.

By Sandeep Gupta, GMAT Perfect 800 | www.top-one-percent.com 148


THE MOST AUTHORITATIVE GUIDE EVER WRITTEN ON GMAT CRITICAL REASONING

71. One of the limiting factors in human physical performance is the amount of oxygen that is
absorbed by the muscles from the bloodstream. Accordingly, entrepreneurs have begun selling
at gymnasiums and health clubs bottles of drinking water, labeled “SuperOXY,” that has extra
oxygen dissolved in the water. Such water would be useless in improving physical performance,
however, since the only way to get oxygen into the bloodstream so that it can be
absorbed by the muscles is through the lungs.
Which of the following, if true, would serve the same function in the argument as the statement
in boldface?

A. the water lost in exercising can be replaced with ordinary tap water
B. the amount of oxygen in the blood of people who are exercising is already more than the
muscle can absorb
C. world-class athletes turn in record performance without such water
D. frequent physical exercise increases the body’s ability to take in and use oxygen
E. lack of oxygen is not the only factor limiting human physical performance

Another variation of the question above:

72. One of the limiting factors in human physical performance is the amount of oxygen that is
absorbed by the muscles from the bloodstream. Accordingly, entrepreneurs have begun selling
at gymnasiums and health clubs bottles of drinking water, labeled “SuperOXY,” that has extra
oxygen dissolved in the water. Such water would be useless in improving physical performance,
however, since the amount of oxygen in the blood of someone who is exercising is
already more than the muscle cells can absorb.

Which of the following, if true, would serve the same function in the argument as the statement
in boldface?

A. world-class athletes turn in record performances without such water


B. frequent physical exercise increases the body’s ability to take in and use oxygen
C. the only way to get oxygen into the bloodstream so that it can be absorbed by the muscles is
through the lungs
D. lack of oxygen is not the only factor limiting human physical performance
E. the water lost in exercising can be replaced with ordinary tap water

By Sandeep Gupta, GMAT Perfect 800 | www.top-one-percent.com 149


THE MOST AUTHORITATIVE GUIDE EVER WRITTEN ON GMAT CRITICAL REASONING

73. During the past year, Pro-Tect Insurance Company's total payout on car-theft claims has been
larger than the company can afford to sustain. Pro-Tect cannot reduce the number of car-
theft policies it carries, so it cannot protect itself against continued large payouts
that way. Therefore, Pro-Tect has decided to offer a discount to holders of car-theft policies
whose cars have antitheft devices. Many policyholders will respond to the discount by installing
such devices, since the amount of the discount will within two years typically more than cover
the cost of installation. Thus, because cars with antitheft devices are rarely stolen,
Pro-Tect's plan is likely to reduce its annual payouts.
In the argument above, the two portions in boldface play which of the following roles?

A. The first and the second are each evidence offered by the argument as support for its main
conclusion.
B. The first presents a problem a response to which the argument assesses; the second is the
judgment reached by that assessment.
C. The first is the position the argument seeks to establish; the second is a judgment the
argument uses to support that position.
D. The first is a development that the argument seeks to explain; the second is a prediction the
argument makes in support of the explanation it offers.
E. The first presents a development whose likely outcome is at issue in the argument; the second
is a judgment the argument uses in support of its conclusion about that outcome.

By Sandeep Gupta, GMAT Perfect 800 | www.top-one-percent.com 150


THE MOST AUTHORITATIVE GUIDE EVER WRITTEN ON GMAT CRITICAL REASONING

Another variation of the question above:

74. During the past year, Pro-Tect Insurance Company’s total payouts on car-theft claims were larger
than the company can afford to sustain. Pro-Tect cannot reduce the number of car-theft
policies it carries, so cannot protect itself against continued large payouts that way. Therefore,
Pro-Tect has decided to offer a discount to holders of car-theft policies whose cars
have antitheft devices. Many policyholders will respond to the discount by installing antitheft
devices, since the amount of the discount will within two years typically more than cover the cost
of installation. Thus, because cars with antitheft devices are rarely stolen, Pro-Tect’s plan is likely
to reduce its annual payouts.
In the argument above, the two portions in boldface play which of the following roles?
A. The first rules out a certain strategy for achieving a goal; the second presents the strategy that
was adopted instead and whose effectiveness the argument assesses.
B. The first is a judgment made in support of a certain conclusion; the second is that conclusion.
C. The first has been used as a consideration to support adopting a certain strategy for achieving a
goal; the second reports a decision to adopt an alternative strategy.
D. The first provides evidence in favor of adopting a certain strategy for achieving a goal; the second
reports a decision to pursue an alternative goal.
E. The first is a consideration offered against adopting a certain strategy for achieving a goal; the
second is the main conclusion that the argument is seeking to establish.

By Sandeep Gupta, GMAT Perfect 800 | www.top-one-percent.com 151


THE MOST AUTHORITATIVE GUIDE EVER WRITTEN ON GMAT CRITICAL REASONING

75. Many winemakers use cork stoppers; but cork stoppers can leak, crumble, or become moldy, so
that those winemakers must often discard a significant proportion of their inventory of bottled
wine. Bottlemaster plastic stoppers, which cannot leak, crumble, or mold, have long been
available to winemakers, at a price slightly higher than that of traditional cork stoppers. Cork
prices, however, are expected to rise dramatically in the near future. Clearly, therefore,
winemakers who still use cork but wish to keep production costs from rising will
be forced to reconsider plastic stoppers. And since the wine-buying public’s association of
plastic stoppers with poor-quality wine is weakening, there is an excellent chance that the
Bottlemaster plastic stopper will gain an increased share of the marked for wine-
bottle stoppers. In the argument given, the two portions in boldface play which of the following
roles?

A. The first is a judgment that has been advanced in support of a position that the argument
opposes; the second is the main conclusion of the argument.
B. The first is a judgment that has been advanced in support of a position that the argument
opposes; the second is a conclusion drawn in order to support the main conclusion of the
argument.
C. The first is the main conclusion of the argument; the second provides evidence in support of
that main conclusion.
D. The first is the main conclusion of the argument; the second is a restatement of that main
conclusion.
E. The first is a conclusion drawn in order to support the main conclusion of the argument; the
second is that main conclusion.

By Sandeep Gupta, GMAT Perfect 800 | www.top-one-percent.com 152


THE MOST AUTHORITATIVE GUIDE EVER WRITTEN ON GMAT CRITICAL REASONING

76. Museums that house Renaissance oil paintings typically store them in environments that are
carefully kept within narrow margins of temperature and humidity to inhibit any deterioration.
Laboratory tests have shown that the kind of oil paint used in these paintings actually adjusts to
climatic changes quite well. If, as some museum directors believe, paint is the most sensitive
substance in these works, then by relaxing the standards for temperature and humidity
control, museums can reduce energy costs without risking damage to these
paintings. Museums would be rash to relax those standards, however, since results of
preliminary tests indicate that gesso, a compound routinely used by Renaissance artists to help
paint adhere to the canvas, is unable to withstand significant variations in humidity.
In the argument above, the two portions in boldface play which of the following roles?

A. The first is an objection that has been raised against the position taken by the argument; the
second is the position taken by the argument.
B. The first is the position taken by the argument; the second is the position that the argument
calls into question.
C. The first is a judgment that has been offered in support of the position that the argument calls
into question; the second is a circumstance on which that judgment is, in part based.
D. The first is a judgment that has been offered in support of the position that the argument calls
into question; the second is that position.
E. The first is a claim that the argument calls into question; the second is the position taken by
the argument.

By Sandeep Gupta, GMAT Perfect 800 | www.top-one-percent.com 153


THE MOST AUTHORITATIVE GUIDE EVER WRITTEN ON GMAT CRITICAL REASONING

77. Critics of certain pollution-control regulations have claimed that the money spent over the
last decade in order to reduce emissions of carbon monoxide and of volatile organic
compounds has been wasted. The evidence they offer in support of this claim might appear
compelling: despite the money spent, annual emissions of these pollutants have been increasing
steadily. This evidence is far from adequate, however, since over the last decade a
substantial number of new industrial facilities that emit these pollutants have been
built. In the reasoning given, the two portions in boldface play which of the following roles?

A. The first identifies a claim that the reasoning seeks to show is false; the second is evidence
that has been cited by others in support of that claim.
B. The first identifies a claim that the reasoning seeks to show is false; the second is a position
for which the reasoning seeks to provide support.
C. The first is a position that the reasoning contends is inadequately supported by the evidence;
the second is a position for which the reasoning seeks to provide support.
D. The first is a position that the reasoning contends is inadequately supported by the evidence;
the second is evidence used to support the reasoning’s contention.
E. The first is a position that the reasoning contends is inadequately supported by the evidence;
the second is evidence that has been used to support that position.

By Sandeep Gupta, GMAT Perfect 800 | www.top-one-percent.com 154


THE MOST AUTHORITATIVE GUIDE EVER WRITTEN ON GMAT CRITICAL REASONING

78. Paleontologist: About 2.8 million years ago, many species that lived near the ocean floor
suffered substantial population declines. These declines coincided with the onset of an ice age.
The notion that cold killed those bottom-dwelling creatures outright is misguided, however;
temperatures near the ocean floor would have changed very little. Nevertheless, the cold
probably did cause the population declines, though indirectly. Many bottom-dwellers
depended for food on plankton, small organisms that lived close to the surface and sank to the
bottom when they died. Most probably, the plankton suffered a severe population
decline as a result of sharply lower temperatures at the surface, depriving many
bottom-dwellers of food.
In the paleontologist's reasoning, the two portions in boldface play which of the following roles?

A. The first introduces the hypothesis proposed by the paleontologist; the second is a judgment
offered in spelling out that hypothesis.
B. The first introduces the hypothesis proposed by the paleontologist; the second is a position
that the paleontologist opposes.
C. The first is an explanation challenged by the paleontologist; the second is an explanation
proposed by the paleontologist.
D. The first is a judgment advanced in support of a conclusion reached by the paleontologist; the
second is that conclusion.
E. The first is a generalization put forward by the paleontologist; the second presents certain
exceptional cases in which that generalization does not hold.

By Sandeep Gupta, GMAT Perfect 800 | www.top-one-percent.com 155


THE MOST AUTHORITATIVE GUIDE EVER WRITTEN ON GMAT CRITICAL REASONING

79. Rain-soaked soil contains less oxygen than does drier soil. The roots of melon plants
perform less efficiently under the low-oxygen conditions present in rain-soaked
soil. When the efficiency of melon roots is impaired, the roots do not supply sufficient amounts
of the proper nutrients for the plants to perform photosynthesis at their usual levels. It follows
that melon plants have a lower-than-usual rate of photosynthesis when their roots
are in rain-soaked soil. When the photosynthesis of the plants slows, sugar stored in the fruits
is drawn off to supply the plants with energy. Therefore, ripe melons harvested after a prolonged
period of heavy rain should be less sweet than other ripe melons. In the argument given, the two
highlighted portions play which of the following roles?

A. The first states the conclusion of the argument as a whole; the second provides support for
that conclusion.
B. The first provides support for the conclusion of the argument as a whole; the second provides
evidence that supports an objection to that conclusion.
C. The first provides support for an intermediate conclusion that supports a further conclusion
stated in the argument; the second states that intermediate conclusion.
D. The first serves as an intermediate conclusion that supports a further conclusion stated in the
argument; the second states the position that the argument as a whole opposes.
E. The first states the position that the argument as a whole opposes; the second supports the
conclusion of the argument.

By Sandeep Gupta, GMAT Perfect 800 | www.top-one-percent.com 156


THE MOST AUTHORITATIVE GUIDE EVER WRITTEN ON GMAT CRITICAL REASONING

80. Exercise is an integral part of sustaining a high level of health. The body needs to be challenged
if it is to become fitter and stronger. However, overexertion can lead to injury. An exercise
program should always maintain a slow and natural pace of progression. In the
argument given, the two portions in boldface play which of the following roles?

A. The first is an opinion; the second is a conclusion based on that opinion.


B. The first is a factual possibility; the second is an opinion that opposes that possibility.
C. The first is a general opinion; the second is a conclusion that supports that opinion.
D. The first is a factual possibility; the second is a conclusion that presents a method of
preventing the occurrence of that possibility.
E. The first is a possible event of cause and effect; the second denies the possibility of such an
event to occur.

By Sandeep Gupta, GMAT Perfect 800 | www.top-one-percent.com 157


THE MOST AUTHORITATIVE GUIDE EVER WRITTEN ON GMAT CRITICAL REASONING

81. Consumer advocate: it is generally true, at least in this state, that lawyers who advertise a specific
service charge less for that service than lawyers who do not advertise. It is also true that each
time restrictions on the advertising of legal services have been eliminated, the
number of lawyers advertising their services has increased and legal costs to
consumers have declined in consequence. However, eliminating the state requirement
that legal advertisements must specify fees for specific services would almost certainly increase
rather than further reduce consumer’s legal costs. Lawyers would no longer have an incentive to
lower their fees when they begin advertising and if no longer required to specify fee
arrangements, many lawyers who now advertise would increase their fees.
In the consumer advocate’s argument, the two portions in boldface play which of the following
roles?
A. The first is a generalization that the consumer advocate accepts as true; the second is
presented as a consequence that follows from the truth of that generalization.

B. The first is a pattern of cause and effect that the consumer advocate argues will be repeated
in the case at issue; the second acknowledges a circumstance in which that pattern would not
hold.

C. The first is pattern of cause and effect that the consumer advocate predicts will not hold in
the case at issue; the second offers a consideration in support of that prediction.

D. The first is evidence that the consumer advocate offers in support of a certain prediction; the
second is that prediction.

E. The first acknowledges a consideration that weighs against the main position that the
consumer advocate defends; the second is that position.

By Sandeep Gupta, GMAT Perfect 800 | www.top-one-percent.com 158


THE MOST AUTHORITATIVE GUIDE EVER WRITTEN ON GMAT CRITICAL REASONING

82. Even those who believe that the art of each age and culture has its own standards of beauty must
admit that some painters are simply superior to others in the execution of their
artistic visions. But this superiority must be measured in light of the artist’s purposes, since
the high merits, for example, of Jose Rey Toledo’s work and his extraordinary artistic skills are
not in doubt, despite the fact that his paintings do not literally resemble what they represent.
The boldface portion plays which one of the following roles in the argument?
A. It is a hypothesis that the argument attempts to refute.

B. It is a generalization, one sort of objection to which the argument illustrates by giving an


example.

C. It is a claim that, according to the argument, is to be understood in a manner specified by the


conclusion.

D. It is a claim that the argument derives from another claim and that it uses to support its
conclusion.

E. It is a generalization that the argument uses to justify the relevance of the specific example it
cites.

By Sandeep Gupta, GMAT Perfect 800 | www.top-one-percent.com 159


THE MOST AUTHORITATIVE GUIDE EVER WRITTEN ON GMAT CRITICAL REASONING

83. Mathematician: Recently, Zubin Ghosh made headlines when he was recognized to have solved
the Hilbert Conjecture. Ghosh posted his work on the internet, rather than submitting it to
established journals. In fact, he has no job, let alone a university position; he lives alone
and has refused all acclaim. In reporting on Ghosh, the press unfortunately has reinforced the
popular view that mathematicians are antisocial loners. But mathematicians clearly form a
tightly knit community, frequently collaborating on important efforts; indeed, teams of
researchers are working together to extend Ghosh's findings.
In the argument above, the two portions in boldface play which of the following roles?

A. The first is an observation the author makes to illustrate a social pattern; the second is a
generalization of that pattern.
B. The first is evidence in favor of the popular view expressed in the argument; the second is a brief
restatement of that view.
C. The first is a specific example of a generalization that the author contradicts; the second is a
reiteration of that generalization.
D. The first is a specific counterexample to a generalization that the author asserts; the second is
that generalization.
E. The first is a judgment that counters the primary assertion expressed in the argument; the second
is a circumstance on which that judgment is based.

By Sandeep Gupta, GMAT Perfect 800 | www.top-one-percent.com 160


THE MOST AUTHORITATIVE GUIDE EVER WRITTEN ON GMAT CRITICAL REASONING

84. Media Critic: Network executives allege that television viewership is decreasing due to the
availability of television programs on other platforms, such as the internet and mobile devices.
These executives claim that declining viewership will cause advertising revenue to fall
and networks will thus be unable to spend the large sums necessary to produce
high-quality programming. That development, in turn, will lead to a dearth of programming
for the very devices that cannibalized television's audience. However, research shows that users
of alternative platforms are exposed to new programs and, as a result, actually increase the
number of hours per week that they watch television. This demonstrates that alternative
platforms will not prevent networks from increasing advertising revenue.
The portions in boldface play which of the following roles in the media critic's argument?

A. The first is a trend that weighs against the critic's claim; the second is that claim.
B. The first is a prediction that is challenged by the argument; the second is a finding upon which
the argument depends.
C. The first clarifies the reasoning behind the critic's claim; the second demonstrates why that claim
is flawed.
D. The first acknowledges a position that the network executives accept as true; the second is a
consequence of that position.
E. The first opposes the critic's claim through an analogy; the second outlines a scenario in which
that claim will not hold.

By Sandeep Gupta, GMAT Perfect 800 | www.top-one-percent.com 161


THE MOST AUTHORITATIVE GUIDE EVER WRITTEN ON GMAT CRITICAL REASONING

85. Many people praise High Renaissance painting for creating very realistic images from
observation, but scholars have documented that some High Renaissance painters
used pinhole cameras to project the likeness of their subjects onto the canvas and
painted from there. Thus, people who credit High Renaissance painters with superior artistic
skills are misguided. Painting from a projected image requires only an insignificant
amount of additional skill beyond that needed to copy a picture outright.
In the argument given, the two boldfaced portions play which of the following roles?
A. The first is a finding that has been used to support a conclusion that the argument rejects; the
second is a claim that supports that conclusion.
B. The first is a finding that has been used to support a conclusion that the argument rejects; the
second is that conclusion.
C. The first is a claim put forth to support a conclusion that the argument rejects; the second is a
consideration that is introduced to counter the force of that evidence.
D. The first is evidence that forms the basis for the position that the argument seeks to establish;
the second is a claim presented to solidify that position.
E. The first is evidence that forms the basis for the position that the argument seeks to establish;
the second is that position.

By Sandeep Gupta, GMAT Perfect 800 | www.top-one-percent.com 162


THE MOST AUTHORITATIVE GUIDE EVER WRITTEN ON GMAT CRITICAL REASONING

86. In an attempt to explain the cause of malaria, a deadly infectious disease, early European settlers
in Hong Kong attributed the malady to poisonous gases supposedly emanating from low-lying
swampland. In the 1880s, however, doctors determined that Anopheles mosquitoes were
responsible for transmitting the disease to humans after observing that the female of the
species can carry a parasitic protozoan that is passed on to unsuspecting humans
when a mosquito feasts on a person's blood.
What function does the statement in boldface fulfill with respect to the argument presented
above?
A. It provides support for the explanation of a particular phenomenon.
B. It presents evidence that contradicts an established fact.
C. It offers confirmation of a contested assumption.
D. It identifies the cause of an erroneous conclusion.
E. It proposes a new conclusion in place of an earlier conjecture.

By Sandeep Gupta, GMAT Perfect 800 | www.top-one-percent.com 163


THE MOST AUTHORITATIVE GUIDE EVER WRITTEN ON GMAT CRITICAL REASONING

87. Products with innovative and appealing designs relative to competing products can often
command substantially higher prices in the marketplace. Because design innovations are
quickly copied by other manufacturers, many consumer technology companies charge as
much as possible for their new designs to extract as much value as possible from them. But large
profits generated by the innovative designs give competitors stronger incentives to copy the
designs. Therefore, the best strategy to maximize overall profit from an innovative
new design is to charge less than the greatest possible price.
In the argument above, the two portions in boldface play which of the following roles?
A. The first is an assumption that supports a described course of action; the second provides a
consideration to support a preferred course of action.
B. The first is a consideration that helps explain the appeal of a certain strategy; the second presents
an alternative strategy endorsed by the argument.
C. The first is a phenomenon that makes a specific strategy unlikely to be successful; the second is
that strategy.
D. The first is a consideration that demonstrates why a particular approach is flawed; the second
describes a way to amend that approach.
E. The first is a factor used to rationalize a particular strategy; the second is a factor against that
strategy.

By Sandeep Gupta, GMAT Perfect 800 | www.top-one-percent.com 164


THE MOST AUTHORITATIVE GUIDE EVER WRITTEN ON GMAT CRITICAL REASONING

88.Journalist: Every election year at this time the state government releases the
financial disclosures that potential candidates must make in order to be eligible to
run for office. Among those making the required financial disclosure this year is a prominent
local businessman, Arnold Bergeron. There has often been talk in the past of Mr. Bergeron's
running for governor, not least from Mr. Bergeron himself. This year it is likely he finally
will, since those who have discounted the possibility of a Bergeron candidacy have
always pointed to the necessity of making financial disclosure as the main obstacle
to such a candidacy.
In the journalist's argument, the two boldfaced portions play which of the following roles?
A. The first provides information without which the argument lacks force; the second states the
main conclusion of the argument.
B. The first provides information without which the argument lacks force; the second states an
intermediate conclusion that is used to support a further conclusion.
C. The first describes a practice that the journalist seeks to defend; the second cites a likely
consequence of this practice.
D. The first states evidence bearing against the main conclusion of the argument; the second states
that conclusion.
E. Each provides evidence in support of an intermediate conclusion that supports a further
conclusion stated in the argument.

By Sandeep Gupta, GMAT Perfect 800 | www.top-one-percent.com 165


THE MOST AUTHORITATIVE GUIDE EVER WRITTEN ON GMAT CRITICAL REASONING

89. Hunter: Hunters alone are blamed for the decline in Greenrock National Forest's deer population
over the past ten years. Yet clearly, black bears have also played an important role in
this decline. In the past ten years, the forest's protected black bear population has risen sharply,
and examination of black bears found dead in the forest during the deer hunting season showed
that a number of them had recently fed on deer.
In the hunter's argument, the boldface portion plays which of the following roles?
A. It is the main conclusion of the argument.
B. It is an objection that has been raised against the main conclusion of the argument.
C. It is a judgment that the argument opposes.
D. It is a finding that the argument seeks to explain.
E. It provides evidence in support of the main conclusion of the argument.

By Sandeep Gupta, GMAT Perfect 800 | www.top-one-percent.com 166


THE MOST AUTHORITATIVE GUIDE EVER WRITTEN ON GMAT CRITICAL REASONING

90. Studies have shown that people who keep daily diet records are far more successful at
losing weight than people who don’t keep track of what they eat. Researchers believe
that many weight-loss efforts fail because people eat more calories than they intend to consume.
One study followed a group of patients who reported that they could not lose weight when
consuming only 1,200 calories a day. The study found that the group consumed, on
average, 47% more than it claimed and exercised 51% less. In contrast, when dieters
record what they eat, their actual consumption more closely matches their reported
consumption.
The two boldface portions in the argument above are best described by which of the following
statements?
A. The first is a conclusion reached by researchers; the second is evidence that that conclusion is
correct.
B. The first is an explanation of why a certain theory is thought to be true; the second is an example
of research results that support this theory.
C. The first is an example illustrating the truth of a certain theory; the second is a competing theory.
D. The first is a premise upon which the researchers base their opinion; the second illustrates that
their opinion is correct.
E. The first introduces a theory that the researchers have disproved; the second is the basis for the
researchers’ argument.

By Sandeep Gupta, GMAT Perfect 800 | www.top-one-percent.com 167


THE MOST AUTHORITATIVE GUIDE EVER WRITTEN ON GMAT CRITICAL REASONING

91. Weight loss programs that guarantee results mislead their customers. No program
can ensure that someone who follows it will lose weight. These programs prey upon unhappy and
insecure people who are often driven more by emotion than by reason. Moreover, many
people who lose weight while on a program eventually regain the weight within a
year. So, while the programs’ claims may be true for a short period, customers will be
disappointed in the long run.
The two portions in boldface play which of the following roles?
A. The first is a generalization; the second is a consequence of that generalization.
B. The first is the author’s position; the second is a consideration in support of that position.
C. The first is an opinion under examination; the second is evidence weighing against that opinion.
D. The first is an assertion that the author questions; the second is evidence in support of that
assertion.
E. The first is evidence in support of the author’s main point; the second is the author’s main point.

By Sandeep Gupta, GMAT Perfect 800 | www.top-one-percent.com 168


THE MOST AUTHORITATIVE GUIDE EVER WRITTEN ON GMAT CRITICAL REASONING

92. For nearly a century, physiologists erroneously believed that a buildup of lactic acid in
muscle tissue was responsible for the soreness that many people experience after
strenuous exercise. The acid, they claimed, is the waste produced by metabolic activity in the
muscle and reaches “threshold” levels, causing soreness, when the muscle has depleted its
oxygen supply. Researchers have recently discovered, however, that lactic acid is
actually the fuel that powers muscular activity. Therefore, the cause of muscle soreness
remains unknown.
In the argument above, the portions in boldface play which of the following roles?
A. The first is an assertion that the author accepts as true; the second is a consideration in support
of that assertion.
B. The first is an assertion that the author accepts as true; the second describes a situation that the
author posits as contrary to that assertion.
C. The first is an assertion that the author argues against; the second is evidence presented as
contrary to the author’s argument.
D. The first is evidence that the author believes is no longer valid; the second is additional evidence
that the author uses to support his main point.
E. The first is a claim that the author believes to be invalid; the second is the author’s main point.

By Sandeep Gupta, GMAT Perfect 800 | www.top-one-percent.com 169


THE MOST AUTHORITATIVE GUIDE EVER WRITTEN ON GMAT CRITICAL REASONING

93. The president’s nominees to federal circuit courts have been judged conservative
for their stands on hot-button issues. But a review of their financial disclosure forms and
Senate questionnaires reveals that the nominees are more notable for their close ties to corporate
and economic interests, especially the energy and mining industries. Some of them were paid
lobbyists for those same interests. Further, the nominees with industry ties were
overwhelmingly appointed to circuit courts regarded as traditional battlegrounds
over litigation affecting these industries. Independent observers who follow the federal
bench believe that the extensive corporate involvement among so many of the nominees is
unprecedented.
In the argument above, the two portions in boldface play which of the following roles?
A. The first is a generalization that the author aims to attack; the second is that attack.
B. The first is a pattern that the author acknowledges as true; the second is the author’s conclusion
based on that acknowledgment.
C. The first is a phenomenon that the author accepts as true; the second is evidence in support of
the author’s conclusion.
D. The first is the author’s position based on the evidence cited; the second is a pattern presented
in support of that position.
E. The first is an exception to a rule introduced in the argument; the second provides the reasoning
behind the exception.

By Sandeep Gupta, GMAT Perfect 800 | www.top-one-percent.com 170


THE MOST AUTHORITATIVE GUIDE EVER WRITTEN ON GMAT CRITICAL REASONING

94. If interest rates remain at their current high levels, many people who currently rent
their residences will hesitate to purchase homes. As the price of real estate continues to
climb, the costs of a mortgage will be too burdensome. Sellers will be forced to lower their asking
prices. So, high interest rates will eventually cause the real estate market to stabilize.
In the argument above, the portions in boldface play which of the following roles?
A. The first is the author’s main point; the second is a prediction that follows from that point.
B. The first is a consideration that the author believes will result in a certain situation; the second
is that situation.
C. The first is a consideration that weighs against the author’s main point; the second is the author’s
main point.
D. The first is a prediction; the second is evidence in support of that prediction.
E. The first is the author’s main point; the second is evidence used to argue against that point.

By Sandeep Gupta, GMAT Perfect 800 | www.top-one-percent.com 171


THE MOST AUTHORITATIVE GUIDE EVER WRITTEN ON GMAT CRITICAL REASONING

95. Corporate Strategist: It is generally true that a reduction in the price of a good results in an
increase in the demand for this product, leading to higher sales. However, I believe that the
management’s strategy of stimulating the sales of our luxury cars by implementing
a series of aggressive price reductions is seriously flawed. Dramatic price reductions
on our luxury cars will erode the image of exclusivity and premium quality associated with these
vehicles. If our cars become substantially cheaper, they will no longer represent the
symbol of status and financial success, thus losing their main appeal to our
customers.
Which of the following statements best describes the role of each portion in boldface in the
argument above?
The first represents the main position of the corporate strategist; the second acknowledges a
consideration that weighs against that position.
The first is an assumption made by the corporate strategist about the efficacy of the
management’s strategy; the second is evidence that supports the strategist’s reasoning.
The first is evidence supporting the main position of the corporate strategist; the second is that
position.
The first is evidence supporting the position of the corporate strategist; the second is a
generalization that will not hold in the case at issue.
The first is the main position of the corporate strategist; the second is evidence in support of that
position.

By Sandeep Gupta, GMAT Perfect 800 | www.top-one-percent.com 172


THE MOST AUTHORITATIVE GUIDE EVER WRITTEN ON GMAT CRITICAL REASONING

96. The fight against the drug trade in Country X should focus for the time being on
tightening the country’s borders and targeting its major smugglers. Wiping out poppy
fields in rural areas means even greater hardship for an economically depressed farming
population. Rather, the United Nations and the government of Country X must
carefully rebuild agricultural infrastructure in areas where the economy depends
on these poppy fields.
What purpose do the two boldface sentences serve in the passage?
A. The first is the conclusion drawn by the speaker; the second is the alternative to that conclusion.
B. The first is a short-term solution to a problem; the second is a long-term solution to the same
problem.
C. The first presents a problem; the second poses an ideal solution to the problem.
D. The first presents a popular solution to a problem; the second presents a solution preferred by
the author.
E. The first presents an argument; the second presents evidence to support the argument.

By Sandeep Gupta, GMAT Perfect 800 | www.top-one-percent.com 173


THE MOST AUTHORITATIVE GUIDE EVER WRITTEN ON GMAT CRITICAL REASONING

97. Mayor: Some of my critics claim that the city’s current budget deficit has been caused by my
policies, and that I am responsible for the deficit. Although I admit that the city has run a
budget deficit during my tenure, I do not agree that I am at fault for this problem. The
economic policies of the prior administration caused the current deficit, and were it not for
the economic policies of my administration, the current deficit would be even
worse.
In the mayor’s argument, the two boldface portions play which of the following roles?
A. The first is a premise that has been used against the mayor; the second supports the critics of the
mayor.
B. The first is a statement accepted by the mayor; the second is a consequence of the critics’ claims.
C. The first is a fact that the mayor believes does not contradict his conclusion; the second offers
support in consideration of that conclusion.
D. The first is evidence of unlawful activity by the mayor; the second is evidence offered by the
mayor to explain that activity.
E. The first is evidence that undermines the mayor’s main position; the second is a statement that
follows from that position.

By Sandeep Gupta, GMAT Perfect 800 | www.top-one-percent.com 174


THE MOST AUTHORITATIVE GUIDE EVER WRITTEN ON GMAT CRITICAL REASONING

98. Seemingly inconsequential changes in sea temperature due to global warming eventually result
in declines in fish and seabird populations. A rise of just two degrees prevents the vertical mixing
of seawater from different strata. This restricts the availability of upwelling nutrients to
phytoplankton. Since zooplankton, which feed upon phytoplankton, feed the rest of the
food chain, the declines are inevitable.
The boldface portion plays which one of the following roles in the argument?
A. It is a hypothesis supported by the fact that phytoplankton feed on upwelling nutrients.
B. It is intended to provide an example of the ways in which the vertical mixing of seawater affects
feeding habits.
C. It helps show how global temperature changes affect larger sea animals directly.
D. It is offered as one reason that global warming must be curtailed.
E. It is offered in support of the idea that global warming poses a threat to all organisms.

By Sandeep Gupta, GMAT Perfect 800 | www.top-one-percent.com 175


THE MOST AUTHORITATIVE GUIDE EVER WRITTEN ON GMAT CRITICAL REASONING

99. Pedigreed dogs, including those officially classified as working dogs, must conform to standards
set by organizations that issue pedigrees. Those standards generally specify the physical
appearance necessary for a dog to be recognized as belonging to a breed but stipulate nothing
about other genetic traits, such as those that enable breeds originally developed as working dogs
to perform the work for which they were developed. Since dog breeders try to maintain only those
traits specified by pedigree organizations, and traits that breeders do not try to maintain risk
being lost, certain traits like herding ability risk being lost among pedigreed dogs.
Therefore, pedigree organizations should set standards requiring working ability in pedigreed
dogs classified as working dogs.
The boldface portion plays which one of the following roles in the argument?
A. It is a claim on which the argument depends but for which no support is given.
B. It is a subsidiary conclusion used in support of the main conclusion.
C. It acknowledges a possible objection to the proposal put forth in the argument.
D. It summarizes the position that the argument as a whole is directed toward discrediting.
E. It provides evidence necessary to support a claim stated earlier in the argument.

By Sandeep Gupta, GMAT Perfect 800 | www.top-one-percent.com 176


THE MOST AUTHORITATIVE GUIDE EVER WRITTEN ON GMAT CRITICAL REASONING

100. Professor Jones has claimed that chemical compound chlorocetin contained in industrial
waste and previously considered harmless is in fact very dangerous. Jones has examined several
areas with high concentration of chlorocetin and found that certain vital biological
processes, such as photosynthesis, are slower in these areas than is usual for the
inspected species. The professor says that although he failed to establish an exact mechanism
by which chlorocetin hampers photosynthesis, his findings are sufficient to state that chlorocetin
is a dangerous chemical affecting natural world. This conclusion, however, is unwarranted
because all that Professor Jones actually established is a mere correlation between
the level of chlorocetin and the speed of photosynthesis – correlation that might not
reflect any causal relationship between the factors.
The highlighted portions of the text perform which of the following functions in the argument?
A. The first is the position the author disagrees with while the second is the author’s own position
B. The first contains the argument the author critiques while the second identifies a logical flaw in
this argument
C. The first is an evidence in the argument the author disputes while the second is a statement
supporting the author’s own position
D. The first describes a phenomenon which weighs against the author’s opinion while the second
reasons in favor of this opinion
E. The first contains information that, if true, could overthrow the author’s argument while the
second discredits this information

Explanations on the following pages

By Sandeep Gupta, GMAT Perfect 800 | www.top-one-percent.com 177


THE MOST AUTHORITATIVE GUIDE EVER WRITTEN ON GMAT CRITICAL REASONING

11. B
Delta switched from technologies using fossil fuels to ones using electricity. It has been asked whether
this results in less fossil fuel used per level of output. The answer is that it does.
What roles do the two boldfaced portions play in the argument? The first boldfaced statement is simply
asserted by the passage. But the second boldfaced statement, when it is first introduced, is not asserted
to be true, but rather is identified as something that might be inferred from the first statement. By the
end of the passage the argument concludes that the second statement is true.
A. This option simply reverses the roles that the statements play in the argument.
B. Correct. This option identifies the roles the boldfaced portions play.
C. Nothing in the passage is intended to support the first statement; and the second statement is
not supposed to call the first into question.
D. This correctly identifies the role of the first statement, but the second boldfaced portion does not
call the argument's conclusion into question—it is part of a sentence that refers to the question
whether that conclusion can be drawn from the first statement.
E. Again, this is only half right. The second boldfaced portion is not offered as support for the
conclusion; if it were offered as such support, the argument would be guilty of circular reasoning,
since the second boldfaced portion states exactly what the argument concludes.

Delta Products Inc. has recently switched at least partly from older technologies using fossil fuels
to new technologies powered by electricity.
The author provides an observation to set the context of his argument.
She talks about a company called Delta Products Inc. Until recently it had been using older
technologies employing fossil fuels.
Now the company has changed at least partly to using new technologies powered by electricity.
The question has been raised whether it can be concluded that for a given level of output Delta's
operation now causes less fossil fuel to be consumed than it did formerly.
The author brings forth a question that has been raised on the outcome of the company’s decision to
switch.
The question is whether the switch has led to reduction in the consumption of fossil fuels for a certain
level of production as compared to before the switch.
The answer, clearly, is yes.
The author makes her conclusion while answering the question.
She says that the switch has indeed led to a decrease in the amount of fossil fuels used for a certain
level of production.
Since the amount of fossil fuel used to generate the electricity needed to power the new technologies is
less than the amount needed to power the older technologies, provided level of output is held constant
Here the author provides the reason behind his affirmative answer.
She says the new technologies need a lower amount of fossil fuels to generate electricity for them.
On the other hand, the older technologies needed a higher amount of fossil fuels to produce a certain
level of output
The entire passage goes in the same direction.

1st Bold Face


Role = Observation/evidence
Relationship = Used to set the context for the argument.
2nd Bold Face
Role = question raised to reach the conclusion.
Relationship =seeks to judge the prudence of the move in the observation that sets the context.
Overall
Both go hand in hand in the same direction.

By Sandeep Gupta, GMAT Perfect 800 | www.top-one-percent.com 178


THE MOST AUTHORITATIVE GUIDE EVER WRITTEN ON GMAT CRITICAL REASONING

A.
The first identifies the content of the conclusion of the argument
No. This is just an observation to set the context of the conclusion;
The second provides support for that conclusion.
No. It is the explanation of the conclusion.
Thus, this is not the correct choice.
B
The first provides support for the conclusion of the argument;
Yes. It sets the context and, in that sense, provides the premise for the conclusion.
the second identifies the content of that conclusion.
Yes. It raises the question, the answer to which becomes the conclusion. So, it definitely calls up the
content of the conclusion.
Thus, this is the correct answer choice.
C
The first states the conclusion of the argument;
No. This is not the conclusion of the argument
The second calls that conclusion into question.
No. It raises a question, the answer to which is the conclusion. It does not challenge the conclusion
Thus, this is not the correct choice.
D
The first provides support for the conclusion of the argument;
Yes. It sets the context and, in that sense, provides the premise for the conclusion.
The second calls that conclusion into question.
No. It raises a question, the answer to which is the conclusion. It does not challenge the conclusion
Thus, this is not the correct choice.
E
Each provides support for the conclusion of the argument
Only the first provides the premise for the conclusion. The second boldface actually builds up the
content of the conclusion. It is a question and in part it contains the answer to the question itself.
Thus, this is not the correct choice.

12. C
CONCLUSION: "real reason why scientists over forty rarely produce highly creative work is not due to
age but rather because most have spent too long in their fields"
A. The first is the position that the argument as a whole defends; the second is evidence that is advanced
as part of that defense.
- BF1 is what the argument OPPOSES. BF1 says aging makes people less creative, but the argument
enforces that too much time, REGARDLESS of age, re-enforces less creativity
B. The first and second are both claims that have been advanced in support of a position that the
argument as a whole opposes.
- wrong. BF support contrasting ideas
C. The first is an explanation that the argument challenges; the second provides evidence in support of
a competing explanation that the argument defends.
- correct as is
D. The first is an explanation that the argument challenges; the second is evidence that has been used
against an alternative explanation that the argument defends.
- BF2 is not evidence AGAINST another explanation...it is evidence SUPPORTING another explanation that
the argument defends.

By Sandeep Gupta, GMAT Perfect 800 | www.top-one-percent.com 179


THE MOST AUTHORITATIVE GUIDE EVER WRITTEN ON GMAT CRITICAL REASONING

E. The first is an explanation that the argument defends; the second is evidence that has been used to
challenge that explanation.
- exact opposite -- BF1 challenges, BF2 defends
Given than we know the argument challenges BF1 and BF2 provides evidence to support the reason for
challenging BF1, C is the correct option.

13. E
It is generally thought that the reason scientists tend to do their most creative work before age forty is
that creative capacity declines with age. Yet those scientists who do creative work after forty tend,
disproportionately, to have started their careers in science later in life. So, a better explanation is that
many scientists over forty have just been at it too long.
What roles do the two portions of the argument that are in boldface play? The argument describes a
phenomenon and what is commonly thought to explain it. Then, the first boldfaced statement
introduces evidence that suggests that there may be another explanation. After this evidence is further
developed, the argument then concludes that there is indeed a better explanation for the phenomenon;
that explanation is stated in the second boldfaced portion.
A. The accuracy of the first statement is never called into question by the argument; rather, it is
relied upon as the basis for the argument's conclusion.
B. The first statement is not an objection against the position the argument defends; instead, it is a
basis for that position.
C. The first statement is not used to support a position the argument challenges, and the second
statement is the explanation the argument supports, not the one it challenges.
D. The second statement is indeed an explanation that the argument favors; but the first statement
is not used to support a competing explanation that the argument challenges.
E. Correct. This option correctly identifies the roles played by the boldfaced portions of the
argument.

Argument: Scientists typically do their most creative work before the age of forty.

The author presents a generic observation.


She says that generally scientists put forth their most creative work, before they cross the
age of forty

It is commonly thought that this happens because aging by itself brings about a loss of creative capacity.

Here the author provides the reason behind the generic observation.
Aging leads to a reduction in the creative capacity and that is why a scientist does his most
creative work before forty

However, studies show that of scientists who produce highly creative work beyond the age of forty,
a disproportionately large number entered their field at an older age than is usual.

But there are studies that tell a different tale. It has been observed that of the scientists who
produce their most creative work after forty, a considerably large number of them started
their career as a scientist at an older age than the usual age at which scientists start their
careers.
Here the author provides a second observation to show a disagreement with the first
generic observation.
So, this part of the argument is going in the opposite direction of the generic observation

By Sandeep Gupta, GMAT Perfect 800 | www.top-one-percent.com 180


THE MOST AUTHORITATIVE GUIDE EVER WRITTEN ON GMAT CRITICAL REASONING

Since by the age of forty the large majority of scientists have been working in their field for at least
fifteen years,

The large majority of scientists will have been working in their fields for at least fifteen
years by the time they reach forty; and because of this
(Here the author begins to explain the logic behind the second observation. From here
onwards, the argument goes in the same direction as the second observation)

the studies’ finding strongly suggests that the real reason why scientists over forty rarely produce highly
creative work is

The results of the studies indicate that the actual reason why scientists older than forty
hardly ever produce highly creative work
(continuing with the logic/explanation behind the second observation)

not that they have aged but rather that scientists over forty have generally spent too long in their
field.

is not because they have grown old but because they have spent too much time in their
field.

1st Boldface-
Role = Observation/evidence
Relationship = Used to support the explanation behind that observation.
2nd Boldface-
Role = explanation behind the observation why scientists over forty rarely produce highly creative
work.
Relationship =goes in the same direction as the observation in the 1st boldface.
Overall
Both go hand in hand in the same direction.
The first is a surface observation and the second explains that observation.

A) The first is a claim,


No. It is not a claim being made by the author. It is an observation.
the accuracy of which is at issue in the argument;
No. It is not refuted in the passage.
the second is a conclusion drawn
No. It is part of an explanation
on the basis of that claim.
No. No claim is the basis of this explanation.
Thus, this is not the correct choice
B) The first is an objection that has been raised
No. It is not an objection
against a position defended in the argument;
No. It is not against the argument
the second is that position
No. It is not any position or claim. It is merely an explanation of an earlier observation
Thus, this is not the correct choice

By Sandeep Gupta, GMAT Perfect 800 | www.top-one-percent.com 181


THE MOST AUTHORITATIVE GUIDE EVER WRITTEN ON GMAT CRITICAL REASONING

C) The first is evidence that has been used to


Yes. It is an observation that has been used
support an explanation that the argument challenges;
No. It has been used to support an explanation that goes in the same direction as the evidence
the second is that explanation.
Yes. This is the explanation that is supported by the evidence in the first boldface.
Thus, this is not the correct choice.
D) The first is evidence that has been used
Yes. It is an observation that has been used
to support an explanation that the argument challenges;
No. It has been used to support an explanation that goes in the same direction as the evidence
the second is a competing explanation that the argument favors.
Yes, we can say that the second statement is a competing explanation that is favoured by the
argument.
Thus, this is not the correct choice.
E) The first provides evidence
Yes. It is an observation that has been used
to support an explanation that the argument favors;
Yes. This observation supports and goes in the same direction as the argument. It is used to explain
what the argument is trying to state.
the second is that explanation
Yes. This is the explanation of the observation that the argument uses in the first boldface
Thus, E is the correct choice.

14. E
Here, the conclusion of the argument appears in the last sentence, that “these findings suggest that the
real reason why musicians over thirty rarely make significant contributions to the musical canon is not
that they have aged but rather that they have spent too much time as musicians.” This conclusion is
drawn in contrast to the conventional explanation that age erodes creative ability, which appears in the
first boldface. The second boldface is a description of the findings that are used to support the
argument’s conclusion.
(A) The description of the first boldface is accurate, as it does present an explanation that is challenged
by the argument. However, the second boldface is not a finding that supports the challenged
explanation. Rather, the second boldface is a finding that supports a different explanation altogether,
and is the primary evidence for the challenge as opposed to the explanation itself.
(B) The description of the first boldface is accurate, as it does present an explanation that is opposed by
the argument. However, the second boldface is not an objection raised against the alternative
explanation advocated by the argument, but the primary evidence in support of the alternative
explanation.
(C) The first boldface is not a claim advanced in support of a position, but is itself a position or
explanation. Additionally, the second boldface does not serve to clarify the position proposed by the
first boldface.
(D) The first boldface is not an explanation advocated by the argument, as the argument advocates an
alternative explanation. The description of the second boldface is accurate.
(E) CORRECT. The first boldface does present an explanation that the argument challenges. The second
is a finding that provides the basis for the challenge to the conventional explanation.

By Sandeep Gupta, GMAT Perfect 800 | www.top-one-percent.com 182


THE MOST AUTHORITATIVE GUIDE EVER WRITTEN ON GMAT CRITICAL REASONING

Among C, D and E, we want to see whether they align with or contrast one another. Since "of scientists
who produce highly creative work beyond the age of forty, a disproportionately large number entered
their field at an older age than is usual" is used to support the idea that "the real reason why scientists
over forty rarely produce highly creative work is not that they have aged but rather that scientists over
forty have generally spent too long in their field" - the two bold portions align with one another (they're
making the same point) - so we can eliminate (D) and since these portions align with the author's
argument, we can also eliminate (C) - leaving (E) as our correct answer.
There are two opposing explanations: Explanation-1: loss of creative capacity | Explanation-2: they
have spent a long time in the field (and hence they do not have anything new to contribute).
BF2 is the explanation-2 mentioned above, and BF1 is a statement supporting explanation-2. Hence
option E is correct.

15. A
Top executives at a bank that has been rumored to be in financial trouble have been buying shares in
the bank. Bank depositors see this as a good sign, because they believe that it indicates that the
executives have faith in the bank. However, corporate executives sometimes do this just to dispel
rumors about a company's health.

Since this is a Boldface question, let's start by ignoring the BF and focusing on the conclusion and
argument. The conclusion is that "[the reasoning of the bank's depositor's] might well be over
optimistic." Now let's breakdown the argument (again, ignore the BF for now):

First, there were "rumors that the bank faced-impending financial collapse." These rumors made the
bank's depositors worried.

Then, it became known that "several of a bank's top executives have been buying shares in their own
bank." This news made the bank's depositors relieved (i.e., made them stop worrying).

Why were they relieved by this news? "They reason that, since top executives evidently have faith in
the bank's financial soundness, those worrisome rumors must be false." - If several of the bank's top
execs have been buying shares in their own bank, they must have faith in the bank's financial soundness.

Thus, according to the depositors, the actions of the execs shows that the rumors about financial
collapse must be false.

Why does the author believe that the depositors are being overoptimistic? Because "corporate
executives have been known to buy shares in their own company in a calculated attempt to dispel
negative rumors about the company's health"

According to the depositors, the execs actions are evidence that the execs have faith in the bank's
financial soundness. The author suggests an alternative explanation for the exec’s actions. The execs
might be buying the shares NOT because they have faith but because they want to dispel negative
rumors about the company's health. Obviously. such rumors, true or not, would be bad for the company,
so it makes sense that execs would want to dispel those rumors.

Now that we understand the argument, let's see how the boldfaced portions fit into that argument:

1) "several of a bank's top executives have been buying shares in their own bank" - The depositors took
this as evidence that the execs had faith in the bank's financial soundness. According to the author, this
evidence does NOT necessarily support the depositors' belief.

By Sandeep Gupta, GMAT Perfect 800 | www.top-one-percent.com 183


THE MOST AUTHORITATIVE GUIDE EVER WRITTEN ON GMAT CRITICAL REASONING

2) "since corporate executives have been known to buy shares in their own company in a calculated
attempt to dispel negative rumors about the company's health" - In other words, "BECAUSE there's
another possible explanation," the author questions the depositors' logic. The 2nd boldfaced portion is
the reason why the author questions their logic.

Which answer choice best describes the boldfaced portions?


A. The depositors believe that the first BF portion is evidence that the execs had faith in the bank's
financial soundness. Thus, the first describes evidence that has been taken as supporting the conclusion
of the depositors. The first half looks good.

The second BF portion explains WHY the author questions the depositors' logic. The 2nd includes an
alternative explanation for the exec’s actions. So perhaps those actions do NOT support the conclusion
of the depositors. Thus, the 2nd gives a reason for questioning whether the 1st actually supports the
depositors' conclusion. (A) looks good.

B. As described for (A), the first part looks good. But the second part is not the author's conclusion. The
author's conclusion (the contrary conclusion) is that "[the reasoning of the bank's depositor's] might
well be over optimistic." The second BF portion supports the author's conclusion, but it is not the
conclusion itself. Eliminate (B).

C. The first is evidence in support of the depositors' conclusion, not the main conclusion of the
argument. Again, the second is not the main conclusion. Eliminate (C).

D. The author does not claim that the execs are buying shares because "corporate executives have been
known to buy shares in their own company in a calculated attempt to dispel negative rumors about the
company's health." In other words, the 2nd portion is not presented as an explanation itself. Instead, it
is the reason the author questions the depositors' explanation.
Also, the author's goal is not to explain why the execs are buying shares. The author does not pretend
to know the motives behind the exec's actions. All the author wants to show is that the depositors'
reasoning might be wrong. So, the first half of (D) does not accurately describe the purpose of the
argument as a whole. Eliminate (D).

E. The author is not necessarily trying to establish that the execs were buying shares in their own
company in a calculated attempt to dispel negative rumors about the company's health. The author
simply wants to present this possibility to show that the depositors might be wrong.
Also, as described for (D), the first half does not accurately describe the purpose of the argument as a
whole. Eliminate (E).

(A) is the best option.

What is the role that the two boldfaced portions play in the argument? The first boldfaced portion states
that bank executives are buying bank shares, which the passage indicates is taken by bank depositors
to be evidence of the executives' faith in the bank. The passage then tells us what some have inferred
from this, and finally offers in the second boldfaced statement evidence that undermines this inference.

A. Correct. This option correctly identifies the roles played by the boldfaced portions.
B. This correctly describes the first statement's role, but the second statement is not offered as a
conclusion—no evidence is given for it; rather it is evidence for something else.
C. The second statement is not offered as a conclusion; no evidence is given for it.

By Sandeep Gupta, GMAT Perfect 800 | www.top-one-percent.com 184


THE MOST AUTHORITATIVE GUIDE EVER WRITTEN ON GMAT CRITICAL REASONING

D. The second statement is not itself offered as an explanation of why these bank executives are
investing in the bank; if it were, that would mean that the bank executives are doing so because
corporate executives are known to do such things in a calculated effort to dispel worries.
Furthermore, the argument does not conclude that this other explanation (which the boldfaced
portion points to) is correct, only that the one inferred by depositors may not be.
E. The argument is not so much seeking to establish an explanation of its own as it is trying to
undermine that inferred by the depositors.

16. D
BF1: top executives evidently have faith in the bank’s financial soundness
This is an intermediate conclusion that supports the conclusion” those worrisome rumors must be
false” which is the position opposed by the argument.
BF2: They might well be overoptimistic
This is the main conclusion supported by the argument.
A. The first summarizes the evidence used in the reasoning called into question by the argument →
incorrect
BF1 is the reasoning called into question by the argument and not just evidence used to support the
reasoning.
the second states the counterevidence on which the argument relies → incorrect. The second BF is the
main conclusion.
B. The first summarizes the evidence used in the reasoning called into question by the argument → same
as A
the second is an intermediate conclusion supported by the evidence → second is the main conclusion
and not intermediate.
C. The first is an intermediate conclusion that forms part of the reasoning called into question by the
argument → correct
the second is evidence that undermines the support for this intermediate
conclusion → incorrect. Second is the main conclusion that opposes the conclusion” those worrisome
rumors must be false”
D. CORRECT
The first is an intermediate conclusion that forms part of the reasoning called into question by the
argument → correct.
the second is the main conclusion of the argument → correct

'Since top executives evidently have faith in the bank’s financial soundness, those worrisome rumors
must be false.' The bolded portion along with the rest of the portion in the sentence forms an
intermediate conclusion. The highlighted portion is the actual intermediate conclusion. But, since the
other 4 options are completely wrong, we can pick option D.
E. The first is an intermediate conclusion that forms part of the reasoning called into question by the
argument → correct
the second states a further conclusion supported by this intermediate conclusion → false. BF1 in no way
supports BF2

17. C
This is a tough one - there are essentially two conclusions: the main point of view and a contrasting
point of view. S1 Plant scientists have used genetic engineering on seeds to produce crop plants that
are highly resistant to insect damage.

By Sandeep Gupta, GMAT Perfect 800 | www.top-one-percent.com 185


THE MOST AUTHORITATIVE GUIDE EVER WRITTEN ON GMAT CRITICAL REASONING

S2 Unfortunately, the seeds themselves are quite expensive, and the plants require more fertilizer and
water to grow well than normal ones.
S3 Thus, for most farmers the savings on pesticides would not compensate for the higher seed costs
and the cost of additional fertilizer.
S4 However, since consumer demand for grains, fruits, and vegetables grown without the use of
pesticides continues to rise, the use of genetically engineered seeds of this kind is likely to become
widespread.
The key thing to notice is, first, the two boldface statements are on opposite sides of the fence. Second,
the first boldface is a fact that is used to support sentence 3. The second boldface goes against sentence
3 - it can't actually go against sentence 2, since S2 is a fact, but it does go against the main point of view
given in S3. We want a choice that is consistent with the above relationships.
A. The first is not context (or background) - it is a premise used to draw a conclusion. The second is not
the main point of view but a contrasting point of view. (This one also implies the two are on the same
side of the fence and they are not!)
B. The first part is fine, but the second is not. The argument does not deny that the second boldface will
be part of the outcome - rather, it says that the contrasting viewpoint is likely to overcome the main
point of view.
C. Correct.
The first presents a development (S2) that the argument predicts will have a certain outcome (S3); the
second acknowledges a consideration (S4) that weighs against that prediction (S3).
D. The author does not actually attempt to defend either conclusion - it just presents the two. This choice
also says the two are on the same side of the fence when they're not.
E. This one says that S2 and S4 are on the same side of the fence.

18. C

Conclusion: “However, these environmentalists are probably wrong (about their assessment of
Tropicorp’s intent).” It’s not one of the bolded phrases, so… Back to the boldfaced phrases to
determine their relationship to the main conclusion: Boldface 1: Main conclusion of the argument says,
essentially, that the environmentalists are wrong in thinking that Tropicorp’s decision not to pursue
rubber tapping and, thereby, leave the forests intact INDICATES that Tropicorp is not entirely profit-
motivated. In other words, the main conclusion disagrees with the environmentalists’ assessment of
Tropicorp’s motives. So, that means that the first BF is reasoning that supports the position that the
argument opposes. Boldface 2: This is the environmentalists’ conclusion, which means that it is the
conclusion which the argument refutes. This is the easier of the two Boldface roles to work with first
so C is the only choice that offers this option so C is the answer. Return to the answer choices
and consider the first BF description in remaining choices. Once again, C is the only choice that
offers the option for which we are searching.

Environmentalist: Tropicorp not acting in their own interest; Economist: they are (acting in their
own interest).
The Goal: We need to identify the purpose of each statement, and the relationship between the two.

1. The environmentalists’ conclusion


2. The economist’s conclusion

The relationship between the two: The BF1 provides proof to environmentalists’ conclusion (BF2).
It really tests who's paying attention. Sloppy readers will conflate the economist and the
environmentalists, and chances are, if you missed this question, that’s exactly what you did too.
So let’s take stock:

By Sandeep Gupta, GMAT Perfect 800 | www.top-one-percent.com 186


THE MOST AUTHORITATIVE GUIDE EVER WRITTEN ON GMAT CRITICAL REASONING

Economist: Tropicorp, which constantly seeks profitable investment opportunities, has been buying
and clearing sections of tropical forest for cattle ranching, although pastures newly created there
become useless for grazing after just a few years. The company has not gone into rubber tapping, even
though greater profits can be made from rubber tapping, which leaves the forest intact. Thus, some
environmentalists argue that Tropicorp’s actions do not serve even its own economic interest.
However, the initial investment required for a successful rubber-tapping operation is larger than that
needed for a cattle ranch; there is a shortage of workers employable in rubber-tapping operations; and
taxes are higher on profits from rubber tapping than on profits from cattle ranching. Consequently, the
environmentalists’ conclusion is probably wrong.

A. The first supports the conclusion of the economist’s argument; the second calls that conclusion into
question.
B. The first states the conclusion of the economist’s argument; the second supports that conclusion.
C. The first supports the conclusion of the environmentalists’ argument; the second states that
conclusion. Correct.
D. The first states the conclusion of the environmentalists’ argument; the second states the
conclusion of the economist’s argument.
E. Each supports the conclusion of the economist’s argument.

The first bolded statement is a conclusion, but by whom? The environmentalists! How many options
screw up even who is even making the argument in the first bolded statement? A, B, and E.
That leaves us with C and D.

Next let’s determine who is making the argument in the second bolded statement? The
Environmentalist. Notice we can now eliminate option D?

That leaves us with the correct option: Option C.

19. B
The boreal owl range over a much larger area than owls of similar size. Scientists hypothesize they do
so because of prey scarcity. Counting the owls' prey—small mammals—in the boreal owls' habitat is
inherently difficult. This makes the scientists' hypothesis hard to confirm directly. However, it has been
found that boreal owls range widely when they inhabit regions with relatively little food for the small
mammals they prey on.

What function is served by the statement that it is scarcity of prey that leads the owls to range so widely?
What function is served by the statement that boreal owls range widely if food for their small-mammal
prey is relatively sparse in the region they inhabit? The first statement expresses a hypothesis that seeks
to explain the comparatively wide range of boreal owls. The second statement serves to provide some
indirect evidence for the scientists' hypothesis.

A. The main conclusion of the argument is that the scientists' hypothesis is not sheer speculation, i.e.,
that the scientists have based their hypothesis on some evidence that they have discovered. The first
statement presents the scientists' hypothesis. The second statement cites some evidence for the
hypothesis and is not the main conclusion of the argument.
B. Correct. As stated, the first presents an explanatory hypothesis, while the second cites some indirect
evidence for the hypothesis.
C. The second statement cites some indirect evidence for the scientists' hypothesis, not for some other
hypothesis.

By Sandeep Gupta, GMAT Perfect 800 | www.top-one-percent.com 187


THE MOST AUTHORITATIVE GUIDE EVER WRITTEN ON GMAT CRITICAL REASONING

D. The argument does not oppose the scientists' hypothesis, presented in the first statement; the
second statement cites evidence for the hypothesis, and does not cite evidence for a position the
argument opposes.
E. The second statement does not present the argument's main conclusion. The main conclusion is that
the scientists' hypothesis is not mere speculation.

If the structure is "BF1, contradiction word immediately after that, BF2", then they both are
contradictory. But the structure in this question is, BF1-another sentence contradiction word BF2.
Here BF2 is contradicted by the statement before itself and not by BF1.

We need to identify the purpose of each statement, and the relationship between the two.

Statement 1: Scientists’ hypothesis (scarcity of prey that leads the owls to range so widely)
Statement 2: Wider ranging owls in areas where food that owl’s prey eats is scarcer (lends
support to the idea that scarcer prey causes boreal owls to range over a wider area.

The relationship between the two: The second provides support for the first, which leads the
author to state that the hypothesis is plausible.

Formal Recap of Argument

Conclusion: This argument has two conclusions. The author’s conclusion is that the scientists’
hypothesis is not sheer speculation – it does have support. The scientists’ conclusion/hypothesis is
that boreal owls range over unusually large areas because of a scarcity of prey.

Evidence: Boreal owls range over unusually large areas when they live where food eaten by small
mammals is relatively sparse. Since this evidence supports the scientists, it also supports the author of
the argument.

Assumption: The scarcity of small mammal food indicates a scarcity of small mammals. It should be
noted, however, that the author does not make a definite claim; it merely says that the scientists are
not merely speculating.

Notice that D and E botch the first statement. The argument DEFINITELY does not oppose the
hypothesis. In fact, the author makes the case that the hypothesis is pretty reasonable.

Between A, B, and C, let’s see what’s going on with the back half of the options:

A: the second statement is not the conclusion. It’s proof that the author finds it promising. Eliminate it.
Now, between B, and C there’s more of a nuance, but C references some sort of alternative hypothesis.
Nowhere in this prompt do we see any indication of an alternative hypothesis. Drop option C.

Option B is correct. The first statement does provide a hypothesis; it hypothesizes an explanation for
the wide range of boreal owls: scarcity of prey. The second portion, while not definitive, does provide
some support to the hypothesis, and the author acknowledges that this proof adds credence to the
hypothesis. Since the owls range over larger areas when the food of small mammals is scarce, this
might well be because the small mammals eaten by the owls are scarce.

By Sandeep Gupta, GMAT Perfect 800 | www.top-one-percent.com 188


THE MOST AUTHORITATIVE GUIDE EVER WRITTEN ON GMAT CRITICAL REASONING

20. A
An investor has criticized a company, based on the company's recent increase in inventory and on its
not decreasing production as a result of this increase.

What roles do the two boldfaced statements play in the argument? The argument suggests that the
investor's criticism is based on a principle that increased inventory of finished products often indicates
that production is faster than it should be, given the existing demand for a company's products.
However, the argument then states that the increase in inventory at the company in question is “entirely
attributable” to existing orders of products. The argument thus suggests that the investor's criticism is
misplaced, based on a suggestion as to (1) a principle that the investor could be using to support her
argument and (2) an explanation as to why the principle does not apply to the company. The two
boldfaced portions state these respective elements.

A. Correct. The first boldfaced portion states the principle that may provide the basis of the investor's
criticism, which the argument as a whole opposes. The second boldfaced portion is a statement that,
if true, the generalization would not apply to the company in question.
B. This option correctly describes the first of the boldfaced portions. However, rather than clarifying
an aspect of the meaning of the first generalization, the second boldfaced portion indicates why the
first generalization may not apply to the company.
C. This option incorrectly describes both of the boldfaced portions. The first boldfaced portion states
a general principle that could support the position that the argument opposes. The second boldfaced
portion then criticizes the application of the principle.
D. Because the second boldfaced portion describes a fundamental premise rather than the conclusion,
the description in this option of the second boldfaced portion is incorrect.
E. If we think of an argument as a set of statements that are meant to support, or provide evidence for,
a conclusion, then, because the boldfaced statements are indeed part of the argument, they may be
seen as providing evidence for the position the argument opposes. However, a description of the
roles of the boldfaced statements in this argument would need to provide more detail, such as what
option A provides.

20 A … Ans. C

• There has been a recent rise in Burton's inventory of finished products.


• An increased inventory of finished products often indicates that production is outstripping
demand.
• Despite the increase in inventory, Burton has not slowed production.

Based on this evidence, the prominent investor claims that the company is mismanaged. After all, if
production is outstripping demand, shouldn't Burton slow production? But this is not the whole story...
• In Burton's case, the increase in inventory is entirely attributable to products that have
already been assigned to orders received from customers.
• This implies that Burton's production is NOT outstripping demand and that there is no
reason to slow production. This undermines the evidence cited by the investor (evidence
supporting the investor's position that the company is mismanaged).
• Therefore, the investor's criticism of management is clearly not justified (author's
conclusion).

Now that we understand the conclusion and the argument, let's take a look at the boldfaced portions:

By Sandeep Gupta, GMAT Perfect 800 | www.top-one-percent.com 189


THE MOST AUTHORITATIVE GUIDE EVER WRITTEN ON GMAT CRITICAL REASONING

• "the company is mismanaged" - This is the position of the prominent investor, not the
author.
• "in this case [the investor's sniping at management] is clearly not justified" -
The author concludes that the investor's claim is not justified.

A. The 1st BF portion is the position of the prominent investor. The author opposes that position, so the
first half of (A) looks good. The 2nd BF portion is not evidence. Instead, it is simply the author's
position/conclusion. By itself, this statement does not undermine support for investor's position. The
second half of (A) is inaccurate, so eliminate this one.

B. As in choice (A), the first half of (B) looks good. But, again, the 2nd BF portion is not evidence. Also,
this statement in no way supports the investor's position. Instead, the 2nd BF portion is simply the
author's conclusion (that the investor's claim is not justified). Eliminate (B).

C. As in (A) and (B), the first half of (C) looks good. The 2nd BF portion is the author's conclusion, so the
second half of (C) looks good too. Choice (C) accurately expresses the roles of the two boldfaced
portions, so keep this one.

D. The 1st BF portion, "the company is mismanaged", is the position of the prominent investor, not
evidence to support the investor's position. The evidence cited to support that position is "the
company's failure to slow production in response to a recent rise in its inventory of finished products",
and this portion is not boldfaced.
The 2nd BF portion does not undermine the evidence cited above ("the company's failure to slow
production in response to a recent rise in its inventory of finished products"). Stating that the investor's
sniping was not justified does not, by itself, undermine the evidence. Rather, the 2nd BF portion is the
conclusion of the author's argument. Eliminate (D).
E. The 1st BF portion is not evidence supporting the investor's position. Instead, it is simply the
investor's position. The second half of (E) is okay, but since the first is inaccurate, (E) must be
eliminated.

(C) is the best answer.

21. B

The answer to this problem should be choice (B). in fact, if you correctly ascertain the structure of the
argument, you can come to this conclusion just by looking at what is said about the FIRST boldface in
each of the choices, since there is actually just one choice that correctly states the role of the first
boldface: the first boldface is the principal conclusion.

The passage starts out by quoting someone else (i.e., "a prominent investor"). If a passage starts out by
quoting a different person, then the purpose is almost always ultimately to contradict that person's
argument.

If you are unsure about which of two statements is the ultimate conclusion, then use the "therefore test"

"Because X; therefore, Y"


Because "Y; therefore, X"

Whichever of these make sense is correct, and whatever follows "therefore" is the conclusion.

By Sandeep Gupta, GMAT Perfect 800 | www.top-one-percent.com 190


THE MOST AUTHORITATIVE GUIDE EVER WRITTEN ON GMAT CRITICAL REASONING

In this case:
In Burton's case the increased inventory of finished products does not indicate blah blah blah;
THEREFORE, the investor's claim is not justified"
--> this makes sense.

The investor's claim is not justified; THEREFORE, in burton's case the increased inventory of finished
products does not indicate blah blah blah"
--> this doesn't make sense.

hence, "the investor's claim is not justified" is the conclusion.

Let's review the argument, rearranging it somewhat to illustrate the logic:

• There has been a recent rise in Burton's inventory of finished products.


• An increased inventory of finished products often indicates that production is outstripping
demand.
• Despite the increase in inventory, Burton has not slowed production.

Based on this evidence, the prominent investor claims that the company is mismanaged. After all, if
production is outstripping demand, shouldn't Burton slow production? But this is not the whole
story...

• In Burton's case, the increase in inventory is entirely attributable to products that have
already been assigned to orders received from customers.
• This implies that Burton's production is NOT outstripping demand and that there is no
reason to slow production. This undermines the evidence cited by the investor (evidence
supporting the investor's position that the company is mismanaged).
• Therefore, the investor's criticism of management is clearly not justified (author's
conclusion).

22. D

Manufacturing jobs are created either within existing companies or in start-ups. Manufacturing jobs are
being created at a much slower rate this year than last year. It seems likely that the number of new
start-ups will not exceed last year's number and that the average number of manufacturing jobs per
start-up will not exceed last year's number. So fewer manufacturing jobs are likely to be created this
year than last year.

What function is served by the statement that within existing firms, new jobs have been created this
year at well below last year's record pace? What function is served by the statement that there is no
reason to think that the new companies starting up this year will create more jobs per company than
did last year's start-ups? The first statement is one of the statements used as support for the argument's
main conclusion (the prediction about this year's job creation). The second statement gives another
premise used as support for that prediction.

By Sandeep Gupta, GMAT Perfect 800 | www.top-one-percent.com 191


THE MOST AUTHORITATIVE GUIDE EVER WRITTEN ON GMAT CRITICAL REASONING

A. The argument does not challenge the claim made by the first statement; it uses the first and the
second statement as support for the argument's main conclusion, the prediction about this year's
job creation.
B. The argument does not challenge the claim made by the first statement, but uses the first and
second statements as support for the argument's main conclusion.
C. The first provides evidence in support of the main conclusion of the argument; the second is not
an objection that has been raised against the main conclusion.
D. Correct. The first provides evidence in support of the main conclusion of the argument; the
second also provides support for the main conclusion.
E. Neither the first nor the second is meant to support a position that the argument opposes; rather,
they are both meant to support the argument's main conclusion.

23. E
Manufacturing jobs are created either within existing companies or in start-ups. Manufacturing jobs are
being created at a much slower rate this year than last year. It seems likely that the number of new
start-ups will not exceed last year's number and that the average number of manufacturing jobs per
start-up will not exceed last year's number. So fewer manufacturing jobs are likely to be created this
year than last year.

What function is served by the statement that any new manufacturing job is created either within an
existing company or by the start-up of a new company? What function is served by the statement that
there is no reason to think that the new companies starting up this year will create more jobs per
company than did last year's start-ups? The first statement makes explicit a general background
assumption that there are just two ways in which manufacturing jobs are created. This assumption is
used, along with other information, to support the argument's main conclusion, i.e., the prediction about
job creation this year. The second statement gives a premise meant to help support the prediction about
this year's manufacturing-job creation.

A. The first is a general statement making explicit an assumption on which the argument's reasoning
depends, but the second is a statement affirmed as part of the argument and does not express a claim
that the argument challenges.
B. The first is a generalization that is simply stated, without any support being offered. The second is
not a conclusion and is not offered in support of the first.
C. The second is not presented as a challenge to the generalization that is given in the first statement.
The argument does not seek to establish the first statement, but merely asserts it.
D. The second is information offered in support of the argument's main conclusion rather than a
statement offered in support of a position opposed by the argument.
E. Correct. The first, stating a truism, is merely asserted and requires no support in the argument, for
which it provides a foundation; the second is a piece of information meant to support the prediction
that is the argument's main conclusion.

By Sandeep Gupta, GMAT Perfect 800 | www.top-one-percent.com 192


THE MOST AUTHORITATIVE GUIDE EVER WRITTEN ON GMAT CRITICAL REASONING

24. A
Manufacturing jobs are created either within existing companies or in start-ups. Manufacturing jobs are
being created at a much slower rate this year than last year. It seems likely that the number of new
start-ups will not exceed last year's number and that the average number of new manufacturing jobs
per start-up will not exceed last year's number. So fewer manufacturing jobs are likely to be created
this year than last year.

What function is served by the statement that any new manufacturing job is created either within an
existing company or by the start-up of a new company? What function is served by the statement that
the number of new jobs created this year will fall short of last year's record number? The first statement
makes explicit a general background assumption that manufacturing jobs are created in just two ways.
This assumption is used, along with other information, to support the argument's main conclusion. The
second statement gives the argument's main conclusion, a prediction about how this year's
manufacturing-job creation will compare with last year's.

A. Correct. The first statement states a truism that is meant to provide support for the second
statement; the second statement is the argument's main conclusion.
B. The second statement is the argument's main conclusion, not an intermediate conclusion used to
support the argument's main conclusion.
C. The second statement is the main conclusion of the argument, not a statement used as support for
the main conclusion.
D. The argument merely asserts, and does not “seek to establish,” the first statement. The first
statement is a truism that does not need to be supported with evidence.
E. The second statement is the argument's main conclusion and is not meant to present a challenge to
the first statement. The first statement serves to provide partial support for the argument's main
conclusion.

Facts/premises are considered true. Hence, it is the truth. 'Generalization' is also correct in this context.
The problem in option D is '"that the argument seeks to establish".

D. The first is a generalization that the argument seeks to establish; the second is the main conclusion
of the argument.

The problem with this option lies in the underlined part because if you look back to the stem of the
argument:

Last year a record number of new manufacturing jobs were created. Will this year bring another record?
Well, any new manufacturing job is created either within an existing company or by the start-up of a
new company. Within existing firms, new jobs have been created this year at well below last year's
record pace. At the same time, there is considerable evidence that the number of new companies
starting up will be no higher this year than it was last year and there is no reason to think that the new
companies starting up this year will create more jobs per company than did last year's start-ups. So
clearly, the number of new jobs created this year will fall short of last year's record.

The underlined portion is dedicated to establish the main conclusion of the argument i.e., the number
of new jobs created this year will fall short of last year's record. The argument is not at all trying to
establish that any new manufacturing job is created either within an existing company or by the start-
up of a new company, leaving us with option A which is the best choice among the possible options.

By Sandeep Gupta, GMAT Perfect 800 | www.top-one-percent.com 193


THE MOST AUTHORITATIVE GUIDE EVER WRITTEN ON GMAT CRITICAL REASONING

A. The first is presented as an obvious truth on which the argument is based; the second is the main
conclusion of the argument.

Option A is pretty straightforward to present the lay-out of the argument and the best option available.

25. E
BF1: any new manufacturing job is created either within an existing company or by the start-up of a
new company.
This is stated as a fact on which the rest of the argument is based.
BF2: there is no reason to think that the new companies starting up this year will create more jobs per
company than did last year’s startups.
This is used to support the main conclusion of the argument which is “the number of new jobs created
this year will fall short of last year’s record”

A. The first provides evidence in support of the main conclusion of the argument → incorrect. The first
is simple a fact on which the argument is based. It neither helps nor opposes the conclusion
the second is a that argument challenges → incorrect. The argument accepts the claim in BF2.

B. The first is a generalization that the argument seeks to establish → false. The argument already
assumes this to be true and does not try and establish/evaluate this fact

C. The first is a generalization that the argument seeks to establish → same as above

D. The first is presented as obvious truth on which the argument is based → correct
the second is a claim that has been advanced in support of a position that the argument opposes →
incorrect. The second supports a position that the argument endorses.

E. CORRECT.

26. A
Main conclusion of the Argument:
"So clearly, the number of new jobs created this year will fall short of last year’s record."
A. CORRECT.
BF1 is indeed a fact on which the argument is based.
BF2 provides evidence to support the conclusion stated above.

B. BF1 is indeed a fact on which the argument is based.


BF2 is not an objection but rather evidence that the argument accepts.

C. BF1 is indeed a fact on which the argument is based.


We know that "So clearly, the number of new jobs created this year will fall short of last year’s record."
is the main conclusion of the argument.

D. The argument is not trying to establish that any new manufacturing job is created either within an
existing company or by the start-up of a new company.
Rather, it's trying to establish that the → false number of new jobs created this year will fall short of last
year’s record.
Also, BF2 is evidence that supports the position the argument accepts.

E. BF2 does not challenge BF1. Rather, it accepts it as a fact on which the argument is based.

By Sandeep Gupta, GMAT Perfect 800 | www.top-one-percent.com 194


THE MOST AUTHORITATIVE GUIDE EVER WRITTEN ON GMAT CRITICAL REASONING

27. D

Let's understand what the consultant says, step by step.


Ace Repairs ends up having to redo a significant number of the complex repair jobs it undertakes, but
when those repairs are redone, they are invariably done right.

Here, the consultant introduces a discrepancy. Apparently, a "significant number" of repair jobs are not
done correctly the first time. However, they are "invariably done right" on the second attempt. Why is
this the case? The rest of the passage explores possible reasons for this discrepancy.

Since we have established that there is no systematic difference between the mechanics who are
assigned to do the initial repairs and those who are assigned to redo unsatisfactory jobs, we must reject
the hypothesis that mistakes made in the initial repairs are due to the mechanics' lack of competence.

In this piece of the passage, the consultant eliminates a possible explanation for the differing success
rates. It's not that crappy mechanics mess up on the first attempt and then great mechanics fix it on the
second attempt.

Rather, it is likely that complex repairs require a level of focused attention that the company's
mechanics apply consistently only to repair jobs that have not been done right on the first try.

Finally, the consultant gives us his/her own view about why there is a difference in repair rate success
in Ace Repairs. He/she thinks that the mechanics pay more attention to jobs that have to be redone,
which explains why the second attempt is so much more successful than the first attempt.

In this question, we're trying to determine which answer choice best describes the role the boldface
portions play. Understanding the structure of the passage in the way outlined above should make it
easier for us to work through the answer choices.

A. The first boldface portion describes the situation the consultant is trying to explain, it is not their
main conclusion.
So (A) is out.

B. The first boldface portion is the finding the consultant is trying to explain. It isn't providing evidence
to serve as the basis for rejecting one explanation -- that comes in the second, non-boldface sentence of
the passage.
(B) is out.

C. The truth of the first boldface is not "at issue" -- it is presented as a fact that is definitely true. The
consultant then attempts to explain why it is true.
The second boldface is the consultant's explanation for why the discrepancy occurs, not evidence that
the discrepancy occurs.
For these reasons, we can cross (C) out.

D. This looks good -- the first section does describe the striking difference between two scenarios and
contrasts them against each other. The second portion does provide the consultant's explanation for
the situation introduced in the first portion.
Let's keep (D).

By Sandeep Gupta, GMAT Perfect 800 | www.top-one-percent.com 195


THE MOST AUTHORITATIVE GUIDE EVER WRITTEN ON GMAT CRITICAL REASONING

E. The first part of (E) is the same as (D), so that bit is fine.

However, the second boldface portion is the consultant's explanation of the contrast -- so, it's not used
to challenge the consultant's explanation.

This means we can rule out (E), leaving us with (D) as our winner.

The following information is attributed to a consultant: Some complex repair jobs done by Ace Repairs
have to be redone. The repairs, when redone, are usually successful. But the mechanics who do the
initial repairs and any others who redo those repairs are, overall, competent to do the repairs
successfully.

What role in the consultant's reasoning do the boldfaced statements play? The consultant's first
sentence describes a phenomenon that could be puzzling and needs explanation. One might be inclined
to argue that the mechanics who redo the repairs are more competent than those who did the initial
repairs. But the second boldfaced statement rebuts this explanation by telling us that it has been
established that there are no systematic differences in competence. The final sentence of the
consultant's reasoning puts forward another explanation: that the redoing of a repair elicits from
mechanics a higher level of focused attention than did the performance of the initial repair.

A. The first describes a puzzling phenomenon for which the consultant seeks an explanation. It is not
presented as a conclusion, i.e., a statement that is asserted on the basis of other statements. The
second is not a statement presented in support of the first; it gives an explanation offered by the
consultant for the puzzling phenomenon described in the first boldfaced portion.
B. The first describes a puzzling phenomenon for which the consultant seeks an explanation, and it is
not offered to show that a certain explanation does not fit. The second gives the consultant's own
explanation of that finding.
C. The reasoning does not question the accuracy of the first boldfaced portion; that portion is a
description of a phenomenon that the consultant believes needs explanation. The second is not
meant as evidence to indicate that the first is true; rather, it is offered as an explanation for the
puzzling phenomenon described in the first.
D. Correct. The first boldfaced portion contrasts the success of repairs that are redone with the failure
of those repairs when they were first done. The second gives an explanation proposed by the
consultant for the difference.
E. The first contrasts the success of repairs that are redone with the failure of those repairs when they
were first done. Rather than giving evidence to challenge the consultant's explanation, the second
provides that explanation itself.

28. C
The following information is attributed to a consultant. Some complex repair jobs done by Ace Repairs
have to be redone under warranty and the repairs, when redone, are usually successful. But the
mechanics who do the initial repairs and those who redo them are, overall, equally competent to do the
repairs successfully.

By Sandeep Gupta, GMAT Perfect 800 | www.top-one-percent.com 196


THE MOST AUTHORITATIVE GUIDE EVER WRITTEN ON GMAT CRITICAL REASONING

What role in the consultant's reasoning do the boldfaced statements play? The first sentence describes
a situation that is puzzling and needs explanation. One might be inclined to argue that the mechanics
who redo the repairs are more competent that those who did the initial repairs. But the second
boldfaced statement rebuts this explanation by telling us that it has been definitely established that
there are no systematic differences in competence. The final sentence of the consultant's reasoning
offers another explanation: that the redoing of a repair elicits from mechanics a higher level of focused
attention than did the performance of the initial repair.

A. The first is an assertion made by the consultant concerning a puzzling phenomenon. It does not
attribute a denial of any claim to the consultant; so, the second does not provide a reason for a denial
made by the consultant in the first boldfaced portion.
B. The first is not an explanation, or even part of one, for a finding, but rather, a description of a puzzling
finding concerning a difference between success rates of initial repairs and those of repairs that are
redone. The first, not the second, describes the finding itself.
C. Correct. The first is a statement of a puzzling fact that the consultant seems to have found and that
needs explanation. The second provides evidence to exclude the hypothesis that the higher success
rates in redoing repairs than in the initial doing of the repairs is explainable by reference to different
levels of competence in the mechanics in each case.
D. The first is a statement of a puzzling fact that the consultant seems to believe needs explanation.
Regarding the second, first note that the explanation that the consultant offers is to be found in the
final sentence of the passage. The second boldfaced portion is part of the reasoning on which the
consultant bases the explanation, not a claim that someone else has made in opposition to the
consultant's explanation.
E. The first is an assertion by the consultant; the consultant presents it as established fact, not as a
position that the consultant seeks to establish (i.e., provide evidence for). The second does not give
evidence that helps establish the consultant's initial assertion.

29. E
A public health expert argues against increasing the urgency of public health messages by pointing out
negative effects that may arise from such an increase, as well as by questioning its efficacy.

What roles are played in the argument by the two claims in boldface? The first claim in boldface states
that increasing the urgency of public health messages may be counterproductive. After making this
claim, the public health expert mentions two specific reasons this could be so: it could irritate people
who already behave responsibly, and it could convince people that all public health messages are too
cautious. (The latter reason in the second claim in boldface). The phrase [i]n addition to indicates that
neither claim in the second sentence is intended to support or explain the other. However, since each
claim in the second sentence gives a reason to believe the claim in the first sentence, each independently
supports the first sentence as a conclusion. The word and beginning the third sentence reveals that its
intended role in the argument is the same as that of the two claims in the second sentence.

Intermediate conclusions can function as a premise, as they can strengthen or weaken the main
conclusion.

2nd Bold Face

Role = opinion of the author- intermediate conclusion


Relationship = offered in support of the main conclusion

By Sandeep Gupta, GMAT Perfect 800 | www.top-one-percent.com 197


THE MOST AUTHORITATIVE GUIDE EVER WRITTEN ON GMAT CRITICAL REASONING

C: The first is the argument's main conclusion; the second supports that conclusion and is itself a
conclusion for which support is provided. There is definitely no support for the second BF part.

E: The first is the argument’s only explicit conclusion (Yes. It is the main conclusion) the second is a
premise (Yes. It is just the author’s opinion acting as a premise to the main conclusion) supporting that
conclusion (Yes).

A. Everything stated after the first sentence is intended to help support it, so the first sentence is the
argument's main conclusion.
B. Everything stated after the first sentence is intended to help support it, so the first sentence is a
conclusion, not a premise.
C. Each of the three claims in the second and third sentences is presented as an independent reason to
accept the general claim in the first sentence. Therefore, nothing in the passage is intended to
support the second statement in boldface as a conclusion.
D. Everything stated after the first sentence is intended to help support it, so the first sentence is a
conclusion, not a premise.
E. Correct. Each of the three claims in the second and third sentences is presented as an independent
reason to accept the general claim in the first sentence. Thus, each of those claims is a premise
supporting the claim in the first sentence as the argument's only conclusion.

30. D
Reported whiplash injuries are twice as common in countries where car insurance companies pay
compensation for such injuries as they are in countries where insurance companies do not. Although
there is no objective test for whiplash, this does not mean, as some suggest, that half of the reports of
such injuries are fake. It could simply be that where insurance will not pay for such injuries, people are
less inclined to report them.

What roles do the two boldfaced portions play in the argument? The first portion tells us about the
correlation between reported cases of whiplash in countries and the willingness of insurance
companies in those countries to compensate for whiplash injuries. The argument next states that
whiplash is difficult to objectively verify. The argument then asserts that although this last fact, taken
together with the first boldfaced portion, has led some to infer that over half of the reported cases in
countries with the highest whiplash rates are spurious, such an inference is unwarranted. The second
boldfaced portion then helps to explain why such an inference is not necessarily warranted by offering
an alternative explanation.

A. The claim made in the first boldfaced portion is never disputed in the argument; at dispute is how
to account for the fact that this claim is true. The second is not the argument's conclusion.
B. In a manner of speaking, perhaps, the argument uses the first portion to support its conclusion; but
there is no indication that it has been used elsewhere to do so. In any case, the second boldfaced
portion is not the argument's conclusion.
C. The first has been used to support a conclusion that the argument rejects; the second boldfaced
portion is not the argument's conclusion.
D. Correct. This option correctly identifies the roles played in the argument by the boldfaced portions.
E. The accuracy of the first boldfaced portion is never questioned in the argument; nor is the second
intended to somehow help show that the first is accurate. Rather, the argument assumes that the
first portion is accurate.

By Sandeep Gupta, GMAT Perfect 800 | www.top-one-percent.com 198


THE MOST AUTHORITATIVE GUIDE EVER WRITTEN ON GMAT CRITICAL REASONING

Apply the THEREFORE TEST to find the conclusion:

There might be difficulty in the two statements below: one is a conclusion and one is supporting the
conclusion.

Nevertheless, these facts do not warrant the conclusion drawn by some commentators that in
the countries with the higher rates of reported whiplash injuries, half of the reported cases are
spurious.

Can be rephrased as: Commentators are wrong in reaching their conclusion that half of the reported
cases are spurious.

Clearly, in countries where automobile insurance does not include compensation for whiplash,
people often have little incentive to report whiplash injuries that they actually have suffered.

Can be rephrased as: People have little incentive to report whiplash injuries.

As per the THEREFORE test, Try applying THEREFORE in front of one statement at a time and see
which makes sense.

Case 1: Commentators are wrong in reaching their conclusion that half of the reported cases are
spurious. Therefore, People have little incentive to report whiplash injuries.

Case 2: People have little incentive to report whiplash injuries. Therefore, Commentators are wrong in
reaching their conclusion that half of the reported cases are spurious.
Clearly Case 2 makes sense. So, Commentators are wrong is the … is the conclusion.

Let’s understand the different parts of the argument:


1) In countries where automobile insurance includes compensation for whiplash injuries
sustained in automobile accidents, reports of having suffered such injuries are twice as
frequent as they are in countries where whiplash is not covered ->
This is a fact as this can be verified.
2) Presently, no objective test for whiplash exists, so it is true that spurious reports of whiplash
injuries cannot be readily identified ->
This is a claim based on the reasoning provided
3) Nevertheless, these facts do not warrant the conclusion drawn by some commentators that in the
countries with the higher rates of reported whiplash injuries, half of the reported cases are spurious.
This is main conclusion of the argument
4) Clearly, in countries where automobile insurance does not include compensation for
whiplash, people often have little incentive to report whiplash injuries that they actually have
suffered.
This is the supporting argument which supports the argument

In the argument given, the two boldfaced portions play which of the following roles?
(A) The first is a claim that the argument disputes; the second is a conclusion that has been based on
that claim.
1st BF is an observation and it is not a claim.
2nd BF is not the conclusion but a support for a conclusion
(B) The first is a claim that has been used to support a conclusion that the argument accepts;
the second is that conclusion.

By Sandeep Gupta, GMAT Perfect 800 | www.top-one-percent.com 199


THE MOST AUTHORITATIVE GUIDE EVER WRITTEN ON GMAT CRITICAL REASONING

1st BF is an observation and it is not a claim.


2nd BF is not the conclusion but a support for a conclusion
(C) The first is evidence that has been used to support a conclusion for which the argument
provides further evidence; the second is the main conclusion of the argument.
1st BF is the observation and whole argument is trying to evaluate that observation
2nd BF is not the conclusion but a support of the conclusion
(D) The first is a finding whose implications are at issue in the argument; the second is a claim
presented in order to argue against deriving certain implications from that finding.
Correct as 1st BF is a finding which is evaluated in the argument which is same as saying whose
implications are at issue in the argument
and as 2nd BF is supporting the argument by refuting the previous conclusion - "these facts do not
warrant the conclusion"
(E) The first is a finding whose accuracy is evaluated in the argument; the second is evidence
presented to establish that the finding is accurate.
1st BF is not a finding whose accuracy is evaluated. The observation is taken as true but the
conclusion is evaluated in the argument
2nd BF is not supporting the 1st BF finding. It is supporting the conclusion that "these facts do not
warrant the conclusion drawn ...

31. C
A. The first is a finding whose accuracy is evaluated in the argument → incorrect. The accuracy of BF1
is not being debated in the argument. Rather the argument discusses the reason for such behavior.
B. The first is a finding whose accuracy is evaluated in the argument- → same as A
C. CORRECT
The first is a finding whose implications are at issue in the argument → correct.
This indicates that BF1 is a finding and the implications of this finding (i.e., the reason for such behavior)
is being discussed in the argument.

the second is an intermediate conclusion that has been used to support a conclusion that the argument
criticizes → correct
D. The first is a claim that the argument disputes → incorrect. The argument does not dispute the claim.
It only disputes the conclusions drawn from it.

The second is a narrower claim that the argument accepts → We know this is incorrect.

E. The first is a claim that has been used to support a conclusion that the argument accepts. The second
is that conclusion.

This is completely incorrect for various reasons

1) BF1 does not support a conclusion. It simply states a finding which is further used to draw a
conclusion “that in the countries with higher rates of reported whiplash injuries, half of the reported
cases are spurious” which the argument rejects.
2) BF2 is a conclusion that the argument rejects.

By Sandeep Gupta, GMAT Perfect 800 | www.top-one-percent.com 200


THE MOST AUTHORITATIVE GUIDE EVER WRITTEN ON GMAT CRITICAL REASONING

32. D
This is the same as the previous one except for BF2.
BF2:
these facts do not warrant a conclusion that has been drawn by some commentators:
This is the conclusion of the argument regarding the implication that “half of the reported cases are
spurious”.
Let’s analyze just the correct option.

D. CORRECT.
The first is a finding whose implications are at issue in the argument → correct
the second is the judgment reached by the argument concerning one alleged implication → correct. That
“half of the reported cases are spurious” is one of the implications of BF1. And D is a judgement that
refutes this implication.

33. B
Country X recently had a stock-trading scandal.
What role does the statement that the scandal's discovery confirms that Country X has a strong
regulatory system play in the argument? In the sentence containing the boldface statement, the phrase,
for one thing indicates that the statement is being used to justify the claim in the preceding sentence.
Thus, the boldface statement must support that preceding sentence as a conclusion. Directly after the
boldface statement, the phrase as the following considerations show indicates that the subsequent
sentences are being used to support the boldface statement. Thus, the boldface statement is a
conclusion supported by the sentences following it, and this statement itself supports the sentence
preceding it, which must be the argument's main conclusion.

The author makes the following general statements:


"In any stock market, some fraudulent activity is inevitable"
"If a stock market is well regulated, any significant stock-trading fraud in it will very likely be
discovered."
When significant stock-trading fraud is discovered, it deters potential perpetrators and facilitates
improvement in regulatory processes.
If this general pattern is true, then "the discovery of the scandal [in Country X] confirms that Country X
has a strong regulatory system." Thus, the statement in bold can be concluded based on the ensuing
statements. This intermediate conclusion is used in support of the author's main conclusion, which is
that "Country X's recent stock-trading scandal should not diminish investors' confidence in the
country's stock market." In other words, investors should view the discovery as evidence that the
country has a well-regulated stock market. The scandals are inevitably going to happen, so if scandals
were never discovered, that would suggest that the stock market is not well-regulated, which should
worry investors more than the discovery of a single scandal should.

So, statement (B) accurately describes the function of the boldfaced portion.

A. As explained above, the boldface statement supports the claim in the preceding sentence, so it
cannot be the argument's only conclusion.
B. Correct. As explained above, the boldface statement is supported by the statements following it
and in turn is used to support the argument's main conclusion in the statement preceding it.
C. As explained above, the boldface statement cannot be the argument's main conclusion, because
it supports a further conclusion presented in the sentence preceding it.

By Sandeep Gupta, GMAT Perfect 800 | www.top-one-percent.com 201


THE MOST AUTHORITATIVE GUIDE EVER WRITTEN ON GMAT CRITICAL REASONING

D. As explained above, the sentences following the boldface statement are the explicit support
provided for it.
E. As explained above, the argument's main conclusion is stated only in the first sentence, which
precedes the boldface statement. It is not repeated anywhere in the boldface statement.

Main Conclusion - Recent scandals shouldn't diminish investor confidence.


Support Conclusion - Discovery of scandal => country has a strong regulatory system.
Premises Supporting the Support Conclusion
1) fraudulent activity is inevitable.
2) Well regulated market => fraud gets discovered, and it deters perpetrators from committing more
fraud.

A - It is the argument's only conclusion. Out! It's not the main conclusion.
B - It is the conclusion for which the argument provides support and which itself is used to support
the argument's main conclusion. The Boldface is supporting the main conclusion, and the premises are
supporting the boldface. Correct.

C -It is the argument's main conclusion and is supported by another explicitly stated conclusion for
which further support is provided. Out. It's not the main conclusion.

D - It is an assumption for which no explicit support is provided and is used to support the
argument's only conclusion. Out. An assumption is never directly stated in the argument. It is
however, an unstated fact that the author presumes to be true, to draw a conclusion.

E - It is a compound statement containing both the argument's main conclusion and an assumption
used to support that conclusion. It's not the main conclusion, and is not an assumption. It's a
statement that the author is backing up.

34. B
Companies generally charge the greatest price the market will bear when they have a product that
represents a technological advance. This is because they want to make large profits while they can. But
making large profits inspires competition. As a result, profits can be maximized by charging less than
the greatest price possible.

What logical roles do the two portions in boldface play in the argument? The first sentence of the
passage introduces a connection between technological advances and price. The second sentence
discusses a pricing strategy related to such advances and offers certain considerations that help explain
that strategy. The first boldfaced portion of the passage, which is contained in the second sentence,
presents one of these considerations. The third sentence begins with the word “But,” which suggests
that what follows—the second boldfaced section—presents a consideration that may be at least
superficially at odds with the strategy just described. The final sentence of the argument presents an
alternative strategy that is supported by the preceding discussion.

A. The course of action endorsed by the argument is described in the passage's fourth and final
sentence, not in the second boldfaced portion, which is found in the passage's third sentence.
B. Correct. The first boldfaced portion is part of an explanation of why many companies follow the
strategy of charging as much as the market will bear when they have a product representing a
technological advance. The second boldfaced portion gives a reason not to follow that strategy.
C. The second boldfaced portion does not cast doubt on an assumption used to justify a strategy, but
rather casts doubt on the strategy itself.

By Sandeep Gupta, GMAT Perfect 800 | www.top-one-percent.com 202


THE MOST AUTHORITATIVE GUIDE EVER WRITTEN ON GMAT CRITICAL REASONING

D. The first boldfaced portion is raised in support of a strategy that the argument calls into question,
not a strategy that the argument endorses.
E. The first boldfaced portion helps explain the appeal of adopting a certain strategy; it does not show
that the strategy is likely to fail. The second boldfaced portion does not explain the appeal of the
strategy, but rather calls the strategy into question.

35. C

Often, when a company comes out with an innovative product, it will price the product as high as it can
to maximize profits before the competitors quickly catch up. But this is not a good strategy because the
very high price of the new product only encourages competitors to match the technological advance
more quickly.

Which answer choice best describes the roles that the boldface portions play in the argument? This type
of item concerns only the argument's structure—the way it is intended to work, not the quality of the
argument or what might strengthen or weaken the argument. So even if a boldface portion could be
used by the argument in a certain way, all that matters is its actual intended role. The fact that
technological advances tend to be quickly surpassed serves to partly explain why many companies
charge the maximum possible price for such a product. In other words, the first boldface portion helps
explain the popularity of the strategy presented in the second boldface portion. The conclusion of the
argument, however, is that the strategy exemplified in this latter boldface portion is unwise, so the
argument as a whole opposes that strategy.

A. Although the first boldface portion could be used as part of an argument that the strategy presented
in the second boldface portion is counterproductive, that is not how it is used here. Rather, it
immediately follows the word because and serves to explain the occurrence of what is described in
the second boldface portion.
B. This is clearly wrong because the second boldface portion presents the strategy that the argument
opposes.
C. Correct. It is the only answer choice that is consistent with the analysis of the reasoning presented
above.
D. The first boldface portion is not an assumption rejected by the argument; rather, it is affirmed in the
argument.
E. The argument does not expressly claim that the first boldface portion has been used to justify the
strategy of setting the price as high as possible, although it implies that this is part of the justification
that those adopting the strategy would give. More clearly, the second boldface portion does not
describe the intended outcome of the strategy, but rather the means of bringing about that intended
outcome (maximizing profits, by means of high prices).

36. A

Premise A product that represents a clear technological advance over competing products can generally
command a high price.

Bold face Surprisingly, perhaps, the strategy to maximize overall profits from a new product is to
charge less than the maximum price the market will bear. This is what the argument proposes.

A second strategy is presented Many companies charge the maximum possible price for such a product,
because they want to make as much profit as they can and technological advances tend to be quickly
surpassed.

By Sandeep Gupta, GMAT Perfect 800 | www.top-one-percent.com 203


THE MOST AUTHORITATIVE GUIDE EVER WRITTEN ON GMAT CRITICAL REASONING

Boldface The drawback is that large profits on the new product give competitors an incentive to
quickly develop a product to match the rival product's capabilities<== this is a drawback of the
alternative strategy.

A) The first is the position that the argument advocates (CORRECT); the second presents grounds for
rejecting an alternate position (CORRECT).

B) The first is the position that the argument advocates (CORRECT); the second is an alternative
position that the argument rejects (the second is NOT the position, is a drawback of the position).

C) The first presents a strategy for achieving a certain goal (seems fine), the second presents a drawback
to that strategy (incorrect, as the drawback is of the alternative strategy, not of the one presented in the
first BF)

D) The first presents a strategy for achieving a certain goal (seems fine), the second presents grounds
for preferring a different goal (the second is not a goal).

E) The first presents a strategy that, according to the argument, is ineffective (wrong); the second
presents a way of improving the effectiveness of that strategy (of course incorrect, the second BF is not
anything like this).

37. A
Homeowners in a condominium association can buy products and services collectively. A management
company handles maintenance of condominium common areas.

What roles are played in the argument by the statement that a condominium generally offers more value
for its cost than a house because of economies of scale and by the statement that condominium owners
spend less time and money on maintenance than owners of individual homes do? In the passage, the
first sentence (the first boldface statement) is a generalization. The second sentence provides an
example of the economies of scale mentioned in the first sentence, so it helps support the first sentence
as a conclusion. In the third sentence, the word since indicates that the first clause is a premise
supporting the second clause (the second boldface statement) as a conclusion. That conclusion itself
provides another example of the economies of scale mentioned in the first sentence, so it also helps
support that first sentence as a conclusion.

A. Correct. As explained above, the first boldface statement is supported by the rest of the statements
in the argument, so it is the main conclusion. The second boldface statement supports the first, but
is itself a conclusion supported by the since clause preceding it.
B. The second and third sentences in the argument provide examples of economies of scale. These
examples are evidence supporting the first boldface statement as a conclusion.
C. Since the second boldface statement provides evidence of the economies of scale described by the
first, it supports the first as a conclusion.
D. The since clause immediately preceding the second boldface statement provides evidence that
supports it, so the second boldface statement is a conclusion.
E. Both the second and the third sentences of the argument support the first boldface statement as a
conclusion. And the since clause immediately preceding the second boldface statement supports it
as a conclusion.

By Sandeep Gupta, GMAT Perfect 800 | www.top-one-percent.com 204


THE MOST AUTHORITATIVE GUIDE EVER WRITTEN ON GMAT CRITICAL REASONING

38. D
Aroca City plans to switch the source of its public-school funding from property taxes to a new local
sales tax.

What argumentative roles do the two portions in boldface play in the passage? The first boldface portion
simply describes the city's plan. The next two sentences in the passage describe an observation some
critics have made in objecting to the plan and say that the observation is correct. But then the second
boldface portion rejects the critics' implicit conclusion that the plan will reduce school funding. The
final two sentences in the passage present reasons to accept the statement in the second boldface
portion, so they are premises supporting it as a conclusion.

A. The argument concludes that the plan is unlikely to reduce funding for the schools. The passage does
not mention the plan's goal, but presumably that goal is not to reduce school funding.
B. The second boldface portion presents the argument's conclusion, not evidence to support the
conclusion. The passage does not mention the plan's goal, but presumably that goal is not to reduce
school funding.
C. The passage does not say whether the plan is better than any other possible school funding plans.
D. Correct. The plan's likely effect on the amount of school funding is at issue in the argument, whose
conclusion is that the plan probably will not reduce that funding.
E. The second boldface portion does not criticize the plan, but rather rejects a criticism of the plan by
stating that the plan will probably not reduce school funding.

39. A
The argument accepts the point of the critics but at the same time introduces a prediction that could
overcome the reason why Critics are objecting to the plan.

Main conclusion:
Nevertheless, implementing the plan will probably not reduce the money going to Aroca’s schools.

BF1:
three percent of current retail sales falls short of the amount raised for schools by property taxes.
This is the reason for critiquing the plan.

BF2:
retail sales in Aroca City are bound to increase substantially.
This is the outcome in case the author's prediction comes true.

Prediction:
Several large retailers have selected Aroca City as the site for huge new stores, and these are certain to
draw large numbers of shoppers from neighboring municipalities, where sales are taxed at rates of six
percent and more.

A. CORRECT.
First is an objection that has been raised against a certain plan → true.
the second is a prediction that, if accurate, undermines the force of that objection → true.
In case the retail sales increase substantially it would mean that the author's prediction has come true
and would undermine the critics' objection

B. There are no alternate plans discussed. The plan is the same. The argument simply discusses whether
it will be effective or not.

By Sandeep Gupta, GMAT Perfect 800 | www.top-one-percent.com 205


THE MOST AUTHORITATIVE GUIDE EVER WRITTEN ON GMAT CRITICAL REASONING

C. the second is the main reason cited by the argument for its endorsement of the criticism → false. The
second, in fact, provides a reason against criticism.

D. The first is a claim that the argument seeks to refute → incorrect. The argument accepts the critics'
point as correct but offers further considerations to weaken it.

E. BF1 is accepted fully and is not subject to any conditions (reservations). Also, BF2 presents a
prediction for the future and not a claim that is free from conditions that BF1 was subjected to.

40. E

To achieve self-sufficiency in electricity production, the Hasarian government proposes to construct


eleven large hydroelectric power plants. But the project might not be large enough to achieve its goal.
It will take fifteen years to complete, but by then many of the existing power plants will not be able to
function at full capacity.

What logical roles in the argument do the boldfaced portions play? The first reports the proposed plan.
But it is argued that the plan might not achieve its goal. It is conceded that the amount of power
projected to be generated by the new plants might be sufficient if added to the existing power
generation capacity. But since it will take fifteen years to complete the project, some of the existing
power generation capacity will no longer be fully available.

A. The first introduces a proposed plan. But the argument is critical of that proposal and indicates that
the plan, if adopted, might ultimately NOT achieve its goal. The second does not give evidence in
support of the plan.
B. The first introduces a proposed plan. The second does not give evidence against adopting an
alternative course of action. No alternative plan is considered.
C. The first does introduce a plan that the argument evaluates, but the second does not provide
evidence to support that plan against possible alternatives. No possible alternative plan is
considered.
D. The first introduces a proposed plan for achieving a goal of energy self-sufficiency. The second gives
a claim that the argument treats as accurate.
E. Correct. The first introduces a proposed plan for achieving a goal of energy self-sufficiency. The
second provides support for the argument's evaluation of the plan. It provides information to
indicate that the planned new energy generation capacity would provide energy self-sufficiency if
existing generation capacity were added. However, the argument indicates a flaw in the plan:
Hasaria's existing power plants will have significantly reduced generation capacity in fifteen years,
the time it will take for the new plants to become operational.

41. B
A historian discusses a controversy about whether or not Leibniz developed calculus concepts and
techniques independently of Newton.

What argumentative roles do the two portions in boldface play in the passage? The first four sentences
of the passage simply provide background information. Both boldface sections are within the fifth
sentence, which reports an argument by several scholars. The key word since indicates that the first
boldface section is a premise in the scholars' argument. A second premise preceded by another since
follows in the next clause. The final clause of the fifth sentence reveals that the second boldface section
is the conclusion of the scholars' argument. In the sixth sentence, the historian expresses misgivings
about the scholars' conclusion, for reasons presented in the seventh and final sentence.

By Sandeep Gupta, GMAT Perfect 800 | www.top-one-percent.com 206


THE MOST AUTHORITATIVE GUIDE EVER WRITTEN ON GMAT CRITICAL REASONING

A. The historian does not reject the claim that Newton's book includes a presentation of Newton's
calculus concepts and techniques. Instead, the historian merely points out that Leibniz's notes do
not cover those sections of Newton's book.
B. Correct. The first boldface section is one of two premises in the scholars' argument, and the second
boldface section is that argument's conclusion. In the following sentence the historian expresses
reservations about that conclusion.
C. The historian does not defend the scholars' conclusion but rather expresses misgivings about it.
D. The second boldface section is the scholars' conclusion and does not present any evidence. Nor does
it support the historian's position that a more cautious conclusion is called for.
E. The second boldface section presents not the historian's conclusion but rather the scholars'
conclusion, about which the historian expresses misgivings.

42. B
To determine what roles the two portions in boldface play, it is useful to look first for certain “inference
indicator” words: words that indicate that what follows is a premise (words like because and since) or
a conclusion (words like thus and therefore).

Here, there is only one, therefore; it immediately precedes the second boldfaced portion. This indicates
that that portion is a conclusion. Because of this, we can effectively rule out answer choice A.

However, we must investigate the rest of the argument to determine whether this is the main
conclusion. If it is, we can rule out answer choice E as well. Alternatively, it may be an intermediate
conclusion, in which case E would be the correct answer.

To determine which sort of conclusion it is, ask whether this conclusion is used in support of another
claim in the argument. This conclusion is not, which makes it the main conclusion. This rules out answer
choice E.

To make a correct choice among options B, C, and D, we must determine the role of the first highlighted
portion.
A. This choice is incorrect because the second boldfaced portion is a conclusion drawn in the argument.
It is not, of course, a hypothesis that the first boldfaced portion is used to undermine. Furthermore,
the argument presents no data to confirm the first boldfaced portion.
B. Correct. The first boldfaced portion is not a conclusion; it is merely an assertion that is not
supported by any claims presented in the argument. This portion, along with the statement
immediately following it, are offered in support of the second boldfaced portion. This second
boldfaced portion is the argument's main conclusion.
C. The first boldfaced portion does not undermine a position that the argument is directed against. In
fact, the argument is not explicitly directed against any position. Note that the argument is
rhetorically positive, arguing for a specific position rather than against one.
D. The argument provides no evidence in support of the first boldfaced position.
E. As noted above, this cannot be the correct answer because the second boldfaced portion is in fact
the main conclusion of the argument.

By Sandeep Gupta, GMAT Perfect 800 | www.top-one-percent.com 207


THE MOST AUTHORITATIVE GUIDE EVER WRITTEN ON GMAT CRITICAL REASONING

In order to conclude that "the entire flute must surely have had more holes," we need evidence directly
pertaining to the missing pieces of that flute. However, the passage provides no such evidence.
The existence of four holes is not evidence of three more holes. What if the diatonic scale only had four
notes at this point in history? What if this particular bone flute was intentionally created with a partial
diatonic scale, not the entire scale? Without any concrete information about the missing pieces of the
flute, we don't have any evidence to back up the author's statement (or, for that matter, any other
hypothetical statement about the entire flute).

That's why it's more accurate to call that statement an opinion, and that's why choice (B) better
describes what the boldface lines are doing.

43. A

Conclusion: Therefore, the Neanderthals who made the flute probably used a diatonic musical scale.

Only Option A and C recognize the second Bold-face as the main conclusion. Rule out B, D and E.

A. CORRECT.
BF 1 is the evidence that is provided to support the conclusion that the Neanderthals made flute that
used a diatonic musical scale.
BF 2 is that conclusion.

B. WE know that BF2 states the conclusion and does not oppose it.

C. The reason that the passage is able to conclude that the Neanderthals made flute that used a diatonic
musical scale (BF 2) is because of a fragment bone flute that was found at the campsite (BF1). Therefore,
BF1 in no way undermines BF2.

D. BF 1 is not a phenomenon but a piece of evidence that is cited.


BF 2 presents the position that the argument concludes(supports)

E. BF 1 is not a phenomenon but a piece of evidence that is cited.


BF 2 does not rule out any possible explanation. Rather, it accepts that the Neanderthals may have used
a flute with diatonic musical scale.

44. E
The author concludes that raising the fine to $1,000 would have the unintended effect of increasing the
amount of litter in the picnic area. When determining the function of the two bold statements, we must
consider how they relate to this conclusion: the first bold portion weighs against the conclusion, while
the second bold portion supports the conclusion. The correct answer will represent these relationships.

(A) The “prediction” mentioned here refers to the author’s conclusion (raising the fine to $1,000 would
increase the amount of litter). This answer choice incorrectly states that the first bold portion supports
this conclusion. Also, this choice incorrectly states that the second bold statement is the prediction, or
conclusion.

(B) This choice incorrectly states that the author’s prediction, or conclusion, is consistent with the first
bold statement when in fact it predicts the exact opposite outcome. Further, this answer states that the
second bold portion weighs against the author’s conclusion when in fact it supports the conclusion.

By Sandeep Gupta, GMAT Perfect 800 | www.top-one-percent.com 208


THE MOST AUTHORITATIVE GUIDE EVER WRITTEN ON GMAT CRITICAL REASONING

(C) The second bold portion does not come as a consequence of the first. In fact, the two bold portions
are in complete contrast to one another.

(D) The second bold portion is not the main position that the author defends. The main position is that
raising the fine to $1,000 would increase the amount of litter in the picnic area.

(E) CORRECT. This answer choice correctly identifies the first bold portion as a statement of causation
that does not support the author’s claim, and the second bold statement as a line of logic that does
support this claim.

45. D
This argument presents a claim that America needs a permanent third party. This claim is the
argument's conclusion, in fact. The author then cites the view held by critics of this claim and notes their
subordinate assertions that a third party would slow the legislative process and impede governance.
Finally, the author cites European countries to contradict the opponents’ claim and their evidence.

(A) This choice is incorrect because the first boldface is not the main point of the argument. It is the
conclusion of the argument’s opponents. Furthermore, the second boldface is a premise that supports
the actual conclusion, not the claim made by the opponents.

(B) This choice is incorrect because the first boldface does not oppose the premises of the argument; it
opposes the conclusion. Furthermore, the second boldface is a premise that supports the conclusion; it
is not the conclusion.

(C) This choice is incorrect because the first boldface is the actual position held by the opponents, not a
premise for their position. This choice does correctly state that the second boldface is a premise that
argues against the position held by the opponents.

(D) CORRECT. The first is the claim of the argument’s opponents, and the second is evidence that
contradicts the opponents’ claim.

(E) This choice is incorrect because the second is not the claim that the opponents are opposing by
asserting the first boldface. They assert the first boldface to oppose the conclusion of the argument.

46. E
The author of the letter believes that the city’s proposed handgun ban will actually make the city a more
dangerous place. In support of this prediction, the author offers the second boldfaced portion of the
argument: an assertion that criminals will feel increased confidence to act since their victims will no
longer be armed. This assertion contradicts the first boldfaced portion of the argument, a relationship
between fewer guns and less violent crime cited by proponents of the ban.

(A) The first boldface portion argues that the handgun ban would lead to less violent crime, a position
that directly contradicts the author’s stated position. The second boldface portion offers support for the
author’s position, but is not the position itself.

(B) The first boldface portion is a pattern of cause and effect that the author believes not to be true in
this case. By demonstrating that violent crime will increase, the second boldface portion does offer
evidence to contradict the first boldface pattern.

By Sandeep Gupta, GMAT Perfect 800 | www.top-one-percent.com 209


THE MOST AUTHORITATIVE GUIDE EVER WRITTEN ON GMAT CRITICAL REASONING

(C) The first boldface portion does present a position that the author argues will not hold in this case.
The second boldface portion, however, is not the author’s position; instead, it offers evidence in support
of the author’s position.

(D) The first boldface portion is a prediction that the author believes to be untrue. The second boldface
portion, however, does not undermine the author’s position that the handgun ban will increase violent
crime. Instead, it supports the author’s position.

(E) CORRECT. The first boldface portion shows a direct relationship between the number of handguns
and violent crime; proponents cite this relationship to support the handgun ban, but the author refutes
this relationship. The second boldface portion provides evidence to support the author’s position that
the handgun ban will actually make the citizens of the city less safe.

47. C
The first two sentences of the argument address the currently-held theory that certain physical features
that serve only to indicate "attractiveness" have developed in order to help those males distribute their
own genes more widely. For example, a male peacock's plume of tailfeathers does not actively help him
to survive; the theory holds that it helps him to procreate by attracting females. The first boldface
portion explains to us how this theory works. The final two sentences of the argument introduce
information that contradicts this theory. In this particular species of bird, the homelier birds are better
at passing on their genes than the attractive birds. The second boldface portion provides an example of
the specific research results that contradict the theory.

(A) The first bold statement is not a conclusion; it is an explanation of how the theory works. The second
bold statement is not a contention (or conclusion); it simply presents new evidence and allows the
reader to draw a conclusion.

(B) The two bold statements address opposite sides of the argument, but this choice says that the
second statement supports the theory advanced by the first statement.

(C) CORRECT. The first bold statement explains how the theory is thought to work and the second bold
statement presents research results that contradict this theory.

(D) Nothing in the argument indicates that the long-held theory discussed in the first two sentences is
not still prevalent. Indeed, the research that contradicts the theory was only discovered "recently." In
addition, the second bold statement does not introduce a new theory; it simply presents research
results that contradict the original theory.

(E) The scientist does not completely disprove the long-held theory; she merely presents one piece of
data that does not support the theory. In addition, the second bold statement does not introduce a new
theory; it simply presents research results that contradict the original theory.

48. B
The author explains that devoted gamers traditionally dictate the design of video games. However, due
to changes in the market, the author argues that this system is no longer in the best interest of the
industry. Instead, to infuse new life into the video game market, manufacturers should simplify their
games in order to attract non-gamers into the gaming fold.

By Sandeep Gupta, GMAT Perfect 800 | www.top-one-percent.com 210


THE MOST AUTHORITATIVE GUIDE EVER WRITTEN ON GMAT CRITICAL REASONING

(A) The first boldface portion does relate a situation that the author believes to be true. The second
boldface portion, however, does not explain this situation; instead, it offers evidence to demonstrate
why this situation should not continue.

(B) CORRECT. The first boldface portion is a situation that the author believes to be true now. Due to
changes in the market, however, the author believes this situation should not continue. The second
boldface portion provides evidence to support the author’s contention that the best way to grow the
gaming market is to attract new gamers.
(C) The first boldface portion is a statement of fact that contradicts the author’s position. The second
boldface portion, however, provides evidence to support the author’s position, but it is not the position
itself.

(D) The first boldface portion is a statement of fact that contradicts the author’s position. The second
boldface portion provides evidence to support the author’s position.

(E) The first boldface portion is not a prediction; rather, it is a statement of fact (or description of a
situation) that the author believes should not hold in this case. The second statement is not an
assumption, nor does it weigh against the author's position; instead, it is a premise that provides
evidence in support of the author’s position.

49. A
The conclusion of the argument is that the city will see many of its prized industries relocate to more
convenient cities and the city's financial health will be jeopardized if the city does not make changes
soon to the transportation network. This is also the second bolded sentence. The first bolded sentence
states that most of the network was put in place at a time when the city was much smaller in both area
and population. We need to find a choice that correctly describes both of these bolded statements.

(A) CORRECT. This choice states that the first statement is an explanation of a current state of affairs.
This explanation is consistent with the passage. The answer choice goes on to describe the second
bolded statement as a prediction based on that state of affairs. This is also consistent with the passage:
the second bolded statement predicts what will happen as a result of the inadequacy of the current
transportation network.

(B) The first statement is indeed a statement of fact, but the author cites it in order to bolster his or her
claim; thus, the statement is not in opposition to the conclusion. The second statement is the conclusion
of the argument.

(C) The first statement does not "emphasize an existing problem" but rather explains that existing
problem (of an overtaxed subway). Moreover, the second statement does not "offer a proposal to solve
that problem" but rather warns of what will happen if the problem is not solved.

(D) The first statement arguably presents information that "the author suggests has been overlooked
in the situation at hand"; however, the second statement does not describe that situation, but rather
proposes a hypothetical outcome in the future.

(E) The first statement is not really "justification" (a term that implies approval on the part of the
author) but rather an "explanation"; nor does it refer to an "impending problem" but rather an existing
problem. Also, the second statement does not describe "consequences" exactly but rather "potential
consequences" if the problem is left unchecked.

By Sandeep Gupta, GMAT Perfect 800 | www.top-one-percent.com 211


THE MOST AUTHORITATIVE GUIDE EVER WRITTEN ON GMAT CRITICAL REASONING

50. B

The chief economist begins his argument by describing the usual relationship of cause and effect. Thus,
the first statement in boldface represents a generalization that the chief economist accepts as accurate.
The economist then goes on to conclude that this time, however, the usual cause-and-effect relationship
will not hold and strong figures of the GDP will cause a decrease rather than an increase in stock prices.
To support this conclusion, the economist offers evidence explaining how strong GDP figures may lead
to lower stock prices. Therefore, the second statement in boldface represents evidence that supports
the main conclusion of the economist.

(A) This answer choice correctly describes the role of the first statement but incorrectly states that the
second statement in boldface represents the conclusion of the economist rather than the evidence that
supports that conclusion. Remember, the conclusion of the economist is that strong GDP figures will
result in a decrease rather than an increase in stock prices.
(B) CORRECT. This answer choice correctly identifies the role of each of the two parts in boldface. The
first part represents a generalization that is typically accurate but will not be repeated in the case at
issue. The second portion presents evidence in support of the economist’s prediction.

(C) This answer choice correctly describes the role of the first portion but mistakenly states that the
second part in boldface follows from this generalization. The second statement in boldface presents
evidence that supports the opposite effect from that described in the first portion. Namely, the
economist claims that this time, stock prices will decrease rather than increase, as would be usual.

(D) This answer choice incorrectly states that the first portion supports rather than weighs against that
economist’s prediction. In addition, this answer choice incorrectly states that the second portion in
boldface represents the economist's prediction rather than evidence supporting it.

(E) This answer choice incorrectly states that the first statement will be repeated in the case at issue.
Remember, the economist argues that the usual pattern will not hold this time. The second statement
is correctly described as acknowledging a circumstance in which the usual pattern will not hold.

51. B
The analyst recounts a proposal by the Russian government to increase the Russian population. The
analyst then dismisses that proposal and makes a counterproposal that he or she then supports with
hypothetical scenarios. The claim that the counterproposal (to try good governance) is preferable is the
conclusion of the argument.

(A) This choice is incorrect. The first boldface is a fact that indicates the government plan has failed
before; it is not the conclusion of the argument. However, the second is a premise in support of the
argument’s proposal.

(B) CORRECT. The first is the fact that the government plan has "been tried before, to no avail," a fact
that undermines the alternative proposal made by the government. The author's proposal is to improve
the country's governance, and the second boldface supports that plan by showing one way in which
better governance might lead to a population increase.

By Sandeep Gupta, GMAT Perfect 800 | www.top-one-percent.com 212


THE MOST AUTHORITATIVE GUIDE EVER WRITTEN ON GMAT CRITICAL REASONING

(C) The first does not contradict the argument’s conclusion that improved governance will reverse the
decline in population. Rather, it undermines the other proposal presented by the government, which is
a claim that the argument does not support. The second is not the argument’s main point, but an
assertion that supports the conclusion of the argument by showing one way in which better governance
might lead to a population increase.

(D) The first is a premise that the government plan has "been tried before, to an avail," which weighs
against the preceding proposal. The second is not the proposal that the first directly supports, but an
assertion that supports the conclusion of the argument by showing one way in which better governance
might lead to a population increase.

(E) The first is not a conclusion at all, rather a factual premise that these measures "have been tried
before, to little avail." The second boldface does not oppose the first boldface, rather it is an assertion
that supports the conclusion of the argument by showing one way in which better governance might
lead to a population increase.

The conclusion: A better plan to reverse the population decline is to improve the country's governance
in both the public and the private sphere.

A) The first is the main point of the analyst's argument; the second is a premise that supports the first.
--> FIRST PART IS WRONG

B) The first is a premise that undermines an alternative to the analyst's proposal; the second is a
premise that supports the analyst's main claim.
--> CORRECT. The premise is that the previous plan was not effective. The analyst's proposal is the
population sharing in wealth and decisions. The alternative to the analyst's proposal is wide range
financial incentives, but the premise of them being tried before and being ineffective undermines the
government's plan to try again.

C) The first is a premise that contradicts the main point made by the analyst; the second is the main
point of the argument.
--> BOTH ARE WRONG

D)The first is a premise that supports a proposal; the second is that proposal.
--> SECOND IS WRONG

E) The first is a conclusion that the argument endorses; the second is a premise that opposes that
conclusion.
--> FIRST PART IS WRONG

By Sandeep Gupta, GMAT Perfect 800 | www.top-one-percent.com 213


THE MOST AUTHORITATIVE GUIDE EVER WRITTEN ON GMAT CRITICAL REASONING

52. C
The argument concludes that United Energy may be acting in a manner consistent with reaching its
financial goals. However, this conclusion must be distinguished from the assertion of the
environmentalists described in the question -that the actions taken by United Energy indicate that the
company is putting environmental concerns ahead of financial returns. The answer choices may
consider either the author's conclusion (which is considered the overall conclusion) or the conclusion
asserted by the environmentalists (which is considered the opposing opinion).

(A) The first boldface does not support the author's conclusion that United Energy may be acting in a
manner consistent with its financial goals; the second does not call the conclusion into question as much
as state its opposite.

(B) The first boldface does not state the conclusion of the argument; the second boldface does not
support the conclusion of the main argument, which is that United Energy may be acting in a manner
consistent with its financial
goals.

(C) CORRECT. The first boldface supports the environmentalists’ conclusion that United Energy is
acting in a manner that places environmental impact ahead of financial returns. The second boldface
states this conclusion.

(D) The first boldface is a relationship that does support the environmentalists' conclusion; however,
the second states this conclusion, and does not undermine it.

(E) The first boldface does not support the author's conclusion, which is that United Energy may be
acting in its financial interest; the second boldface also does not support this conclusion.

(a) Fact/Premise: United Energy recently invested in a series of large windmills. The company has not
drilled oil wells in the same area, even though, greater revenues and profits could be generated from oil
wells. (Supports Environmentalist's conclusion)
(b) Environmentalist's conclusion: United Energy has established that it places environmental impact
over financial returns.
(c) Author's conclusion: United Energy may be acting in a manner consistent with its financial goals.
(B) and (D) state that the first statement is a conclusion, so we eliminate them.
(A) and (E) state that the second boldface stuff is the "premise" which supports the conclusion or calls
it into question, so they are also wrong.
So, the only right answer is (C) The first boldface is supporting the environmentalist conclusion as
stated. The second boldface is the environmentalist conclusion.

53. D
The marketing analyst begins his argument by describing the usual relationship between the
introduction of a new product and the impact on corporate revenues. Note the use of the word
“traditionally” at the beginning of this premise, indicating that the suggested pattern of cause and effect
typically holds. Thus, the first statement in boldface represents a generalization that the market analyst
accepts as accurate. The analyst then goes on to conclude that this time, however, the usual cause-and-
effect relationship will not hold and the introduction of a new product will reduce rather than increase
the company’s profits. To support this conclusion, the analyst offers evidence explaining why the new
product launch may erode the company’s profits. Therefore, the second statement in boldface
represents evidence that supports the main conclusion of the analyst.

By Sandeep Gupta, GMAT Perfect 800 | www.top-one-percent.com 214


THE MOST AUTHORITATIVE GUIDE EVER WRITTEN ON GMAT CRITICAL REASONING

(A) This answer choice incorrectly states that the first statement will be repeated in the case at issue.
Remember, the analyst argues that the usual pattern will not hold this time. The second statement is
correctly described as acknowledging a circumstance in which the usual pattern will not hold.

(B) This answer choice correctly describes the role of the first portion but mistakenly states that the
second part in boldface follows from this generalization. The second statement in boldface presents
evidence that supports the opposite effect from that described in the first portion. Specifically, the
analyst claims that the new product launch will decrease rather than increase profits.

(C) This answer choice correctly describes the role of the first statement but incorrectly states that the
second statement represents the conclusion rather than the supporting evidence for that conclusion.

Remember, the conclusion of the analyst is that the new product launch will decrease rather than
increase profits.

(D) CORRECT. This answer choice correctly identifies the role of each of the two parts in boldface. The
first part represents the generalization that is typically accurate but will not be repeated in the case at
issue. The second portion presents evidence in support of the analyst’s prediction.

(E) This answer choice incorrectly states that the first portion supports rather than weighs against that
analyst’s prediction. In addition, this answer choice incorrectly states that the second portion in
boldface represents the analyst’s prediction rather than the evidence supporting it.

54. D
The letter writer believes that if criminal penalties for drug use are eliminated, the incidence of armed
robbery and other violent crimes will decrease. In support of that belief, the letter writer offers the
second boldface portion of the argument: an assertion that crimes are committed by drug users because
they need money to buy expensive illegal drugs, and that if drugs were legal and therefore cheaper, the
crimes would become unnecessary. The first boldface portion of the argument mentions an observed
relationship between drug use and other crimes: when drug use declines, other crimes decline as well.
This observation is counter to the letter writer’s ultimate claim.

(A) The letter writer forecasts that violent crime will decline even if drug use is decriminalized. The first
boldface portion does not offer support for that forecast, but rather evidence that violent crime
decreases when anti-drug laws are enforced. The second boldface portion is not the letter writer’s
forecast, but rather the support given for it.

(B) The first boldface portion is an observation that violent crime decreases when anti-drug laws are
enforced; that observation weighs against the letter writer’s main position, but falls short of refuting
his claim that violent crime will decrease as a result of decriminalizing drug use. The second boldface
portion is support for the letter writer’s main position, not the position itself.

(C) The first boldface portion shows a direct relationship between a decline in drug activity and a
decline in violent crime, but the letter writer does not argue that future events are predicted by this
relationship. In fact, the letter writer ultimately claims the opposite: that violent crime will decrease
when criminal penalties for drug use are eliminated, even if drug use increases as a result.

By Sandeep Gupta, GMAT Perfect 800 | www.top-one-percent.com 215


THE MOST AUTHORITATIVE GUIDE EVER WRITTEN ON GMAT CRITICAL REASONING

(D) CORRECT. The first boldface portion shows a direct relationship between a decline in drug activity
and a decline in violent crime, but the letter writer claims that violent crime will decrease when criminal
penalties for drug use are eliminated, even if drug use increases as a result. If true, the information in
the second boldface section explains why the letter writer makes that claim: that the high cost of illegal
drugs is the reason drug users commit violent crimes, so cheaper, legalized drugs will cause crime to
decline.

(E) The first boldface portion is presented by the letter writer as true. However, the second boldface is
not an inference drawn from the first boldface portion; rather, it contradicts the first boldface portion.

55. D
In the first bold statement, the agent predicts, or concludes, that occupancy rates will increase despite
decreasing population. The second bold statement describes the reasoning behind this prediction: if
potential home-buyers are discouraged by high interest rates on mortgages, they will opt to rent
instead. So, the first statement is a prediction, and the second statement supports this prediction.

(A) The second statement does not weigh against the conclusion, but rather supports it.

(B) The agent’s final prediction is made in the first bold statement, not the second bold statement.
(C) The first bold statement does not describe a pattern of cause and effect. In fact, it predicts the
disjointed relationship between population and occupancy rates: as population decreases, occupancy
rates will increase.

(D) CORRECT. This choice describes the correct relationship between the two statements: the first is
the conclusion, and the second supports this conclusion.

(E) The first statement is not an undisputed fact, but rather a prediction.

56. D
The conclusion of the policy analyst is that the experts' recommendation—to eliminate every
government agency after 10 years and create it anew—is impractical. The first bold-faced statement,
the generalization that government agencies become less effective over time, is used as evidence to
support the experts' position. The second bold-faced statement, the observation that certain
governmental agencies cannot afford even temporary upheaval, is used to defend the analyst's position.

The conclusion of the technology analyst is that the experts' recommendation is impractical. The first
bold-faced statement, the generalization that technologies become less effective over time, is used as
evidence to support the experts' position. The second bold-faced statement is used to defend the
technology analyst's position.

A) The first is evidence offered in support of an opinion that the technology analyst rejects; the second
offers information that contradicts that evidence.
The first part of this choice is correct, the second is not. The second bold-faced statement does not
contradict the premise that technologies lose their effectiveness over time.

By Sandeep Gupta, GMAT Perfect 800 | www.top-one-percent.com 216


THE MOST AUTHORITATIVE GUIDE EVER WRITTEN ON GMAT CRITICAL REASONING

B) The first is a premise that the technology analyst accepts but argues against; the second offers
evidence that supports the analyst’s position.
This choice correctly states that the second bold-faced statement offers evidence in support of the
technology analyst's position. It is true that the first bold-faced statement is a premise that the
technology analyst accepts but she does not argue against this premise; instead, the analyst argues
against the experts' position, which is based on this premise.

C) The first is a position that the technology analyst argues against; the second is the position that the
analyst defends.
The first bold-faced statement is not a position that the technology analyst argues against. The first
bold-faced statement is one that the analyst accepts as true. Furthermore, the second bold-faced
statement is not the technology analyst's position; it is the evidence offered in support of the analyst's
position.

D) The first is a generalization that the technology analyst accepts as accurate and is used as the basis
for an opinion that the analyst rejects; the second is a consideration used to defend the analyst’s
position.

Correct. The first bold-faced statement, the fact that technologies become less effective over time, is
accepted by the technology analyst as true and yet is used as evidence to support the experts' position,
which the technology analyst rejects. The second bold-faced statement is used to defend the analyst's
position.

E) The first is a generalization that the technology analyst accepts as accurate and is used as the basis
for the analyst’s position; the second offers another consideration used to defend that position.

This choice correctly states that the second bold-faced statement offers a consideration in support of
the technology analyst's position. It is also true that the first bold-faced statement is a generalization
that the technology analyst accepts as accurate but, the first bold-faced statement is not used as the
basis for the analyst's position; it is used as the basis for the experts' position.

(A) This choice correctly states that the first bold-faced statement offers evidence in support of an
opinion (the experts') that the policy analyst rejects. However, the second bold-faced statement does
not contradict the premise that government agencies lose their effectiveness over time. Instead, the
second bold-faced statement simply offers another observation—that certain government agencies
cannot afford even temporary upheaval.

(B) This choice correctly states that the second bold-faced statement offers evidence in support of the
policy analyst's position. Additionally, it is true that the first bold-faced statement is a premise that the
policy analyst accepts. However, the analyst does not argue against this premise; instead, the analyst
argues against the experts' position, which is based on this premise.

(C) The first bold-faced statement is not a position that the policy analyst argues against; in fact, the
first bold-faced statement is one that the analyst accepts as true. Moreover, the second bold-faced
statement is not the policy analyst's position; it is evidence offered in support of the analyst's position,
which is that the experts' recommendation is impractical.

By Sandeep Gupta, GMAT Perfect 800 | www.top-one-percent.com 217


THE MOST AUTHORITATIVE GUIDE EVER WRITTEN ON GMAT CRITICAL REASONING

(D) CORRECT. The first bold-faced statement, the fact that government agencies become less effective
over time, is accepted by the policy analyst as true and yet is used as evidence to support the experts'
position, which the policy analyst rejects. The second bold-faced statement, the observation that certain
government agencies cannot afford even temporary upheaval, is used to defend the analyst's position.

(E) This choice correctly states that the second bold-faced statement offers a consideration in support
of the policy analyst's position. It is also true that the first bold-faced statement is a generalization that
the policy analyst accepts as accurate. However, the first bold-faced statement is not used as the basis
for the analyst's position; it is used as the basis for the experts' position.

57. C
The conclusion, or ultimate position, of the political candidate is that older and disabled individuals
should be offered drug coverage alternatives that, in contrast to plans built around individual choice,
do not force them to gamble with their health. The first bold-faced statement is an observation that the
candidate makes about the appeal of the choice-based plans; the use of the phrase "deceptively
appealing" and the continuation of the argument makes it clear that the candidate views the appeal of
these plans as unfortunate. The second bold-faced statement, that consumers cannot predict their
future health needs, is an assertion that the candidate uses to support his ultimate position that
alternative plans should be offered.

(A) This choice incorrectly states that the candidate argues against the fact that choice plans are
deceptively appealing to numerous stakeholders. The candidate views this fact as unfortunate but one
cannot argue against a "fact." Moreover, the second bold-faced statement is not the candidate's ultimate
claim, or conclusion; instead, it is a claim used to support the candidate's ultimate conclusion that
alternative plans should be offered.

(B) This choice correctly states that the second bold-faced statement is a claim that the candidate uses
as evidence to support his ultimate position. However, the first bold-faced statement is not an
observation to which the candidate is ultimately opposed; it is his own observation that the current
prescription drug plans are "deceptively appealing." His opposition is to the drug plans themselves, but
that is not the observation made in the first statement.

(C) CORRECT. The first bold-faced statement, that coverage plans centered around choice are
deceptively appealing, is an observation that the candidate acknowledges as true but unfortunate. The
second bold-faced statement—that consumers cannot predict their future health needs—is an
assertion that the candidate makes to support his ultimate position that alternative plans should be
offered.

(D) This choice incorrectly states that the candidate argues against the observation that choice plans
are deceptively appealing to numerous stakeholders. This is the candidate's own observation; though
he does view the fact as unfortunate, one cannot argue against one's own observation. Moreover, the
second bold-faced statement is not an observation; instead, it is a claim used to support the candidate's
ultimate conclusion that alternative plans should be offered.

(E) This choice correctly states that the first bold-faced statement is an observation the candidate
makes. However, the second bold-faced statement—that consumers cannot predict their future health
needs—is not an assertion that the candidate opposes; instead, he uses this claim to support his
ultimate position.

By Sandeep Gupta, GMAT Perfect 800 | www.top-one-percent.com 218


THE MOST AUTHORITATIVE GUIDE EVER WRITTEN ON GMAT CRITICAL REASONING

58. D
Once again, our first task is to identify the conclusion. In this case, our job is made a bit easier for us
because there is only one opinion, or claim, in the argument: ambiguity inspires interpretation. What
follows―information about how different people interpret a particular statement―is fact, and therefore
cannot be the author's main point.

(A) It is used to support the argument's conclusion. We know this answer choice represents an incorrect
role.

(B) It is an illustration of the claim that we are the measure of all things. The statement "We are the
measure of all things" is used as part of the support for the conclusion. This answer states a reverse
relationship.

(C) It is compatible with either accepting or rejecting the argument's conclusion. This answer would be
tempting if we misunderstood what the conclusion of the argument was. Otherwise, it can be easily
eliminated. The part in question is the argument's conclusion.

(D) It is a view that other statements in the argument are intended to support. This is another way of
saying that the part of the argument in the question stem is the conclusion, or main point, of the
argument. This is the correct answer.

(E) It sets out a difficulty the argument is intended to solve. It is not a difficulty, and there is nothing the
argument is intended to solve.

59. A

(A) is correct. The first sentence is the conclusion of the argument. The following sentences provide
support for that conclusion. What's interesting about this particular problem is that we are asked to
define the role of one phrase within the conclusion. So, what does that phrase actually do? By saying
“To the extent that”, the author is qualifying, or setting boundaries on the conclusion. Let's look for a
choice that reflects this.

(A) is the one! The key word "limits" is another of saying "To a certain extent." To part of which
problem? The whole problem of homelessness.

(B) is certainly incorrect-this particular phrase does not say anything about the primacy of the cause.

(C) is not quite right. While the phrase certainly relates to the conclusion in an intimate way, it is not a
fact (it's not even a complete grammatical sentence) and thus cannot be called "evidence."

(D) is very close. It is part of the conclusion, but certainly does not represent the conclusion itself, for
the main conclusion is about private motive not being at fault.

(E) is not even close.

By Sandeep Gupta, GMAT Perfect 800 | www.top-one-percent.com 219


THE MOST AUTHORITATIVE GUIDE EVER WRITTEN ON GMAT CRITICAL REASONING

60. B

(B) is correct. The first and last sentences both seem to state the conclusion, and the "But" does not
really represent the same kind of logical pivot we've come to expect from it―here it's merely used to
define "activity." The sentence could begin with the word "and," a non-pivot word, and retain its
meaning. So, let's try assembling the facts to see how they lead to the conclusion.

1. many people complain about government intervention in their lives


2. they tend not to reelect inactive politicians
3. active politicians pass laws that affect people's lives

Therefore, voters often reelect politicians whose behavior they resent (political behavior does not
match rhetoric). "Often" and "resent" are rather strong, but the flow of logic generally works. If people
do not reelect inactive politicians, we might infer that they reelect active ones, and complaining about
something is roughly equivalent to resenting it.

So how does the claim in question relate to the conclusion? It supports it! In the end, this argument is
actually rather straightforward, though it may not have seemed so at the beginning.

(A) is not quite right. The conclusion does not tell us why people do not reelect inactive politicians.
(B) is spot on.
(C) is not even close.
(D) is incorrect because the claim in question is not based on the claim about peoples' complaints simply
because the two follow one another in a sentence-rather, they are both claims upon which the
conclusion is based.
(E) is too extreme.

61. D
(D) is correct. This argument follows a recognizable pattern. When we see "Some vegetarians have two
ideas," we should anticipate a pivot followed by a counter of some sort to one or both of the vegetarians'
ideas. "But" is the pivot, and begins the sentence that the question asks us about. What follows is the
conclusion based on that pivot. It helps if we recognize that a "supposition" is indicated by the word
"suppose." Suppose means "if," so the argument basically says, "if it were true that blablabla, then it
would be less clear that [vegetarian idea #2] is enough to stop eating meat."

So how is that sentence used in the argument? In a nutshell, it supports the final conclusion, which is to
say it helps to hurt vegetarian idea #2.
(A) is a tempting answer, but goes too far. Nothing was concretely disproven.
(B) is actually the reverse of what is true. The author uses the phrase to show that #2 is NOT sufficient,
or enough by itself.
(C) is too extreme. Again, nothing was concretely disproven, only made "less clear."
(D) is what we said above: "helps to hurt vegetarian idea #2." The wording of this answer is an
extremely roundabout way of saying that the example is meant to show that reason #2 is not sufficient
by itself.
(E) is too extreme. One sufficient reason is made less clear, and that's all.

By Sandeep Gupta, GMAT Perfect 800 | www.top-one-percent.com 220


THE MOST AUTHORITATIVE GUIDE EVER WRITTEN ON GMAT CRITICAL REASONING

62. E
(E) is correct. The argument begins with "Some people," so we anticipate that the author will pivot the
argument and disagree with those people. Things take an unexpected turn, however, when we run into
"Critics." At this point, we must be curious as to which side the author will finally settle on. We find out
at the very beginning of the next sentence, which begins with that all-important pivot word "But,"
leading into a criticism of the criticism! What we are left with is nothing more than the implied
conclusion that the author disagrees with the critics. Notice how the structure of this tricky argument
can be easily seen by focusing on three tiny but important pieces: some people, critics, but. Now, what
is the role of the point mentioned in the question? It's the criticism of the critics!

(A) is incorrect. If anything, the point in question supports that claim, by criticizing the critics of that
claim.
(B) is wrong for the same reasons. When we know the sides of the argument (structure), we can
eliminate choices like this without getting tangled up in heady interpretations.
(C) is on the right side of the argument, but goes too far. The point is designed merely to weaken the
criticism of that claim―a far cry from proving that claim.
(D) is tempting, but it too is on the opposite side of the argument. It is not meant to be support for the
critics, but rather a counter to the critics.
(E) is the answer. It is the only one that accurately represents the role-the point in question is meant
to counter the critics.

63. D
1) Let’s look at BF 1

“Environmental organizations want to preserve the land surrounding the Wilgrinn Wilderness Area
from residential development.”
This is the goal of the Environmental organizations.

2) Plan 1
“They plan to do this by purchasing that land from the farmers who own it. That plan is ill-conceived: if
the farmers did sell their land, they would sell it to the highest bidder, and developers would outbid any
other bidders”
This outlines why the plan to achieve the above goal will not work

3) BF2
“On the other hand, these farmers will never actually sell any of the land, provided that farming it
remains viable.”
This brings in further consideration used to support 4) i.e., the conclusion stated below.

4) Plan 2
“And that is exactly why a more sensible preservation strategy would be to assist the farmers to
modernize their farms to the extent needed to maintain viability”
This gives us the plan that is finally endorsed by the argument.

Now let’s consider the options:

A. the first presents a goal that the argument rejects as ill-conceived → false. The argument does not
reject the goal; it only rejects the plan outlined to achieve the above goal.

By Sandeep Gupta, GMAT Perfect 800 | www.top-one-percent.com 221


THE MOST AUTHORITATIVE GUIDE EVER WRITTEN ON GMAT CRITICAL REASONING

B. The first presents a goal that the argument concludes cannot be attained → false. Once again, the goal
is never stated to be unachievable. The argument, in fact, finally provides a strategy to achieve this goal.

C. The first presents a goal that the argument concludes can be attained → true
the second is a judgment disputing that conclusion → False. The second bold face does not dispute that
the goal cannot be achieved. It just provides another reason to prove that Plan 1 will not succeed.

D. CORRECT.
The first presents a goal, strategies for achieving which are being evaluated in the argument → true. A
goal is presented. Strategies for achieving the goals are evaluated and ultimately rejected.
The second is a judgment providing a basis for the argument's advocacy of a particular strategy → The
argument ultimately promotes Plan 2). and BF2 provides the reason for which Plan 2) is promoted by
the argument.
E. The first presents a goal that the argument endorses → true
the second presents a situation that the argument contends must be changed if that goal is to be met in
the foreseeable future. → false
The argument provides a plan that will cause the goal to succeed without the farmers selling their land.
i.e., without changing the situation presented in BF 2.

64. B

BF1:
the Greeks actually adopted alphabetic writing at least two centuries earlier.
Now the first thing to understand here is that BF1 is THE position that the argument seeks to establish.

BF2:
they would surely have adopted whatever convention the Phoenicians were then using with respect to
the direction of writing.
Since we also know the Phoenicians used to write either from left to right or from right to left before 8
B.C, this clearly supports BF1.
Let’s look at the options.
Notice only option A and B indicate BF1 as the position the argument is trying to establish. So, we can
eliminate the other choices. Now between A and B

A. The first is the position that the argument seeks to establish → Correct.
the second reports a discovery that has been used to support a position that the argument opposes →
Incorrect.
We know that the second BF is used to support the position (BF1) of the argument and not oppose it.

B. CORRECT. One can arrive at this answer by the virtue of elimination. But here is a quick explanation
The first is the position that the argument seeks to establish → correct.
the second presents an assumption on which the argument relies → Correct.
A negation of BF2 would cause the entire argument to crumble:
“In the process they would surely NOT have adopted whatever convention the Phoenicians were then
using with respect to the direction of writing”

By Sandeep Gupta, GMAT Perfect 800 | www.top-one-percent.com 222


THE MOST AUTHORITATIVE GUIDE EVER WRITTEN ON GMAT CRITICAL REASONING

65. B

This one is almost similar to the previous one. The only difference is BF1. Let’s analyze it:
the text of these earliest surviving Greek inscriptions sometimes runs from right to left and sometimes
from left to right.

The final position of the argument is still “the Greeks actually adopted alphabetic writing at least two
centuries earlier.” Therefore, BF1 is used as evidence to support this position. BF2 is still the assumption
on which the position of the argument relies. Check the explanation above. Therefore, the only option
that first our analysis is B.

B → Correct.

66. B

Main conclusion of the Argument: Filling our prisons with such individuals would have exactly the
opposite of the desired effect, since it would limit our ability to incarcerate younger criminals, who
commit a far greater proportion of serious crimes.

BF1: such a policy would reduce crime dramatically → this clearly refutes the main conclusion. Let’s
look at our options. Only option A and B indicate that BF1 is the conclusion that the argument seeks to
refute. Eliminate the rest

A.
The first is a conclusion that the argument as a whole seeks to refute → Correct
the second is a claim that has been advanced in support of that conclusion → Incorrect. The second BF
is the main conclusion of the argument.

B. CORRECT.

67. B

Conclusion of the main expert: The expert therefore recommends that the watch commander’s approval
should no longer be required (BF1) since the officers’ time spent obtaining approval is largely wasted.
Conclusion of the editorial: This recommendation should be rejected as dangerous. (BF2)

Clearly the conclusion of the editorial refutes the conclusion of the main expert

A. The first is a recommendation made by the editorial → incorrect

B. CORRECT.

The first is a proposal against which the editorial is directed → correct.


the second is a judgment reached by the editorial concerning that proposal → Correct

C. The first provides evidence in support of a recommendation that the editorial supports → incorrect.

D. The first is a position that the editorial challenges → correct.


the second is a judgment that was made in support of that challenged position → incorrect
the second is a judgement in opposition of the challenged position.

By Sandeep Gupta, GMAT Perfect 800 | www.top-one-percent.com 223


THE MOST AUTHORITATIVE GUIDE EVER WRITTEN ON GMAT CRITICAL REASONING

E. The first is a recommendation that the editorial questions → correct.


the second provides evidence against that recommendation → incorrect. The second BF is not evidence
against BF1 but the main conclusion itself

68. C

A. The first is a claim, the accuracy of which is disputed by the editorial --> incorrect. The editorial does
not dispute BF1 but disputes the conclusion drawn from it.

B. The first is an observation that the editorial disputes → same as A.

C. CORRECT
The first is a finding that was used in support of a proposal that the editorial opposes → correct.
The second is a judgment that was based on that finding and in turn was used to support the
proposal → correct. The second BF is a judgement based on BF1 that the argument eventually opposes.

D. The first is a finding introduced to support the main conclusion of the editorial → incorrect. The main
conclusion of the editorial is “This recommendation should be rejected as dangerous”

E. The first is a conclusion, the evidence for which the editorial evaluates → incorrect
The first is an evidence that is used to draw the conclusion “that because the officers’ time spent
obtaining approval is largely wasted, the watch commander’s approval no longer be required”

69. B

A. The first presents an obstacle to achieving a certain goal → incorrect. There are no goals being
discussed here.
The business consultant is simply talking about the pros and cons of using titles within an organization.

B. CORRECT
The first is a consideration that has led to the adoption of a certain strategy → correct.
The strategy here is shunning the use of titles.
the second presents a reason against adopting that strategy → by suggesting “use of a title can facilitate
an executive’s dealings with external businesses” this provides a reason to not shun the usage of titles.

C. The first describes a concern that the consultant dismisses as insignificant → Incorrect. The
consultant does not dismiss BF1 or BF2 but finds a solution that takes both into consideration.

D. The second BF in no way offers support for BF1.

E. The first is a belief against which evidence is offered → incorrect. The consultant does not state BF1
is untrue.

By Sandeep Gupta, GMAT Perfect 800 | www.top-one-percent.com 224


THE MOST AUTHORITATIVE GUIDE EVER WRITTEN ON GMAT CRITICAL REASONING

70. A

BF1: Some corporations shun the use of executive titles.


This explains the strategy that has been adopted by certain companies. The passage goes on to explain
why this strategy has been adopted.

BF2: use of a title can facilitate an executive’s dealings with external businesses.
This tells us the downside of adopting BF1 as a strategy
Main conclusion: Clearly, corporations should adopt the compromise of encouraging their executives to
use their corporate titles externally but not internally

A. The first describes a strategy that has been adopted to avoid a certain problem → correct.
the second presents a drawback to that strategy → correct.
B. The first describes a strategy that has been adopted to avoid a certain problem → correct.
the second is a consideration raised to call into question the effectiveness of that strategy as a means of
achieving that goal → incorrect.
We are not discussing a goal here. The second BF is simple another fact that could cause BF to be
ineffective
C. The first describes a strategy that has been adopted to avoid a certain problem → correct
the second is a consideration the consultant raises in questioning the significance of that problem →
incorrect. the consultant does not question the significance of the problem (the problem here is “use of
titles indicating position in the corporation tends to inhibit communication up and down the corporate
hierarchy”)
D. The first is part of an explanation that the consultant offers for a certain phenomenon → incorrect.
BF1 is a strategy that the consultant evaluates
E. The first describes a policy for which the consultant seeks to provide a justification → incorrect. The
consultant does not justify the strategy of shunning titles but finds a middle ground.

71. B

The boldface basically indicates that selling water with extra oxygen would not improve physical
performance.
A. irrelevant. We are not discussing the use of water.
B. CORRECT. If this were true then the main conclusion of the argument “water would be useless in
improving physical performance” would still remain true
C. irrelevant. Does not prove the ineffectiveness of SuperOxy.
D. This indicates that water with extra oxygen maybe useful. This is exactly the opposite of what
boldface indicates.
E. That maybe. This is irrelevant as long as oxygen is still a factor.

Conclusion: 'SuperOXY' water would be useless in improving physical performance.


The bold portion "the only way to get oxygen into the bloodstream so that it can be absorbed by the
muscles is through the lungs" is a premise supporting the conclusion.

So, we have to find the option that, if incorporated in the argument, will also function as a premise i.e.,
it will also support the conclusion. So, in short, we are trying to find the option that strengthens the
conclusion.

Option (A) is incorrect because it says that water lost can be replaced by tap water. It doesn't say how
or why SuperOXY is useless in improving physical performance.

By Sandeep Gupta, GMAT Perfect 800 | www.top-one-percent.com 225


THE MOST AUTHORITATIVE GUIDE EVER WRITTEN ON GMAT CRITICAL REASONING

Option (B) says that amount of oxygen is already more than what the muscles can absorb. This means
drinking SuperOXY will not improve physical performance because muscles anyway cannot absorb the
extra oxygen. This option strengthens the conclusion. Hence this is the answer.

Option (C) says that people turn in great performance without this water. But it doesn't say that this
water cannot further improve their performance.

Option (D) says that frequent physical exercise increases the body’s ability to take in and use oxygen. It
doesn't say anything about how this water does not improve performance.

Option (E) says there are other factors affecting human physical performance but doesn't say that
SuperOXY doesn't affect human physical performance.

You have to attack the main argument here. The main argument says that these waters are trying to
replenish the body's oxygen content and promote physical performance. So, you need to find an answer
choice that says that even if this water contains the oxygen, it's not going to assimilated.

72. C

The conclusion of this argument is that SuperOXY would be useless in improving physical
performance. Here's how the author reaches this conclusion:

• The amount of oxygen that muscles absorb from the bloodstream is a limiting factor in
physical performance.
• SuperOXY is water with extra oxygen.
• Muscle can't absorb more oxygen than the amount that's already in the blood of people
who are exercising.
• Therefore, SuperOXY won't do anything to improve physical performance.

The argument concludes that SuperOXY won't do anything to improve physical performance. The
bolded statement tells us the reasoning behind this conclusion. More specifically, the bolded statement
tells us that muscle can't absorb additional oxygen from the bloodstream.
This supports the conclusion by confirming that the oxygen in SuperOXY can't be absorbed by
muscles. The correct answer choice will do both of these things! Now, let's start eliminating:
A. Who cares about record performances and world-class athletes? We're trying to support an
argument about the effectiveness of SuperOXY. Choice (A) goes nowhere near that argument, so
let's eliminate it.

B. Choice (B) doesn't get specific enough to tell us anything about how muscle absorbs
oxygen and doesn't imply anything about SuperOXY. Eliminate (B).

By Sandeep Gupta, GMAT Perfect 800 | www.top-one-percent.com 226


THE MOST AUTHORITATIVE GUIDE EVER WRITTEN ON GMAT CRITICAL REASONING

C. CORRECT … (C) tells us that the only way that oxygen can be absorbed by muscles is through
the lungs. Ingesting water through your lungs would kill you! So, choice (C) effectively tells us
that SuperOXY can't be ingested in the only way that leads to more oxygen absorption. This has
nothing at all to do with the amount of oxygen that can be absorbed through blood. But it doesn't
have to, because we're asked if choice (C) would serve the same function as the bolded statement.
We're not adding (C) to the passage. Instead, we're replacing the bolded statement with (C). And
it works in exactly the same way, supporting the conclusion by confirming that the oxygen in
SuperOXY can't be absorbed by muscles. So, let's keep it around. (C) is far and away the best
choice. It might have been tough to see if we hadn't identified what exactly we were being asked
to do. But once the question stem is cleared up, the process of elimination becomes as easy as...
breathing water?

D. So what? Choice (D) says absolutely nothing about SuperOXY, and the argument doesn't care at
all about any of these other potential factors. We need a choice that supports the conclusion that
SuperOXY won't increase oxygen absorption. (D) isn't that choice, so get rid of it.

E. Choice (E) doesn't even pretend to care about oxygen! And we certainly don't care about
lost water. So, eliminate (E).

73. B
Let’s first find out which of the two boldface statements (if any) form the conclusion. The first bold-face
is followed immediately by “therefore, xxxxx”, so we know at once that it is not the main conclusion of
the argument (since it's being used to justify something else).

The second boldface on the other hand, is used to establish a position by indicating what maybe the
RESULT of the action taken by the Pro-Tect company.

Cause: Pro-Tect Insurance Company's total payout on car-theft claims has been larger than the
company can afford to sustain. Pro-Tect cannot reduce the number of car-theft policies it carries, so it
cannot protect itself against continued large payouts that way

Effect: Therefore, Pro-Tect has decided to offer a discount to holders of car-theft policies whose cars
have antitheft devices. Many policyholders will respond to the discount by installing such devices, since
the amount of the discount will within two years typically more than cover the cost of installation.

Result: Thus, because cars with antitheft devices are rarely stolen, Pro-Tect's plan is likely to reduce its
annual payouts.

Now let’s look at the options that indicate the second bold-face to be the conclusion.

A) indicates both the bold-faces are conclusions. INCORRECT.


B) CORRECT. Aptly suggests that bold-face 1 is the problem and bold-face the conclusion to that
problem
C) Indicates boldface 1 to be the conclusion.
D) the second bold-face is not a prediction but the conclusion. The prediction in the para is “Many
policyholders will respond to the discount by installing such devices, since the amount of the discount
will within two years typically more than cover the cost of installation”
E) indicates that neither of the bold-faces form the conclusion. We know this to be incorrect.

By Sandeep Gupta, GMAT Perfect 800 | www.top-one-percent.com 227


THE MOST AUTHORITATIVE GUIDE EVER WRITTEN ON GMAT CRITICAL REASONING

74. A

The goal is: "protecting itself against continued large payouts"

Strategy 1: reduce the number of car-theft policies it carries

Strategy 2: offer a discount to holders of car-theft policies whose cars have antitheft devices

So (A) fits.

(C) is incorrect. If statement 1 is viewed as a consideration to support adopting a certain strategy, what
is this "certain strategy"? This consideration actually supports the "alternative strategy" only.
The "alternative strategy" of course is "offer a discount to holders of car-theft policies whose cars have
antitheft devices". But (C) talks about two different strategies.

75. E

This is one argument that progressively advances toward the main conclusion. That eliminates answer
choices A and B immediately, because there is nothing contradictory to the argument that is presented.
The argument can be summarized as follows:
P) Cork stoppers have problems that result in wasted inventory
P) Bottlemaster plastic stoppers do not have these problems
P) Bottlemaster plastic stoppers are slightly more expensive than traditional cork stoppers
P) Cork prices are going to rise dramatically
Intermediate Conclusion) Winemakers who use cork but wish to keep production costs low will have
to reconsider plastic stoppers
P) Public's negative association with plastic wine stoppers is declining
Final Conclusion) Bottlemaster plastic stoppers will gain an increased market share
Any conclusion must be supported with reasons in the argument, and the ultimate conclusion here is
supported by the fact that the public is no longer so negative about plastic stoppers and winemakers
who currently use cork stoppers but who wish to keep production costs low are going to need to
reconsider using plastic stoppers. Any ultimate conclusion of an argument cannot serve as the "why" to
something else - it is in no way a premise. Here the intermediate conclusion (as we just saw) is also a
conclusion, because it is supported by the fact that the plastic stoppers are a good substitution for the
cork stoppers, and cork stoppers are going to rise drastically. It then itself, however, serves as a "why"
to the main conclusion, as we just saw, meaning that it cannot itself be the main conclusion - it does not
pass the "reverse why test."
That makes answer choice E the correct answer. Both C and D incorrectly cite the intermediate
conclusion as the main conclusion.

By Sandeep Gupta, GMAT Perfect 800 | www.top-one-percent.com 228


THE MOST AUTHORITATIVE GUIDE EVER WRITTEN ON GMAT CRITICAL REASONING

76. D

Main conclusion: Museums would be rash to relax those standards

A. the second is the position taken by the argument → incorrect. The main conclusion is the position
taken by the argument.

B. The first is the position taken by the argument → incorrect. The main conclusion is the position taken
by the argument.

C. The first is a judgment that has been offered in support of the position that the argument calls into
question → correct
the second is a circumstance on which that judgment is, in part based → incorrect. The second BF is
based on the judgement offered in BF and not the other way around.

D. The first is a judgment that has been offered in support of the position that the argument calls into
question → correct the second is that position → correct
E. the second is the position taken by the argument → incorrect. The argument does not endorse
relaxing standards

77. D

BF1: Conclusion of the critics: money spent over the last decade in order to reduce emissions of carbon
monoxide and of volatile organic compounds has been wasted.

Conclusion of the passage: This evidence (offered by the critics) is far from adequate.

BF2: over the last decade a substantial number of new industrial facilities that emit these pollutants
have been built.

This is offered in support of the main conclusion of the passage.

A. The first identifies a claim that the reasoning seeks to show is false → correct
the second is evidence that has been cited by others in support of that claim → incorrect.
The second is cited against the claim made in BF1

B. The first identifies a claim that the reasoning seeks to show is false → correct
the second is a position for which the reasoning seeks to provide support → incorrect. The second is the
support provided for the position

C. The first is a position that the reasoning contends is inadequately supported by the evidence →
correct
the second is a position for which the reasoning seeks to provide support → incorrect. The second is the
support provided for the position

D. CORRECT.
The first is a position that the reasoning contends is inadequately supported by the evidence → correct
the second is evidence used to support the reasoning’s contention → correct. This is evidence used to
explain why the BF1 is false.

By Sandeep Gupta, GMAT Perfect 800 | www.top-one-percent.com 229


THE MOST AUTHORITATIVE GUIDE EVER WRITTEN ON GMAT CRITICAL REASONING

E. The first is a position that the reasoning contends is inadequately supported by the evidence →
correct
the second is evidence that has been used to support that position → false. The evidence has been used
against that position

78. A.

The conclusion of this passage is "the cold probably did cause the population declines, though
indirectly". Once you figure out that's the conclusion, there are only two answer choices left in play: (B)
and (B), the only two choices that actually say that's the conclusion (the "hypothesis proposed by the
paleontologist").

Note that "a generalization" and "an explanation" are not going to represent conclusions. a "judgment"
could be a conclusion, but not in the case of choice (D), because there it's followed immediately by "...in
support of X". Also (D) is backwards. (D) states that bold-face 1 supports bold-face 2, when in fact the
argument is written in such a way that bold-face 2 supports bold-face 1. Between (A) and (B), you don't
have to think that hard. choice (A) says that the second boldface is for the conclusion, while choice (B)
says the second boldface is against the conclusion. since the former is true - the second boldface is the
rationale behind the paleontologist's hypothesis - you go with (A).

1. About 2.8 million years ago, many species that lived near the ocean floor suffered substantial
population declines. - It's a factual statement that tells us something that happened a long time ago.

2. These declines coincided with the onset of an ice age. - This is also factual. However, this fact
happened at the same time as the one in the first statement.

3. The notion that cold killed those bottom-dwelling creatures outright is misguided, however; - Now,
this is Paleo's opinion. Saying that something is misguided is not a fact, it's an opinion that tells us that
Paleo doesn't agree that cold killed those bottom dwelling creatures.

4. temperatures near the ocean floor would have changed very little. - This is a reason to back up the
opinion of Paleo. Since the temperatures changed very little, this temperature change should not have
led to the death of bottom dwelling creatures.

5. Nevertheless, the cold probably did cause the population declines, though indirectly. - This is
our first Boldface statement or BF1. This is a causative statement where the Paleo links cold to
population decline, with a keyword "indirectly". So, in a previous statement, the Paleo opined that cold
did not lead to death of bottom dwelling creatures and provided a reason for the same. In this
statement, Paleo introduces his own theory of how cold might have affected bottom dwelling
creatures. So, this statement could be called an opinion or conclusion or judgement or
hypothesis of the Paleo.

6. Many bottom-dwellers depended for food on plankton, small organisms that lived close to the
surface and sank to the bottom when they died. - This is most probably a fact because what bottom
creatures eat is not a matter of opinion, they eat what they eat. However. the presence of "many" could
make it an opinion since different people would define "many" differently. This statement, along with
the next statement (BF2) provides support to the opinion of the Paleo (BF1). This statement says that
bottom dwelling creatures depended on Plankton for food.

By Sandeep Gupta, GMAT Perfect 800 | www.top-one-percent.com 230


THE MOST AUTHORITATIVE GUIDE EVER WRITTEN ON GMAT CRITICAL REASONING

7. Most probably, the plankton suffered a severe population decline as a result of sharply lower
temperatures at the surface, depriving many bottom-dwellers of food. - This is BF2. This says
that Plankton suffered a severe population decline. Read this statement with the preceding statement,
which said Plankton was food for bottom dwelling creatures. So, combining these statements, we find
that the food of bottom dwelling creatures declined severely. This could now explain BF1 (indirect
cause of decline).

Now, let's look at the options:


A. The first introduces the hypothesis proposed by the paleontologist; the second is a judgment
offered in spelling out that hypothesis. - As we understand, BF1 can be termed a hypothesis of
the paleontologist. We know BF2 supports BF1. So, the roles of both BF1 and BF2 roughly match with
our understanding. Let's keep this option in the fray.
B. The first introduces the hypothesis proposed by the paleontologist; the second is a position that the
paleontologist opposes. - Here, the explanation of BF2 is easily wrong. Paleo doesn't oppose BF2 in
any way. Thus, this is incorrect.
C. The first is an explanation challenged by the paleontologist; the second is an explanation proposed
by the paleontologist. - Here, BF1 is not challenged by Paleo; instead, BF1 is proposed by Paleo. So,
Incorrect.
D. The first is a judgment advanced in support of a conclusion reached by the paleontologist; the
second is that conclusion. - In our analysis we found that BF2 supports BF1 and this option says just
the opposite. However, I would advise you to eliminate this option only after reading BF1 and BF2
again. But do eliminate this because BF1 is no way to explain BF2. So, this is also incorrect.
E. The first is a generalization put forward by the paleontologist; the second presents certain
exceptional cases in which that generalization does not hold. - BF2 doesn't present any exceptional
circumstances. Therefore, this is also incorrect.
So, after going through the options, we find that option A is the correct choice.

79. C

“It follows that” – indicates some kind of conclusion.

But then the argument builds on this. The lower rates of photosynthesis will force the plant to use stored
sugars; THEREFORE (another conclusion) the fruit will be less sweet.

So, the first highlighted portion is a premise (fact) that helps (in conjunction with other premises) lead
to a conclusion that ALSO served as a support for the overall argument that melons that have been
harvested after too much rain will be less sweet. Let’s see if there’s an answer that reflects this.

The first states the conclusion of the argument as a whole; the second provides support for that conclusion.

Nope. The first is NOT a conclusion because it is part of the basis of the argument. We can eliminate A.

The first provides support for the conclusion of the argument as a whole; the second provides evidence that
supports an objection to that conclusion.

The first part is a little questionable, but we could say that in supporting the conclusion in the middle
of the argument that the first highlighted portion does provide support for the overall conclusion.
HOWEVER, the middle chunk does NOT challenge the overall conclusion. We can eliminate B.

By Sandeep Gupta, GMAT Perfect 800 | www.top-one-percent.com 231


THE MOST AUTHORITATIVE GUIDE EVER WRITTEN ON GMAT CRITICAL REASONING

The first provides support for an intermediate conclusion that supports a further conclusion stated in the
argument; the second states that intermediate conclusion.

Yes, the first portion does support an intermediate conclusion—and that second portion is said
intermediate conclusion, which we know from the phrase “it follows that.” C seems like a good answer.

The first serves as an intermediate conclusion that supports a further conclusion stated in the argument;
the second states the position that the argument as a whole opposes.

Nope. The first portion is not a conclusion. We can eliminate D without reading further (also the second
part is not true either so, all around D is a dud).

The first states the position that the argument as a whole opposes; the second supports the conclusion of
the argument.

Nope. There’s no part of this argument that the rest of the argument opposes, so we can eliminate E
because it does not properly characterize the first portion.

The correct answer is C.

80. D

• Sentence 1 is a premise because it cites a fact.


• Sentence 2 is a premise because it presents an additional fact.
• The first boldface portion in sentence 3 is a fact (premise) presented as an event that may
occur. Note the phrase can lead to.
• The second boldface portion, or the last sentence, is a judgment (conclusion) supported by
sentences 2 and 3.

A. An opinion is the personal belief of the author. The first boldface, however,
presents factual information.
B. While this answer choice defines the first boldface part correctly, it defines the second
incorrectly. Because the second boldface portion is supported by the first, it is incorrect to say
that it opposes it.
C. The first boldface presents factual information. You can immediately eliminate this.
D. Correct! The first boldface portion presents us with a possible situation that is based purely on
fact. The second boldface portion concludes the argument by presenting a judgment about what
must be done to prevent the situation (injury) from occurring.
E. While this answer choice defines the first boldface part correctly, it defines the second
incorrectly. Because the second boldface is supported by the first, it is incorrect to say that it
denies its possibility to occur. If the event would never occur, then sentence 3 wouldn't be a fact.

By Sandeep Gupta, GMAT Perfect 800 | www.top-one-percent.com 232


THE MOST AUTHORITATIVE GUIDE EVER WRITTEN ON GMAT CRITICAL REASONING

81. C

In the past, there was a clear pattern: fewer restrictions --> more ads --> more consumer savings. Clear
cause and effect.

Now, when they abrogate the requirement for displaying fees, consumer prices will rise. Clearly, the
author, a consumer advocate, is concerned about this, and this prompt may well be part of a larger
argument about why we should NOT get rid of the requirement for lawyers to display their fees---that
would likely be a consumer advocate's larger agenda with respect to this issue. Clearly, this consumer
advocate is on the lookout for consumers and is bothered about the fact that, if this requirement is lifted,
the consumers will be paying more and more. Therefore, this is the "case at issue," the thing that really
concerns the author.

That's the gigantic problem with (B). The "case at issue," namely what would happen if the fee
requirement is abolished, is clearly a case in which the past pattern would no longer hold. Option (B)
mistakenly calls some other unspecified concern the "case at issue," and treats this important issue, the
author's main point, as a throwaway extra detail. It entirely misses the point of what is important to the
author.

Choice (C) gets it right: the cause-and-effect pattern discussed in the first BF statement will NOT play
out in the "case at hand." The "case at hand" that deeply concerns this author is the case in which fees
would go up, and the second BF "offers a consideration in support of" the prediction that fees could go
up. The best answer is (C).

IDENTIFY CONCLUSION: However, eliminating the state requirement that legal advertisements must
specify fees for specific services would almost certainly increase rather than further reduce consumer’s
legal costs.

> BF2 = premise supporting the Conclusion

(A) The first is a generalization that the consumer advocate accepts as true; the second is presented as
a consequence that follows from the truth of that generalization.

- BF2 is NOT a consequence...

(B) The first is a pattern of cause and effect that the consumer advocate argues will be repeated in the
case at issue; the second acknowledges a circumstance in which that pattern would not hold.
- BF2 does not acknowledge a circumstance in which a pattern (identified by BF1) does not hold.

(C) The first is pattern of cause and effect that the consumer advocate predicts will not hold in the case
at issue; the second offers a consideration in support of that prediction.

- correct as is. pattern will not hold b/c author goes on to say that eliminating requirements would
increase costs (instead of eliminating them). also, BF2 is supporting the prediction (conclusion)

(D) The first is evidence that the consumer advocate offers in support of a certain prediction; the second
is that prediction.

- BF2 is not a conclusion or prediction -- it is evidence/a premise

By Sandeep Gupta, GMAT Perfect 800 | www.top-one-percent.com 233


THE MOST AUTHORITATIVE GUIDE EVER WRITTEN ON GMAT CRITICAL REASONING

(E) The first acknowledges a consideration that weighs against the main position that the consumer
advocate defends; the second is that position.

- BF2 is not a conclusion

82. C
All cultures have different definitions of beauty. Still, there are some people that are simply more
beautiful than others. However, this beauty must always be considered along with one's personality.
The part in question is analogous to the statement "still, there are some people that are simply more
beautiful than others."

Notice two things about this statement:

1. It's a claim (an opinion)

2. According to the author, it's an opinion that must be considered alongside something else
(personality).

That's what (C) says -- it's a claim that must be considered in a specific way --

(A) is incorrect in part b/c the author doesn't try to refute the hypothesis

(B) is incorrect in part b/c it is not an objection

(D) is very tempting, but this isn't a claim that is derived from another claim.

(E) is incorrect because the statement is not used to justify evidence.

83. D
The author's conclusion is that mathematicians actually form a tightly knit community. The
counterargument is that mathematicians are antisocial loners. Now, label each statement as either
Conclusion, Premise For, or Premise Against. In the above argument, the first boldface represents an
example that supports the counterargument. Thus, the first statement is Premise Against. The second
boldface represents the author's conclusion. Now we can write down our assessment of the boldface
statements:

#1 = Premise against
#2 = Conclusion

Turning to the answer choices, we should assess each one methodically.

Evaluate the first half of choice (A) first. This says that the author uses this statement to illustrate a
social pattern. In other words, this choice asserts that statement #1 is Premise For. We have labeled the
first statement as Premise Against, so this choice is incorrect.

The first half of choice (B) says the first statement supports the popular view. The popular view is the
counterargument, so this choice argues that #1 is Premise Against. The second half of choice (B),
however, says that the second statement is the popular view. The second statement is the author's
conclusion, not the counterargument. Eliminate this 'choice.

By Sandeep Gupta, GMAT Perfect 800 | www.top-one-percent.com 234


THE MOST AUTHORITATIVE GUIDE EVER WRITTEN ON GMAT CRITICAL REASONING

The first half of choice (C) says that the first statement is an example of the counterargument (in other
words, Premise Against). The second half of choice (C) says that the second statement reiterates the
counterargument, but the second statement is the author's own conclusion. Eliminate this choice.

The first half of choice (D) says that the first statement is a counterexample to the author's conclusion
(in other words, Premise Against). The second half of choice (D) says that the second statement is the
author's conclusion. We agree with these labels, so this is the correct answer. As always, you. should
read all the answer choices, but you may be at the two-minute mark already, in which case you should
select (D) and move on.

The first half of choice (E) says that the first statement is a premise against the author's conclusion (in
other words, Premise Against). The second half of this choice, however, says that the second statement
is another Premise Against. Eliminate this choice.

84. B

1. Evidence -> Intermediate conclusion -> Following final Conclusion

2. However - changes the direction

3. Counter Evidence -> Counter Conclusion = This is the Main Conclusion of the argument

So, the first boldface is an intermediate conclusion or a prediction and second bold face is a supporting
evidence for the Final conclusion of the argument. The However explains that what comes after it
challenges what came before it. So, Answer should be (B)

The first is a prediction that is challenged by the argument; the second is a finding upon which the
argument depends.
Why not C? "These executives claim that declining viewership will cause advertising revenue to fall and
networks will thus be unable to spend the large sums necessary to produce high quality programming".
--> BF1 is a claim made by the executives and not by the media critic.

Moving over to BF2: "research shows that users of alternative platforms are exposed to new programs
and, as a result, actually increase the numbers of hours per week that they watch television" --> The
colored portion should help you determine that it is a finding which supports the position of the media
critic.

Network executives: other platforms -> decline in tv viewership -> ad rates will fall -> death of
programming for other platforms

Media Critic: research shows: users of alternative platforms exposed to new programs -> increase in tv
viewership. Hence -> alternative platforms won't prevent networks from increasing advertising
revenue.

BF1: declining viewership will cause advertising revenue to fall and networks will thus be unable to
spend the large sums necessary to produce high quality programming.

BF2: as a result, actually increase the numbers of hours per week that they watch television.

BF1 supports the network executive's conclusion while BF2 supports the Media Critic's conclusion.

By Sandeep Gupta, GMAT Perfect 800 | www.top-one-percent.com 235


THE MOST AUTHORITATIVE GUIDE EVER WRITTEN ON GMAT CRITICAL REASONING

We are looking for the answer choice, in which BF1 and BF2 are on opposite side.

(A) The first is an inevitable trend that weighs against the critic's claim; the second is the claim. -->
Wrong: BF1 and BF2 on same side

(B) The first is a prediction that is challenged by the argument; the second is a finding upon which the
argument depends. --> Correct: BF1 and BF2 on opposite side

(C) The first clarifies the reasoning behind the critic’s claim; the second demonstrates why that claim is
flawed. --> Wrong: BF1 does not clarify critic's claim. It clarifies execs claim

(D) The first acknowledges a position that the technology executives accept as true; the second is a
consequence of that position. --> Wrong: First part of the choice is correct - BF1 is premise to execs
conclusion. Second part is wrong - BF2 goes against the exec's position.

(E) The first opposes the critic's claim through an analogy; the second outlines a scenario in which that
claim will not hold. --> Wrong: Second part of the answer choice wrong - BF2 is premise for critic's claim

85. D

Thus, people who credit High Renaissance painters with superior artistic skills are misguided.

The word “thus” might mean this is the conclusion. The previous sentence only said that “some” painters
did the tracing thing, not all of them. But this sentence seems to be condemning all of them.

Painting from a projected image requires only an insignificant amount of additional skill beyond that
needed to copy a picture outright.

Okay, the last sentence was definitely the conclusion. This sentence is supporting the conclusion. If this
is true, then yes, painters who use this technique aren't that great.

A. A “finding” could be a fact, and a claim is an opinion, so this one is okay so far – contender

B. A “finding” could be a fact, and the conclusion is technically an opinion. But the boldface opinion is
FOR the conclusion; it's not actually the conclusion itself. – eliminate

C. A “claim” is not a fact. I can eliminate this one. – Eliminate

D. “Evidence” can be a fact, and a claim is an opinion. This one has to stay in, too. – contender

E. “Evidence” can be a fact, but the second boldface is an opinion supporting the conclusion, while this
choice says that the second boldface is the “position,” or conclusion. I can eliminate this one. – eliminate

The main technique can distinguish between (A) and (D): both boldfaces are premises used to support
the author's conclusion.

Answer (A) says that the first boldface is used “to support a conclusion that the argument rejects.”
Eliminate answer (A).

Many people praise High Renaissance painting for creating very realistic images from observation - Fact

By Sandeep Gupta, GMAT Perfect 800 | www.top-one-percent.com 236


THE MOST AUTHORITATIVE GUIDE EVER WRITTEN ON GMAT CRITICAL REASONING

BF1: scholars have documented that some High Renaissance painters used pinhole cameras to project
the likeness of their subjects onto the canvas and painted from there - Evidence - Goes against the
previous statement and supports the conclusion

Thus, people who credit High Renaissance painters with superior artistic skills are misguided -
Conclusion - Based on the evidence of BF1

BF2: Painting from a projected image requires only an insignificant amount of additional skill beyond
that needed to copy a picture outright - Claim - Supports Conclusion and BF1

86. A

In an attempt to explain the cause of malaria, a deadly infectious disease, early European settlers in
Hong Kong attributed the malady to poisonous gases supposedly emanating from low-lying swampland.
This is a fact. Likely either background or premise.

In the 1880s, however, doctors determined that Anopheles mosquitoes were responsible for
transmitting the disease to humans after observing that the female of the species can carry a parasitic
protozoan that is passed on to unsuspecting humans when a mosquito feasts on a person’s blood. They
used to think it was one thing, and then they figured out it was really the mosquitoes. The boldface
language, in particular, is the evidence used to show that it was mosquitoes. That's a Premise.

The question specifically asks me what role this information plays: “the female carries a PP that is
passed to humans when a mosquito bites someone. "Because of that, the scientists decided that the
mosquitoes were transmitting the disease. That's the most like a premise that supports some further
conclusion.

A. Provides support for something that happened.

B. "Evidence"— And that evidence does “contradict" what the earlier settlers thought! Was that an
established fact? No, they thought that, but the argument doesn't say it was an “established fact." Cross
this one off.

C. Nothing was contested here. First, some people thought one thing, and later, new evidence led some
doctors to conclude something else. No.

D. The only thing we might be able to describe as an erroneous conclusion is what the early settlers
thought. But the boldface supports the doctors' conclusion.

The first half of the sentence, the non-bold part, is the new conclusion. The bold part is the evidence
supporting that. This isn't it after all!

By Sandeep Gupta, GMAT Perfect 800 | www.top-one-percent.com 237


THE MOST AUTHORITATIVE GUIDE EVER WRITTEN ON GMAT CRITICAL REASONING

87. B

There are two strategies discussed here:

1. charge as much as possible for their new designs

2. CHARGE LESS THAN THE GREATEST POSSIBLE PRICE.

The argument endorses the second strategy.

The first statement in bold "BECAUSE DESIGN INNOVATIONS ARE QUICKLY COPIED BY OTHER
MANUFACTURERS" explains the reason for the appeal of the first strategy.

The second statement in bold is the second strategy, the strategy endorsed by the argument.

88. A

A. The first provides information without which the argument lacks force; the second states the main
conclusion of the argument. Correct. The first provides support for the main conclusion. The second
sentence is the main conclusion

B. The first provides information without which the argument lacks force; the second states an
intermediate conclusion that is used to support a further conclusion. The second is not the intermediate
conclusion rather the main conclusion.

C. The first sites a practice that the journalist seeks to defend; the second sites a likely consequence of
this practice. The author is not defending the first statement, rather it is a fact.

D. The first states evidence bearing against the main conclusion of the argument; the second is that main
conclusion. The first provides support to the main conclusion.

E. Each provides evidence in support of an intermediate conclusion that supports a further conclusion
stated in the argument. The second statement is the main conclusion.

C says:
The first is a phenomenon that justifies a specific strategy - this is fine. It justifies "charge as much as
possible for their new designs"
the second is that strategy. - this is incorrect. The second is not the strategy that the first part justifies.
The second is the strategy "CHARGE LESS THAN THE GREATEST POSSIBLE PRICE."

B says:
The first is a consideration that helps explain the appeal of a certain strategy; - this is fine. It explains
the appeal of "charge as much as possible for their new designs"
the second presents an alternative strategy endorsed by the argument. - this is fine. It presents
"CHARGE LESS THAN THE GREATEST POSSIBLE PRICE", the strategy endorsed by the argument.

By Sandeep Gupta, GMAT Perfect 800 | www.top-one-percent.com 238


THE MOST AUTHORITATIVE GUIDE EVER WRITTEN ON GMAT CRITICAL REASONING

89. A

Hunters alone are blamed for the decline in Greenrock National Forest's deer population over the past
ten years. - It is what was believed in the past ten years.

Yet clearly, black bears have also played an important role in this decline. - Yet is indicating sort of
contrast against belief held.

Clearly- is followed by the final statement which is more like a conclusion.

In the past ten years, the forest's protected black bear population has risen sharply, and examination of
black bears found dead in the forest during the deer hunting season showed that a number of them had
recently fed on deer. - This gives the reason why the final statement was given.

After looking through the option, the bolded part is the main conclusion.

A is the correct choice.

90. D

The conclusion of this argument is that “many weight-loss efforts fail because people eat more calories
than they intend to consume.” The first boldface portion is a factual premise (“Studies have shown…”)
that there is an observed correlation between keeping a diet record and losing weight successfully. This
premise (indirectly) supports the researchers’ conclusion. The second boldface portion is another
supporting premise, this one citing a specific study showing that dieters who do not keep a diet record
eat far more than they realize.

(A) The first boldface is not the conclusion; it is an observed fact. The second boldface is evidence that
the researchers’ conclusion is correct, but is not evidence that the first boldface is correct.

(B) The first boldface is a fact that supports the researchers’ theory, but it does not explain why their
conclusion is correct—the other premises do so.

(C) The first boldface is a fact that supports the researchers’ theory, but it does not illustrate the truth
of that theory—the second boldface does. The second boldface is a fact that supports the researchers’
theory; it is not a competing theory.

(D) CORRECT. The first boldface (diet record = diet success) is a basis for the researchers’ conclusion
that many weight-loss efforts fail because people consume more than they intended. The second
boldface directly illustrates how weight-loss efforts of a certain group failed for exactly that reason.

(E) The first boldface is a factual statement, not a theory. Furthermore, the first boldface supports the
theory of the researchers; it is not something they have disproved.

By Sandeep Gupta, GMAT Perfect 800 | www.top-one-percent.com 239


THE MOST AUTHORITATIVE GUIDE EVER WRITTEN ON GMAT CRITICAL REASONING

91. B

The conclusion of this argument is the first sentence: “Weight loss programs that guarantee results
mislead their customers.” The rest of the text is evidence in support of that conclusion. The correct
answer will identify the first boldface portion as the conclusion, and the second boldface portion as
evidence in support of the conclusion.

(A) The first boldface portion is a fairly strong assertion, not a generalization. The second boldface
portion is not a consequence of the first.

(B) CORRECT. The first boldface portion is the author's position, or conclusion.
The second boldface portion is a consideration in support of the position stated in the first boldface
portion.

(C) The second boldface portion is evidence in support of the opinion stated in the first boldface portion,
not weighing against it.

(D) The first boldface portion is the author’s assertion, not one that he or she questions.

(E) The first boldface portion is the author’s main point, not evidence in support of it. The second
boldface portion is not the author's main point, but rather support for it. This answer choice incorrectly
reverses the roles of the boldface portions.

92. D

The question asks us to analyze the structure of the argument. Specifically, it asks us to determine the
relationship of the two bolded sentences to the argument as a whole. To do this effectively, we need
first to determine the conclusion of the argument. In this case, the conclusion is the final sentence, which
follows from the evidence presented in the rest of the argument. So, we know that the correct answer
cannot describe either bolded portion as the author's final conclusion. The first bolded portion is
presented as an earlier, erroneous belief. In evaluating the answer choices, we should look to eliminate
any that suggest the author agrees with this first statement. The second bolded statement shows that
the earlier theory (that lactic acid causes soreness) was incorrect, and thus also provides evidence in
support of the author's conclusion. So, we know that the correct answer must describe the second
bolded portion as supporting the conclusion. The correct answer must describe both portions correctly,
not just one or the other.

(A) This choice misrepresents the first bolded portion by claiming that the author accepts it as true,
when the author actually presents it as an incorrect, outdated belief. The second portion is also incorrect
because it indicates that the second bolded portion supports the first one.

(B) This choice also misrepresents the first portion, for the reason described above. It corrects the
second bolded portion, however, by describing it as contradictory to the first portion.

(C) The first portion is correctly described as an assertion that the author does not believe to be true.
Yet the second portion is incorrectly described as going against the author's final conclusion.

(D) CORRECT. This choice correctly represents the first portion by saying the author believes it is no
longer valid. The second portion is correctly described as evidence in support of the author's conclusion.

By Sandeep Gupta, GMAT Perfect 800 | www.top-one-percent.com 240


THE MOST AUTHORITATIVE GUIDE EVER WRITTEN ON GMAT CRITICAL REASONING

(E) The first portion is correctly described as evidence considered by the author to be invalid. However,
this choice misrepresents the second portion as the conclusion, when it is actually evidence given in
support of the conclusion.

93. C

The conclusion of the argument is that the nominees "are more notable for their close ties to corporate
and economic interests" than for their positions on controversial issues. The first boldfaced statement
is a recognition of the fact that the president's nominees have been branded conservative. The second
boldfaced statement offers information in support of the assertion that the nominees are more notable
for their corporate ties. So, we need to find a choice that describes both statements accurately.

(A) The author does not seek to attack the assertion made in the first statement.

(B) The author does acknowledge the first statement as true. However, the second statement is not the
conclusion.

(C) CORRECT. The author does accept the first statement as true, and the second statement is indeed
given in support of the conclusion.

(D) The first statement is not the author's "position" (i.e., conclusion).

(E) The first statement is not an exception to a rule, making the description of the second statement
false as well.

94. B
The question asks us to analyze the structure of the argument. Specifically, we are asked to determine
the respective roles of the two portions in boldface. To do so, we need first to find the conclusion of the
argument and then determine the relationship of each boldface portion to that conclusion. The author
argues that high interest rates will cause people to refrain from buying homes because their mortgage
costs will be too high. In turn, sellers will be forced to lower their asking prices. In the end, this chain of
events will work to stabilize the real estate market. So, the last sentence, which happens to be the second
boldface portion, is the conclusion. The first boldface portion is a fact that the author believes will lead
to the stabilization of the market. We need to find a choice that reflects this assessment.

(A) The first portion is not the author's main point (another term for conclusion).

(B) CORRECT. The first portion is indeed a consideration that the author believes will result in a
particular situation, namely the stabilization of the market. The second is indeed that situation.

(C) The first portion does not weigh against (contradict) the author's main point (conclusion).

(D) The first is indeed a prediction, but the second portion is not evidence; it is the conclusion.

(E) The first portion is not the main point (conclusion).

By Sandeep Gupta, GMAT Perfect 800 | www.top-one-percent.com 241


THE MOST AUTHORITATIVE GUIDE EVER WRITTEN ON GMAT CRITICAL REASONING

95. E
The corporate strategist begins the argument by describing the usual relationship between the
reduction in price and the resulting effect on product demand. After describing the traditional
relationship, he concludes, however, that the management’s price-reduction strategy is flawed. Thus,
the first statement in boldface represents the conclusion of the corporate strategist. Finally, the
strategist finishes his argument by providing evidence that justifies his reasoning. Therefore, the second
statement in boldface provides evidence that supports the main position of the corporate strategist.

(A) This answer choice correctly describes the first statement but incorrectly states that the second
statement weighs against rather than supports the main position of the equity strategist.

(B) This answer choice incorrectly states that the first statement represents an assumption made by
the strategist. Remember that assumptions are never stated in the body of the argument. The second
statement is correctly described as evidence supporting the strategist’s reasoning.

(C) This answer choice incorrectly labels the first statement as evidence supporting the conclusion and
the second statement as the conclusion itself. The order of the two descriptions should be the reverse
of that presented in this answer choice.

(D) This answer choice incorrectly describes the first statement as evidence rather than the conclusion,
and incorrectly states that the scenario outlined in the second statement will not hold in the case at
issue.

(E) CORRECT. This answer correctly describes the first statement as the strategist’s conclusion and the
second statement as the evidence supporting that conclusion.

96. B

The first boldface sentence states that the fight against the drug trade in Country X should focus for the
time being on tightening the country’s borders and targeting its major smugglers. The second boldface
sentence states that the United Nations and the government of Country X should eventually replace the
poppy fields with other farming ventures ("agricultural infrastructure"). We need to find a choice that
describes the relationship between these two sentences.

(A) This choice states that the first sentence is the conclusion and that the second sentence is an
alternative to that conclusion. This misrepresents the relationship. The first sentence is a shorter-term
conclusion and the second sentence is a longer-term conclusion of the argument.

(B) CORRECT. This choice states that the first sentence is a short-term solution to a problem and the
second a long-term solution to the same problem. This accurately describes the relationship. The first
sentence states that the fight should focus "for the time being" on borders and smuggling while the
second sentence states that the United Nations and the government of Country X should work to replace
the poppy crop with something else.

(C) This choice states that the first sentence presents a problem. According to the text, however, the
first sentence is not a problem but a solution to a problem.

(D) This choice states that the first sentence presents a popular solution to a problem. Since we have no
way of knowing whether the solution is popular, this cannot be correct.

By Sandeep Gupta, GMAT Perfect 800 | www.top-one-percent.com 242


THE MOST AUTHORITATIVE GUIDE EVER WRITTEN ON GMAT CRITICAL REASONING

(E) This choice states that the first sentence presents an argument and that the second sentence
provides evidence to support that argument. Though the first sentence does present an argument, the
second sentence, rather than providing evidence, presents a second argument.

97. C

This argument begins with the classic “some of my critics claim” construction. As we know from that
discussion, the conclusion of the argument will typically be the opposite of the claim. In this case, the
conclusion comes in the second sentence when the mayor states the following:

Conclusion: I do not agree that I am at fault for this problem [the budget deficit]. Because neither
bolded portion overlaps the conclusion, the bolded portions must be premises or counter-premises.
Take a moment to go back and look at some of the indicator words-see the “although” just before the
first bolded portion? The presence of that word means that the first bolded portion is given as a
counter-premise to the author’s conclusion. That is, the mayor admits that there was a budget deficit,
and this fact possibly undermines his or her argument in some way, but the mayor still believes that the
conclusion is true despite this fact. The second bolded portion comes after the conclusion and is used
as a premise to support the conclusion. Thus, one bolded portion is a counter-premise, and the other is
a premise, and the correct answer must reflect that fact. In summary, the pertinent portions of the
argument appear as follows:

Critics claim: The critics claim that the mayor is responsible for the current budget deficit.

Bolded portion: In this counter-premise the mayor admits that there is a budget deficit.

Conclusion: The conclusion indicates that even though there is a budget deficit, the mayor is not
responsible for the deficit, contrary to the claim of the critics.

Bolded portion: This is a premise that indicates that the mayor’s economic policies have actually
benefited the city, not hurt the city. A quick scan of the answer choices reveals that each will be broken
into two parts: the first part will describe the first bolded section and the second part will describe the
second bolded section.

Answer choice (A): The first half of this answer is a classic Contender. It may very well be that the
counter-premise has been used against the mayor. Setting that aside, however, the description of the
second boldface portion is inaccurate, so this answer choice is incorrect.

Answer choice (B): This is classic Half-Right, Half-Wrong answer choice. The first bolded portion is a
statement accepted by the mayor; however, it is not the case that the second bolded portion is a
“consequence of the critics’ claims.”

Answer choice (C): This is the correct answer. In this case, although the mayor admits that the first
bolded portion is true, he or she does not believe that fact has a negative impact on the conclusion.

Answer choice (D): This answer begins poorly because we do not know that the first boldface portion
is evidence of unlawful activity by the mayor.

By Sandeep Gupta, GMAT Perfect 800 | www.top-one-percent.com 243


THE MOST AUTHORITATIVE GUIDE EVER WRITTEN ON GMAT CRITICAL REASONING

Answer choice (E): This is another answer where the first bolded portion causes many people to leave
the answer as a Contender. However, the description of the second bolded portion is inaccurate
because the second portion is not a consequence of the first bolded portion (this is a direct test of your
ability to discern a premise from a conclusion).

98. C

The argument is structured as follows:

Premise: A rise of just two degrees prevents the vertical mixing of seawater from different strata.

Premise: This restricts the availability of upwelling nutrients to phytoplankton.

Premise: Zooplankton, which feed upon phytoplankton, feed the rest of the food chain.

Conclusion: Seemingly inconsequential changes in sea temperature due to global warming eventually
result in declines in fish and seabird populations. The conclusion in the first line is echoed again in the
final sentence. The argument part referenced in the question stem is a premise (note the use of the
premise indicator “since” in the last line), and your answer must indicate that the role played by the
argument part is that of a premise.

Answer choice (A): The portion referenced in the question stem is not a hypothesis, but rather a
statement of fact.

Answer choice (B): The statement referenced in the question stem is not an example of the way the
mixing of seawater affects feeding habits, but rather another premise that is then combined with the
vertical mixing premise to help support the conclusion.

Answer choice (C): This is the correct answer. The phrase “it helps show” describes a premise, and
in this case the premise is used to support a statement about the effect of temperature changes on fish
and seabirds.

Answer choice (D): The argument does not take a position that global warming should be curtailed.
Instead, the argument shows how small changes in sea temperature lead to population declines, and no
opinion of those effects is stated.

Answer choice (E): This is an Exaggerated Answer. The argument specifically indicates that fish and
seabird populations will decline. This answer choices states that all organisms are threatened.

99. B

The argument has an interesting structure. Visually, the argument appears as follows:

Premise: Pedigreed dogs, including those officially classified as working dogs, must conform to
standards set by organizations that issue pedigrees.

Premise: Those standards generally specify the physical appearance necessary for a dog to be
recognized as belonging to a breed but stipulate nothing about other genetic traits, such as those that
enable breeds originally developed as working dogs to perform the work for which they were
developed.

By Sandeep Gupta, GMAT Perfect 800 | www.top-one-percent.com 244


THE MOST AUTHORITATIVE GUIDE EVER WRITTEN ON GMAT CRITICAL REASONING

Premise: Dog breeders try to maintain only those traits specified by pedigree organizations, and traits
that breeders do not try to maintain risk being lost.

Sub-conclusion: Certain traits like herding ability risk being lost among pedigreed dogs.

Conclusion: Therefore, pedigree organizations should set standards requiring working ability in
pedigreed dogs classified as working dogs. Given the size of the stimulus, this is a tough problem to
analyze. The second to last sentence contains both a premise and a conclusion. The final sentence
contains the main conclusion. Perhaps because of the size of the problem, the test makers kindly
inserted the conclusion indicator “therefore” before the main conclusion.

Answer choice (A): This is a Half Right, Half Wrong answer. The phrase referenced in the question is a
“claim on which the argument depends,” but it is not one for which no support is given. In fact, several
premises back up the statement.

Answer choice (B): This is the correct answer. The statement in question is a sub-conclusion, described
in this answer as a subsidiary conclusion.

Answer choice (C): The phrase in question is in agreement with the argument, and does not reference
a possible objection. If you were to choose this answer, you would have to ask yourself, “What is the
possible objection mentioned in this answer choice?”

Answer choice (D): The argument as a whole works towards supporting the recommendation that
“pedigree organizations should set standards requiring working ability in pedigreed dogs classified as
working dogs.” The phrase in the question stem does not summarize the antithesis of that position.

Answer choice (E): This answer has the order of the argument backwards. The phrase referenced in the
question stem provides evidence necessary to support a claim stated later in the argument.

100. C

There are two persons in the question: Professor Jones and the author. As one can see, the author does
not agree to the argument by Professor Jones. “This conclusion, however, is unwarranted because …
(the second boldface) “. Therefore, the opinion of the author is very much at odds with the opinion of
Professor Jones. Then, let’s take a look at the first boldface “certain vital biological processes, such
as photosynthesis, are slower in these areas than is usual for the inspected species”. This is result
of a test or lab or discovery, so it is an evidence professor Jones used to back up his conclusion
“Professor Jones has claimed that...”. Therefore, answer C is the correct choice. Why is B incorrect?
“The first contains the argument the author critiques” is incorrect, because the author does not agree
with the evidence (the first boldface), but only disagrees the conclusion (claim) made by Professor
Jones. We can rule out A, D and E as they are unrelated. B states that ‘the first contains the argument the
author critiques. Nowhere in the passage does the author critique the argument; he merely attacks the
conclusion that is based on the argument. Furthermore, the first sentence is clearly evidence and not an
argument in itself. The information here isn’t correct, necessarily, so this is not a case of challenging
a fact.

By Sandeep Gupta, GMAT Perfect 800 | www.top-one-percent.com 245

You might also like